You are on page 1of 798

ki fakultet

Matematic
Univerzitet u Beogradu

ZBIRKA ZADATAKA
IZ OSNOVA GEOMETRIJE

Dr Dragomir Lopandic

SAVEZ STUDENATA

PRIRODNOMATEMATICKOG
FAKULTETA
Beograd
1971.

Sadr
zaj
1 APSOLUTNA GEOMETRIJA
1.1 Konveksni i konkavni likovi . . . . . . . . . . . . . . . . . . . . .
1.2 Zadaci . . . . . . . . . . . . . . . . . . . . . . . . . . . . . . . .

3
3
4

2 PARALELNOST
69
2.1 Paralelnost pravih . . . . . . . . . . . . . . . . . . . . . . . . . . 69
3

I SLICNOST

PROPORCIONALNOST DUZI
LIKOVA

4 KARAKTERISTICNE
TEOREME I NJIHOVE PRIMENE
4.1 Apolonijeva teorema . . . . . . . . . . . . . . . . . . . . . . . . .
4.2 Lajbnicova teorema i njena primena . . . . . . . . . . . . . . . .
4.3 Karnoova teorema i njena primena . . . . . . . . . . . . . . . . .

4.4 Cevijeva
teorema i njena primena . . . . . . . . . . . . . . . . . .
4.5 Menelajeva teorema i njena primena . . . . . . . . . . . . . . . .
4.6 Dezargova teorema i njena primena . . . . . . . . . . . . . . . .
4.7 Paskalova teorema i njena primena . . . . . . . . . . . . . . . . .
4.8 Brijansonova teorema i njena primena . . . . . . . . . . . . . . .
4.9 Van Obelova teorema i njena primena . . . . . . . . . . . . . . .
4.10 Ptolomejeva teorema . . . . . . . . . . . . . . . . . . . . . . . .
4.11 Ojlerova teorema iz geometrije cetvorougla i njena primena . . .
5

210
210
224
229
234
245
263
269
272
273
276
289

HARMONIJSKI SPREGNUTI ELEMENTI. DVORAZMERA292


5.1 Harmonijske tacke i harmonijske prave . . . . . . . . . . . . . . 292
5.2 Dvorazmera cetiri tacke i dvorazmera cetiri prave . . . . . . . . 313

6 GEOMETRIJA KRUGOVA
6.1 Ortogonalni krugovi . . . . . . . . . . . . . . . . . . . . . . . . .
6.2 Pol i polara u odnosu na krug . . . . . . . . . . . . . . . . . . .
6.3 Spregnute tacke i spregnute prave u odnosu na krug . . . . . . .
6.4 Polarni i autopolarni trouglovi. Polarni krug trougla . . . . . . .
6.5 Slicnost krugova . . . . . . . . . . . . . . . . . . . . . . . . . . .
6.6 Potencija tacke u odnosu na krug . . . . . . . . . . . . . . . . .
6.7 Radikalna osa dvaju krugova. Radikalno srediste triju krugova .
6.8 Pseudoradikalna osa dvaju krugova. Pseudoradikalno srediste
triju krugova. . . . . . . . . . . . . . . . . . . . . . . . . . . . . .
6.9 Elipticki, parabolicki i hiperbolicki pramenovi krugova . . . . .
7

133

329
329
335
343
349
356
364
371
388
390

INVERZIJA
402
7.1 Inverzne tacke . . . . . . . . . . . . . . . . . . . . . . . . . . . . 402
7.2 Inverzni likovi . . . . . . . . . . . . . . . . . . . . . . . . . . . . 408
INVOLUCIJA

426

9 ZNACAJNE
TACKE
I LINIJE U GEOMETRIJI TROUGLA

I CETVOROUGLA
439
9.1 Simsonova prava trougla i cetvorougla . . . . . . . . . . . . . . . 439
9.2 Mikelova tacka trougla i cetvorougla . . . . . . . . . . . . . . . . 448
9.3 Ojlerova prava trougla i cetvorougla . . . . . . . . . . . . . . . . 452

9.4
9.5
9.6
9.7
9.8
9.9
9.10
9.11
9.12
9.13
9.14
9.15
9.16
9.17
9.18

Ojlerov krug trougla i cetvorougla . . . . . . . . . . . .


Nagelova tacka trougla . . . . . . . . . . . . . . . . . . .
Spikerov krug trougla . . . . . . . . . . . . . . . . . . .
Furmanov krug trougla . . . . . . . . . . . . . . . . . .
Izometricke tacke u odnosu na duz i na trougao . . . . .
Izogonalne prave u odnosu na ugao. Izogonalne tacke u
na trougao. . . . . . . . . . . . . . . . . . . . . . . . . .
Lemoanova tacka i Lemoanova prava trougla . . . . . .
Lemoanovi krugovi trougla . . . . . . . . . . . . . . . .
Tikerovi krugovi trougla . . . . . . . . . . . . . . . . .
Brokarove tacke trougla . . . . . . . . . . . . . . . . . .
Brokarov krug trougla . . . . . . . . . . . . . . . . . . .
Apolonijevi krugovi i izodinamicke tacke trougla . . . .
Droz-Farnijevi krugovi trougla . . . . . . . . . . . . . .
Adamsovi krugovi trougla . . . . . . . . . . . . . . . . .
Ortopol prave u odnosu na trougao . . . . . . . . . . . .

. . . . .
. . . . .
. . . . .
. . . . .
. . . . .
odnosu
. . . . .
. . . . .
. . . . .
. . . . .
. . . . .
. . . . .
. . . . .
. . . . .
. . . . .
. . . . .

457
460
463
465
466
468
471
479
482
485
488
490
493
496
497

10 GEOMETRIJA POLIGONA
501
10.1 Opsti poligoni . . . . . . . . . . . . . . . . . . . . . . . . . . . . 501
10.2 Tetivni i tangentni poligoni . . . . . . . . . . . . . . . . . . . . . 508
10.3 Pravilni poligoni . . . . . . . . . . . . . . . . . . . . . . . . . . . 523

I ODREDIVANJE POVRSINA
543
11 RAZLAGANJE POVRSI
11.1 Razlaganje povrsi . . . . . . . . . . . . . . . . . . . . . . . . . . . 543
11.2 Odredjivanje povrsina . . . . . . . . . . . . . . . . . . . . . . . . 554
12 NEJEDNAKOSTI U PLANIMETRIJI
13 KONSTRUKTIVNI ZADACI
13.1 Metoda geometrijskih mesta .
13.2 Metoda transformacije . . . .
13.3 Inverzija . . . . . . . . . . . .
13.4 Translacija . . . . . . . . . .
13.5 Simetrija . . . . . . . . . . . .
13.6 Rotacija . . . . . . . . . . .

.
.
.
.
.
.

.
.
.
.
.
.

.
.
.
.
.
.

.
.
.
.
.
.

.
.
.
.
.
.

617
.
.
.
.
.
.

.
.
.
.
.
.

.
.
.
.
.
.

.
.
.
.
.
.

.
.
.
.
.
.

.
.
.
.
.
.

.
.
.
.
.
.

.
.
.
.
.
.

.
.
.
.
.
.

.
.
.
.
.
.

.
.
.
.
.
.

.
.
.
.
.
.

.
.
.
.
.
.

.
.
.
.
.
.

.
.
.
.
.
.

686
686
715
737
743
751
756

APSOLUTNA GEOMETRIJA

Euklidska geometrija izvedena sintetickim metodom zasniva se na aksiomama koje su podeljene u pet grupa i to: aksiome rasporeda, aksiome incidencije, aksiome podudarnosti, aksiome neprekidnosti i aksiomu paralelnosti.
Deo te teorije koji se izvodi iz prve cetiri grupe aksioma, dakle bez aksiome
paralelnosti, naziva se apsolutnom geometrijom. Zadaci iz ovog clana su iz te
apsolutne geometrije, pa se njihovo resavanje zasniva iskljucivo na primeni poznatih stavova iz tog dela geometrije.

1.1

Konveksni i konkavni likovi

Definicija 1.1. Lik nazivamo konveksnim ako sve tacke duzi koja je
odredena bilo kojim dvema tackama toga lika pripadaju tome liku; ako taj
uslov nije zadovoljen, lik nazivamo konkavnim. Tako su likovi predstavljeni
na slikama 1 i 3 konveksni, a na slikama 2 i 4 konkavni.
Definicija 1.2. Lik nazivamo ogranicenim ako postoji kruzna povrs koja
sadrzi sve tacke lika ; ako takva kruzna povrs ne postoji, lik nazivamo
neogranicenim. Tako su likovi predstavljeni na slikama 1 i 2 ograniceni, a na
slikama 3 i 4 neograniceni.
Definicija 1.3. Najvecu od duzi koju spajaju po dve tacke ogranicenog lika
nazivamo precnikom ili dijametrom lika .
Definicija 1.4. Pravu s koja se nalazi u ravni lika i koja s tim likom ima
jednu ili vise zajednickih tacaka, pri cemu se sve ostale tacke lika nalaze s iste
strane od prave s nazivamo pravom oslonca lika .
Definicija 1.5. U odnosu na svaku ravnu povrs sve tacke ravni dele se
na unutrasnje, spoljasnje i granicne. Tacku A nazivamo unutrasnjom tackom
povrs i ako postoji kruzna povrs sa sredistem A cije sve tacke pripadaju povrsi
. Tacku B nazivamo spoljasnjom u odnosu na ravnu povrs ako u ravni te
povrsi postoji kruzna povrs sa sredistem B koja ne sadrzi ni jednu tacku povrsi
. Tacku C ravne povrsi nazivamo granicnom ako u ravni povrsi svaka
kruzna povrs sa sredistem C sadrzi i tacaka koje pripadaju i tacaka koje ne
pripadaju povrsi . Skup svih granicnih tacaka povrsi predstavlja izvesnu
liniju koju nazivamo granicom ili rubom povrsi .
Definicija 1.6. Za ravnu liniju kazemo da je konveksna ako ona predstavlja
granicu neke konveksne povrsi.

1.2

Zadaci

1. Dokazati da je svaka
(a) trougaona povrs,
(b) kruzna povrs,
konveksna.
Uputstvo:

Slika 5
a) Da bismo dokazali da je neka trougaona povrs ABC konveksna, na osnovu
definicije konveksnog lika, mi treba da dokazemo da svaka tacka R koja se nalazi izmedu dveju proizvoljnih tacaka P i Q te trougaone povrsi pripada toj
trougaonoj povrsi. Kako su tacke A,B,C temena jedne trougaone povrsi, one su
nekolinearne, pa najmanje jedna od njih, npr. tacka A, ne pripada pravoj koja
je odredena tackama P i Q. Pri tom na stranici BC postoje dve razlicite tacke
P i Q takve da je tacka P izmedu tacaka A i P , a tacka Q izmedu tacaka A i
Q ili je istovetan s tackom Q . Ako je P P i Q Q , dokaz je jednostavan.
Ako je npr. tacka P izmedu tacaka A i P , bice A,P ,Q tri nekolinearne tacke, a
P i R tacke takve da je B(A,P,P) i B(P,R,Q), te prema jednoj aksiomi postoji
tacka S takva da je B(A,S,Q) i B(P,R,S). Kako je B(A,S,Q) i Q tacka duzi
AQ , bice B(A,S,Q). Sad su A,P ,Q tri nekolinearne tacke, a R i S tacke takve
da je B(A,S,Q) i B(P,R,S), te prema istoj aksiomi postoji tacka R takva da
je B(A,R,R) i B(P,R,Q). S obzirom da tacke P i Q pripadaju duzi BC i da
je B(P,R,Q), bice B(B,R,C). Najzad iz B(B,R,C) sledi da tacka R pripada
trougaonoj povrsi (ABC). Stoga je trougaona povrs konveksna (Slika 5).

Slika 6
4

b) Da bismo dokazali da je neka kruzna povrs konveksna, saglasno definiciji


konveksnog lika, mi treba da dokazemo da svaka unutrasnja tacka R duzi koja
spaja dve proizvoljne tacke P i Q povrsi pripada toj povrsi.
Obelezimo sa O srediste povrsi i sa r poluprecnik kruzne povrsi . Tacke
O,P ,Q su temena izvesnog trougla ili su na jednoj pravoj. U prvom slucaju
jedan od uglova ORP i ORQ je veci ili jednak 90o . Neka je to npr. ugao ORP .
Kako je u trouglu OP R ugao ORP veci od ugla OP R, to je OR < OP . No
OP r, pa je OR < r. Stoga tacka R pripada povrsi . U drugom slucaju
neposredno zakljucujemo da iz svake od mogucnosti P O Q, O P Q,
sledi da je OR < r, pa i u tom slucaju tacka R pripada povrsi . Prema tome
povrs je konveksna (Slika 6).
2. Ako sva temena neke poligonske povrsi (A1 . . . An ) pripadaju nekoj konveksnoj povrsi , dokazati da sve tacke poligonske povrsi (A1 . . . An ) pripadaju
povrsi .
Uputstvo:

An
P
Q

A1
q

A2

A3

Slika 7
S obzirom da sva temena poligonske povrsi (A1 . . . An ) pripadaju konveksnoj
povrsi , i sve tacke ruba te poligonske povrsi pripadaju povrsi . Dokazimo
da i svaka unutrasnja tacka M poligonske povrsi (A1 . . . An ) pripada povrsi .
S obzirom da je broj temena pomenute poligonske povrsi konacan, postoji u
ravni te poligonske povrsi prava s koja sadrzi tacku M i koja ne sadrzi ni jedno
teme te poligonske povrsi. Tacka M razlaze pravu s na dve poluprave p i q.
Svaka od tih dveju polupravih p i q ima za kraj tacku M koja se nalazi u poligonu A1 . . . An , pripada ravni tog poligona a ne sadrzi ni jedno njegovo teme,
prema tome sece taj poligon u neparnom broju tacaka. Ako je P bilo koja od
presecnih tacaka poluprave p s poligonom A1 . . . An , a Q bilo koja od presecnih
tacaka poluprave q s poligonom A1 . . . An , bice tacka M izmedu tacaka P i Q.
S obzirom da su tacke P i Q na rubu poligonske povrsi (A1 . . . An ) koja pripada
povrsi , tacke P i Q takode pripadaju konveksnoj povrsi , te i unutrasnja
tacka M duzi P Q pripada povrsi . Prema tome, sve tacke poligonske povrsi
(A1 . . . An ) pripadaju povrsi (Slika 7).
3. Dokazati da je presek konacnog broja od n konveksnih likova takode
konveksan lik.
Uputstvo:

A
1

B
2

Slika 8
Neka je lik presek konveksnih likova 1 . . . n . Dokazimo da je lik takode
konveksan.
Obelezimo sa A i B ma koje dve tacke lika , a sa P bilo koju tacku izmedu
njih. S obzirom da sve tacke lika pripadaju konveksnim likovima 1 . . . n ,
bice da su A i B tacke tih konveksnih likova. Stoga tacka P koja se nalazi
izmedu tacaka A i B pripada svim likovima 1 . . . n , dakle i liku .
Otuda sledi da sve tacke izmedu tacaka A i B pripadaju liku , pa je lik
konveksan (Slika 8).
4. Ako je a1 , . . . , an konacan skup od n duzi koje pripadaju jednoj pravoj i
od kojih svake dve imaju najmanje jednu zajednicku tacku, dokazati da svih n
duzi toga skupa imaju najmanje jednu zajednicku tacku.
Uputstvo: Najpre dokazimo ovaj stav kada je n = 3. Obelezimo sa S1 , S2 , S3
proizvoljne tacke zajednicke duzima a2 i a3 , a3 i a1 , a1 i a2 . S obzirom da duzi
a1 , a2 , a3 pripadaju jednoj pravoj, i tacke S1 , S2 , S3 su na jednoj pravoj. Ako su
dve ili tri tacke S1 , S2 , S3 tetivne, one pripadaju svim trima duzima a1 , a2 , a3 .
Ako su tacke S1 , S2 , S3 razlicite, jedna od njih, npr. tacka S2 je izmedu ostalih
dveju tacaka S1 i S3 . Pri tome tacke S1 i S3 pripadaju duzi a2 , te i svaka tacka
duzi S1 S3 , dakle i tacka S2 pripada duzi a2 . Stoga je S2 zajednicka tacka duzi
a1 , a2 , a3 .
Da bismo dokazali ovaj stav i za slucaj kada je n > 3, primenimo metod matematicke indukcije. Zato pretpostavimo da je stav istinit kada je n = k, a
dokazimo da vazi i za n = k + 1. Presek b duzi ak i ak+1 je takode duz, ili u krajnjem slucaju jedna tacka. Pretpostavimo da je b duz; dokaz koji navodimo vazi i
u slucaju kada je b tacka. Dokazimo da svake dve od k duzi a1 , . . . , ak1 , b jedne
prave imaju imaju najmanje jednu zajednicku tacku. Saglasno pretpostavci,
svake dve od duzi a1 , . . . , ak1 imaju najmanje jednu zajednicku tacku. Duz b
ima takode sa svakom od duzi ai (i = 1, . . . , k 1) najmanje jednu zajednicku
tacku, jer prema dokazanom delu svake tri duzi ai , ak , ak+1 imaju najmanje
jednu zajednicku tacku. Dakle, svake dve od duzi a1 , . . . , ak1 , b imaju najmanje po jednu zajenicku tacku, te saglasno pretpostavci svih k tih duzi imaju
najmanje jednu zajednicku tacku. Otuda sledi da i svih k + 1 duzi a1 , . . . , ak+1
imaju najmanje jednu zajednicku tacku. Dokazali smo da je stav istinit kada je
n = 3, prema tome on je istinit i za n > 3.
5. Ako konacan skup od n polupravih neke prave p pokriva celu tu pravu,

dokazati da u tom skupu polupravih postoje takve dve poluprave koje takode
pokrivaju celu tu pravu.
Uputstvo: Sve poluprave konacnog skupa polupravih koje pokrivaju pravu
p ne mogu biti istosmerne, jer bi u tom slucaju, u tom skupu polupravih, postojala takva poluprava koja bi sadrzala sve ostale poluprave tog skupa, pa
tacke prave p koje se nalaze na produzenju poluprave ne bi pripadale ni jednoj
polupravoj datog skupa polupravih, sto je suprotno pretpostavci. Medu polupravama koje imaju jedan isti smer, postoji takva poluprava a koja sadrzi sve
ostale poluprave tog podskupa, a medu polupravama koje imaju drugi smer postoji takva poluprava b koja sadrzi ostale poluprave tog podskupa. Poluprave
a i b pokrivaju celu pravu p. Zaista, ako one ne bi pokrivale celu pravu p, iz
suprotne usmerenosti tih polupravih sledilo bi da tacke prave p koje se nalaze
izmedu krajeva A i B polupravih a i b ne pripadaju ni je- dnoj polupravoj datog
skupa polupravih, sto je suprotno pretpostavci. Dakle, u datom skupu polupravih postoje dve poluprave a i b koje pokrivaju celu pravu p.
6. Ako je 1 , . . . , n konacan skup od n (n 4) konveksnih povrsi jedne
ravni od kojih svake tri imaju najmanje jednu zajednicku tacku, dokazati da
svih n povrsi ima takode bar jednu zajednicku tacku (Helijeva teorema u ravni).
Uputstvo: Dokaz izvedimo metodom matematicke indukcije. Stoga najpre
dokazi- mo slucaj kada je n = 4. U tom cilju obelezimo sa S1 zajednicku tacku
povrsi 2 , 3 , 4 ; sa S2 zajednicku tacku povrsi 1 , 3 , 4 ; sa S3 zajednicku
tacku povrsi 1 , 2 , 4 ; sa S4 zajednicku tacku povrsi 1 , 2 , 3 . Pri tome su
neke od tacaka S1 , S2 , S3 , S4 istovetne ili su sve medusobno razlicite. Ako su bar
dve od tih tacaka npr. S1 i S2 istovetne, ta tacka pripada povrsima 1 , 2 , 3 , 4 .
Ako su sve tacke S1 , S2 , S3 , S4 medusobno razlicite, postoje medu njima najmanje tri tacke koje pripadaju jednoj pravoj, ili pak nikoje tri od tih tacaka ne
pripadaju jednoj pravoj.

S1

S3

S2

Slika 9a
U slucaju da medu pomenutim tackama postoje takve tri tacke npr. S1 , S2 , S3
koje pripadaju jednoj pravoj, bice jedna od njih npr. S3 izmedu ostalih dveju
S1 i S2 . S obzirom da obe tacke S1 i S2 pripadaju konveksnom liku 3 , i tacka
S3 koja se nalazi izmedu njih pripada konveksnom liku 3 , te je S3 zajednicka
tacka likova 1 , 2 , 3 , 4 .

S1

S4

S3

S2

Slika 9b
U slucaju da medu tackama S1 , S2 , S3 , S4 nikoje tri ne pripadaju jednoj pravoj,
bice jedna od tih tacaka u trouglu koji je odreden ostalim trima tackama, ili
pak nikoja od njih nije u trouglu koji je odreden ostalim trima tackama. Ako je
npr. tacka S4 u trouglu S1 S2 S3 , tacka S4 je izmedu tacke S1 i neke unutrasnje
tacke S duzi S2 S3 .
S obzirom da tacke S2 i S3 pripadaju konveksnom liku 4 , i tacka S koja se
nalazi izmedu njih pripada liku 4 . Sad su S1 i S tacke konveksnog lika 4 , te
i tacka S4 koja se nalazi izmedu njih pripada liku 4 . Stoga tacka S4 pripada
svim likovima 1 , 2 , 3 , 4 .

S4
S3

S1

S2

Slika 9c
Ako nikoja od tacaka S1 , S2 , S3 , S4 nije u trouglu koji je odreden ostalim trima
tackama, te tacke predstavljaju temena konveksnog cetvorougla S1 S2 S3 S4 . Dijagonale S1 S3 i S2 S4 tog konveksnog cetvorougla seku se u nekoj tacki S. S
obziom da obe tacke S1 i S3 pripadaju konveksnim likovima 2 i 4 , i tacka S
koja se nalazi izmedu njih pripada likovima 2 i 4 .
Takode, kako tacke S2 i S4 pripadaju konveksnim likovima 1 i 3 , i tacka S
koja se nalazi izmedu njih pripada likovima 1 i 3 . Dakle, tacka S pripada
svim likovima 1 , 2 , 3 , 4 . Time je stav za slucaj n = 4 dokazan.
Da bismo dokazali ovaj stav za n > 4 primenicemo princip matematicke indukcije. Zato je dovoljno, pretpostavljajuci da k(4 k < n) takvih povrsi ima
najmanje jednu zajednicku tacku, dokazati da i k + 1 takvih povrsi ima najmanje jednu zajednicku tacku. Povrsi k i k+1 su konveksne te je prema zadatku
3. i njihov presek konveksna povrs. Pri tom svake tri od k konveksnih povrsi
1 . . . k1 , imaju najmanje po jednu zajednicku tacku. Zaista, svake tri od
8

povrsi 1 . . . k1 po pretpostavci imaju najmanje po jednu zajednicku tacku.


Povrs i bilo koje dve od povrsi 1 . . . k1 , npr. povrsi i i j imaju najmanje
jednu zajednicku tacku, jer prema dokazanom delu ovog stava za n = 4 povrsi
i , j , k , k+1 imaju najmanje jednu zajednicku tacku. Dakle, svake tri od k
konveksnih povrsi 1 . . . k1 , imaju najmanje po jednu zajednicku tacku, te
saglasno pretpostavci svih k tih povrsi imaju najmanje jednu zajednicku tacku.
Otuda sledi da i svih k + 1 povrsi 1 . . . k+1 imaju najmanje jednu zajednicku
tacku. Time je stav dokazan.
7. Ako je A1 , . . . , An konacan skup od n tacaka jedne ravni, pri cemu svake
tri od tih n tacaka pripadaju nekoj kruznoj povrsini poluprecnika r, dokazati
da postoji kruzna povrs poluprecnika r koja sadrzi svih n tacaka.
Uputstvo: Da bismo dokazali da postoji kruzna povrs koja sadrzi svih n
tacaka A1 , . . . , An , dokazimo najpre da u toj ravni postoji tacka koja pripada
svim kruznim povrsima 1 , . . . , n kojima su sredista A1 , . . . , An , a poluprecnici
jednaki duzi r. Saglasno pretpostavci, bilo koje tri od navedenih n tacaka, npr.
tacke Ai , Aj , Ak pripadaju nekoj kruznoj povrsi poluprecnika r, pa je srediste te
kruzne povrsi zajednicka tacka povrsi i , j , k . Dakle sve tri od n konveksnih
povrsi 1 , . . . , n imaju bar jednu zajednicku tacku, te prema Helijevoj teoremi
svih n kruznih povrsi 1 , . . . , n ima bar jednu zajednicku tacku O. S obzirom da tacka O pripada svim kruznim povrsima 1 , . . . , n kojima su sredista
A1 , . . . , An a poluprecnici jednaki duzi r, kruz na povrs kojoj je srediste O, a
poluprecnik jednak duzi r sadrzi svih n tacaka A1 , . . . , An . Time je stav dokazan.
8. Ako konacan skup {1 , . . . , n } od n poluravni neke ravni pokriva celu
tu ravan, dokazati da u tom skupu poluravni postoje takve tri poluravni koje
pokrivaju celu tu ravan.
Uputstvo:Dokaz izvedimo indirektno. Stoga pretpostavimo da iskazani stav
nije istinit, tj. da nikoje tri poluravni i , j , k iz datog skupa poluravni ne
pokrivaju celu ravan . Obelezimo sa 1 , . . . , n poluravni komplementne s poluravnima 1 , . . . , n u odnosu na ravan , tj. skupove svih tacaka ravni koje
ne pripadaju respektivno poluravnima 1 , . . . , n . Na taj nacin mi smo formirali u ravni konacan skup 1 , . . . , n od n poluravni, dakle konacan skup od n
konveksnih povrsi. Dokazimo da svake tri poluravni tog skupa imaju najmanje
jednu zajednicku tacku. S obzirom da po pretpostavci poluravni i , j , k ne
pokrivaju celu ravan , postoji u ravni tacka P koja ne pripada ni jednoj od
poluravni i , j , k . Stoga tacka P pripada svakoj od poluravni i , j , k te
svake tri poluravni iz skupa 1 , . . . , n imaju najmanje jednu zajednicku tacku.
Prema Helijevoj teoremi za ravne konveksne likove postoji u ravni tacka Q
koja pripada svim poluravnima 1 , . . . , n . S obzirom da tacka Q pripada svim
poluravnima 1 , . . . , n i ona ne pripada ni jednoj od poluravni 1 , . . . , n . No
to je nemoguce jer po pretpostavci sve poluravni 1 , . . . , n pokrivaju celu ravan . Ovim je stav dokazan.
9. Ako je {l1 , . . . , ln } konacan skup od n kruznih lukova sadrzanih na istom
krugu l pri cemu je svaki od tih lukova manji od poluobima kruga l, a svaka tri
od tih lukova imaju najmanje jednu zajednicku tacku, dokazati da svi likovi tog
kruga imaju najmanje jednu zajednicku tacku.
Uputstvo: Odsecci 1 , . . . , n kruzne povrsi l omedeni lucima l1 , . . . , ln i
9

odgovarajucim tetivama su konveksne povrsi jedne ravni, od kojih svake tri


imaju najmanje jednu zajednicku tacku, te prema Helijevoj teoremi, sve te
povrsi imaju najmanje jednu zajednicku tacku. S obzirom da odsecci 1 , . . . , n
jedne iste kruzne povrsi imaju najmanje jednu zajednicku tacku, i odgovarajuci
lukovi l1 , . . . , ln tih povrsi imaju najmanje jednu zajednicku tacku.
10. Ako za svake tri tacke P , Q, R prostog ravnog poligona A1 , . . . , An
postoji u tom poligonu tacka S takva da sve unutrasnje tacke duzi P S, QS, RS
se takode nalaze u tom poligonu, dokazati da u tom poligonu postoji tacka O
takva da se unutrasnje tacke svih duzi koje spajaju tacku O s tackama tog
poligona takode nalaze u tom poligonu (Teorema M.A. Krasnoseljskog).
Uputstvo:

Ai+1 Ai
Mi

An

A4

A3
A1

A2

A6
A5

Slika 10
Ako je Ai Ai+1 bilo koja stranica prostog ravnog poligona A1 . . . An , postoji u
tom poligonu tacka Mi takva da sve unutrasnje tacke trougaone povrsi Mi Ai Ai+1
budu u tom poligonu. Prava Ai Ai+1 razlaze ravan tog poli- gona na dve poluravni, obelezimo sa i onu od tih poluravni koja sadrzi tacku Mi .
Na taj nacin mi smo dobili n poluravni 1 , . . . , n koje pripadaju istoj ravni,
ravni poligona A1 . . . An . Dokazimo da svake tri od tih n poluravni imaju najmanje jednu zajednicku tacku. Neka su i , j , k bilo koje tri od tih n poluravni, a Pi , Pj , Pk bilo koje unutrasnje tacke stranica Ai Ai+1 , Aj Aj+1 , Ak Ak+1
koje odreduju rubove poluravni i , j , k . Saglasno pretpostavci, u poligonu
A1 . . . An postoji tacka S takva da su unutrasnje tacke duzi SPi , SPj , SPk
takode u tom poligonu. Pri tome tacka S pripada poluravnima i , j , k . Zaista, prava SPi sece stranicu Ai Ai+1 trougla Mi Ai Ai+1 u tacki Pi , prema tome
ona sece jos jednu stranicu tog trougla u nekoj tacki Qi . S obzirom da se unutrasnje tacke obeju duzi Pi Qi i Pi S nalaze u poligonu A1 . . . An , tacka S je
na polupravoj Pi Qi , te pripada poluravni i . Istim postupkom dokazuje se da
tacka S pripada i poluravnima j i k . Na taj nacin, poluravni 1 , . . . , n predstavljaju konacan skup konveksnih likova jedne ravni od kojih svaka tri imaju
najmanje jednu zajednicku tacku, te prema Helijevoj teoremi postoji najmanje
jedna tacka, obelezimo je sa O, koja pripada svim tim likovima. Dokazimo da
je O trazena tacka.
U tom cilju dokazimo najpre da je tacka O u poligonu A1 . . . An . Neka je s proizvoljna prava koja sadrzi tacku O, ne sadrzi ni jedno teme poligona A1 . . . An , a
sece taj poligon; obelezimo sa X onu od presecnih tacaka koja je susedna tacki
10

O. Pri tome je X unutrasnja tacka neke stranice Ap Ap+1 poligona A1 . . . An .


Na taj nacin prava s sece stranicu Ap Ap+1 trougla Mp Ap Ap+1 , prema tome ona
sece jos jednu njegovu stranicu u nekoj tacki Y . Kako su obe tacke O i Y u
poluravni p , one su na pravoj s sa iste strane tacke X. S obzirom da duz XO
sem tacke X nema s datim poligonom zajednickih tacaka i da se unutrasnje
tacke duzi OY koja pripada polupravoj XO nalaze u datom poligonu, i tacka
O je u tom poligonu.
Sad dokazimo da su ove unutrasnje tacke duzi koja spaja tacku O s proizvoljnom tackom P datog poligona u tom poligonu. Neka izmedu tacaka O i P
postoji tacka E koja se nalazi izvan poligona A1 . . . An . Duz P E ima s tim poligonom konacan broj zajednickih tacaka. Neka je Q ona od tih tacaka koja je
susedna tacki E. Tacka Q pripada izvesnoj stranici Aq Aq+1 poligona A1 . . . An
ili se pak poklapa s nekim temenom Aq tog poligona. U prvom slucaju prava
P O sece stranicu Aq Aq+1 trougla Mq Aq Aq+1 u tacki Q, prema tome ona sece jos
jednu stranicu tog trougla u nekoj tacki R. Prema tome, unutrasnje tacke duzi
QR su u poligonu A1 . . . An , a unutrasnje tacke duzi QE izvan tog poligona, te
poluprava, dakle i tacka O ne pripada poluravni q , sto je nemoguce. Ako se
tacka Q poklapa s temenom Aq , tada na jednoj od stranica koje se susticu u
temenu Aq postoji tacka Q takva da izmedu tacaka O i Q ima tacaka koje su
izvan poligona A1 . . . An pa se dokaz izvodi kao u prvom slucaju. Otuda sledi
da su sve tacke duzi OP u poligonu A1 . . . An , pa je stav dokazan.
11. Ako su A1 i A2 bilo koje dve unutrasnje tacke konveksne povrsi ,
dokazati da su sve ostale tacke duzi A1 A2 unutrasnje tacke povrsi .
Uputsvo:

M2

M1
A1

N1

A2
N2

Slika 11
S obzirom da su A1 i A2 unutrasnje tacke povrsi , postoje kruzne povrsi 1 i
2 sa sredistima A1 i A2 kojima sve tacke pripadaju povrsi (Slika 11).
Neka su M1 M2 i N1 N2 spoljasnje dirke povrsi 1 i 2 . Unija cetvorougaone
povrsi M1 M2 N2 N1 i kruznih povrsi 1 i 2 je neka konveksna povrs koja
pripada povrsi .
Svaka unutrasnja tacka P duzi A1 A2 je srediste neke kruzne povrsi koja pripada povrsi , dakle i povrsi . Stoga su sve tacke duzi A1 A2 unutrasnje tacke
povrsi .

11

12. Ako je A1 unutrasnja i A2 granicna tacka konveksne povrsi , dokazati


da su sve ostale tacke duzi A1 A2 unutrasnje tacke povrsi .
Uputstvo:

A2

A1

Slika 12
S obzirom da je A1 unutrasnja tacka povrsi , postoji kruzna povrs 1 sa
sredistem A1 koja pripada povrsi (Sl.12). Neka su A2 M i A2 N dirke povrsi
1 kroz tacku A2 . Unija trougaone povrsi A2 M N i kruzne povrsi 1 je neka
konveksna povrs koja pripada povrsi .
Svaka unutrasnja tacka duzi A1 A2 je srediste neke kruzne povrsi koja pripada
povrsi , dakle i povrsi . Stoga su sve unutrasnje tacke duzi A1 A2 unutrasnje
tacke povrsi .
13. Ako su A1 i A2 dve granicne tacke konveksne povrsi , dokazati da su
unutrasnje tacke duzi A1 A2 ili sve unutrasnje ili sve granicne tacke povrsi .
Uputstvo:

A1

A3
P

A2

Slika 13
Ako bi duz A1 A2 sadrzala neku unutrasnju tacku P povrsi (Sl.13), prema
prethodnom zadatku, sve unutrasnje tacke duzi A1 P i A2 P , prema tome i sve
unutrasnje tacke duzi A1 A2 bile bi unutrasnje tacke povrsi . Ako duz A1 A2
ne sadrzi ni jednu unutrasnju tacku povrsi , sve tacke duzi A1 A2 su granicne
tacke povrsi . U tom slucaju sve tacke duzi A1 A2 pripadaju istoj pravoj
oslonca povrsi .

12

14. Dokazati da svaka prava s kroz bilo koju unutrasnju tacku P konveksne
povrsi moze da ima s rubom te povrsi najvise dve zajednicke tacke.
Uputstvo:

s
A

Slika 14a
s

Slika 14b

Slika 14c
S obzirom da je P unutrasnja tacka povrsi , postoji kruzna povrs sa
sredistem P koja pripada povrsi . Prava s kroz srediste P kruzne povrsi ,
sece tu povrs po dijametru koji pripada preseku prave s i povrsi . Stoga prava
s i povrs imaju pored tacke P jos zajednickih tacaka. Kako su prava s i povrs
konveksni likovi, njihov presek je takode konveksan lik koji se nalazi na jednoj
pravoj. Otuda sledi da je taj presek duz, poluprava ili prava (Sl.14).
Ako je presek duz AB, njeni krajevi su granicne tacke povrsi . S obzirom da
duz AB sadrzi unutrasnju tacku P povrsi , sve unutrasnje tacke duzi AB su
unutrasnje tacke povrsi . U tom slucaju prava s ima s rubom povrsi dve
zajednicke tacke. Ako je presek prave s s povrsi poluprava AP , prava s ima
s rubom povrsi samo jednu zajednicku tacku. Ako je presek prave s s povrsi
prava, tada ona s rubom povrsi nema zajednickih tacaka.

13

15. Dokazati da svaka prava s kroz bilo koju unutrasnju tacku P ogranicene
konveksne povrsi sece granicu te povrsi u dvema tackama.
Uputstvo:

s
P

Slika 15
S obzirom da je P unutrasnja tacka povrsi , postoji kruzna povrs sa
sredistem P koja pripada povrsi (Slika 15).
Prava s kroz srediste te kruzne povrsi sece istu povrs po dijametru koji pripada preseku prave s i povrsi . Stoga prava s ima s povrsi pored tacke P jos
zajednickih tacaka. Kako su prava s i povrs konveksni likovi, njihov presek
je takode konveksan lik. Sem toga, povrs je ogranicena, pa je i njen presek s
pravom s ogranicen. S obzirom da je ogranicen i konveksan lik koji sadrzi
beskonacno mnogo tacaka rasporedenih na pravoj s, s obeju strana od tacke P
je duz kojoj je P unutrasnja tacka. Krajevi A i B te duzi su granicne tacke
povrsi , a prema zadatku 13, unutrasnje tacke duzi AP i BP , dakle i duzi AB
su unutrasnje tacke povrsi . Otuda sledi da prava s ima s granicom povrsi
samo dve zajednicke tacke.
Napomena: Ako konveksna povrs nije ogranicena, svaka prava s kroz bilo
koju unutrasnju tacku P ima s rubom dve, jednu ili pak nema zajednickih
tacaka, prema tome da li je presek prave s s povrsi duz, poluprava ili prava.
16. Ako svaka prava kroz bilo koju unutrasnju tacku ogranicene povrsi
sece rub te povrsi u dvema tackama, dokazati da je povrs konveksna.
Resenje: Pretpostavimo da povrs nije konveksna vec konkavna. To znaci
da postoje dve tacke A i B na povrsi takve da postoje unutrasnje tacke duzi
AB koje ne pripadaju povrsi . Tacka B moze biti ili unutrasnja ili granicna
tacka povrsi . U slucaju da je tacka B granicna, onda postoji njoj bliska unutrasnja tacka B povrsi , takva da duz AB sadrzi i tacke koje ne pripadaju
ovoj povrsi. Zato cemo pretpostaviti da je B unutrasnja tacka povrsi . Neka
je C jedna od tacaka duzi AB koja ne pripada povrsi . Kako tacka A pripada
povrsi , a tacka C ne, onda ce duz AC sadrzati bar jednu granicnu tacku M
povrsi . Analogno, duz BC sadrzi bar jednu granicnu tacku N povrsi .

14

Slika 16
Posto je B unutrasnja tacka ogranicene povrsi , onda prava AB sadrzi bar
jednu granicnu tacku P povrsi koja se nalazi iza B u odnosu na C. Prema
tome, M , N i P su tri granicne tacke prave AB i povrsi , a to je suprotno nasoj
pretpostavci da je povrs konkavna. Odatle sledi da je povrs konveksna, sto
je i trebalo dokazati.
17. Ako se kroz svaku tacku ruba ogranicene povrsi moze konstruisati
najmanje jedna prava oslonca te povrsi, dokazati da je povrs konveksna.
Resenje: Pretpostavimo da povrs nije konveksna vec konkavna. Tada na
povrsi postoje dve tacke A i B takve da sve unutrasnje tacke duzi AB ne
pripadaju povrsi . Tacka A je ili unutrasnja ili granicna tacka povrsi . U
slucaju da je A granicna tacka povrsi , onda postoji njoj bliska unutrasnja
tacka A povrsi , takva da duz A B sadrzi tacke koje ne pripadaju povrsi .
Zato cemo pretpostaviti da je A unutrasnja tacka povrsi . Takode na duzi
AB postoji tacka P koja je granicna tacka povrsi , jer na duzi AB postoje i
tacke koje pripadaju i tacke koje ne pripadaju povrsi . Posto prava AB sadrzi
unutrasnju tacku A povrsi , onda ona nije prava oslonca te povrsi.

Slika 17
Svaka druga prava koja prolazi kroz tacku P nije prava oslonca povrsi jer
na povrsi postoje dve tacke A i B koje su sa raznih strana od te prave. Iz
ovoga sledi da postoji granicna tacka P povrsi kroz koju se ne moze konstruisati nijedna prava oslonca te povrsi. Ovo je suprotno pretpostavci, pa sledi da
je povrs konveksna.
18. Ako je P zajednicka tacka povrsi i prave l oslonca povrsi , dokazati
da je P granicna tacka povrsi .
Resenje: Tacka P pripada povrsi i pravoj l. Ako bi tacka P bila unutrasnja
tacka povrsi , onda bi postojala kruzna povrs sa sredistem p cije sve tacke
pripadaju povrsi . Posto prava l sadrzi srediste p kruzne povrsi , onda povrs
sadrzi tacke s obe strane prave l. Odatle i povrsi pripadaju tacke s obe
15

strane prave, sto je nemoguce jer je l prava oslonca povrsi . Neizostavno sledi
da je P granicna tacka povrsi .
19. Ako je duz M N dijametar konveksne povrsi , dokazati da su prave m
i n, koje su u tackama M i N upravne na pravoj M N , prave oslonca povrsi .
Resenje:

M
n
m
N
n

Slika 18

Pretpostavimo da prave m i n nisu prave oslonca povrsi , vec da su to prave


m i n koje su takode upravne na pravoj M N i koje, saglasno definiciji, imaju
s povrsi zajednicke tacke M i N . Tada je duz M N manja od duzi M N , a
to je nemoguce jer duz M N , kao dijametar povrsi , mora biti najveca od duzi
koje spajaju dve tacke povrsi .

1.2.Kombinatorni zadaci iz apsolutne geometrije


20. Dokazati da skup koji se sastoji iz konacnog broja od n pravih neke
ravni , pri cemu se svake dve od tih pravih seku, a nikoje tri i vise ne seku u
jednoj tacki, razlaze ravan na n = 12 (n2 + n + 2) konveksnih oblasti od kojih
je n = 2n neogranicenih i n = 12 (n2 3n + 2) ogranicenih.
Resenje: U datom skupu k pravih sece (k + 1)-vu pravu u k raznih tacaka
koji razlazu (k +1)-vu pravu na k +1 delova. Svaki od tih delova razlaze neku od
ranije dobijenih oblasti na dve, pa se dodavanjem (k + 1)-ve prave broj oblasti
povecava za k + 1 oblast. Kako je za k = 1 broj oblasti 1 = 2, onda ce za
k = n biti

n = 2 + 2 + 3 + 4 + + n
= 1 + (1 + 2 + 3 + + n)

16

=1+
=

n(n + 1)
2

1 2
(n + n + 2).
2

Da bi se odredio broj neogranicenih i broj ogranicenih oblasti, odredimo najpre


za koliko se povecava broj neogranicenih oblasti dodavanjem (k + 1)-ve prave.
Svaki od ona dva neogranicena dela na (k + 1)-oj pravoj razlaze izvesnu oblast
dobijenu razlaganjem prethodnih k pravih na dve neogranicene oblasti, a svaki
ograniceni deo na (k+1)-oj pravoj razlaze oblast u kojoj se nalazi na dve oblasti,
od kojih je jedna istog tipa a druga obavezno ogranicena. Stoga se dodavanjem
(k + 1)-ve prave broj neogranicenih oblasti povecava za dva. Kako je za k = 1
broj neogranicenih oblasti 1 = 2, onda ce za k = n taj broj iznositi n = 2n.
Znamo da je n = n + n (iz formulacije zadatka). Odatle je

n = n n =

1 2
1
(n + n + 2) 2n = (n2 3n + 2)
2
2
.

Svaka od dobijenih oblasti je presek od n poluravni, dakle n konveksnih povrsi,


pa je samim tim svaka od tih oblasti konveksna.
21. Dokazati da skup koji se sastoji iz konacnog broja od n krugova neke
ravni , pri cemu se svaka dva kruga iz tog skupa seku, a nikoja tri i vise ne
seku u jednoj tacki, razlaze ravan na n2 n + 2 oblasti.
Resenje: Najpre dokazujemo da se dodavanjem (k + 1)-og kruga skupu od
k datih krugova broj oblasti poveca za 2k. Zaista, s obzirom da (k + 1)-vi krug
sece prvih k krugova u 2k tacaka, 2k tacaka razlaze taj (k + 1)-vi krug na 2k
delova i svaki od tih delova razlaze neku od oblasti koje su dobijene razlaganjem
ravni sa prvih k krugova na dve oblasti, tada se dodavanjem (k + 1)-og kruga
ravni dobija novih 2k oblasti. Stoga, ukupan broj oblasti za k = n iznosi

n = 2 + 2 + 4 + 6 + 8 + + 2(n 1)
= 2 + 2[1 + 2 + 3 + + (n 1)]
=2+2

n(n 1)
2

= n2 n + 2
.
22. Ako je l broj presecnih tacaka svih dijagonala konveksnog poligona
A1 , . . . , An , kod kojeg se nikoje tri i vise dijagonala ne seku u jednoj tacki,
dokazati da je
17

l=

1
n(n 1)(n 2)(n 3)
24
.

Resenje: Poligon A1 . . . An je konveksan pa se s jedne strane prave odredene


dijagonalom A1 Ak tog poligona nalazi k 2 temena, a s druge strane n k temena. Svaka dijagonala koja spaja dva temena s raznih strana prave A1 Ak sece
dijagonalu A1 Ak . Prema tome, dijagonala A1 Ak sece ostale dijagonale poligona
A1 . . . An u (k 2)(n k) tacaka. Odatle sledi da broj l1 presecnih tacaka svih
dijagonala iz temena A1 iznosi

l1 = 1 (n 3) + 2 (n 4) + + (n 3) 1
= 1 (n 1 2) + 2 (n 1 3) + + (n 3)(n 1 (n 2))

= (n 1)(1 + 2 + 3 + + (n 3)) (1 2 + 2 3 + 3 4 + + (n 3)(n 2))


=

1
1
(n 1)(n 2)(n 3) (n 1)(n 2)(n 3)
2
3
=

1
(n 1)(n 2)(n 3)
6

.
Kako i dijagonale iz svakog drugog temena seku ostale dijagonale tog poligona takode u l1 tacaka i kako se pri tome svaka presecna tacka pojavljuje cetiri
puta, imamo da je

l=

1
nl1
=
n(n 1)(n 2)(n 3)
4
24
.

23. Ako je fn broj poligonskih povrsi koje se dobijaju razlaganjem konveksne


poligonske povrsi A1 , . . . , An njenim dijagonalama, pri cemu se nikoje tri i vise
dijagonala ne seku u jednoj tacki, dokazati da je

fn =

1
(n 1)(n 2)(n2 3n + 12)
24
.

18

Resenje: S obzirom da je poligonska povrs (A1 . . . An ) konveksna, onda je i


poligonska povrs (A1 . . . Ak ) za k + 1 < n takode konveksna. S toga, s jedne
strane prave odredene dijagonalom Ak+1 Ai nalazi se i 1 temena, a s druge
strane ki temena poligonske povrsi (A1 . . . Ak+1 ). Dijagonale poligonske povrsi
(A1 . . . Ak+1 ) koje spajaju dva temena s raznih strana prave Ak+1 Ai seku dijagonalu Ak+1 Ai u (i 1)(k i) tacaka. Te tacke razlazu dijagonalu Ak+1 Ai
na (i 1)(k i) + 1 odsecaka. Svaki od tih odsecaka razlaze na dve izvesnu od
fk + 1 povrsi koje su dobijene razlaganjem povrsi (A1 . . . Ak+1 ) dijagonalama
koje ne polaze iz temena Ak+1 . Primenjujuci tu osobinu na sve dijagonale koje
polaze iz temena Ak+1 , nalazimo da je
fk+1 = fk +1+(21)(k2)+1+(31)(k3)+1+ +((k1)1)(k(k1))+1
= fk + 1 (k 2) + 2 (k 3) + + (k 2) 1 + k 1
= fk + k(1 + 2 + 3 + + k 2)) (1 2 + 2 3 + 3 4 + + (k 2)(k 1)) + k 1
= fk + 1 +

k(k 1)(k 2) k(k 1)(k 2)

+k1
2
3
= fk +

k(k 1)(k 2)
+ k 1.
6

Analogno nalazimo da je

fk = fk1 +

(k 1)(k 2)(k 3)
+k2
6
..
.
f4 = 3 +

321
.
6

Odatle je

1
fk+1 = 3 + [3 2 1 + 4 3 2 + + k(k 1)(k 2)] + [3 + 4 + 5 + + (k 1)]
6
=

1
1
(k + 1)k(k 1)(k 2) + k(k 1)
24
2

19

Prema tome,

fn =

1
1
n(n 1)(n 2)(n 3) + (n 1)(n 2)
24
2
=

1
(n 1)(n 2)(n2 3n + 12)
24

.
24. Dokazati da se oblasti dobijene razlaganjem ravni proizvoljnim pravama
a1 , . . . , an mogu podeliti na dva skupa tako da svaka oblast pripada samo jednom
od tih skupova i da nikoje dve susedne oblasti ne pripadaju istom skupu.
Resenje: Primenicemo metod indukcije. Pretpostavimo da se oblasti na koje
je razlozena ravan sa k pravih a1 , . . . , ak mogu podeliti na dva skupa tako da
svaka oblast pripada jednom od tih skupova i da nikoje dve susedne oblasti ne
pripadaju istom skupu. Prava ak+1 razlaze neke od tih oblasti na po dve oblasti.

Slika 19

Slika 20
Podelimo ponovo sve te oblasti na dva skupa tako da sve oblasti s jedne strane
prave ak+1 ostanu u istim skupovima, a sve oblasti s druge strane ove prave
zamene skupove. Pri tome, svake dve oblasti koje imaju zajednicku stranicu na
pravoj ai (i = 1, . . . , k) pripadaju skupovima (jer obe takve oblasti ili menjaju
20

ili ne menjaju skupove). Saglasno pretpostavci, svake dve oblasti koje imaju
zajednicku stranicu na pravoj ak+1 takode pripadaju raznim skupovima. Na
taj nacin i oblasti dobijene razlaganjem ravni pravama a1 , . . . , ak+1 podeljene
su na dva skupa, pri cemu svaka oblast pripada jednom i samo jednom od tih
skupova i nikoje dve susedne oblasti ne pripadaju istom skupu. S obzirom da je
teorema tacna kada je k = 1, ona je tacna i kada je k = n pa je stav dokazan.
25. Dokazati da ne postoji poligon A1 . . . A2n+1 sa neparnim brojem temena
kome bi sve stranice sekle izvesnu pravu p.
Resenje: Pretpostavimo suprotno, da postoji poligon A1 . . . A2n+1 s neparnim brojem temena kome sve stranice seku izvesnu pravu p. Stranica A1 A2 sece
pravu p, pa su temena A1 i A2 s raznih strana prave p; stranica A2 A3 sece pravu
p, pa su temena A2 i A3 takode s raznih strana prave p. Zato su temena A1 i
A3 s iste srane prave p. Stranica A3 A4 sece pravu p pa su temena A3 i A4 s
raznih strana prave p; stranica A4 A5 sece pravu p, pa su temena A4 i A5 takode
s raznih strana prave p. Zato su temena A3 i A5 , prema tome i temena A1 i
A5 s iste strane prave p. Nastavljajuci ovaj postupak, nalazimo da su temena
A1 i A2n+1 s iste strane prave p, pa bi stranica A1 A2n+1 kojoj su sve tacke s
iste strane prave p sekla pravu p, sto je nemouce. Stoga, ne postoji poligon s
neparnim brojem temena kome bi sve stranice sekle pravu p.
26. Ako su A, B, C tri nekolinearne tacke, dokazati da
(a) postoji prava koja sece produzenja polupravih AB, BC, AC;
(b) ne postoji prava koja sece produzenja polupravih AB, BC, CA.
Resenje: (a) S obzirom da su tacke A, B, C nekolinearne, one su medu sobom razlicite. Iz A 6= B i B 6= C sledi (prema jednoj aksiomi) da postoje tacke
D i E takve da je [B A D] i [C B E]. Pri tome su C, D, E tri nekolinearne
tacke, a B i A tacke takve da je [C B E] i [B A D], pa prema jednoj aksiomi
postoji tacka F takva da je [DF E] i [C A F ]. Iz relacije [DF E] sledi da tacke
D, E, F pripadaju jednoj pravoj s, a iz relacija [B A D], [C B E] i [C A F ]
da su tacke D, E, F na produzenjima polupravih AB, BC, AC. Dakle, postoji
prava s koja sece produzenja polupravih AB, BC, AC.

F
A

Slika 21
(b) Prema dokazanom delu ovog zadatka, svaka prava s koja sece produzenja
polupravih AB i BC, sece i produzenje poluprave AC. S obzirom da prava s
21

sece produzenje poluprave AC, ona ne moze seci i produzenje poluprave CA.
Prema tome, ne postoji prava koja sece produzenja polupravih AB, BC, CA.
27. Ako je cetvorougao ABCD konveksan, dokazati da se njegove dijagonale
seku, i obratno, ako se dijagonale cetvorougla ABCD seku, dokazati da je on
konveksan.
Resenje: Ako je cetvorougao konveksan, tacka C je u konveksnom uglu BAD,
a tacka D u konveksnom uglu ABC, prema tome, poluprava AC sece duz BD

u nekoj tacki O, a poluprava BD sece duz AC u nekoj tacki O .

D
O

A
B

Slika 22
S obzirom da su AC i BD dve razne prave, one, prema tome i poluprave

AC i BD mogu imati najvise jednu zajednicku tacku. Stoga su tacke O i O


istovetne, pa se dijagonale AC i BD cetvorougla ABCD seku u tacki O (Slika
22). Obratno, ako se dijagonale AC i BD cetvorougla ABCD seku u nekoj
tacki O, tacka O je izvan prave AB, tacka C iza O u odnosu na A, a tacka D
iza O u odnosu na B, pa su temena C i D sa iste strane prave AB.
Isto tako su temena D i A sa iste strane prave BC, tacke A i B sa iste strane
prave CD, tacke B i C sa iste strane prave AD. Stoga je cetvorougao ABCD
konveksan.
28. Ako je cetvorougao ABCD konkavan, dokazati da se njegove dijagonale
ne seku, i obratno, ako se dijagonale cetvorougla ABCD ne seku, dokazati da
je on konkavan.
Resenje:

22

Slika 23
Ako bi se dijagonale AC i BD cetvorougla ABCD sekle, prema prethodnom
zadatku, cetvorougao ABCD bio bi konveksan, sto je suprotno pretpostavci,
prema tome, dijagonale konkavnog cetvorougla ABCD se ne seku (Slika 23).
Obratno, ako bi cetvorougao ABCD bio konveksan, prema prethodnom zadatku, njegove dijagonale bi se sekle, sto je suprotno pretpostavci, prema tome,
cetvorougao ABCD kome se dijagonale ne seku je konkavan.

29. Ako duz koja spaja dve unutrasnje tacke naspramnih stranica AD i BC
sece dijagonale AC i BD cetvorougla ABCD, dokazati da se dijagonale tog
cetvorougla seku, tj. da je taj cetvorougao konveksan.
Resenje:
C

D
O
N
M

A
B

Slika 24
Uput. Neka su M i N unutrasnje tacke stranica AD i BC, a P i Q tacke u
kojima duz M N sece dijagonale AC i BD. Iz M P N sleduje da su tacke
M i N , prema tome i tacke B i D sa raznih strana prave AC. Dakle, duz BD
sece pravu AC u nekoj tacki O. Analognim postupkom nazivamo da su tacke
A i C sa raznih strana prave BD, pa je O presek dijagonala AC i BD. Otuda
sleduje da je cetvorougao ABCD konveksan (sl.24).

23

30. Dokazati da je kod svakog konveksnog cetvorougla zbir dijagonala veci


od zbira bilo kojih dveju naspramnih stranica.
Resenje: Prema zadatku 27 dijagonale AC i BD konveksnog cetvorougla
ABCD seku se u nekoj tacki S. Pri tome je AS + BS > AB i CS + DS > CD,
pa je
AS + BS + CS + DS > AB + CD
AC + BD > AB + CD
.
31. Ako je S presek dijagonala konveksnog cetvorougla ABCD i P proizvoljna tacka njegove ravni, dokazati da je
SA + SB + SC + SD 6 P A + P B + P C + P D.
Uputstvo.
S obzirom da je
SA + SC 6 P A + P C

SB + SD 6 P B + P D,

bice
SA + SB + SC + SD 6 P A + P B + P C + P D.
Znak jednakosti vazi samo u slucaju kada je
SA + SC = P A + P C

SB + SD = P B + P D,

tj. kada je tacka P na svakoj od dijagonala AC i BD. No to ce nastupiti samo


u slucaju kada je tacka P istovetna s tackom S.
32. Dokazati da je zbir dijagonala konveksnog cetvorougla veci od poluobima, a manji od obima tog cetvorougla.
Uputstvo.
Dijagonale AC i BD konveksnog cetvorougla ABCD se seku u nekoj tacki S.
Pri tome je
AB < AS + BS,

BC < BS + CS,

CD < CS + DS,

DA < DS + AS,

pa je
1
(AB + BC + CD + DA) < AS + BS + CS + DS = AC + BD.
2
Da bismo dokazali drugi deo stava koristimo relacije
AC < AB + BS,

AC < CD + DA,

BD < AB + AD,

BD < BC + CD.

Sabiranjem odgovarajucih strana nalazimo da je


AC + BD < AB + BC + CD + DA.
Napomena: poslednja nejednakost vazi i u slucaju kada je cetvorougao ABCD
konkavan.
33. Dokazati da je zbir svih dijagonala konveksnog petougla veci od obima
tog petougla.
24

Uputstvo.
Ako je A1 A2 A3 A4 A5 konveksan petougao, bice cetvorougli
A2 A3 A4 A5 , A3 A4 A5 A1 , A4 A5 A1 A2 , A5 A1 A2 A3 , A1 A2 A3 A4
takode konveksni. Prema poznatom stavu, kod konveksnog cetvorougla zbir
dijagonala je veci od zbira bilo kojih dveju njegovih naspramnih stranica, pa je
(slika 25)
A2 A4 + A3 A5

> A2 A3 + A4 A5

A3 A5 + A4 A1
A4 A1 + A5 A2

> A3 A4 + A5 A1
> A4 A5 + A1 A2

A5 A2 + A1 A3
A1 A3 + A2 A4

> A5 A1 + A2 A3
> A1 A2 + A3 A4

Sabiranjem odgovarajucih strana ovih nejednakosti nalazimo da je


A1 A3 + A2 A4 + A3 A5 + A4 A1 + A5 A2 > A1 A2 + A2 A3 + A3 A4 + A4 A5 + A5 A1 .
A4

A5

A3

A1

A2

Slika 25
34. Ako je P proizvoljna tacka u ravni konveksnog poligona A1 . . . An ,
dokazati da je
P A1 + P A2 + . . . + P An >

1
(A1 A2 + A2 A3 + . . . + An A1 ) .
2

Uputstvo.
Iz trojki tacaka P, A1 , A2 ; P, A2 , A3 ; . . . P, An A1 respektivno nalazimo da je
P A1 + P A2 > A1 A2 , P A2 + P A3 > A2 A3 , . . . , P An + P A1 > An A1 .
S obzirom da tacka P ne moze biti istovremeno na svim stranicama poligona
A1 . . . An , u svim dobijenim relacijama ne moze istovremeno vaziti znak jednakosti, pa je otuda
P A1 + P A2 + . . . + P An >

1
(A1 A2 + A2 A3 + . . . + An A1 ) .
2
25

35. Ako je P proizvoljna tacka u konveksnom poligonu A1 . . . An , dokazati


da je
P A1 + P A2 + . . . + P An <

n1
(A1 A2 + A2 A3 + . . . + An A1 ) .
2

Uputstvo.
S obzirom da je
P A1 + P A2
P A2 + P A3
...........
P An + P A1

<
<

A2 A3 + A3 A4 + . . . + An A1
A3 A4 + A4 A5 + . . . + A1 A2
..............................
< A1 A2 + A2 A3 + . . . + An1 An ,

posle sabiranja odgovarajucih strana, nalazimo da je


P A1 P A2 + . . . + P An <

n1
(A1 A2 + A2 A3 + . . . + An A1 ) .
2

36. Tacka D je srediste stranice BC trougla ABC. Ako je AC > AB,


dokazati da je BAD > CAD, i obrnuto, ako je BAD > CAD, dokazati
da je AC > AB.
Uputstvo.
Ako je E tacka simetricna s tackom A u odnosu na tacku D, bice ABD
=
ECD, pa je AB = CE i BAD = CED.
A

Slika 26
Kod trougla ACE je AC > CE, pa je AEC > CAE, tj. BAD > CAD
(slika 26). Drugi deo zadatka dokazuje se indirektnim postupkom.
37. Ako je D bilo koja unutrasnja tacka stranice BC trougla ABC, dokazati da je
AB + AC BC < 2AD.
Uputstvo.
Kod trouglova ABC i ACD imamo da je
AB BD < AD

i AC CD < AD,
26

pa je
AB + AC (BD + CD) < 2AD
i prema tome
AD + AC BC < 2AD.
38. Ako su a, b, c stranice trougla i ma tezisna linija koja odgovara stranici
a, dokazati da je
b + c a < 2ma < b + c.
Uputstvo.
Obelezimo sa ABC trougao cije su stranice BC, CA, AB, jednake duzinama
a, b, c, redom; sa D srediste stranice BC i sa E tacku simetricnu sa tackom A u
odnosu na tacku D. Prema prethodnom zadatku, imamo da je AB+AC BC <
2AD, pa je b + c a < 2ma . Kod trougla ABC imamo da je AE < AC + CE,
pa je 2ma < b + c.
39. Ako su ma , mb , mc tezisne linije trougla i p njegov poluobim, dokazati
da je
p < ma + mb + mc < 2p.
Uputstvo.
Ako su a, b, c stranice trougla, prema prethodnom zadatku imamo da je
b+ca
c+ab
a+bc
pa je

< 2ma
< 2mb
< 2mc

<
<
<

b+c
c+a
a+b

1
(a + b + c) < ma + mb + mc < a + b + c i prema tome
2
p < ma + mb + mc < 2p.

40. Ako je P proizvoljna tacka simetrale unutrasnjeg ugla A, a O proizvoljna tacka simetrale spoljasnjeg ugla A trougla ABC, dokazati da je
|P B P C| 6 |AB AC|

i QB + QC > AB + AC.

Uputstvo.
Ako je C tacka simetricna s tackom C u odnosu na simetralu unutrasnjeg ugla
A, tacka C ce biti na polupravoj AB i to takva da je AC = AC i P C = P C .
S toga je kod trougla P BC
|P B P C| < |AB AC|.
Ako je C tacka simetricna s tackom C u odnosu na simetralu spoljasnjeg ugla
A, bice tacka C iza tacke A u odnosu na tacku B takva da je AC = AC i
QC = QC . S toga je kod trougla QBC
QB + QC > AB + AC.
Ovim smo analizirali slucaj kada su tacke P i Q razlicite od tacke A. Uzmemo
li u obzir i ovaj slucaj dobijamo trazene relacije.
41. Ako je kod trougla ABC stranica AB manja od stranice AC, tacka
D srediste stranice BC, tacka E presek raspolovnice ugla A sa stranicom BC
27

i H podnozje visine iz temena A, dokazati da je AEB < AEC, BE < CE,


a tacka E izmedu tacaka H i D.
Uputstvo.
S obzirom da je AB < AC, izmedu tacaka A i C postoji tacka F takva da je
AB = AF . Pri tome je ABE
= AF E, pa je AEB = AEF . Tacka F je u
konveksnom uglu AEC, pa je AEF < AEC i prema tome AEB < AEC.
Iz navedenih podudarnih trouglova takode sleduje da su uglovi ABE i AF E
jednaki, pa su jednaki i njima naporedni uglovi A BC i EF C. S obzirom da je
BCA < A BC, kod trougla CEF bice i ECF < EF C, pa je EF < CE,
i prema tome BE < CE. S obzirom da je tacka D srediste duzi BC, a F tacka
1
duzi BC takva da je BE < CE, bice CE > BC, tj. CE > CD. Stoga
2
je tacka E iza tacke D u odnosu na tacku C. Ugao AEB manji je od njemu
naporednog ugla AEC, dakle on je ostar, pa je podnozje H upravne iz A na
pravoj BC iza E u odnosu na C. S toga je tacka E izmedu tacaka H i D (slika
27).
A

B
A

Slika 27
42. Ako su ABC i A B C dva trougla kod kojih je AB = A B , AC =
A C i A > A , dokazati da je BC > B C .
Uputstvo.
S obzirom da je A > A u uglu BAC postoji poluprava AD koja sa polupravom AB zahvata ugao jednak s uglom B A C (slika 28).
Neka je D tacka te poluprave takva da je AD = A C . Pri tome je ABD
=
A B C , pa je BD = B C . Ako bi tacka D bila na duzi BC, tj. izmedu tacaka
B i C, imali bismo da je BC > BD, i prema tome BC > B C . Ako tacka D nije
na duzi BC, odnosno pravoj BC, ugao CAD sadrzan je u uglu BAC, te njegova
raspolovnica sece duz BC u nekoj tacki S. Pri tome je ASC
= ASD, pa je
SC = SD. Kod trougla SBD je BS + SD > BD, i prema tome BC > B C .

28

Slika 28
43. Ako su ABC i A B C dva trougla kod kojih je AB = A B , AC =
A C i BC > B C , dokazati da je A > A .
Uputstvo.
Uputstvo: Dokaz se izvodi indirektnim postupkom, koristeci prethodni zadatak.
44. Dokazati da su kod trougla ABC stranice AB i AC medu sobom jednake
ako su mu jednake:

(a) tezisne linije BB1 i CC1 ;


(b) simetrale BB2 i CC2 unutrasnjih uglova B i C;
(v) visine BB3 i CC3 .
Uputstvo.
(a) Dokaz izvodimo indirektno. Zato pretpostavimo da stranice AB i AC nisu
jednake, vec da je npr. AC > AB. Pri tome je kod trouglova BB1 C i
CC1 B B1 BC > C1 CB.

29

C1

B1

Slika 29
Ako je B tacka iza B1 u odnosu na B takva da je BB1 = B1 B i C iza
tacke C1 u odnosu na C takva da je CC1 = C1 C , kod trouglova BB C i
CC B bice BC = CB, BB = CC i CBB > BCC , pa je CB > BC .
S obzirom da je ABB1
= BC C1 imamo da je
= CB B1 i ACC1
CB = AB i BC = AC, pa je AB > AC, sto je suprotno pretpostavci.
Dakle nije AC > AB. Istim postupkom dokazuje se da nije ni AB > AC,
prema tome bice AB = AC (slika 29);
(b) kao u prethodnom delu, dokaz navedimo indirektno. Neka je AC > AB, i
prema tome B > C. Otud sleduje da u uglu ABB2 postoji poluprava
BD takva da je B2 BD = ACC2 .

D
E
B2
C2

Slika 30

30

Kako je ugao ABB2 konveksan, poluprava BD sece duz AB2 u nekoj


tacki D. U trouglu BCD je CBD > BCD, pa je CD > BD. Otud
sleduje da izmedu tacaka C i D postoji tacka E takva da je BD = CE. Iz
podudarnosti trouglova BB2 D i CEC2 sledi da su saglasni uglovi BDC i
C2 EC jednaki, pa su prave BD i C2 E uporedne. No to je nemoguce, jer
su tacke E i C2 sa raznih strana prave BD. Dakle nije AC > AB. Istim
postupkom dokazuje se da nije ni AB > AC, prema tome bice AB = AC
(slika 30);
(v) trouglovi ABB3 i ACC3 su podudarni, pa je AB = AC (slika 31).

C3

B3

Slika 31
45. Neka su AA1 , BB1 , CC1 tezisne linije i AA2 , BB2 , CC2 visine trougla ABC, a A A1 , B B1 , C C1 tezisne linije i A A2 , B B2 , C C2 visine trougla
A B C . Dokazati da su trouglovi ABC i A B C podudarni ako je:
(a) AB = A B , AC = A C , AA1 = A A1 ;
(b) AB = A B , AC = A C , BB1 = B B1 ;
(v) AB = A B , A = A , CC2 = C C2 ;
(g) BC = B C , BB2 = B B2 , CC2 = C C2 ;
(d) BC = B C , AA1 = A A1 , AA2 = A A2 .
Uputstvo.
Neka su AA1 , BB1 , CC1 tezisne linije i AA2 , BB2 , CC2 visine trougla ABC, a
A A1 , B B1 , C C1 tezisne linije i A A2 , B B2 , C C2 visine A B C .
(a) Neka su D i D tacke simetricne tackama A i A u odnosu na tacke A1 i
A1 . Tada su cetvorouglovi ABDC i A B D C paralelogrami (dijagonale
se polove), pa je AA1 B
= D A1 C , odnosno
= DA1 C i A A1 B


AB = CD i A B = C D . Sad je ACD
= A C D , odakle sledi
31

A1 AC = A1 A C . Zatim je AA1 C
= A A1 C , pa je A1 C = A1 C

i prema tome BC = B C . Iz jednakosti AB = A B , BC = B C , CA =
C A sledi da je ABC
= A B C ;
(b) iz podudarnosti trouglova ABB1 i A B B1 sledi da je BB1 C = B B1 C .
Sad je BB1 C = B B1 C , odnosno BC = B C i prema tome ABC
=
A B C ;
(v) iz podudarnosti trouglova ACC2 i A C C2 sledi da je AC = A C , pa je
ABC
= A B C ;
(g) trouglovi BBC2 i B C C2 su podudarni, pa je CBC2 = C B C2 i prema
tome B = B . Isto tako, trouglovi BB2 C i B B2 C su podudarni, pa
je BCB2 = B C B2 i prema tome C = C . Iz jednakosti BC =
B C , B = B , C = C , sledi da je ABC
= A B C ;
(d) trouglovi AA1 A2 i A A1 A2 su podudarni, pa je AA1 A2 = A A1 A2 i
prema tome AA1 B = A A1 B i AA1 C = A A1 C . Sad je AA1 B
=
A A1 B i AA1 C
= A A1 C , pa je AB = A B i AC = A C . Iz
jednakosti AB = A B , BC = B C , CA = C A sledi da je ABC
=
A B C .
46. Ako su tezisne linije AD i uglovi BAD, CAD trougla ABC jednaki
tezisnoj liniji A D i uglovima B A D , C A D trougla A B C , dokazati da je
ABC
= A B C .
Uputstvo.

32

Ako je E tacka simetricna tacki A u odnosu na tacku D, a E tacka simetricna


tacki A u odnosu na tacku D bice ABD
= E C D ,
= ECD i A B D


pa je BAD = DEC i B A D = D E C (sl. 32).
Pri tome je AEC
= A E C , pa je AC = A C i CE = C E . Uz to je jos

vazi i CE = AB i C E = A B pa je i AB = A B . Iz jednakosti AB = A B ,
AC = A C i A = A sledi da je ABC
= A B C .
47. Ako je kod trouglova ABC i A B C razlika stranica AB i AC jednaka
razlici stranica A B i A C , stranica BC jednaka stranici B C i tezisna linija
AD jednaka tezisnoj liniji A D , dokazati da je ABC
= A B C .
Uputstvo.

Uglovi ADB i A D B su jednaki. Zaista, ako bi bilo ADB > A D B ,


odnosno ADC < A D C , imali bismo kod trouglova ABD i A B D da je
AB > A B , a kod trouglova ACD i A C D da je AC < A C (sl. 33).
Otuda je AB AC > A B A C sto je suprotno pretpostavci, dakle nije
ADB > A D B . Analognim postupkom dokazuje se da nije ADB <
A D B , prema tome, bice ADB = A D B . Iz podudarnih trouglova ABD
i A B D sledi da je AB = A B , a iz podudarnih trouglova ACD i A C D da
je AC = A C . Stoga je ABC
= A B C .
48. Ako je kod prostog cetvorougla ABCD A=B i AD=BC, dokazati
da je C=D, a tacka odredena sredistima stranica AB i CD osa simetrije
cetvorougla ABCD.
Uputstvo.

33

P BC, pa je ADP = BCP


Ako je P srediste stranice AB, bice P AD =
i P D = P C. Trougao CDP je jednokrak, te jeCDP = DCP . Sem toga,
tacka P je u uglovima BCD i CDA, pa je BCD = BCP +P CD i CDA =
CDP + P DA
(sl. 34).
Otuda je i BCD = CDA. Da bismo dokazali drugi deo stava, obelezimo
sa Q srediste stranice CD. Pri tome je CP Q
= DP Q, pa su naporedni
uglovi P QC i P QD jednaki, dakle pravi. Otuda sledi da su tacke C i D upravno
simetricne u odnosu na pravu PQ. Isto tako su tacke A i B upravno simetricne
u odnosu na pravu PQ, pa je prava PQ osa simetrije cetvorougla ABCD.
49. Ako je kod prostog konveksnog cetvorougla ABCD A=B i C=D,
dokazati da je AD=BC i AC=BD.
Uputstvo.

34

Pretpostavimo suprotno, tj. da stranice AD i BC nisu jednake, vec da je


npr. AD > BC. U tom slucaju, izmedu tacaka A i D postoji tacka D takva
da je BC = AD . Prema prethodnom zadatku, kod cetvorougla ABCD je
BCD = CD A (sl. 35).
Kako je tacka D u uglu BCD, ugao BCD je manji od ugla BCD te je
ugao CD A manji od ugla CDA. To je nemoguce, jer je ugao CD A spoljasnji,
a ugao CDA unutrasnji nesussedni ugao trougla CDD . Dakle nije AD > BC.
Iz istih razloga nije AD < BC, pa je AD = BC. Iz podudarnosti trouglova
ABC i BAD sledi da je AC = BD.
50. Ako je kod prostog cetvorougla ABCD A = B i D > C, dokazati
da je BC > AD.
Uputstvo.

35

C
D

Ako bi bilo BC < AD, izmedu tacaka A i D postojala bi tacka D takva da


je BC = AD . Prema zadatku 44, imali bismo kod cetvorougla ABCD da je
AD C = BCD . Tacka D je u uglu BCD, te je BCD < BCD, i prema
tome AD C < CDA (sl. 36).
To je nemoguce, jer je AD C spoljasnji, a ugao CDA unutrasnji nesusedni
ugao trougla CDD. Dakle nije BC < AD. Ako bi bilo BC = AD, prema
zadatku 44, imali bismo da je C = D sto je suprotno pretpostavci. Dakle
nije ni BC = AD. Stoga je BD > AD.
51. Ako je kod prostog cetvorougla ABCD A=B i BC AD, dokazati
da je D C.
Uputstvo.

36

Kako je BC > AD, izmedu tacaka B i C postoji tacka C takva da je AD = BC .


Pri tome je ADC = ADC , ADC = BC D i BC D = BCD, pa je
D = C (sl. 37).
52. Ako je kod prostog cetvorougla ABCD AD=BC i A B, dokazati
da je C D.
Uputstvo.

37

Prema zadatku 38, kod trouglova BAD i ABC je BD > AC, a prema zadatku
39, kod trouglova BCD i ADC je C > D (sl. 38).
53. Ako su naspramni uglovi prostog cetvorougla ABCD medu sobom jednaki, dokazati da su njegove naspramne stranice medu sobom takode jednake.
Uputstvo.

38

Slika 39
Neka je kod prostog cetvorougla ABCD A = C i B = D. Pored toga,
pretpostavimo da uglovi CAD i ACB nisu medu sobom jednaki. Neka je
CAD < ACB. U tom slucaju bice i ACD > CAB, pa u trouglu ACD
postoji i tacka D takva da je ABC
= CD A (sl. 39).

Sad je ABC = CD A i CD A > CEA > CDA, pa je ABC >


CDA, sto je suprotno pretpostavci. Dakle nije CAD > ACB. Isto tako
nije CAD < ACB, prema tome je CAD = ACB i ACD = CAB.
Sad su trouglovi ABC i CDA podudarni, pa je AB = CD i BC = DA.
54. Ako je kod prostog cetvorougla ABCD B=D i ako je O srediste
dijagonale AC na dijagonali BD, dokazati da su kod tog cetvorougla naspramne
stranice medu sobom jednake.
Uputstvo.

39

Slika 40
Tacka D simetricna tacki B u odnosu na O je izmedu tacaka O i D, ili je iza
D u odnosu na O ili je istovetna sa D. Ako je D izmedu tacaka O i D, D je u
trouglu ACD , pa je CD A > CDA. Kako je AOB
= COD i BOC
=

D OA. bice AB = CD i BC = D A. Sad je ABC


= CD A. pa je
ABC = CD A i prema tome ABC > CDA, sto je suprotno pretpostavci
(sl. 40).
Dakle, tacka D nije izmedu tacaka O i D. Istim postupkom se dokazuje da
tacka D nije iza D u odnosu na O, pa prema tome, Tacka D je istovetna sa
tackom D, pa su naspramne stranice cetvorougla ABCD medu sobom jednake.
55. Ako je kod prostog cetvorougla ABCD zbir stranica AB i BC jednak
zbiru stranica CD i DA i ako je srediste O dijagonale AC tacka dijagonale BD,
dokazati da su kod tog cetvorougla ABCD naspramne stranice medu sobom
jednake.
Uputstvo.

40

Slika 41
Kao u prethodnom primeru, tacka D simetricna tacki B u odnosu na tacku O
je izmedu tacaka O i D, ili je iza D u odnosu na O, ili je istovetna sa D. Ako je
D izmedu O i D, D je u trouglu ACD, te je AC + CD > AD + CD (sl. 41).
Kako je AOB
= COD i BOC
= D OA bice AB = CD i BC =

AD , pa je AB + BC = AD + CD . Otuda je AD + CD > AB + BC, sto


je suprotno pretpostavci. Dakle, tacka D nije izmedu tacaka O i D. Istim
postupkom dokazuje se se da D nije iza D u odnosu na O. Prema tome, D je
istovetna sa D. Otud sledi da je AB = CD i BC = AD.
56. Ako prave odredene stranicama prostog cetvorougla dodiruju neki krug
koji se nalazi u tom cetvorouglu, dokazati da je kod tog cetvorougla zbir dveju
naspramnih stranica jednak zbiru drugih dveju naspramnih stranica.
Uputstvo.

41

Q
S

Slika 42a
U ovom zadatku, potrebno je razlikovati dva slucaja, prvi kada krug k dodiruje
sve cetiri stranice cetvorougla ABCD (sl. 42(a)) i drugi kada dodiruje samo
dve stranice i produzenja ostalih dveju (sl. 42(b)). Neka su P , Q, R, i S tacke u
kojima krug k dodiruje prave AB, BC, CD i DA redom. U prvom slucaju bice
AB +CD = AP +BP +CR+DR = AS +BQ+CQ+DS = BC +AD (sl.42(a)).

42

Q
C
R

Slika 42b
U drugom slucaju bice npr. [AP B], [ASD], [BCQ], [DCR], pa je AB + CD =
AP + BP + DR CR = AS + BQ + DS CQ = AD + BC (sl. 42(b)).
57. Ako su stranice prostog cetvorougla ABCD na tangentnom krugu k koji
se nalazi izvan tog cetvorougla, dokazati da je razlika dveju naspramnih stranica
jednaka razlici drugih dveju stranica tog cetvorougla.
Uputstvo.

43

Q
C
R

Slika 43a
Kao i u prethodnom primeru, potrebno je razlikovati dva slucaja, prvi kada
krug k ne dodiruje ni jednu stranicu, vec produzenja stranica cetvorougla (sl.
43(a)) i drugi kada krug k dodiruje dve i produzenja drugih dveju(sl.43(b)).
Neka su P , Q, R, S tacke u kojima krug k dodiruje prave AB, BC, CD, DA.
U prvom slucaju bice npr. [ABP ], [BCQ], [DCR], [ADS], pa je AB CD =
BP DR + CR = AS BQ DS + CQ = AD BC (sl. 43(a)).

44

S
D
R
C
Q
A

Slika 43b
U drugom slucaju bice npr. [ABP ], [BCQ], [CED], [ADS], pa je AB CD =
AP BP CR DR = AS BQ CQ DS = AD BC (sl. 43(b)).
58. Ako su stranice slozenog cetvorougla ABCD na tangentama kruga k,
dokazati da je razlika dveju naspramnih stranica jednaka razlici drugih dveju
stranica tog cetvorougla.
Uputstvo.

45

C
D

Slika 44
U ovom slucaju krug k dodiruje dve naspramne stranice i produzenja drugih
dveju. Ako su P , Q, R, S tacke u kojima krug k dodiruje prave AB, BC, CD,
DA, bice npr. [ABP ], [BQC], [CDR], [DSA], pa je AB CD = AP BP
CR + DR = AS BQ CQ + DS = AD BC (sl. 44).
59. Ako je ABCD konveksan cetvorougao kod koga krugovi upisani u trouglove ABC i CDA dodiruju dijagonalu AC u istoj tacki, dokazati da se u
cetvorougao ABCD moze upisati krug.
Uputstvo.

46

C
S

T
Q

Slika 45
Ako su P , Q, R tacke u kojima krug upisan u trougao ABC dodiruje stranice
AB, BC, CA i R, S, T tacke u kojima krug upisan u trougao CDA dodiruje
stranice AC, CD, DA, bice AB + CD = AP + BP + CS + SD = AR + BQ +
CR + T D = AT + T D + BQ + CQ = AD + BC (sl. 45).
60. Ako je ABCD konveksan cetvorougao takav da krugovi upisani u trouglove ABC i CDA dodiruju dijagonalu AC u istoj tacki, dokazati da i krugovi
upisani u trouglove ABD i BCD
Uputstvo.

47

C
S

T
Q

Koristiti prethodni zadatak.


61. Ako su A i B dodirne tacke dveju tangenata a i b nekog kruga k sa
sredistem O, a M1 i M2 dodirne tacke drugih dveju tangenata m1 i m2 , od
kojih prva sece prave a i b u tackama A1 i B1 , a druga sece prave a i b u
tackama A2 i B2 ,dokazati da su uglovi A1 OB1 i A2 OB2 jednaki ili suplementni
zavisno od toga da li su tacke M1 i M2 na istom ili na raznim lucima AB kruga
k.

48

A2

B1

M1

M2

A1

B2

O
A

Slika 46
Uputstvo.
Uputstvo. Uglovi A1 OB1 i A2 OB2 jednaki su polovini istog ili razlicitih
uglova AOB zavisno od toga da li su tacke M1 i M2 na istom ili na raznim
lucima AB kruga k (sl. 46).
62. Dokazati da je poligon
A1 ...An
pravilan ako su sve njegove stranice medu sobom jednake i
n2
uzastopnih unutrasnjih uglova medu sobom jednaki.

49

An1

An2

A1

A2

An

Slika 47
Uputstvo.
Neka su kod polinoma A1 ...An sve stranice medu sobom jednake i uzastopni
uglovi A1 , ..., An2 medu sobom jednaki. Da bismo dokazali da je poligon
A1 ...An pravilan, treba da dokazemo da su i preostala dva unutrasnja ugla
An2 i An jednaka s ostalim unutrasnjim uglovima tog poligona.
S obzirom da su kod poligona A1 ...An1 stranice A1 A2 , ..., A2 A3 , ..., An2 An1
jednake i uglovi A2 , A3 , ..., An2 jednaki bice A2 A1 An1 = An2 An1 A1 .
Trougao A1 An An1 ima jednake stranice A1 An i An1 An , pa je i
An1 A1 An = A1 An1 An .Pri tome je An A1 A2 =
A2 A1 An1 +An1 A1 An i An2 An1 An = An2 An1 A1 + A1 An1 An ,
pa su kod poligona A1 ...An unutrasnji uglovi A1 i An1 medu sobom jednaki.
Istim postupkom dokazujemo da su i uglovi An i An2 medu sobom jednaki,
pa je poligon A1 ...An pravilan(sl. 47).
63. Ako je
A1 ...An
prost poligon s parnim brojem temena cije stranice dodiruju neki krug k sa
sredistem S, dokazati da je (a)
A1 A2 + A3 A4 + ... + An1 An = A2 A3 + A4 A5 + ... + An A1
; (b)
A1 SA2 + A3 SA4 + ... + An1 SAn = A2 SA3 + A4 SA5 + ... + An SA1
50

. Uputstvo. Koristiti stav prema kome su odsecci na tangentama iz iste tacke


na nekom krugu medu sobom jednaki.
64. Dva konveksna n-tougla A1 ...An i A1 ...An imaju sve odgovarajuce stranice jednake izuzev stranica A1 A2 i A1 A2 . Ako su za
= 3, 4, ..., n
svi ili samo neki od uglova A veci od odgovarajucih uglova A , a ostali medu
sobom jednaki, dokazati da je
A1 A2 > A1 A2
.

Ai

Ai

A3

An

An

A1

A2

A1

A2

Slika 48
Uputstvo.
Ako je n=3 stav je poznat. Analizirani slucaj kada je n > 3, pretpostavljajuci
najpre da je za svako v = 3, 4, ..., n Av = Av sem za v = i kada je Ai > Ai
(sl.48). Iz podudarnih poligona A2 ...Ai i A2 ...Ai nalazimo da je A2 Ai = A2 Ai
i A2 Ai Ai1 = A2 Ai Ai1 , a iz podudarnih poligona
Ai Ai+2 ...An A1 i Ai Ai+1 ...An A1 da je A1 Ai = A1 Ai i
A1 Ai Ai+1 = A1 Ai Ai+1 . S obzirom da su poligoni A1 ...An i A1 ...An
konveksni bice Ai = A2 Ai Ai1 + A1 Ai A2 + A1 Ai Ai+1 i
Ai = A2 Ai Ai1 + A1 Ai A2 + A1 Ai Ai+1 te je
51


A1 Ai A2 > A1 Ai A2 Kod
touglova A1 A2 Ai je A1 Ai = A1 Ai , A2 Ai = A2 Ai i

Ai > Ai , pa je A1 A2 > A1 A2 .
Sad pretpostavimo da je za svako v = 3, 4, ..., n Av = Av sem za v = i, j, ..., l
(i < j < ... < l) kada je Ai > Ai , Aj > Aj , ..., Al > Al . Neka je
A1 ...An poligon kome su stranice jednake odgovarajucim stranicama poligona
A1 ...An izuzev stranica A1 A2 i A1 A2 , a za v = 3, 4, ..., n uglovi Av i Av
jednaki sem sto je Ai = Ai , dakle Ai > Ai . Prema dokazanom bice

A1 A2 > A1 A2 . Neka je zatim A


1 ...An poligon kome su stranice jednake
odgovarajucim stranicama poligona A1 ...An izuzev stranice A1 A2 , a za
v = 3, 4, ..., n uglovi Av i A
sto je A
v jednaki sem
j = Aj , dakle


Aj > Aj . Prema dokazanom bice A1 A2 > A1 A2 , itd. Otuda je
A1 A2 > A1 A2 .
65. Dva ravna konveksna n-tougla A1 ...An i A1 ...An imaju sve odgovarajuce
stranice jednake izuzev stranica A1 A2 i A1 A2 , a za
= 3, 4, ..., n
uglovi A nisu manji od odgovarajucih uglova A . Ako je
A1 A2 > A1 A2
, dokazati da je bar jedan od uglova A veci od njemu odgovarajuceg ugla A .

Ai

Ai

An

A3
An

A1

A2

A1

Uputstvo.

52

A2

Uputstvo. Dokaz se moze izvesti indirektnim postupkom, koristeci prethodni


zadatak.
66. Dva konveksna n-tougla
A1 ...An
i
A1 ...An
sa jednakim odgovarajucim stranicama nemaju sve jednake odgovarajuce unutrasnje uglove. Ako temenima n-tougla
A1 ...An
dodelimo znake plus ili minus zavisno da li su uglovi kod tih temena veci ili
manji od odgovarajucih uglova n-tougla
A1 ...An
, dokazati da broj promena ovih znakova uzetih redom kod temena
A1 , ..., An
nije manje od cetiri.

Ak+1

An1

An

M
+

A1

A2

Slika 49

53

Ak

Uputstvo.
S obzirom da svi odgovarajuci uglovi n-touglova A1 ...An i A1 ...An nisu jednaki, n-tougao A1 ...An ima temena kojima su dodeljeni znaci plus ili minus.
Dokazimo da taj n-tougao ima i temena sa znakom plus i temena sa znakom
minus. Pretpostavimo da sva temena n-tougla A1 ...An raspolazu istim znakom,
npr. plus, sem onih temena kojima nije dodeljen nikakav
znak jer su kod njih uglovi jednaki sa odgovarajucim uglovima n-tougla
A1 ...An . Pri tome su kod konveksnih n-touglova A1 ...An i A1 ...An
odgovarajuce stranice Ai Ai+1 i Ai Ai+1 za i = 1, ..., n medusobom jednake a
uglovi Aj veci ili jednaki s uglovima Aj za j = 2, ..., n 1 pa je prema zadatku
64. An A1 > An A1 , sto je nemoguce jer su ove stranice po pretpostavci takode
jednake. Ovim smo dokazali da sva temena poligona A1 ...An ne mogu
raspolagati istim znakom. Stoga postoje promene ovih znakova uzetih redom
kod temena A1 , ..., An . Broj tih promena ne moze biti neparan, jer se polazeci
od nekog temena sa znakom plus obilazeci redom ostala temena tog poligona
dolazi do istog temena koji ima znak plus. Dokazimo da broj tih promena ne
moze biti jednak dvojci. Ako bi broj promena tih znakova bio jednak dvojci,
bice npr. temena A1 , ..., Ak sa znakom plus, a temena Ak+1 , ..., An sa znakom
minus. Medu njima moze da bude i temena bez znaka. Neka su M i N
proizvoljne unutrasnje tacke stranica A1 An i Ak Ak+1 , a M i N tacke stranica
A1 An i Ak Ak+1 takve da je A1 M = A1 M i Ak N = Ak N , i prema tome
M An = M An i N Ak+1 = N Ak+1 . Stoga je prema zadatku 64. kod
konveksnih poligona M A1 ...Ak N i M A1 ...Ak N M N > M N , a kod
konveksnih poligona N Ak+1 ...An M i N Ak+1 ...An M M N < M N . Dobijene
dve nejednakosti su protivrecne, te broj promena uvedenih znakova ne moze
biti jednak dvojci. Ovim smo dokazali da broj promena uvedenih znakova
uzetih redom kod temena A1 , ..., An nije manji od cetiri.
67. Ako od pet komplanarnih tacaka A1 , A2 , A3 , A4 , A5 nikoje tri ne pripadaju jednoj pravoj, dokazati da medu njima postoje cetiri tacke koje predstavljaju temena konveksnog cetvorougla.

54

A1

A5
A4
N

A2

Slika 50
Uputstvo.
Ako medu datim tackama A1 , ..., A5 nikoje cetiri ne bi odredivale konveksan
cetvorougao, tada bi dve od tih tacaka bile u trouglu koji je odreden ostalim
trima tackama. Neka su npr. tacke A4 i A5 u trouglu A1 A2 A3 (v. sl. 50).
Prava A4 A5 sece dve stranice trougla A1 A2 A3 , recimo stranice
A1 A2 i A1 A3 u tackama M i N . S obzirom da se obe tacke A4 i A5 nalaze s iste strane od prave A2 A3 i da prava A4 A5 ne sece duz A2 A3 , tacke
A2 , A3 , A4 , A5 odredivale bi konveksan cetvorougao, sto je suprotno pretpostavci. Ovim je teorema dokazana.
68. Ako svake cetiri od n komplanarnih tacaka A1 ...An predstavljaju temena
konveksnog cetvorougla, dokazati da za datih n tacaka predstavljaju temena
nekog konveksnog n-tougla.

55

Aj

Ak1

Ai

A3

A2

A4

Ak+1

Ai

A3

Ak

Ak+1

A1

A2

Aj

Ak1

Slika 51
Uputstvo.
Ako je n=4, stav sleduje neposredno. Ako je n > 4 dokaz izvodimo indukcijom. Stoga pretpostavimo da je teorema tacna za k (k 4) tacaka, dokazimo
da je ona tacna i za k+1 tacaka. Neka su A1 , ..., Ak+1 tacke jedne ravni takve da
svake cetiri tacke iz tog skupa odreduju neki konveksan cetvorougao. S obzirom
da je teorema tacna za k takvih tacaka, postoji koveksan poligon od k stranica
kome su temena tacke A1 , ..., Ak . Te tacke prenumerisemo tako da pomenuti
konveksni poligon bude A1 ...Ak . Tacka Ak+1 ne moze biti u poligonu A1 ...An .
Zaista, ako bi se tacka Ak+1 nalazila u njemu,ona bi se nalazila u jednom od
trouglova A1 A2 A3 , A1 A3 A4 , ..., A1 Ak1 Ak , npr. u trouglu A1 Ai Ai+1 . U tom
slucaju tacke A1 , Ai , Ai+1 , Ak+1 ne
predstavljaju temena konveksnog cetvorougla sto je suprotno pretpostavci. Stoga
tacka Ak+1 nije u poligonu A1 ...Ak . Tacka Ak+1 ne moze biti na tom poligonu,
jer bi u tom slucaju u datom skupu tacaka postojale tri kolinearne tacke, sto je
nemoguce. Stoga je tacka Ak+1 izvan poligona A1 ...Ak .
Konstruisimo sve poluprave koje polaze iz tacke Ak+1 a sadrze temena tog
poligona. S obzirom da je taj poligon konveksan, a tacka Ak+1 izvan njega, u
konstruisanom skupu polupravih postoje dve poluprave, obelezimo ih sa Ak+1 Ai
i Ak+1 Aj , koje obrazuju konveksan ugao pri cemu su sve ostale poluprave iz pomenutog skupa u tom uglu.Dokazimo da su Ai i Aj susedna temena poligona
A1 ...Ak . Ako te tacke ne bi bile susedna temena, postojalo bi najmanje jedno
teme Ae tog poligona koje se nalazi u trouglu Ai Aj Ak+1 . U tom slucaju tacke
Ai , Aj , Ak+1 , Ae ne bi predstavljale temena konveksnog cetvorougla, sto je su56

protno pretpostavci. Stoga su Ai i Aj susedna temena poligona A1 ...Ak pa su


i tacke A1 , ..., Ak+1 temena izvesnog konveksnog poligona. Ovim je teorema
dokazana.
69. Dokazati da je zbir dvaju unutrasnjih uglova trougla manja od zbira
dvaju pravih uglova.

Slika 52
Uputstvo.
Dokazimo da je kod trougla ABC npr. B + C < 2R, gde je R prav ugao.
Spoljasnji ugao ACD veci je od unutrasnje nesusednog ugla B tog trougla, pa
je B + C < C + ACD = 2R (sl. 52).
70. Ako je bilo koji trougao, dokazati da postoji trougao 1 takav da je
zbir S1 unutrasnjih uglova trougla 1 jednak zbiru S unutrasnjih uglova trougla
, a jedan od uglova trougla 1 bar dvaput manji od naznacenog ugla trougla
.

57

Slika 53
Uputstvo.
Ako obelezimo sa A, B, C temena trougla , sa D srediste stranice AC i
sa E tacku simetricnu sa B u odnosu na D, bice ABD
= CED, pa je
ABD = CED i BAD = ECD, tj ABE = CEB i BAC = ECA.
Pri tome je
ABC + BCA + CAB = ABE + EBC + BCA + CAB =
CEB + EBC + BCA + ACE = CEB + EBC + BCE, pa su
zbirovi unutrasnjih uglova trouglova ABC i EBC jednaki. Sem toga, ugao B
trougla ABC jednak je zbiru uglova B i E trougla EBC, pa je jedan od
uglova E i B trougla EBC bar dva puta manji od ugla B trougla ABC. Stoga
su E, B, C temena 1 koji ima navedene osobine (sl. 53).
71. Dokazati da zbir unutrasnjih uglova trougla ne moze biti veci od zbira
dvaju pravih uglova.

58

An1

A1

B1

B2

Bn1

Bn

Slika 54
Uputstvo.
Prvi nacin. Pretpostavimo naprotiv da postoji trougao kod koga je zbir
S unutrasnjih uglova veci od zbira dva prava ugla, tj. da je S = 2R + , gde
je R prav a bilo koji ugao. Prema prethodnom zadatku, postoji trougao 1
kod koga je zbir S1 unutrasnjih uglova jednak zbiru S unutrasnjih uglova trougla , a jedan njegov unutrasnji ugao 1 , bar dvaput manji od naznacenog
unutrasnjeg ugla , zatim postoji trougao 2 kod koga je zbir S2 unutrasnjih
uglova jednak zbiru S1 unutrasnjih uglova trougla 1 , a jedan njegov unutrasnji
ugao 2 bar dvaput manji od unutrasnjeg ugla 1 trougla 1 . Nastavljajuci taj
postupak, zakljucujemo da postoji trougao n kod koga je zbir Sn unutrasnjih
uglova jednak zbiru Sn1 unutrasnjih uglova trougla n1 , a jedan njegov ugao
n bar dvaput manji od ugla n1 trougla n1 . Stoga je takode Sn =2R+
i n 21n . Ako broj n izaberemo tako veliki da je 21n < , bice tim
pre n < . U tom slucaju bice zbir ostala dva ugla trougla n veci od zbira
dvaju pravih uglova, sto je protivrecno sa zadatkom 69. Otuda sleduje da zbir
unutrasnjih uglova trougla ne moze biti veci od zbira dvaju pravih uglova.
Drugi nacin. Naprotiv,neka postoji trougao ABB1 kome je zbir unutrasnjih
uglova veci od zbira dvaju pravih uglova. Na polupravoj BB1 odredimo tacke
B2 , ..., Bn takve da je [BB1 B2 ...Bn ] i BB1 = B1 B2 = ... = Bn1 Bn (sl. 54),
zatim s one strane prave BB1 s koje je A odredimo tacke A1 , ..., An1 takve da
je ABB1
= ...
=
= ...
= An1 Bn2 Bn . Pri tome je AA1 B1
= A1 B1 B2
An2 An1 Bn1 pa je AA1 = A1 A2 = ... = An2 An1 . Zatim je BAB1 >
59

A1 B1 A, pa iz trouglova ABB1 i A1 B1 A sledi da je BB1 > AA1 . Pri tome


je BA + AA1 + ... + An2 An1 + An1 Bn > BBn tj. n (BB1 AA1 ) <
ABAA1 +AB1 . S obzirom da je BB1 > AA1 , bice BB1 AA1 > 0, pa izvedena
nejednakost protivreci Arhimedovoj aksiomi neprekidnosti. Otud sleduje da zbir
unutrasnjih uglova trougla ne moze biti veci od zbira dvaju pravih uglova.
72. Dokazati da zbir unutrasnjih uglova prostog ravnog n-tougla ne moze
biti veci od zbira (2n 4) pravih uglova. Prema poznatoj teoremi, unutrasnjim
dijagonalama moze se povrs ogranicena tim poligonom razloziti na n 2 trougaonih povrsi kojima unutrasnji uglovi sacinjavaju unutrasnje uglove n-to ugla.
S obzirom da ne postoji trougao kome je zbir unutrasnjih uglova veci od zbira
dvaju pravih uglova, ne moze zbir unutrasnjih uglova svih dobijenih trouglova
biti veci od zbira (2n 4) pravih uglova. Stoga, ne moze ni zbir unutrasnjih
uglova prostog ravnog n-tougla biti veci od zbira (2n 4) pravih uglova.
73. Ako su M i N dve razne tacke jednog kraka ostrog ugla XOY , a M
i N upravne projekcije tih tacaka na drugom kraku i ako je tacka M izmedu
tacaka O i N , dokazati da je (a) OM M ON N ; (b) M M N N .
y

Slika 55
Uputstvo.
(a)Kod konveksnog cetvorougla M N N M unutrasnji ugloviM i N su pravi,
te zbir druga dva unutrasnja ugla M i N nije veci od zbira dva prava ugla. Otuda
sleduje da spoljasnji ugaoOM M kod temenaM nije manji od unutrasnjeg ugla
n-tog cetvorougla, tj. da je OM M ON N (sl. 55).
60

(b)S obzirom da je ugao OM M ostar, njegov naporedni ugao M M N je tup.


Neka je N tacka polupraveN N takva da je M N =N N . Unutrasnji uglovi
M i N konveksnog cetvorouglaM N N M su pravi, a stanice M M i N N
jednake, pa jednaki unutrasnji uglovi kod temena M i N nisu tupi. Otuda
sleduje da je poluprava M N u konveksnom uglu M M N , i prema tome da je
tacka N izmedu tacaka N i N . Stoga je N N N N , tj. M M N M .
74. Ako je C podnozje visine iz temena C pravog ugla trougla ABC, dokazati da je ACC ABC.

A
C
D

Slika 56
Uputstvo.
Neka su D i D tacke polupravih AC i AB takve da je AD=AB i AD =AC.
Pri tome je ABC
= ADD , pa je ABC = ADD , a ugao D , C i D
na jednom kraku ostrog ugla A trougla ABC, a C i D upravne projekcije tih
tacaka na drugom kraku tog ugla i pri tome tacka C izmedu tacaka A i D, pa je
prema prethodnom zadatku ACC ADD , i prema tome ACC ABC
(sl. 56).
75. Dokazati da manjoj visini trougla odgovara veca stranica, i obrnuto, da
manjoj stranici trougla odgovara veca visina.

61

B
D
C

Slika 57
Uputstvo.
Neka je kod trougla ABC visina BB manja od visine CC ; dokazimo da
je AC > AB. Ako obelezimo sa D i D tacke polupravih AC i AB tako da je
AB = AD i AB = AD , bice ABB
= ADD , pa je BB = DD , a ugao

D prav. Ako bi bilo AC < AB, tacka D bi bila iza C u odnosu na A, pa bi


prema zadatku 65, kod cetvorougla C D DC bilo DD > CC , i prema tome
BB > CC , sto je suprotno pretpostavci. Ako bi bilo AC = AB, tacke C i D
bile bi istovetne, a duzi CC i DD jednake, sto je takode suprotno pretpostavci.
Dak
76. Ako neka prava s kroz srediste P osnovice BC jednakokrakog trougla
ABC sece prave AC i AB u tackama Q i R takvim da je tacka P izmedu tacaka
Q i R, dokazati da je QR = BC.
Uputstvo.
Ako je prava s razlicita od prave BC, ona ne sadrzi ni jedno teme trougla
ABC. U tom slucaju prava s sece stranicu BC trougla ABC, te prema Pasovom
stavu, ona sece jos jednu njegovu stranicu, npr. AC. Pri tome je [AQC] i [ABR].
Spoljasnji ugao P BR trougla ABC veci je od unutrasnjeg ugla P CQ, pa u uglu
P BR postoji poluprava BD takva da je /angleP CQ = /angleP BD. Kako
je ugao P BR konveksan, poluprava BD sece duz P R u nekoj tacki D, pa je
P R > P D. Iz podudarnosti trouglova P BD i P CQ sledi da je P D = P Q, pa
je P R > P Q. Ako zatim obelezimo sa Q i R podnozja upravnih iz tacaka
62

Q i R na pravoj BC, bice RR > QQ , pa je kod trouglova BRR i CQQ


BR > CQ , i prema tome Q R > BC. Iz QR > Q R i Q R > BC, sledi da je
QR > BC. Ukljucujuci I slucaj kada je prava s istovetna s pravom BC, imamo
da je QR = BC (sl. 58).

B
P

s D
R

Sl. 58
77. Dokazati da je simetrala jedne stranice trougla upravna na pravoj koja
je odredena sredistima drugih dveju stranica tog trougla.
Uputstvo.
Obelezimo sa P , Q, R sredista stranica BC, CA, AB trougla ABC, sa A ,
B , C podnozja upravnih iz tacaka A, B, C na pravoj QR i sa S srediste duzi
B C . Pri tome je

AA R
= BB R
AA Q
= BB Q

, pa je AA = BB i AA = CC, i prema tome BB = CC. Sad je


SBB
= SCC
, pa je BS = CS. Zatim je
SP B
= SP C
63

pa je
SP B = SP C
S obzirom da su uglovi SP B i SP C naporedni i jednaki, oni su pravi, dakle
bice SP ?BC. Stoga je prava SP simetrala stranice BC. Iz
SBB
= SCC
i

SP B
= SP C

sledi i takode da je
BSB = CSC
i
BSP = CSP
. No tacka B je u uglu BSP , a tacka C u uglu CSP , pa je
B SP = C SP
S obzirom da su uglovi B SP i C SP jednaki i naporedni, oni su pravi, pa je
SP B C , tj. SP QR (sl. 59).

Sl. 59
78. Ako su A, B, C tri razne tacke neke prave l i A , B , C istim redom
tacke neke druge prave l takve da je AB = A B i BC = B C , dokazati da
64

sredista P , Q, R duzi AA , BB , CC pripadaju jednoj pravoj ili se poklapaju


(Teorema Hjelmsleva).
Uputstvo.
Obelezimo sa B i C tacke simetricne s tackama B i C u odnosu na tacku
P . S obzirom da su tacke A, B, C kolinearne, i njima centralno simetricne
tacke A , B, C su kolinearne i na isti nacin rasporedene. Sem toga, duzi
te su i duzi A B i
AB i BC respektivno su jednake sa duzima A B i B C,

B C respektivno jednake sa duzima A B i B C. Stoga su trouglovi A B B i


A C C jednakokraki sa zajednickim uglom pri vrhu A , te su im simetrale stranice B B i C C istovetne. Prema prethodnoj teoremi ta zajednicka simetrala
duzi B B i C C upravna je na srednjoj liniji P Q trougla BB B i srednjoj
liniji P R trougla CC C. S obzirom da postoji samo jedna prava koja sadrzi
tacku P i upravna je na toj simetrali, bice prave P Q i P R istovetne. Stoga
tacke P , Q, R pripadaju jednoj pravoj (sl. 60).

C
B

A
B
C

Sl. 60
79. Ako sve tacke konacnog skupa tacaka ne pripadaju jednoj pravoj, dokazati da postoji prava koja sadrzi samo dve tacke tog skupa tacaka.
Uputstvo.
U konacnom skupu tacaka koji ne pripada jednoj pravoj postoje tri tacke A,
B, C takve da je odstojanje jedne od njih, npr. tacke A, od prave p odredene
65

ostalim dvema tackama B i C, najmanje. Prava p sadrzi samo dve tacke B i


C datog skupa tacaka. Zaista ako bi prava p pored tacaka B i C sadrzala jos
neku tacku D iz ovog skupa, bar dve od tacaka B, C, D, npr. C i D bice s iste
strane podnozja N upravne iz tacke A na pravoj p. Pri tome je npr. tacka C
izmedu tacaka N i D. Ako obelezimo sa C i N podnozja upravnih iz tacaka C
i N na pravoj AD, bice CC < N N < AN , pa je odstojanje tacke C od prve
AD manje od odstojanja tacke A od prave BC, sto je nemoguce. Stoga prava
p sadrzi samo dve tacke B i C datog skupa tacaka (sl.61).

p
B

Sl. 61

Napomena. Moze se dokazati da takvih pravih ima vise. Stavi


se L, M . Keli
3m
i V.O. Mozer su dokazali da broj takvih pravih nije manji od
, gde je m broj
7
tacaka datog skupa.
80. Ako svaka prava odredena dvema tackama nekog konacnog skupa od n
tacaka sadrzi najmanje jos jednu tacku tog skupa, dokazati da svih n tacaka tog
skupa pripadaju jednoj pravoj.
Uputstvo.
Pretpostavimo da svih n tacaka datog skupa ne pripadaju jednoj pravoj.
Pri tome, u datom skupu postoje takve tri tacke A, B, C da je odstojanje jedne
od njih, npr. tacke A, od prave p odredeno drugim dvema tackama najmanje.
Prava p sadrzi pored tacaka B i C bar jos jednu tacku D datog skupa. Neka
je N podnozje upravne iz tacke A na pravoj p. Od tri tacke B, C, D, dve su s
66

iste strane tacke N , npr. C i D. Neka je tacka C izmedu tacaka N i D. Ako


obelezimo sa C i N podnozja upravnih iz tacaka C i N na pravoj AD, imamo
da je CC N N < AN , pa je odstojanje tacke C od prave AD manje od
odstojanja tacke A od prve BC, sto je nemoguce. S toga sve tacke datog skupa
pripadaju jednoj pravoj (sl. 62).

Sl. 62
Napomena. Ovaj se stav moze dokazati i indirektnim postupkom koristeci
prethodni zadatak.
81. Ako je u ravni dat konacan skup od n pravih pri cemu se svake dve od
tih pravih seku u tacki koja pripada bar jos jednoj pravoj iz tog skupa, dokazati
da se svih n datih pravih seku u jednoj tacki.
Uputstvo.
Pretpostavimo da se sve prave datog skupa ne seku u jednoj tacki; drugim
recima pretpostavimo da u datom skupu postoji prava p koja ne sadrzi sve
presecne tacke ostalih pravih tog skupa. Neka je A ona od tih tacaka kojoj je
odstojanje od prave p najmanje; ako takvih tacaka ima vise, neka je A bilo koja
od njih. Saglasno pretpostavci, u datom skupu postoje najvise tri prave koje
sadrze tacku A, neka one seku pravu p u tackama, recimo B, C, D. Jedna od
tacaka B, C, D je izmedu ostalih dveju, neka je npr. tacka D izmedu tacaka B
i C. Sem pravih AD i BC u datom skupu postoji bar jos jedna prava recimo
s, koja sadrzi tacku D. Prava s ne sadrzi ni jedno teme trougla ABC, a sece
stranicu BC u tacki D, te prema Pasovom stavu, mora seci jos jednu stranicu
67

tog trougla, npr. stranicu AC u nekoj tacki E. Pri tome je odstojanje tacke E
od prave p manje od odstojanja tacke A od prave p, sto je suprotno pretpostavci.
Stoga u datom skupu ne postoji prava koja ne sadrzi sve presecne tacke ostalih
pravih tog skupa, prema tome sve prave datog skupa seku se u jednoj tacki (sl.
63).

s
A

p
B

Sl. 63

68

PARALELNOST

2.1

Paralelnost pravih

U ovom clanu svrstani su zadaci za cije je resavanje neophodno pretpostaviti


da je uvedena aksioma paralelnosti euklidske geometrije i da su izvedene osnovne
teoreme koje iz nje proizilaze. To je pre svega stav o zbiru unutrasnjih uglova
poligona, stav prema kome je u svakom krugu sredisnji ugao dva puta veci od
periferijskog nad istim lukom, stav prema kome se prave odredene visinama
trougla seku u jednoj tacki koju nazivamo ortocentrom tog trougla, stav prema
kome se tezisne linije trougla seku u jednoj tacki koju nazivamo tezistem tog
trougla, itd.
82. Ako je ABC trougao kome ugao A nije prav. D tacka iza A u odnosu
na B takva da je AD = AC, a E podnozje upravne iz B na pravoj koja sadrzi
D i uporedna je sa stranicom AC, dokazati da je duz BE jednaka zbiru visina
BB1 i CC1 trougla ABC.
Uputstvo.
Ako obelezimo sa F podnozje upravne iz tacke A na pravoj DE, bice ADF
=
CAC1 , pa je CC1 = B1 E. Tacka B1 je izmedu tacaka B i E, pa je BE =
BB1 + B1 E = BB1 + CC1 (sl. 64).

Sl. 64
83. Ako je ABC trougao kome ugao A nije prav. D tacka poluprave AB
69

takva da je AD = AC, a E podnozje upravne iz B na pravoj koja sadrzi D I


uporedna je sa stranicom AC, dokazati da je duz BE jednaka razlici visina B1
i CC1 trougla ABC.
Uputstvo.
Ako obelezimo sa F podnozje upravne iz tacke D na pravoj AC, bice ADF
=
ACC1 , pa je DF = EB1 . No tacka E je na polupravoj B1 B, pa je duz GE
jednaka razlici duzi BB1 i EE1 , tj. razlici visina BB1 i CC1 trougla ABC
(sl.65).

C
F
B

D
E

Sl. 65
84. Ako ugao A trougla ABC nije prav iako su M i N tacke polupravih
BC i CB takve da je BAM = C i CAN = B, dokazati da je AM N
jednakokrak trougao.
Uputstvo.
Tacke M i N na pravoj BC su razlicite. Zatim, ako bi one bile istovetne,
ugao A trougla ABC bio bi razlozen polupravom AM na uglove B i C, daleke
bio bi prav, sto je suprotno pretpostavci. Otud zakljucujemo da su A, M , N
temena trougla. Pri tome su tacke B i C trougla ABC. U drugom slucaju bice
uglovi AM N i AN M istovetni uglovima M i N trouglova ABM i ACN , pa su
jednaki uglu A trougla ABC. U oba slucaja uglovi AM N i AN M su jednaki
pa je trougao AM N jednakokrak (sl. 66).
70

Sl. 66
85. Ako je D tacka u kojoj simetrala ugla A sece stranicu BC trougla
ABC, S srediste upisanog kruga i P tacka u kojoj taj krug dodiruje stranicu
BC, dokazati da je BSP = CSD.
Uputstvo.
Ako sa R obelezimo prav ugao,iz pravouglog trougla BSP imamo da je
1
BSP = R ABC
2
ugao CSD je spoljasnji za trougao ACS, pa je
CSD =

1
1
(BAC + BCA) = R ABC
2
2

. Otuda je BSP = CSD (sl. 67).

71

Sl. 67
86. Ako su A , B , C sredista stranica BC, CA, AB trougla ABC i ako
je D podnozje visine iz temena A, dokazati da je pri AC > AB
A B D = A C D = B C
.
Uputstvo.
S obzirom da je AC > AB tacka D je iza tacke A u odnosu na C, pa je
B A C spoljasnji ugao kod temena A trougla A B D. Otuda je A B D =
B A C B DA . Medutim, B A C = B, a kod pravouglog trougla ACD
kome je tacka B srediste hipotenuze AC, tj. opisanog kruga, imamo da je
B DA B DC = C. S toga je A B D = B C. Istim postupkom
dokazuje se da je i A C D = B C (sl. 68).

72

Sl. 68
87. Ako su A1 , B1 , C1 tacke u kojima upisani krug trougla ABC dodiruje stranice BC, CA, AB; A2 , B2 , C2 tacke u kojima upisani krug trougla
A1 B1 C1 dodiruje stranice B1 C1 , C1 A1 , A1 B1 ; itd, zatim R prav ugao, doka2
2
1
zati da je Bn An Cn = R +
(A R).
3
(2)n
3
Uputstvo.
Kod trouglova BA1 C1 i CA1 B1 imamo da je BA1 = BC1 i CA1 = CB1 , pa
1
1
je BA1 C1 = R B i CA1 B1 = R C. Stoga je
2
2
B1 A1 C1 = 2B BA1 C1 CA1 B1 =

1
1
(B + C) = R A
2
2

. Ovim smo dokazali slucaj kada je n = 1. Da bismo dokazali opsti stav,


primenimo princip matematicke indukcije. Stoga pretpostavimo da je stav tacan
za n = R - 1, dokazimo da je i on tacan i za n = k. Kao u dokazanom delu ovog
stava, imamo da je (sl. 69)

73

B1

A2

C1

B2

C2

A1

Sl. 69
1
12
1
2
2
1
2
Bk Ak Ck = R Ak Ak1 R R+
(A R) = R+
(A R)
2
23
(2)k1
3
3
(2)n
3
Ovim je teorema dokazana.
88. Ako obelezimo sa E i F tacke u kojima simetrale unutrasnjeg i spoljasnjeg ugla A trougla ABC seku pravu BC, sa D podnozje visine iz temena A, sa G tacku poluprave AC takvu da je AB = AG, sa O srediste kruga
l opisanog oko trougla ABC i sa L tacku u kojoj tangenta kruga l u tacki A
1
sece pravu BC, dokazati da je (a) DAE = AF E = GBC = (B C)
2
(b) poluprava AE bisektrisa ugla OAD; (c) tacka L srediste duzi EF .
Uputstvo.
(a) Ako je AC > AB, bice tacke D i F iza tacke E u odnosu na tacku C,
pa je
1
1
1
DAE = R AED = R C A = R C (2R B C) = (B C)
2
2
2
1
1
1
AF E = R AEF = R C A = R C (2R B C) = 1 (B C)
2
2
2
gde je r prav ugao. Tacka G je izmedu tacaka A i C, pa je
GBC = AGB C = ABG C = B GBC C =
74

1
(B C)
2

1
(C B).
2
(b) S obzirom da je poluprava AE bisektrisa ugla A trougla ABC, ona sadrzi
srediste N luka BC kruga l na kome nije teme A. No ON k AD a tacke O i
D s raznih strana prave AN , pa su uglovi DAE i AN O naizmenicni i jednaki.
Kod trougla OAN imamo da je OA = ON , pa je i ON A = OAN . Otuda je
DAN = OAN , pa je poluprava AN bisektrisa i ugla DAO.
(c) Kako su uglovi OAL i EAF jednaki i istosmerni, i uglovi OAE i LAF
su jednaki. Stoga su uglovi A i F trougla LAF jednaki, pa je AL = LF . Sad su
uglovi A i E trougla LAE jednaki, pa je i AL = LE. Otuda je tacka L srediste
duzi EF .
Ako je AB > AC, bice DAE = AF E = GBC =

G
O

Sl. 70
89. Ako je D proizvoljna tacka prave koja sadrzi stranicu BC trougla
ABC, a O1 i O2 sredista krugova opisanih oko trouglova ABD i ACD.
Dokazati da su konveksni uglovi O1 AO2 i BAC jednaki i istosmerni.
Uputstvo.
Uglovi BAO1 i CAO2 su jednaki i istosmerni s uglom HAD, pa su i medu
sobom jednaki i istosmerni. Stoga su i uglovi O1 AO2 i BAC jednaki I istosmerni
(sl. 71).

75

O2
O1

Sl. 71
90. Ako su S, Sa, Sb, Sc sredista upisanih krugova trougla ABC i R prav
1
1
ugao, dokazati da je (a) BSC = R + A (b) BSa C = R A (c) BSb C =
2
2
1
CSc B = A
2
91. Ako su l i k opisani krug i upisani krug trougla ABC i ka , kb , kc
spolja upisani krugovi koji odgovaraju respektivno stranicama BC, CA, AB,
dokazati da su zajednicka unutrasnja tangenta krugova k i ka koja je razlicita
od prave BC i zajednicka spoljasnja tangenta krugova kb i kc koja je razlicita
od prave BC paralelne s pravom t koja u temenu A dodiruje krug l, zatim da su
odstojanja pomenutih tangenata od prave t jednaka visini iz temena A trougla
ABC.
Uputstvo.
Koristiti prethdne zadatke ili simetriju prave BC u odnosu na simetrle unutrasnjeg i spoljasnjeg ugla A.
92. Dokazati da kod prostog cetvorougla (a) simetrale dvaju uzastopnih
unutrasnjih uglova zahvataju ugao jednak poluzbiru drugih dvaju unutrasnjih
uglova; (b) simetrale dvaju naspramnih uglova zahvataju ugao jednak polurazlici drugih dvaju unutrasnjih uglova; (c) simetrale uglova koji su odredeni
naspramnim stranicama zahvataju ugao jednak poluzbiru dvaju naspramnih
uglova.
93. Ako su P , Q, R, S podnozja upravnih kroz presek O dijagonala, na
stranicama AB, BC, CD, DA cetvorougla ABCD, dokazati da je SOQ =

76

1
(R P ) gde je T prav ugao. Ako je P QRS prost cetvorougao i O
2
tacka u njemu (sl. 72) bice
2T

SOQ = P + OSP + OQP


. Medutim,
OSP +OQP = OAB+OBA = BOC = OCD+ODC = OQR+OSR = 4T RSOQ
, pa je

1
SOQ = 2T (R P )
2
. Ostale slucajeve neka citalac izvede sam.

C
R

S
O

Sl. 72
94. Ako je ABCD pravougaonik kome je AB = 3BC, zatim E, F par
tacaka stranice AB takvih da je AE = EF = F B i R prav ugao, dokazati da je
AED + AF D + ABD = R.
Uputstvo.
Ako obelezimo sa Q tacku simetricnu s tackom F u odnosu na pravu CD,
bice AED = DBG i AF D = GBC. Stoga je (v. sl.73)
AED + AF D + ABD = DBG + GBC + ABD = R
77

Sl. 73
95. Ako su ABCD i A B C D dva konveksna cetvorougla s odgovarajucim
jednakim stranicama i ako je A > A , dokazati da je B < B , C > C ,
D < D .
Uputstvo.
S obzirom da su cetvorouglovi ABCD i A B C D konveksni njihove dijagonale su unutrasnje. Kod trouglova ABD i A B D je AB = A B , AD = A D
i A > A , pa je BD > B D. Zatim, kod trouglova BCD i B C D je
BC = B C , CD = C D i BD > B D , pa je C > C . Dokazati da je
B < B . Ako bi bio B B , kod trouglova ABC i A B C imali bismo da je AB = A B , BC = B C i B B , pa je AC A C , pa je
D D . U tom slucaju bio bi zbir unutrasnjih uglova cetvorougla ABCD
veci od zbira unutrasnjih uglova cetvorougla ABCD veci od zbira unutrasnjih
uglova cetvorougla A B C D , sto je nemoguce. Dakle, bice B < B . Isto
tako je D < D pa je stav dokazan (sl. 74a i 74b).

78

Sl. 74a

79

Sl. 74b
96. Dokazati da su sredista stranica svakog cetvorougla ABCD s neparalelnim dijagonalama temena paralelograma cije su stranice paralelne s dijagonalama AC i BD i jednake polovinama tih dijagonala. Ako su dijagonale AC i BD
cetvorougla ABCD paralelne, dokazati da sredista njegovih stranica pripadaju
jednoj pravoj.
Uputstvo.
Koristiti stav prema kome je srednja linija trougla paralelna sa odgovarajucom stranicom i jednaka njenoj polovini.
97. Ako su dijagonale cetvorougla medu sobom upravne, dokazati da su duzi
koje spajaju sredista naspramnih stranica tog cetvorougla medu sobom jednake.
Uputstvo.
Ako su P , Q, R, S, sredista stranica AB, BC, CD, DA, cetvorougla ABCD,
bie P Q k AC, SR k AC, i GR k BD, P S k BD, pa je P Q k SR i QR k P S.
Stoga je cetvorougao P QRS paralelogram. Kako je po pretpostavci AC BD,
bice i P Q QR, pa je paralelogram P QRS pravougli. Otuda sleduje da je
P R = QS. (sl. 75a)

80

R
C

Sl. 75a
98. Ako su duzi koje spajaju sredista naspramnih stranica cetvorougla medu
sobom jednake, dokazati da su dijagonale tog cetvorougla medu sobom upravne.
Uputstvo.
Ako su P , Q, R, S, sredista stranica AB, BC, CD, DA, cetvorougla ABCD,
bie P Q k AC, SR k AC, i GR k BD, P S k BD, pa je P Q k SR i QR k P S.
Stoga je cetvorougao P QRS paralelogram. S obzirom da su duzi P R i QS po
pretpostavci jednake, paralelogram P QRS je pravougaon, te je P Q QR, i
prema tome AC BD (sl. 75b).

81

C
R
D

A
P
B

Sl. 75b
99. Ako su P , Q, R, S sredista stranica AB, BC, CD, DA i M , N sredista
dijagonala AC i BD cetvorougla ABCD, dokazati da se duzi P R, QS, M N
seku u jednoj tacki koja je srediste svake od njih.
Uputstvo.
Neka su P , Q, R, S i Q, M , S, N temena dvaju paralelograma P QRS i
QM SN , a O i O preseci njihovih dijagonala. Pri tome je O zajednicko srediste
dijagonala P R i QS, a O zajednicko srediste dijagonala QS i M N , pa su tacke
O i O istovetne. Otud sleduje da se duzi P R, QS, M N seku u istoj tacki O koja
je srediste svake od njih. Analogan dokaz izvodi se i u slucaju kada cetvorke
tacaka P , Q, R, S i Q, M , S, N pripadaju pravim linijama (sl. 76).

82

C
R
D

Sl. 76
100. Dokazati da tacke simetricne nekoj tacki O u odnosu na srediste stranica
cetvorougla ABCD i neparalelnim dijagonalama predstavljaju temena paralelograma cije su stranice uporedne i jednake dijagonalama tog cetvorougla.
Uputstvo.
Neka su P , Q, R, S sredista stranica AB, BC, CD, DA cetvorougla ABCD,
a P1 , Q1 , R1 , S1 tacke simetricne tacki O u odnosu na P , Q, R, S. Duzi P Q
i RS uporedne su I jednake polovini duzi AC, kao i duz P1 Q1 , pa je P1 Q1 AC
i R1 S1 AC. Isto tako je Q1 R1 BD i S1 P1 BD. Po pretpostavci, dijagonale AC
i BD nisu uporedne, pa ni njima uporedne duzi P1 Q1 i Q1 R1 sa zajednickim
krajem Q1 nisu na jednoj pravoj. Prema tome, P , Q, R, S, je paralelogramsa
pomenutim o osobinama. (sl. 77).

83

R
D

R1

Q
S

Q1
S1

P1

Sl. 77
101. Dokazati da sredista krakova i sredista dijagonala bilo kojeg trapeze
pripadaju jednoj pravoj.
Uputstvo.
Ako su M i N sredista krakova AD i BC, a P i Q sredista dijagonala AC
i BD, trapeza ABCD, bice M P k CD k AB i M Q k AB, pa su prave M P i
M Q istovetne, prema tome tacke M , P , Q pripadaju jednoj pravoj i tacka N
pripada toj pravoj (sl. 78).

84

Sl. 78
102. Dokazati da je srednja linija konveksnog trapeza jednaka poluzbiru, a
duz odredena sredistima dijagonala jednaka polurazlici uporednih stranica tog
trapeza.
Uputstvo.
Obelezimo sa M i N sredista krakova AD i BC, sa P i Q sredista dijagonala
AC i BD konveksnog trapeza ABCD, a sa E i F tacke simetricne tackama D
i C u odnosu na N i Q. Pri tome je E iza B u odnosu na A, a F na polupravoj
BA, pa su M N i P Q srednje linije trouglova ADE i ACF . Otuda je duz
M N jednaka poluzbiru, a duz P Q polurazlici uporednih stranica AB i CD tog
trapeze (sl. 79).

85

N
P

Sl. 79
103. Ako je duz koja spaja sredista dveju naspramnih stranica AD i BC
cetvorougla ABCD jednaka poluzbiru ili polurazlici drugih dveju stranica, dokazati da su stranice AB i CD tog cetvorougla medu sobom uporedne.
Uputstvo.
Neka su M i N sredista naspramnih stranica AD i BC cetvorougla ABCD,
a P i Q tacke takve da su M ABP i M DOQ paralelogrami. Ako naspamne
stranice AB i CD ne bi bile uporedne, bile bi tacke M , P , Q temena trougla
kome je duz M N tezisna linija, pa bi ta tezisna linija bila jednaka poluzbiru ili
polurazlici stranica M P i M Q, sto nemoguce. Otud sledi da su stranice AB i
CD cetvorougla ABCD medu sobom uporedne (sl. 80).

86

D
Q
N
P

Sl. 80
104. Ako duz odredena sredistima M i N naspramnih stranica AD i BC
bilo kojeg cetvorougla ABCD sece dijagonale AC i BD u tackama P i Q, i ako
je duz M P jednaka duzi N Q, dokazati da su druge dve naspramne stranice AB
i CD medu sobom uporedne.
Uputstvo.
Prema zadatku 28. dijagonale AC i BD cetvorougla ABCD se seku. Ako
nije AB k CD, postojace na pravama AC i BD tacke E i F razlicite od tacaka
C i D takve da je DE k AB i CE k AB. Pri tome su jednake I istosmerne duzi
M P i QN srednje linije trouglova ADE i EF C, pa su i duzi F C i DE jednake i
istosmerne. Dakle bice DECF paralelogram, sto je nemogue, jer se prave koje
sadrze njegove naspramne stranice CE i DF seku. Prema tome, bice AB k CD
(sl. 81).

87

D
E

N
M

Sl. 81
105. Ako su naspramne stranice AB i CD cetvorougla ABCD medu sobom
jednake, dokazati da je prava odredena sredistima drugih dveju stranica upravne
na pravoj koja je odredena sredistima dijagonala tog cetvorougla.
Uputstvo.
Ako su M i N sredista stranica AD i BC, a P i Q sredista dijagonala AC i
BD, bice P , M , Q, N temena romba ili pak tacke jedne prave. U prvom slucaju
je M N P Q , a u drugom P Q (sl. 82).

88

Sl. 82
106. Ako su naspramne stranice AB i CD cetvorougla ABCD medju sobom
jednake, dokazati da prava odredjena sredistima drugih dveju stranica zahvata
sa pravama AB i CD jednake uglove.
uputstvo:
Ako su M i N sredista stranica AD i BC, a E tacka takva da je CDAE
paralelogram, bice ABE jednakokrak trougao, kome je tezisna linija AL iz
vrha A uporedna duzi M N , te su i uglovi koje prava M N zahvata sa pravama
AB i CD medu sobom jednaki,ili su pak A,B,E, tacke jedne prave, pa je M N k
AB k CD (sl.83).
C

D
N
E
M
L
A

Slika 83
107. Ako zbir duzi koje spajaju sredista naspramnih stranica cetvorougla
89

jednak poluobimu tog cetvorougla, dokazati da je taj cetvorougao paralelogram.


uputstvo:
Neka su K,L,M ,N sredista stranica AB,BC,CD,DA cetvorougla ABCD. Ako
sa O obelezimo srediste dijagonale AC, bice
1
AB
2

OL =

1
CD
2
1
OK = BC
2
1
OM = AD
2
ON =

pa je
OL + ON =

1
(AB + CD)
2

OK + OM =

1
(BC + AD)
2

Otuda je
LN

1
(AB + CD)
2

1
(BC + AD)
2
Sabiranjem odgovarajucih strana poslednjih
KM

dveju nejednakosti dobijamo da je


KM + LN

1
(AB + BC + CD + DA)
2

S obirom da je po pretpostavci
KM + KN =

1
(AB + BC + CD + DA)
2

bice
LN =

1
(AB + CD)
2

1
(BC + AD)
2
pa je tacka O na obema duzima LN i KM . U tom slucaju bice
KM =

AB k CD
i
BC k AD
pa je cetvorougao ABCD paralelogram(sl.84).

90

D
O

A
K

Slika 84
108. Neka se kod cetvorougla ABCD prave odredjene naspramnim stranicama AB i CD seku u tacki P , a prave odredjene naspramnim stranicama AD
i BC seku u tacki P . Ako su M ,N ,M ,N tacke polupravih P A,P C,P A,P C
takve da je P M = AB, P N = CD, P N = AD, P N = BC, dokazati da su
duzi M N i M N jednake i istosmerne.
uputstvo:
Ako obelezimo sa E tacku takvu da je cetvorougao BCDE paralelogram, bice
cetvorougli AM N E i AM N E paralelogrami, pa su duzi M N i M N jednake
i istosmerne s duzi AE,dakle i medu sobom(sl.85).
P

N
M
D

C
E

N
B

Slika 85
109. Ako je duz koja spaja sredita osnova AB i CD trapeza ABCD jednaka
polurazlici tih osnova i ako je AB > CD, dokazati da je zbir unutranjih uglova
A i B tog trapeza prav ugao.
uputstvo:
Neka su M i N sredista osnova AB i CD trapeza ABCD. Po pretpostavci
je M N = 21 (AB CD). Ako sa C1 i D1 obelezimo tacke u kojima prave
kroz temena C i D uporedne sa duzi M N seku stranicu AB, bice cetvorougao
D1 C1 CD paralelogram, a trouglovi ADD1 i BCC1 jednakokraki, jer
je AD1 = DD1 i BC1 = CC1 . Stoga je
A =

1
DD1 C1
2

B =

1
CC1 D1
2

91

i prema tome
A + B =

1
(DD1 C1 + CC1 D1 ) = R
2

gde je R prav ugao (sl. 86).

D1

C1

Slika 86
110. Dokazati da upravne projekcije jednog temena trougla na simetralama
unutrasnjih i spoljasnjih uglova kod druga dva temena pripadaju jednoj pravoj.
uputstvo:
Ako su P i Q upravne projekcije temena A na simetralama unutrasnjih uglova
B i C, a R i S upravne projekcije temena A na simetralama spoljasnjih
uglova B i C bice BP AR pravougli paralelogram, pa su dijagonale AB i RP
jednake, one se seku u nekoj tacki M koja ih polovi. U jednakokrakom trouglu
M BP uglovi B i P su jednaki, pa kako je P na raspolovnici ugla ABC,
bice naizmenicni uglovi M P B i P BC jednaki. Otud sleduje da
prava RP sadrzi srediste M stranice AB i uporedna je sa BC. Isto tako, prava
SQ sadrzi srediste N stranice AC i uporedna je sa BC, pa su P , Q, R, S tacke
jedne prave, i to one koja sadrzi sredista M i N stranica AB i AC (sl. 87).

MQ

PN

Slika 87
111. Ako je k krug opisan oko jednakostranicnog trougla ABC i P proizvoljna tacka kruga k, dokazati da je duz AP jednaka zbiru ili razlici duzi BP
i CP , prema tome da li je tacka P na luku BC kruga k na kome nije teme A ili
je pak na kruznom luku BAC.
uputstvo:

92

Analizirajmo najpre slucaj kada je tacka P na luku BC kruga k na kome nije


teme A. Kod trougla ABP je
B > A
pa je AP > BP . Stoga, izmedu tacaka A i P postoji tacka S takva da je
BP = SP . S obzirom da je BP = SP i
BP S =

2R
3

gde je R prav ugao, bice SP B jednakostranican, pa je i BS = BP . Iz


jednakosti AB = CD, BS = BP i
ABS = CBP
sledi da je

ASB
= CP B

, te je AS = CP .
Kako je A S P , bice AP = AS + SP i prema tome AP = BP + CP (sl.
88). Analizirajmo sad slucaj kada je tacka P na kruznom luku BAC. Pri tome
je tacka P na luku AB koji ne sadrzi C ili na luku AC koji ne sadrzi B.
Saglasno prethodnom slucaju, ako je tacka P na luku AB koji ne sadrzi C,
bice CP = AP + BP , pa je AP = CP BP ; ako je tacka P na luku AC koji
ne sadrzi B, bice BP = AP + CP , pa je AP = BP CP .
A

Slika 88
112. Ako su a, b, c tri prave jedne ravni koje se seku u istoj tacki O i koje
razlazu ravan u kojoj se nalaze na sest jednakih uglova, dokazati da su podnozja
A, B, C upravnih iz proizvoljne tacke P razlicite od O na pravama a, b, c temena
jednakostranicnog trougla.
uputstvo:
S obzirom da su uglovi CAP , OBP , OCP pravi, tacke A, B, C pripadaju
krugu l kome je duz OP precnik. Ako obelezimo sa R prav ugao, bice
ACB = AOB =
pa je trougao ABC jednakostranican (sl. 89).
93

2
R
3

B
C

Slika 89
113. Ako su a, b, c tri prave koje se seku u istoj tacki O i koje razlazu ravan
u kojoj se nalaze na sest jednakih uglova, dokazati da je odstojanje proizvoljne
tacke P te ravni od prave a jednako zbiru ili razlici odstojanja te tacke od drugih dveju pravih b i c.
uputstvo:
Koristiti prethodna dva zadatka.
114. Ako je O srediste, AB precnik i P proizvoljna tacka nekog kruga k,
zatim M podnozje upravne iz tacke P na pravoj AB, O tacka poluprave P O
takva da je P O = 2AM i R druga presecna tacka prave AO s krugom k, dokazati
da je AOR = 3AOP .
uputstvo:
Obelezimo sa N srediste duzi P Q i sa K drugu presecnu tacku prave AN s
krugom k. Iz podudarnosti trouglova AM P i P AN sledi da je
P M A = AN P
No P M A je prav, te je i njemu jednak ugao AN P prav. Stoga je tacka K
simetricna s tackom A u odnosu na pravu OP , i prema tome
AOP = P OK
Iz podudarnosti trouglova AP N i AQN sledi da je
P AN = QAN
tj.
P AK = RAK
Otuda je i
P OK = ROK
i prema tome
AOR = 3AOP
(sl. 90).

94

K
P

R
N
Q

Slika 90
krugova k1 i k2 , P1 i P2 tacke u kojima
115. Ako su O1 i O2 srediSta
proizvoljna prava koja sadrzi zajednicku tacku M tih krugova sece k1 i k2 , a N
tacka simetricna s tackom M u odnosu na srediste O duzi O1 O2 , dokazati da je
tacka N na simetrali s duzi P1 P2 .
uputstvo:
Ako su K, S, S1 , S2 podnozja upravnih kroz N , O, O1 , O2 na pravoj P1 P2 , bice
S zajednicko srediste duzi M K i S1 S2 , pa je K srediste duzi P1 P2 (sl. 91).
s
P1

S1

O1

KS M

S2

P2

O2

N
k2
k1

Slika 91
116. Ako su B i C tacke u kojima proizvoljan krug kroz temena B i C
sece stranice AB i AC trougla ABC, dokazati da je prava B C uporedna s
pravom koja u temenu A dodiruje krug opisan oko trougla ABC.
uputstvo:
Ako je D proizvoljna tacka prave koja u temenu A dodiruje opisani krug trougla
ABC, a nalazi se s one strane prave AB s koje nije tacka C, odnosno tacka
C , bice
DAB = DAB = BCA = BCC = AB C
S obzirom da su uglovi DAB i AB C jednaki i naizmenicni, prave AD i
B C su medu sobom uporedne (sl. 92).

95

A
D

C
B

Slika 92
117. Ako su B i C tacke u kojima neka prava uporedna s pravom koja u
temenu A dodiruje opisani krug trougla ABC sece stranice AB i AC, dokazati
da tacke B, C, B , C pripadaju jednom krugu.
uputstvo:
Ako obelezimo sa D proizvoljnu tacku prave koja u temenu A dodiruje opisani
krug trougla ABC, a nalazi se s one strane prave AB s koje nije tacka C,
odnosno tacka C , bice
AB C = DAB = DAB = BCA = BCC
Dakle kod konveksnog cetvorougla BCC B spoljasnji ugao kod temena B jednak je s unutrasnjim uglom kod temena C, pa je pomenuti cetvorougao tetivan,
i prema tome tacke B, C, B , C na jednom krugu (sl. 92).
118. Ako je dijagonala AC precnik kruga opisanog oko tetivnog cetvorougla
ABCD, dokazati da su upravne projekcije bilo kojih dveju naspramnih stranica
tog cetvorougla na drugoj dijagonali medju sobom jednake.
uputstvo:
Ako je O srediste dijagonale AC, a O , A , C upravne projekcije tacaka O, A, C
na dijagonali BD bice O zajednicko srediste duzi BD i A C , pa je A B = C D
i A D = C B (sl. 93).
D
C
A
O
O

Slika 93
119. Ako je P proizvoljna tacka kruga l opisanog oko trougla ABC, a
P1 , P2 , P3 tacke simetricne s tackom P u odnosu na simetrale uglova A, B, C,
96

dokazati da su prave AP1 , BP2 , CP3 medju sobom uporedne.


uputstvo:
Tacka P je na jednom od lukova BC, CA, AB kruga l na kome nije trece teme
trougla, npr. na luku BC na kome nije teme A. U tom slucaju tacke P1 i P2
su s raznih strana prave AB, pa su uglovi BAP1 i ABP2 naizmenicni. Iz
simetrije u odnosu na simetralu ugla B sleduje da su uglovi ABP2 i CBP
medu sobom jednaki. No CBP = CAP = BAP1 , pa su naizmenicni uglovi
ABP2 i BAP1 i jednaki. Stoga su prave AP1 i BP2 medu sobom upravne.
Isto
tako, uporedne su i prave AP1 i CP3 , pa su sve tri prave AP1 , BP2 , CP3 medu
sobom uporedne (sl. 94).

A
P2
P3

C
P

P1

Slika 94
120. Ako su AB i CD paralelne tetive dvaju krugova koji se seku u tackama
M i N , dokazati da su uglovi AM C i BN D jednaki ili suplementni.
uputstvo:
Prava CD sece pravu AM u nekoj tacki E. Ako je tacka E npr. na polupravoj
M A i iza C u odnosu na tacku D, bice (sl. 95)
AM C = M V DM EC = M V DM AB = 2RM N D = 2R+M N B = M N BM N D = BN D
Ostale slucajeve neka citalac analizira sam.

A
E
M

D
B

Slika 95

97

121. Ako su A, B i C, D tacke u kojima neka prava p sece dva kruga k i


l koji se seku u tackama M i N , dokazati da su uglovi AM C i BN D jednaki
ili suplementni zavisno od toga imaju li tetive AB i CD zajednickih tacaka ili ne.
Uputstvo. Postupiti kao u prethodnom zadatku.
122. Ako su M i N tacke u kojima se seku dva kruga k i l, A i B tacke
u kojima proizvoljna tacka kruga k sece krug l, a C dodirna tacka, dokazati
da su uglovi AM C i BN C jednaki ili suplementni, zavisno od toga da li je C
unutrasnja tacka tetive AB ili je na njenom produzenju.
Uputstvo. Postupiti kao u zadatku 120.
123. Ako su A i B dodirne tacke bilo koje zajednicke tangente dvaju krugova k i l koji se seku u tackama M i N , dokazati da su uglovi AM B i AN B
suplementni.
Uputstvo. Ako prava MN sece pravu AB u tacki koja se nalazi iza N u
odnosu na M, bice uglovi A i B trougla NAB jednaki sa uglovima AMN i BMN.
124. Ako su AB i CD dve paralelne tetive dvaju krugova k i l koji se dodiruju
u nekoj tacki M , dokazati da su uglovi AM C i BN D jednaki ili suplementni.
Uputstvo. Postupiti kao u zadatku 120.
125. Ako je M dodirna tacka dvaju krugova k i l, a A, B i C, D tacke u kojima neka prava sece krugove k i l, dokazati da su uglovi AM C i BM D jednaki
ili suplementni, zavisno od toga da li se krugovi k i l dodiruju iznutra ili spolja.
Uputstvo. Postupiti kao u zadatku 121.
126. Ako je M dodirna tacka dvaju krugova k i l, p proizvoljna prava koja
sece krug k u tackama A, B i dodiruje krug l u tacki C, dokazati da su uglovi
AM C i BN D jednaki ili suplementni, zavisno od toga da li se krugovi k i l
dodiruju iznutra ili spolja.
Uputstvo. Postupiti kao u zadatku 122.

98

127. Ako su M i N presecne tacke dvaju krugova k i l, p i q dve prave kroz


tacku N od kojih prva sece krugove k i l u tackama A i B, a druga sece krugove
k i l u tackama C i D, dokazati da su uglovi AM B i CM D jednaki.

A
B

C
D
N

Slika 96
Tacka N ne pripada duzima AB i CD, pripada obema tim duzima ili samo
jednoj od njih. Analizirajmo prvi slucaj pretpostavljajuci da je tacka N iza B
u odnosu na A i iza D u odnosu na C. Pri tome je (sl. 96)
AM B = M BN M AN = M DN M CN = CM D
Ostale slucajeve neka citalac dokaze sam.
128. Ako su O1 i O2 sredista dvaju krugova k1 i k2 koji se seku u tackama
M i N , a P1 i P2 tacke u kojima proizvoljna prava kroz N sece k1 i k2 , dokazati
da je P1 M P2 = O1 M O2 .

O1

O2
P2
N

P1

Slika 97
Uputstvo. Uglovi P1 i P2 trougla MP1 P2 jednaki su uglovima O1 i O2 trougla
MO1 O2 (sl. 97).

99

129. Ako su O1 i O2 sredista dvaju krugova k1 i k2 koji se seku, P jedna


njihova presecna tacka, a M1 , M2 i N1 , N2 dodirne tacke zajednickih dirki tih
krugova pri cemu su M1 i M2 s one strane prave O1 O2 s koje nije P , dokazati
da je M1 P M2 = 12 O1 P O2 i N1 ON2 = 21 2R 21 O1 P O2 .

N1
N2
P

O1

O2

M2

M1

Slika 98
Uputstvo. Ako je Q podnozje upravne iz P na pravoj M1 M2 bice poluprave
PM1 i PM2 raspolovnice uglova O1 PQ i O2 PQ. Analizirati slucaj kada je Q
tacka duzi M1 M2 i slucaj kada je na njenom produzenju. Iste osobine koristiti
i pri resavanju drugog dela zadatka (sl. 98).
130. Ako su M i N presecne tacke dvaju krugova k1 i k2 , p i q poluprave
kojima je zajednicki kraj M i koje s polupravom M N zahvataju jednake uglove.
Ako su P1 i P2 preseci poluprave p s krugovima k1 i k2 , a O1 i O2 preseci poluprave q s krugovima k1 i k2 , dokazati da su duzi P1 P2 i Q1 Q2 medu sobom
jednake.

k1
M
Q1
k2

P1

P2

Q2

Slika 99
Trouglovi NP1 P2 i NQ1 Q2 su podudarni, jer su stranica NP1 i uglovi P1 i
P2 trougla NP1 P2 jednaki stranici NQ1 i uglovima Q1 i Q2 trougla NQ1 Q2 (sl.
99).

100

131. Ako su P i Q sredista lukova AB i AC kruga opisanog oko trougla


ABC, dokazati da je tetiva P Q upravna na simetrali ugla A tog trougla.

A
Q
P
S

Slika 100
Ako je S presek simetrala unutrasnjih uglova trougla ABC, bice AP Q
=
SP Q, pa je tacka S simetricna sa tackom A u odnosu na pravu PQ. Otuda je
ASP Q (sl. 100).
132. Ako su P , Q, R, S sredista lukova AB, BC, CD, DA kruga opisanog
oko konveksnog tetivnog cetvorougla ABCD, pri cemu navedeni luci ne sadrze
ostala temena tog cetvorougla, dokazati da se tetive P R i OS seku pod pravim
uglom.

R
D
O

B
P

Slika 101

Cetvorougao
PQRS je takode konveksan, pa se njegove dijagonale PR i QS
seku u istoj tacki O. Stoga je
P OQ = P SQ + RP S
i
P OS = P QS + RP Q
Kako je
P SQ + RP S = P QS + RP Q

101

bice i naporedni uglovi POQ i POS jednaki, pa je P RQS (sl. 101).


133. Ako su S i T tacke u kojima se seku dva kruga k1 i k2 , P i Q tacke u
kojima proizvoljna prava kroz tacku T sece krugove k1 i k2 , a R tacka u kojoj
se seku dirke krugova k1 i k2 konstruisane u tackama P i Q, dokazati da tacke
P , Q, R, S pripadaju jednom krugu.

k1
S

k2
Q
T
Q

P
P
R

Slika 102
Analizirajmo slucaj kada se tacka T nalazi izmedu tacaka P i Q. Ako su PP
i QQ prave koje u tackama P i Q dodiruju krugove k1 i k2 i pri tome tacke P
i Q s one strane od prave PQ s koje nije tacka S, bice
T P P = P ST
i
T QQ = T SQ
pa je
T P P + T QQ = P ST + T SQ = P SQ
No ugao PSQ, dakle i zbir uglova TPP i TQQ je manji od zbira dvaju pravih
uglova, pa se prave PP i QQ seku u nekoj tacki
R koja se nalazi s one strane prave PQ s koje su tacke P i Q. Stoga su tacke
R i S s raznih strana prave PQ. Sem toga ugao PRQ je suplementan sa zbirom
uglova TPP i TQQ, tj. s uglom PSQ, pa su tacke P, Q, R, S na jednom krugu
(sl. 102). Ostale slucajeve neka citalac analizira sam.

102

134. Ako su P i Q tacke u kojima proizvoljan krug kroz temena B i C


trougla ABC sece stranice AB i AC, a P i Q tacke u kojima prave kroz P i Q
uporedne sa stranicama AC i AB seku stranicu BC, dokazati da tacke P , P ,
Q, Q pripadaju jednom krugu.

Slika 103
S obzirom da su tacke P i Q izmedu tacaka B i C, bice [BPQC] ili [BQPC]
ili su pak tacke P i Q istovetne.Ako je [BPQC], bice cetvorougao PPQQ
konveksan, a
P P Q = CQR = P BC = QQ C
pa su tacke P,P,Q,Q na jednom krugu (v. sl. 103).
Ako je [BQPC] , bice tacke P i Q s iste strane od prave QP , a
P P Q = CQR = CBP = P Q Q
pa su i u tom slucaju tacke P,P,Q,Q na jednom krugu. Ako su tacke P i Q
istovetne dokaz je neposredan.

103

135. Ako su O1 i O2 sredista dvaju krugova k1 i k2 koji se seku u tackama


A i B, a C i D tacke u kojima prave AO1 i AO2 seku krugove k2 i k1 , dokazati
da tacke B, C, D, O1 , O2 pripadaju jednom krugu.
Tacke O1 , O2 , B ne pripadaju jednoj pravoj , te odreduju neki krug l.
Dokazimo da se tacke C i D nalaze na tom krugu; najpre to ucinimo za tacku
C. Pri izvodenju dokaza treba posebno analizirati sve slucajeve koje moze da
ima tacke C u odnosu na tacke A i O1 . Ako je tacka C iza tacke A u odnosu
na tacku O1 (sl. 104), bice tacke B i C s raznih strana prave O1 O2 . Sem toga,
ako obelezimo sa R prav ugao, imamo da je
O1 BO2 = O1 AO2 = 2R O2 AC = 2R ACO2 = 2R O1 CO2
S toga je C tacka kruga l.

O1
O2
B

Slika 104
Ako je tacka C izmedu tacaka A i O1 , bice tacke B i C takode s raznih
strana od prve O1 O2 i pri tome
O1 BO2 = O1 AO2 = ACO2 = 2R O1 CO2
pa je tacka C takode na krugu l.
Ako je tacka C iza O1 u odnosu na tacku A ,bice tacke B i C s iste strane
od prave O1 O2 i pri tome
O1 BC = O1 AO2 = O1 CO2
pa je tacka C i u tom slucaju na krugu l.
Istim postupkom dokazuje se da i tacka D pripada krugu l.
136. Ako su P i Q tacke u kojima simetrale stranica AC i AB seku prave
odredene stranicama AB i AC trougla ABC, a O srediste kruga opisanog oko
trougla ABC, dokazati da tacke B, C, P , Q, O pripadaju jednom krugu, ili pak
jednoj pravoj.
104

Slika 105
Razlikovacemo slucajeve kada je kod trougla ABC ugao A ostar, prav i tup.
Ako je ugao A ostar (sl. 105), srediste O kruga opisanog oko trougla ABC
je s one strane od prave BC s koje je teme A, pri cemu je
BOC = 2A.
Sem toga, tacka P je na polupravoj AB, dakle izmedu tacaka A i B, iza tacke
B u odnosu na tacku A, ili je istovetna sa tackom B.
Ako je tacka P izmedu tacaka A i B, tacke O i P su s iste strane od prave
BC, a uglovi BOC i BP C jednaki; ako je tacka P iza B u odnosu na A, tacke
O i P su s raznih strana od prave BC, a uglovi BOC i BP C suplementni.
Stoga je tacka P na krugu koji je odreden tackama O, B i C. Istim postupkom
dokazuje se da i tacka Q pripada tom krugu. Ako je ugao A prav, tacke P i Q
su istovetna s tackama B i C, a tacka O je istovetna sa sredistem duzi BC, te
su tacke B, C, P i Q, na jednoj pravoj.
Ako je ugao A tup, tacka O je s one strane od prave BC s koje nije A, a
tacka P iza A u odnosu na B, pa su tacke O i P s raznih strana od prave BC.
Pri tome su uglovi BOC i BP C suplementni, pa je tacka P na krugu koji je
odreden s tackama O, B i C. Istim postupkom dokazuje se da i tacka Q pripada
tome krugu.

105

137. Ako su AD, BE, CF visine trougla ABC, a M i N tacke simetricne s


tackom D u odnosu na prave AB i AC, dokazati da tacke E, F , M , N pripadaju
jednoj pravoj.

N
A

Slika 106
Prema zadatku ..., prava BE je simetrala ugla DBF , a po pretpostavci prava
CE simetrala njemu susednog ugla BET . S obzirom da su simetrale BE i CE
dvaju susednih uglova DEF i DEN upravne medu sobom, pomenuti uglovi
DEF i DEN su naporedni, pa su tacke E, F i N kolinearne. Istim postupkom
dokazuje se da su i tacke E, F i M kolinearne. Otuda sleduje da tacke E, F ,
M i N pripadaju jednoj pravoj (sl. 106).

106

138. Dokazati da podnozja upravnih kroz bilo koju tacku kruga opisanog oko
nekog trougla na pravama koje su odredene stranicama tog trougla, pripadaju
jednoj pravoj (Simsonova teorema).

C
P
A

Slika 107
Obelezimo sa l opisani krug trougla ABC, a P bilo koju tacku toga kruga,
a sa A , B i C podnozja upravnih iz P na pravama BC, CA i AB. Ako se
tacka P poklapa s nekim temenom trougla ABC, stav je jednostavan. Ako je
tacka P razlicita od temena A, B i C, ona je unutrasnja tacka jednog od lukova
AB, BC i CA kruga l, npr. luka AC koji ne sadrzi teme B. Sem toga, tacke
A i C su na kracima ugla B ili je jedna od njih na produzenju odgovarajueceg
kraka. Pretpostavimo da su tacke A i C na kracima ugla B. Iz uvedenih
pretpostavki sleduje da su kod tetivnih cetvorouglova P ABC i P C BA uglovi
AP C i C P A jednaki i istosmerni. Stoga su i uglovi C P A i A P C jednaki i
istosmerni. Iz tetivnih cetvorouglova P C AB i P B A C sleduje da su uglovi
C P A i A P C jednaki i istosmerni s uglovima C B A i A B C, pa su i uglovi
C B A i A B C jednaki i istosmerni, i prema tome tacke A , B i C na jednoj
pravoj. Analogan dokaz izvodi se i u slucaju kada je jedna od tacaka A i C na
produzenju odgovarajuceg kraka ugla B.
Napomena. Prava kojoj pripadaju tacke A , B i C naziva se Simsonova prava
tacke P u odnosu na trougao ABC (sl. 107).

107

139. Ako je ABC jednakostranican trougao i P proizvoljna tacka njegove


ravni koja nije na opisanom krugu oko tog trougla, dokazati da postoji trougao
cije su stranice jednake duzima P A, P B, P C (Teorema Pompejca).

Slika 108
Obelezimo sa A , B i C tacke u kojima prave kroz tacku P uporedne sa

stranicama AB, BC i CA seku prave BC, CA i AB. Cetvorouglovi


AB P C ,


BA P C i CA P B su jednokraki trapezi, pa je AP = B C , BP = C A i
CP = A B . S obzirom da tacka P nije na krugu opisanom oko trougla ABC,
tacke A , B i C nisu na jednoj pravoj, dakle postoji trougao A B C kome su
stranice jednake duzima AP , BP i CP (sl. 108).

108

140. Krug k sece stranice BC, CA, AB trougla ABC u tackama P i P , Q i


Q , R i R . Ako se normale u tackama P , Q, R na pravama BC, CA, AB seku
u jednoj tacki dokazati da se normale u tackama P , Q , R na pravama BC,
CA, AB takode seku u jednoj tacki.

A
Q
R
B

R C

Slika 109
Neka se upravne u tackama P , Q i R na stranicama BC, CA i AB seku
u tacki O. Ako su A , B i C sredista tetiva P P , QQ i RR upravne u tim
tackama na BC, CA i AB tj. P P , QQ i RR se seku u jednoj tacki, sredistu
S kruga l. Upravna u tetivi P na pravoj BC sece pravu OS u tacki O koja je
simetricna s tackom O u odnosu na tacku S. Isto tako upravne u tackama Q i
R na stranicama CA i AB seku pravu OS u tacki O , te se upravne u tackama
P , Q i R na stranicama BC, CA i AB seku u jednoj tacki (sl. 109).

109

141. Ako su AA , BB , CC visine ostrouglog trougla ABC, dokazati da je


ortocentar H tog trougla srediste upisanog kruga trougla A B C .

C
H

Slika 110

Cetvorouglovi
BA HC i CB HA su konveksni i tetivni, pa je
C BH = CA H

HCB = HA B .

C BH = ABB

HCB = C CA,

No
zatim
ABB = C CA,
pa je
C AH = HA B .
Sem toga, tacka B je u uglu C A B , pa je prava A H simetrala ugla C A B .
Istim postupkom dokazuje se da je i prava B H simetrala ugla A B C pa je H
srediste kruga upisanog u trougao A B C (sl. 110).
142. Ako su AA , BB , CC visine trougla ABC kome je ugao A tup, dokazati da je ortocentar H tog trougla srediste spolja upisanog kruga trougla
A B C koji dodiruje stranicu B C .
Uputstvo. V. z. 141.

110

143. Dokazati da tacke simetricne s ortocentrom u odnosu na prave odredene


stranicama trougla pripadaju krugu koji je opisan oko tog trougla.

B
A

C
C

Slika 111
Obelezimo sa H tacku u kojoj se seku prave odredene visinama AA , BB i
CC trougla ABC, a sa A , B i C tacke simetricne s ortocentrom H u odnosu
na prave BC, CA i AB. Jedan od uglova B i C, npr. ugao B, je ostar, pa su
tacke B i A s iste strane prave AC. Sem toga je
AA C = HA C = A HC = A HC = C B A = ABC,
pa je tacka A na krugu koji je opisan oko trougla ABC. Analogno se dokazuje
da i tacke B i C pripadaju tome krugu (sl. 111).

111

144. Ako je H ortocentar trougla ABC, dokazati da su poluprecnici krugova


opisanih oko trouglova ABC, HBC, HCA, HAB medu sobom jednaki.

Slika 112
Prema zadatku 143, tacka A simetricna s tackom H u odnosu na pravu BC
je na opisanom krugu trougla ABC. Iz podudarnosti trouglova A BC i HBC
sleduje da su i poluprecnici krugova opisanih oko tih trouglova medu sobom
jednaki. Istim postupkom dokazuje se jednakost poluprecnika ostalih krugova
(sl. 112).

112

145. Ako je H ortocentar, T srediste, O srediste kruga opisanog oko trougla


ABC i A1 srediste stranice BC, dokazati
(a) da je duz OA1 istosmerna s duzi AH i jednaka njenoj polovini
(b) da tacke O, T , H pripadaju jednoj pravoj, pri cemu je HT = 2T O.

A1

Slika 113
(a) Heka je D tacka u kojoj prava OC sece krug l. Kako su tacke O i
A1 sredista duzi BC i CD, duz OA1 je srednja linija trougla BCD, prema
tome, ona je istosmerna s duzi BD i jednaka njenoj polovini. Pored toga, prave
BD i AH upravne su na pravoj BC, dakle, uporedne medu sobom. Isto tako,
prave AD i BH upravne su na pravoj AC, te su i one medu sobom uporedne.
Otuda sleduje da je cetvorougao AHBD paralelogram, pa je duz BD jednaka i
istosmerna s duzi AH, prema tome, duz OA1 je istosmerna s duzi AH i jednaka
njenoj polovini.
(b) S obzirom da je tacka T izmedu tacaka A i A1 , takva da je AT = 2T A1 ,
a duzi AH i OA1 istosmerne pri cemu je AH = 2OA1 , bice tacka T izmedu
tacaka O i H takva da je HT = 2T O (sl. 113).
Napomena. Pravu koja sadrzi srediste opisanog kruga, ortocentar i teziste
trougla nazivamo Ojlerovom pravom tog trougla.

113

146. Ako je ABCD tetivan cetvorougao, E ortocentar trougla ABD i F


ortocentar trougla ABC, dokazati da je cetvorougao CDEF paralelogram.

O
E

Slika 114
Ako obelezimo sa O srediste kruga opisanog oko tetivnog cetvorougla ABCD
i sa P podnozje upravne iz tacke O na stranici AB, prema zadatku ... bice duz
OP istosmerna s duzima CF i DE, i jednaka polovini svake od tih duzi. Stoga
su duzi CF i DE jednake i istosmerne, pa je cetvorougao CDEF paralelogram
(sl. 114).

114

147. Dokazati da tacke simetricne s ortocentrom u odnosu na sredista stranica trougla pripadaju krugu koji je opisan oko tog trougla.

H
O

Slika 115
Obelezimo sa O srediste kruga oplsanog oko trougla ABC, sa H ortocentar
tog trougla, sa A srediste stranlce BC i sa A tacku koja je simetricna s tackom
H u odnosu na A . S obzirom da je tacka A srediste duzi HA a duz A O
istosmerna s duzi HA i jednaka njenoj polovini, bice tacka O srediste duzi
AA , pa je tacka A na krugu opisanom oko trougla ABC (sl. 115).

115

148. Ako su H i O ortocentar i srediste opisanog kruga trougla ABC, a M


i N srediste duzi AH i tezisne linije AD iz temena A, dokazati da tacke O, M ,
N pripadaju jednoj pravoj,stavise da je tacka N srediste duzi OM .

M
N
H
O

Slika 116
S obzirom da je tacka M sredlste duzi AH, a prema zadatku 145 duz OD
istosmerna s duzi AH i jednaka njenoj polovini, bice duz OD istosmerna i
jednaka s duzi M N , pa je cetvorougao HDOM paralelogram. Stoga je duz
M O istosmerna i jednaka sa duzi HD. No tacke M i N su sredista stranica
AH i AD trougla AHD, pa je duz M N istosmerna s duzi HD i jednaka njenoj
polovini. Otuda sleduje da su tacke O, M i N na jednoj pravoj, stavise da je
tacka N sredista duzi OM (sl. 116).

116

149. Ako je H ortocentar, O srediste opisanog kruga, i D podnozje visine


iz temena A trougla ABC, zatim M tacka u kojoj se seku prave AO i BC, E
srediste duzi OH i F srediste duzi AM , dokazati da tacke D, E, F pripadaju
jednoj pravoj.

A
F
H
E

Slika 117
Ako obelezimo sa A srediste stranice BC i sa A srediste duzi AH, prema
zadatku 145, bice duzi OA i AA jednake i istosmerna, pa je cetvorougao
OAA A paralelogram, i prema tome OA k A A , tj. M A k A A . No duzi
OA i HA su jednake i suprotno usmerene, te se srediste E duzi OH poklapa
sa sredistem duzi A A . S obzirom da su tacke E i F sredista dveju uporednih
duzi AM i A A , prava EF sadrzi presek D pravih M A i AA (sl. 117).

117

150. Ako su H i D ortocentar i srediste stranice BC trougla ABC, a E i F


podnozja upravnih iz tacke H na simetrali unutrasnjeg i simetrali spoljasnjeg
ugla A, dokazati da tacke D, E, F pripadaju jednoj pravoj.

K
E
H

Slika 118
S obzirom da su kod cetvorougla HEAF uglovi E, A i F pravi, on je pravougli paralelogram, pa je presek K njegovih jednakih dijagonala AH i EF
srediste svake od tih duzi. Stoga je AK = EK, pa je kod trougla AKE
AEK = EAK. Ako je O srediste kruga opisanog oko trougla ABC, prema
zadatku ... poluprava AS je raspolovnica ugla KAO te su uglovi AEK i EAO
naizmenicni i jednaki. Stoga je EK k AO. Prema zadatku 145 duzi AK i OD
su jednake i istosmerne, pa je cetvorougao KDOA paralelogram, i prema tome
KD k AO. S obzirom da su obe prave KD i KE uporedne s pravom AO, one
su istovetne. Otud sleduje da tacke D, E i F pripadaju jednoj pravoj. (sl. 118)

118

151. Ako je O srediste opisanog kruga trougla ABC, M tacka simetricna s


ortocentrom H tog trougla u odnosu na teme A i N tacka simetricna s temenom
A u odnosu na srediste D stranice BC, dokazati da tacke O,M ,N pripadaju jednoj pravoj.
Resenje:

C
D

Slika 119
S obzirom da je tacka D srediste duzi AN, a prema zadatku 145 duz DO
istosmerna s duzi HA, odnosno duzi AM, i jednaka njenoj polovini, tacke O, M,
N pripadaju jednoj pravoj (sl. 119).
152. Dokazati da sredista stranica, podnozja visina i sredista duzi koje
spajaju ortocentar s temenima trougla pripadaju jednom krugu.
Resenje:

119

A
B2

A3
C2
C1

B1
H
E T
O

B3
B

C3
C

A2

A1

Slika 120
Obelezimo sa H ortocentar trougla ABC, sa A1 , B1 , C1 sredista stranica BC,
CA, AB, sa A2 , B2 , C2 podnozja visina iz temena A, B, C i sa A3 , B3 , C3 sredista
duzi AH, BH, CH. Kako su duzi B1 C1 i B3 C3 srednje linije trouglova ABC i
HBC koje odgovaraju zajednickoj stranici BC, one su uporedne sa BC, dakle i
medu sobom.
Isto tako, duzi C1 , B3 i B1 , C3 su srednje linije trouglova BAH i CAH koje
odgovaraju zajednickoj stranici AH, pa su iste uporedne sa AH, dakle i medu
sobom. Medutim duzi AH i BC upravne su jedna na drugoj, pa je cetvorougao
B1 C1 B3 C3 pravougli paralelogram. Stoga su njegove dijagonale B1 B3 i C1 C3
medu sobom jednake, a presek E tih dijagonala srediste svake od tih duzi. Na
isti nacin dokazuje se da je cetvorougao A1 B1 A3 B3 pravougli paralelogram, pa
je duz A1 A3 jednaka s duzi B1 B3 , a srediste duzi A1 A3 istovetno sa sredistem E
duzi B1 B3 . Otuda sleduje da su duzi EA1 , EB1 , EC1 , EA3 , EB3 , EC3 medu
sobom jednake, i prema tome tacke A1 , B1 , C1 , A3 , B3 , C3 na izvesnom krugu
l kome je srediste E. Kako su duzi A1 A3 , B1 B3 , C1 C3 precnici toga kruga, a
uglovi A1 A2 A3 , B1 B2 B3 , C1 C2 C3 pravi i tacke A2 , B2 , C2 pripadaju krugu l (sl.
120).
Napomena: Krug koji sadrzi sredista stranica, podnozja visina i sredista duzi
koja spajaju ortocentar s temenima nekog trougla naziva se Ojlerovim krugom
tog trougla.
153. Dokazati da se srediste Ojlerovog kruga bilo kojeg trougla poklapa sa
sredistem duzi koja spaja ortocentar sa sredistem opisanog kruga tog trougla,
120

zatim da je poluprecnik toga kruga jednak polovini poluprecnika opisanog kruga.


Resenje:
Pored oznaka uvedenih u prethodnoj teoremi, obelezimo sa O srediste kruga l
opisanog oko trougla ABC. Prema teoremi 145, duz OA1 jednaka je i istosmerna
s duzi A3 H, pa je cetvorougao A1 HA3 O paralelogram. Stoga se srediste E duzi
A1 A3 , tj. srediste Ojlerovog kruga l trougla ABC, poklapa sa sredistem duzi
OH (sl. 120). S obzirom da su tacke E i A3 sredista starnica OH i AH trougla
OAH, duz EA3 jednaka je polovini duzi OA, pa je poluprecnik Ojlerovog kruga
l jednak polovini poluprecnika opisanog kruga trougla ABC.
154. Ako obelezimo sa A1 ,B1 ,C1 sredista stranica BC = a, CA = b, AB = c
trougla ABC, sa p poluobim tog trougla, sa l(o, r) opisani krug tog trougla, sa
P ,Q,R tacke u kojima upisani krug k(S1 , ) dodiruje stranice BC,CA,AB, sa
Pi ,Qi , Ri za i = a, b, c tacke u kojima spolja upisani krug ki (Si , i ) dodiruje
prave BC,CA,AB, sa M i N tacke u kojima simetrala stranice BC sece krug
pri cemu je tacka M na luku BAC, a sa M i N podnozja upravnih iz tacaka
M i N na pravoj AB, dokazati da je:
(a) AQa = ARa = p;
(dj) P A1 = A1 Pa , Pc A1 = A1 Pb ;
(b) QQa = RRa = a;
(e) A1 M = 12 (b + c ), A1 N = 21 (a );
(v) Qb Qc = Rb Rc = a;
(z) M M = 21 (b c ), N N = 21 (a + );
(g) AQ = AR = BRc = P Pc = CPb = CQb = p a; (z) AM = 21 (b
c), AN = 12 (b + c);
(d) P Pa = bc, Pb Pc = b+c;
(i) a + b + c = 4r + .
Resenje:
(sl.121)
(a) Kako su B i C unutrasnje tacke duzi ARa i AQa , bice ARa + AQa =
AB + BRa + AC + CQa = AB + AC + (BPa + Pa C) = AB + AC + BC = 2p. No
ARa = AQa , pa je 2Ra = 2p, i prema tome ARa = AQa = p. Istim postupkom
dokazuje se da je BPb = BRb = p i CPc = CQc = p.
(b) Kako su B i C unutrasnje tacke duzi RRa i QQa , bice RRa + QQa =
RB + BRa + QC + CQa = BP + BPa + P C + Pa C = 2BC. No RRa = QQa ,
pa je 2RRa = 2BC i prema tome RRa = QQa =BC. Istim postupkom dokazuje
se da je P Pb = RRb = AC i P Pc = QQc = AB.
(v) Kako su C i Qb unutrasnje tacke duzi BPb i QQc , bice Qb Qc = QQc
CQb = BP b BC = a.
(g) Kako su P, Q, R unutrasnje tacke stranica BC, CA, AB, bice AQ = AR
1
1
= (AB + AC QB QC) = (AB + AC BC) = p a. Istim postupkom
2
2
dokazuju se i ostale jednakosti.
(d) Tacke P i Pa su istovetne ili razlicite. Ako su tacke P i Pa istovetne,
bice SSa BC, tj. ASBC, pa je AB = BC. Ako su tacke P i Pa razlicite, bice
tacka Pa izmedu tacaka P i C ili tacka P izmedu tacaka Pa i C. U prvom slucaju
imamo da je
121

P Pa = P C Pa C = PC - BP = CQ - BR = AC - AB.
U drugom slucaju bice P Pa = AB AC. Drugi deo dokazuje se neposredno,
jer je

Pb Pc = Pb P + P Pc = AC + AB
.
(d) Tacke P i Pa su izmedu tacaka B i C takve da je BP = CPa , pa se srediste
stranice BC poklapa sa sredistem duzi P Pa . Tacka Pb je iza C u odnosu na B, a
tacka Pc iza B u odnosu na C pri cemu je CPb = BPc , pa se srediste A1 stranice
BC poklapa sa sredistem duzi Pb Pc .
(e) Duz A1 M je srednja linija konveksnog trapeza Pb Pc Sc Sb . Zaista, prema
prethodnom delu ovog zadatka, tacka A1 je srediste kraka Pb Pc , a duz A1 M
upravna na pravoj BC, dakle uporedna sa stranicama Sb Pb i Sc Pc tog trapeza.
Kako je duz MN precnik kruga l, ugao MAN je prav, pa je prava AM upravna
na simetrali AN unutrasnjeg ugla trougla ABC. Otuda sleduje da je tacka M
na simetrali Sb Sc spoljasnjeg ugla trougla ABC, i prema tome da je duz A1 M
srednja linija trapeza Pb Pc Sc Sb . Stoga je
1
1
(Sb Pb + Sc Pc ) = (b + c )
2
2
Isto tako duz A1 N spaja sredista krakova konkavnog trapeza P Pa Sa S, pa je
A1 M =

A1 N =

1
1
(Sa Pa SP ) = (a )
2
2

(z) Duz M M spaja sredista krakova konkavnog trapeza Rb Rc Sc Sb , a duz


N N sredista krakova konveksnog trapeza RRa Sa S, pa je
1
1
(Sb Rb Sc Rc ) = (b c ) i
2
2
1
1
N N = (SP + Sa Pa ) = ( + a ).
2
2
MM =

(z) S obzirom da su tacke M i N sredista duzi Sb Sc i SSa , tacke M i N su


sredista Rb Rc i RRa . Otuda je s obzirom na raspored tacaka
1
1
a = (b c) i
2
2
1
1
AN = ARa - N Ra = p - a = ( b+ c ).
2
2

AM = ARb M Rb = (p c)

(i) Kako je tacka A1 izmedu tacaka M i N, bice A1 M + A1 N = M N , odakle


s obzirom na jednakosti

A1 M =

1
(b + c )
2
122

A1 N =

1
(a )
2

MN = 2r
nalazimo da je

a + b + c = 4r +
Rb
Qc
Sb
M
M

A
Sc
Rc

Qb

R
S

Pc

B
N

A1

Pa

Pb
C
Qa

Ra

Sa

Slika 121
155. Ako su A1 , B1 , C1 sredista stranica BC,CA,AB ostrouglog ili pravouglog trougla ABC, O i r srediste i poluprecnik opisanog kruga, a poluprecnik
upisanog kruga, dokazati da je
OA1 + OB1 + OC1 = r + .
Resenje:
S obzirom da trougao ABC nije tupougli, srediste O opisanog kruga je u tom
trouglu ili na njemu, pa je
1
OA1 = r (a )
2
1
OB1 = r (b )
2

123

1
OC1 = r (c )
2
i prema tome
3
1
OA1 + OB1 + OC1 = 3r (a + b c ) +
2
2
Prema prethodnom zadatku, imamo da je
a + b + c = 4r +
pa je

OA1 + OB1 + OC1 = r + .

156. Ako su A1 , B1 , C1 sredista stranica BC, CA, AB trougla ABC kome je


ugao A tup, O i r srediste i poluprecnik kruga l opisanog oko trougla ABC i
poluprecnik upisanog kruga, dokazati da je
OA1 + OB1 + OC1 = r + .
Resenje:
Kako je ugao A trougla ABC tup, srediste opisanog kruga je izvan tog trougla. Uz oznake iz zadatka 154. , smatramo da je tacka N sa one strane stranice
BC sa koje je tacka A.
Dalje se zadatak resava kao prethodni.
157. Ako obelezimo sa ABCD konveksan cetvorougao upisan u krugu k i sa
1 , 2 , 3 , 4 poluprecnici krugova upisanih u trouglove BCD, CDA, DAB, ABC,
dokazati da je
1 + 3 = 2 + 4 .
Resenje:
Srediste O kruga k moze da ima razne polozaje u odnosu na trouglove BCD,
CDA, DAB, ABC. Neka je npr. tacka O u trouglovima ABC i ABD i prema
tome izvan trouglova CDA i BCD. Ako obelezimo sa d1 , d2 , d3 , d4 , d5 , d6 odstojanja tacke O od tetiva AB, BC, CD, DA, AC, BD i sa r poluprecnik kruga k,
prema Karnoovoj teoremi, imamo da je

r + 1 = d2 + d3 d6
r + 2 = d3 + d4 d5
r + 3 = d1 + d4 + d6

124

r + 4 = d1 + d2 + d5
Iz prve i trece, zatim druge i cetvrte od ovih jednakosti nalazimo da je

1 + 3 = 2 + 4 = d1 + d2 + d3 + d4 2r
Analogan postupak primenjuje se i u slucaju kada tacka O ima neki drugi
polozaj u odnosu na trouglove BCD, CDA, DAB, ABC.
158. Ako je H ortocentar ostrouglog trougla ABC, r poluprecnik opisanog
kruga, poluprecnik upisanog kruga i a poluprecnik spolja upisanog kruga koji
dodiruje stranicu BC, dokazati da je
(a) AH = 2r + a
(b) AH BH + CH = 2(r + ).

Resenje:

H S
O

A1 P a

DP

Sa

Slika 122

125

(a) Obelezimo sa O srediste opisanog kruga, sa A1 srediste stranice BC i sa


N tacku u kojoj simetrala ugla A sece opisani krug. Pri tome je tacka A1 izmedu
1
1
tacaka O i N, pa je OA1 =ON-A1 N. Kako je OA1 = AH, ON=r, A1 N= (a -),
2
2
imamo da je AH=2r+-a (sl.122).
(b) Prema prethodnom delu ovog zadatka, a prema zadatku 154, imamo da
je
AH + BH + CH = 6r + 3 (a + b + c ) = 2(r + )
159. Ako su M iN tacke u kojima krug odredjen temenima B, C i sredistem
S upisanog kruga trougla ABC sece prave AB i AC, dokazati da je prava M N
zajednicka dirka upisanih krugova trougla ABC koji dodiruju stranicu BC.
Resenje:
A

M
Sa

Slika 123
Kod tetivnih cetvorouglova SBCN i SBMN imamo da je ANS=SBC i
SNM=SBA. No SBA=SBC, pa je ANS=SNM. Stoga je prava NS simetrala ugla ANM, te i prava MN dodiruje krug k upisan u trougao ABC.
Analognim postupkom dokazuje se da prava MN dodiruje i spolja upisani krug
ka trougla ABC (sl. 123).
160. Ako su Sb i Sc sredista spolja upisanih krugova kb i kc trougla ABC, a
M i N tacke u kojima krug opisan oko tetivnog cetvorougla BCSb Sc sece prave
AB i AC, dokazati da je prava M N zajednicka spoljasnja dirka krugova kb i kc .
Resenje:
Ako su Sb i Sc sredista spolja upisanih krugova kb i kc trougla ABC, a M i
N tacke u kojima krug opisan oko tetivnog cetvorougla BCBb Sc sece prave AB
i AC, dokazati da je prava MN zajednicka spoljasnja dirka krugova kb i kc .
161. Ako se dijagonale AC i BD tetivnog cetvorougla ABCD seku u taci O
pod pravim uglom, dokazati da su podnozja, P, Q, R, S upravnih iz tacke O na
126

pravama AB, BC, CD, DA temena tangentnog i tetivnog cetvorougla.


Resenje:

D
R
C
S

Slika 124
S ozirom da su cetvorouglovi OSAP, OPBQ, ABCD tetivni, imamo da je
SPO=SAO=DAC=DBC=
=OBQ=OPQ pa je tacka O na simetrali ugla P cetvorougla PQRS. Na isti
nacin dokazuje se da je tacka O i na simetralama ostalih unutrasnjih uglova
cetvorougla PQRS, pa je taj cetvorougao tangentni. Kako je SAO=SPO i
SDO=SRO, bice SAO+SDO=SPO+SRO. Trougao AOD je pravougaoni, pa je zbir uglova SAO i SDO, dakle zbir uglova SPO i SRO jednak pravom
uglu. Na isti nacin dokazuje se da je zbir uglova OPQ i ORQ jednak pravom
uglu, pa je zbir uglova SPO, OPQ, SRO, ORQ, dakle i zbir uglova SPQ i SRQ
jednak zbiru dva prava ugla. Otuda sleduje da je cetvorougao PQRS tetivan
(sl.124).
162. Ako je ABCD tetivan i tangentan cetvorougao cije stranice AB, BC, CD, DA
dodiruju upisani krug u tackama P, Q, R, S dokazati da se duzi P R i QS seku
pod pravim uglom.
Resenje:

127

D
R
C
S
O
Q

A
P
B

Slika 125

Cetvorougao
ABCD je konveksan, pa se duzi PR i QS koje spajaju tacke na
naspramnim stranicama seku u nekoj tacki O. Ako sa T obelezimo prav ugao,
kod tetivnih cetvorouglova APOS i CROQ imamo da je SAP+GCR=
DAP+BCD=2T, ASO+OQC=
ASO+OSD=2T, APO+ORC=
APO+OPB=2T. S obzirom da je zbir svih unutrasnjih uglova cetvorouglova
APOS i CROQ jednak 8T, iz dobijenih jednakosti nalazimo da je SOP+QOR=
2T. Kako su ti uglovi suplementni i unakrsni, oni su pravi, pa je PRQS (sl.125).
163. Ako je ABCD tetivan cetvorougao, dokazati da su sredista SA , SB , SC , SD
krugova upisanih u trouglove BCD, CDA, DAB, ABC temena pravouglog paralelograma.
Resenje:

128

D
Sa

Sb

O
S
Sd

Sc
A

B
P

Slika 126
Neka su P, Q, R, S sredista lukova AB, BC, CD, DA kruga l opisanog oko
trapeza ABCD. Tacke A, B, SC , SD pripadaju jednom krugu kome je srediste
P, pa je trougao PSC SD jednakokrak. Prava PR je simetrala ugla CPD, tj. ugla
P jednakokrakog trougla PSC SD , pa je SC SD PR. Isto tako je SA SB PR, pa
je SA SB kSC SD . Na isti nacin dokazuje se da je SB SC SQ i SA SD SQ, pa je
SB SC kSA SD . Otuda sleduje da je cetvorougao SA SB SC SD paralelogram. No
prave PR i QS su medu sobom upravne, pa su i uglovi pomenutog paralelograma
pravi, prema tome, on je pravougaoni (sl.126).
164. Ako je D podnozje visine koje odgovara hipotenuzi BC pravouglog
trougla ABC, dokazati da je prava odredjena sredistima S1 i S2 krugova k1 i
k2 upisanih u trouglove ABD i ACD upravna na simetrali ugla A trougla ABC.
Resenje:

129

S2

F2

S1
F1
B

E1 D

E2

Slika 127
Prvi nacin: S obzirom da je kod trougla ABC ugao A prav, podnozje D
visine iz temena A je izmedu tacaka B i C. Raspolovnica AS2 ugla CAB sece
duz CD dakle i polupravu BD u nekoj tacki E2 . Stoga raspolovnica BS1 ugla
AE2 dakle i polupravu AS2 u nekoj tacki B. S obzirom da su
ABD sece duZ
uglovi ABD i CAD komplementni sa istim uglom BAD, oni su jednaki, te su
i polovine tih uglova jednake, naime bice ABS1 =CAS2 . No ugao CAS2 je
komplementan sa uglom BAS2 , pa je ugao ABS1 komplementan sa uglom BAS2 .
Dakle bice u trouglu ABB uglovi ABB i BAB komplementni, pa je ABB
prav, i prema tome BS1 AS2 . Istim postupkom dokazuje se da je CS2 AS1
u nekoj tacki C.Ovim smo dokazali da se visine S1 B i S2 C trougla AS1 S2
nalaze na simetralama BS1 i CS2 uglova B i C, trougla ABC, pa se srediste S
kruga upisanog u trougao ABC poklapa sa ortocentrom trougla AS1 S2 . Otuda
je S1 S2 AS (sl.127).
Drugi nacin: Ako obelezimo sa E1 i E2 tacke u kojima simetrale AS1 i AS2
uglova BAD i CAD seku duzi BD i CD, a sa F1 i F2 tacke polupravih AB i AC
takve da je AF1 =AD i AF2 =AD, bice cetvorouglovi ADE1 F1 i ADE2 F2 tangentni i prosti kojima se upisani krugovi k1 i k2 nalaze u njima. Iz simetricnosti
cetvorougla ADE1 F1 u odnosu na pravu AE1 sledi da je ADE1 =AF1 E1 .
No ugao ADE1 je prav, pa je i ugao AF1 E1 takode prav. Srediste S1 kruga
k1 je na simetrali ugla AF1 E1 , te je AF1 S1 jednak polovini pravog ugla. S
druge strane iz jednakosti AF1 =AD i AF2 =AD sledi da je AF1 =AF2 , pa je kod
pravouglog trougla AF1 F2 svaki od uglova AF1 F2 i AF2 F1 jednak polovini pravouglog trougla. S obzirom da su tacke S1 i F2 sa iste strane prave AF1 , a uglovi
AF1 S1 i F1 F2 jednaki, tacka S1 je na pravoj F1 F2 . Istim postupkom dokazuje
130

se da je tacka S2 na pravoj F1 F2 . No prava F1 F2 odredena osnovicom F1 F2 jednakokrakog trougla AF1 F2 upravna je na simetrali AE ugla A, te je i S1 S2 AE.
165.Ako su pb i pc , qc i qa , ra i rb trisektori unutrasnjih uglova A, B, C proizvoljnog trougla ABC pri cemu se poluprave pb i pc nalaze do polupravih AC i
AB koje sadrze stranice naspram temena B i C, itd, dokazati da su tacke P , Q,
R u kojima se seku poluprave qa i ra , rb i pb , pc i qc temena jednakostranicnog
trougla (Morlejeva teorema).
Resenje:

A
N

M
D
R

P
B

Slika 128
Ako obilezimo sa P tacku u kojoj se seku poluprave qc i rb , bice tacka P
srediste kruga k upisanog u trougao PBC. Neka su M i N tacke u kojima krug
k dodiruje stranice BP i CP, a M i N tacke u kojima prave PM i PN seku stranice AB i AC.Tacka M je izvan kruga k te postoje dve prave koje sadrze tacku
M i dodiruju krug k. Obelezimo sa D onu od dodirnih tacaka tih tangenata
koja se nalazi sa one strane prave MP s koje je tacka Q, a sa Q tacku u kojoj
prava MD sece polupravu rb . Kod pravouglog trougla PDM hipotenuza MP je
2
1
dva puta veca od katete PD, pa je MPD= T i PMD= T, gde je T prav
3
3
ugao. S obzirom da je cetvorougao PMPN tetivan, imamo da je MPN=2T
2
2
- MPN=2T-BPC= (B+C)= (2T-A). Iz simetricnosti tacaka P i N u od3
3
1
nosu na pravu MQ i iz dobijenih jednakosti sledi da je QNN=QPN=
2
131

DPN=
1
1
1
1
(MPN=MPD)= (2T-A)- T= (T-A). Kod trougla PMN stranice PM i
2
3
3
3
1
1
PN su jednake, pa je MNP=NMP= (2T-MPN)= (2T-MPN)=
2
2
1
(T+A). Najzad, kod trougla MQN imamo da je MNQ=MNP-QNP=
3
1
2
A i NMQ=NMP-QMN= A, pa je MQN=2T-A. Stoga su uglovi MAN
3
3
i MQN suplementni i prema tome tacke A, M, Q, N na jednom krugu. Sad
2
1
je MNQ=MAQ= A i NMQ=NAQ= A, te su poluprave AQ i AQ, a
3
3
prema tome i tacke Q i Q istovetne. Na taj nacin dokazano je da prava MQ
dodiruje krug k u tacki D. Istim postupkom dokazuje se da i prava NR dodiruje krug k u nekoj tacki E. Pri tome su tacke M i D simetricne sa tackama
N i E u odnosu na simetralu ugla MPN, pa su i tacke Q i R simetricne medu
sobom u odnosu na simetralu ugla MPN. Stoga je PQ=PR i RPQ=MPN2
2QPN= T, pa je trougao PQR jednakostranican (sl.128).
3

132

I SLICNOST

PROPORCIONALNOST DUZI
LIKOVA

166. Ako su C i C tacke dveju uporednih duzi AB i A B takve da je


AB : CB = A C : C B , dokazati da se prave AA , BB , CC seku u jednoj
tacki ili su medu sobom uporedne.

Slika 129
Prave AA i BB se seku ili su uporedne. Ako se prave AA i BB seku u
nekoj tacki O, i prava CC sadrzi tacku O. Zaista, ako obelezimo sa C tacku
u kojoj se seku prave OC i A B ,bice AC : CB = A C : C B i AC : CB =
A C : CB , pa je A C : C B = A C : CB .Iz ove jednakosti sleduje da su
tacke C i C istovetne, te i prava CC sadrzi tacku O(sl. 129).
Ako su prave AA i BB uporedne,prave kroz tacku C uporedna s njima
sece pravu A B u nekoj tacki C. Pri tome je AC : CB = A C : C B i
AC : CB = A C : CB , pa je A C : C B = A C : CB . Stoga su tacke C
iC istovetne, i prema tome prave AA ,BB ,CC uporedne.
167. Ako su A, B, C tri tacke neke prave p, a A , B , C tacke neke druge
prave p takve da je AB k BA i AC k CA , dokazati da je i BC k CB
(Paposova teorema).

A
B
C
p

Slika 130a
Prave p i p se seku ili su medu sobom uporedne. Ako se prave p i p seku
u nekoj tacki O, bice OA : OB = OB : OA i OA : CO = OC : OA . Iz ovih
133

dveju jednakosti nalazimo da je OB : OC = OC : OB te je BC k CB (sl.


130a).

pA

Slika 130b
Ako su prave p i p uporedne medu sobom, bice cetvorouglovi ABA B i
ACA C paralelogrami, te su duzi AB i AC jednake i istosmerne sa duzima
B A i C A , i prema tome duzi BC i C B jednake i istosmerne. Otuda sleduje
da je cetvorougao BCB C paralelogram, te je BC k CB (sl. 130b).
168. Ako su A, B, C tri tacke jedne prave, a A , B , C tacke izvan te prave
takve da je AB k BA , AC k CA i BC k CB , dokazati da tacke A , B , C
takode pripadaju jednoj pravoj (Obratna Paposova teorema).
p

pA

C = C

Slika 131
Dokaz izvedimo indirektno. Stoga pretpostavimo da tacke A , B , C ne pripadaju jednoj pravoj. Pri tome prava A B sece pravu AC ne u tacki C , vec
u nekoj tacki C. Sad su A, B, C tacke jedne prave, a A , B , C tacke druge
prave pri cemu je AB k BA i AC k CA , te je prema prethodnom zadatku
BC k CB . U tom slucaju postoje dve razne prave BC i BC uporedne sa
pravom CB sto je nemoguce. Stoga tacke A , B , C pripadaju jednoj pravoj(sl.131).
169. Ako su A1 , B1 , C1 sredista stranica BC, CA, AB trougla ABC, a
M i N tacke u kojima proizvoljna prava kroz teme A sece prave A1 C1 i A1 B1 ,
dokazati da je BM k CN .

134

B1

C1
M
B

A1

Slika 132
S obzirom da su A1 , B, C tri razne tacke jedne prave i A, M , N tri razne
tacke neke druge prave takve da je BA k A1 N i CA k A1 M , prema direktnojPaposovoj teoremi bice i BM k CN (sl. 132).
170. Kroz naspramna temena A i C paralelograma ABCD konstruisane su
dve paralelne prave od kojih prva sece prave odredene stranicama BC i CD u
tackama P i Q, a druga sece prave odredene stranicama AB i AD u tackama R
i S, dokazati da je P R k QS.
Uputstvo: Primeniti direktnu Paposovu teoremu.
171. Ako su H i D ortocentar i podnozje visine iz temena A trougla ABC,
a M i N take u kojima upravne iz take D na pravama AB i AC seku prave
koje su u tackama B i C upravne na stranici BC, dokazati da tacke H, M , N
pripadaju jednoj pravoj.
N

M
B

Slika 133
Tacke B,C,D pripadaju jednoj pravoj, a tacke H,M ,N su takve da je BM k
CN , BH k DN i DM k CH,te su prema obratnoj Paposovoj teoremi tacke
H,M ,N na jednoj pravoj (sl.133).
172. Ako su A , B , C tacke u kojima proizvoljna prava s sece prave
odredene stranicama BC, CA, AB trougla ABC, dokazati da ortocentri trouglova AB C , A BC , A B C pripadaju jednoj pravoj.
135

Hc

A
Ha

C
H
B

Hb

Slika 134
Obelezimo sa H,Ha,Hb,Hc ortocentre trouglova ABC, AB C ,A BC ,A B C.
S obzirom da su A ,B,C tri razne tacke jedne prave,a H,Hb,Hc tri tacke izvan
te prave takve da je BHb k CHc,BH k A Hc i A Hb k CH, prema obratnoj
Paposovoj teoremi tacke H,Hb,Hc pripadaju jednoj pravoj.
Istim postupkom dokazuje se da i tacke H,Ha,Hb pripadaju jednoj pravoj. No kako dve razne tacke odreduju uvek jednu i samo jednu pravu, tacke
H,Ha,Hb,Hc pripadaju jednoj pravoj (sl.134).
173. Dokazati da su ortocentri cetriju trouglova koji su odredeni sa cetiri
prave od kojih nikoje dve nisu paralelne i nikoje tri nisu konkurentne pripadaju
jednoj pravoj.
F

H1
H4
D

C
H3

H2

Slika 135

Cetiri
prave od kojih nikoje dve nisu paralelne i nikoje tri nisu konkurentne
odreduju izvestan cetvorougao ABCD kome se prave odredene naspramnim
stranicama AB i CD seku u nekoj tacki E, a prave odredene naspramnim
stranicama BC i AD seku u nekoj tacki F .Neka su H1 ,H2 ,H3 ,H4 ortocentri
trouglova ADE,BCE,ABF ,CDF . S obzirom da su A,B,E tri razne tacke jedne
prave, a H1 ,H2 ,H3 tri tacke takve da je AH1 k BH2 ,AH3 k EH2 i BH3 k EH1 ,
prema obratnoj Paposovoj teoremi tacke H1 ,H2 ,H3 pripadaju jednoj pravoj.
Istim postupkom dokazuje se da i tacke H1 ,H2 ,H4 pripadaju jednoj pravoj.
No kako dve razne tacke odreduju jednu i samo jednu pravu,tacke H1 ,H2 ,H3 ,H4
pripadaju jednoj pravoj (sl.135).

136

174. Ako su E i F tacke u kojima simetrale unutrasnjeg i spoljasnjeg ugla


A trougla ABC seku pravu BC, dokazati da je
(a) BE : CE = AB : AC;
(b) BF : CF = AB : AC.

A
D
F

Slika 136
(a) Ako je D tacka iza A u odnosu na C takva da je AB = AD, bice
BD k AE, pa je BE : CE = AD : AC i prema tome BE : CE = AB : AC
(sl.136).
(b) Ako je D tacka poluprave AC takva da je AB = AD , bice BD k AF ,
pa je BF : CF = AD : AC, i prema tome BF : CF = AB : AC
175. Ako obelezimo sa S srediste upisanog kruga trougla ABC, sa Sa , Sb ,
Sc sredista spolja upisanih krugova koji odgovaraju redom stranicama BC, CA,
AB a sa E i F tacke u kojima simetrale unutrasnjeg i spoljasnjeg ugla A seku
pravu BC, dokazati da je
(a) AS : SE = ASa : Sa E = (AB + AC) : BC;
(b) ASb : Sb F = ASc : Sc F = |AB AC| : BC.
Uputstvo:Koristiti prethodni zadatak i osobine proporcija.
176. Ako su S, Sa , Sb , Sc sredista upisanih krugova trougla ABC, zatim P ,
Pa , Pb , Pc tacke u kojima ti krugovi dodiruju pravu BC i A1 srediste stranice
BC, dokazati da je
(a) SA1 k APa ;
(b) Sa A1 k AP ;
(v) Sb A1 k APc ;

137

(g) Sc A1 k APb .
A
P
S
B

EA1 P a

Sa

Slika 137
A sece stranicu BC i sa P tacku u kojoj se seku SP i APa ,bice ASP
ASa Pa kao i SP E Sa Pa E, pa je zbog toga AS : ASa = SP :
Sa Pa ,takode i ES : ESa = SP : Sa Pa .Ali kako je AS : ASa = ES : ESa , bice i
SP : Sa Pa = SP : Sa Pa .Iz dobijene proporcije nalazimo da je SP = SP ,pa je
tacka S srediste duzi P P .Prema zadatku 154 ,tacka A1 je srediste duzi P Pa ,te
je SA1 k P Pa i prema tome SA1 k APa .
177. Ako su D i A1 podnozje visine iz temena A i srediste stranice BC
trougla ABC, zatim k(S, ), ka (Sa , a ), kb (Sb , b ), kc (Sc , c ) upisani krugovi
trougla ABC i X, Xa , Xb , Xc tacke u kojima prave SA1 , Sa A1 , Sb A1 , Sc A1
seku pravu AD, dokazati da je
(a) AX =
(b) AXa = a
(v) AXb = b
(g) AXc = c

138

A
K

S
B

DPE A1 P a

Sa

Slika 138
(a)Ako obelezimo sa P i Pa tacke u kojima krugovi k i ka dodiruju stranicu
BC,sa E tacku u kojoj prava AS sece stranicu BC i sa K tacku u kojoj se seku
prave SP i APa , bice ASK ASa Pa i takode SP E Sa Pa E, pa
dobijamo da je SA : Sa A = SK : Sa Pa i SE : Sa E = SP : Sa Pa . No kako je
SA : Sa A = SE : Sa E, bice i SK : Sa Pa = SP : Sa Pa , i prema tome SK = SP .
Sad su tacke S i A1 sredista stranica P K i P Pa trougla P KPa , te je SA1 k KPa ,
i prema tome AK k XS.Kako je pored toga AX k KS,cetvorougao AXSK je
paralelogram,te je AX = KS = SP = (sl.138).
Analognim postupkom dokazuju se i ostali.
178. Ako su X i Y tacke u kojima simetrale uglova B i C trougla ABC
seku duz koja spaja teme A s tackom Pa u kojoj spolja upisani krug ka dodiruje
stranicu BC, dokazati da je
AX : AY = AB : AC.
A

X
Y
B

Pa

Slika 139
S obzirom da je X tacka u kojoj simetrala ugla B sece stranicu APa trougla
ABPa ,a Y tacka u kojoj simetrala ugla C sece stranicu APa trougla ACPa ,
imamo da je AX : XPa = AB : BPa i AY : Y Pa = AC : CPa , odnosno da je
AX : APa = AB : p i AY : APa = AC : p, gde je p poluobim trougla ABC.Iz
dobijenih dveju proporcija nalazimo da je AX : AY = AB : AC.

139

179. Dokazati da se duzi koje spajaju temena Ai cetvorougla A1 A2 A3 A4 sa


tezistima Ti trouglova koji su odredeni ostalim temenima, seku u jednoj tacki,
tezistu tog cetvorougla, pri cemu je
Ai T : T Ti = 3 : 1.
A3
M
A4

T2

T1

A1

A2

Slika 140
Ako obelezimo sa M srediste stranice A3 A4 ,bice tacke T1 i T2 na duzima
A2 M i A1 M takve da je A2 M : T1 M = A1 M : T2 M = 3 : 1. Stoga je
T1 T2 k A1 A2 i A1 A2 : T1 T2 = 3 : 1. Sem toga,duzi A1 T1 i A2 T2 se seku u nekoj
tacki T ,te su trouglovi T A1 A2 i T T1 T2 slicni,pa je A1 T : T T1 = A2 T : T T2 =
3 : 1.Istim postupkom dokazuje se da i duzi A3 T3 i A4 T4 seku duz A1 T1 u tacki
T ,pa se sve cetiri duzi Ai Ti za i = 1, 2, 3, 4 seku u jednoj tacki.
180. Ako je AA1 precnik kruga k, B proizvoljna tacka kruga k razlicita
od tacaka A i A , a C tacka duzi AA takva da je AB = CA , dokazati da se
simetrala ugla A, tezisna linija iz temena B i visina iz temena C trougla ABC
seku u jednoj tacki.

B
F

D
K

Slika 141
Ako obelezimo sa K tacku u kojoj se seku tezisna linija BE i visina CF ,
bice KC k BA , pa je EK : KB = EC : CA . No EC = AE i CA = AB,pa
je EK : KB = AE : AB, i prema tome prava AK simetrala ugla A trougla
BAE,dakle i trougla ABC.

140

181.Ako je D srediste stranice BC ABC, P tacka u kojoj simetrala ugla


ADB sece stranicu AB i Q tacka u kojoj simetrala ugla ADC sece stranicu
AC,dokazati da je ABC AP Q.
Uputstvo:

Slika 142
Prema zadatku 139, bice AP : BP = AD : BD i AQ : CQ = AD : CD. S
obzirom da su duzi BD i CD medusobno jednake, iz navedenih proporcija sledi
da je AP : BP = AQ : CQ, pa je P Q k BC, i prema tome ABC AP Q
(sl.142).
182.Ako je ugao A ABC ostar i ako su B i C podnozja visina iz temena
B i C,dokazati da je ABC AB C .
Uputstvo:

141

Slika 143
S obzirom da je ugao A ostar,tacke B i C su na polupravama AC i AB, pa je
BAC = B AC . . . (1).Kako su uglovi BB C i BC C pravi,tacke B,C,B ,C
pripadaju krugu l kome je duz BC precnik.Stoga je ABAC = AB AC i prema
tome AB : AC = AB : AC . . . (2). Iz (1) i (2) sledi da je ABC AB C
(sl.143).
183.Ako je A ABC ostar i ako su B i C podnozja visina iz temena B i
C, a O srediste kruga k opisanog oko tog trougla, dokazati da je OA B C .
Uputstvo:

142

D
A

Slika 144
Ako je AD dirka kruga k u tacki A i D tacka te dirke koja se nalazi sa one
strane prave AC s koje nije teme B,bice ABC = CAD. Otuda iz jednakosti
ABC = AB C ,sleduje AB C = CAD. S obzirom da je A ostar,tacke
B i C su na polupravama AC i AB. Stoga je CAD = B AD i prema tome
AB C = B AD. Sem toga,tacke C i D su s raznih strana prave AB ,te
su uglovi AB C i B AD naizmenicni. Iz jednakosti tih naizmenicnih uglova
sleduje da su prave AD i B C uporedne. Kako je prava koja sadrzi poluprecnik
OA kruga k upravna na dirku AD, ona je upravna i na pravoj B C koja je
uporedna sa AD,pa je tvrdenje dokazano (sl.144).
184.Neka je tacka S izvan kruga k. Ako su P i Q tacke u kojima tangente
kroz S dodiruju k, a A i B tacke u kojima proizvoljna prava kroz S sece k,
dokazati da je AP : BP = AQ : BQ.
Uputstvo:

143

O
A

Slika 145
S obzirom da je SAP SP B i SAQ SQB, imamo da je AP : BP =
AS : P S i AQ : BQ = AS : QS. No P S = QS, pa je i AP : BP = AQ : BQ
(sl.145).
185.Ako su A i C tacke u kojima krug kroz temena A, B, C paralelograma
ABCD sece prave AD i CD, dokazati da je A B : A C = A C : A D.
Uputstvo:

144

Slika 146
Trouglovi A BC i A C D su slicni, pa je A B : A C = A C : A D (sl.146).
186.Ako obelezimo sa D proizvoljnu tacku prave koja je odredena stranicom
BC ABC, a sa O1 i O2 sredista krugova opisanih oko ABD i ACD, dokazati da je ABC AO1 O2 .
Uputstvo:

145

O2
O1

Slika 147
Uglovi B i C trougla ABC jednaki su, respektivno sa uglovima O1 i O2 trougla
AO1 O2 , pa je ABC AO1 O2 (sl.147).
187.Ako su p i p dve prave koje se seku u tacki O, zatim A, B, C tacke
prave p i A , B , C tacke prave p takve da je O(A, B, C), O(A , B , C ) i
AB : BC = A B : B C , dokazati da se krugovi opisani oko OAA , OBB ,
OCC seku u istim tackama ili se medu sobom dodiruju u tacki O.
Uputstvo:

146

C
C

B
B

Slika 148
Ako se krugovi opisani oko trouglova OAA i OBB seku u tacki O i tacki S,
tada i krug opisan oko OCC sadrzi tacku S.Zaista, iz tetivnih cetvorouglova
OAA S i OBB S sledi da je OAS = OA S i OBS = OB S, pa je i
BAS = B A S i ABS = A B S. Stoga je SAB SA B , pa sledi
da je AB : A B = SB : SB . No kako je AB : A B = BC : B C , vazice
i BC : B C = SB : SB . Sem toga je SBC = SB C , pa je SBC
SB C .Otuda je i OCS = OC S, pa je tacka S na krugu opisanom oko
OCC (sl.148). Slucaj kada se pomenuti krugovi dodiruju u tacki O neka
citalac dokaze sam.
188.Ako su A , B , C podnozja visina iz temena A, B, C ABC, dokazati
da je ABC A B C A BC A B C.
Uputstvo: Kako su tacke B i C na polupravama AC i AB ili na produzenjima
tih polupravih, imamo da je BAC = B AC . S obzirom da su uglovi BB C i
BC C pravi, bice cetvorougao BC B C tetivan, pa je ABC = AB C .Otuda
je ABC AB C . Analogno se dokazuje da je i ABC A BC i
ABC A B C
189.Ako je P AQ prav, A proizvoljna tacka poluprave OQ, a B, C, D tacke
poluprave OP , takve da je [OBCD] i OA = OB = BC = CD, dokazati da je
ABC DBA.
Uputstvo:

147

Slika 149
Ugao O trougla OAB je prav, pa je AB 2 = OA2 + OB 2 .S obzirom da jeOA =
OB = BC = CD i [BCD], bice AB 2 = 2 OA OA, tj. AB 2 = BC BD i prema
tome AB : BC = BD : AB. Kako je ABC = DBA i AB : BC = BD : AB,
bice ABC DBA (sl.149).
190.Ako su P i Q tacke stranica AB i AC ABC takve da je AB = nAP i
AC = (n+1)AQ, dokazati da su za sve vrednosti broja n prave P Q konkurentne.
Uputstvo:

148

Slika 150
Neka je D tacka u kojoj prava kroz teme C paralelna sa stranicom AB sece fiksiranu pravu P Q. Pri tome je AP Q CDQ, pa je AP : CD = AQ : QC =
l : n. Iz ove proporcije AP : P B = l : n sledi da je AP : CD = AP : P B, i prema
tome da je AB = CD. Stoga je cetvorougao ABCD paralelogram, te za svako
n prava P Q sadrzi istu tacku D (sl.150).
191.Ako su P , Q, R tacke stranica BC, CA, AB ABC takve da je
BP : P C = CQ : QA = AR : RB, dokazati da se tezista ABC i P QR
poklapaju.
Uputstvo:

149

R
Q

T
R

Slika 151
Neka su AA i RR tezisne linije trouglova ABC i P QR. Ako je M tacka simetricna s tackom P u odnosu na tacku A , bice cetvorougao ARM Q paralelogram,
pa je M Q k RA i M Q = RA.No, A R k M Q i A R = 21 M Q, pa je A R k RA
i A R = 21 RA. Stoga se duzi AA i RR seku uizvesnoj tacki T pri cemu je
AT : T A = RT : T R , pa setezista trouglova ABC i P QR poklapaju (sl.151).
192.Ako su P , Q, R tacke stranica BC, CA, AB ABC takve da je
BP : P C = CQ : QA = AR : RB = k, dokazati da postoji trougao kome
su stranice jednake duzima AP , BQ i CR.
Uputstvo:

150

Slika 152
Ako obelezimo sa S tacku takvu da je cetvorougao BP SQ paralelogram, bice
AP S trougao kome je AP = AP i P S = SQ,dokazimo da je i SA = CR. U
tom cilju obelezimo sa T tacku u kojoj prava SQ sece stranicu AB. S obzirom
da je BC k T Q, bice (sl.152) BC : T Q = AC : AQ = k + 1.Iz ove jednakosti
i iz BC : P C = k + 1 sledi da je T Q = P C, i prema tome BC = T S. Iz
BC k T Q imamo da je AB : AT = AC : AQ = k + 1, pa, s obzirom na jednakost
AB : RB = k + 1, imamo da je AT = RB. Stoga je RBC
= AT S, pa je i
SA = CR. Ovim je teorema dokazana.
193.Ako je D srediste osnove BC jednakokrakog ABC, E podnozje upravne
iz D na stranici AC, F srediste duzi DE, dokazati da je duz BEupravna na
duzi AF .
Uputstvo:

151

G
E
F
B

Slika 153
Neka je BG visina iz temena B ABC. S obzirom da je tacka D srediste
stranice BC, a DF k BG, bice tacka E srediste duzi CG. Iz jednakosti uglova
ACD i ADE, zatim uglova ADC i BGE sledi da su trouglovi BCG i ADE
slicni. Kako su tacke E i F sredista odgovarajucih stranica CG i DE tih slicnih
trouglova, bice i trouglovi BEG i AF E slicni, pa je EAF = GBE. Kako je
EA GB i EAF = GBE sledi da je BE AF (sl.153).
194.Neka su ABC i A B C dva homoteticna trougla u odnosu na neku
tacku O, a p, q,r prave kroz tacku O uporedne sa pravama BC, CA, AB.Ako
P QR, koji je upisan u trougao ABC, odreduje na stranicama A B C jednake odsecke, dokazati da P QR odreduje i na pravama p, q, r jednake odsecke.
Uputstvo:

152

A
R1

Q2
R2

Q2
R2
P1
B

R1
P1

Q
Q1
P2

Q1
P2

Slika 154
Obelezimo sa P1 i P2 , Q1 i Q2 , R1 i R2 tacke u kojima prave p,q,r seku stranice
P R i P Q,QP i QR, RQ i RP , a sa P1 i P2 , Q1 i Q2 ,R1 i R2 tacke u kojima
prave B C , C A ,A B seku stranice P R i P Q, QP i QR, RQ i RP . Pri tome
je P1 P2 : P1 P2 = CO : CC = QQ1 : QQ1 = Q1 Q2 : Q1 Q2 . Po pretpostavci je
P1 P2 = Q1 Q2 , pa je i P1 P2 = Q1 Q2 . Isto tako je i Q1 Q2 = R1 R2 , pa su
odsecci P1 P2 , Q1 Q2 , R1 R2 koje odreduje P QR na pravama p, q, r medu
sobom jednaki (sl.154).
195.Ako su B i C tacke u kojima prave AB i AC dodiruju krug k, a P , Q,
R podnozja upravnih iz proizvoljne tacke S toga kruga na pravama BC, CA,
AB,dokazati da je SP 2 = SQ SR.
Uputstvo:

153

B
k
R

A
Q

Slika 155
S obzirom da je SP Q = SCQ = SBC = SBP = SRP i SQP =
SCP = SCB = SBR = SP R bice SP Q SRP pa je SP : SQ =
SR : SP i prema tome SP 2 = SQ SR (sl.155).
196. Ako je k krug opisan oko trougla ABC, t dirka kruga k u tacki A i
D tacka u kojoj prava kroz B uporedna sa t sece AC, dokazati da je
AB 2 = AC AD.
t

A
T
D

Slika 156
Ako sa T obelezimo proizvoljnu tacku dirke t koja se nalazi s one strane
prave AB s koje nije tacka C, bice T AB = ACB i T AB = ABD, pa
154

je ACB = ABD. Sem toga je BAC = DAB, pa je ABC ADB.


Otuda je AB : AD = AC : AB, i prema tome AB 2 = AC AD ( sl. 156 ).
197. Ako je D tacka u kojoj simetrala ugla A sece stranicu BC trougla
ABC, E tacka u kojoj upravna kroz D na simetrali unutrasnjeg ugla B sece
pravu AB i F tacka u kojoj upravna kroz D na simetrali spoljasnjeg ugla C
sece pravu AC, dokazati da je
AD2 = AE AF.
P1M
A
Q

S O

C
N Q1
Sa

Slika 197
Kod trouglova ADE i AF D je EAD = DAF , a AED = ADF ,
jer je AED = R + B2 i ADF = R + B2 gde je R prav ugao. Otuda je
ADE AF D, pa je AE : AD = AD : AF , a prema tome AD2 = AE AF
(sl. 157).
198. Ako upravna kroz proizvoljnu tacku P hipotenuze BC pravouglog
trougla ABC sece prave AC i AB u tackama Q i R, a opisani krug oko
trougla ABC u tacki S, dokazati da je
P S 2 = P Q P R.
A

Slika 198
Duz SP je visina koja odgovara hipotenuzi BB pravouglog trougla BSC, pa
je SP 2 = BP CP ...( 1 ). Kod pravouglih trouglova P BR i P QC jednaki su
ostri uglovi B i Q, pa je P BR P QC i prema tome BP : P R = P Q : CP .
Otuda je BP CP = P Q P R...( 2 ). Iz jednakosti ( 1 ) i ( 2 ) sledi da je
P S 2 = P Q P R ( sl. 198 ).

155

199. Ako je P QRS kvadrat upisan u pravougli trougao ABC pri cemu su
temena P i Q hipotenuze BC, a temena R i S na stranicama AC i AB, dokazati
da je
P Q2 = BP CQ.
X

O
A

Slika 199
Ako je T tacka poluprave CB takva da je RS = CT , bice ST k AC, pa je ugao
BST prav. Stoga je duz SP visina koja odgovara hipotenuzi BT pravouglog
trougla SBT i prema tome SP 2 = BP T P . Kako je P QRS kvadrat, bice
SP = P Q, a iz podudarnih trouglova P ST i QRC nalazimo da je T P = CQ.
Otuda je P Q2 = BP CQ ( sl. 159 ).
200. Ako su P , Q, R tacke u kojima proizvoljna prava kroz teme A paralelograma ABCD sece prave BC, CD, BD, dokazati da je
AR2 = P R QR.
D

Slika 200
S obzirom da je
ARD P RB
i
QRD ARB
imamo da je
AR : P R = DR : ER
i
QR : AR = DR : BR
Iz ovih proporcija sledi da je
AR : P R = QR : AR
156

pa je
AR2 = P R QR
( sl. 200 ).
201. Prava kroz presek S dijagonala AC i BD uporedna sa stranicom AB
cetvorougla ABCD sece prave CD, BC, AD u tackama P , Q, R. Dokazati da
je
P S 2 = P Q P R.

C
Y

Slika 201
Ako sa O obeleimo tacku u kojoj se seku prave AB i CD, bice
SQ : QP = AB : BO
i
RS : SP = AB : BO
Iz ovih dveju proporcija sledi da je
SQ : QP = RS : SP
Otuda je
(SQ + QP ) : CP = (RS + SP ) : SP
tj.
SP : CP = RP : SP
i prema tome
P S2 = P Q P R
( sl. 201 ).
202. Ako su A, B, C tri razne tacke kruga k, A i B upravne projekcije
tacaka A i B na pravoj c koja u tacki C dodiruje krug k i C upravna projekcija
tacke C na pravoj AB, dokazati da je
AA BB = CC 2 .

157

Slika 202
Ako su prave p i AB uporedne, stav je jednostavan. Ako se prave p i AB
seku u nekoj tacki O, bice trouglovi OAA , OBB , OCC slicni pa je
OA
AA
=

CC
OC
BB
OB
=

CC
OC
tj.
AA =

OA
OB
CC , BB =
CC
OC
OC

. Otuda je
AA BB =

OA OB
CC 2 = CC 2
OC 2

( sl. 202 ).
203. Ako su AB i CD osnovice jednakokrakog trapeza ABCD opisanog oko
kruga poluprecnika r, dokazati da je
AB CD = 4 r2 .
D

Slika 203
Ako su P , Q, R, S tacke u kojima stranice AB, BC, CD, DA, dodiruju
opisani krug k kome je srediste O, bice P OQ = QCR, pa je i P OB =
RCO. Stoga su pravougli trouglovi P OB i RCO slicni, pa je BP : OR =
OP : CR. No, BP = 21 AB, CR = 21 CD, OP = OR = r, pa je AB CD = 4r2
( sl. 203 ).
204. Ako su a, b, c, d duzi jednake stranicama AB, BC, CD, DA a x i y
duzi jednake dijagonalama AC i BD konveksnog i tetivnog cetvorougla ABCD,
dokazati da je
ad+bc
x
=
.
y
ab+cd
158

M
A

B
N

Slika 204
Ako obelezimo sa O presek dijagonala AC i BD, bice OAB OBC, pa
je ( sl. 204 )
OA
OD OA
OB
=
,
=
AB
CD AD
BC
OD
OC
=
BC
AD
otuda je
OA
OD
OA
=
,
=
AB AD
CD AD AB AD
OC
OD
OB
,
=
BC AD BC CD
AD CD
i prema tome
OB
OC
OD
OA
=
=
=
.
AB AD
AB BC
BC CD
CD AD
I z ovih jednakosti nalazimo da je
OA + OC
AB AD + BC CD
=
,
OB + OD
AB BC + AD CD
tj. da je

x
ad + bc
=
.
y
ab + cd
159

205. Ako su kod trougla ABC stranice BC, CA, AB jednake duzima a, b, c
i ako je A = 2B, dokazati da je
a2 = b (b + c).

S
D
B

Slika 205
Ako je D tacka u kojoj raspolovnica ugla A sece stranicu BC, bice ABC
DAC, pa je
BC : AC = AC : CD
( sl. 205 ).
206. Ako su kod trougla ABC stranice BC, CA, AB jednake duzima a, b, c
i ako je A = 2B, dokazati da je
a2 = b (b + c).
Ako je D tacka iza A u odnosu na C takva da je AB = AD, bice BAC
spoljasnji ugao pri vrhu jednakokrakog trougla ABD, pa je BDA = 12 BAC,
i prema tome ABC = BDC. S obzirom da je BCA = DCB i ABC =
BDC, bice ABC BDC, pa je BC : AC = CD : BC tj. BC 2 = AC CD,
i prema tome a2 = b(b + c).

B
B
B

C
E

Slika 206
Ako je D tacka iza A u odnosu na C takva da je AB = AD, bice ugao BAC
spoljasnji ugao pri vrhu jednakokrakog trougla ABD, pa je BAC = 2BDA,
160

tj. BAC = 2BDC. S obzirom da je a2 = b(b + c), bice BC 2 = CA CD, tj.


BC : CA = CD : BC. Iz ove proporcije i jednakosti uglova BCA i DCB sledi
da je ABC BDC, pa je ABC = BDC. Otuda je A = 2B ( sl. 206
).
207. Ako su kod trougla ABC stranice BC, CA, AB jednake duzima a, b, c
i ako je a2 = b (b + c), dokazati da je
A = 2B.

Slika 207
Ako je D tacka iza A u odnosu na C takva da je AB = AD, bice ugao BAC
spoljasnji ugao pri vrhu jednakokrakog trougla ABD, pa je BAC = 2BDA,
tj. BAC = 2BDC. S obzirom da je a2 = b(b + c), bice BC 2 = CA CD, tj.
BC : CA = CD : BC. Iz ove proporcije i jednakosti uglova BCA i DCB sledi
da je ABC BDC, pa je ABC = BDC. Otuda je A = 2B ( sl. 166
).
208. Ako su P , Q, R tacke u kojima proizvoljna prava kroz teme A paralelograma ABCD sece prave BC, CD, BD, dokazati da je
1
1
1
=
+
.
AR
AP
AQ
S obzirom da su Q i C tacke u kojima prava uporedna sa AB sece prave
AP i BP , imamo da BP : BC = AP : AQ. Isto tako, P i B su tacke u kojima
prava uporedna sa AD sece prave RA i RD, pa je BP : AD = P R : RA. Kako
je BC = AD, leve strane dobijenih proporcija su jednake, pa je AP : AQ =
P R : RA, tj. AP : AQ = (P A + AR) : RA. Otuda je
AP
AP
=
+ 1,
AR
AQ
i prema tome

1
1
1
=
+
.
AR
AP
AQ

209. Ako su P , Q, R tacke u kojima proizvoljna prava s kroz teziste T


trougla ABC sece prave BC, CA, AB pricemu su tacke Q i R s iste strane od
T , dokazati da je
1
1
1
=
+
.
TP
TQ TR
161

D
M
C
N

M
P
S

O
N

Slika 209
Neka su B i C tacke u kojima prava kroz tacku T uporedna sa stranicom
BC sece stranice AB i AC,a B i C tacke u kojima prave kroz temena B i C
uporedne s pravom s seku pravu B C .S obzirom da su cetvorouglovi T P CC i
T P B paralelni ,a trouglovi QC T i RB T slicni s trouglovima CC C i BB B,
imamo da je
C C
C C
TP
=
=
TQ
TQ
T C
i

BB
BB
TP
.
=
=
TR
TR
T B
Otuda i iz jednakosti B T = T C = BB + C C ,sledi da je
BB
TP
TP
C C
+
=1
+
=
TQ TR
T C
T B
,pa je

1
1
1
=
+
.(sl.209).
TP
TQ TR

210. Ako su dijagonale AC i BD cetvorougla ABCD seku u tacki O i ako


prava kroz tacku O uporedna sa stranicom AB sece stranice AD i BC u tackama
A1 i B2 , prava kroz tacku O uporedna sa stranicom BC sece stranice AB i CD
u tackama B1 i C2 , prava kroz tacku O uporedna sa stranicom CD sece stranice

162

BC i AD u tackama C1 i D2 , a prava kroz tacku O uporedna sa stranicom DA


sece stranice CD i AB u tackama D1 i A2 , dokazati da je
B1 C2
C1 D2
D1 A2
A1 B2
+
+
+
= 4.
AB
BC
CD
DA

T
B

Slika 210
S obzirom da je DA1 O DAB i COB2 CAB, bice (sl.210)
A1 O
OD
=
AB
BD

OC
OB2
=
.
AB
AC
Sabiranjem odgovarajucih strana ovih jednakosti nalazimo da je
OD
OC
A1 B2
=
+
.
AB
BD
AC
Analognim postupkom nalazimo i da je
AO
OD
B1 C2
=
+
BC
AC
BD
C1 D2
BO
A
=
+
CD
BD AC
OC
OB
D1 A2
=
+
.
DA
AC
BD
Sabiranjem odgovarajucih strana izvedenih jednakosti dobijamo da je

A1 B2 B1 C2 C1 D2 D1 A2
AO + OC
BO + OD
AC
BD
+
+
+
= 2
+2
= 2
+2
= 4.
AB
BC
CD
DA
AC
BD
AC
BD
211. Ako je D proizvoljna tacka stranice BC trougla ABC, a E i F tacke
stranica AC i AB takve da je AB k DE i AC k DF , dokazati da je
ED ED
+
= 1.
AB
AC

Upustvo.

163

Slika 169
S obzirom da je EDC ABC i F BD ABC, imamo da je
DC
ED
=
AB
BC
i

FD
BD
=
.
AC
BC

Otuda je

BD + DC
BC
ED F D
+
=
=
= 1.
AB
AC
BC
BC

(sl.169)
212. Ako su A , B , C tacke u kojima uporedne prave kroz temena A, B,
C trougla ABC seku prave BC, CA, AB, dokazati da je
1
1
1
+
+
= 0.

AA
BB
CC

Upustvo.

Slika 170

164

S obzirom da je CAA CB B i BAA BC C, imamo da je AA :


BB = A C : BC i A A : CC = BA : BC. Otuda je A A : BB + A A : CC =
(BA + A C) : BC = 1,i prema tome
1
1
1
+
+
= 0.(sl.170)
AA BB
CC
213. Ako su A , B , C sredista stranica BC, CA, AB trougla ABC; P , Q,
R tacke u kojima proizvoljna prava s sece prave BC, CA, AB i P , Q , R tacke
u kojima ta ista prava sece prave B C , C A , A B , dokazati da je
1
1
1
+
+
= 0.

PP
QQ
RR

Upustvo.
Ako obelezimo sa A, B, C tacke u kojima prave kroz A ,B , C uporedne
sa s seku prave B C , C A , A B , prema prethodnom zadatku , bice
1
1
1
+
+
= 0.

PP
QQ
RR
S obzirom da je A A = P P , B B = QQ i C C = RR , bice i
1
1
1
+
+
= 0.
PP
QQ
RR
214. Ako krug upisan u trougao ABC sece tezisnu liniju AA1 u tackama M
i N takvim da je AM = M N = N A1 , dokazati da je pri AC > AB,
AB : BC : CA = 5 : 10 : 13.

Upustvo.

Q
M
R
N
B

165

Slika 171
Obelezimo sa P , Q, R tacke u kojima upisani krug dodiruje srtranice BC, CA,
AB .Stavimo li da je AM = M N = N A1 = x, bice AR2 = AM AN = 2x2 i
A1 P 2 = A1 M A1 N = 2x2 , pa je AR = A1 F .No AB = AR + RB, A1 B + A1 P +
P B i RB = P B, pa je AB = A1 B, i prema tome
ab =

1
bc.
2

(1)

koristeci neke od pomenutih jednakosti nalazimo da je

ac = aq + qc = ar + pa1 + a1 c = 2ar + ab = 2x 2 + ab.

(2)

Zamenom dobijenih jednakosti (1) i (2) u relaciji 4AA21 = 2(AB 2 + AC 2 )


BC 2

nalazimo da je 36x2 = 2[AB 2 + (2x 2 + AB)2 ] 4AB 2 . Otuda je x = 2 5 2 AB,


i pri tome AC = 13
5 AB. (sl. 171).
215. Ako je prava koja sadrzi visinu AD trougla ABC dirka kruga opisanog
oko trougla, dokazati da je razlika unutrasnjih uglova B i C prav ugao.

Upustvo.

Slika 172
S obzirom da prava AD dodiruje krug opisan oko trougla ABC, tacka D prave
BC je izvan toga kruga , dakle iza B u odnosu na C ili iza C u odnosu na B
.Neka je npr. tacka D iza B u odnosu na C. U tom slucaju je ugao DAB
odreden tagentom AD i tetivom AB jednak periferijskom uglu ACB , pa je
B C = ABC DAB = ADB(sl. 172).
216. Ako je razlika unutrasnjih uglova B i C trougla ABC prav ugao, dokazati da je prava koja sadrzi visinu AD dirka kruga opisanog oko trougla ABC.

Upustvo.

166

Ako je B C, a BE poluprava u uglu B takva da je ABE = C


, bice CBE prav , pa je BEBC, i prema tome BEkAD.Stoga je ugao
BAD = ABE = C , pa je prava AD dirka kruga opisnog oko trougla ABC.
217. Ako je prava koja sadrzi visinu AD trougla ABC dirka kruga opisanog
oko tog trougla, dokazati da je
AD2 = BD CD.

Upustvo.
Prema poznatom stavu , imamo da je BAD = ACD , pa su pravougli
trouglovi ABD i V AD slicni. Stoga je AD : CD = BD : AD , i prema tome
AD2 = BD CD.
218. Ako je podnozje D visine iz temena A na produzenju stranice BC trougla ABC i pri tome AD2 = BD CD, dokazati da je prava AD dirka kruga
opisanog oko trougla ABC.

Upustvo.
Iz AD2 = BD CD sledi da je AD : CD = BD : AD , pa su pravougli
trouglovi ABD i V AD slicni .Stoga su uglovi DAB i CAD tj. uglovi DAB i
ACB jednaki , pa je prava AD dirka kruga opisanog oko trougla ABC.
219. Neka simetrala ugla A sece stranicu BC trougla ABC u tacki E,
dokazati da je ugao A tog trougla prav ako i samo ako je
r
1
1
1
1
+
.
+
=
AB
AC
BE 2
CE 2

Upustvo.
Datu relaciju mozemo napisati u obliku

BE 2 + CE 2
AB + AC
=
AB AC
BE EC

(3)

s obzirom da je ab : ac = be : ec , bice i

BE + EC
bc
AB + AC
=
=
AC
EC
ec
deljenjem odgovarajucih strana relacijia (1) i (2) nalazimo da je

1
BE 2 + CE
=
AB
BC BE
167

(4)

, pa je otuda

BC 2
CE 2
AC 2
=1+
=1+
.
2
2
AB
BE
AB 2

Stoga je AB 2 +AC 2 = BC 2 , pa je uugao A trougla ABC prav.Obratno tvrdenje


neka citalac izvede sam.
220. Ako je kod trougla ABC zbir ili razlika unutrasnjih uglova B i C prav
ugao i D podnozje visine iz temena A, dokazati da je
1
1
1
+
=
.
AB 2
AC 2
AD2

Upustvo.
U oba slucaja, trouglovi ABD i CAD su slicni ,pa je AD : AB = CD : AC.
Otuda je AD2 : AB 2 = CD2 : AC 2 , tj. AD2 : AB 2 = (AC 2 AD2 ) : AC 2 i
prema tome
1
1
1
+
=
.
AB 2
AC 2
AD2

221. Ako je D podnozje visine iz temena A trougla ABC i pri tome


1
1
1
+
=
,
AB 2
AC 2
AD2
dokazati da je zbir ili razlika unutrasnjih uglova B i C tog trougla prav ugao.

Upustvo.
Iz

1
1
1
+
=
AB 2
AC 2
AD2

nalazimo da je AD2 : AB 2 = (AC 2 AD2 ) : AC 2 tj. da je AD2 : AB 2 = CD2 :


AC 2 . Otuda je AD : AB = CD : AC. Iz ove proporcije sleduje da su pravougli
trouglovi ABD i CAD slicni , pa je BAD = ACD, i prema tome , zbir ili
razlika unutrasnlih uglova B i C prav ugao.
222. Ako je kod trougla ABC zbir ili razlika unutrasnjih uglova B i C prav
ugao i ako je D podnozje visine iz temena A, dokazati da je
(a)
AD2 = BD CD
(b)
AB 2 : AC 2 = BD : CD

168

Upustvo.
(a) U oba slucaja ,trouglovi ABD i CAD su slicni , pa je AD : BD = CD :
AD i prema tome AD2 = BD CD. (b)U oba slucaja , iz pravouglih trouglova
ABD i ACD imamo da je AB 2 = AD2 + BD2 i AC 2 = AD2 + CD2 .Otuda,
koristeci (a) nalazimo da je
AB 2
AD2 + BD2
BD CD + BD2
BD(BD + CD)
BD
=
=
=
=
.
2
2
2
2
AC
AD + CD
BD CD + CD
CD(BD + CD)
CD
223. Ako je kod trougla ABC zbir ili razlika unutrasnjih uglova B i C prav
ugao i ako je r poluprecnik kruga opisanog oko trougla ABC, dokazati da je
AB 2 + AC 2 = 4 r2 .

Upustvo.

Slika 173
Ako je zbir unutrasnjih uglova B i C prav ugao , ugao A trougla ABC je prav ,
pa je AB 2 + AC 2 = 4r2 .U tom slucaju , hipotenuza BC je precnik kruga opisanog oko trougla ABC pa je AB 2 + AC 2 = 4r2 .Pretpostavimo da je kod trougla
ABC razllika unutrasnjih uglova B i C prav ugao.Ako obelezimo sa D podnozje
visine iz temena A i sa E tacku koja se nalazi iza D u odnosu na B takvu
da je BD = DE, bice ugao A trougla ABC prav, pa je AE 2 + AC 2 = CE 2 i
prema tome AB 2 = AC 2 = CE 2 .Neka je S srediste duzi CE , tj. srediste kruga
opisanog oko trougla AEC , a O srediste kruga opisanog oko trougla ABC.
Jednakokraki trouglovi OAB i SAE imaju jednake osnovice AB i AE i jednake
uglove kod temena O i S , pa su medusobom podudarni.Otuda je OA = SE,
pa je CE = 2OA = 2r, i prema tome AB 2 + AC 2 = 4r2 (sl. 173).

169

Napomena.Ovaj stav predstavlja jedan vid uopstenja Pitagorine teoreme.


224. Ako je ugao A trougla ABC prav i T teziste tog trougla, dokazati da
je
BT 2 + CT 2 = AT 2 .

Upustvo.
Ako su M i N podnozja upravnih kroz T na stranicama AB i AC , bice
BT 2 = BM 2 + M T 2 i CT 2 = CN 2 + N T 2 , pa je
BT 2 + CT 2 = BM 2 + CN 2 + M T 2 + N T 2 .
S obzirom da je BM = 2AM = N T i CN = 2AN = 2M T , bice
BT 2 + CT 2 = 5AT 2 .

225. Ako su E i F tacke u kojima simetrale unutrasnjih uglova B i C seku


naspramne stranice trougla ABC, a P, Q, R podnozja upravnih iz proizvoljne
tacke M duzi EF na stranicama BC, CA, AB, dokazati da je
M P = M Q + M R.

Upustvo.

A
F
E
R
F

Q
M
H

E
G

B F

Slika 174
Ako obelezimo sa E i E podnozja upravnih iz tacke E na stranicama AB
i BC , a sa F i F podnozja upravnih iz tacke F na strtanicama AC i BC
bice EE = EE = m i F F = F F = n. Neka je zatim M P = x, M Q = y,
M R = z.S obzirom da je EM Q EF F i F M R simF EE , imamo
170

da je (sl. 174) ny = EM
EF i
jednakosti nalazimo da je

z
m

MF
EF

.Sabiranjem odgovarajucih strana ovih

z
EM + M F
y
+
=
=1
n m
EF
,tj. da je
my + nz = mn

(5)

ako zatim obelezimo sa g podnozje upravne iz tacke F na pravoj EE i sa H


tacku u kojoj se seku prave M P i F G, bice F M H F EG , pa je
xn
MF
z
=
=
mn
EF
m
i pri tome
m(x n) = z(m n)

(6)

iz jednakosti (1) i (2) sledi da je


x = y + z.

226. Ako su M i N tacke u kojima prava kroz presek O dijagonala AC i


BD trapeza ABCD uporedna s osnovicama AB i CD sece prave AD i BC,
dokazati da je tacka O srediste duzi M N .

D
M

C
N

Slika 175
Slika 175 Iz slicnih ova AOM i ACD imamo da je M O : DC = AM : AD,
a iz slicnih trouglova BCN i BDC imamo da je CN : DC = BN : BC.No
AM : AD = BN : BC, pa je i M O : DC = ON : DC.Otuda je M O = ON , pa
je tacka O srediste duzi M N (sl. 175)
227. Ako je ABCD trapez sa nejednakim kracima AD i BC, dokazati da
je
AB + CD
AC 2 BD2
=
.
2
2
AD BC
AB CD

171

Slika 176
Ako obelezimo sa E i F podnozja upravnih iz temena C i D na stranici AB
,tada iz trouglova ABC, ABD, BCD dobijamo da je (sl. 176)
AC 2 = AB 2 + BC 2 2AB EB

(7)

bd2 = ab2 + ad2 2ab af

(8)

bd2 = bc2 + cd2 + 2cdoeb

(9)

iz jednakosti (1) i (3) nalazimo da je


AC 2 BD2 = AB 2 CD2 2EB(AB + CD) = (AB + CD)(AB CD 2EB)
a iz nejednakosti (2) i (3)nalazimo da je
AD2 BC 2 = CD2 AB 2 + 2ABoAF + 2CDoEB =
CD2 AB 2 + 2AB(AB CD EB) + 2CDoEB =

CD2 AB 2 + 2AB 2 2AB CD 2AB EB + 2CDoEB =


AB 2 2AB CD + CD2 2EB(AB CD)

= (AB CD)2 2EB(AB CD) = (AB CD)(AB CD 2EB)


Otuda je

(AB + CD)(AB CD 2EB)


AB + CD
AC 2 BD2
=
=
AD2 BC 2
(AB CD)(AB CD 2EB)
AB CD

228. Ako su P i Q tacke na kracima AD i BC trapeza ABCD takve da je


AP : P D = BQ : QC = m : n, dokazati da je
PQ =

n AB + m DC
.
m+n

172

C
R

Slika 177
Ako sa R obelezimo tacku u kojoj se seku prave P Q i AC , bice
AP R ADC
i
CRQ CAB
pa je
P R : DC = AP : AD = m : (m + n)
i
RQ : AB = CQ : CB = n : (m + n)
tj. da je
PR =

mDC
m+n

nAB
.
m+n
S obzirom da su duzi orjentisane , bice P Q = P R + RQ, i prema tome
RQ =

PQ =

nAB + nDC
(sl.177).
m+n

229. Ako je T teziste trougla ABC i s proizvoljna prava u ravni tog trougla,
a T , A , B , C uporedne projekcije tacaka T, A, B, C na pravoj s, dokazati da
je
1
T T = (AA + BB + CC ).
3

173

A
T

C
D

Slika 178
Ako obelezimo sa D srediste stranice BC trougla ABC , a sa D uporednu
projekciju tacke D na pravoj s ,prema predhodnom zadatku , imamo da je
TT =

1
(AA + DD ).
3

No
DD =
,pa je
TT =

1
(BB + CC )
3

1
(AA + BB + CC )(sl.168).
3

230. Dva kruga k1 i k2 s poluprecnicima r1 i r2 dodiruju se spolja u tacki


P . Ako je Q podnozje upravne iz tacke P na bilo kojoj spoljasnjoj dirki tih
krugova, dokazati da je
2r1 r2
.
PQ =
r1 + r2

A1

O1

A2

O2

Slika 179
Ako su O1 i O2 sredista krugova k1 i k2 , a A1 i A2 dodirne tacke spoljasnje dirke
tih krugova na kojoj sa nalazi tacka Q , bice cetvorougao O1 A1 A2 O2 trapez , a

174

P i Q tacke na njegovim kracima O1 O2 i A1 A2 takve da je O1 P : P O2 = A1 Q :


QA2 = r1 : r2 , pa je prema zadatku 229 (sl. 179)
PQ =

2r1 r2
r2 O1 A2 + r1 O2 A2
=
.
r1 + r2
r1 + r2

231. Ako su M i N tacke stranica AB i CD, a P i Q tacke stranica AD


i BC cetvorougla ABCD takve da je AM : M B = DN : N C = m : n i
AP : P D = BQ : QC = p : q, dokazati da se duzi M N i P Q seku u tacki O
takvoj da je M O : OM = p : q i P O : OQ = m : n.
C
N
H
D

G
F

E
Q

O
P

Slika 180
Obelezimo sa E tacku takvu da je cetvorougao ABCD paralelogram , sa F i
G tacke na duzima DE i CE takve da je M F kAD i QGkBE , a sa H tacku u
kojoj se seku duzi DG i N F .Pri tome je
DN : N C = AM : M B = DF : F E
, pa je CEkN F .Iz jednakosti
AD : P D = BC : QC = BE : QG
i AD = BE sledi da je P D = QG.No duzi P D i QG su istomerne , pa je
cetvorougao P QGD paralelogram.Ako je O tacka stranice P Q tog paralelograma takva da je HO kQG , imamo da je
BE : QG = EC : GC = F N : HN = M F : O H

.Stoga je tacka O i na na duzi M N , te su tacke O i O istovetne.Na taj nacin


nalazimo da je
M O : ON = F H : HN = EG : GC = BQ : QC = p : q
i
P O : OQ = DH : HG = DN : N C = m : n
(sl. 180) 232. Ako je kod trougla ABC stranica BC jednaka poluzbiru drugih
dveju stranica, dokazati,
(a) da teme A, sredista M i N stranica AB i AC, srediste O opisanog kruga
i srediste S upisanog kruga pripadaju jednom krugu k;
(b) da je simetrala AS ugla A upravna na pravoj OS;
(v) da dirka kruga k u tacki S sadrzi teziste T trougla ABC.
175

N
S

Slika 181
Obelezimo sa L tacku u kojoj simetrala AS ugla A sece stranicu BC.Iz
BC =

1
(AB + AC)
2

i
BL : LC = AB : AC
sledi da je
BL =

1
AB = BM
2

CL =

1
AC = CN
2

i
Pri tome je
i

SM B
= SLB
SLC
= SN C

pa je SM = SL i SL = SN , pa je SM = SN .Ako simetrala AS ugla A sece krug


k opisan oko trougla AM N u nekoj tacki S , bice S M = S N .Otuda sleduje
da su obe tacke S i S na simetrali duzi M N . Pri AB = AC trougao ABC je
jednakostranican, u kom je slucaju teorema jednostavna.Kad stranice AB i AC
nisu jednake, simetrala AL ugla A nije upravna na pravoj M N , prema tome,
na njoj postoji samo jedna tacka jednako udaljena od tacaka M i N . Stoga se
tacke S i S poklapaju, pa je S tacka kruga k. S obzirom da je tacka A srediste
slicnosti kruga k i kruga opisanog oko trougla ABC s koeficijentom 1 : 2, duz
OA je precnik kruga k i prema tome tacka O na krugu k.Dakle, tacke A, M ,
N , O, S pripadaju jednom krugu (sl. 181).
(b)S obzirom da je duz OA precnik kruga k , a tacka S na krugu k, ugao ASO
je prav , pa je simetrala AL ugla A upravna na pravoj OS.
(v) Kako je tacka S srediste luka M N kruga k , dirka kruga k u tacki S uporedna
je s pravom M N , dakle na pravom BC. Otuda i iz AS : SL = (AB + BC) :
BC = 2 : 1 sledi da pomenuta dirka sadrzi teziste Z trougla ABC.
233. Ako je kod trougla ABC stranica BC jednaka poluzbiru drugih dveju
stranica, dokazati da je

176

(a)
ha = 3 = a
(b)
a2 = 4(2r )
M
A

S
C
B

A P E A1P a

Sa

Slika 182
(a) Koristeci uobicajene oznake iz slicnih trouglova AA E i SP E imamo da je
AA SP = AS : SE. No, AE : SE = (a + b + c) : a = 3a : a = 3 : 1, pa je AA :
SP = 3 : 1, i prema tome ha = 3.Isto tako , iz slicnih trouglova AA E i Sa Pa E
imamo da je AA : Sa Pa = AE : Sa E.No AE : Sa E = (b + c a) : a = 1,pa je
AA : Sa Pa = 1 i prema tome ha = a (sl. 182)
(b) Kod pravouglog trougla BA1 N imamo da je
BA21 = BN 2 A1 N 2
No

1
a
2
A1 N =

BA21 =

BN 2 = M N A1 N = 2r
pa je

a
( )2 = 2r 2
2

i prema tome
a2 = 4(2r )
(sl. 182)
234. Neka je AD visina koja odgovara hipotenuzi BC pravouglog trougla ABC, DE visina trougla ABD i DF visina trougla ACD. Ako je BC =
a, AD = ha , BE = m, CF = n dokazati da je
(a)
h3a = amn;
177

(b)
a2 = m2 + n2 + 3h2a ;
(v)

3
3
3
a 2 = m2 + n 2 .

E
F

Slika 183
(a) Iz pravouglih trouglova ABC , ABD, ACD nalazimo da je
AD2 = BD CD
BD2 = AB EB
CD2 = AC F C

pa je
AD4 = BD2 CD2 = AB AC EB F C
Otuda je
AD3 = BC EB F C
a prema tome
h3a = amn
(sl. 183)
(b)S obzirom da je
BC 2 = (BD+DC)2 = BD2 +DC 2 +2BDDC = (BE 2 +DE 2 )+(CF 2 +DF 2 )+2AD2 = BE 2 +CF 2 +3AD2
bice
a2 = m2 + n2 + 3h2a
(v)Iz jednakosti
BD : BC = EB : AB
i
BD2 = AB BE
sledi da je
BD3 = BC BE 2
a iz jednakosti
DC : BC = F C : AC
178

i
DC 2 = F C AC
da je
CD3 = BC F C 2
Stoga je
BC = BD + DC =
i prema tome

3
3
BC BE 2 + BC F C 2

3
3
3
a 2 = m2 + n 2 .

235. Ako je O srediste duzi AB, a C i D tacke prave AB takve da je


OC : OD = k, dokazati da je
AC 2 BC 2 = k(AD2 BD2 ).
S obzirom da je
AC 2 BC 2 = (AC BC)(AC + BC) = 2AB OC = 2kAB OD
i
AD2 BD2 = (AD BD)(AD + BD) = 2AB OD
bice
AC 2 BC 2 = k(AD2 BD2 ).
236. Ako su AB i CD dve tetive kruga k koje se seku u tacki S pod pravim
uglom i ako su O i r srediste i poluprecnik kruga k, dokazati da je
AB 2 + CD2 = 4(2r2 OS 2 ).
D

O
A

N
S

Slika 184
Ako sa M i N obelezimo srediste tetive AB i CD, bice
AM 2 = OA2 OM 2
i
CN 2 = OC 2 ON 2
179

Sabiranjem odgovarajucih strana ovih jednakosti dobijamo da je


AM 2 + CN 2 = 2r2 (CM 2 + ON 2 )
tj. da je
AB 2 + CD2 = 4(2r2 OS 2 )
(sl. 184) 237. Ako su AB i CD dve tetive kruga k(O, r) koje se seku u nekoj
tacki S pod pravim uglom, dokazati da je
AS 2 + BS 2 + CS 2 + DS 2 = 4r2 .
C
k

O
B
A

Slika 185
Iz pravouglih trouglova ASC i BSD sledi da je
AS 2 + CS 2 = AC 2
i
BS 2 + DS 2 = BD2
pa je
AS 2 + BS 2 + CS 2 + DS 2 = AC 2 + BD2 .
Ako je CE precnik kruga k, bice ugao CAE, prav, pa je
CAB + BAE = CDB + ABD
S obzirom da je
CAB = CDB
bice i
BAE = ABD
pa je BD = AE.Otuda je
AS 2 + BS 2 + CS 2 + DS 2 = AC 2 + AE 2 = CE 2 = 4r2 .(sl.185).
238. Neka je ugao A trougla ABC prav i D podnozje visine iz temena A.
Ako su r, r1 , r2 poluprecnici opisanih i , 1 , 2 poluprecnici upisanih krugova
trouglova ABC, ABD, ACD dokazati da je
(a)
r2 = r12 + r22 ;
180

(b)
2 = 21 + 22 .
(a) S obzirom da je ugao A trougla ABC prav , bice BC 2 = AB 2 +AC 2 .Duzi
BC, AB, AE su precnici krugova opisanih oko pravouglih trouglova ABC, ABD
i ACD pa je (2r)2 = (2r1 )2 + (2r2 )2 i prema tome
r2 = r12 + r22 .
(b) S obzirom da je
ABC DBA DAC
bice
a : c : b = : 1 : 2
Otuda i iz
a2 = b 2 + c2
sledi da je
2 == 21 + 22 .
239. Ako se krugovi k1 (O1 , r1 ) i k2 (O2 , r2 ) dodiruju spolja u tacki S, a prave p1
i p2 seku u tacki S pod pravim uglom, pri cemu prava p1 sece k1 i k2 u tackama
A i B, a prava p2 sece k1 i k2 u tackama C i D, dokazati da je
AB 2 + CD2 = 4(r1 + r2 )2 .
Ako je P tacka u kojoj se seku prave kroz O1 i O2 upravne na pravama AB i
CD , bice O2 P 2 + O1 P 2 = O1 O22 .Otuda je 41 AB 2 + 41 CD2 = (r1 + r2 )2 i prema
tome
AB 2 + CD2 = 4(r1 + r2 )2 (sl.158)

240. Ako su r1 i r2 poluprecnici dvaju krugova k1 i k2 koji se spolja dodiruju


u tacki P i ako je d odstojanje tacke P od jedne njihove spoljasnje zajednicke
dirke, dokazati da je
1
1
2
+
= .
r1
r2
d
Obelezimo sa O1 i O2 sredista krugova k1 i k2 , sa A1 i A2 tacke u kojima jedna
njihova zajednicka spoljasnja dirka dodiruje krugove k1 i k2 , sa B1 i B2 tacke u
kojima prava kroz tacku P upopredna s pravom A1 A2 sece prave O1 A1 i O2 A2
. Iz sliccnosti trouglova P O1 B1 i P O2 B2 sledi da je O1 P : P O2 = O1 B1 : O2 B2
pa je pri r1 r2 r1 : r2 = (r1 d)(d r2 ).Stoga je d(r+ r2 ) = 2r1 r2 .Deljenjem
obeju strana ove jednakosti sa r1 r2 d nalazimo da je
1
2
1
+
= (sl.159).
r1
r2
d

181

241. Ako su r1 i r2 poluprecnici dvaju krugova k1 i k2 koji se medu sobom


spolja dodiruju i ako su T1 i T2 dodirne tacke jedne njihove spoljasnje dirke,
dokazati da je

T1 T2 = 2 r1 r2 .

Uputstvo.

T1
T2
K
O2

O1

Sl.186

Slika 186
Neka je r1 > r2 .Obelezimo sa O1 i O2 sredista krugova k1 i k2 i sa K tacku
duzi O1 T1 takvu da je
KT1 = O2 T2 .U trouglu KO1 O2 ugao K je prav,
O1 O2 = r1 + r2
O1 K = r1 r2
a
KO2 = T1 T2
pa je
T1 T22 = (r1 + r2 )2 (r1 r2 )2
i prema tome

T1 T2 = 2 r1 r2 (sl.186).

242. Neka su k, k1 , k2 tri kruga upisana u ugao , pri cemu krugovi k1 i


k2 dodiruju krug k, zatim l1 i l2 krugovi koji dodiruju jedan krug ugla a pri
tome krug l1 dodiruje krugove k i k1 , a krug l2 dodiruje krugove k i k2 . Ako su
r, 1 , 2 poluprecnici krugova k, l1 , l2 dokazati da je

1 + 2 = r.

182

243. Ako su BB i CC visine iz temena B i C trougla ABC, a r poluprecnik


opisanog kruga i d odstojanje njegovog sredista od stranice BC, dokazati da je
BC : B C = r : d.

Uputstvo.
Ako je H ortocentar trougla ABC, bice tacke A, B, H, Cna jednom krugu
kome je duz AH precnik. Iz slicnosti trouglova ABC i ABC sledi da je BC :
B C = 2r : AH = r : d.
244. Ako je H ortocentar trougla ABC i O srediste kruga opisanog oko tog
trougla, dokazati da je
BC 2 + AH 2 = 4OA2 .

Uputstvo.
Ako obelezimo sa D srediste stranice BC trougla ABC, bice duz OB hipotenuza pravouglog trougla OBD, pa je BD2 + OD2 = OB 2 . Pri tome je
1
1
BD = BC, OD = AH, i OB = OA, pa je BC 2 + AH 2 = 4OA2
2
2
245. Ako je H ortocentar trougla ABC, r poluprecnik kruga opisanog oko
tog trougla i a, b, c duzi jednake stranicama BC, CA, AB, dokazati da je
AH 2 + BH 2 + CH 2 = 12r2 (a2 + b2 + c2 ).

Uputstvo.
Prema prethodnom zadatku, bice
AH 2 = 4r2 a2 , BH 2 = 4r2 b2 , OH = 4r2 c2
pa je
AH 2 + BH 2 + CH 2 = 12r2 (a2 + b2 + c2 )
246. Ako su AA , BB , CC visine i H ortocentar trougla ABC, dokazati
da je
(a)
1
AH AA = (AB 2 + AC 2 + BC 2 )
2
(b)
AH AA + BH BB + CC =
183

1
(AB 2 + BC 2 + CA2 )
2

Uputstvo.
(a) S obzirom da su cetvorouglovi ABHC, BCHA, CAHB tetivni, imamo
da je
AHAA = ABAC = AB(AB+BC ) = AB 2 +ABBC = AB 2 +CBBA = AB 2 +CB(BC+CA ) = AB 2 BC
. Otuda je
1
(AB 2 + AC 2 BC 2 )
2
(b)Prema izvedenom delu ovog zadatka, imamo da je
AH AA =

AH AA =

1
(AB 2 + AC 2 BC 2 )
2

1
(AB 2 + BC 2 CA2 )
2
1
CH CC = (BC 2 + AC 2 AB)
2

BH BB =

pa je
AH AA + BH BB + CH CC =

1
(AB 2 + BC 2 + CA2 )
2

.
247. Ako je M srediste kvadrata nad hipotenuzom BC trougla ABC koji
se nalazi s one strane od prave BC s koje nije teme A, sa N srediste kvadrata
nad hipotenuzom BC koji se nalazi s one strane od prave BC sa koje je teme
A, dokazati da
je
2
(AB + AC)
(a) AM =
2
2
(b) AN =
(AB AC)
2
Uputstvo.
(a) S obzirom da je stranica BC hipotenuza pravouglog trougla ABC, duz
BC je precnik kruga l opisanog oko trougla ABC.Ugao BMC je prav, pa je i
tacka M na krugu l. Kako je tacka na krugu l, a s one strane od prave BC s
koje nije teme A, pri cemu je BM=CM, a tacka M je srediste luka BC kruga l
na kome nije tacka A, pa je prava AM simetrala ugla A trougla ABC. Ako je M
podnozje upravne iz tacke M na pravoj AB, bice kod pravouglog trougla AMM
1
katete AM i MM jednake, a prema zadatku 154, AM = (AB + AC), te je
2

2
(Sl.188) AM =
(AB + AC)
2

184

A1
B

Slika 188
Slucaj pod (b) dokazuje se analognim postupkom.
248. Ako su P , Q, R tacke u kojima upisani krug dodiruje stranice BC,
CA, AB trougla ABC, a A , B , C podnozja upravnih iz proizvoljne tacke M
kruga k na pravama BC, CA, AB i P , Q , R podnozja upravnih iz tacke M
na pravama QR, RP , P Q, dokazati da je
M A M B M C = M P M Q M R .

Uputstvo.
Prema zadatku 195, imamo da je
M P 2 = M B M C
M Q2 = M C M A

M R2 = M A M B
pa je

M A M B M C = M P M Q M R
(Sl.189)

185

A
B
Q

C M

R P

Q
R
S

A P

Slika 189
249. Ako su , a , b , c poluprecnici upisanih krugova k, ka , kb , kc zatim
P , Pa , Pb , Pc tacke u kojima ti krugovi dodiruju pravu BC i P , Pa , Pb , Pc
tacke u kojima prave AP , APa , APb , APc seku krugove k, ka , kb , kc , dokazati
da je
(a) AP P P = 2ha
(b) APa Pa Pa = 2a ha
(c) APb Pb Pb = 2b ha
(d) APc Pc Pc = 2c ha
Uputstvo.
Visina trougla ABC razlicita je od duzi AP ili se poklapa s njom. Ako je
razlicita, obelezimo sa P tacku kruga k dijametralno suprotnu sa P. Iz slicnih
trouglova ADP i PPP imamo da je AD : AP = P P : P P . Otuda je
AP P P = AD P P i prema tome
AP P P = 2 ha .Ako se visina poklapa sa duzi AP, duz PP je precnik kruga
k, pa je takode
AP P P = 2 ha
(Sl.190)

186

P P

P A1

Slika 190
250. Ako su S, Sa , Sb , Sc sredista i , a , b , c poluprecnici upisanih krugova
trougla ABC, M i N tacke u kojima simetrale spoljasnjeg i unutrasnjeg ugla A
seku opisani krug trougla ABC, kome je poluprecnik r, dokazati da je
(a) SA SN = 2r
(b) Sa A Sa N = 2ra
(c) Sb A Sb M = 2rb
(d) Sc A Sc M = 2rc
Uputstvo.
Ako je R tacka u kojoj krug k dodiruje stranicu AB, bice
ARS M BN
pa je SA : M N = RS : BN . Otuda je
SA BN = M N RS
S obzirom da je BN = SN , M N = 2r, RS = , bice
SA SH = 2r
Ostala tri dela ovog zadatka dokazuju se na analogan nacin (Sl.191)

187

M
A

O
R

C
N

Slika 191
251. Ako su a, b, c stranice trougla, r poluprecnik opisanog kruga i ha visina
koja odgovara stranici a, dokazati da je
bc = 2rha .

Uputstvo.
Neka su kod trougla ABC stranice BC,CA,AB duzine a,b,c.Ako sa D obelezimo
podnozje visine iz temena A i sa AE precnik opisanog kruga, bice
ABD AEC
pa je AB : AE = AD : AC. Otuda je
AB AC = AD AE
i prema tome
bc = 2r ha
(Sl.192)

188

C
D
E

Slika 192
252. Ako su r i ha poluprecnik opisanog kruga i visina iz temena A trougla
ABC, la i
la simetrale unutrasnjeg i spoljasnjeg ugla A, a N i M tacke u kojima
te simetrale seku opisani krug, dokazati da je
2rha
(a) AN =
la
2rha
(b) AM =
la
Uputstvo.
(a)
Ako obelezimo sa D podnozje visine iz temena A i sa E tacku u kojoj
simetrala unutrasnjeg ugla A sece stranicu BC trougla ABC,
M AN EDA
pa je AN : M N = AD : DE i prema tome
AN =

2rha
M N AD
=
DE
la

(Sl.193)

189

M
A

D
N

Slika 193
(b)
Ako obelezimo sa D podnozje visine iz temena A i sa F tacku u kojoj
simetrala spoljasnjeg ugla A sece pravu odredenu stranicom BC trougla ABC,
bice
M AN ADF
pa je AM : M N = AD : AF i prema tome
AM =

2rha
M N AD
=
AF
la

(Sl.193)
253. Ako su b i c duzi jednake stranicama AC i AB trougla ABC, la i la
simetrale unutrasnjeg i spoljasnjeg ugla A, a N i M tacke u kojima te simetrale
seku opisani krug, dokazati da je
bc
(a) AN = ;
la
bc
(b) AM = .
la
Uputstvo.

190

(a) Prema zadacima 252 i 251 imamo da je


AN =

2rha
la

i
bc = 2rha
pa je
AM =

bc
la

(b) Prema zadacima 252 i 251 imamo da je


AM =

2rha
la

i
bc = 2rha
pa je
AN =

bc
la

254. Ako je ha visina iz temena A, la simetrala ugla A i r poluprecnik


opisanog kruga trougla ABC, dokazati da je
s
la2
m2a h2a
r=

.
2ha
la2 h2a
Uputstvo.
Ako obelezimo sa D podnozje visine iz temena A, sa E tacku u kojoj simetrala
ugla sece stranicu BC, sa A srediste stranice BC, sa H presecnu tacku prave
AB sa opisanim krugom koja je razlicita od tacke A i sa K tacku opisanog kruga
dijametralno suprotnu s temenom trougla ABC, bice AN:AE=DA:DE. No
AE = la
q
DA = ma 2 ha 2
q
DE = la 2 ha 2

a kod slicnih trouglova ANK i ADE AN:AD=AK:AE, tj.


AN =

2rha
la

pa je (Sl.194)
la 2
r=
2ha

m2a h2a
la2 h2a

191

E A

D
K
N

Slika 194
255. Ako obelezimo sa b i c stranice naspram temena B i C trougla ABC, sa
k(S, ) upisani krug, sa ka (Sa , a ) kb (Sb , b ), kc (Sc , c ) spolja upisane krugove
koji odgovaraju, respektivno, temenima A, B, C a sa d, da , db , dc odstojanja
tacaka S, Sa , Sb , Sc od prave koja sadrzi tezisnu liniju AA1 = ma , dokazati da
je
(b c)
(a) d =
2ma
(b + c) b
(b) db =
2ma
(b c) a
(c) da =
2ma
(b + c) c
(d) dc =
2ma
Uputstvo.
(a) Obelezimo sa P i Pa tacke u kojima krugovi k i ka dodiruju stranice
BC. Prema zadatku 176 imamo da je APa k SAl , te trougloviASAl i Pa SAl
sa zajednickom osnovicom SAl imaju visine AA i P Pa . Otuda je SAl AA =
SAl Pa Pa . Ako zatim obelezimo sa S podnozje normale iz tacke S na pravoj
AAl , bice
SAl AA = AAl SS
i
SAl Pa Pa = Al SP
192

S toga je
AAl SS = Al Pa SP
i prema tome (Sl.195)
d=

(b a)
Al Pa SP
=
AAl
2ma

(b) Kao u prethohodnom slucaju, obelezimo sa P i Pa tacke u kojima krugovi


k i ka dodiruju stranicu BC. Prema zadatku 176 imamo da je AP k Al Sa , te
trouglovi ASa Al i P Sa Al sa zajednickom osnovom Sa Al imaju jednake visine
AA i PP. Otuda je Sa Al AA = Sa Al P P . Ako zatim obelezimo sa Sa
podnozje normale iz tacke Sa na pravoj AAl , bice
Sa Al AA = AAl Sa Sa = Sa Al P P = AAl Sa Pa
S toga je
AAl Sa Sa = AAl Sa Pa
i prema tome (Sl.195)
da = Sa Sa =

Al P Sa Pa
(b a)a
=
AAl
2ma

(v) Obelezimo sa Pb i Pc tacke u kojima krugovi kb i kc dodiruju pravu BC.


Prema zadatku 176 imamo da je APc k Al Sb , te trouglovi ASb Al i Pc Sb Al sa
zajednickom stranicom Sb Al imaju jednake visine AA i Sc Sc . Otuda je
Sb Al AA = Sb Al Pc Pc
Ako zatim obelezimo sa Sb podnozje upravne iz tacke Sb na pravoj AA, bice
Sb Al AA = AAl Sb Sb
i
Sb Al Pc Pc = Al Pc Sb Pb
. S toga je
AAl Sb Sb = Al Pc Sb Pb
i prema tome (Sl.195)
db = Sb Sb =

Al Pc Sb Pb
(b + a)b
=
AAl
2ma

(g) Ovaj slucaj je analogan prethodnom

193

Sb

A
Sc

P
S

Pc

A1 P a

Pb

Sa

Slika 195
256. Ako su a, b, c stranice i ha , hb , hc njima odgovarajuce visine nekog trougla, zatim p poluobim, r poluprecnik opisanog kruga i poluprecnik
ab+bc+ca
upisanog kruga, dokazati da je (a) ha + hb + hc =
; (b)
2r
1 1 1
ha + hb + hc = 2 p ( + + ).
a b
c
Resenje: (a) Ako je AD visina iz temena A i AE precnik opisanog kruga
trougla ABC,bice ... pa je AD:AC=AB:AE,tj. ha + b = c : 2r.Otuda je ha =
bc
ca
ab
.Isto tako je hb =
i hc =
. Sabiranjem odgovarjucih strana dobijenih
2r
2r
2r
triju jednakosti nalazimo da je
1
(ab + bc + ca).
2r
(b) Ako je S srediste upisanog kruga u trougao ABC i P tacka u kojoj
simetrala AS ugla A sece stranicu BC,bice AP:SP.
Otuda je ha : = 2p : a, i prema tome
ha + hb + hc =

ha =

2p
2p
, hc =
.
b
c
194

Sabiranjem odgovarajucih strana dobijenih triju jednakosti nalazimo da je


1 1 1
ha + hb + hc = 2p( + + ).
a b
c
257. Ako su , a , b , c poluprecnici upisanih krugova trougla ABC i ha
2
1
1
2
1
1
=
; (b)
+
=
.
visina iz temena A, dokazati da je (a)
a
ha
b
c
ha
Resenje: (a) Obelezimo sa S i Sa srediste upisanog kruga i spolja upisanog
kruga koji se nalazi u uglu A, sa B tacku B u kojoj prava AS sece pravu BC
i sa S i Sa upravne projekcije tacaka S i Sa na pravoj koja sadrzi visinu AD.
Prema zadatku 175 bice AS : SA = ASa : Sa A, pa je AS : S D = AS D. Stoga
1
2
1
je (ha ) : = (ha + a ) : a i prema tome +
=
.
a
ha
(b) Obelezimo sa Sa i Sc srediste spolja upisanih krugova koji se nalaze
respektivno u uglovima B i C, sa F tacku u kojoj prava Sb Sc sece pravu BC
i sa Sb , Sc upravne projekcije tacaka Sb , Sc na pravoj koja sadrzi visinu AD.
Prema zadatku 175, bice
ASb : Sb F = ASc : Sc F
pa je i
ASb : Sb D = ASc : Sc D
Stoga pri
[ASb D](ha b ) : b = (c ha ) : c
a pri
[ASc D](ha c ) : c = (b ha ) : b
U oba slucaja imamo da je

1
2
1
+
=
b
c
ha

258. Ako su ha , hb , hc visine trougla ABC, a , a , b , c poluprecnici


1
1
1
1
1
1
+ +
= ; (b)
+
+
njegovih upisanih krugova, dokazati da je (a)
a b c

ha hb
1
1
1
1
1
1
= ; (v)
+

= .
hc

ha
hb
hc
c
(a) Koristeci prethodni zadatak, nalazimo da je
1
1
1
1

=0

a
b
c
pa je

1
1
1
1
+
+
=
a
b
c

(b) Prema prethodnom zadatku, imamo da je


2
1
1
=
ha
a
195

1
1
2
=
hb
b
2
1
1
=
hc
c
pa je

1
1
1
1
+
+
=
ha
hb
hc

(c) Prema prethodnom zadatku, imamo da je


2
1
1
=
ha
a
1
1
2
=
hb
b
2
1
1
=
hc
c
pa je

1
1
1
1
+

=
ha
hb
hc
c

259. Ako su a, b, c stranice i p poluobim trougla ABC, a , a , b , c


poluprecnici upisanih krugova tog trougla, dokazati da je (a) a = (p b)
(p c); (b) b c = p (p a).
(a) Ako su S i Sa sredista upisanih krugova koji se nalaze u uglu A a P i Pa
tacke u kojima ti krugovi dodiruju stranicu BC,bice ...pa je SP : CP = CPa :
Sa Pa . Otuda je : (a c) = (p b) : a i prema tome a = (p b)(p c).
(b) AKo su Sb i Sc sredista spolaj upisanih krugova koji se nalaze respektivno
u uglovima B i C,a Pb i Pc tacke u kojima ti krugovi dodiruju pravu BC,bice ...
pa je
Sb Pb : CPb = CPc : Sc Pc .
Otuda je
b : (p a) = p : c
i prema tome
|rhob c = p(p a).
260. Ako su a, b, c stranice i p poluobim trougla ABC, a , a , b , c
poluprecnici upisanih krugova, dokazati da je (a) a (b + c ) = ap ; (b) a +
b 2 + c2 a2
; (d) a b + b c + c a = p2 .
b c = bc ; (c) b c a =
2
Resenje: (a) Ako je A1 srediste stranice BC, M srediste kruznog luka BAC,
Sa srediste spolja upisanog kruga koji se nalazi u uglu A, a Ra tacka u kojoj
taj krug dodiruje pravu AB, bice
M BA1 ASa Ra
196

pa je
ARa : Sa Ra = M A1 : BA1
Otuda je
p : a = (b + c ) : a
i prema tome
a (b + c ) = ap

(b) Prema prethodnom zadatku, bice


a + b c = (p b)(p c) + p(p a) = 2p2 p(a + b + c) + bc = bc

(v) Prema prethodnom zadatku, bice


b c a = p(p a) (p b)(p c) = p(b + c a) bc =

1
(b2 + c2 a2
2

(g) Prema prethodnom zadatku, bice


b c + c a + b a = p(p a) + p(p b) + p(p c) = p2

261. Ako su , a , b , c poluprecnici upisanih krugova trougla ABC i p


poluobim tog trougla, dokazati da je (a) a b c = p2 ; (b) b c = (p a)2 a .
Resenje: (a) Koristeci uobicajne oznake, imamo da je
ARa Sa ARS
pa je
ARa : AR = Ra Sa : RS
tj.
p : (p a) = a :
Otuda je
a =
Prema zadatku 259 imamo da je

p
. . . . . . (1)
pa

b c = p(p a) . . . . . . (2)
Iz (1) i (2) nalazimo da je
a b c = p 2

(b) Iz proporcije
p : (p a) = a :
197

sledi da je
=

pa
a . . . . . . (3)
p

Iz (1) i (3) nalazimo da je


b c = (p a)2 a

262. Ako su a, b, c stranice trougla ABC, a , a , b , c poluprecnici upisanih


krugova i r poluprecnik opisanog kruga, dokazati da je
2 + 2a + 2b + 2c = 16r2 (a2 + b2 + c2 ).
Resenje: Ako je A1 srediste stranice BC i O srediste kruga opisanog oko trougla
ABC, bice
BA21 + OA21 = OB 2
pa je

a2
a 2
+r(
) = r2
4
2

tj.
a2 + (a )2 = 4r(a ) . . . (1)
Isto tako je
b2 + (b )2 = 4r(b ) . . . (2)
i
c2 + (c ) = 4r(c ) . . . (3)
Iz jednakosti (1), (2) i (3) nalazimo da je
a2 + b2 + c2 + (a )2 + (b )2 + (c )2 = 4r(a + b + c 3)
Otuda se, primenom zadatka 154 dobija da je
2 + 2a + 2b + 2c = 16r2 (a2 + b2 + c2 )

263. Ako su a, b, c stranice i p poluobim trougla ABC, a , a , b , c


poluprecnici upisanih krugova i r poluprecnik opisanog kruga, dokazati da je
(a) a2 = (a )(b + c ); (b) a2 + b2 + c2 = 2p2 22 8r; (v) ab + bc + ca =
p2 + 2 + 4r.
Resenje: (a) Ako je A1 srediste stranice BC, M srediste kruznog luka ABC
i N srediste luka BC na kome nije teme A, bice
M BA BN A1
pa je
BA1 : M A1 = N A1 : BA1
tj.
BA1 = N A1 M A1
198

Otuda je

a
a b + c
( )2 =

2
2
2

i prema tome
a2 = (a )(b + c )
(b) Prema izvedenom delu ovog zadatka imamo da je
a2 = (a b )(b + c )
b2 = (b )(c + a

c2 = (c )(a + b )
Pa je
a2 + b2 + c2 = 2(b a + b c + c a ) 2(a + b + c))
Otuda primenom zadataka 260 i 154 nalazimo da je
a2 + b2 + c2 = 2p2 22 8r
(v) Kako je
(a + b + c)2 = a2 + b2 + c2 + 2(ab + ac + bc)
prema prethodnom delu, bice
2(ab + ac + bc) = 4p2 (2p2 22 8r)
tj.
ab + ac + ca = p2 + 2 + 4r

264. Ako su a, b, s, stranice i p poluobim trougla ABC, r i poluprecnici


opisanog i upisanog kruga, a a poluprecnik spolja upisanog kruga koji se nalazi
u uglu A, dokazati da je (a) abc = 4rp; (b) abc = 4ra (p a).
Resenje: (a). Ako je D podnozje visine iz temena A ,a AE precnik kruga
opisanog oko trougla ABC,bice
ABD AEC
pa je AB:AD=AE:AC,tj,
c : ha = 2r : b.
Otuda je
bc = 2rha ...(1)
Obelezimo sa S srediste upisanog kruga ,sa P tacku u kojoj je taj krug dodiruje
stranicu BC i sa E tacku u kojoj A sece starnicu BC.Iz slicnih trouglova ADE
i SPE sledi da je AD:SP=AE:SE,tj.
ha : = (a + b + c) : a.
Otuda je
ha =

2p
...(2).
a
199

Iz (1) i (2) sledi da je


abc = 4rp.
(b)Oblezimo sa Sa srediste spolja upisanog kruga koji se nalazi u uglu A i
sa Pa tacku u kojoj taj krug dodiruje stranicu BC. Iz slicnih trouglova ADE i
Sa Pa E sledi sa je
AD : Sa Pa = AE : Sa E
tj.
ha : a = (b + c a) : a.
Otuda je
ha =

2(p a)a
....(3).
a

Iz (1) i (3) sledi da je


abc = 4ra (p a).
265. Ako su P i Q tacke u kojima simetrale unutrasnjeg i spoljasnjeg ugla
A trougla ABC seku pravu BC, dokazati da je (a) AP 2 = AB AC BP P C;
(b) AQ2 = BQ CQ AB AC.

P
N

Slika 196
ABC,bice
ABP AN C
pa je AB : AN = AP : AC,i prema tome AB AC = AP AN. S obzirom da je
tacka P izmedju tacaka A i N ,bice AN = AP + P N ,pa je
AB AC = AP 2 + AP P N,
i prema tom
AP 2 = AB AC + BP P N.
(b) AKo je M presek prave AQ s krugom oko trougla ABC ,bice
AM C ABQ
,pa je AB : AQ = AM : AC,i prema tome AB AC = AQ AM . S obzirom da
je tacka A izmedu tacaka M i Q, bice AM = QM QA, pa je
AB AC = AQ M Q AQ2 ,
200

i prema tome
AQ2 = BQ AC
(sl.196)
266. Ako su lA i
la simetrale unutrasnjeg i spoljasnjeg ugla A trougla ABC,
a, b, c, duzi jednake stranicama BC, CA, AB i p poluobim tog trougla, dokazati
4bc(p b)(p c)
4bcp(p a)
; (b) la2 =
.
da je (a) la2 =
(b + c)2
(b c)2
Resenje: (a) Ako obelezimo sa P presek simetrale unutrasnjeg ugla A sa
stranicom BC trougla ABC ,bice ABC AN C pa je BP : P C = c : b,pa
ac
ab
je BP =
i PC =
.Prema prethodnom zadatku je AP 2 = AB AC
b+c
b+c
BP P C, pa je
2
lA
= bc

a2 bc
bc
bc
4bcp(p a)
=
[(b+c)2 a2 ] =
(b+c+a)(b+ca) =
2
2
2
(b + c)
(b = c)
(b + c)
(b + c)2

(b)Ako obelezimo s Q presek simetrale spoljasnjeg ugla A sa pravom BC ,pri


cemu je npr tacka Q iza B u odnosu na C,bice BQ : CQ = c : b ,pa je
BQ =

ac
bc

CQ =

ab
.
bc

Prema prethodnom zadatku je

AQ2 = BQ CQ AB AC,
pa je
2
IA
=

bc
bc
4bc(p b)(p c)
a2 bc
bc =
[a2 (bc)2 ] =
(a+bc)(ab+c) =
.
2
2
2
(b c)
(b c)
(b c)
(b c)2

267. Ako su a, b, c, stranice i p poluobim trougla ABC, a S, Sa , Sb , Sc


bc(p a)
sredista upisanih krugova tog trougla, dokazati da je (a) AS 2 =
; (b)
p
bcp
bc(p c)
bc(p b)
ASa2 =
; (c) ASb2 =
; (d) ASc2 =
.
pa
pb
pc
Resenje: (a) Ako je P tacka u kojoj simetrala AS ugla A sece stranicu
BC,bice AS : SP = (b + c) : a,pa je
AS =

b+c
AP.
2p

Otuda ,primenom prethodnog zadatka,nalazimo da je


AS 2 =

(b + c)2 4bcp(p a)
bc(p a)
(b + c)2

AP
=

=
.
2
2
2
4p
4p
(b + c)
p

201

(b)S obzirom da je
ASa : Sa P = (b + c) : a,
bice
AS =

b+c
AP.
2(p a)

Otuda ,primenom prethodnog zadatka,nalazimo da je


AS 2 =

(b + c)2
4bcp(p a)
bcp
(b + c)2
2

AP
=

=
.
2
2
2
4(pa )
4 (p a)
(b + c)
pa

(v) Ako je Q tacka u kojoj simetrala ASb spoljasnjeg ugla A sece stranicu
BC,bice npr tacka Q iza B u odnosu na C,pa je
ASb : Sb Q = (b c) : a
i prema tome
ASb =

(b c)
AQ.
2(p b)

Otuda ,primenom prethodnog zadatka,nalazimo da je


2
ASB
=

(b c)2 4bc(p b)(p c)


bc(p c)
(b c)2
AQ2 =
=
.
2
4(p b)
4(p b)2
(b c)2
pb

(g)Dokaz isti kao u prethodnom delu.


268. Ako su S, Sa , Sb , Sc sredista i , a , b , c poluprecnici upisanih krugova
trougla ABC, a r i p poluprecnik opisanog kruga i poluobim, dokazati da je (a)
AS 2 + BS 2 + CS 2 = p2 + 2 8r; (b) ASa2 + BSa2 + CSa2 = p2 2 + 2a2
4r + 4ra ; (c) ASa2 + BSb2 + CSc2 = p2 + 2 + 8r + 16r2 .
Resenje: Ako sa P, Q, R obelezimo tacke u kojima upisani krug dodiruje
stranice BC, CA, AB; iz pravouglih trouglova ASR, BSP, CSQ nalazimo da je

AS 2 +BS 2 +CS 2 = 32 +(pa)2 +(pb)2 +(pc)2 = 32 +3p2 2p(a+b+c)+(a2+b2 +c2 ) = 32 +3p2 4p2 +(2p2 2
(b)Ako sa
Pa , Qa , Ra
obelezimo tacke u kojima spolja upisani krug koji se nalazi u uglu A dodiruje
BC, CA, AB;iz pravouglih trouglova
ASa Ra , BSa Pa , CSa Qa
nalazimo da je

ASa2 +BSa2 +CSa2 = p2 +2a +(pc)2 +2a +(pb)2 +2a = 3p2 +32a 2p(b+c)+(b2+c2 ) = 3p2 +32a 2p(2pa)+(a2 +b
odakle koristeci zadatke 260,263,154 dobijamo da je
ASa2 + BSa2 + CSa2 = p2 2 + 22a 4r + 4r2a .
202

(v) S obzirom da je
2

ASa2 = p2 + 2a , BSb2 = p2+b , CSc2 = p2 + 2c ,


bice
ASa2 +BSb2 +CSc2 = 3p2 +(2a +2b +2c ) = 3p2 +(16r2 +2 2p2 +8r) = p2 +2 +8r+16r.

269. Ako su S, Sa , Sb , Sc sredista upisanih krugova trougla ABC i a, b, c duzi


AS 2
BS 2
CS 2
jednake stranicama BC, CA, AB, dokazati da je (a)
+
+
= 1;
bc
ca
ab
ca
ab
bc
+
+
= 1.
(b)
ASa2
BSb2
CSc2
Resenje: Prema zadatku 267, imamo da je
(a)
BS 2
CS 2
pa pb pc
AS 2
+
+
=
+
+
= 1.
bc
ca
ab
p
p
p

(b)
bc
ca
ab
papbpc
+
+
=
= 1.
2
2
2
ASa
BSb
CSc
p
p
p
270. Ako su S, Sa , Sb , Sc sredista upisanih krugova trougla ABC, a ha , hb , hc
AS2 BS 2
+
+
njegove visine i r poluprecnik opisanog kruga, dokazati da je (a)
ha
hb
2
CS
ha
hb
hc
1
= 2r; (b)
+
+
= .
2
2
2
hc
ASa
BSb
CSc
2r
Resenje: (a)Prema zadacima 269 i 251,imamo da je
AS 2 BSb2 CSc2
=1
bc ca ab
i
bc = 2rha , ca = 2rhb , ab = 2rhc
pa je

bc
ca
ab
+
+
= 2r.
ASa2
BSa2
CS

(b)Prema zadacima 269 i 251,imamo da je


BS 2
CS 2
AS 2
+
+
=1
ba
ca
ab
i
bc = 2rha , ca = 2rhb , ab = 2rhc ,

203

pa je

ha
hb
hc
1
+
+
= .
2
2
2
ASa
BSb
CSc
2r

271. Ako su S, Sa , Sb , Sc sredista i , a , b , c poluprecnici upisanih krugova


trougla ABC, a O i r srediste i poluprecnik opisanog kruga, dokazati da je

(a)
OS 2 = r(r 2)
(b)
OSa2 = r(r + 2a )
(c)
OS 2 + OSa2 + OSb2 + OSc2 = 12r2

Uputstvo.
(a) Ako su P i Q tacke u kojima prava OS sece krug koji je opisan oko trougla
ABC,M i N sredista lukova BC kruga , pri cemu je M na luku BAC ,R tacka
u kojoj upisani krug k dodiruje stranicu AB i d=OS.bice tacka S na tetivi AN
kruga l.pa je

P1M
A
Q

S O

C
N Q1
Sa

Slika 197
ASBN = r2 d2 ....(1). Iz slicnih troglova ARS i MBN nalazimo AS : RS =
M : BN, odnosno da je ASBN = M N RS. Kako je MN precnik kruga ,a
RS poluprecnik kruga k,bice (sl.197) ASBN = 2r.....(2). Iz jednakosti (1) i (2)
nalazimo da je r2 d2 = 2r pa je OS 2 = r(r 2).
(b)Kao u prethodnom delu ,ako su P1 iQ1 tacke u kojima prava OSa sece krug
,Ra podnozje upravne Sa na pravoj AB i da = OSa , bice Sa na pravoj AN,pa
je
ASa N Sa = P1 Sa Q1 Sa = (da + r)(da r).
No
N Sa = N B,

204

pa je
ASa N B = d2 r2 ....(1).
Iz slicnih trouglova
ARa Sa iM BN
nalazimo da je
ASa : Ra Sa = M N : N B,
odnosno da je
ASa N B = M N Ra Sa = 2ra ....(2).
Iz jednakosti (1) i (2) sledi da je
d2 r2 = 2ra ,
pa je
OS = r(r + 2a ).

(c)Iz prethodna dva dela ovog zadatka,nalazimo da je


OS 2 + OSa2 + OSb2 + OSc2 = 4r2 + 2r(a + b + c ) = 4r2 + 8r2 = 12r2

272. Ako su a, b, c stranice i p poluobim trougla ABC, a S, Sa , Sb , Sc sredista


upisanih krugova, dokazati da je
(a)
a2 bc
SSa2 =
p(p a)
(b)

Sb Sc2 =

a2 bc
(p b)(p c)

Uputstvo.
(a) Duz SSa je hipotenuza pravouglog trougla BSSa , pa je prema zadatku 267
SSa2 = BS 2 + BSa2 =

a2 bc
ac(p b) ac(p c)
+
=
.
p
pa
p(p a)

(b) Duz Sb Sc je hipotenuza pravouglog trougla BSb Sc pa je prema zadatku


Sb Sc2 = BSb2 + BSc2 =

acp
ac(p a)
a2 bc
+
=
.
pb
pc
(p b)(p c)

OS 2 + OSa2 + OSb2 + OSc2 = 4r2 + 2r(a + b + c ) = 4r2 + 8r2 = 12r2 .

205

273. Ako su S, Sa , Sb , Sc sredista i , a , b , c poluprecnici upisanih krugova


trougla ABC, a r poluprecnik kruga opisanog oko tog trougla, dokazati da je
(a)
SSa2 = 4r(a )
(b)
Sb Sc2 = 4r(b + c )
(c)
SSa2 + SSb2 + SSc2 = 8r(2r )
(d)
Sb Sc2 + Sc Sa2 + Sa Sb2 = 8r(4r + )

Uputstvo.
(a) Obelezimo sa A1 srediste stranice BC sa N srediste luka BC opisanog kruga
na kome nije tacka A.S obzirom daje tacka N srediste hipotenuze SSa pravouglog trougla BSSa ,bice SSa = 2BN, i prema tome
SSa2 = 4BN 2 = 4(BA21 + N A21 ) = a2 + (a )2 = (a )(a + ) + 4r(a ).

(b) Obelezimo sa A1 srediste stranice BC i sa M srediste kruznog luka BAC.S


obzirom daje tacka M srediste hipotenuze Sb Sc pravouglog trougla BSb Sc , bice
Sb Sc = 2BM,i prema tome
Sb Sc2 = 4BM 2 = 4(BA1 +M A21 ) = a2 +(b +c )2 = (a )(b +c )+(b +c )2 ) = 4r(b c ).

(c) Prema prvom delu ovog zadatka ,imamo da je


SSa2 + SSb2 + SSc2 = 4r(a ) + 4r(c ) = 4r(a + b + c 3) = 8r(2r ).

(d) Prema drugom delu ovog zadatka ,imamo da je


Sb Sc2 +Sc Sa2 +Sa Sb2 = 4r(b +c )+4r(c +a )+4(a b ) = 8r(a +b +c ) = 8r(4r).

274. Ako su S, Sa , Sb , Sc sredista upisanih krugova trougla ABC i ako je


poluprecnik upisanog kruga, r poluprecnik opisanog kruga i p poluobim tog
trougla, dokazati da je
(a)
AS BS CS = 4r2
(b)
ASa BSb CSc = 4rp2
206

(c)
SSa SSb SSc = 16r2
(d)
Sa Sb Sb Sc Sc Sa = 16r2 p
Uputstvo.
(a) Koristeci zadatke 267, 264, 261, nalazimo da je
AS 2 BS 2 CS 2
=

3

bc(p a) ca(p b) ab(p c)
= 16r2 2 p2 2 = 16r2 4 .
= a2 b 2 c2
p
p
p
a b c
p

(b) Koristeci zadatke 267,264 i 261 nalazimo da je


ASa2 BSb2 CSc2 =

a b c
p2
bcp cap abp
= a2 b 2 c2
= 16r2 2 p2 3 = 16r2 p4 .
papbpc

Otuda je
ASa BSb CSc = 4rp2 .

(c) Prema zadatku 273,imamo da je


SSa2 SSb2 SSc2 = 64r3 (a )(b )(c ) = 64r3 [a b c (a b +b c +c a )+2 (a +b +c )].
Otuda primenom zadataka 261,,260 ,154 nalazimo da je
SSa2 SSb2 SSc2 = 256r4 2 ,
pa je
SSa2 SSb2 SSc2 = 16r2 .

(d) Prema zadatku 273 ,imamo da je

Sa Sb Sb Sc Sc Sa = 64r3 (a +b )(b +c )(c +a ) = 64r3 (a +b )(a b +b c +c a +3 ) = 64r3 (a +b )(p2 +2c ) = 64


.Otuda je
Sa Sb Sb Sc Sc Sa = 16r2 p.

275. Ako su S, Sa , Sb , Sc sredista upisanih krugova trouglova dokazati da je


(a)
AS ASa = AB AC
(b)
ASb ASc = AB AC
207

.
Uputstvo.
(a) Kako su uglovi SBSa iSCSa pravi tacke S, Sa , B, C su na krugu kome je
duz SSa precnik.ako je B druga presecna taka toga kruga s pravom AC,bic e

AS ASa = AB AC. No,AB = AB ,pa je ASa AS = AB AC.


(b) Kako su uglovi Sb BSc iSb CSc pravi bice Sb , Sc , B, C na krugu kome je
duzSb Sc precnik.Ako je B druga presecna tacka toga kruga s pravom AC bice
ASb ASc = AB AC. No AB = AB,pa je ASb ASc = AB AC.
276. Ako je r poluprecnik opisanog kruga, poluprecnik upisanog kruga,
S srediste upisanog kruga i Sa srediste spolja upisanog kruga koji odgovara
stranici BC, dokazati da je
AS SSa = 4.
Uputstvo.
Obelezimo sa O srediste opisanog kruga trougla ABC , sa P i Q tacke ku kojima prava OS sece krug l i sa d duz odredenu tackama O i S . S obzirom da je
srediste N duzi SSa na krugu l , imamo da je AS.SSa = 2AS.SN = 2SP.SQ =
2(rd)(r+d) = (r2 +d2 ). Prema Ojlerovoj teoremi, imamo da je d2 = r2 = 2r,
pa je AS.SSa = 4r.
277. Ako su duzi b i c jednake stranicama AC i AB trougla ABC, S i
srediste i poluprecnik upisanog kruga r poluprecnik opisanog kruga tog trougla,
dokazati da je
AS 2 = bc 4r.
Uputstvo.
Ako obelezimo sa Sa srediste spolja upisanog kruga koji odgovara stranici BC
trougla ABC, prema zadacima 275 i 276, imamo da je
AB.AC = AS.ASa = AS(AS+SSa ) = AS 2 +ASS.SSa = AS 2 4r. OtudajeAS 2 = bc4r
.
278. Ako obelezimo sa ha , hb , hc visine iz temena A, B, C trougla ABC, sa
S srediste upisanog kruga i sa r poluprecnik opisanog kruga, dokazati da je
SB 2
SC 2
SA2
+
+
= 2r.
ha
hb
hc

Uputstvo.
Ako obelezimo sa poluprecnik upisanog kruga, prema zadatcima 277 i 258
imamo da je
SB 2
SC 2
bc 4r ca 4r ab 4r
SA2
+
+
=
+
+
ha
hb
hc
ha
hb
hc
208

ca ab
1
1
1
bc
+
+
4r(
+
+ )
ha
hb
hc
ha
hb
hc
= 2r + 2r + 2r 4r = 2r

279. Ako su na , nb , nc odsecci koje trougao ABC odredjuje na pravama


koje sadrze srediste upisanog kruga, a paralelne su respektivno sa stranicama
BC = a, CA = b, AB = c, dokazati da je
nb
nc
na
+
+
= 2.
a
b
c

Uputstvo.
Ako obelezimo sa ha , hb , hc visine iz temena A, B, C trougla ABC, a sa poluprecnik upisanog kruga, imamo da je
ha
na
=
a
hb
nb
=
b
hb
nc
hc
=
c
hc
otuda je
na
nb
nc
ha hb hc
+
+
+
+
=
a
b
c
ha
hb
hc
1
1
1
1
+
+ )=3 =2
= 3 (
ha
hb
hc

209

4
4.1

KARAKTERISTICNE
TEOREME I NJIHOVE
PRIMENE
Apolonijeva teorema

280. (Apolonije) Ako je X tacka stranice BC trougla ABC takva da je


BX : XC = m : n, dokazati da je
nAB 2 + mAC 2 + nBX 2 + mCX 2 + (m + n)AX 2 .

Uputstvo.

Slika 198
Ako je D podnozje visine iz temena A, bice X = D ili X 6= D (sl.198). Kada
je X = D, iz pravougaonih trouglova ABX i ACX nalazimo da je AB 2 =
AX 2 + BX 2 i AC 2 = AX 2 + CX 2 . Pomnozicemo prvu od ovih jednakosti sa n a drugu sa m, posle sabiranja odgovarajucih strana dobijanih jednakosti, nalazimo da je nAB 2 + mAC 2 = nBC 2 + mCX 2 + (m + n)AX 2 .
Kada je X 6= D neposredni uglovi ABX i ACX AXBG ostar i prema tome
ugao ACX tup. Iz ABX i ACX nalazimo da je AB 2 = AX 2 2BX.DX i
AC 2 = AX 2 + CX 2 + 2CX.DX. Mnozenjem prve od ovih jednakosti sa n a
druge sa m, posle sabiranja odgovarajucih strana dobijenih jednakosti nalazimo
da je nAB 2 + mAC 2 + nBX 2 + mCX 2 + (m + n)AX 2 .
Napomena. Obrazac kojima je iskazana Apolonijeva teorema moze se napisati i u sledecem obliku
nAB 2 + mAC 2 = (m + n)(AX 2 + BX.XC)
jer je
nAB 2 +mAC 2 = (m+n)AX 2 +nBX 2 +mCX 2 = (m+n)AX 2 +nBX.BX+mCX.CX =
(m + n)AX 2 + mBX.CX + nBX.XC = (m + n)(AX 2 + BX.CX)
.
281. Ako je X proizvoljna tacka ravni pravougaonika ABCD, dokazati da
je AX 2 + CX 2 = BX 2 + DX 2 .
210

Uputstvo.

O
A

Slika 199
bice
AX 2 + CX 2 = 2OX 2 + OA2 + OC 2
i
BX 2 + DX 2 = 2OX 2 + OB 2 OD2
Otuda je
AX 2 + CX 2 = BX 2 + DX 2
(sl. 199)
282. Dokazati da je kod paralelograma zbir kvadrata stranica jednak zbiru
kvadrata dijagonala.
Uputstvo.

Slika 200
teoremi bice
AB 2 + BC 2 = 2OB 2 + 2OA2
i
CD2 + DA2 = 2OD2 + 2OA2
Otuda je
AB 2 + BC 2 + CD2 + DA2 = 4OA2 + 4OB 2 = AC 2 + BD2
211

(sl. 200)
283. Ako je kod cetvorougla ABCD zbir kvadrata stranica jednak zbiru
kvadrata dijagonala, dokazati da je taj cetvorougao paralelogram.
Uputstvo.
Ako je O srediste dijagonale AC, primenom Apolonijeve teoreme, imamo da je
AB 2 + BC 2 = 2BO2 + 2AO2
i
CD2 + DA2 = 2DO2 + 2AO2
pa je
AB 2 + BC 2 + CD2 + DA2 = AC 2 + 2BO2 + 2DO2
Iz ove jednakosti i iz
AB 2 + BC 2 + CD2 + DA2 = AC 2 + BD2
sledi da je
BO2 + 2DO2 = BD2
pa je tacka O srediste duzi BD, i prema tome, cetverougao ABCD paralelogram
(sl.200).
284. Ako je X tacka stranice BC trougla ABC takva da je BX : XC = m :
n, Y tacka u kojoj prava AX sece krug upisan oko trougla ABC, dokazati da je
nAB 2 + mAC 2 = (m + n)AX AY.

Uputstvo.

C
Y

Slika 201
Prema Apolonijevoj teoremi iskazanoj u napomeni (v. res. zad. 280), bice (sl.
201)
nAB 2 +mAC 2 = (m+n)(AX 2 +BX.XC) = (m+n)(AX 2 +AX.XY ) = (m+n)(AX+XY )AX = (m+n)AX.XY
212

285. Ako je D podnozje visine iz temena A i A1 stranice BC trougla ABC,


dokazati da je
AC 2 AB 2 = 2BC DA1 .
Uputstvo.
Primenom Apolonijeve teoreme nalazimo da je
1
AB 2 + AC 2 = 2AA21 + BC 2
2
Otuda i iz jednakosti
1
AA21 = AD2 + DA21 = AB 2 BD2 + DA21 = AB 2 ( BC DA1 )2
2
sledi da je
AC 2 AB 2 = 2BC.DA1
286. Ako je D tacka stranice BC trougla ABC takva da je BD = 2DC,
dokazati da je
AB 2 2AC 2 = 3AD2 + 6AD2 .
Resenje: Primenom Apolonijeve teoreme nalazimo da je
AB 2 + 2AC 2 = 3AD2 + BD2 + 2CD2
te s obzirom da je BD = 2DC, dobijamo da je
AB 2 + 2AC = 3AD2 + 6CD2

287. Ako su X i Y tacke stranice BC trougla ABC takve da je BX = XY =


Y C, dokazati da je
AB 2 + AC 2 = AX 2 + AY 2 + 4XY 2 .
A

Slika 202
213

Resenje: Primenom Apolonijeve teoreme nalazimo da je (sl. 202)


AB 2 + AY 2 = 2AX 2 + 2XY 2
i
AC 2 + AX 2 = 2AY 2 + 2XY 2
te sabiranjem odgovarajucih strana dobijamo da je
AB 2 + AC 2 = AX 2 + AY 2 + 4XY 2

288. Ako su AA1 , BB1 , CC1 tezisne linije trougla ABC i T njegovo teziste,
dokazati da je
(a) AA21 = 41 (2AB 2 + 2AC 2 BC 2 );
(b) AA21 + BB1 + CC12 = 34 (AB 2 + BC 2 + CA2 );
(v) AT 2 + BT 2 + CT 2 = 13 (AB 2 + BC 2 + CA2 ).
Resenje: (a) S obzirom da je tacka A1 srediste stranice BC, primenom
Apolonijeve teoreme nalazimo da je
AB 2 + AC 2 = BA21 + A1 C 2 + 2AA1
tj. da je
AB 2 + AC 2 =
Otuda je
AA21 =

BC 2
+ 2AA1
2

1
(2AB 2 + 2AC 2 BC 2 )
4

(b) Primenom prethodnom delu ovog zadatka je


AA21 =

1
(2AB 2 + 2AC 2 BC 2 )
4

1
(2AB 2 + BC 2 AC 2 )
4
1
CO12 = (2AC 2 + 2BC 2 AB 2 )
4
odakle sabiranjem odgovarajucih strana nalazimo da je
BB12 =

AA21 + BB12 + CC12 =

(v) Iz jednakosti

3
(AB 2 + BC 2 + CA2 )
4

2
AA1
3
2
BT = BB1
3
AT =

214

CT =

2
CC1
3

nalazimo da je
1
(AA21 + BB12 + CC12 )
3
pa je s obzirom na prethodni deo ovog zadatka
AT 2 + BT 2 + CT 2 =

AT 2 + BT 2 + CT 2 =

1
(AB 2 + BC 2 + CA2 )
3

289. Ako su P i Q sredista dijagonala AC i BD cetvorugla ABCD, dokazati


da je
1
P Q2 = (AB 2 + BC 2 + CD2 + DA2 AC 2 BD2 ).
4
D

Slika 203
Resenje: Linije trouglova P BD, BAC, DAC, prema zadatku 288. imamo da
1
1
je (v. sl. 203.) BP 2 + DP 2 = 2P Q2 + BD2 , AB 2 + BC 2 = 2BP 2 AC 2 ,
2
2
2
2
21
2
2
2
CD + DA = 2DP AC . Ako vrednosti za BP i DP iz poslednjih dveju
2
1
jednakosti zamenimo u prvu jednakost, dobijamo da je P Q2 = (AB 2 + BC 2 +
4
CD2 + DA2 AC 2 BD2 ).
290. Ako su l1 i la simetrale unutrasnjeg i spoljasnjeg ugla A trougla ABC,
zatim a, b, c stranice naspram temena A, B, C i p poluobim tog trougla, dokazati
4abc(p b)(p c)
4bcp(p a)
; (b) la2 =
.
da je (a) la2 =
(b + c)2
(b c)2
Resenje: (a) Ako obelezimo sa S tacku u kojoj simetrala unutrasnjeg ugla
c
BC
A sece stranicu BC trougla ABC, bice BE : EC = c : b, pa je BE =
b+c
b
i EC =
BC. Primenom Apolonijeve teoreme i ovih jednakosti nalazimo
b+c
da je
bAB 2 + cAC 2 = (b + c)AE 2 + cCE 2 = (b + c)AE 2 +

bc
BC 2
b+c

Otuda je
Ia2 =

bc
4bcp(p a)
bc
[(b + c)2 a2 ] =
(b + c + a)(b + c a) =
(b + c)2
(b + c)2
(b + c)2
215

(b) Postupak analogan.


291. Ako je tacka M srediste tetive AB kruga k kome je srediste O, N
proizvoljna tacka kruga l kome je duz OM precnik i C, dokazati da je
AC 2 + BC 2 = 4N C 2 .
C

M
A

B
N

Slika 204
Resenje: M C i SC su tezisne linije trouglova ABC i OM C, te primenom
zadatka 288. nalazimo da je (sl. 204)
1
AC 2 +BC 2 = 2M C 2 + AB 2 = 2M C 2 +2AM 2 = 2M C 2 +2OC 2 2OM 2 = 4SC 2 OM 2 = 4SC4SN 2 = 4N C 2
2

292. Ako su H, T , O, S ortocentar, teziste, srediste opisanog kruga i srediste


upisanog kruga bilo kojeg trougla, a r i poluprecnik opisanog i upisanog kruga,
dokazati da je: (a) SH 2 + 2SO2 = 3(ST 2 + 2OT 2 ) (b) 3(ST 2 + 2OT 2 ) SH 2 =
2r(r 2)
Resenje: (a) Prema Ojlerovoj teoremi, tacka T je izmedju tacaka O i H,
takva da je HT : T O = 2 : 1, te primenom Apolonijeve teoreme, nalazimo da je
SH 2 + 2SO2 = 3ST 2 + HT 2 + 2OT 2
216

Otuda i iz
HT 2 = 4OT 2
sledi da je
SH 2 + 2SO2 = 3(ST 2 + 2OT 2 )

(b) Prema prethodnom delu ovog zadatka i zadatku 271, imamo da je


3(ST 2 + 2OT 2 ) SH 2 = 2SO2
i
SO2 = r(r 2)
pa je
3(ST 2 + 2OT 2 ) SH 2 = 2r(r 2)
293. Dokazati da je poluprecnik kruga koji dodiruje katete i opisani krug
pravouglog trougla jednak precniku upisanog kruga.
Resenje: Ako su a, b, c hipotenuza i katete provouglog trougla ABC, r
poluprecnik i O srediste opisanog kruga, poluprecnik i S srediste upisanog
kruga, a poluprecnik i S srediste kruga koji dodiruje opisani krug i katete

a
a
AB i AC, bice AS = 2, AS = 2, SS = 2( ), AO = , OS = .
2
2
Prema Ojlerovoj teoremi (v.zad. 271) imamo da je
OS 2 = r2 2r
pa je

a2
a
4
. Primenom Apolonijeve teoreme na trougao AOS , nalazimo da je
OS 2 =

AO2 .SS + OS 2 + AS.SS 2 + SS 2 .AS


Otuda je
a2

a2

a2
2( )+ 2( a +2 ) = 2 2( a)+2 2( )+2 2(2 2 +2 )
4
4
4
odakle dobijamo da je = 2.
294. Ako su stranice BC, CA, AB trougla ABC jednake duzima a, b, c i
ako je O srediste i r poluprecnik kruga opisanog oko trougla ABC, a O tacka
simetricna sa O u odnosu na pravu BC, dokazati da je
AO2 = r2 + b2 + c2 a2 .
Resenje: Ako je A srediste stranice BC, bice duz AA tezisna linija trouglova
ABC i AOO , pa je prema zadatku 288
AA2 =

1
(2AB 2 + 2AC 2 BC 2 )
4
217

1
(2AO2 + 2AO2 + OO2 ).
4
Iz ovih jednakosti nalazimo da je :
AA2 =

2AB 2 + 2AC 2 BC 2 = 2AO2 + 2AO2 OO2 .


BC 2
, bice OO2 = 4OB 2 BC 2 i prema
Kako je OO = 2OA i OA2 = OB 2
4
tome
2AB 2 + 2AC 2 BC 2 = 2AO2 + 2AO2 4OB 2 + BC 2 .
Otuda je: AO2 = r2 + b2 + c2 a2 .

295. Ako je kod trougla ABC AB 2 + AC 2 = 5BC 2 , dokazati da su tezisne


linije iz temena B i C medu sobom upravne. Resenje: Obelezimo sa B i C
sredista stranica AC i AB, a sa T teziste trougla ABC. Prema zadatku 288
imamo da je
1
BB 2 = (2BC 2 + 2AB 2 AC 2 )
4
i
1
CC 2 = (2BC 2 + 2AC 2 AB 2 ).
4
Kako je
2
BT = BB
3
i
2
CT = CC ,
3
bice:
1
BT 2 = (2BC 2 + 2AB 2 AC 2 )
9
i
1
CT 2 = (2BC 2 + 2AC 2 AB 2 ).
9
Otuda je
1
BT 2 + CT 2 = (4BC 2 + AB 2 + AC 2 )
9
tj.
BT 2 + CT 2 = BC 2
, pa su duzi BT i CT , prema tome i tezisne linije BB i CC medu sobom
upravne.
296. Ako je S srediste kruga upisanog u trougao ABC i P proizvoljna
tacka ravni tog trougla, dokazati da je
BCBA2 +CAP B 2 +ABP C 2 = BCSA2 +CASB 2 +ABSC 2 +(BC+CA+AB)P S 2 .

218

S
D
B

Slika 205
Resenje: Ako je D tacka u kojoj simetrala AS ugla A sece stranicu BC, bice
AS : SD = (AB + AC) : BC i BD : DC = AB : AC, pa primenom Apolonijeve
teoreme na trouglove P AD, P BC, SBC (sl.205), nalazimo da je:
BCP A2 +(AB+AC)P D2 = BCSA2 +(AB+AC)SD2 +(BC+CA+AB)P S 2 ,
AC P B 2 + AB P C 2 = AC BD2 + AB DC 2 + (AB + AC)P D2 ;
AC SB 2 + AB SC 2 = AC BD2 + ABDC 2 + (AB + AC)SD2 .

Otuda je:
BCP A2 +CAP B 2 +ABP C 2 = BCSA2 +CASB 2 +ABSC 2 +(BC+CA+AB)P S 2 .
je

297. (M. Stewart) Ako je X tacka stranice BC trougla ABC, dokazati da


BC AX 2 = XC 2 AB 2 + BX AC 2 BX XC bc.
Resenje: Primenom Apolonijeve teoreme, imamo da je:
XC AB 2 + BX AC 2 = BC AX 2 + XC BX 2 + BX XC 2 .

Otuda je:
BC AX 2 = XC AB 2 + BX AC 2 BX XC(BX + XC)
i prema tome
BC AX 2 = XC AB 2 + BX AC 2 BX XC BC.
298. Ako su r i poluprecnik opisanog kruga i poluprecnik upisanog kruga
trougla ABC kome je stranica BC najmanja, a M i N tacke stranica AB i
AC takve da je M B = N C = BC = a, dokazati da je
MN2 =

a2
(r 2).
r

Resenje: Primenom Stjuartove teoreme na trouglove ABC i ABN , nalazimo


da je
ac2
a3
BN 2 =

= ab + 2a2
b
b
219

i
MN2 =

a
ca
BN 2 + (b c)2 a(a c),
c
c

pa je

b 2 + c2 a2
c2 + a2 b 2
a2 + b 2 c2
MN2
=
2

a2
bc
ca
ab
Iz ove jednakosti, koristeci zadatke 246, 251, 158 dobijamo da je:
MN2 =

a2
(r 2 ).
r

299. Ako su r i a poluprecnik opisanog kruga i poluprecnik upisanog kruga


koji odgovara stranici BC trougla ABC, a M i N tacke pravih AB i AC takve
da je A B M i M B = N C = BC = a, dokazati da je
MN2 =

a2
(r + 2 a ).
r

Resenje: Dokaz se izvodi kao u prethodnom zadatku.


300. Ako su B i C podnozja upravnih iz temena B i C na simetrali unutrasnjeg ugla A trougla ABC, dokazati da je:
r
r
c
b

BB =
(p b) (p c) i CC =
(p b) (p c).
b
c

B
B
B

C
E

Slika 206
Resenje: Ako obelezimo sa B tacku u kojoj prava BB sece pravu AC, primenom Stjuartove teoreme , nalazimo da je (v. sl. 206).
AC BB 2 = AB 2 B C + BC 2 AB AC AB B C,
tj. da je
BB = c2 (b c) + bc(b c).
Otuda je
c
c
c
BB 2 = [a2 (b c)2 ] = (a b + c)(a + b c) = 4 (p b)(p c)
b
b
b

220

i prema tome

BB = 2

c
(p b)(p c).
b

r
c
1
BB , pa je BB =
(p b)(p c). Istim postupkom dokazuje
2
b
se drugi deo stava.
301. Ako su a i c osnovice, a b i d kraci, e i f dijagonale trapeza, dokazati
da je
No BB =

e2 = ac+ =

ad2 cb2
ab2 cb2
i f 2 = ac +
.
ac
ac

Resenje:
Ako su stranice AB,BC,CD,DA trapeza ABCD jednake duzine a,b,c,d i ako je
E tacka na osnovici AB takva da je CE k AD, primenom Stjuartove teoreme
na trougao ABC nalazimo da je
ad2 = (a c)e2 + cb2 ac(a c)
tj. da je
e2 = ac+ =

ad2 cb2
ac

Istim postupkomizvodi se i drugi deo stava.


302. Odrediti skup svih tacaka P za koje je mAP 2 + nBP 2 = l2 gde su A
i B date tacke, m i n dati brojevi i l data duz.
Resenje:
Analizirajmo prvo slucaj kada je m + n = 0, tj. kada je AP 2 BP 2 =
221

l2
.Ako
m

je Q podnozje upravne iz P na AB, bice AP 2 BP 2 = AQ2 BQ2 ,pa su sve


tacke P na izvesnoj pravoj p koja je u tacki Q upravna na pravu AB. Obrnuto,
ako je P proizvoljna tacka prave p,bice AP 2 BP 2 = AQ2 BQ2 ,prema tome,
tacka P pripada trazenom skupu. Otuda sledi da je skup svih tacaka P takvih
l2
prava p koje je u tacki Q upravna na pravoj AB.
da je AP 2 BP 2 =
m
Sad analizirajmo slucaj kad je m + n 6= 0. Ako obelezimo sa C tacku prave DB
takvu da je AC : CB = n : m, bice AC : CB : AB = n : m : (m + n). Primenom Stjuartove teoreme na tri kolinearne tacke A,B,C i tacku P , nalazimo
l2
n
mn
m
AB 2 , pa je CP 2 =
AP 2 +
BP 2

da je CP 2 =
2
m+n
m+n
(m n)
m+n
mn
AB 2 . Obratno, ako je P bilo koja tacka koja zadovoljava dobijenu jed(m + n)2
nakost, koristeci Stjuartovu teoremu nalazimo da je AP 2 + nBP 2 = l2 . Otuda
l2
mn
sleduje da pri m + n 6= 0 i
AB 2 > 0 skup svih tacaka P

m+n
(m + n)2
l2
mn
AB 2 .
predstavlja krug kome je srediste C,a poluprecnik r2 =

m + n (m + n)2
303. Neka je O srediste i r poluprecnik kruga k, a A tacka u njegovoj ravni
i d duz jednaka duzi OA. Ako su M i N promenljive tacke kruga k takve da je
ugao M AN prav:
(a) dokazati da je skup svih sredista svih tetiva M N kruga k takodje krug
l
kome se srediste poklapa sa sredistem duzi OA, a poluprecnik je jednak duzi
1
2
2
2 2r d ;
(b) dokazati da je skup svih presecnih tacaka dirki kruga k u tackama M i
N takodje izvestan krug l1 koji je homotetican s krugom l u odnosu na tacku O.

N
R

O
S

A
M

Resenje:
(a) Srediste P tetive M N je srediste hipotenuze M N pravouglog trougla AM N ,
pa je AP = M P = N P . Stoga je OP 2 + AP 2 = OP 2 + M P 2 = OM 2 = r2 .Ako
je P S tezisna linija trougla AOP , prema Apolonijevoj teoremi imamo da je
1
OP 2 + AP 2 = 2SP 2 + 2OA2 ,i prema tome SP 2 = (2OP 2 + 2AP 2 OA2 ) =
4
1 2
(2r d2 ).Otud sleduje da duz SP ne zavisi od polozaja tacaka M i N ,
4
pa je skup sredista P svih tetiva M N kruga k na izvesnom krugu l kome je
1 2
srediste S, a poluprecnik jednak duzi =
2r d2 . Dokazimo da svaka
2
222

tacka P kruga l pripada navedenom skupu taaka. Duz SP je tezisna linija


1
trougla AOP , pa je prema Apolonijevoj teoremi OP 2 +AP 2 = 2SP 2 + OA2 .
2
1
No,SP 2 = (2r2 d2 ) i OA = d, pa je OP 2 + AP 2 = r2 . Ako je M N tetiva
2
kruga k koja je u tacki P upravna na duzi OP, bice OP 2 + P M 2 = r2 .Iz
poslednjih dveju jednakosti sledi da je AP = M N = N P , pa se srediste P
stranice M N poklapa sa sredistem kruga opisanog oko trougla M AN . Stoga
je ugao M AN prav. Prema tome, skup sredista P svih tetiva M N kruga k je l.
(b) Presecna taka Q dirku kruga k u tackama M i N je na pravoj OP . Ta
prava sece l u tacki P i jos nekoj tacki R.Pri tome je OP OR = m2 .Prema
Euklidovoj teoremi kod pravouglog trougla OM Q je OP . OQ = OM 2 = e2 .
r2
Iz poslednjih dveju jednakosti nalazimo da je OQ = 2 OR. Otuda sleduje da
m
se tacka Q nalazi na izvesnom krugu l1 koji s krugom l ima za srediste homor2
tetije tacku O i koeficijent 2 . Dokazimo da svaka tacka Q1 kruga l1 pripada
m
navedenom skupu tacaka. Ako su M1 i N 1 dodirne tacke tangenata kroz Q1 na
krugu k P1 presecna tacka duzi OQ1 i M1 N1 , a R1 presecna tacka prava OQ1
s krugom l pri cemu je R1 iza O u odnosu na Q1 , bice OP1 OQ1 = OM 2 .No
r2
OQ1 = 2 OR1 i OM1 = r, pa je OP1 OR1 = m2 . Otud sleduje da je srediste
m
P1 tetive M1 N1 tacka kruga l, pa je prema prethodnom delu ugao M1 AN1 prav.
Prema tome, skup svih tacaka Q je krug l1 .
304. Ako je ABCD tangentan i tetivan cetvorougao, r poluprecnik opisanog
kruga, poluprecnik upisanog kruga i d duz koja spaja sredista tih krugova,
dokazati da je
1
1
1
+
= 2.
(r + d)2
(r d)2

Resenje:
Obelezimo sa O srediste opisanog kruga l, sa S srdiste upisanog kruga k i sa
K, L, M , N tacke u kojima stranice AB, BC, CD, DA dodiruju krug k.
Prema zadatku 162, prave KM i LN seku se u izvesnoj tacki P pod pravim
uglom. Bilo koje dve prave upravne medju sobom u tacki P , a koje seku krug
k prema zadatku 303, imaju takvu osobinu da se parovi uzastopnih dirki kruga
k u presecnim tackama seku na krugu l i opisan oko kruga k. Prema istom
zadatku tacke O, S, P pripadaju jednoj pravoj. Neka je A B C D onaj od tih
cetvorouglova upisanih u krug l cije stranice A B , B C , C D , D A dodiruju
krug k u tackama K , L , M , N takvim da prave K M i L N koje se seku
u tacki P pod pravim uglovima zahvataju s pravom OS jednake uglove. Prava
OS sadrzi dva naspramna temena tog cetvorougla, npr. temena A i C . Ako su
M i N podnozja upravnih kroz M i N na OS, bice SM M
= AN N

, pa je SM = N N .Otuda je
SM 2 + SN 2 = 2

(10)

Kod pravouglovih touglova SA N i SC M imamo da je SA SN = SN 2 i


SC SM = SM 2 . No SA = SO+OA = r+d i SC = OC OS = rd, pa je
223

SN =

SN 2
2
=
SA
r+d

(11)

SM =

2
SM 2
=

SC
rd

(12)

Iz jednakosti (1),(2),(3) nalazimo da je

1
1
1
+
= 2
(r + d)2
(r d)2

4.2

Lajbnicova teorema i njena primena

305. (G.W. Leibnitz) Ako je T teziste trougla ABC i P proizvoljna tacka,


dokazati da je
P A2 + P B 2 + P C 2 = T A2 + T B 2 + T C 2 + 3P T 2.

T
B

Resenje:
Ako obelezimo sa D srediste stranice BC,bice tacka T izmedju tacaka A i
D takva da je AT : T D = 2 : 1. Otuda, primenom Apolonijeve teoreme
CAD, P BC, T BC nalazimo da je P A2 + 2P D2 = T A2 + 2T D2 + 3P T 2,
P B 2 + P C 2 = BD2 + DC 2 + 2P D2 , T B 2 + T C 2 = BD2 + DC 2 + 2T D2 .
Sabiranjem odgovarajucih strana prvih dveju jednakosti te primenom trece nalazimo da je P A2 + P B 2 + P C 2 = T A2 + T B 2 + T C 2 + 3P T 2
306. Ako je T teziste cetvorougla ABCD i P proizvoljna tacka, dokazati da
je
P A2 + P B 2 + P C 2 + P D2 = T A2 + T B 2 + T C 2 + T D2 + 4P T 2.

224

Resenje:
Ako obelezimo sa T teziste trougla ABC, prema Lejbnicovoj teoremi imamo
da je
P A2 + P B 2 + P C 2 = T A2 + T B 2 + T C 2 + 3T P
(13)
T A2 + T B 2 + T C 2 = T A2 + T B 2 + T C 2 + 3T T

(14)

Prema zadatku tacka T je izmedju tacaka D i T takva da je DT : T T = 3 : 1.


Stoga se primenom Apolonijeve teoreme na trougao P DT i tacku T koja se
nalazi na stranici DT tog trougla dobija da je
P D2 + 3T P 2 = DT 2 + 3T T 2 + 4P T 2

(15)

Iz relacije (1), (2), (3) sledi da je


P A2 + P B 2 + P C 2 + P D2 = T A2 + T B 2 + T C 2 + T D2 + 4P T 2
. Napomena. Ovaj zadatak predstavlja Lajbnicovu teoremu u geometriji cetvorougla.
Zadatkom Lajbnicova teorema bice generisana na proizvoljne poligone.
307. Ako je ABCD tetivan cetvorougao sa upravnim dijagonalama, a r poluprecnik opisanog kruga dokazati da je
AB 2 + CD2 = BC 2 + AD2 = 4r2 .

C
D

S
T

Resenje:
Ako obelezimo sa T teziste, sa S presek dijagonala i sa O srediste opisanog
kruga cetvorougla ABCD, prema Lajbnicovoj teoremi, bice
SA2 + SB 2 + SC 2 + SD2 = T A2 + T B 2 + T C 2 + T D2 + 4ST 2
OA2 + OB 2 + OC 2 + OD2 = T A2 + T B 2 + T C 2 + T D2 + 4OT 2
. Iz ovih dveju relacija u jednakosti SA2 + SB 2 = AB 2 , SC 2 + SD2 = CD2 ,
ST = OT , OA = OB = OC = OD = r sledi da je AB 2 + CD2 = 4r2 . Isto tako
je BC 2 + AD2 = 4r2 .

225

308. Ako su a, b, c stranice trougla ABC, O i r srediste i poluprecnik opisanog kruga, a T i H teziste i ortocentar tog trougla, dokazati da je
(a)

1
OT 2 = r2 (a2 + b2 + c2 );
9

(b)
OH 2 = 9r2 (a2 + b2 + c2 );
(v)

4
T H 2 = 4r2 (a2 + b2 + c2 );
9

(g)
AH 2 + BH 2 + CH 2 = 12r2 (a2 + b2 + c2 ).
Resenje:
Uput. (a) Prema Lajbnicovoj teoremi OA2 +OB 2 +OC 2 = T A2 +T B 2 +T C 2 +
1
3OT 2 , tj. OT 2 = r2 (T A2 + T B 2 + T C 2 ) odakle, s obzirom na zadatek 288,
3
nalazimo da je
1
OT 2 = r2 (a2 + b2 + c2 )
9
.
(b) Prema Ojlerovoj teoremi je OH = 3OT , pa iz prethodnog dela nalazimo da
je OH 2 = 9r2 (a2 + b2 + c2 ).
(v) Prema Ojlerovoj teoremi je T H = 2OT , pa iz (a) nalazimo da je T H 2 =
4
4r2 (a2 + b2 + c2 ).
9
(g) Prema Lajbnicovoj teoremi je AH 2 +BH 2 +CH 2 = AT 2 +BT 2 +CT 2 +3T H,
1
odakle s obzirom na jednakosti AT 2 + BT 2 + CT 2 = (a2 + b2 + c2 ) i T H 2 =
3
4 2
2
2
2
2
2
4r (a + b + c ) dobijamo da je AH + BH + CH 2 = 12r2 (a2 + b2 + c2 ).
9
309. Ako obelezimo sa O, H, T srediste opisanog kruga, ortocentar i teziste
trougla ABC, sa p njegov poluobim sa r poluprecnik opisanog kruga i sa poluprecnik upisanog kruga, dokazati da je
(a)
OT 2 =

1 2
(9r 2p2 + 22 + 8r);
9

(b)
OH 2 = 9r2 2p2 + 22 + 8r;
226

(v)
HT 2 =

4 2
(9r 2p2 + 22 + 8r);
9

(g)
AH 2 + BH 2 + CH 2 = 12r2 2p2 + 22 + 8r.
Resenje:
Ako obelezimo sa a,b,c stranice trougla ABC, prema prethodnom zadatku i za1
1
datku 263 nalazimo da je (a) OT 2 = r2 (a2 +b2 +c2 ) = (9r2 2p2 +22 +8r);
9
9
(b) OH 2 = 9r2 (a2 + b2 + c2 ) = 9r2 2p2 + 22 + 8r;
4
4
(v) HT 2 = 4r2 (a2 + b2 + c2 ) = (9r2 2p2 + 22 + 8r);
9
9
(g) AH 2 + BH 2 + CH 2 = 12r2 (a2 + b2 + c2 ) = 12r2 2p2 + 22 + 8r;
310. Ako su S, Sa , Sb , Sc sredicta i , a , b , c poluprecnici upisanih krugova trougla ABC, a a, b, c stranice, p poluobim, r poluprecnik opisanog kruga
i T teziste tog trougla, dokazati da je
(a)
9T S 2 = p2 + 52 16r;
(b)
9T Sa2 = p2 2 + 62a 4r + 12ra ;
(v)

4
T S 2 + T Sa2 + T Sb2 + T Sc2 = 16r2 (a2 + b2 + c2 ).
9

Resenje:
(a) Primenom Lajbnicove teoreme, nalazimo da je AS 2 + BS 2 + CS 2 = AT 2 +
BT 2 + CT 2 + 3T S 2 . Otuda, koristeci zadatke 268,288, i 261, dobijamo da je
9T S 2 = p2 + 52 16r. (b)Primenom Lajbnicove teoreme, nalazimo da je
ASa2 + BSa2 + CSa2 = AT 2 + BT 2 + CT 2 + 3T Sa2 . Otuda koristeci zadatatke
268,288 i 261, dobijamo da je 9T Sa2 = p2 2 + 62a 4r + 12ra . (v) Koristeci
prethodna dva dela ovog zadatka, nalazimo da je T S 2 + T Sa2 + T Sb2 + T Sc2 =
1 2
[4p + 22 28r + 6(2a + 2b + 2c ) + 12r(a + b + c )], odakle s obzirom na
9
zadatke 262, 263 i 154, dobijamo da je
8
4
T S 2 + T Sa2 + T Sb2 + T Sc2 = 16r2 (p2 2 4r) = 16r2 (a2 + b2 + c2 )
9
9
227

311. Ako su S, Sa , Sb , Sc sredista i , a , b , c poluprecnici upisanih krugova trougla ABC, a a, b, c stranice, p poluobim, r poluprecnik opisanog kruga,
T teziste i H ortocentar tog trougla, dokazati da je
(a)
HS 2 = 4r2 + 4r + 32 p2 ;
(b)
HSa2 = 4r2 + 4r + 2 + 22a p2 ;
(v)
HS 2 + HSa2 + HSb2 + HSc2 = 48r2 4(a2 + b2 + c2 ).
Resenje:
(a) Primenom Apolonijeve teoreme nalazimo da je HS 2 + 2OS 2 = 3T S 2 +
HT 2 + 2OT 2 , odakle, s obzirom na zadatke 271, 310 i 309, dobijamo da je
HS 2 = 4r2 + 4r + 32 p2 .
(b) Primenom Apolonijeve teoreme nalazimo da je HSa2 + 2OSa2 = 3T Sa2 +
HT 2 + 2OT 2 , odakle, s obzirom na zadatke 271, 310 i 309, dobijamo da je
HSa2 = 4r2 + 4r + 2 + 2(()a )2 p2 .
(v) Koristeci prethodna dva dela ovog zadatka, nalazimo da je
HS 2 + HSa2 + HSc2 = 16r2 + 62 + 16r 4p2 + 2(2a + 2b + 2c ), odakle s obzirom
na zadatak 262, dobijamo da je
HS 2 + HSa2 + HSb2 + HSc2 = 48r2 + 82 32r 8p2 = 48r2 4(a2 + b2 + c2 )
.
312. Ako je O1 srediste Ojlerovog kruga, H ortocentar i r poluprecnik opisanog kruga trougla ABC, dokazati da je
(a)
O1 A2 + O1 B 2 + O1 C 2 =

1 2
(3r + a2 + b2 + c2 );
4

(b)
O1 A2 + O1 B 2 + O1 C 2 + O1 H 3 = 3r2 .

Resenje:
(a) Ako obelezimo sa O srediste opisanog kruga i sa T teziste trougla ABC,
bice tacka O1 srediste duzi OH, a tacka T izmedju tacaka O i H takva da
je HT : T O = 2 : 1. Stoga je O1 T : T O = 1 : 2. Primenom Lajbnicove
teoreme, zatim teoreme 288 i 308, nalazimo da je O1 A2 + O1 B 2 + O1 C 2 =
1
1
T A2 + T B 2 + T C 2 + 3O1 T 2 = (T A2 + T B 2 + T C 2 ) + OH 2 = (a2 + b2 +
12
3
228

1
1
[9r2 (a2 + b2 + c2 )] = (3r2 + a2 + b2 + c2 ).
12
4
(b) Prema izvedenom delu ovog zadatka i zadacima 153 i 303, nalazimo da je

c2 ) +

1
1
1
O1 A2 +O1 B 2 +O1 C 2 +O1 H 2 = (O1 A2 +O1 B 2 +O1 C 2 )+ OH 2 = (3r2 +a2 +b2 +c2 )+ [9r2 (a2 +b2 +c2 )] = 3r2
4
4
4
.

4.3

Karnoova teorema i njena primena

313. (L. Carnot) Ako su P , Q, R, tacke pravih koje su odredjene stranicama


BC, CA, AB trougla ABC, dokazati da se prave upravne u tackama P , Q, R
na pravama BC, CA, AB seku u jednoj tacki ako i samo ako je
BP 2 P C 2 + CQ2 QA2 + AR2 RB 2 = 0.

A
Q
R
O

Resenje:
Najpre dokazimo da je navedeni uslov potreban. Normale na BC, CA, AB u
tackama P , W , R se seku, ali se u opstem slucaju ne seku u jdnoj tacki. Ako se
seku u jednoj tacki O bice BP 2 P C 2 = OB 2 OC 2 , CQ2 QA2 = OC 2 OA2 ,
AR2 RB 2 = OA2 OB 2 , te sabiranjem odgovarajucih strana dobijamo
BP 2 P C 2 + CQ2 QA2 + AR2 RB 2 = 0

(16)

Dokazimo sad da je taj uslov dovoljan. Drugim recima, pretpostevljajuci da je


uslov (1) zadovoljen, dokazimo da se normale u tackama P , Q, R na pravama
BC, CA, AB seku u jednoj tack. Neka je O presek normala u tackama P i Q
na pravama BC i CA, a R podnozje upravne kroz O na pravoj AB. Tacke R i
R su istovetne. Zaista, prema dokaznom delu ove teoreme bice
BP 2 P C 2 + CQ2 QA2 + AR2 R B 2 = 0
229

(17)

Iz (1) i (2) nalazimo da je AR2 RB 2 = AR2 R B 2 tj. (AR + RB)(AR RB) =


(AR + R B)(AR R B), odnosno AB(AR RB) = AB(AR R B). Otuda
je (AR AR ) + (R B RB) = 0, tj. RR + RR = 0, pa je RR = 0 i tacka
R,R su istovetne.
314. Ako obelezimo sa O proizvoljnu tacku koja se nalazi u ravni poligona A1 , . . . , An i sa P1 , . . . , Pn podnozja upravnih iz tacke O na pravama
A1 A2 , . . . , An A1 , dokazati da je
A1 P12 + A2 P22 + . . . + An Pn2 = P1 A22 + P2 A23 + . . . + Pn A21 .

Resenje:
Iz pravouglovih trouglova OA1 P1 i OA2 P1 , . . . , OAn P1 , . . . , OAn Pn i
OA1 Pn nalazimo da je OA21 A1 P12 = OA22 , . . . , OA2n An Pn OA21 Pn A21 .
Sabiranjem odgovarajucih strana ovih jednakosti dobijamo da je
A1 P12 + A2 P22 + ... + An Pn2 = P1 A22 + P2 A23 + ... + Pn A21
.
315. Primenom Karnoove teoreme dokazati da se simetrale stranica trougla
seku u jednoj tacki.
Resenje:
Uputstvo. Dokaz neposredan.
316. Primenom Karnoove teoreme dokazati da se prave odredene visinama
trougla seku u jednoj tacki, ortocentru tog trougla.
uputstvo:
Ako su P , Q, R podnozja visina iz temena A, B, C trougla ABC, bice
BP 2 P C 2 = AB 2 AC 2 ,
CQ2 QA2 = BC 2 AB 2 ,
AR2 RB 2 = AC 2 BC 2 .
Sabiranjem odgovarajucih strana ovih jednakosti nalazimo da je
BP 2 P C 2 + CQ2 QA2 + AR2 RB 2 = 0,
te se prema Karnoovoj teoremi prave odredene visinama trougla seku u jednoj
tacki (Slika 213).

230

Q
R
H

317. Dokazati da se normale kroz sredista spolja upisanih krugova trougla


na odgovarajucim stranicama seku u jednoj tacki.
uputstvo:
Podnozja normala kroz sredista spolja upisanih krugova na odgovarajucim stranicama trougla ABC su tacke Pa , Qb , Rc u kojima spolja upisani krugovi
dodiruju stranice BC, CA, AB. Prema zadatku 135 je:
BPa = p c, Pa C = p b, CQb = p a,
Qb A = p c, ARc = p b, Rc B = p a,
te je
BPa2 Pa C 2 + CQ2b Qb A2 + ARc2 Rc B 2 = 0
i prema Karnoovoj teoremi pomenute prave seku se u jednoj tacki.

318. Od tri kruga kojima sredista nisu na jednoj pravoj svaka dva kruga se
seku. Dokazati da se prave odredene zajednickim tetivama tih krugova seku u
jednoj tacki, radikalnom sredistu tih krugova.
uputstvo:
Neka su S1 , S2 , S3 sredista, a r1 , r2 , r3 poluprecnici takvih krugova k1 , k2 , k3 .
Prave s1 , s2 , s3 odredene zajednickim tetivama krugova k2 i k3 , k3 i k1 , k1 i k2
upravne su na pravama S2 S3 , S3 S1 , S1 S2 u tackama P , Q, R pri cemu je
S2 P 2 P S32 = r22 r32

S3 Q2 QS12 = r32 r12


pa je

S1 R2 RS22 = r12 r22

S2 P 2 P S32 + S3 Q2 QS12 + S1 R2 RS22 = 0

Otuda se prema Karnoovoj teoremi prave odredene zajednickim tetivama krugova k1 , k2 , k3 seku u jednoj tacki.

231

319. Ako su A , B , C podnozja visina iz temena A, B, C trougla ABC,


dokazati da se normale kroz temena A, B, C na pravama B C , C A , A B seku
u jednoj tacki, sredistu O kruga opisanog oko trougla ABC.
uputstvo:
Neka su P , Q, R podnozja normala iz tacaka A, B, C na pravama B C , C A ,
A B . Prave AA , BB , CC seku se u jednoj tacki i upravne su na stranicama
BC, CA, AB u tackama A , B , C pa je
BA2 A C 2 + CB 2 B A2 + AC 2 C B 2 = 0
Kako je
B A2 AC 2 = B P 2 P C 2

C B 2 BA2 = C Q2 QA2
A C 2 CB 2 = A R2 RB 2

bice i
B P 2 P C 2 + C Q2 QA2 + A R2 RB 2 = 0,
te se prema Karnoovoj teoremi prave AP, BQ, CR seku u jednoj tacki
(Slika 214).

H
R
Q

O
C

320. Ako je O proizvoljna tacka ravni trougla ABC a A , B , C podnozja


upravnih kroz O na pravama BC, CA, AB, dokazati da se normale kroz temena
A, B, C na pravama B C , C A , A B seku u jednoj tacki.
uputstvo:
Predhodni zadatak je specijalan slucaj ovog opstijeg stava koji se dokazuje na
isti nacin.
321. Ako su A , B , C upravne projekcije temena A, B, C trougla ABC
na nekoj pravoj p, dokazati da se prave kroz tacke A , B , C upravne na pravama BC, CA, AB seku u izvesnoj tacki P , ortopolu prave p u odnosu na
232

trougao ABC.
uputstvo:
Ako obelezimo sa A , B , C podnozja upravnih iz tacaka A , B , C na pravama BC, CA, AB; bice
BA2 A C 2 = AB 2 A C 2 = BB 2 + B A2 CC 2 A C 2
CB 2 B A2 = CB 2 B A2 = CC 2 + B C 2 AA2 A B 2

Sabiranjem odgovarajucih strana ovih jednakosti nalazimo da je

BA2 A C 2 + CB 2 B A2 + AC 2 C B 2 = 0
Stoga se prema Karnoovoj teoremi prave A A , B B , C C seku u jednoj tacki
P , koju nazivamo ortopolom prave p u odnosu na trougao ABC (Slika 215).
A

P
B
A
C

C p

322. Dva trougla ABC i A B C pripadaju istoj ravni. Ako se upravne


kroz tacke A, B, C na pravama B C , C A , A B seku u jednoj tacki, dokazati
da se i upravne tacke A , B , C na pravama BC, CA, AB takode seku u jednoj
tacki.

uputstvo:
Neka su P , Q , R podnozja upravnih kroz temena A, B, C na pravama B C ,
C A , A B i P , Q, R podnozja upravnih kroz temena A , B , C na pravama
BC, CA, AB. Pri tome je
(BP 2 P C 2 ) + (CQ2 QA2 ) + (AR2 RB 2 ) =
(BA2 A C 2 ) + (CB 2 B A2 ) + (AC 2 C B 2 ) =

(B A2 AC 2 ) (C B 2 BA2 ) (A C 2 CB 2 ) =

(B P 2 P C 2 ) (C Q2 Q A2 ) (A R2 R B 2 ) = 0

Stoga se prema Karnoovoj teoremi upravne kroz temena A , B , C na pravama


BC, CA, AB seku u jednoj tacki (Slika 216).
233

A
Q
R
Q
O

O
R

P
B
B
P
C

Napomena: Dva trougla ABC i A B C kod kojih vazi osobina navedena


u zadatku 322 nazivamo ortologickim trouglovima.

4.4

Cevijeva
teorema i njena primena

323.(Giovanni Ceva) Ako su P , Q, R tacke pravih koje su odredene stranicama BC, CA, AB trougla ABC, dokazati da se prave AP, BQ, CR seku u
jednoj tacki ako i samo ako je
BP CQ AR

= 1.
P C QA RB
uputstvo:
Najpre dokazimo da je navedeni uslov potreban. Prave AP , BQ, CR u opstem
slucaju se ne seku u jednoj tacki. Ako se seku u nekoj tacki S i ako obelezimo
sa Q i R tacke u kojima prave BQ iCR seku pravu kroz A uporednu sa BC
bice
Q A
BP
=
PC
AR
CQ
BC
=
QA
AQ
AR
AR
,
=
RB
BC

234

pa je

BP CQ ar

=1
P C QA rb

(18)

Sada dokazimo da je taj uslov i dovoljan. Naime, pretpostavljajuci da je


uslov (1) zadovoljen, dokazimo da se prave AP , BQ, CR seku u jednoj tacki.
Neka je S presecna tacka pravih BQ i CR, a P tacka u kojoj prava AS sece
pravu BC. Tacke P i P su istovetne. Zista, prema dokazanom delu ove teoreme,
bice
bp cq ar

=1
(19)
p c qa rb
iz (1) i (2) nalazimo da je

BP
BP
=
PC
P C

te su tacke P i P istovetne i prema tome prave AP , BQ, CR su konkurentne


(Slika 217).

324. Primenom Cevijeve


teoreme dokazati da se:
(a) tezisne linije trougla seku u jednoj tacki;
(b) prave odredene visinama trougla seku u jednoj tacki;
(c) simetrale unutrasnjih uglova trougla seku u jednoj tacki;
(d) simetrale jednog unutrasnjeg ugla i simetrale dva spoljasnja ugla kod
druga dva temena trougla seku u jednoj tacki.
uputstvo:
(a) Dokaz se izvodi neposredno.
(b) Ako su AA1 , BB1 , CC1 visine trougla ABC, bice
ABA1 CBC1
BCB1 ACA1
235

CAC1 BAB1
pa je

AB
BA1
=
C1 B
BC
CB1
BC
=
A1 C
CA
AC1
CA
=
B1 A
AB

Otuda je

BA1 CB1 AC1

=1
A1 C B1 A C1 B

te se prema Cevijeovoj
teoremi prave odredene visinama trougla seku u jednoj
tacki.
(c) Ako su A , B , C tacke u kojima simetrale unutrasnjih uglova trougla ABC seku naspramne stranice, bice
BA
BA
=
A C
AC
CB
CB
=
BA
BA

AC
AC
=
C B
CB
Otuda je

BA CB AC

=1
A C B A C B

te se prema Cevijeovoj
teoremi simetrale unutrasnjih uglova trougla seku u jednoj tacki.
(d) Postupak isti kao u predhodnom delu ovog zadatka.
325. Ako su P, Q, R tacke u kojima krug upisan u trougao ABC dodiruje
stranice BC, CA, CB, dokazati da se prave AP, BQ, CR seku u jednoj tacki

(Zergonova
teorema).
uputstvo:
Kako je AQ = AR, BR = BP, CP = CQ bice
BP CQ AR

= 1.
P C QA RB

Otuda se prema Cevijevoj


teoremi prave AP, BQ, CR seku u jednoj tacki,

Zergonovoj
tacki trougla ABC (Slika 218).

236

R
O

326. Ako su Pa , Qa , Ra tacke u kojima krug upisan u trougao ABC


dodiruje stranicu BC i produzenja stranica CA i AB, dokazati da se prave
APa , BQa , CRa seku u jednoj tacki.
uputstvo:
Kao u prethodnom zadatku,bice:
AQa = ARa , BRa = BPa , CPa = CQa
pa je

BPa CQa ARa

= 1.
Pa C Qa A Ra B

Otuda se u Cevijevoj
teoremi prave
APa , BQa , CRa
seku u jednoj tacki (Slika 219).

237

Pa

Qa

Ra

Sa

327. Ako su Pa , Qb , Rc tacke u kojima spolja upisani krugovi trougla ABC


dodiruju stranice BC, CA, AB dokazati da se prave APa , BQb , CRc seku u jednoj tacki.
uputstvo:
Prema zadatku 135 bice
BRc = CQb , CPa = ARb , AQb = BPa
pa je

BPa CQb ARc

= 1.
Pa C Qb A Rc B

Otuda se prema Cevijevoj


teoremi prave
AP1 , BQ2 , CR3
seku u jednoj tacki, Nagelovoj tacki trougla ABC (Slika 220).

238

Sb

kb

A
Sc
kc

Rc
O

Qb
C

Pa

Sa

ka

sl.220
328. Neka je ABC proizvoljan trougao i k krug koji sece prave BC, CA, AB
u tackama P i P , Q i Q , R i R . Ako se pri tome prave AP, BQ, CR seku u
jednoj tacki, dokazati da se i prave AP , BQ , CR takode seku u jednoj tacki
(Slika 221).
uputstvo:
Prema poznatom stavu bice

BP BP = RB R B, CQ CQ = P C P C, AR AR = QA Q A
pa je

BP BP
CQ CQ
AR AR
=1
=1
=1
RB R B
P C P C
QA Q A

Mnozenjem odgovarajucih strana tih jednakosti nalazimo da je

BP CQ AR BP CQ AR

= 1.
P C QA RB P C Q A R B

Po pretpostavci prave AP, BQ, CR seku se u jednoj tacki, te se prema Cevijevoj

teoremi i prave AP , BQ , CR seku u jednoj tacki (Slika 221).

239

Q
R

R
Q

329. Dokazati da se prave od kojih svaka sadrzi po jedno teme trougla i


razlaze obim tog trougla na dva jednaka dela, seku u jednoj tacki.
uputstvo:
Ako prave kroz temena A, B, C trougla ABC polove obim tog trogla,one seku
naspramne stranice BC, CA, AB tog trougla u tackama Pa , Qa , Ra u kojima
spolja upisani krugovi dodiruju stranice BC, CA, AB pa se prema prethodnom
zadatku pomenute prave APa , BQb , CRc seku u jednoj tacki.

330. Ako je P srediste stranice BC trougla ABC i ako su Q i R tacke u


kojima neka prava uporedna sa stranicom BC sece prave AC i AB, dokazati da
se prave AP , BQ, CR seku u jednoj tacki.
uputstvo:
S obzirom da je
RP = P C
i

bice

BR
CQ
=
QA
RA
BP CQ AR

= 1.
P C QA RB

Otuda se prema Cevijevoj


teoremi prave AP, BQ, CR seku u jednoj tacki
(Slika 222).

240

331. Neka je ABC proizvoljan trougao i neka su P , Q, R tacke pravih


BC, CA, AB takve da se prave AP , BQ, CR seku u jednoj tacki. Ako su A ,
B , C sredista duzi AP , BQ, CR, dokazati da se prave A P , B Q , C R seku
u jednoj tacki.
Uputstvo.
A
R

B
P
R

A P

Slika 223

S obzirom da su tacke P , Q , R na srednjim linijama

B C ,C A ,A B

trougla ABC,bice

C P CQ
A Q AR
BR
BP
= ,
= ,
=
PC
P B QA
A C RB
RA
241

Mnozenjem odovarajucih strana ovih jednakosti nalazimo da je

CP BR AQ
BP CQ AR
=
P C QA RB
P B RA QC

Po pretpostavci ,prave AP,BQ,CR seku se u jednoj tacki.,te je prema Cevijevoj


teoremi
BP CQ AR
= 1.
P C QA RB
Otuda je i

CP BR AQ
= 1,
P B R A Q C

pa se prema Cevijevoj
teoremi i prave

A P ,B Q ,C R

seku u jednoj tacki(sl.223).


332. Neka su A , B , C sredista stranica BC, CA, AB trougla ABC. P ,
Q, R tacke pravih BC, CA, AB i P , Q , R tacke simetricne sa P , Q, R u
odnosu na na A , B , C . Ako se pri tome prave AP , BQ, CR seku u jednoj
tacki, dokazati da se i prave AP , BQ , CR takode seku u jednoj tacki.
Uputstvo.

A
Q

R
S
C

B
Q

R
S
B

Slika 224

S obzirom da se prave AP,BQ,CR seku u jednoj tacki,prema Cevijevoj


teoremi
bice
BP CQ QR
= 1...(1)
P C QA RB
Tacke

P ,Q ,R

su simetricne s tackama P,Q,R u odnosu na sredista A,B,C stranica BC,CA,AB


pa je

BP
CP CQ
AQ AR
AR
= ,
= ,
= ...(2)
PC
P B QA
Q C RB
RB
242

Iz jednakosti (1)i (2) nalazimo da je

BP CQ AR
= 1,
P C Q A R B

pa se prema Cevijevoj
teoremi prave

AP , BQ , CR

seku u jednoj tacki(sl.224).


333. Neka su AA , BB , CC simetrale unutrasnjih uglova trougla ABC,
a P i P , Q i Q , R i R tacke pravih BC, CA, AB takve da su prave AP ,
BQ , CR simetricne sa pravama AP , BQ, CR u odnosu na prave AA , BB ,
CC . Ako se pri tome prave AP , BQ, CR seku u jednoj tacki S, dokazati da
se i prave AP , BQ , CR takode seku u jednoj tacki S . Dokazati zatim da
upravne projekcije tacaka S i S na pravama BC, CA, AB pripadaju jednom
krugu.
Uputstvo.
A
Q

Q
R
S
B

Slika 225
Obelezimo sa 1 i2 uglove koje poluprava AP zahvata s polupravama AB i AC
,sa 1 i2 uglove koje poluprava BQ zahvata s polupravama BC i BA,sa 1 i2
uglove koje poluprava CR zahvata sa polupravama CA i CB, a sa a,b,c starnice
BC,CA,AB trogla ABC. Pri tome je
c sin 1 CQ
a sin 1 AR
b sin 1
BP
,
,
...(1)
=
=
=
PC
b sin 2 QA
c sin 2 RB
a sin 2

S obzirom da su prave AP , BQ , CR simetrale s pravama AP,BQ,CR u

odnosu na prave AA , BB , CC bice

BP
c sin 2 CQ
a sin 2 AR
b sin 2
,
,
...(2)
=
=
=
P C
b sin 1 Q A
c sin 1 R B
a sin 1
Mnozenjem odgovarajucih strana jednakosti (1)i(2) nalazimo da je

BP CQ AR BP CQ AR
= 1.
P C QA RB P C QA R B
243


Po pretpostvaci prave AP,BQ,CR seku se u jednoj tacki,pa je prema Cevijevoj
teoremi
BP CQ AR
= 1.
P C QA RB
Otuda je

BP CQ AR
= 1,
P C Q A R B

pa se prema Cevijevoj
teoremi i prave AP , BQ , CR seku u jednoj tacki. Sada

treba da dokazemo drugi deo stava.Ako saLiL , M iM , N iN obelezimo upravne

projekcije tacaka SiS na pravama BC,CA,AB bice

ASM AS N iASN simAS M


pa je

AM
AS AN
AS
=
i
=
.
AN
AS AM
AS

Otuda je

AN
AM
=
AN
AM

tj.

AM AM = AN AN ,

pa se tacke M, M , N, N nalaze na jednom krugu kome je srediste presek sime

trala tetivaM M iN N ,dakle srediste duzi SS .Istim postupkom se dokazuje se

da i tacke L, L pripadaju tom krugu(sl.225).


334. Neka je A1 . . . A2n1 ravan mnogougao s neparnim brojem stranica i S
proizvoljna tacka njegove ravni. Ako prave SA1 , . . . , SA2n1 odredene tackom S
i temenima mnogougla seku prave odredene naspramnim stranicama u tackama
Pn , Pn+1 , . . . , P2n1 , . . . , Pn1 , dokazati da je
A1 P1 A2 P2 A2n1 P2n1

= 1.
P1 A2 P2 A3
P2n1 A1

Uputstvo.
A5
P5

P4

A6

A4

P6

P3
S

A7

A3
P2
P7
A1

P1

A2

Slika 226
244

Iz parova trouglova
SA1 P1 iSA2 P1 , ..., SA2n1 P2n1 iSA1 P2n1
(sl.226),imamo da je
SA1 sin (A1 SP1 )
SA2n1 sin (A2n1 SP2n1 )
A2n1 P2n1
A1 P1
=
=
, ....,
P1 A2
SA2 sin (P1 SA2 )
P2n1 A1
SA1 sin (P2n1 SA1 )
gde navedene uglove smatramo pozitivnim ako imaju isti smer , a negativnim
ako imaju suprotan smer.Mnozenjem odgovarajucih strana dobijenih jednakosti
i vodeci racuna da je
(A1 SP1 ) = (Pn SAn+1 )
nalazimo da je

4.5

A1 P1 A2 P2 A2n1 P2n1
....
= 1.
P1 A2 P2 A3
P2n1 A1

Menelajeva teorema i njena primena

335. ( Menelaus) Dokazati da tacke P , Q, R pravih koje su odredene


stranicama BC, CA, AB trougla ABC pripadaju jednoj pravoj ako i samo ako
je
BP CQ AR

= 1.
P C QA RB
Uputstvo.

R
A
Q
C
P

Slika 227
Najpre dokazimo da je navedeni uslov potreban.U opstem slucaju tacke P,Q,R ne

pripadaju jednoj pravoj.Ako one pripadaju izvesnoj pravoj s i ako sa A , B , C


obelezimo projekcije tacaka A,B,C na pravoj s,bice

BP
BB CQ
CC AR
AA
=
=
=
,
,
PC
CC QA
AA RB
BB
pa je

BP CQ AR
= 1...(1)
P C QA RB
245

Sad dokazimo da je taj uslov i dovoljan.Naime,pretpostavljajuci da je uslov


(1) zadovoljen,dokazimo da tacke P,Q,R pripadaju jednoj pravoj.Neka je a prava

odredjena tackaam Q i R ,a P njen presek sa pravom BC.Tacke P iP su negativne.Zaista,prema dokazanom delu ove teoreme bice

CQ AR
BP

= 1...(2)

P C QA RB
Iz(1) i (2) nalazimo da je

BP
BP
= ,
PC
P C

pa su tacke P iP istovetne i prema tome tacke P,Q,R su kolinearne.(sl.227).


336. Dokazati da tacke P , Q, R u kojima simetrale spoljasnjeg ugla A i
unutrasnjih uglova B i C seku prave odredene naspramnim stranicama trougla
ABC, pripadaju jednoj pravoj.
Uputstvo.

A
Q
R

Slika 228
S obzirom da je
BP
AB CQ
BC AR
AC
=
,
=
,
=
,
PC
AC QA
AB RB
BC
bice

BP SQ AR
= 1,
P C QA RB

pa su tacke P,Q,R na jednoj pravoj(sl.228).


337. Dokazati da tacke P , Q, R u kojima simetrale spoljasnjih uglova A, B,
C seku prave odredene naspramnim stranicama trougla ABC, pripadaju jednoj
pravoj.
Uputstvo.

246

Q
A
P
B

Slika 229
S obzirom da je
BP
AB CQ
BC AB
AC
=
,
=
,
=
,
PC
AC QA
AB RB
BC
bice

BP CQ AR
= 1,
P C QA RB

pa su tacke P,Q,R na jednoj pravoj(sl.229).


338. Dokazati da tacke P , Q, R u kojima dirke kruga opisanog oko trougla ABC u njegovim temenima seku prave odredene naspramnim stranicama,
pripadaju jednoj pravoj, Menelajevoj pravoj trougla ABC.
Uputstvo.

Q
A
P

Slika 230
AP
AB
=
PC
AC
,tj. da je

AB 2
AP 2
=
2
CP
BC 2

.Otuda je
AP 2 = BP CP
247

sledi da je

.Isto tako je

BP
AB 2
=
PC
AC 2
CQ
BC 2 AR
AC 2
i
=
=
QA
AB 2 RB
BC 2

.Kako jsu tacke P,Q,R na produzenjima stranica BC,CA,AB,iz izvedenih proprcija nalazimo da
BP CQ AR

= 1
P C QA RS
,pa su tacke P,Q,R na jednoj pravoj(sl.230).
339. Dokazati da su kod trougla srediste jedne visine, dodirna tacka odgovarajuce stranice sa spolja upisanim krugom i srediste upisanog kruga tri
kolinearne tacke.
Uputstvo.

K
S
B

C
P E Pa

Slika 231
tacka u kojoj simetrala ugla A sece stranicu BC,K srediste visine AD ,Pa tacka
u kojoj stranica BC dodiruje spolja upisani krug i srediste S upisanog kruga,bice
tacke K, Pa , Sna pravama koje su odredene stranicama AD,DE,EA trogla ADE
takve da je
Stoga su prema Menelajevoj teoremi tacke K, Pa , S kolinerane(sl.231).
340. Dokazati da su kod trougla srediste jedne visine, dodirna tacka odgovarajuce stranice sa upisanim krugom i srediste spolja upisanog kruga koji
odgovara toj stranici tri kolinearne tacke.
Uputstvo.

248

A
K
S
B

D
P
E

Sa

Slika 232
u kojoj simetrala ugla A sece stranicu BC,K srediste visine AD,P tacka u kojoj
stranica BC dodiruje upisani krug i Saa srediste spolja upisanog kruga koji odgovara stranici BC ,bice tacke K, P, Sa na pravama koje su odredene stranicama
AD,DE,EA trougla ADE takve da je
AK DP ESa
AB + AC
BC

= 2
KD P E Sa A
BC
AB + AC
. S toga prema Manelajevoj teoremi tacke K, P, Sa su kolinearne(sl.232).
341. Dokazati da su kod trougla srediste visine iz jednog temena, srediste
spolja upisanog kruga koji odgovara drugom temenu i dodirna tacka spolja upisanog kruga koji odgovara trecem temenu, sa pravom koja sadrzi stranicu odgovarajucu s pomenutom visinom, tri kolinearne tacke.
Uputstvo.

A
Sc
F

B D

Pb

Slika 233
u kojoj simetrala spoljasnjeg ugla A sece pravu BC , K srediste visine AD ,Sc
srediste spolja upisanog kruga koji odgovara temenu C i Pb tacka u kojoj spolja
upisani krug koji odgovara temenu B dodiruje pravu BC ,bice tacke K, Pa , Ab
na pravama koje su odredene stranicama AD,DF,FA trougla takve da je
AK DPb F Sc
BC
AC AB

= 1
KD Pb F Sc A
AC AB
BC
.Stoga su prema Manelajevoj teoremi tacke K, Pb , Sc kolinearne(sl.233).
342. Ako je O srediste opisanog kruga trougla ABC, M tacka simetricna s
ortocentrom H tog trougla u odnosu na teme A i N tacka simetricna s temenom
249

A u odnosu na srediste D stranice BC, dokazati da tacke O, M, N pripadaju


jednoj pravoj.
Uputstvo.

M
A
B
C H T O
B A D

C
N

Slika 234
isticemo drugi.Ako obelezimo sa T teziste trougla ABC ,bice tacke O,M,N na
pravama koje su odredene stranicama HT,TA,AH trougla AHT takve da je
HO

3 TN
2 AM
1
= ,
= ,
=
1 NA
3 MN
2

pa je

HO T N AM

= 1
OT N A M H
S toga prema Manelajevoj teoremi tacke O,M,N pripadaju jednoj pravoj(sl.234).
343. Ako su P , Q, R tacke stranica BC, CA, AB trougla ABC takve da
je BP : P C = CQ : QA = AR : RB i P tacka u kojoj se seku prave BC i P Q,
dokazati da je
BP : CP = CP 2 : BP 2 .

Uputstvo.

A
Q
R
P B

Slika 235
Prema Menelajevoj teoremi imamo da je

BP
CQ AR

= 1.
P C QA RB
250

No

AR
BP
CQ
=
=
,
QA
RB
PC

te je

CQ AR
BP 2

=
,
QA RB
PC
i prema tome

BP
CP 2
=
CP
BP 2
(sl.235).
344. Ako su A , B , C sredista stranica BC, CA, CB, M presek duzi AA
i BC , a N presek pravih AB i CM , dokazati da je
AB = 3 AN.

Uputstvo.

A
N
C

Slika 236
S obzirom da su O,M,N tacke u kojima jedna prava sece prave odredene starnicama

AB, B C , C A
trogla

AB C ,
prema Ma nelajevoj teoremi imamo da je

AC B M C N

= 1.
CB M C N A
Medutim,

AC
BM
= 2i
= 1,
CB
M C
pa je

CN
1
= ,
NA
2
251

i prema tome AB=3AN(sl.236).


345. Ako su M i N tacke na stranicama AB i AC trougla ABC takve da je
AM = AN a D tacka u kojoj tezisna linija AD trougla sece duz M N , dokazati
da je
M D : D N = AC : AB.

Uputstvo.

A
B
M

NC

Slika 237

Ako obelezimo sa B iC tacke u kojima prava kroz tacku D uporedna sa stranicom BC sece stranice MB i AC ,imamo da je

BD : DC = B D : D C .
S obzirom da je BD=DC,bice i

B D =DC .

Primenom Menelajeve teoreme na secicu MN trougla AB C ,iamamo da je

BD
C N AM

= 1,


DC
CA M B

C N
pa je MB
cicu B C trogla
= 1.Najzad ,primenom Menelajeve teoreme na se
AMN ,nalazimo da je

M D N C AB
= 1

DN CABM
pa je

AB
MD

= 1,
D N C A
i prema tome

CA
AC
MD
=
=
D N
AB
AB
(sl.237).

252

346. Ako su P , Q, R tacke u kojima neka prava sece prave odredene stranicama BC, CA, AB trougla ABC, dokazati da njima simetricne tacke P , Q ,
R u odnosu na sredista stranica BC, CA, AB pripadaju jednoj pravoj.
Uputstvo.

A
Q
R
C
R

B
Q

Slika 346
Primenom Menelajeve teoreme nalazimo da je
BP CQ AR

= 1.
P C QP RB
Otuda i iz jednakosti

BP
CP
CQ
AQ AR
BR
= =
= ,
=
PC
P B
QA
Q C RB
RA
sledi da je

BR AQ
CP

= 1,
P C R A Q C
pa su tacke

P ,Q ,R
na jednoj pravoj(slika 346).

253

347. Ako dve prave s i s seku prave odredene stranicama BC, CA, CB
trougla ABC u tackama P , Q, R i P , Q , R , dokazati da presecne tacke X, Y, Z
pravih BC i QR , CA i RP , AB i P Q pripadaju jednoj pravoj.
Uputstvo.

A
Z

Y
Q

R
R
Q
P

Slika 347
Prema Menelajevoj teoremi imamo da je

BX CQ AR
BP
CY AR
BP CQ
AZ

= 1,

= 1,

= 1,
XC QA R B
P C Y A RB
P C Q A ZB
pa je s obzirom da je

BP
CQ
AR
BP CQ AR

= 1i = 1
P C QA RB
P C QA RB
bice

BX CY AZ

= 1,
XC Y A ZB
prema tome,tacke X,Y,Z pripadaju jednoj pravoj(slika 347).
348. Ako je O proizvoljna tacka ravni trougla ABC, dokazati da tacke
P , Q, R u kojima upravne u tacki O na duzima OA, OB, OC seku respektivno

254

prave BC, CA, AB pripadaju jednoj pravoj.


Uputstvo.

A
Q1

R2

P2
O
P1
B

R1 Q2

Slika 348
Obelezimo sa P1 i P2 tacke u kojima prava OP sece prava AC i AB, pa sa Q1 i
Q2 tacke u kojima prava OQ sece prave AB i BC, a sa R1 i R2 tacke u kojima
prava OR sece prave BC i CA. Prema Menalajevoj teoremi imamo da je
BP CP1 AP2

= 1
P C P1 A P2 B
CQ AQ1 BQ2

= 1
QA Q1 B Q2 C
AR BR1 CR2

= 1
RB R1 C R2 A
pa je
(

CP1
AQ1 BQ2 BR1 CR2
BP CQ AR

)(

) = 1

P C QA RB P1 A PAPB2 Q1 B Q2 C R1 C R2 C
2

S obzirom da je

BQ2
BO OQ2
=
R1 C
CO OR1
255

CO OR2
CR2
=
P1 A
AO P1 O
AP2
AO OP2
=
Q1 B
Q1 O OB
BR1 BO OR1

Q1 C CO OQ1
CP1
CO OP1
=
R2 A
AO OR2
AO OQ1
AQ1
=
P2 D
BO OP1

bice

CP1 AP2 AQ1 BQ2 BR1 CR2

=1
P1 A P2 P Q1 B Q2 C R1 C R2 A

Otuda je

BP CQ AR

= 1
P C QA RB
pa su tacke P, Q, R, na jednoj pravoj(slika 348).

349. Ako su AD i AE tezisna linija i simetrala ugla A trougla ABC, a P ,


Q, R upravne projekcije proizvoljne tacke M prave AE na pravama BC, CA,
AB, dokazati da se prave AD, M P, QR seku u jednoj tacki.
Uputstvo.

Q
N

R
M

256

Slika 349
Dokazimo da se presecn tack N pravih AD i MP nalazi na pravoj QR.U tom

cilju obelezimo sa B iC tacke u kojima prava kroz tacku N uporedna sa stra

nicom BC sece prave AB i ACIz podudarnosti trouglova M QC iM RB sledi

da je C Q = B R.Pri tome su N,Q,R tacke pravih koje su odredene stranicama

B C , C A, AB trougla ABC,takve da je

B N = N C , AQ = ARiC Q = B R
pa je

B N C Q AR

= 1.
N C QA RB
Stoga su prema Menelajevoj teoremi tacke N,Q,R na jednoj pravoj(slika 349).
350. Ako je ABCD proizvoljan cetvorougao, E tacka u kojoj se seku prave
AB i CD, F tacka u kojoj se seku prave BC i DA, dokazati da sredista P , Q,
R duzi AC, BD, EF pripadaju jednoj pravoj.
Uputstvo.

Q
C

A
B

Slika 350
S obzirom da su C,E,D tacke u kojima jedna prava sece produzene stranice
FB,BA,AF trougla AFB ,prema Menelajevoj teoremi bic
EC BE AD

= 1.....(1).
CB EA DF
257

Ako sA

A ,B ,F

obelezimo sredista starnica BF,FA,AB trougla ABF,bice tacke P,Q,R na pravama

B P ,F A ,A B
pre cemu je

FC
B P BE A R AD F Q
=
,
,
.....(2)
CB
P F EA RB DF QA
. Iz jdnakosti (1) i (2) nalazimo da je

BP F Q AR

= 1,
P F QA RB
te prema Menelajevoj teoremi tacke P,Q,R pripadaju jednoj pravoj
351. Ako je ABCD ravan cetvorougao, E presecna tacka pravih AB i CD,
F presecna tacka pravih BC i AD, A , B , C , D tacke u kojima cetiri uporedne
prave kroz temena A, B, C, D seku pravu EF , dokazati da je
1
1
1
1
+
=
+
.
AA
CC
BB
DD

Uputstvo.
Prava CD sece stranice ili produzenje stranica AB, BF, FA ABF , pa je prema
Menelajevoj teoremi
AE
BC
FD

= 1
EB
CF
DA

258

C
C

Slika 351
S obzirom da je
AA
AE
=
EB
BB
BC
BB CC
=
CF
CC
DD
FD
=
DA
AA DD

bice

AA DD BB AA CC DD = BB CC DD + AA BB CC

Otuda je
BB DD (AA + CC ) = AA CC (BB + DD )
tj.
(AA + CC ) : AA CC = (BB + DD ) : BB DD
259

i prema tome:
1
1
1
1
+
=
+
AA
CC
BB
DD
352. Ako obelezimo sa A, B, C, D cetiri razne tacke neke prave p takve da
je AB = BC = CD, sa O proizvoljnu tacku izvan prave p i sa A , B , C , D
tacke u kojima neka prava p koja ne sadrzi tacku O sece prave OA, OB, OC,
OD, dokazati da je
AA
DD
BB
CC
+ = + .

AO
DO
BO CO
Uputstvo.
Ako je prava p paralelna sa pravom p dokaz je neposredan jer je u tom slucaju
BB
AA
=

AO
BO
i

pa je

DD
CC
=

DO
CO
DD
BB
CC
AA
+
=
+
A O
D O
BO
C O

260

D
C
B
A
S

Slika 352
Ako prava p sece pravu p u nekoj tacki S, primenom Menelajeve teoreme
nalazimo da je
BB
SA
AA
=

AO
BO
SB
DD
CC
AD
=

DO
CO
SC
CC
SB
BB
=
BO
CO
SC
Odatle je
AA
DD
BB
SA CC
AD
+ =
+
=

AO
DO
BO
SB
CO
SC

BB
AB
CC
CD
(1

)
+
(1 +
)
BO
SB
C O
SC

261

BB
CC
CC
BC
BB
BC
+
+
(

)=
BO
CO
CO
SC
BO
SB

CC
CC
BC
CC
BS
BC
BB
CC
BB
+ +(
+

)= +

BO
CO
CO
SC
CO
SC
SB
BO
CO

353. Ako su P1 , . . . , Pn tacke u kojima neka prava p sece prave odredene


stranicama A1 A2 , . . . , An A1 n-tougla A1 . . . An , dokazati da je
A1 P1 A2 P2
An Pn

...
= (1)n .
P1 A2 P2 A3
Pn A1

Uputstvo.
Obelezimo sa S tacku u kojoj prava p sece proizvoljnu pravu s koja se nalazi u
ravni n-tougla A1 ...An . Ako je A1 ...An uporedne s pravom p projekcije tacaka
A1 ...An na pravoj s, imamo da je
A1 F1
A S
= 1
P1 A2
SA2
A S
A2 P2
= 2
P2 A3
SA3
...
An Pn
A S
= n
Pn A1
SA1

262

P6

A5

P5
A6

P4
P3

A4

A3
P2

A1

A2

A1

P1

A2

A6 A4

A5A3

Slika 353
S obzirom da je broj presecnih tacaka prave p sa poligonom A1 ...An paran,
mnozenjem odgovarajucih strana dobijenih jednakosti, nalazimo da je
A1 P1
A2 P2

P1 A2
P2 A3
...
An Pn
= (1)n
Pn A1

4.6

Dezargova teorema i njena primena

354. (G. Dezargues) Neka su ABC i A B C dva trougla koji pripadaju


jednoj ravni i kojima se prave odredene odgovarajucim stranicama BC i B C ,
CA i C A , AB i A B seku u tackama P , Q, R. Ako se pri tome prave AA ,
BB , CC seku u jednoj tacki, dokazati da tacke P , Q, R pripadaju jednoj
pravoj, i obratno, ako tacke P , Q, R pripadaju jednoj pravoj, dokazati da se
prave AA , BB , CC seku u jednoj tacki.
Uputstvo.
Izvedimo najpre prvi deo stava. Neka je O presecna tacka pravih AA, BB,
CC. Prave BC, CA, AB seku prave odredene stranicama trouglova OBC,

263

OCA, OAB, pa je prema Menelajevoj teoremi


CC
OB
BP
= 1
PC
CO BB
CQ AA
OC
= 1
QA A O C C
BB
OA
AR
= 1
RB
BO
AA

B
A

Q
P

R
C
A

Slika 354
Mnozenjem odgovarajucih strana ovih jednakosti nalazimo da je
BP
CQ AR

= 1
PC
QA RB
te prema Menelajevoj teoremi tacke P, Q, R pripadaju jednoj pravoj.
Izvedimo sad drugi deo stava: neka se prave BB i CC seku u nekoj tacki O.
Dokazimo da i prava AA sadrzi tacku O. Trouglovi RBB i QCC pripadaju jednoj ravni, a prave RQ, BC, BC odredene njihovim odgovarajucim temenima
seku se u tacki P, Te prema dokazanom delu ovog stava presecne tacke O, A,
A pravih BB i CC, RB i QC, RB i QC pripadju jednoj pravoj.
355. Ako su tacke P , Q, R na pravama odredenim stranicama BC, CA, AB
trougla ABC takve da se prave AP , BQ, CR seku u jednoj tacki O, dokazati
264

da presecne tacke P , Q , R pravih BC i QR, CA i RP , AB i P K pripadaju


jednoj pravoj.
Uputstvo.

A
R

Q
O
B

Slika 355
Prave odredene odgovarajucim temenima trouglova ABC i PQR seku se u
tacki O, te prema Dezargovoj teoremi tacke P, Q, R u kojima se seku prave
BC i QR, CA i RP, AB i PQ pripadaju jednoj pravoj.
356. Ako su AA , BB , CC visine trougla ABC, dokazati da tacke P , Q,
R u kojima se seku prave BC i B C , CA i C A , AB i A B pripadaju jednoj
pravoj.
Uputstvo.

265

R
Q
A
B

Slika 356
Ortocentar E trougla ABC je srediste perspektive trouglova ABC i ABC,
pa se prema Dezargovoj teoremi tacke P, Q, R u kojima se seku prave odredene
odgovarajucim stranicama BC i BC, CA i CA, AB i AB nalaze na jednoj
pravoj.
Napomena: trougao ABC kome su temena podnozja visina trougla ABC, nazivamo ortickim trouglom, A pravu kojoj pripadaju tacke P, Q, R nazivamo
ortickom pravom trougla ABC.
357. Ako su P , Q, R tacke u kojima upisani krug dodiruje stranice BC,
CA, AB trougla ABC, dokazati da se tacke X, Y , Z u kojima se seku prave
QR i BC, RP i CA, P Q i AB nalaze na jednoj pravoj
Uputstvo.

266

G
R
P
B

Slika 357

Prema zadatku 325. ptave AP, BQ, CR seku se u jednoj tacki, Zergonovoj
tacki G trougla ABC, pa su trouglovi ABC i PQR perspektivni. Stoga se prema
Dezargovoj teoremi tacke X, Y, Z u kojima se seku prave QR i BC, RP i CA,
PQ i AB nalaze na jednoj pravoj.
358. Ako su Pa , Qb , Rc tacke u kojima spolja upisani krugovi dodiruju
stranice BC, CA, AB trougla ABC, dokazati da se tacke X, Y , Z u kojima se
seku Qb Rc i BC, Rc Pa i CA, Pa Qb i AB nalaze na jednoj pravoj.
Uputstvo.

267

Rc
N
B

Pa

Qb
X

C
Y

Slika 358
Prema zadatku 327. prave APa , BQb , CRc seku se u jednoj tacki, Nagelovoj
tacki H trogla ABC. Stoga su trouglovi Pa Qb Rc i ABC perspektivni, pa se tacke
X, Y, Z u kojima se seku prave odredene odgovarajucim stranicama Qb Rc i BC,
Rc Pa i CA, Pa Qb i AB nalaze na jednoj pravoj.
359. Dokazati da se ose perspektiva triju trouglova, perspektivnih u odnosu
na istu tacku, seku u jednoj tacki.
Uputstvo.

268

P1

P2

O
P3
A1

C1
Q1
B1

Q3 Q2

C2

A2

C3
S

B2

A3

B3

Slika 359
Obelezimo sa A1 B1 C1 , A2 B2 C2 , A3 B3 C3 tri trougla perspektivna u odnosu
na istu tacku O. Prema Dezargovoj teoremi P1 , Q1 , R1 u kojima se seku prave
B2 C2 i B3 C3 ,C2 A2 i C3 A3 ,A2 B2 i A3 B3 pripadaju nekoj pravoj s1 , osi perspektive trouglova A2 B2 C2 i A3 B3 C3 ; tacke P2 , Q2 , R2 u kojima se seku prave B3 C3
i B1 C1 , C3 A3 i C1 A1 , A3 B3 i A1 B1 pripadaju nekoj pravoj s2 , osi perspektive
trouglova A3 B3 C3 i A1 B1 C1 ; tacke P3 , Q3 , R3 u kojima se seku prave B1 C1 i
B2 C2 , C1 A1 i C2 A2 , A1 B1 i A2 B2 pripadaju nekoj pravoj s3 , osi perspektive
trouglova A1 B1 C1 i A2 B2 C2 . Prema tome se prave odredene odgovarajucim
stranicama trouglova Q1 Q2 Q3 i R1 R2 R3 seku u tackama A1 , A2 , A3 koje pripadaju jednoj pravoj pa se prema Dezargovoj teoremi prave Q1 R1 , Q2 R2 , Q3 R3 ,
tj. prave s1 , s2 , s3 seku u jednoj tacki.

4.7

Paskalova teorema i njena primena

360. (B. Pascal) Ako je A1 , . . . , A6 tetivan sestougao, dokazati da tacke P ,


Q, R u kojima se seku prave odredene naspramnim stranicama A1 A2 i A4 A5 ,
A2 A3 i A5 A6 , A3 A4 i A6 A1 pripadaju jednoj pravoj.
Uputstvo.
Neka su X, Y, Z tacke u kojima se seku A3 A4 i A5 A6 , A1 A2 i A3 A4 , A1 A2 i
A5 A6 . Prave A4 A5 , A2 A3 , A6 A1 seku prave odredene stranicama YZ, ZX, XY
trougla XYZ pa je prema Menelajevoj teoremi

269

ZA5
XA4
YP

= 1
PZ
A5 X
A4 Y
ZQ
XA3
Y A2

= 1
QX
A3 Y
A2 Z
XR Y A1
ZA6

= 1
RY
A1 Z
A6 X

Q
X
A5
A4
A6

A3
P
A2

A1
Z

Slika 360
Mnozenjem odgovarajucih strana ovih jednakosti nalazimo da je

Y A1 Y A2
ZA5 ZA6
ZQ
XR XA3 XA4
YP

= 1

PZ
QX
RY
XA5 XA6
Y A3 Y A4
ZA1 ZA2
S obzirom da prema poznatom stavu

XA3 XA4 = XA5 XA6


Y A1 Y A2 = Y A3 Y A4

bice

ZA5 ZA6 = ZA1 ZA2


YP
ZQ
XR

= 1
PZ
QX
RY
270

Odatle, prema Menelajevoj teoremi, tacke P, Q, R pripadaju jednoj pravoj.


361. Neka se tri tetive AA , BB , CC istog kruga k seku u jednoj tacki
S. Ako obelezimo sa X proizvoljnu tacku kruga k i sa P , Q, R tacke u kojima
prave XA , XB , XC seku respektivno prave BC, CA, AB, dokazati da tacke
P , Q, R, S pripadaju jednoj pravoj.

B
A
C

R
S
C
X

A
B

P
Q

Slika 361

Sestougao
AAXBBC je upisan u krug k, te su prema Paskalovoj teoremi
presecne tacke S, P, Q pravih odredenim njegovim naspramnim stranicama kolinearne. Isto tako, sestougao BBXCCA je upisan u krug k, te su prema Paskalovoj teoremi presecne tacke S, Q, R pravih odredenim njegovim naspramnim
stranicama takode kolinearne. Odatle sledi da tacke P, Q, R, S pripadaju jednoj
pravoj.
362. Ako obelezimo sa D proizvoljnu tacku trougla ABC, sa P i Q podnozja
upravnih iz tacke D na pravama AB i AC, a sa R i S podnozja upravnih iz
tacke A na pravama DB i DC, dokazati da su prave BC, P S, QR konkurentne.
Slika 362
S obzirom da su uglovi APD, AQD, ARD, ASD pravi, tacke P, Q, R, S pripadaju
jednom krugu kome je duz AD precnik. Stoga je sestougao APSDRQ tetivan,
pa su prema Paskalovoj teoremi presecne tacke B, O, C pravih koje su odredjene
parovima naspramnih stranica AP i DR, PS i RQ, SD i QA kolinearne. Stoga
su i prave BC ,PS , QR konkurentne.
271

363. Ako su P , Q, R podnozja upravnih iz proizvoljne tacke O na pravama


koje su odredene stranicama BC, CA, AB trougla ABC, a P , Q , R tacke
u kojima krug l odreden tackama P , Q, R sece prave BC, CA, AB, dokazati
da presecne tacke X, Y, Z pravih QR i Q R, RP i R P , P Q i P Q pripadaju
pravoj koja je odredena tackom O i sredistem S kruga l.
Slika 363
Obelezimo sa Q i R tacke kruga l dijametralno suprotne s tackama Q i R.
Pri tome su uglovi QQO i QQQ pravi, pa su tacke Q,O,Q kolinearne; isto

tako uglovi RRO i RRR su pravi, pa su tacke R,O,R kolinearne.Sestougao


QQQRRR je tetivan te prema Paskalovoj teoremi tacke O,S,X u kojima se
seku prave odredene naspramnim stranicama QQ i RR,QQ i RR QR i RQ
pripadaju jednoj pravoj. Dakle tacka X pripada pravoj OS. Istim postupkom
dokazuje se da i tacke Y i Z pripadaju pravoj OS.
364. Ako su P , Q, R tacke u kojima neka prava s kroz izvesnu tacku O
sece prave odredene stranicama BC, CA, AB trougla ABC, a P , Q , R tacke
u kojima prave OA, OB, OC seku opisni krug oko tog trougla, dokazati da se
prave P P , QQ , RR seku u jednoj tacki koja se nalazi na opisanom krugu oko
tog trougla.
Slika 364
Ako obelezimo sa S tacku u kojoj prava PP sece krug l opisan oko trougla ABC, bice sestougao APSQBC tetivan pa se prema Paskalovoj teoremi
presecne tacke O,P,Q1 pravih AP i QB, PS i BC, SQ i CA nalaze na jednoj
pravoj. S obzirom da su prave AC i OP razlicite, njihove zajednicke tacke Q i
Q1 su istovetne. Dakle prava QQ sadrzi tacku S. Istim postupkom dokazuje se
da i prava RR sadrzi tacku S.

4.8

Brijan
sonova teorema i njena primena

365. (M. Brianchon) Dokazati da se prave odredene naspramnim temenima


tangentnog sestougla seku u jednoj tacki, Brijans onovojtac ki tog sestougla.
Slika 365
Ako obelezimo sa A1 ,...,A6 proizvoljni tangentni sestougao, sa D1 ,...,D6
tacke u kojima prave odredene njegovim stranicama A1 A2 ,...,A6 A1 dodiruju
upisani krug k i sa P1 ,P2 ,P3 tacke u kojima se seku prave odredene naspramnim
stranicama D2 D1 i D5 D6 , D1 D4 i D6 D3 , D4 D5 i D3 D2 , sestougla D1 ...D6
prema poznatom stavu bice prave D2 D1 , ..., D3 D2 polare tacaka A1 , ..., A6 a
prave A1 A4 , A2 A5 , A3 A6 polare tacaka P1 , P2 , P3 u odnosu na krug k. Prema
Paskalovoj teoremi, tacke P1 , P2 , P3 u kojima se seku prave odredene naspramnim stranicama D2 D1 i D3 D4 , D1 D4 i D6 D5 , D4 D5 i D3 D2 tetivnog sestougla
D1 ...D6 pripadaju jednoj pravoj, te se prema poznatoj teoremi i polare A1 A4 , A2 A5 , A3 A6
tih kolinearnih tacaka P! .P2 , P3 seku u jednoj tacki. Prema tome, pave odredene
naspramnim temenima tangentnog sestougla seku se u jednoj tacki koju nazivamo Brijansakovom tackom tog tangentnog sestougla.
366. Ako su P , Q, R tacke u kojima stranice BC, CA, AB dodiruju upisani
krug trougla ABC, dokazati da se prave AP , BQ, CR seku u jednoj tacki.
272

Slika 366
Prave odredene stranicama degenerisanog sestougla RPCQAR su dirke ugla
upisanog u trougao ABC, pa se prema Brijansonovoj teoremi prave AD, BQ,
CR seku u jednoj tacki.
367. Ako su P , Q, R, S tacke u kojima prave odredene stranicama AB, BC,
CD, DA tangentnog cetvorougla ABCD dodiruju upisani krug k, dokazati da
se prave AC, BD, P R, QS seku u jednoj tacki.
Slika 367
Prave odredene stranicama degenerisanog sestougla APBCRD dodiruju krug k,
to se prema Brijansonovoj teoremi prave AC, BD , PR seku u jednoj tacki.
Dakle prava PR sadrzi presek X pravih AC i BD. Istim postupkom dokazuje se
da i prava QR sadrzi presek X pravih AC i BD, te se cetiri prave AC,BD,PR,QS
seku u jednoj tacki.
368. Ako su P , Q, R tacke u kojima upisani krug k dodiruje stranice BC,
CA, AB trougla ABC, zatim A , B , C tacke pravih BC, CA, AB takve da
se prave AA , BB , CC seku u jednoj tacki O, a P , Q , R tacke u kojima se
seku druge dirke kruga k iz tacaka B i C , C i A , A i B , dokazati da se prave
P P , QQ , RR seku u tacki O.
Slika 368
Prave odredene stranicama degenerisanog sestougla BPCBPC su dirke
kruga k, pa se prema Brijansonovoj teoremi prave BB , CC , PP seku u
jednoj tacki. No prave BBi CC seku se u tacki O te i prava PP sadrzi tacku
O. Istim postupkom dokazuje se da i prave QQi RR sadrze tacku O.

4.9

Van Obelova teorema i njena primena

369. (Van Aubel) Ako su A , B , C tacke pravih koje su odredene stranicama


BC, CA, AB trougla ABC takve da se prave AA , BB , CC seku u jednoj
tacki O, dokazati da je
AO
AC
AB
= + .
OA
CB
BC
Slika 369
Ako su B i C tacke u kojima prave BB i CC seku prave kroz C i B
uporedno s pravama AA, bice
AC
AO
=
BC
CB
AO
AB
=
CB
BC

OA
A C
=
BC
BC
A B
OA
=
CB
BC
273

Deljenjem zbira odgovarajucih strana poslednjih dveju jednakosti, nalazimo da


je
AO
AC
AB
= 6=
OA
CB
BC

370. Ako je T presek tezisnih linija AA , BB , CC trougla ABC, dokazati


da je
AT : T A = 2 : 1.
Prema Van Obelovoj teoremi, imamo da je
AT : T A = AC : C B 6= AB : B C = 2 : 1.
371. Ako je S srediste upisanog kruga trougla ABC, a E tacka u kojoj prava
AS sece stranicu BC, dokazati da je
AS : SE = (b + c) : a.
Prema Van Obelovoj teoremi, imamo da je
AS : SE = AC : C B + AB : B C = b : a + c : a = (b + c) : a.

372. Ako je G Zergonova


tacka trougla ABC, a P tacka u kojoj upisani krug
dodiruje stranicu BC, dokazati da je
a(p a)
AG
=
.
GP
(p b)(p c)
Ako su B i C tacke u kojima prave BSa i ASc seku prave AC i AB, prema
Van Obelovoj teoremi imamo da je
ASa : Sc F = AC : C B + AB : B C = b : c c : a = (b c) : a.
373. Ako je Sc srediste spolja upisanog kruga trougla ABC kome je AC >
AB i F tacka u kojoj prava ASc sece pravu BC, dokazati da je
ASc : Sc F = (b c) : a.
Ako obelezimo sa Q i R tacke u kojima upisani krug dodiruje stranice CA i
AB trougla ABC, prema Van Obelovoj teoremi, imamo da je
AG
AQ
AR
pa pa
a(p a)
=
6=
=
+
=
GP
QC
RB
pc
pb
(p b)(p c)
374. Ako je r poluprecnik opisanog kruga i poluprecnik upisanog kruga
trougla ABC, zatim P , Q, R tacke u kojima upisani krug dodiruje stranice BC,

CA, AB i G Zergonova
tacka trougla ABC, dokazati da je
AG BG CQ
4r

= .
GP GQ GR

Prema prethodnom zadatku, imamo da je


a(p a)
AG
=
,
GP
(p b)(p c)
274

b(p b)
BG
=
,
GQ
(p c)(p a)
c(p c)
CG
=
.
GR
(p a)(p b)

Otuda se primenom zadatka 264 , 259, 261, dobija da je


AG
4r
BG
CG
abc
4rp
= .
=
=
=
=
2
GP
CQ
GR
(p a)(p b)(p c)
p

375. Ako je N Nagelova tacka trougla ABC i Pa u kojoj spolja upisani krug
dodiruje stranicu BC. Dokazati da je
a
AN
=
.
N Pa
pa
Ako obelezimo sa Qb i Rc tacke u kojima spolja opisani krugovi dodiruju
stranice CA i AB trougla ABC, prema Van Obelovoj teoremi imamo da je
AN
AGb
ARc
pb
pc
a
=
+
=
+
+
N Pa
Qb c
Rc B
pa pa pa

376. Ako je r poluprecnik opisanog kruga i poluprecnik upisanog kruga


trougla ABC, zatim Pa , Qb , Rc tacke u kojima spolja upisani krugovi dodiruju
stranice BC, CA, AB i N Nagelova tacka tog trougla, dokazati da je
4r
AN BN CN

=
.
N Pa N Qb N Rc

Prema prethodnom zadatku, imamo da je


a
AN
=
N Pa
pa
BN
b
=
N Qb
pb
c
CN
=
N Rc
pc

Otuda, primenom zadatka 264, 259 , 261, nalazimo da je


AN BN CN
abc
4r
4rp

=
=
=
N Pa N Qb N Rc
(p a)(p b)(p c)
p2

377. Ako su M1 i N1 tacke stranica AB i AC takve da je BM1 : M1 A =


AN1 : N1 C = k1 , a M2 i N2 tacke tih istih stranica takve da je BM2 : M2 A =
AN2 : N2 C = k2 , dokazati da se prave M1 N1 i M2 N2 seku u nekoj tacki S pri
cemu je M1 S : SN1 = k2 i M2 S : SN2 = k1 .

275

A
M2
N1
S

M1

P
B

N2
C

Slika 262.
Iz BM1 : M1 A = k1 iBM2 : M2 A = k2 nalazimo da je M1 M2 : M2 A =
(k2 k1 ) : (1 + k1 ) a iz AN1 : N1 C = k1 iAN2 : N2 C = k2 da je AN1 : N1 N2 =
k1 (1 + k2 ) : (k2 k1 ) Ako obelezimo sa P tacku u kojoj se seku prave AS i

M1 N2 , primenom Cevijeve
teoreme nalazimo da je
N2 P : P M1 = (1 + k1 ) : k1 (1 + k2 )
Najzad primenom Van Obelove teoreme dobijamo da je M1 S : SN1 = k2 i
M2 S : SN2 = k1 .

4.10

Ptolomejeva teorema

378.(K. Ptolomej) Ako je ABCD konveksan i tetivan cetvorougao, dokazati


da je proizvod njegovih dijagonala jednak zbiru proizvoda njegovih naspramnih
stranica, tj. da je
AC BD = AB CD + BC AD.

276

Slika 263.
S obzirom da je cetvorougao ABCD konveksan , poluprava DB je u konveksnom uglu, ugla CDA. Odatle sledi da poluprava DE sece duz AC u nekoj
tacki E. Pri tom je
AED BCD
i ABD ECD pa je AD : AE = BD : BC i AB : AD = EC : CD. Iz ovih
proporcija nalazimo da je AE BD = BC AD i BC BD = AB CD pa je
(AE + EC) BD = AB CD + BC AD. Kako je tacka E izvan tacaka A i C,
imamo da je
AE + EC = AC
i prema tome
AC BD = AB CD + BC AD.
379. Primenom Ptolomejeve teoreme dokazati Pitagorin stav.
Neka je ugao kod temena B prav ugao trougla ABC, a tacka D takva da je
cetvorougao ABCD pravougaonik. Prema Ptolomejevoj teoremi imamo da je
AC BD = AB CD + BC AD
pa je
AC 2 = AB 2 + BC 2 .
380. Ako je k krug opisan oko kvadrata ABCD, a M proizvoljna tacka luka
CD kruga k na kome nisu temena A i B, dokazati da je

M A + M C = M B 2 i M A M C = M D 2.
277

Primenom Ptolomejeve teoreme na konveksan i tetivan cetvorougao ABCM


nalazimo da je
AC BM = AB M C + BC M A.

S obzirom da je AC = AB 2 i BC = AB imamo da je

M A + M C = M B 2.
Analogno i drugi deo tvrdjenja.
381. Ako je a stranica i d dijagonala pravilnog petougla A1 A5 , dokazati
da je

a
d = (1 + 5).
2

A4

A5
A3

A1
A2

278

Primenom Ptolomejeve teoreme na cetvorougao


A1 A2 A3 A4
nalazimo da je
a2 + ad = d2
pa je

a
(1 + 5).
2
382. Ako je A1 A7 pravilan sedmougao, dokazati da je
d=

1
1
1
=
+
.
A1 A2
A1 A3
A1 A4

A5

A6

A4
A7

A3
A1
A2

Primenom Ptolomejeve teoreme na cetvorougao


A1 A3 A4 A5
nalazimo da je
A1 A3 A4 A5 + A3 A4 A5 A1 = A1 A4 A3 A5
Ako obe strane ove jednakosti podelimo sa
A1 A2 A1 A3 A1 A4
koristeci jednakosti
A1 A2 = A3 A4 = A4 A5
A1 A3 = A3 A5
i
A1 A4 = A1 A5
279

dobijamo da je

1
1
1
=
+
.
A1 A2
A1 A3
A1 A4

383. Ako je ABC jednakostranican trougao i P tacka kruga k opisanog oko


tog trougla, dokazati da je duz AP jednaka zbiru ili razlici duzi BP i CP , prema
tome da li tacka P pripada luku BC krugu k na kome nije A ili je na kruznom
luku BAC.

Slika 264.
Ako je tacka P na luku BC kruga na kome nije tacka A cetvorougao ABP C
je konveksan i tetivan pa je prema Ptolomejevoj teoremi
AP BC = BP AC + CP AB.
S obzirom da je trougao ABC jednakostranican imamo da je
AB = BC = CA
pa je i
AP = BP + CP.
Analogan postupak primenjuje se i u slucaju kada je tacka P na kruznom luku
BAC.
384. Ako je ABCD konveksan i tetivan cetvorougao, a E tacka u kojoj prava
kroz C uporedna sa dijagonalom BD sece opisani krug, dokazati da je
AE BD = AB BC + CD DA.
280

E
D
C

Slika 265.
Prema Ptolomejevoj teoremi kod konveksnog i tetivnog cetvorougla ABED
imamo da je
AE BD = AB ED + BE DA.
S obzirom da je cetvorougao BCED jednakokraki trapez bice
BE = CD
i
ED = BC
pa je
AE BD = AB BC + CD DA.
385. Ako je ABCD paralelogram i E tacka u kojoj krug opisan oko trougla
ABC sece polupravu AD, dokazati da je
AD AE = AC 2 AB 2 .

281

Prema Ptolomejevoj teoremi kod konveksnog i tetivnog cetvorougla ABCE


imamo da je
AC BE = AE BC + AB CE.
S obzirom da je cetvorougao ABCE takode i jednakokraki trapez bice
AC = EB
i
AB = EC.
Posto je cetvorougao ABCD paralelogram sledi da je
AD = BC
pa iz svega sledi da je
AD AE = AC 2 AB 2 .

386. Ako su B , C , D tacke u kojima proizvoljan krug kroz teme A paralelograma ABCD sece poluprave AB, AC, AD, dokazati da je
AC AC = AB AB + AD AD .

282

C
D

387. Ako su a, b, c, d duzi jednake stranicama AB, BC, CD, DA a x i y


duzi jednake dijagonalama AC i BD konveksnog i tetivnog cetvorougla ABCD,
dokazati da je
ad + bc
x
=
.
y
ab + cd

D
C

Slika 266.

Ako obelezimo sa C tacku u kojoj prava kroz C uporedna na BD sece


opisani krug k, a sa D tacku u kojoj prava kroz D uporedna sa AC sece krug
283

k bice kruzni luci ADC i BCD medu sobom jednaki pa je


AC = BD = z.
Iz konveksnih i tetivnih cetvorouglova ABCD i ABC D primenom Ptolomejeve
teoreme,nalazimo da je
xz = ad + bc
i
yz = ab + cd.
Deljenjem odgovarajucih strana ovih dveju jednakosti dobijamo da je
x
ad + bc
=
.
y
ab + cd
388. Ako su a, b, c, d duzi jednake stranicama AB, BC, CD, DA a x i y
duzi jednake dijagonalama AC i BD konveksnog i tetivnog cetvorougla ABCD,
dokazati da je
x2 =

(ac + bd)(ad + bc)


,
ab + cd

y2 =

(ac + bd)(ab + cd)


.
ad + bc

Prema zadacima 378. i 387. imamo da je xy = ac + bd i

ad + bc
x
=
pa je
y
ab + cd

(ac + bd) (ad + bc)


(ac + bd) (ab + cd)
i y2 =
.
ab + cd
ad + bc
389. Ako je AB precnik i r poluprecnik kruga k, a C i D tacke na raznim
lucima AB tog trougla, dokazati da je

x2 =

CD =

p
p
1
(AC 4r2 AD2 + AD 4r2 AC 2 ).
2r

284

Prema Ptolomejevoj teoremi bice AB CD = AC BD + AD BC pa je


CD =

p
p
1
1
(AC BD + AD BC) =
(AC 4r2 AD2 + AD 4r2 AC 2 ).
2r
2r

390. Ako je AB precnik kruga k, a C i D tacke na istom luku AB toga


kruga takve da je cetvorougao ABCD konveksan, dokazati da je
CD =

p
p
1
(AC 4r2 AD2 AD 4r2 AC 2 ).
2r

Prema Ptolomejevoj teoremi bice AC BD = AB CD + AD BC pa je


CD =

p
p
1
1
(AC BD AD BC) =
(AC 4r2 AD2 AD 4r2 AC 2 ).
2r
2r

391. Ako obelezimo sa S presechnu tacku dijagonala cetvorougla ABCD


upisanog u krugu poluprecnika r, a sa r1 , r2 , r3 , r4 poluprecnike krugova opisanih oko trouglova SAB, SBC, SCD, SDA, dokazati da je
r2 =

(r1 r2 + r3 r4 )(r1 r4 + r2 r3 )
.
r1 r3 + r2 r4

Uputstvo.

285

D
O3
C
A

O4

O2

O1

Slika 267
Ako obelezimo sa O1 ,O2 ,O3 ,O4 sredista krugova opisanih oko trouglova
SAB, SBC, SCD, SDA bice
ABC O1 BO2
BCD O2 CO3
CDA O3 DO4
DAB O4 AO1
pa je

BC
CD
DA
AB
=
=
=
= k...(1).
r1
r2
r3
r4

Ako zatim sa A obelezimo podnozje upravne iz temena A na pravoj BD, zatim


na trouglove SAB i DAB primenimo zadatak 321. nalazimo da je
AS = 2r1

AA
AB

AA
AD
.Iz ovih dveju relacija i poslednje od relacija (1) dobijamo da je
AB = 2r

AS =

r4 r1
k
r

Analognim postupkom nalazimo da je i


BS =

r1 r2
k
r

r2 r3
k
r
r3 r4
k
DS =
r
CS =

286

Stoga je
AC =

r4 r1 + r2 r3
k
r

r1 r2 + r3 r4
k...(2).
r
Zamenom kolicnika AB, BC, CD, DA, AC, BD i relacija (1) i (2) u Ptolomejevu jednakost
AC BD = AB CD + AD BC
BD =

dobijamo trazenu relaciju.


392. Ako je A1 . . . A6 konveksan sestougao upisan u krug k i ako njegove
uzastopne stranice A1 A2 , . . . , A6 , A1 obelezimo sa a1 , . . . , a6 dijagonale A1 A4 ,
A2 A5 , A3 A6 sa d1 , d2 , d3 , dokazati da je
d1 d2 d3 = a1 a3 a5 + a2 a4 a6 + a1 a4 d3 + a2 a5 d1 + a3 a6 d2 .

Uputstvo.

A5

A4

A6

A3

A1

A2

Slika 268
Dijagonale A1 A3 , A3 A5 , A5 A1 , A2 A4 obelezimo sa e1 , e2 ,e3 , e4 . Primenom
Ptolomejeve teoreme na cetvorouglove
A1 A2 A3 A4
i
A2 A3 A4 A5
nalazimo da je
a1 a3 + a2 d1 = e1 e4
287

i
a2 a4 + a3 d2 = e2 e4
Ako prvu od ovih jednakosti pomnozimo sa a5 a drugu sa a6 sabiranjem nalazimo
da je
a1 a3 a5 + a2 a4 a6 + a2 a5 d1 + a3 a6 d2 = d3 e3 ed
Najzad primenom Ptolomejeve teoreme na cetvorougao
A1 A2 A4 A5
dobijamo da je
e3 e4 = d1 d2 a1 a4
pa je
d1 d2 d3 = a1 a3 a5 + a2 a4 a6 + a2 a5 d1 + a3 a6 d2

393. Ako su d1 , . . . , d2n+1 rastojanja temena pravilnog poligona A1 . . . A2n+1


s neparnim brojem stranica od proizvoljne tacke P koja se nalazi na manjem
luku A1 A2n+1 kruga opisanog oko tog poligona, dokazati da je
d1 + d3 + d5 + . . . + d2n+1 = d2 + d4 + d6 + . . . + d2n .

Uputstvo.

A3
A4

A2

A2n + 1

A1

Slika 269
Obelezimo sa a stranicu i sa b najmanju dijagonalu pravilnog poligona
A1 ...A2n+1 . Primenom Ptolomejeve teoreme na tetivne cetvorouglove
P A1 A2 A3 , P A2 A3 A4 , ..., P A2n+1 A1 A2

288

nalazimo da je
a (d1 + d2 ) = bd2 , bd3 = a(d2 + d4 ), ..., bd2n+1 + ad1 = ad2n , bd1 + ad2n+1 = ad2
Sabiranjem odgovarajucih strana ovih jednakosti nalazimo
d1 + d3 + d5 + ... + d2n+1 = d2 + d4 + ... + d2n

4.11

Ojlerova teorema iz geometrije


cetvorougla i njena
primena

394. (L. Euler) Ako su P i Q sredista dijagonala AC i BD cetvorougla


ABCD, dokazati da je
AB 2 + BC 2 + CD2 + DA2 = AC 2 + BD2 + 4P Q2 .

Uputstvo.

C
D

P
Q

Slika 270
Duzi AQ, CQ, P Q su tezisne linije trouglova ABD, CBD, QAC pa je prema
zadatku 299.
1
2AQ2 = AB 2 + AD2 BD2
2
1
2CQ2 = BC 2 + CD2 BD2
2
1
2P Q2 = AQ2 + CQ2 AC 2
2
Ako poslednju od ovih jednakosti pomnozimo sa 2 i oduzmemo od zbira odgovarajucih strana prve dve jednakosti nalazimo da je
AB 2 + BC 2 + CD2 + DA2 = AC 2 + BD2 + 4P Q2
289

395. Dokazati da je zbir kvadrata stranica paralelograma ABCD jednak


zbiru kvadrata njegovih dijagonala, tj. da je
AB 2 + BC 2 + CD2 + DA2 = AC 2 + BD2 .

Uputstvo.
Kako se kod paralelograma ABCD sredista P i Q dijagonala AC i BD
poklapaju, prema Ojlerovoj teoremi imamo da je
AB 2 + BC 2 + CD2 + DA2 = AC 2 + BD2
396. Ako su kod cetvorougla ABCD tacke K i L sredista naspramnih stranica AB i CD, a tacke M i N sredista naspramnih stranica BC i DA, dokazati
da je
AC 2 + BD2 = 2(KL2 + M N 2 ).

Uputstvo.
Prema poznatom stavu cetvorougao KM LN je paralelogram pa je prema
prethodnom zadatku
KL2 + M N 2 = 2(KM 2 + M L2 ) =

1
(AC 2 + BD2 )
2

Otuda je
AC 2 + BD2 = 2(KL2 + M N 2 )
397. Ako su kod cetvorougla ABCD dijagonale AC i BD medu sobom
upravne, dokazati da su zbirovi kvadrata naspramnih stranica tog cetvorougla
medu sobom jednaki, tj. da je
AB 2 + CD2 = BC 2 + AD2 .

Uputstvo.

290

C
D
O

Slika 271
Neka je O tacka u kojoj se seku prave odredene dijagonalama AC i BD.
AB 2 + CD2 = AO2 + BO2 + CO2 + DO2
i
BC 2 + AD2 = AO2 + BO2 + CO2 + DO2
pa je
AB 2 + CD2 = BC 2 + AD2
398. Ako su kod cetvorougla ABCD zbirovi kvadrata naspramnih stranica
jednaki, dokazati da su prave odredene dijagonalama AC i BD medu sobom
upravne.
Uputstvo.
Iz jednakosti
AB 2 + CD2 = BC 2 + AD2
sledi da je
AB 2 BC 2 = AD2 CD2
stoga se podnozja upravnih iz tacaka B i D na pravoj AC poklapaju, pa je
prava AD upravna na pravu AC.

291

HARMONIJSKI SPREGNUTI ELEMENTI.


DVORAZMERA

5.1

Harmonijske ta
cke i harmonijske prave

Definicija 5.1. Ako su A, B, C, D cetiri razne kolinearne tacke takve da


je AC : CB = AD : DB, tada kazemo da su tacke C i D harmonijski spregnute
sa tackama A i B ili da su tacke A, B, C, D harmonijski spregnute.

Cinjenicu
da su A, B, C, D cetiri harmonijske tacke simbolicki obelezavamo
sa H(A, B; C, D).
Definicija 5.2. Za cetiri konkurentne ili paralelne prave a, b, c, d kazemo da
su harmonijski spregnute ako postoji prava s koja ih sece redom u harmonijskim
tackama A, B, C, D.

Cinjenicu
da su a, b, c, d cetiri harmonijske prave simbolicki obelezavamo sa
H(a, b; c, d).
399. Ako su A, B, C tri razne kolinearne tacke, dokazati da postoji jedna i
samo jedna tacka D takva da je H(A, B; C, D).
Uputstvo.
Ako su A,B,C tri razne kolinearne tacke. Dokazati da postoji jedna i samo
jedna tacka D takva da je H(A, B; C.D).
Ako bi postojala tacka D takva da je
H(A, B; C, D)
imali bismo da je
AC : CB = AD : BD
. Otuda je
(AC + BC) : CB = (AD + DB) : BD
,tj
(AC + BC) : CB = AB : BD
. U dobijenoj proporciji prve tri duzi su poznate, pa je cetvrta jednoznacno
odredena. Stoga postoji jedna i samo jedna tacka D za koju je
H(A, B; C, D)
.
400. Ako su A, B, C, D cetiri kolinearne tacke takve da je H(A, B; C, D),
dokazati da je H(C, D; A, B).
Uputstvo.
Ako su A,B,C,D cetiri kolinearne tacke takve da je
H(A, B; C, D)
, dokazati da je
H(C, D; A, B)
292

.
Iz
H(A, B; C, D)
tj.
AC : CB = AD : DB
sledi da je
CA : AD = CB : BD
, pa je i
H(C, D; A, B)
.
401. Ako su A, B, C, D cetiri razne tacke neke prave p, O tacka izvan prave
p a E i F tacke u kojima prava kroz B uporedna sa OA sece OC i OD, dokazati
da je H(A, B; C, D) ako i samo ako je tacka B srediste duzi EF .
Uputstvo.

Slika 272
Ako su A,B,C,D cetiri razne tacke neke prave p, O tacka izvan prave p, a E
i F tacke u kojima prava kroz B uporedna sa OA sece OC i OD, dokazati da je
H(A, B; C, D)
ako i samo ako je tacka B srediste duzi EF .
Najpre dokazimo da je navedeni uslov potreban. Iz
H(A, B; C, D)
sledi da je
AC : BC = AD : BD
. No
OAC EBC
293

i
OAD F BD
pa je
AC : BC = AO : BE
i
AD : BD = AO : BD
. Otuda je
AO : BE = AO : BF
i prema tome je
BE = BF
. Iz ove jednakosti sledi da je tacka B srediste duzi EF .(slika 274)
Dovoljan uslov dokazuje se reverzibilnim postupkom.
402. Ako su A, B, C, D cetiri razne kolinearne tacke i ako je tacka O srediste
duzi AB, dokazati da je H(A, B; C, D) ako i samo ako je
OB 2 = OC OD.

Uputstvo.
Ako su A,B,C,D, cetiri razne kolinearne tacke i ako je tacka O srediste duzi
AB, dokazati da je
H(A, B; C, D)
ako i samo ako je
OB 2 = OC OD.
Najpre dokazimo da je navedeni uslov potreban.
Iz
H(A, B; C, D)
, tj.
AC : CD = AD : BD
,sledi da je
(AO + OC) : (OB OC) = (AO + OD) : (OD OB)
, pa je
2OB : (OB OC) = 2OD : (OD OB)
,tj.
OB : OC = OD : OB
i prema tome je
OB 2 = OC OD
.
Dovoljan uslov se dokazuje reverzibilnim postupkom.

294

403. Dokazati da su cetiri razne kolinearne tacke A, B, C, D harmonijski


spregnute ako i samo ako je
1
2
1
+
=
.
AC
AD
AB

Uputstvo.
Dokazati da su cetiri razne kolinearne tacke A,B,C,D harmonijski spregnute
ako i samo ako je
1
1
2
+
=
.
AC
AD
AB
Najpre dokazimo da je navedeni uslov potreban. Iz
H(A, B; C, D)
, tj.
AC : CB = AD : BD
, sledi da je
CB : AC = BD : AD
,tj.
(CA + AB) : AC = (BA + AD) : AD
. Otuda je
(AB : AC) + (AB : AD) = 2
, tj.
(1 : AC) + (1 + AD) = 2 : AB
.
Dovoljan uslov se dokazuje reverzibilnim postupkom.
404. Ako su A, B, C, D razne kolinearne tacke, dokazati da je H(A, B; C, D)
ako i samo ako je
OC
OD
2
+
=
.
AC
AD
AB

Uputstvo.
Ako su A,B,C,D razne kolinearne tacke, dokazati da je
H(A, B; C, D)
ako i samo ako je

OC
OD
OB
+
=2
.
AC
AD
AB
Najpre dokazimo da je navedeni uslov potreban. Iz
H(A, B; C, D)
295

sledi da je
(1 : AC) + (1 : AD) = 2 : AB
, pa je
OA : AC = OA : AD = 2 OA : AB
. Otuda je
[(OA : AC) + 1] + [(OA : AD) + 1] = 2 [(OA : AB) + 1]
i prema tome je
(OC : AC) + (OD : AD) = 2 OB : AB
Dovoljan uslov se dokazuje reverzibilnim postupkom.
405. Ako su A, B, C, D razne kolinearne tacke, dokazati da je
(OA + OB)(OC + OD) = 2(OA OB + OC OD).

Uputstvo.
Ako su A,B,C,D razne kolinearne tacke, dokazati da je
H(A, B; C, D)
ako i samo ako je
(OA + OB) (OC + OD) = 2 (OA OB + OC OD.)
Najpre dokazimo da je navedeni uslov potreban. Iz
H(A, B; C, C)
, tj.
AC : CB = AD : DB
nalazimo da je
(AO + OC) : (CO : OB) = (AO + OD) : (DO + OB)
, tj.
(OC OA) : (OB OC) = (OA OD) : (OB OD)
. Otuda je
OA OC + OB OC + OA OD + OB OD = 2 (OA OB + OC OD)
i prema tome
(OA + OB) (OC + OD) = 2(OA OB + OC OD)
296

Dovoljan uslov dokazuje se revarzibilnim postupkom.


406. Ako su A, B, C, D kolinearne tacke takve da je H(A, B; C, D), dokazati
da je
1
1
1
1
+
+
+
= 0.
AC
BC
AD BD

Uputstvo.
Ako su A,B,C,D kolinearne tacke da je H(A,B;C,D), dokazati da je
1
1
1
1
+
+
+
=0
AC
BC
AD BD
. Iz H(A,B;C,D), tj.
AC AD
:
= 1
CB DB
nalazimo da je

odnosno da je

Otuda je

i prema tome

1
AD
=
CB
AC BD
AC + CB + BD
1
=
CB
AC BD
1
CB
1
1
=
+
+
CB
BD AC BD AC

1
1
1
1
+
+
+
=0
AC
BC
AD BD
407. Ako su A, B, C, D kolinearne tacke takve da je H(A, B; C, D) i ako je
O sredisne duzi AB, dokazati da je
1
1
1
+
=
.
AC BC
AD BD
AO BO

Uputstvo.
Ako su A,B,C,D kolinearne tacke takve da je H(A,B;C,D) i ako je O odrediste
duzi AB, dokazati da je
1
1
1
+
=
AC BC
AD BD
AO BO
Prema zadatku 403. imamo da je
1
2
1
+
=
AC
AD
AB
297

pa je

1
1
1
+
=
AC
AD
AO

Isto tako je

1
1
1
+
=
BC
BD
BO
Mnozenjem odgovarajucih strana dobijenih dveju jednakosti nalazimo da je
1
1
1
1
1
+
+
+
=
AC BC
AC CD AD BD AD BD
AO BO

S obzirom da je

bice
pa je

AC
AD
=
CB
DB
1
1
=
AC BD
AD BC
1
1
1
+
=
AC BC
AD BD
AO BO

408. Ako su A, B, C, D kolinearne tacke takve da je H(A, B; C, D) i ako je


tacka O srediste duzi AB, dokazati da je
AD BD = CD OD.

Uputstvo.
Ako su A,B,C,D kolinearne tacke takve da je H(A,B;C,D)i ako je tacka O
srediste duzi AB dokazati da je
AD BD = CD OD.
Prema zadatku 402. imamo da je OB 2 = OC OD, pa je OB 2 = (OD CD)
OD, tj. OB 2 = OD2 CD OD. I otuda je OD2 OB 2 = CD OD, tj.
(OD + OB) (OD OB) = CD OD i prema tome AD BD = CD OD.
409. Ako su A, B, C, D kolinearne tacke takve da je H(A, B; C, D) i ako je
tacka O srediste duzi AB, dokazati da je
OC 2 + OD2 = CD2 + 2OB 2 .

Uputstvo.

298

Ako su A,B,C,D kolinearne tacke takve da je H(A,B;C,D) i ako ja tacka O


srediste duzi AB, dokazati da je
OC 2 + OD2 = CD2 + 2 OB 2 .
Prema zadatku 402. imamo da je
OB 2 = OC OD
te je
2OB 2 = OCOD+OCOD = (OC+CD)OC+(ODCD)OD = OC 2 +OD2 CD2
Otuda je
OC 2 + OD2 = CD2 + 2OB 2
410. Ako su A, B, C, D kolinearne tacke takve da je H(A, B; C, D) a tacke
O i O sredista duzi AB i CD, dokazati da je
OB 2 + OD2 = OO2 .

Uputstvo.
Ako su A,B,C,D kolinearne tacke takve da je H(A,B;C,D), a tacke O i
Osredista duzi AB i CD dokazati da je
OB 2 + O D2 = OO2 .
Prema zadatku 402. imamo da je
OB 2 = OC OD
pa je
OB 2 = (OO + O C) (OO + O D)
i prema tome
OB 2 + O D2 = OO2
411. Ako su A, B, C, A , B , C tacke jedne prave takve da je H(A, A ; B, C),
H(A, A ; C, A), H(C, C ; A, B), dokazati da je
1
1
1
+
+
= 0.
AA
BB
CC

Uputstvo.
Ako su A,B,C,A,B,Ctacke jedne prave takve da je H(A,A;B,C), H(B,B;C,A)
i H(C,C;A,B) dokazati da je
1
1
1
+
+
= 0.

AA
BB
CC
299

Koristeci zadatak 403. nalazimo da je


1
1
1 1
1
1
1
1
1
1
+
+
= (
+
+
+
+
+
)=0
AA
BB
CC
2 AB
AC
BC
BA CA CB
412. Ako su A, B, C, D cetiri kolinearne tacke takve da je H(A, B; C, D) i
ako je pri tome AC : CB = m : n, dakle i AD : DB = m : n, dokazati da je
DB
m+n
=
BC
mn

DA
m+n
=
.
AC
mn

Uputstvo.
Iz proporcija AC:CB=m:n i AD:DB=m:n sledi da je (AC+CB):CB=(m+n):n,
(AD-DB):DB=(m-n):n, AC:(AC+CB)=m:(m+n), AD:(AD-DB)=m:(m-n), tj.
da je AB:CB=(m+n):m...(1), AB:DB=(m-n):n...(2), AC:AB=(m+n)...(3), AD:AB=m:(mn)...(4). Iz jednakosti (1) i (2) sledi da je
DB : BC = (m + n) (m n)
a iz jednakosti (3) i (4) da je
DA : AC = (m + n) (m n)
413. Ako su A, B, C, D cetiri kolinearne tacke takve da je H(A, B; C, D) i
AC : CB = m : n, i ako je tacka N srediste duzi CD, dokazati da je
AN : N B = m2 : n2 .

Uputstvo.
S obzirom da je N na produzenju AB bice
AN =

1
CD + AC
2

1
CD BC
2
Ako duz AB obelezimo sa a, iz proporcija (1) i (3)nalazimo da je
NB =

AC =

am
m+n

BC =

an
m+n

300

a iz proporcija (2) i (4), da je


CD = AD AC =
Otuda je

2amn
m2 n 2

AN =

am2
m2 n 2

BN =

an2
m2 n 2

pa je

AN : N B = m2 : n2
414. Ako su A, B, C, D cetiri kolinearne tacke takve da je H(A, B; C, D) i
AC : CB = m : n, a tacke M i N sredista duzi AB i CD, dokazati da je
MN
1 m2 + n 2
.
= 2
AB
2 m n2

Uputstvo.
Pri m > n imamo da je
MN =

a
+ BN
2

a pri mn da je

a
2
gde je a=AB. Prema prethodnom zadatku imamo da je
M N = AN

AN =

am2
m2 n 2

BN =

an2
m2 n 2

pa je u oba slucaja
MN =
i prema tome

a m2 + n 2

2 m2 n 2

M N : AB = (m2 + n2 ) : 2(m2 n2 )
415. Ako su A, B, C, D cetiri kolinearne tacke takve da je H(A, B; C, D), a
M i N sredista duzi AB i CD, dokazati da je
AB 2 + CD2 = 4M N 2 .

301

Uputstvo.
Pri resavanju zadatka 413. imali smo da je AB=a i
CD =

2amn
m2 n 2

pa se primenom prethodnog zadatka dobija da je


AB 2 + CD2 = a2 +

a2 (m2 + n2 )2
4(amn)2
=
= 4M N 2
2
2
2
(m n )
(m2 n2 )2

416. Ako su A, B, C, D cetiri kolinearne tacke takve da je H(A, B; C, D) i


pri tome AC : CB = m : n, dokazati da je
m2 n 2
AB
=
.
CD
2mn

Uputstvo.
Prilikom resavanja zadatka 413. imali smo da je AB=a i
CD = 2amn : (m2 n2 )
pa je
AB : CD = (m2 n2 ) : 2mn
417. Ako obelezimo sa E i F tacke u kojima simetrale unutrasnjeg i spoljasnjeg ugla A trougla ABC seku pravu BC, i sa S, Sa , Sb , Sc sredista upisanih
krugova tog trougla, dokazati da je
(a)
H(B, C; E, F )
;

(b)
H(A, E; S, Sa )

;
(v)

H(A, P ; Sb , Sc )

Uputstvo.
(a) S obzirom da je BE: EC=AB:AC i BF:FC=-AB:AC, bice BE:EC=BF:FC, pa je H(B,C;E,F).
(b) Prema poznatom stavu imamo da je AS:SE=(AB+AC):BC i
ASa : Sa E = (AB + AC) : BC
302

pa je
AS : SE = ASa : Sa E
i prema tome H
(A, E, S, Sa )
(v) Prema poznatom stavu imamo da je
ASb : Sb F = (AB AC) : BC
i
ASc : Sc F = (AB AC) : BC
pa je
ASb : Sb F = ASc : Sc F
i prema tome H
(A, F ; Sb , Sc )
418. Ako je A1 srediste stranice BC, D podnozje visine iz temena A, a E i
F tacke u kojima simetrale unutrasnjeg i spoljasnjeg ugla A trougla ABC seku
pravu BC, dokazati da je
(a)
4A1 D A1 E = (b c)2
;
(b)
4A1 D A1 F = (b + c)2
;
(v)
A1 D EF = bc
.

Uputstvo.
(a) Obelezimo sa S i Sa sredista upisanih krugova k i ka trougla ABC, a
sa P i Pa tacke u kojima ti krugovi dodiruju stranicu BC s obzirom da je
H(A, E; S, Sa ), bice i H(D, E; P, Pa ). No, tacka A1 je srediste duzi P Pa , a duz
P Pa jednaka razlici stranica AB i AC, pa je prema zadatku 402.
A1 D A1 E = A1 P 2
i prema tome
4A1 D A1 E = (b c)2
(b) Obelezimo sa Sb i Sc sredista spolja upisanih krugova trougla ABC, a sa Pb
i Pc tacke u kojima ti krugovi dodiruju pravu BC. Pri tome je H(A, F ; Sb , Sc ),
pa je i H (D, F ; Pb , Pc ). No tacka A1 je srediste duzi Pb Pc , a duz Pb Pc jednaka
zbiru stranica AB i AC, pa je prema zadatku 402.
A1 D A1 F = A1 Pb2

303

pa prema tome
4A1 D A1 F = (b + c)2
(v) S obzirom da je
4A1 D A1 E = (b c)2
i
4A1 D A1 F = (b + c)2
bice
4A1 D(A1 F A1 E) = 4bc
pa prema tome
A1 D EF = bc
419. Ako su P , Q, R tacke u kojima upisani krug dodiruje stranice BC, CA,
AB trougla ABC, a S tacka u kojoj prava QR sece pravu BC, dokazati da je
H(B, C; P, S).
Uputstvo.
Primenom Menelajeve teoreme na trougao ABC i prvu QR, nalazimo da je
BS CQ AR

= 1
SC QA RB
(slika 275).
Kako je CQ=PC, QA=AR, RB=BP bice
BS BP
:
= 1
SC P C
pa je H(B,C;P,S).

A
R

Slika 273
420. Ako su Pa , Qa , Ra tacke u kojima spolja upisani krug koji odgovara
temenu A trougla ABC dodiruje prave BC, CA, AB i S tacka u kojoj prava
Qa Ra sece pravu BC, dokazati da je H(B, C; Pa , S).
304

421.
Uputstvo.
Koristiti postupak iz zadatka 419.
422. Ako su Qa i Ra tacke u kojima spolja upisani krug koji odgovara stranici BC trougla ABC dodiruje prave AC i AB, a Qb i Rc tacke u kojima spolja
upisani krugovi tog trougla dodiruju stranice AC i AB, dokazati da su prave
BC, Qa Ra , Qb Rc konkurentne.
Uputstvo.
Koristiti predhodna dva zadatka.
423. Ako su P , Q, R tacke u kojima konkurentne prave AO, BO, CO seku
prave odredene stranicama BC, CA, AB trougla ABC i S tacka u kojoj se seku
prave BC i QR, dokazati da je H(B, C; P, S).
Uputstvo.

Primenom Cevijeve
i Menelajeve teoreme nalazimo da je
BP CQ AR

=1
P C QA RB
i

BS CQ AR

= 1
SC QA RB

(slika 274.).

R
Q
O
B

Slika 274
Iz ovih dveju jednakosti sledi da je
BS
BP
=
PC
SC
pa je H(B,C;P,S)
424. Neka su P i P , Q i Q , R i R tacke pravih koje su odredene stranicama BC, CA, AB trougla ABC, takve da je H(B, C; P, P ), H(C, A; Q, Q ),
305

H(A, B; R, R ). Ako su pri tome prave AP , BQ, CR konkurentne, dokazati da


su tacke P , Q , R kolinearne; i obrnuto, ako su tacke P , Q , R kolinearne,
dokazati da su prave AP , BQ, CR konkurentne.
Uputstvo.
S obzirom da je
H(B, C; P, P ), H(C, A; Q, Q ), H(A, B; R, R )
tj.

BP
BP CQ
CQ AR
AR
= ,
= ,
= ,
PC
P C QA
Q A RB
RB

imamo da je

BP CQ AR
BP CQ AR

=
P C QA RB
P C QA RB

Ako su pri tome prave AP,BQ,CR konkurentne, prema Cevijevoj


teoremi imamo
da je
BP CQ AR

= 1,
P C QA RB
pa je i

BP CQ AR

= 1,
P C Q A R B

a prema Menelajevoj teoremi tacke P,Q,R kolinearne.Obratno, ako su tacke


P,Q,R kolinearne, imamo da je
BP CQ AR

= 1,
P C Q R R B
pa je

BP CQ AR

= 1,
P C QA RB

a prema Cevijevoj
teoremi, prave AP,BQ,CR konkurentne (sl. 277).

A
R
B

S
P

Q
C

P
Q

Slika 277
Napomena. Ako obelezimo sa S presek pravih AP,BQ,CR i sa s pravu kojoj
pripadaju tacke P,Q,R kaze se da je S pol prave s u onosu na trougao ABC,
a s polara tacke S u onosu na
306

425. Ako je ABCD proizvoljan cetvorougao, P tacka u kojoj se seku prave


AB i CD, Q tacka u kojoj se seku prave BC i AD, a R i S tacke u kojima prave
AC i BD seku pravu P Q, dokazati da je H(P, Q; R, S).
Uputstvo.
Tacke R,D,B su na stranicama PQ,QA,AP trougla APQ takve da se prave AR,

PD, QB seku u jednoj tacki C , pa je prema Cevijevoj


teoremi
P R QP AB

=1
RQ DA BP
(sl. 278).Tacke S,D,B su preseci jedne prave s pravama koje su odredena stranicama PQ,QA,AP trougla APQ,

D
C

Slika 278
te je prema Menelajevoj teoremi
P S QD AB

= 1.
SQ DA BP
Iz dobijenih dveju jednakosti sledi da je
PR PS
:
= 1,
RQ SQ
426. Ako neka prava p sece cetiri harmonijske prave a, b, c, d u raznim
tackama A , B , C , D , dokazati da je H(A , B ; C , D ).
Uputstvo.
Sobzirom da je H(a,b;c,d) postoji prava p koja sece prave a,b,c,d u tackama
A,B,C,D takvim da je H(A,B;C,D).Ako obelezimo sa E i F tacke u kojima
prava kroz B uporedna sa OA sece prave OC i OD, prema zadatku 401, bice
tacka B srediste duzi EF. Ako zatim obelezimo sa E i F tacke u kojima prava
kroz B uporedna sa OA sece OC i OD, bice
OB E OB E
i
OBF OB F
pa je (sl.279)
307

A
A

E
C

p
D
F

Slika 279
BE:BE=OB:OB. Iz dobijenih dveju proporcija sledi da je BE:BE=BF:BF.
No kako je BE=-BF, bice i BE=-BF, pa je tacka B srediste duzi EF. Stoga
je prema zadatku 401
427. Ako su prave c i d simetrale uglova koje odreduju prave a i b, dokazati
da je H(a, b; c, d).
Uputstvo.
Ako obelezimo sa O tacku u kojoj se seku prave a,b,c,d i sa p proizvoljnu pravu
koja ne sadrzi tacku O, a sece prave a,b,c,d u tackama recimo A,B,C,D, bice
prave OC i OD simetrale unutrasnjeg i spoljasnjeg ili spoljasnjeg i unutrasnjeg
ugla A trougla OAB (sl. 280),

c
C

Slika 280
pa je H(A,B;C,D), i prema tome H(a,b;c,d).

428. Ako obelezimo sa D podnozje visine iz temena A trougla ABC, sa E


i F tacke u kojima simetrale unutrasnjeg i spoljasnjeg ugla A seku pravu BC
i O srediste kruga opisanog oko tog trougla, dokazati da je H(AO, AD; AE, AF ).
Uputstvo.
Koristiti prethodni zadatak.
429. Ako su A , B , C tacke u kojima simetrale unutrasnjih uglova A, B, C
seku naspramne stranice trougla ABC, a B , C tacke u kojima se seku prave
308

A C i BB , A B i CC , dokazati da je prava AA simetrala ugla B AC .


Uputstvo.
Obelezimo sa S tacku u kojoj se seku prave AA, CC, i sa D presecnu tacku
pravih BC i AB. Primenom Paposove teoreme tj. zadatak,..., za cetvorougao
ACBS nalazimo da je H(B,A;D,C), pa je i H(AB,AA;AB,AC).Istim postupkom dokazuje se da je i H(AC,AA;AC,AB). Kod dobijenih dveju harmonijskih
cetvorki pravih tri para odgovarajucih pravih simetricna su u odnosu na pravu
AA,pa je i cetvrti par pravih simetrican u odnosu na pravu AA. Stoga je prava
AA simetrala ugla BAC (sl. 281).
430. Ako je H tacka u kojoj se seku prave odredene visinama AA , BB ,
CC trougla ABC, D srediste stranice BC i E tacka u kojoj se seku prave BB
i CC , dokazati da je AD HE.
Uputstvo.

A
B
C

B
B

Slika 281
Primenom Papasove teoreme tj. zadatka 425, na cetvorougao ACHB nalazimo
da je H(B,C;E,A),pa je i H(HB,HC;HE,HA).Ako obelezimosa AF pravu kroz
tacku A paralelnu sa stranicama BC, bice takode H(AC,AB;AD,AE). Kod dobijenih dveju harmonijskih cetvorki pravih,tri odgoverajuce prave upravne su
medu sobom, pa su upravne i cetvrte odgoverajuce prave AD i HE (sl. 282).
431. Ako je D srediste stranice BC, E podnozje visine iz temena A i F
tacka u kojoj krug opisan oko trougla ABC sece krug u kome je dijametar AD,
dokazati da je H(AB, AC; AE, AF ).
Uputstvo.

309

A
C

F
B

Slika 282
Ako obelezimo sa AP dijametar kruga opisanog oko trougla ABC i sa PQ paralelnu sa stranicom BC, bice (sl. 283) H(PB,PC;PD,PQ).No prave PB,PC,PD,PQ
respektivno su upravne na
432. Ako su M i N tacke u kojima prava kroz presek O dijagonala AC i
BD uporedna sa osnovicom AB sece krake AD i BC trapeza ABCD, dokazati
da je


1
1
1
1
.
=
+
MN
2 AB
CD
Uputstvo.

A
F

O
B

E
Q

D
P

Slika 283 Slika


Obelezimo sa S tacku u kojoj se seku prave AD i BC, sa P i Q tacke u kojima
prava OS sece prave AB i CD, a sa C1 i N1 tacke prave AB takve da je
CC1 k AD
i
N N1 k AD.
Prema zadatku 425 imamo da je H(P,Q;O,S), pa je prema zadatku 426
433. Ako je ABCD pravougaonik i ako su P , Q tacke prave AC takve da je
H(A, C; P, Q), a R, S tacke prave BD takve da je H(B, D; R, S), dokazati da
310

tacke P , Q, R, S pripadaju jednom krugu.

Uputstvo.

D
M

C1 N 1

Slika 284
1 1
1
1
= (
+
), tj.
AN1
2 AC1
AB
1
1 1
1
= (
+
).
AS
2 AB
AC
Presecna tacka O dijagonala AC i BD pravougaonika ABCD je srediste svake
od tih dijagonala, pa je prema zadatku 402
OC 2 = OR OQ
i
OD2 = OR OS.
S obzirom da su dijagonale AC i AD medu sobom jednake, jednake su i duzi
OC i OD, pa je
OP OQ = OR OS.
Otuda sleduje da tacke P,Q,R,S pripadaju
434. Ako obelezimo sa P i Q sredista lukova AB istog kruga k, sa R tacku
prave koja dodiruje krug k u tacki Q, sa D bilo koju od tacaka u kojima krug l
opisan oko trougla P QR sece pravu AB i sa C tacku u kojoj prava kroz tacku
Q paralelna sa pravom RD sece pravu AB, dokazati da je H(A, B; C, D).
Uputstvo.

311

Q
D

C
P
O

B
R

Slika 285
S obzirom da je ugao PQR prav, duz PR je precnik kruga l, pa je i ugao PDR
prav. Iz
RDP DiRD k QC

sledi da je QCP D u nekoj tacki K. Kako je duz PQ precnik kruga k i ugao


PKQ prav, tacka K je na krugu k. Pri tome je tacka K na onom luku AB
kruga k na kome nije tacka Q, pa je prava KQ simetrala unutrasnjeg a prava
KD simetrala spoljasnjg ugla K trougla KAB.

P
K

D
R

Slika 286
Stoga je (sl. 286) H(KA,KB;KC,KD), i prema tome H(A,B;C,D).
435. Ako su A, B, C, D cetiri kolinearne tacke, k1 i k2 krugovi od kojih prvi
sadrzi tacke A i B, a drugi tacke C i D, zatim P i R presecne tacke tih krugova,
S presecna tacka pravih P R i AB, T dodirna tacka jedne od dirki kroz S na k1
ili k2 , a X i Y tacke u kojima krug l(S, ST ) sece pravu AB, dokazati da je
H(A, B; X, Y ) i H(C, D; X, Y ).

312

Uputstvo.
S obzirom na uvedene oznake (sl. 287) bice
SX 2 = ST 2 = SP SQ = SA SB = SC SD
te je prema zadtku 402 H(A,B;X,Y) i H(C,D;X,Y).

5.2

Dvorazmera
cetiri ta
cke i dvorazmera
cetiri prave

Definicija 5.3. Dvorazmerom cetiri kolinearne tacke A, B, C, D nazivamo


kolicnik dveju razmera AC : DB i AD : DB. Ako tu dvorazmeru simbolicki
obelezimo sa R(A, B; C, D), bice
P
k1

k2

Q C
Y
S

Slika 287

R(A, B; C, D) =

AC AD
:
.
CB DB

Definicija 5.3. Dvorazmerom cetiri konkurentne ili paralelne prave nazivamo dvorazmeru tacaka A, B, C, D u kojima neka prava s sece prave a, b, c,
d. Ako dvorazmeru cetiri prave a, b, c, d obelezimo sa R(a, b; c, d), bice
R(a, b; c, d) = R(A, B; C, D).
436. Ako su A, B, C tri tacke orijentisane prave p i k realan broj, dokazati
da je tacka M na pravoj p za koju je R(A, B; C, M ) = k jednoznacno odredena.
Iz (A, B; C, M ) = k, tj.
AC
l AC
=
.
CB
k BC
Stoga je na pravoj p tacka M jednakoznacno odredena.
437. Ako su A, B, C tri tacke neke prave p, A i B tacke neke druge prave
tacka pravih AA i BB i D tacka
kroz C takve da je A C : B C = k, S preseCna
u kojoj prava kroz S uporedna sa A B sece p, dokazati da je R(A, B; C, D) = k.
313

A
B

Slika 288
Saglasno definiciji dvorazmere cetiri tacke jedne prave.
438. Ako su A, B, C, D cetiri tacke jedne prave, dokazati da je
R(A, B; C, D) = (

1
1
1
1

):(

).
AB
AD
AB
AD

Saglasno definiciji dvorazmere cetiri tacke A, B, C, D jedne prave, imamo da


je
(A, B; C, D) =

AC AD
AC(BA + AD)
AC(AD AB)
AC AD
:
=

=
=
=
BC BD
BC AD
AD(BA + AC)
AD(AC AB)

AC(AD AB) AD(AC AB)


1
1
1
1

=(

):(

).
AB AC AD
AB AC AD
AB
AD
AB
AC
439. Ako su O, A, B, C, D tacke orijentisane prave p, dokazati da je
R(A, B; C, D) =

OA OC OA OD
:
.
OB OC OB OD

Saglasno definiciji dvorazmere cetiri tacke jedne prave imamo da je


(A, B; C, D) =

AC AD
AO + OC AO + OD
OA OC OA OD
:
=
:
=
:
.
BC BD
BO + OC BO + OD
OB OC OB OD

440. Ako su A, A , B, B , C, C , D, D i O tacke neke prave takve da je


OA OA = OB OB = OC OC = OD OD = k, dokazati da je
R(A, B; C, D) = R(A , B ; C , D ).

314

Saglasno definiciji dvorazmere cetiri tacke, imamo da je


(A, B; C, D) =
=

k
( OC

k
( OC

AC BD
(OC OA)(OD OB)
AC AD
:
=
=
=
BC BD
BC AD
(OC OD)(OD OA)

k
k
OA )( OD
k
k
OB )( OD

k
OB )
k
OA )

(OC OA )(OD OB
=
(OC OB )(OD OA )

A C A D
A C B D
=
:
= (A , B ; C , D )
B C A D
B C B D

441. Ako su A, B, C, D, A , B , C , D , O kolinearne tacke takve da je OA


OA = OB OB = OC OC dokazati da je

R(A, B; C, C ) = R(A , B ; C , C).

Zadatak predstavlja specijala slucaj prethodnog.


442. Ako su A, B, C, D, A , B , C , D , M, N kolinearne tacke takve da je
R(A, A ; M, N ) = R(B, B ; M, N ) = R(C, C ; M, N ) = R(D, D ; M , N ),
dokazati da je
R(A, B; C, D) = R(A , B ; C , D ).
Ako obelezimo sa O srediste duzi M N , prema poznatom stavu bice
OA OA = OB OB = OC OC = OD OD = OM 2 ,
pa je prema prethodnom zadatku
(A, B; C, D) = (A , B ; C , D).

443. Ako su A, B, C, D, C , D kolinearne tacke takve da je


R(A, B; C, D) = R(A, B; C , D ),
dokazati da je
R(A, B; C, C ) = R(A, B; D, D ).
S obzirom da je
(A, B; C, D) = (A, B; C , D ),

315

tj.

AC AD
AC AD
:
= :
CB DB
CB CB

bice

AD AD
AC AC
: =
:
,
CB C B
DB D B

tj.
(A, B; C, C ) = (A, B; D, D ).
444. Ako su A, B, X, Y, Z tacke jedne prave, dokazati da je
R(A, B; X, Y ) R(A, B; Y, Z) R(A, B; Z, Y ) = 1.

Saglasno definiciji dvorazmere cetiri tacke, imamo da je


(A, B; X, Y ) (A, B; Y, Z) (A, B; Z, X) =
=(

AX AY AY AZ AZ AX
:
)(
:
)(
:
) = 1.
BX BY BY BZ BZ BX

445. Ako su A, B, C, D cetiri razne kolinearne tacke, dokazati da je


R(D, A; B, C) R(D, B; C, A) R(D, C; A, B) = 1.

Koristi postupak iz prethodnog zadatka.


446. Ako su A, B, C, D, E, F razne kolinearne tacke takve da je
R(A, B; C, F ) = R(C, D; B, F ) = R(E, B; D, F ) = 1,
dokazati da je
R(A, E; B, F ) = 1.
Koristiti definiciju dvorazmere cetiri kolinearne tacke.
447. Ako su A, B, C, D cetiri razne kolinearne tacke dokazati da je
H(A, B; C, D) ako i samo ako je
R(A, B; C, D) = R(A, B; D, C).

Koristiti definiciju dvorazmere cetiri kolinearne tacke.

316

448. Ako su A, B, C, D, E proizvoljne tacke jedne prave, dokazati da je


R(A, B; C, D) R(A, B; D, E) = R(A, B; C, E).

Saglasno definiciji dvorazmere cetiri tacke jedne prave, mamo da


(A, B; C, D) (A, B; D, E) = (

AC AD
AD AE
AC AE
:
)(
:
)=
:
=
CB DE
DB EB
CB EB

= (A, B; C, E)
449. Ako su A, B, C, D cetiri tacke orijentisane prave, dokazati da je
(a)
R(A, B; C, D) R(A, B; D, C) = 1;
(b)
R(A, B; C, D) + R(A, C; B, D) = 1.
Saglasno definiciji dvorazmere cetiri tacke, imamo da je
(a)(A, B; C, D) (A, B; D, C) = (

AC AD AD AC
:
)(
:
)=1
BC BD BD BC

AB AD
AC AD
:
+
:
=
BC BD CB CD
(AB + BC)(BC + CD) AB CD
AB + BC + CD
AC BD AB CD

=
=
= 1.
=
AD BC AD BC
AD BC
AD
(b)(A, B; C, D) + (A, C; B, D) =

450. Ako su A, B, C, D cetiri tacke orijentisane prave, dokazati da je


R(A, B; C, D) = R(B, A; D, C) = R(C, D; A, B) = R(D, C; B, A).

Dokaz neposredno sleduje iz definicije dvorazmere cetiri tacke.


451. Ako su A, B, C, D cetiri tacke na orijentisanoj pravoj takve da je
R(A, C; B, D) = k, dokazati da je
(a)

R(A, B; D, C) =

1
k

(b)

R(A, C; B, D) = 1 k

(c)

R(A, C; D, B) =
317

1
1k

(d)

R(A, D; C, B) =

k
k1

(e)

R(A, D; B, C) =

k1
k

Uputstvo.
Koristeci zadatak 449 imamo da je
(a)

R(A, B; D, C) =

(b)

1
1
= ;
R(A, B; C, D)
k

R(A, C; B, D) = 1 R(A, B; C, D) = 1 k;

(v)

R(A, C; D, B) =

(g)

1
1
=
;
R(A, C; B, D)
1k

R(A, D; C, B) = 1 R(A, C; D, B) =

(d)

R(A, D; B, C) =

1
;
1k

k1
1
=
.
R(A, C; D, B)
k

Napomena. Od cetiri tacke A,B,C,D jedne prave mogu se permutovanjem obrazovati 41, dakle 24 dvorazmere, koje se dele u sest grupa. Svaka od tih grupa
ima po cetiri dvorazmere koje su, prema zadatku... medu sobom jednake. Stoga
postoji samo 24, dakle sest razlicitih vrednosti kojima su jednake dvorazmere
obrazovane od tacaka A,B,C,D.

452. Ako su A, B, C, D, E tacke jedne prave takve da je R(A, B; C, D) = m


i R(A, B; C, E) = n dokazati da je
(a)

R(A, B; E, D) =

(b)

R(A, C; E, D) =

m
n

m1
n1

Uputstvo.
Saglasno definiciji dvorazmere, nalazimo da je
(a)

R(A, B; E, D) =

m
R(A, B; C, D)
= ;
R(A, B; C, E)
n
318

(b)

R(A, C; E, D) =

(v)

(g)

1 R(A, B; C, D)
m1
R(A, C; B, D)
=
=
;
R(A, C; B, E)
1 R(A, B; C, E)
n1
R(A, C; E, D)
n(m 1)
=
;
A, B; E, D
m(n 1)

R(B, C; E, D) =

R(C, D; B, E) =

R(A, D; B, E)
1 R(A, B; D, E)
mn
=
=
.
R(A, C; B, E)
1 R(A, B; C, E)
m(1 n)

453. Ako dve prave p i p seku cetiri konkurentne prave a, b, c, d, prva u


tackama A, B, C, D a druga u tackama A , B , C , D , dokazati da je
R(A, B; C, D) = R(A , B ; C , D ).

Uputstvo.
R
R
O
S
S

Q
A

C
Q
P

Slika 289
Ako obelezimo sa P ,Q,P ,Q tacke u kojima prave kroz A,B,A ,B uporedne
s pravom d seku pravu c i sa R,S,R ,S tacke u kojima prave kroz A,B,A ,B
uporedne s pravom c seku pravu d (sl. 289), imamo da je
AC
AP
=
BC
BQ
i

Otuda je

AD
AR
=
BD
BS
AP AR
AC AD
:
=
:
,
BC BD
BQ BS

tj.
R(A, B; C, D) =
319

AP BQ
:
.
AR BS

Isto tako je
R(A , B ; C , D ) =
S obzirom da je

A P B Q
:
.
A R B S

AP
A P
=
AR
AR

B Q
BQ
= ,
BS
BS

bice
R(A, B; C, D) = R(A , B ; C , D ).

454. Neka su A, B, C, D tacke neke prave p i A , B , C , D tacke neke


druge prave p takve da je R(A, B; C, D) = R(A , B ; C , D ). Ako se pri tome
prave AA , BB , CC seku u nekoj tacki S, dokazati da i prava DD sadrzi
tacku S.
Uputstvo.

Slika 290
Ako obelezimo sa D tacku u kojoj prava SD sece pravu p , prema prethodnom
zadatku bice
R(A, B; C, D) = R(A , B ; C , D ).
Iz ove jednakosti i iz
R(A, B; C, D) = R(A B ; C , D )
sledi da je
R(A , B ; C , D ) = R(A , B ; C , D ).

Stoga je D D , te prava DD sadrzi tacku S (sl.290).

320

455. Ako su A, B, C tri tacke jedne prave s i A , B , C tri tacke neke druge
prave s , dokazati da tacke P , Q, R u kojima se seku prave BC i B C, CA i
C A, AB i B A pripadaju jednoj pravoj.
Uputstvo.
Ako obelezimo sa S,M ,N tacke u kojima se seku prave s i s , AB i BC ,
AC i CB , (v.sl. 291)
A

C
S

Q
B

P
N

s
M

Slika 291
imamo da je
Stoga je prema zadatku .....
R(A , B ; C , S) = R(R, B ; M, A)
i
R(A , B ; C , S) = R(Q, N ; C , A)
i prema tome
R(R, B ; M, A) = R(Q, N, C , A).

Sad su prema prethodnom zadatku prave RQ, B N , M C konkurentne. No


prave B N i M C seku se u tacki P , te i prava QR sadrzi tacku P .
Napomena: Pravu koja sadrzi tacke P ,Q,R nazivamo Paposovom pravom dveju
trojki kolinearnih tacaka A,B,C i A ,B ,C .
456. Ako su E i F proizvoljne tacke naspramnih stranica AB i CD cetvorougla
ABCD, dokazati da tacke P , Q, R u kojima se seku dijagonale cetvorougla
ABCD, AEF D, EBCF pripadaju jednoj pravoj.
Uputstvo.
Primeniti Paposovu teoremu, tj. prethodni zadatak.
457. Ako prave odredene dvema tetivama AB i CD nekog kruga k sadrze
srediste S tetive M N tog istog kruga, dokazati da su tacke X i Y u kojima
321

prava M N sece prave AD i BC simetricne medu sobom u odnosu na tacku S.


Uputstvo.
S obzirom da su tacke M ,N ,S,X projekcije tacaka M ,N ,B,D iz tacke A na
pravoj M N , a tacke M ,N ,Y ,S projekcije istih tacaka M ,N ,B,D iz tacke C na
pravoj M N , imamo da je
R(M, N ; S, X) = R(AM, AN ; AB, AD), R(M, N ; Y, S) = R(CM, CN ; CB, CD).
Kako je prema zadatku.......

B
S

Slika 292
R(AM, AN ; AB, AD) = R(CM, CN ; CB, CD)
bice
R(M, N ; S, X) = R(M, N ; Y, S).
Stoga je

MY MS
MS MX
:
=
:
,
SN XN
Y N SN

i prema tome
MX
NY
=
.
XN
YM
Otuda sleduje da su tacke X i Y simetricne u odnosu na srediste S duzi M N .
458. Ako su O i O bilo koje dve tacke ravni trougla ABC, a P , Q, R tacke
u kojima prave O A, O B, O C seku prave BC, CA, AB, dokazati da je
R(B, C; P, P ) R(C, A; Q, Q ) R(A, B; R, R ) = 1.

Uputstvo.

322

Q
R
R

O
B

Slika 293

Primenom Cevijeve
teoreme nalazimo da je (v.sl. 293)
BP CQ AR

=1
P C QA RB
i

BP CQ AR

= 1.
P C Q A R B

Deljenjem odgovarajucih strana ovih dveju jednakosti, nalazimo da je


(

BP BP CQ CQ AR AR
:
)(
:
)(
:
)=1
P C P C QA Q A RB R B

tj. da je
R(B, C; P, P )R(C, A; Q, Q )R(A, B; R, R ) = 1.
459. Neka su P , P ; Q, Q ; R, R tacke pravih koje su odredene stranicama
BC, CA, AB trougla ABC takve da je
R(B, C; P, P ) R(C, A; Q, Q ) R(A, B; R, R ) = 1.

Uputstvo.

Primenom Cevijeve
teoreme nalazimo da je
BP CQ AR

=1
P C QA RB
Iz ove jednakosti i iz jednakosti
R(B, C; P, P ) R(C, A; Q, Q ) R(A, B; R, R ) = 1
sledi da je
BP CQ AR

= 1,
P C Q A R B
323


pa se prema Cevijevoj
teoremi prave AP ,BQ ,CR seku u jednoj tacki.
460. Ako dve prave s i s seku prave odredene stranicama BC, CA, AB
trougla ABC u takama P , Q, R i P , Q , R , dokazati da je
R(B, C; P, P ) R(C, A; Q, Q ) R(A, B; R, R ) = 1.

Uputstvo.

A
R

R
B

s
P

C
s

P
Q

Slika 294
Primenom Menelajeve teoreme nalazimo da je (v.sl. 294)
BP CQ AR

= 1
P C QA RB
i

BP CQ AR

= 1.
P C Q A R B

Deljenjem odgovarajucih strana ovih jednakosti dobijamo da je


(

BP BP CQ CQ AR AR
:
)(
:
)(
:
)=1
P C P C QA Q A RB R B

tj. da je
R(B, C; P, P ) R(C, A; Q, Q ) R(A, B; R, R ) = 1.

461. Neka su P , P ; Q, Q ; R, R tacke pravih koje su odredene stranicama


BC, CA, AB, trougla ABC takve da je
R(B, C; P, P ) R(C, A; Q, Q ) R(A, B; R, R ) = 1.
Ako pri tome tacke P , Q, R pripadaju nekoj pravoj s, dokazati da i tacke P ,
Q , R takode pripadaju nekoj pravoj s .
Uputstvo.

324

S obzirom da su tacke P ,Q,R kolinearne ,prema direktnoj Menelajevoj teoremi imamo da je


BP CQ AR

= 1.
P C QA RB
Iz ove jednakosti i iz jednakosti
R(B, C; P, P ) R(C, A; Q, Q ) R(A, B; R, R ) = 1
nalazimo da je

BP CQ AR

= 1
P C Q A R B

tj. prema obratnoj Menelajevoj teoremi tacke P ,Q ,R pripadaju jednoj pravoj.


462. Ako je S proizvoljna tacka u ravni trougla ABC i ako su P , Q, R tacke
u kojima prave SA, SB, SC seku respektivno prave BC, CA, AB, a P , Q , R
tacke u kojima neka prava s sece prave BC, CA, AB, dokazati da je
R(B, C; P, P ) R(C, A; Q, Q ) R(A, B; R, R ) = 1.

Uputstvo.
S

Q
C
B

Slika 295

Primenom Cevijeve
i Menelajeve teoreme nalazimo da je (v.sl. 295)
BP CQ AR

=1
P C QA RB
i

BP CQ AR

= 1.
P C Q A R B
Deljenjem odgovarajucih strana ovih jednakosti dobijamo da je
(

BP BP CQ CQ AR AR
:
)(
:
)(
:
) = 1,
P C P C QA Q A RB R B

tj. da je
R(B, C; P, P )R(C, A; Q, Q )R(A, B; R, R ) = 1.
325

463. Ako obelezimo sa S proizvoljnu tacku ravni trougla ABC, sa P , Q, R


tacke u kojima prave SA, SB, SC seku respektivno prave BC, CA, AB i sa P ,
Q , R tacke pravih BC, CA, AB takve da je
R(B, C; P, P ) R(C, A; Q, Q ) R(A, B; R, R ) = 1,
dokazati da tacke P , Q , R pripadaju jednoj pravoj.
Uputstvo.
S obzirom da se prave AP , BQ, CR seku u jednoj tacki, prema direktnoj

Cevijevoj
teoremi, imamo da je
BP CQ AR

= 1.
P C QA RB
Iz ove jednakosti
R(B, C; P, P ) R(C, A; Q, Q ) R(A, B; R, R ) = 1
sledi da je

BP CQ AR

= 1.
P C Q A R B

te prema obratnoj Menelajevoj teoremi tacke P ,Q ,R pripadaju jednoj pravoj.


464. Ako su P , Q, R tacke u kojima neka prava s sece prave odredene
stranicama BC, CA, AB trougla ABC, a P , Q , R tacke pravih BC, CA, AB
takve da je
R(B, C; P, P ) R(C, A; Q, Q ) R(A, B; R, R ) = 1.

Uputstvo.
S obzirom da su tacke P ,Q,R kolinearne, prema direktnoj Menelajevoj teoremi imamo da je
BP CQ AR

= 1.
P C QA RB
Iz ove jednakosti i iz jednakosti
R(B, C; P, P ) R(C, A; Q, Q ) R(A, B; R, R ) = 1
sledi da je
BP CQ AR

= 1.
P C Q A R B

pa se prema obratnoj Cevijevoj


teoremi prave AP ,BQ ,CR seku u jednoj tacki.

326

465. Ako su uglovi koje odreduju cetiri prave a, b, c, d jednog pramena


jednaki odgovarajucim uglovima koje odreduju cetiri prave a , b , c , d drugog
pramena, dokazati da je
R(a, b; c, d) = R(a , b ; c , d ).

Uputstvo.
Neka su A,B,C,D tacke u kojima neka prava p sece prave a,b,c,d i A ,B ,C ,D
tacke u kojima neka prava p sece prave a ,b ,c ,d .Obelezimo sa P i Q tacke u
kojima prave kroz A i B uporedne sa d seku c, sa R i S tacke u kojima prave
kroz A i B uporedne sa c seku d,sa P i Q tacke u kojima prave kroz A i B
uporedne sa d seku c , sa R i S tacke u kojima prave kroz A i B uporedne
sa c seku d (sl.296).

O
S

Q
A

Q
D

A C

Slika 296
Pri tome je

pa je

AC
AP
=
BC
BQ
AD AR
,
BD BS
AC AD
AP AR
:
=
:
BC BD
BQ BS

tj.
R(A, B; C, D) =
Isto tako je
R(A , B ; C , D ) =
No

AP BQ

.
AR BS
A P B Q

.
A R B S

A P
AP
=
AR
AR

327

B Q
BQ
= ,
BS
BS

pa je
R(A, B; C, D) = R(A , B ; C , D ),
i prema tome
R(a, b; c, d) = R(a , b ; c , d ).

466. Ako su A,B,C,D,S,S proizvoljne tacke nekog kruga k, dokazati da je


R(SA, SB; SC, SD) = R(S A, S B; S C, S D).

re
senje: S obzirom da su tacke A,B,C,D,S,S na jednom krugu,prave SA,SB,SC,SD
zahvataju uglove jednake s odgovarajucim uglovima koje zahvataju prave S A,S B,S C,S D,pa
je prema prethodnom zadatku (sl.297)

D
A

B
C

sl.297

R(SA, SB; SC, SD) = R(S A, S B; S C, S D).

328

467. Ako su S,S , A,B,C proizvoljne tacke nekog kruga k, a D tacka u ravni
toga kruga takva da je
R(SA, SB; SC, SD) = R(S A, S B; S C, S D).
dokazati da i tacka D pripada krugu k.
re
senje: Pretpostavimo li da prava SD sece krug k sem u tacki S u jos nekoj
tacki D prema prethodnom zadatku bice
R(SA, SB; SC, SD ) = R(S A, S B; S C, S D ).
tj.
R(SA, SB; SC, SD) = R(S A, S B; S C, S D ).
tj.
R(SA, SB; SC, SD ) = R(S A, S B; S C, S D).
pa je
R(S A, S B; S C, S D) = R(S A, S B; S C, S D ).
Stoga su prave S D i S D istovetne , pa su i tacke D i D u kojima one seku
pravu SD takode istovetne.Iz toga sledi da tacka D pripada krugu k.

6
6.1

GEOMETRIJA KRUGOVA
Ortogonalni krugovi

Definicija 6.1. Uglom dveju krivih linija l1 i l2 u njihovoj presecnoj tacki P


nazivamo ostar ili prav ugao odredjen tangentama u toj tacki na tim linijama.
Specijalno, ako je taj ugao prav kaze se da su linije l1 i l2 u tacki P medju
sobom upravne ili ortogonalne.
468. Dokazati da su uglovi u presecnim tackama dvaju krugova medju sobom jednaki.
Resenje: Ako su O1 i O2 sredista dvaju ortogonalnih krugova k1 i k2 koji se seku
u tackama P i Q, bice trouglovi O1 O2 P i O1 O2 Q podudarni. Otud sleduje da
su i uglovi pod kojima se seku krugovi k1 i k2 u tackama P i Q medju sobom
jednaki.
469. Ako su dva kruga k1 i k2 ortogonalna, dokazati da dirke u presecnim
tackama na bilo kojem od tih krugova sadrze srediste drugog kruga.
Resenje: Obelezimo sa A i B presecne tacke krugova k1 i k2 a sa AT1 i AT2 dirke
u tacki A na krugovima k1 i k2 . S obzirom da su krugovi k1 i k2 ortogonalni,
ugao T1 AT2 je prav. Stoga dirka AT1 sadrzi srediste kruga k2 , a dirka AT2
kruga k2 sadrzi srediste kruga k1 .
470. Ako su O1 i O2 sredista i r1 i r2 poluprecnici dvaju ortigonalnih krugova
k1 i k2 , dokazati da je O1 O22 = r12 + r22 .
Resenje: Ako je A bilo koja od presecnih tacaka ortogonalnih krugova k1 i k2 ,
bice duz O1 O2 hipotenuza pravouglog trougla AO1 O2 , pa je
O1 O22 = AO12 + AO22
329

i prema tome,
O1 O22 = r12 + r22

471. Ako su O1 i O2 sredista i r1 i r2 poluprecnici dvaju krugova k1 i k2 pri


cemu je O1 O2 = r12 + r22 , dokazati da se krugovi k1 i k2 seku pod pravim uglom.
Resenje: S obzirom da je
(r1 r2 )2 < r12 + r22 < (r1 + r2 )2
i
r12 + r22 = O1 O22
bice
r1 r2 < O1 O22 < r1 + r2
pa se prema poznatom stavu krugovi k1 i k2 seku. Ako je A bilo koja od
presecnih tacaka, kod trougla AO1 O2 bice
AO12 + AO22 = O1 O22
pa je ugao O1 AO2 prav. Stoga su krugovi k1 i k2 ortogonalni.
472. Dokazati da je proizvod poluprecnika r1 i r2 dvaju ortogonalnih krugova
k1 i k2 jednak polovini proizvoda zajednicke tetive tih krugova i duzi koja spaja
sredista O1 i O2 krugova k1 i k2 .
Resenje: Srediste N duzi AB je na duzi O1 O2 i pri tome AN ortogonalno na
O1 O2 pa je kod pravouglog trougla AO1 O2
AO1 AO 2 = O1 O2 AN
i prema tome je
r1 r2 =

1
O1 O2 AN
2

473. Dva kruga k1 i k2 ciji su poluprecnici jednaki duzi r seku se pod pravim
uglovima. Ako je A jedna od presecnih tacka i s prava kroz A koja sece k1 i k2
u tackama B i C, dokazati da je
AB 2 + AC 2 = 4r2 .

330

C
M2
A
M1
B

O2
O1

Slika 298.
Resenje: Ako su O1 i O2 sredista krugova k1 i k2 , a M1 i M2 sredista tetiva AB
i AC imamo da je trougao AO1 M1 slican trouglu O2 AM2 , pa je O1 M1 = AM2 .
S obzirom da je ugao M1 trougla AO1 M1 bie
AM12 + M1 O12 = AO12
pa je
AM12 + AM22 = AO12
i prema tome
AB 2 + AC 2 = 4r2
(slika 298.)
474. Ako su data dva ortogonalna ugla, dokazati da su svake dve dijametralno suprotne tacke C i D jednog od tih krugova harmonijski spregnute s
tackama A i B u kojima prava CD sece drugi krug.

331

k
P

D
O
C

Slika 229.
Resenje: Neka su k i l dva ortogonalna kruga, C i D dijametralno suprotne
tacke kruga l, a A i B tacke i kojima prava CD sece krug k. Obelezimo sa O
srediste kruga l i sa P jednu od presecnih tacaka krugova k i l. S obzirom da
su krugovi k i l ortogonalni, tacka O je na tangenti kruga k u tacki P , pa je
OA OB = OP 2 = OC 2 = OD2
Stoga su, prema zadatku 402., tacke C i D harmonijski spregnute s tackama A
i B (slika 299.).
475. Ako je par tacaka C, D harmonijski spregnut s parom tacaka A, B,
dokazati da je krug kome je duz CD precnik ortogonalan na svakom krugu koji
sadrzi tacke A, B.
Resenje: Neka je k bilo koji krug koji sadrzi tacke A i B, l krug kome je duz
CD precnik i O srediste duzi CD. Kako su tacke C i D harmonijski spregnute
s tackama A i B, jedna od tacaka, npr. C, nalazi se izmedju tacaka A i B, a
druga iza A u odnosu na B, ili iza B u odnosu na A. Stoga je tacka C u krugu
k, a tacka D izvan kruga k. Krug l sadrzi tacku C koja se nalazi u krugu k i
tacku D koja se nalazi izvan kruga k, prema tome, krugovi k i l se seku. Ako
je P bilo koja od presecnih tacaka, imamo da je
OA OB = OC 2 = OP 2
pa je prava OP dirka kruga k u tacki P . Otud sleduje da su krugovi k i l
ortogonalni.
476. Ako je srediste O kruga l na krugu k i ako proizvoljna prava kroz tacku
O sece krug k u tacki A, pravu koja sadrzi zajednicku tetivu krugova l i k u
tacki B i krug l u tackama C i D, dokazati da su tacke C i D harmonijski
spregnute s tackama A i B.

332

A
P
C
A

B
D
Q

Slika 300.
Resenje: Neka prava kroz O i srediste kruga k sece krug k u tacki A , pravu
P Q u tacki B i krug l u tackama C i D . S obzirom da je duz P B visina koja
odgovara hipotenuzi pravouglog trougla OP A , bice
OA OB = OP 2 = OC 2 = OD2
pa su, prema zadatku 402., tacke C i D harmonijski spregnute s tackama A i
B . Stoga je, prema zadatku 475., krug l kome je duz C D precnik ortogonalan
na svakom krugu koji sadrzi tacke A i B , dakle i na krugu k koji je opisan
oko tetivnog cetvorougla A B BA. Kako su krugovi l i k ortogonalni, prema
zadatku 474., bice dijametralno suprotne tacke C i D kruga l harmonijski spregnute sa tackama A i B u kojima prava CD sece krug k (slika 300.)
477. Dokazati da je opisani krug oko trougla odredjenog sa pravama koje
sadrze dijagonale potpunog cetvorougla, ortogonalna na krugovima kojima su
precnici dijagonale tog cetvorougla.

333

T
Q
S
D

C
R

Slika 301.
Resenje: Obelezimo sa ABCD bilo koji potpuni cetvorougao, sa P , Q i R njegove dijagonalne tacke, a sa S i T tacke u kojima prave odredjene sa dvema
dijagonalama, npr. AC i BD seku pravu odredjenu trecom dijagonalom P Q.
Pri tome su tacke R, S i T tacke trougla odredjenog pravama koje sadrze dijagonale AC, BD i P Q potpunog cetvorougla ABCD. S obzirom da su tacke S
i T harmonijski spregnute s tackama P i Q, prema zadatku 475., krug kome je
dijagonala P Q precnik ortogonalan je na svakom krugu koji sadrzi tacke S i T ,
dakle i na krugu l koji je opisan oko trougla RST (slika 301.)
478. Ako je H ortocentar trougla ABC, dokazati da su krugovi k1 i k2 s
precnicima AH i BC ortogonalni.
Resenje: Duz kojoj su krajevi sredista O1 i O2 krugova k1 i k2 precnik Ojlerovog kruga trougla ABC, a podnozja B i C visina iz temena B i C na tom
Ojlerovom krugu, pa su uglovi O1 B O2 i O1 C O2 pravi. Kako su tacke B i C
presecne tacke krugova k1 i k2 , a uglovi O1 B O2 i O1 C O2 pravi, krugovi k1 i
k2 su ortogonalni.
479. Ako su S, Sa , Sb , Sc sredista upisanih krugova trougla ABC, dokazati
da su krugovi l1 i l2 s precnicima SSa i Sb Sc medju sobom ortogonalni.
Resenje: Sredista M i N krugova l1 i l2 su krajevi precnika kruga opisanog
oko trougla ABC, pa su uglovi M BN i M CN pravi. Kako su B i C presecne
tacke krugova l1 i l2 , a uglovi M BN i M CN pravi, krugovi l1 i l2 su ortogonalni.
480. Ako je AB precnik i O srediste nekog kruga k, a P proizvoljna tacka
tog kruga, dokazati da su krugovi OP A i OP B ortogonalni.
Resenje: Ako sa O1 i O2 obelezimo sredista krugova OP A i OP B, bice
OO1 P = 2 OAP i OO2 P = 2 OBP
S obzirom da su poluprave O1 O2 i O2 O1 raspolovnice uglova OO1 P i OO2 P ,
bice
O2 O1 P = OAP i O1 O2 P = OBP
334

pa su trouglovi P O1 O2 i P AB slicni, i prema tome, uglovi O1 P O2 i AP B


jednaki. Kako je ugao AP B prav i ugao O1 P O2 je prav, pa su krugovi OP A i
OP B ortogonalni.

6.2

Pol i polara u odnosu na krug

481. Ako je P proizvoljna tacka koja se nalazi u krugu k ili izvan njega,
dokazati da skup svih tacaka, od kojih svaka s tackom P obrazuje par harmonijski spregnutih s parom koji se nalazi na krugu k i kolinearan je s tackom P ,
pripada jednoj pravoj, polari tacke P u odnosu na krug k.

Q
l
k

B
O
A

Slika 302.
Resenje: Obelezimo sa O srediste kruga k, sa A i B tacke u kojima prava OP
sece krug k, sa A i B tacke u kojima bilo koja druga prava kroz P sece k, sa Q
tacku koja sa tackom P obrazuje par harmonijski spregnut sa parom A , B i sa
Q podnozje upravne p kroz tacku Q na pravoj AB. Ugao P QQ je prav, pa je
duz P Q precnik kruga l, opisanog oko trougla P QQ . S obzirom da su tacke P
i O harmonijski spregnute sa tackama A i B , krug l kome je duz P Q precnik,
ortogonalan je na svakom krugu koji sadrzi tacke A i B , dakle, i na krugu
k (vidi zadatak 475.). Kako su krugovi k i l ortogonalni, A i B dijametralno
suprotne tacke kruga k, a P i Q tacke u kojima prava AB sece krug l, bice
prema zadatku 474., par tacaka P i Q harmonijski spregnut sa parom tacaka
A, B. Tacka Q je jedina tacka koja s P obrazuje par harmonijski spregnut sa
parom tacaka A i B. Prema tome, skup svih tacaka koje odgovaraju tacki Q ,
a nalaze se na secicama kruga k kroz tacku P , pripada pravoj p, koju nazivamo
polara tacke P u odnosu na krug k (slika 302.)
482. Ako polara tacke P u odnosu na krug k sadrzi tacku Q, dokazati da i
polara tacke Q u odnosu na krug k sadrzi tacku P .

335

Q
k
B
Q
O

B
P

Slika 303.
Resenje: Obelezimo sa O srediste i sa r poluprecnik kruga k, sa p polaru tacke
P u odnosu na krug k, sa P presek prave OP sa tom polarom i sa Q podnozje
upravne q kroz tacku P na pravoj OQ. S obzirom da je prava p polara tacke P
u odnosu na krug k, tacke P i P su harmonijski spregnute sa tackama A i B u
kojima prava OP sece krug k, pa je
OP OP = OA2 = r2
Uglovi P P Q i P Q Q su pravi, pa je cetvorougao P P QQ tetivan. Stoga je
OP OP = OQ OQ = r2
pa su tacke Q i Q harmonijski spregnute s tackama A i B u kojima prava OQ
sece krug k. Otud sleduje da je prava q polara tacke Q u odnosu na krug k, i
prema tome je tacka P na polari tacke Q u odnosu na krug k (slika 303.).
483. Ako se pol prave p nalazi na pravoj q, dokazati da se i pol prave q
nalazi na pravoj p.
Resenje: Neka su P i Q polovi pravih p i q. S obzirom da se tacka P nalazi na
pravoj q, polara tacke Q sadrzi tacku P , te prema prethodnom zadatku (zadatak 482.) i polara tacke P sadrzi tacku Q.
484. Dokazati da se polare svih tacaka jedne prave p u odnosu na neki krug
k seku u jednoj tacki P , poluprave p u odnosu na krug k.
Resenje: Neka je Q proizvoljna tacka prave p, a q njena polara u odnosu na krug
k. S obzirom da polara p tacke P sadrzi tacku Q i polara q take Q sadrzi tacku
P , prema ovome, polare svih tacaka prave p seku se u jednoj tacki, poluprave p
u odnosu na krug k.
485. Neka je p proizvoljna prava pramena S i P pol prave p u odnosu na
krug k. S obzirom da polara tacke P sadrzi tacku S, i polara tacke S sadrzi
tacku P , prema tome, polovi svih pravih pramena S nalaze se na jednoj pravoj,
polari s, tacke S u odnosu na krug k.
336

p
b
B
O

A
a

sl. 304
486. S obzirom da polara tacke A sadrzi tacku P , i polara tacke P sadrzi
tacku A. Isto tako, polara tacke B sadrzi tacku P , pa i polara tacke P sadrzi
tacku B. Otud sleduje da je prava AB polara tacke P u odnosu na krug k (sl.
304).
P
A
C

k
B

sl. 305
487. S obzirom da su A i B dodirne tacke tangenata kroz S na krug k, prava
AB je polara tacke S u odnosu na krug k. Tacka P pripada polari tacke S, pa
je i tacka S na polari tacke P . Dodirna tacka C tangente iz P na krug k takode
je na polari tacke P . Otud sleduje da je prava SC polara tacke P u odnosu na
krug k (sl. 305).
Q

C
D

N
R

337

sl. 306
488. Ako sa M i N obelezimo tacke u kojima prava QR sece prave AB i
CD, bice tacke M i P harmonijski spregnute sa tackama A i B, a tacke N i P
harmonijski spregnute s tackama C i D, pa je prava M N , odnosno QR polara
tacke P u odnosu na krug k (sl. 306).

s
A
B
C
D

B1
C1
D1

C
D

sl. 307
489. Obelezimo sa O srediste datog pramena, sa p polaru tacke O u odnosu
na krug k i sa A1 tacku u kojoj se seku prave AA i p (sl. 307). Prema zadatku
488 tacke B1 , C1 , D1 u kojima se seku prave AB i A B, AC i A C, AD i
A D su takode na pravoj p pa je R(A A, A B; A C, A D) = R(A A1 , A B1 ;
A C1 , A D1 ) = R(AA1 , AB1 ; AC1 , AD1 ) = R(AA , AB ; AC , AD ) = Stoga
je prema zadatku 466 R(SA, SB; SC, SD) = R(S A , S B ; S C , S D ).

a
k

X
B

O S

B
H

sl. 308
490. Neka su B i C bilo koje dve dijametralno suprotne tacke kruga k,
nekolinearne s tackom A. Tacke A, B, C su temena trougla, obelezimo sa AA ,
BB , CC njegove visine, sa H njegov ortocentar i sa S presek pravih BC i
B C . S obziorm da je duz BC precnik kruga k, a uglovi BB C i BC C pravi,
tacke B i C su takode na krugu k. Ako sa X i Y obelezimo tacke u kojima
prava SH sece prave AB i AC, bice tacke A i X harmonijski spregnute s tackom
B i C , a tacke A i Y harmonijski spregnute s tackama B i C, pa je prava XY
polara tacke A u odnosu na krug k. Stoga je ortocentar H trougla ABC na
polari tacke A u odnosu na krug k (sl. 308).
338

O
d
P

A C B

sl. 309
491. S obzirom da se tacke A, B, C, D nalaze na jednoj pravoj, prema
zadatku 484, njihove polare a, b, c, d seku se u jednoj tacki P polu prave p u
odnosu na krug k. Ako sa O obelezimo srediste kruga k bice OA, OB, OC,
OD cetiri harmonijske prave. Kako su prave a, b, c, d pramena P upravne
harmonijskim pravama OA, OB, OC, OD, i prave a, b, c, d su harmonijske (sl.
309).
492. S obzirom da su prave a, b, c, d harmonijske, one se seku u jednoj tacki
P ili medu sobom uporedne, pa su prema zadatku 485, polovi tih pravih na
jednoj pravoj, polari tacke P u odnosu na krug k. Ako sa O obelezimo srediste
kruga k, bice OA a, OB b, OC c, OD d. Otuda iz harmonijske
spregnutosti pravih a, b, c, d sleduje da su prave OA, OB, OC, OD takode
harmonijski spregnute, pa su i A, B, C, D cetiri harmonijske tacke.
493. Ako su P , Q, R tacke u kojima upisani krug k dodiruje stranice BC,
CA, AB trougla ABC i S presecna tacka pravih BC i QR, dokazati da je
H(B, C; P, S).
Resenje:
Polare QR i BC tacaka A i P u odnosu na krug k seku se u tacki S, pa
je prema zadatku 482, prava AP polara tacke S u odnosu na krug k. Stoga
presecna tacka T pravih AP i QR s tackom S obrazuje par harmonijski spregnut s parom R, Q. Otuda sleduje da su prave AT i AS harmonijski spregnute
s pravama AR i AQ, pa su i tacke P i S harmonijski spregnute s tackama B i
C (sl. 310).

R
T

339

slika 310
494. Ako su Pa , Qa , Ra tacke u kojima spolja upisani krug ka dodiruje
stranicu BC i produzenja stranica CA i AB trougla ABC, a Sa presecna tacka
pravih BC i Qa Ra , dokazati da je H(B, C; Pa , Sa ).
Resenje:
Kao u prethodnom zadatku, polare Qa Ra i BC tacaka A i Pa u odnosu na
krug ka seku se u tacki Sa , pa je prava ATa , gde je Ta presecna tacka pravih APa i Qa Ra , polara tacke Sa u odnosu na krug ka . Stoga tacka Ta s
tackom Sa obrazuje par harmonijski spregnut s parom Ra , Qa . Otuda sleduje
da su prave ATa i ASa harmonijski spregnute s pravom ARa i AQa , pa su i
tacke Pa i Sa harmonijski spregnute s tackama B i C (sl. 311)
A

Pa

C
Sa
Qa

Ta
Ra

slika 311
Napomena: Duzi P S i Pa Sa u zadacima 493 i 494 kod istog trougla medu
sobom su jednake i simetricne u odnosu na srediste stranice BC.
495. Ako su P , Q, R tacke u kojima krug k upisan u trougao ABC dodiruje
stranice BC, CA, AB, dokazati da tacke X, Y , Z u kojima prave kroz srediste
S kruga k uporedne s pravama QR, RP , P Q seku prave BC, CA, AB pripadaju
jednoj pravoj.
Resenje:
Polare x, y, z tacaka X, Y , Z u odnosu na krug k sadrze tacke P , Q, R a
uporedne su s pravama AS, BS, CS. Kako su prave AS, BS, CS upravne
na stranicama QR, RP , P Q trougla P QR, njima uporedne prave x, y, z kroz
tacke P , Q, R sadrze visine trougla P QR, dakle, seku se u jednoj tacki, ortocentru trougla P QR. S obzirom da se prave x, y, z seku u jednoj tacki, polovi X,
Y , Z tih definisani u odnosu na krug k pripadaju jednoj pravoj, polari tacke u
odnosu na krug k (sl. 312).

340

A
Q

slika 312
496. (Salmonova teorema) Ako su a i b polare dveju tacaka A i B u odnosu
na krug k(O, r) a C i D podnozja upravnih iz tacaka A i B na pravama b i a,
dokazati da je
OA : AC = OB : BD.
Resenje:
S obzirom da su prave a i b polare tacaka A i B u odnosu na krug k, prave OA
i OB su upravne na pravama a i b u izvesnim tackama A i B . Pri tome,
trapezi OACB i OBDA imaju jednake odgovarajuce uglove i proporcionalne
stranice koje se susticu u tacki O, jer iz OA OA = OB OB sledi da je
OA : OB = OB : OA . Iz tih uslova sleduje da su trapezi OACB i OBDA
slicni, pa su i ostale stranice proporcionalne, naime bice OA : AC = OB : BD
(sl. 313).

341

A
D
A
O
B

C
B
b

slika 313
497. Ako su P i Q sredista dveju tetiva AB i CD nekog kruga k i ako je
prava AB simetrala ugla CP D, dokazati da je i prava CD simetrala ugla AQB.
Resenje:
Obelezimo sa O srediste kruga k, sa P i Q polove pravih AB i CD u odnosu
na krug k. Kako se polare AB i CD tacaka P i Q seku u nekoj tacki S,
prema zadatku 486, bice prava P Q polara tacke S. Stoga se tacka S koja je s
tackom S harmonijski spregnuta s tackama C i D nalazi na pravoj P Q . Simetrala P S ugla CP D upravna je na pravoj P P , pa su P C, P D, P S, P P cetiri
harmonijske prave. Stoga je tacka S i na pravoj P P . Dakle, bice tacke S i P
istovetne. Sad je tacka P na polari CD tacke Q , pa je i tacka Q na polari AB
tacke P . Otud sleduje da su tacke S i Q harmonijski spregnute s tackama A
i B, pa su i prave QS i QQ harmonijski spregnute s pravama QA i QB. No
prave QS i QQ su medu sobom upravne, pa je prava QS simetrala ugla AQB
(sl. 314).

342

Q
B
O

S
P

P = S

slika 314

6.3

Spregnute ta
cke i spregnute prave u odnosu na krug

Definicija 6.3.1. Za dve tacke P i Q kazemo da su spregnute ili konjugovane


u odnosu na neki krug ako polara svake od tih tacaka sadrzi drugu tacku.
Definicija 6.3.2. Za dve prave p i q kazemo da su spregnute ili konjugovane
u odnosu na neki krug ako svaka od tih pravih sadrzi pol druge prave.

498. Ako su A i B dve spregnute tacke u odnosu na krug k, dokazati da je


krug l, kome je duz AB precnik, ortogonalan na krugu k.
Resenje:
Kako su tacke A i B spregnute u odnosu na krug k, tacka A je na polari b tacke
B, a tacka B na polari a tacke A. Ako obelezimo sa O srediste kruga k, prava
OA upravna je na pravoj a u nekoj tacki A , pa je ugao AA B prav, i prema
tome, tacka A je na krugu l. Prava AA sadrzi srediste O kruga k, prema tome,
ona sece krug k u dijametralno suprotnim tackama C i D. Pri tom su tacke A i
A harmonijski spregnute s dijametralno suprotnim tackama C i D kruga k, pa
je prema zadatku 475, svaki krug kroz tacke A i A , dakle i krug l, ortogonalan
na krug k (sl. 315).

343

l
B

C
k

A
O

A
a

slika 315
499. Ako su k i l dva ortogonalna kruga, dokazati da je svaki par dijametralno suprotnih tacaka jednog od tih krugova spregnut u odnosu na drugi krug.
Resenje:
Neka je P Q bilo koji par dijametralno suprotnih tacaka kruga l. Prava odredena
tackom P i sredistem O kruga k sece krug k u dijametralno suprotnim tackama
A i B, a krug l u tacki P i jos nekoj tacki P , pa su prema zadatku 474, tacke
P i P su harmonijske i spregnute s tackama A i B. Stoga je tacka P na polari
tacke P u odnosu na krug k. S obzirom da je duz P Q precnik, a P tacka kruga
l, ugao P P Q je prav, pa je prava P Q upravna na pravoj OP . Otuda sleduje
da je prava P Q polara tacke P u odnosu na krug k. Kako je tacka Q na polari
tacke P , bice i tacka P na polari tacke Q, pa su tacke P i Q spregnute u odnosu
na krug k (sl. 316).

l
Q

A
k

P
O

slika 316
500. Ako su tacke C i D spregnute u odnosu na krug k i ako prava CD sece
krug k u tackama A i B, dokazati da su tacke C i D harmonijski spregnute s
tackama A i B.
Resenje:
S obzirom da su tacke C i D spregnute u odnosu na krug k, bice krug l kome
je duz CD precnik ortogonalan na krug k. Stoga su, prema zadatku 474, dijametralno suprotne tacke C i D kruga l harmonijski spregnute s tackama A i B
u kojima prava CD sece krug k (sl. 317).
344

l
k

D
B
O
C

slika 317
501. Ako su a i b dve dirke kruga k koje se seku u tacki P , a c i d dve
prave kroz P koje su spregnute u odnosu na krug k, dokazati da su prave c i d
harmonijski spregnute s pravama a i b.
Resenje:
S obzirom da su prave c i d spregnute u odnosu na krug k, pol C prave c je na
pravoj d, a pol D prave d je na pravoj c, pa je prema zadatku 486, prava CD
polara tacke P u odnosu na krug k. Otuda sleduje da prava CD sadrzi dodirne
tacke A i B tangenata (dirki) a i b. Pri tome su tacke C i D spregnute u odnosu
na krug k, pa su one harmonijski spregnute s tackama A i B u kojima prava
CD sece krug k. Dakle, prave c i d su harmonijski spregnute s pravama a i b
(sl. 318).

D
B
c
b
P

a
A

slika 318
502. Ako je jedan par suprotnih temena pravougaonika spregnut u odnosu
na neki krug, dokazati da je i drugi par suprotnih temena pravougaonika spregnut u odnosu na taj krug.
Resenje:
Ako su suprotna temena A i C pravougaonika ABCD spregnuta u odnosu na
krug k, prema zadatku 498, bice krug l kome je duz AC precnik, tj. krug koji
je opisan oko pravougaonika ABCD, ortogonalan na krugu k. Stoga je, prema
zadatku 499, par dijametralno suprotnih tacaka B, D kruga l spregnut u odnosu
na krug k (sl. 319).
345

A
O

A
a

slika 319
503. Ako je A, B par spregnutih tacaka u odnosu na krug k, nekolinearnih
sa sredistem O kruga k, dokazati da je pol P prave AB u odnosu na krug k
ortocentar trougla OAB.

slika 320
S obzirom da tacke O, A i B nisu kolinearne, polare a i b tacaka A i B u
odnosu na krug k seku se u nekoj tacki P , koja je prema zadatku 486, poluprave
AB. Po pretpostavci tacke A i B su spregnute u odnosu na krug k, pa je tacka
A na pravoj b, a tacka B na pravoj a. Otuda je AB OB i BP OA, pa je
pol P prave AB ortocentar trougla OAB (sl. 320).

346

504. Ako je A, B par spregnutih tacaka u odnosu na krug k, nekolinearnih


sa sredistem O toga kruga, dokazati da je jedan od uglova koje odreduju polazne tacke A i B u odnosu na krug k jednak uglu AOB.

B
O

P
A

slika 321
S obzirom da tacke O, A, B nisu kolinearne, polare a i b tacaka A i B u
odnosu na krug k, seku se u nekoj tacki P . Ako su A i B tacke u kojima prave
OA i OB seku prave a i b, bice uglovi OA P i OB P pravi, pa su tacke O, P , A ,
B na jednom krugu. Stoga su uglovi A OB i A P B , tj. uglovi AOB i A P B
jednaki ili suplementni, pa je jedan od uglova, koji su odredeni polarama tacaka
A i B u odnosu na krug k, jednak uglu AOB (sl. 321).

505. Ako su O i r srediste i poluprecnik kruga k, a A i B dve spregnute tacke


u odnosu na taj krug, dokazati da je
AB 2 = OA2 + OB 2 2r2 .
Ako obelezimo sa a polaru tacke A u odnosu na krug k i sa A tacku u kojoj prava OA sece pravu a, bice OA a. S obzirom da su tacke A i B spregnute
u odnosu na krug k, tacka B je na pravoj a, pa je i tacka A podnozje visine iz
temena B trougla OAB. Ugao kod temena A tog trougla je ostar , pa je
AB 2 = OA2 + OB 2 2OA OA
347

i prema tome
AB 2 = OA2 + OB 2 2r2 .

506. Ako su A i B dve spregnute tacke u odnosu na krug k, obe izvan njega,
dokazati da je zbir kvadrata tangenata iz tacaka A i B na krugu k jednak kvadratu duzi AB.
Ako sa O i r obelezimo srediste i poluprecnik kruga k, prema prethodnom
zadatku, imamo da je
AB 2 = OA2 + OB 2 2r2 .
Otuda je
AB 2 = (OB 2 r2 ) + (OB 2 r2 ),
i prema tome
AB 2 = d2A + d2B ,
gde su dA i dB odsecci tangenata iz tacaka A i B na krug k.

507. Ako je P tacka u kojoj se seku polare tacaka A i B u odnosu na krug


k sa sredistem O, dokazati da je
AP 2 BP 2 = OA2 OB 2 .

S obzirom da su tacke A i P , a isto tako tacke B i P , spregnute u odnosu


na krug k ciji je poluprecnik r, prema zadatku 505, bice
AP 2 = OA2 + OP 2 2r2
i
BP 2 = OB 2 + OP 2 2r2 .
Otuda je
AP 2 BP 2 = OA2 OB 2 .

508. Ako su k1 , k2 , k3 tri kruga cija sredista ne pripadaju jednoj pravoj, dokazati da je skup svih tacaka, cije se polare u odnosu na krugove k1 , k2 , k3 seku
u jednoj tacki, krug ortogonalan na krugovima k1 , k2 , k3 .
Ako je P tacka cije se polare u odnosu na krugove k1 , k2 , k3 seku u nekoj
tacki Q, bice tacke P i Q spregnute u odnosu na svaki od tih krugova. Stoga je
prema zadatku 498, krug k kome je duz P Q precnik, ortogonalan na krugovima
k1 , k2 , k3 . Dakle, skup svih tacaka koji ima navedenu osobinu pripada krugu
k. Sad dokazimo da takvu osobinu ima svaka tacka P kruga k. S obzirom
348

da je krug k ortogonalan na krugovima k1 , k2 , k3 , prema zadatku 499, par P ,


Q dijametralno suprotnih tacaka kruga k, spregnut je u odnosu na svaki od
krugova k1 , k2 , k3 ; prema tome polare tacke P u odnosu na krugove k1 , k2 , k3
seku se u jednoj tacki Q .

6.4

Polarni i autopolarni trouglovi. Polarni krug trougla

Definicija 6.4. Neka su u ravni k data dva trougla ABC i A B C . Ako su


prave BC, CA, AB polare tacaka A , B , C u odnosu na krug k, a prave B C ,
C A , A B polare tacaka A, B, C u odnosu na isti krug k, tada kazemo da
su trouglovi ABC i A B C spregnuti, ili konjugovani, ili polarni u odnosu na
krug k, a za krug k kazemo da je polaran u odnosu na spregnute trouglove ABC
i A B C . Ako se trouglovi ABC i A B C poklapaju, tj. ako su prave BC,
CA, AB polare tacaka A, B, C u odnosu na krug k, tada kazemo da je trougao
ABC spregnut sa samim sobom ili da je autopolaran u odnosu na krug k. U
tom slucaju krug k nazivamo polarnim krugom trougla ABC.
509. Ako su ABC i A B C dva trougla u ravni kruga k takva da su prave
BC, CA, AB polare tacaka A , B , C u odnosu na krug k, dokazati da su i
prave B C , C A , A B polare tacaka A, B, C u odnosu na krug k, tj. da su
trouglovi ABC i A B C polarni u odnosu na krug k.

A
k
C

O
B

slika 322
S obzirom da su prave AB i AC polare tacaka C i B u odnosu na krug
k, prema zadatku 486, bice i prava B C polara tacke A u odnosu na krug k.
349

Isto tako je prava C A polara tacke B, a prava AB polara tacke C. Stoga su


trouglovi ABC i A B C polarni u odnosu na krug k ( sl. 322).

510. Ako su prave odredene stranicama AB i AC trougla ABC polare tacaka


C i B u odnosu na neki krug k, dokazati da je i prava BC polara tacke A, tj.
da je trougao ABC ortogonalan u odnosu na krug k.

slika 323
S obzirom da polara tacke B sadrzi tacku A, i polara tacke A sadrzi tacku
B (v. sl. 482). Isto tako, polara tacke A sadrzi tacku C. Otud sleduje da je
prava BC polara tacke A u odnosu na krug k, pa je trougao ABC autopolaran
u odnosu na krug k (sl. 323).

511. Dokazati da se ortocentar svakog autopolarnog trougla u odnosu na neki


krug poklapa sa sredistem tog kruga.

350

slika 324
Ako je ABC autopolarni trougao u odnosu na krug k, saglasno definiciji,
bice prave BC, CA, AB polare tacaka A, B, C u odnosu na krug k. Ako sa O
obelezimo srediste kruga k, bice OABC, OBCA, OCAB, pa je srediste O
kruga k ortocentar trougla ABC (sl. 324).
512. Dokazati da je svaki autopolaran trougao u odnosu na neki krug tupougli i da svaki tupougli trougao ima svoj polarni krug.
Resenje: Ako je trougao ABC autopolaran u odnosu na krug k, prema
zadatku 511, srediste O kruga k je ortocentar trougla ABC , tj. tacka u kojoj
se seku prave odredene njegovim visinama AA , BB , CC . Pri tom su tacke A
i A , zatim B i B , zatim C i C s iste strane od tacke O ,pa je tacka O izvan
trougla ABC, i prema tome, trougao ABC je tupougli. Da bismo dokazali drugi
deo teoreme, pretpostavimo da je ABC bilo koji tupougli trougao. S obzirom da
je trougao tupougli, tacka O u kojoj se seku prave odredene njegovim visinama
AA , BB , CC nalazi se izvan tog trougla, pa su tacke A i A , zatim B i B ,
zatim C i C , s iste strane od tacke O. Sem toga je,
OA OA = OB OB = OC OC = r2
pa su prave BC, CA, AB polare tacaka A, B, C u odnosu na krug k kome je
srediste O i poluprecnik jednak duzi r (sl.325).

351

C
A

A
B

Slika 325
513. Dokazati da su krugovi, ciji su precnici stranice trougla autopolarnog
u odnosu na krug k, ortogonalni na krugu k.
Resenje: Ako je ABC autopolarni trougao u odnosu na krug k, svaka dva
njegova temena su spregnuta u odnosu na krug k, pa su prema zadatku 498,
krugovi ciji su precnici stranice trougla ABC ortogonalni na krugu k.
514. Dokazati da je dijagonalni trougao svakog cetvorotemenika upisanog u
neki krug autopolaran u odnosu na taj krug.
Resenje: Obelezimo sa ABCD cetvorotemenik upisan u krug k, a sa P, Q, R
tacke u kojima se seku njegove naspramne stranice AB i CD,BC i AD, AC i
BD. Saglasno definiciji, tacke P, Q, R su temena dijagonalnog trougla cetvorotemenika
ABCD. Ako sa S i T obelezimo tacke u kojima prave AC i BD seku pravu
P Q, bice tacke R i S harmonijski spregnute s tackama A i C, a tacke R i T
harmonijski spregnute s tackama B i D, pa je prava ST , tj. prava P Q, polara
tacke R u odnosu na krug k. Analogno se dokazuje da su i prave QR i RP polare
tacke P i Q u odnosu na krug k pa je trougao P QR autopolaran u odnosu na
krug k(sl.326).

352

S
C
R
B

A
T

Slika 326
515. Dokazati da je dijagonalni trougao svakog cetvorostranika opisaog oko
nekog kruga autopolaran u odnosu na taj krug.
Resenje: Obelezimo sa abcd cetvorostranik opisan oko nekog kruga k, a sa
A, B, C,D, E, F njegova temena, tj. tacke u kojima se seku stranice d i a, a i
b, b i c, c i d, a i c, b i d, sa X, Y, Z tacke u kojima se seku dijagonalne prave
AC i BD, BD i EF , EF i AC; a sa P, Q, R, S tacke u kojima stranice a, b, c, d
dodiruju krug k. Pri tome su prave P Q i RS polare tacaka B i D u odnosu

ns krug k, pa je prema zadatku 486, presecna tacka Y pravih P Q i RS pol


prave BD. Ako sa M obelezimo tacku u kojoj prava P Q sece pravu BD, bice

tacke M i Y harmonijski spregnute s tackama P i Q, pa su i prave BM i BY


harmonijski spregnute s pravama BP i BQ. S obzirom da prave BM ,BQ,BP

seku pravu AC u tackama A, C, X prava BY mora seci pravu AC u tacki Y,


koja sa tackom X obrazuje harmonijski par spregnut sa parom A, C. Isto tako

prava DY sece pravu AC u tacki Y . Stoga su tacke Y i Y istovetne, pa je


prava BD, tj. prava XZ polara tacke Y u odnosu na krug k. Istim postupkom
dokazuje se da su prave XY i Y Z polare tacaka Z i X u odnosu na krug k, pa
je trougao XY Z autopolaran u odnosu na krug k(sl.327).

353

Y Y
R
C

S
X

A
B

Slika 327
516. Ako je P QR dijagonalni trougao cetvorotemenika ABCD upisanog u
krug k sa sredistem O, dokazati da je OP OR, OQ P R, OR P Q.
Resenje: Prema zadatku 514, dijagonalni trougao P QR cetvorotemenika
A, B, C, D upisanog u krug k je autopolaran u odnosu na taj krug, dakle prave
QR, RP , P Q su polare tacaka P, Q, R u odnosu na krug k, te je OP QR,
OQ RP , OR P Q.
517. Ako je XY Z dijagonalni trougao cetvorotemenika abcd opisanog oko
kruga k sa sredistem O, dokazati da je OX Y Z, OY ZX, OZ XY .
Resenje: Prema zadatku 515, dijagonalni trougao XY Z cetvorostranika abcd
opisanog oko kruga k je autopolaran u odnosu na taj krug , tj. prave Y Z, ZX,
XY su polare tacaka X, Y, Z u odnosu na krug k, pa je OX Y Z, OY ZX,
OZ XY .
518. Dokazati da su krugovi, ciji su dijametri stranice dijagonalnog trougla
cetvorotemenika upisanog u neki krug k, ortogonalni na krug k.
Resenje: Prema zadatku 514, dijagonalni trougao P QR cetvorotemenika
ABCD upisanog u krug k je autopolaran u odnosu na krug k. Stoga su, svaka
dva temena trougla P QR spregnuta u odnosu na krug k, pa su prema zadatku
498, krugovi s dijametrima P Q, QR, RS ortogonalni na krug k.
519. Dokazati da su krugovi, ciji su dijametri stranice dijagonalnog trougla
cetvorotemenika opisanog oko nekog kruga k, ortogonalni na krug k.
Resenje: Prema zadatku 515, dijagonalni trougao XY Z cetvorostranika abcd
opisanog oko kruga k je autopolaran u odnosu na krug k. Stoga su, svaka dva
temena trougla XY Z spregnuta u odnosu na krug k, pa su prema zadatku 498,
krugovi s precnicima xy, yz, zx ortogonalni na krugu k.

354

520. Ako je M promenljiva tacka kruga k opisanog oko nekog trougla ABC
i ako su P ,Q,R tacke u kojima prave AM ,BM ,CM seku pravu BC,CA,AC
dokazati da prave odredene odgovarajucim stranicama svih trouglova P QM
obrazuju pramenove pravih.
Uputstvo: Tacke P, Q, R su temena dijagonalnog trougla cetvorotemenika
ABCM upisanog u krug k, pa je prema zadatku 514, trougao P QR autopolaran
u odnosu na krug k. Stoga su prave QR, RP, P Q polare tacaka P, Q, R u odnosu
na krug k. Kako je pol P prave QR na pravoj BC, bice i pol prave BC na pravoj
QR. Dakle, sve prave odredjene stranicama, pomenutih trouglova sadrze pol
prave BC i prema tome one obrazuju pramen pravih. ANalogno se dokazuje
da i prave odredjene ostalim odgovarajucim stranicama trouglova P QR cine
pramenove pravih (sl. 328).

A
M
Q

Sl.328
521. Ako polarni krug trougla sece pravu odredenu sa dva njegova temena,
dokazati da su presecne tacke harmonijski spregnute sa tim temenima.
Uputstvo: Neka polarni krug k trougla ABC sece pravu AB u tackama P
i Q. Pri tome su tacke A i B spregnute u odnosu na krug k, pa su prema
zadatku 500, presecne tacke P i Q prave AB s krugom k harmonijski spregnute
s tackama A i B (sl. 329).

355

H
B

P
B

Sl.329

6.5

Sli
cnost krugova

Definicija 6.5. Za dve tacke P i P krugova k i k koje su kolinearne sa


spoljasnjim ili unutrasnjim sredistem slicnosti tih dvaju krugova, kazemo da su
homologne ili antihomologne u odnosu na to srediste slicnosti zavisno od toga
da li su poluprecnici koji odgovaraju tim tackama istosmerni ili ne.
Definicija 6.6. Za dve tetive P Q i P Q krugova k i k kazemo da su homologne ili antihomolgone u odnosu na srediste S slicnosti tih dvaju krugova,
zavisno od toga da li su take P i Q homologne ili antihomologne sa tackama
P i Q u odnosu na isto srediste slicnosti S tih krugova.
522. Dve prave kroz srediste S slicnosti krugova k i k seku te krugove; jedna
sece krug k u tackama A i B, a krug k u tackama A i B , druga sece krug k u
tackama C i D, a krug k u tackama C i D . Ako su pri tome A i A , B i B ,
D i D homologne tacke krugova k i k , dokazati da je
AA BB = CC DD
.
Uputstvo: Homologne tetive AC i A C , a isto tako tetive BD i B D krugova
k i k medju sobom su uporedne, pa je SA : SC = AA : CC i SB : SD =
SA SB
AA BB
BB : DD . Otuda je SC
SD = CC
DD .
AA BB
SA SB
Medjutim, SC SD = 1, pa je CC DD , i prema tome AA BB = CC DD .
Specijalno, ako krugovi k i k imaju zajednicku tangentu koja sadrzi tacku S i
ako su T i T njene dodirne tacke sa krugovima k i k , bice AA BB = T T 2
(sl. 330).

356

B
A

B
S

O
C

Sl.330
523. Ako su A,B i C,D dva para antihomolognih tacaka dvaju krugova k
i k definisani u odnosu na isto sredi?ste S slicnosti tih krugova, dokazati da je

SA SB = SC SD
.
Uputstvo: Prava SA, sece krugove k i k u antihomolognim tackama A i
B , prema tome, ona sece te krugove jos u dvema antihomolognim tackama B
i A . Isto tako, prava SC sece krugove k i k u antihomolognim tackama C i
D , prema tome, ona sece te krugove jos u dvema antihomolognim tackama D i
C (sl. 330). Pri tome su duzi AC i A C , a isto takoBC i B C uporedne, pa je
SA : SA = SC : SC i SB : SB = SC : SC . Otuda je SA : SA = SB : SB ,
i prema tome SA SB = SB SA ...(1). Isto tako je SC SD = SD SC ...(2).
Kako je SA SB = SC SD i SA SB = SC SD bice SA SB SA SB =
SC SDSC SD ...(3). Iz jednakosti (1), (2), (3) sledi da je SASB = SC SD .
524.Ako su A,B i C,D dva para antihomolognih tacaka krugova k i k
odredenih u odnosu na isto srediste S slicnosti tih krugova, dokazati da tacke
A, B , C, D pripadaju jednom krugu ili jednoj pravoj.
Upustvo: Pretpostavimo da tacke A, B , C, D nisu na jednoj pravoj. S
obzirom da su A, B i C, D parovi antihomolognih tacaka krugova k i k definisani u odnosu na isto srediste S slicnosti tih dvaju krugova, prema zadatku 523,
imamo da je SA SB = SC SD . Otuda sledi da tacke A, B , C, D pripadaju
jednom krugu l (sl. 331).

357

A
B
S

k
k

Sl.331
525.Ako neki krug l dodiruje dva kruga k i k u tackama A i B, dokazati da
su A i B antihomologne tacke krugova k i k .
Uputstvo: S obzirom da je tacka A srediste slicnosti krugova k i l, a tacka
B srediste slicnosti krugova k i l, prema poznatom stavu, bar jedno srediste
slicnosti, npr S, krugova k i k pripada pravoj AB. Kako su tacke A i B kolinearne sa sredistem S slicnosti krugova k i k , a prave odredjene poluprecnicima
OA i O B tih krugova seku, A i B su antihomologne tacke krugova k i k ,
definisane u odnosu na srediste S slicnosti tih dvaju krugova (sl. 332).

358

O
O
B
A

Sl.332
526. Dokazati da je skup svih tacaka kojima su rastojanja od sredista O1 i
O2 krugova k1 i k2 srazmerna poluprecnicima r1 i r2 tih krugova takode krug
kome je precnik duz odredena sredistima S1 i S2 slcnosti tih krugova.
Uputstvo: S obzirom da su S1 i S2 tacke prave O1 O2 takve da je O1 S1 :
O2 S1 = r1 : r2 i O1 S2 : O2 S2 = r1 : r2 , prema poznatom stavu, bice skup svih
tacaka P takvih da je O1 P : O2 P = r1 : r2 krug kome je duz S1 S2 precnik.
Specijalno, ako su poluprecnici r1 i r2 krugova k1 i k2 jednaki, skup svih tacaka
P je simetrala duzi O1 O2 .
Napomena: Krug kome je duz S1 S2 precnik nazivamo krugom slicnosti krugova k1 i k2 .
527.Ako je l krug slicnosti dvaju krugova k1 i k2 , dokazati da je skup svih
tacaka iz kojih parovi dirki na krugovima k1 i k2 zahvataju jednake uglove, onaj
deo kruga l koji se nalazi izvan krugova k1 i k2 .
Uputstvo: Obelezimo sa P bilo koju od tacaka iz kojih parovi dirki na krugovima k1 i k2 zahvataju jednake uglove,sa A1 i B1 dodirne tacke dirke kroz
tacku P na krugu k1 , a sa A2 i B2 dodirne tacke dirke kroz tacku P na krug
k2 . Prave P O1 i P O2 odredene tackom P i sredistima O1 i O2 krugova k1 i
k2 su simetrale jednakih uglova A1 P B1 i A2 P B2 , pa su kod pravouglih trouglova P A1 O1 i P A2 O2 jednaki i ostri uglovi A1 P O1 i A2 P O2 . Otuda sledi da
su ti trouglovi slicni, pa su duzi O1 P i O2 P srazmerne poluprecnicima O1 A1 i
O2 A2 krugova k1 i k2 . Obrnuto, neka je P proizvoljna tacka kruga l koja se
nalazi izvan kruga k1 , dakle i izvan kruga k2 . Obelezimo sa A1 i B1 dodirne
tacke dirki kroz P na k1 , a sa A2 i B2 dodirne tacke dirki kroz P na k2 . Kod
trouglova A1 O1P i A2 O2 P jednaki su pravi uglovi A1 i A2 , a stranice O1 P i
359

O2 P srazmerne stranicama O1 A1 i O2 A2 pa je A1 O1 P A2 O2 P . Otuda


je A1 P O1 = A2 P O2 , i prema tome A1 P B1 = A2 P B2 (sl.333).

A1
A2
l
O1

O2

B2

k1
B1

k2

Sl. 333.
528. Ako su k1 , k2 , k3 tri kruga s nejednakim poluprecnicima i kolinearnim
sredistima, dokazati da se od sest sredista slicnosti tih krugova uzetih u parovima
po tri nalaze na cetiri prave.
Uputstvo: Neka su O1 , O2 , O3 i r1 , r2 , r3 poluprecnici krugova k1 , k2 , k3 a
S1 , S2 , S3 spoljanja i S1 , S2 , S3 unutrsnja sredista slicnosti krugova k2 i k3 , k3
i k1 ,k1 i k2 . Pri tome je O1 S3 : O2 : S3 = r1 : r2 , O2 S1 : O3 S1 = r2 : r3 ,
O3 S2 : O1 S2 = r3 : r1 Otuda je
O2 S1 O3 S2 O1 S3

= 1
S1 O3 S2 O1 S3 O2
pa su prema Menelajevoj teoremi tacke S1 , S2 , S3 na jednoj pravoj. Analogno
se izvodi dokaz iza trojke tacaka S, S2 , S3 ; S2 , S3 , S1 ; S3 , S1 , S2 ..
529. Dokazati da su ortocentar H i teziste T trougla ABC sredista slicnosti
opisanog kruga l i Ojlerovog kruga l trougla ABC.
Uputstvo: S obzirom da krug l sadrzi temena A, B, C trougla ABC, a krug
l sredista duzi HA, HB, HC tacka H je spoljasnje srediste slicnosti krugova l i
l isto tako, krug l sadrzi temena A, B, C a krug l sredista stranica BC, CA, AB
trougla ABC, pa je tacka T unutrasnje srediste slicnosti krugova l i l (sl.334).

360

l
O
H

Sl. 334.
530. Ako su S, Sa , Sb , Sc sredista upisanih krugova trougla ABC, dokazati
da je tacka S spoljasnje srediste slicnosti krugova l i l opisanih oko trouglova
Sa Sb Sc i ABC.
Uputstvo:Tacke A, B, C su sredista stranica Sb Sc , Sc Sa , Sa Sb trougla Sa Sb Sc ,
pa je opisani krug l trougla ABC Ojlerov trougla Sa Sb Sc . Sem toga tacka S je
ortocentar trougla Sa Sb Sc , pa e prema prethodnom zadatku, tacka S spoljasnje
srediste slicnosti krugova l i l .
531. Ako je k upisani krug trougla ABC i k krug kome je visina AA
precnik, dokazati da je tacka u kojoj spolja upisani krug ka dodiruje stranicu
BC spoljasnje srediste slicnosti krugova k i k .
Uputstvo: Obelezimo sa S i Sa sredista krugova k i ka , sa P i Pa tacke u
kojima ti krugovi dodiruju stranicu BC, sa Q tacku u kojoj prava P S sece pravu
APa i sa D tacku u kojoj simetrala AS ugla A sece stranicu BC. S obzirom da
je ASQ ASa Pa i DSP DSa Pa , imamo da je Sa A : SA = Sa PA : SP
i Sa D : SD = Sa Pa : SP . No tacke D i A su harmonijski spregnute s tackama
S i Sa , pa je As A : SA = Sa D : SD. Stoga je Sa Pa : SQ = Sa Pa : SP , i prema
tome je SP = SQ. Dakle duz P Q je precnik kruga k. Pri tome su P, A i Q, A
dva para homogenih tacaka krugova k i k pa je presecna tacka Pa pravih P A
i QA koja se nlazi iza P u odnosu na A ,spoljasnje srediste slicnosti krugova k
i k (sl.335).

361

A
Q
S
B

Pa

Sa

Qa

Sl. 335.
532. Ako je ka spolja upisani krug trougla ABC, i k krug kome je visina
AA precnik, dokazati da je tacka u kojoj upisani krug k trougla ABC dodiruje
stranicu BC unutrasnje srediste slicnosti krugova ka i k .
Uputstvo: Obelezimo sa S i Sa sredista krugova k i ka , sa P i Pa tacke u
kojima ti krugovi dodiruju stranicu BC , sa Qa tacku u kojoj prava Pa Sa sece
pravu AP i sa D tacku u kojoj simetrala ugla A sece stranicu BC. S obzirom
da je ASa Qa ASP i DSa Pa DSP , bice Sa A : SA = Sa Qa : SP
i Sa D : SD = Sa Pa : SP . No Sa A : SA = Sa D : SD, pa je Sa Qa : SP =
Sa Pa : SP , i prema tome Sa Pa = Sa Qa . Stoga je duz Pa Qa precnik kruga
ka . Pri tome su Pa , A i Qa , A parovi homolognih tacaka krugova ka i K , pa
je presecna tacka P pravih Pa A i Qa A koja se nalazi izmedu tacaka Pa i A
unutrasnje srediste slicnosti krugova ka i k .
533. Dokazati da je svaki krug koji sadrzi sredista O1 i O2 dvaju krugova k1
i k2 ortogonalan na krugu slicnosti krugova k1 i k2 .
Uputstvo: Sredista S1 i S2 slicnosti krugova k1 i k2 su harmonijski spregnuta
sa sredistima O1 i O2 tih krugova, pa je prema zadatku 475, krug l kome je duz
S1 S2 precnik ortogonalan na svakom krugu koji sadrzi tacke O1 i O2 .

534. Dokazati da je bilo koje srediste slicnosti dvaju krugova k1 i k2 jednako


udaljeno od dirki na tim krugovima koje su konstruisane na njihovim presecnim
tackama.
Uputstvo: Obelezimo sa S1 i S2 sredista slicnosti krugova k1 i k2 , sa O1 i
O2 sredista tih krugova i sa P bilo koju od presecnih tacaka tih krugova. Pri

362

tome je
O1 S1 : O2 S1 = O1 P : O2 P
i
O1 S2 : O2 S2 = O1 P : O2 P,
pa su prave P S1 i P S2 simetrale uglova odedenih pravama O1 P i O2 P . Stoga
su prave P S1 i P S2 simetrale uglova koje odreduju normale u tacki P na pravama P O1 i P O2 , tj. uglovi koji su odredeni tangentama t1 i t2 krugova k1 i
k2 u njihovoj presecnoj tacki P . Otuda sledi da je bilo koje srediste slicnosti
krugova k1 i k2 jednako udaljeno od tangenti t1 i t2 krugova k1 i k2 .
535. Ako su O1 , O2 , O3 sredista triju krugova l1 , l2 , l3 od kojih svaki dodiruje
ostala dva, P1 , P2 , P3 dodirne tacke krugova l1 i l2 , l3 i l1 , l1 i l2 , a X i Y tacke
u kojima prave P1 P2 i P1 P3 seku krug l1 , dokazati da tacke O1 , X, Y pripadaju
jednoj pravoj koja je uporedna sa pravom O2 O3 .
Uputstvo: Dodirna tacka P3 krugova l1 i l2 je srediste slicnosti tih krugova,
pa su P1 i Y njihove homologne tacke, i prema tome prave O2 P1 i O1 Y su medu
sobom uporedne. Isto tako prave O3 P1 i O1 X su medu sobom uporedne. S
obzirom da je tacka P1 na pravoj O2 O3 , a prave O1 X i O1 Y uporedne sa O3 P1
i O2 P1 , tacke O1 , X, Y su na jednoj pravoj koja je uporedna sa pravom O2 O3
(sl. 336).

l1

O1
Y

P3

P2

O2

P1

l2

O3

l3

Slika 336

363

6.6

Potencija ta
cke u odnosu na krug

Definicija 6.7. Neka je M proizvoljna tacka ravni nekog kruga k. Ako su


P i Q promenljive tacke kruga k kolinearne sa tackom M , tada je proizvod
duzi M P i M Q konstantan. Tu konstantu nazivamo stepenom ili potencijom
tacke M u odnosu na krug k. Potencija tacke M u odnosu na krug k ima pozitivan znak ako je tacka M izvan kruga k, a negativan ako je tacka M u krugu k.
536. Ako je M tacka ravni kruga k, a A i B par tacaka kruga k kolinearnih
sa M , dokazati da je proizvod duzi M A i M B konstantan.
Uputstvo: Neka je A, B jedan, a C, D drugi par tacaka kruga k kolinearnih
sa M . Iz slicnosti trouglova M AC i M DB sledi da je M A M C = M D M B,
pa je M A M B = M C M D (sl. 337).
D

M
B

Slika 337
537. Ako je M tacka izvan kruga k, a T dodirna tacka jedne od dirki kruga
k kroz tacku M , dokazati da je potencija tacke M u odnosu na krug k jednaka
kvadratu duzi M T .
Uputstvo: Ako su A i B tacke u kojima neka prava kroz M sece krug k, bice
M AT M T B, pa je
M A M T = M DT M B,
pa je
MA MB = MT 2
(sl. 338).

364

Slika 338
538. Ako je O srediste i r poluprecnik kruga k, a d duz koja spaja neku
tacku M sa tackom O i p(M ) potencija tacke M u odnosu na krug k, dokazati
da je p(M ) = (d2 r2 ), prema tome da li je tacka M izvan ili u krugu k.
Uputstvo: Neka su A i B tacke u kojima prava OM sece krug k. U slucaju
da je tacka M izvan kruga k, imamo da je
M A M B = (d + r) (d r) = d2 r2 ,
a u slucaju da je tacka M u krugu k, imamo da je
M A M B = (r + d) (r d) = r2 d2 .

539. Dokazati da je potencija ortocentra H trougla ABC u odnosu na


krug l opisan oko tog trougla jednak cetvorostrukoj potenciji iste tacke H u
odnosu na Ojlerov krug k tog trougla.
Uputstvo: Prava odredena visinom AA sece krug k u tacki A i jos nekoj
tacki A , a krug l u tacki A i jos nekoj tacki AH . Pri tome su tacke A i A
sredista duzi HAH i HA, pa je
HA HAH = 2 HA 2 HA = 4 HA HA .
Dakle, potencija tacke H u odnosu na krug l jednaka je cetvorostrukoj potenciji
tacke H u odnosu na krug k (sl 339).

365

B
A

C
H
C

Slika 339
540. Dokazati da je zbir potencija temena trougla ABC u odnosu na Ojlerov
krug k tog trougla jednak cetvrtini zbira kvadrata stranica tog trougla.
Uputstvo: Podnozja A , B , C visina iz temena A, B, C i sredista A , B , C
duzi koje spajaju ortocentar H sa temenima A, B, C trougla ABC su na Ojlerovom krugu tog trougla, pa iz proizvoda AA AA , BB BB , CC CC
potencije temena A, B, C u odnosu na Ojlerov krug trougla ABC (sl. 340). Sad
se primenom zadatka 246 dobija da je
AA AAH + BB BBH + CC CCH
=

1
(AH AA + BH BB + CH CC )
2
1
= (AB 2 + BC 2 + CA2 ).
4

366

B
A

C
H
C
B

slika 340
541. Ako su A1 , B1 , C1 tacke simetricne sa ortocentrom H u odnosu na
temena A, B, C trougla ABC, dokazati da je zbir potencija tacaka A1 , B1 , C1
u odnosu na krug l trougla ABC jednak zbiru kvadrata stranica tog trougla.
Uputstvo: S obzirom da je A1 H AH, AH BC, BC k B1 C1 , bice
A1 H B1 C1 . Isto tako je B1 H C1 A1 i C1 H A1 B1 , pa je H ortocentar A1 B1 C1 . Tacke A, B, C su sredista duzi HA1 , HB1 , HC1 , koje spajaju
ortocentar H s temenima trougla A1 B1 C1 , pa je krug l, koji sadrzi tacke A,
B, C, Ojlerov krug trougla A1 B1 C1 . Prema prethodnom zadatku imamo da
je zbir potencija tacaka A1 , B1 , C1 u odnosu na krug l jednak cetvrtini zbira
kvadrata stranica trougla ABC. S obzirom da je B1 C1 = 2BC, C1 A1 = 2CA,
A1 B1 = 2AB, bice zbir potencija tacaka A1 , B1 , C1 u odnosu na krug l jednak
je zbiru kvadrata stranica trougla ABC (sl. 341).

367

A1

B1

C1

Slika 341
542. Ako su a, b, c stranice trougla ABC, r poluprecnik opisanog kruga i p
potencija ortocentra H u odnosu na opisani krug, dokazati da je
p = a2 + b2 + c2 8r2 .

Uputstvo: Ako obelezimo sa A , B , C podnozja visina iz temena A, B, C


trougla ABC, prema zadacima 246 i 308, imamo da je
1 2

(a + b2 + c2 ) = AH AA + BH BB + CH CC =
2

=AH 2 + BH 2 + CH 2 + AH HA + BH HB + CH HC =
3
= 12r2 (a2 + b2 + c2 ) + p.
2
Otuda je p = a2 + b2 + c2 8r2 .
543. Ako je T teziste trougla ABC, dokazati da su potencije tacaka A, B,
C u odnosu na krugove T BC, T CA, T AB medju sobom jednake.

Uputstvo: Ako obelezimo sa D srediste stranice BC, a sa B i C druge


presecne tacke krugova T AB i T AC sa pravom BC, imamo da je DA DT =

DB DB i DA DT = DC DC , pa je DB DB = DC DC , i prema tome

je CB CB = BC BC . Stoga su potencije tacaka B i C u odnosu na krugove


T AC i T AB jednake. Na isti nacin dokazuje se da su i potencije tacaka A i B
u odnosu na krugove T BC i T AC jednake (sl. 342).

368

B B

C C

Slika 342
544. Dokazati da je kvadrat poluprecnika polarnog kruga trougla jednak
polovini potencije ortocentra u odnosu na opisani krug tog trougla.
Uputstvo: Da bi trougao imao polarni krug, on mora da bude tupougli (v.z.
512). Srediste tog kruga poklapa se sa ortocentrom H tog trougla (v.z. 511).
Ako obelezimo sa r poluprecnik tog polarnog kruga i sa D podnozje visine iz
temena A, imamo da je
r2 = HA HD.
Neka je O srediste opisanog kruga trougla ABC, M srediste stranice BC, N
podnozje upravne iz tacke O na pravoj AD i E tacka u kojoj ta prava sece
opisani krug. Pri tome je tacka N srediste duzi AE, a duz M O, dakle i duz
DN istosmerna sa duzi HA i jednaka njenoj polovini, pa je tacka D srediste
duzi HE, i prema tome
1
r2 = HA HE.
2
(sl. 343).

369

E
N

D
A
H

Slika 343
545. Dokazati da je potencija sredista bilo kojeg upisanog kruga u odnosu
na opisani krug trougla jednak dvostrukom proizvodu poluprecnika tih dvaju
krugova.
Uputstvo: Ako obelezimo sa r poluprecnik opisanog kruga, sa poluprecnik
upisanog kruga, sa S srediste tog upisanog kruga i sa N tacku u kojoj simetrala
AS ugla A sece opisani krug trougla ABC, prema zadatku 250, imamo da je
SA SH = 2r. Analogno se dokazuju i ostali slucajevi.
546. Dokazati da je zbir potencija odredista svih upisanih krugova u odnosu
na opisani krug jednak 8r2 , gde je r poluprecnik opisanog kruga.
Uputstvo: Ako obelezimo sa , a , b , c poluprecnike upisanih krugova
trougla ABC i sa s zbir potencija sredista svih tih upisanih krugova u odnosu
na opisani krug, prema prethodnom zadatku, bice s = 2r(a + b + c ) =
2r 4r = 8r2 .
547. Ako su a, b, c stranice i p poluobim trougla ABC, a d2 i d2a potencija
sredista S i Sa upisanih krugova u odnosu na opisani krug tog trougla, dokazati
da je
abc
abc
, d2a =
.
d2 =
2p
2(p a)

Uputstvo: Ako obelezimo sa r poluprecnik opisanog kruga, sa poluprecnik


upisanog kruga i sa a poluprecnik spolja upisanog kruga koji dodiruje stranicu
BC, prema zadacima 250 i 264 imamo da je
abc
d2 = SA SN = 2r =
,
2p
abc
d2a = ASa N Sa = 2ra =
.
2(p a)
548. Ako je X jedna od tacaka u kojima spolja upisani krug sa sredistem
Sa sece opisani krug l trougla ABC, a Y tacka u kojoj prava Sa X sece krug l,
dokazati da je duz Sa Y jednaka precniku kruga l.

370

Uputstvo: Prema zadatku 250 imamo da je Sa A Sa N = Sa X Sa Y = 2ra .


Kako je Sa X = a , bice Sa Y = 2r.
549. Ako su O i r srediste i poluprecnik kruga k, a A i B tacke kolinearne s
tackom O, takve da je OA OB = r2 , dokazati da je zbir potencija tacaka A i
B u odnosu na krug k jednak kvadratu duzi AB.
Uputstvo: Jedna od tacaka A i B, npr. A se nalazi izvan kruga k, a druga
je u tom krugu. Ako obelezimo sa P dodirnu tacku jedne od tangenata kruga k
kroz tacku A, potencija tacke A u odnosu na krug k jednaka je kvadratu duzi
AP . Ako su C i D tacke u kojima AB sece krug k, duz P B je visina koja
odgovara hipotenuzi CD pravouglog trougla P CD, pa je proizvod duzi BC i
BD, dakle potencija tacke B u odnosu na krug k, jednak kvadratu duzi P B s
negativnim znakom. Ugao B trougla AP B je prav, pa je AP 2 BP 2 = AB 2 .
Stoga je zbir potencija tacaka A i B u odnosu na krug k jednak kvadratu duzi
AB (sl. 344).

P
k

Slika 344

6.7

Radikalna osa dvaju krugova. Radikalno sredi


ste triju
krugova

Definicija. 6. 8. Skup svih tacaka kojima su potencije u odnosu na dva data


kruga medu sobom jednake je prava koju nazivamo radikalnom osom tih dvaju
krugova.
550. Dokazati da je skup svih tacaka kojima su potencije u odnosu na dva
data kruga medu sobom jednake prava upravna na pravoj koja spaja sredista
tih dvaju krugova.
Ako su O1 i O2 sredista, a r1 i r2 poluprecnici dvaju krugova k1 i k2 , zatim
P tacka cije su potencije u odnosu na krugove k1 i k2 medu sobom jednake, bice
O1 P 2 r12 = O2 P 2 r22
371

tj.
O1 P 2 O2 P 2 = r12 r22 .
Iz ove jednakosti sleduje da je razlika kvadrata duzi O1 P i O2 P stalna, pa su sve
tacke s jednakim potencijama u odnosu na krugove k1 i k2 na izvesnoj pravoj
p koja je upravna na pravoj O1 O2 . Obratno, ako je P bilo koja tacka prave p,
bice
O1 P 2 O2 P 2 = r12 r22
tj.
O1 P 2 r12 = O2 P 2 r22 ,
pa su potencije u odnosu na krugove k1 i k2 medu sobom jednake. Stoga je
skup svih tacaka cije su potencije i odnosu na krugove k1 i k2 , prava p koja je
upravna na pravoj O1 O2 (sl. 345).
P

O1

O2

k2

k1

slika 345
551. Dokazati da se radikalne ose triju krugova seku u jednoj tacki, ili su
medu sobom uporedne, ili su pak istovetne.
Neka su O1 , O2 , O3 sredista triju krugova k1 , k2 , k3 , a s1 , s2 , s3 radikalne
ose krugova k2 i k3 , k3 i k1 , k1 i k2 . Ako pretpostavimo da tacke O1 , O2 , O3 nisu
kolinearne, prave O2 O3 i O3 O1 se seku, pa se i prave s1 i s2 upravne na njima
seku u nekoj tacki S. Pri tome su potencije tacke S u odnosu na krugove k2 i k3
jednake; isto tako jednake su potencije tacke S u odnosu na krugove k3 i k1 , pa
su jednake i potencije tacke S u odnosu na krugove k1 i k2 . Otuda sleduje da je
tacka S na pravoj s3 , pa se radikalne ose bilo kojih triju krugova s nekolinearnim
sredistima seku u jednoj tacki S, radikalnom sredistu tih triju krugova. Ako su
sredista O1 , O2 , O3 krugova k1 , k2 , k3 na jednoj pravoj, radikalne ose s1 , s2 ,
s3 tih krugova upravne su na toj pravoj u trima raznim tackama, ili pak u istoj
tacki, prema tome, one su medu sobom uporedne ili su pak istovetne (sl. 346).

372

A
Q
R
C

B
Q

slika 346
552. Dokazati da skup sredista svih krugova koji su ortogonalni na datim
krugovima k1 i k2 predstavljaju tacke radikalne ose tih krugova koje se nalaze
izvan krugova k1 i k2 .
Ako obelezimo sa O i r srediste i poluprecnik proizvoljnog kruga k koji je
ortogonalan na krugovima k1 i k2, bice tacka O na tangentama krugova k1 i k2
u njihovim presecnim tackama sa krugom k, te se tacka O nalazi izvan krugova
k1 i k2. Sem toga, bice potencija tacke O u odnosu na krugove k1 i k2 jednake
kvadratu duzi r, dakle i medu sobom, pa se tacka O nalazi na radikalnoj osi
s krugova k1 i k2. Obratno, ako je O proizvoljna tacka prave s koja se nalazi
izvan krugova k1 i k2, njena potencija u odnosu na krugove k1 i k2 su medu
sobom jednake, pa su jednaki i osecci na tangentama krugova k1 i k2 kroz tacku
O. Stoga je tacka O srediste jednog kruga koji je ortogonalan na krugovima k1
i k2 (sl. 347).

373

A
Z

Y
Q

R
R
Q
P

slika 347
553. Ako je srediste kruga k tacka radikalne ose krugova k1 i k2 , a krug k
ortogonalan na krugu k1 , dokazati da je krug k ortogonalan na krugu k2 .
S obzirom da je srediste O kruga k tacka radikalne ose s krugova k1 i k2,
potencije tacke O u odnosu na krugove k1 i k2 su medu sobom jednake. No
krug k je ortogonalan na krugu k1, pa je potencija tacke O u odnosu na krug k1
jednaka kvadratu poluprecnika r kruga k. Stoga je potencija tacke O u odnosu
na krug k, takode jednaka kvadratu duzi r, pa je krug k ortogonalan na krugu k2.
554. Ako je P tacka radikalne ose s krugova k1 i k2 , dokazati da se polare
p1 i p2 tacke P u odnosu na krugove k1 i k2 seku u tacki koja se takode nalazi
na pravoj s, ili su paralelne s pravom s.
Ako se prave p1 i p2 seku u nekoj tacki Q, bice tacke P i Q spregnute u
odnosu na krugove k1 i k2, pa je krug k, kome je duz P Q precnik, ortogonalan
na krugovima k1 i k2 (v. z. 498). Stoga je srediste O kruga k na radikalnoj osi
s krugova k1 i k2. S obzirom da prava s sadrzi srediste O kruga k i tacku P
koja se nalazi na tom krugu, ona sadrzi i tacku Q koja je dijametralno suprotna
s tackom. Ako su prave p1 i p2 paralelne, tacka P je presek radikalne ose s
krugova k1 i k2 sa pravom koja spaja sredista O1 i O2 tih krugova. Pri tome
su prave s, p1, p2 upravne na pravoj O1O2, te su medu sobom paralelne.
555. Ako se secice AB i CD dvaju krugova k1 i k2 seku na radikalnoj osi s
tih krugova, dokazati da tacke A, B, C, D pripadaju jednom krugu.

374

S obzirom da je presecna tacka P pravih AB i CD na radikalnoj osi s krugova


k1 i k2, potencije tacke P u odnosu na krugove k1 i k2 su medu sobom jednake,
pa je
P A P B = P C P D.
Stoga tacke A, B, C, D pripadaju jednom krugu (sl. 348).

A
Q1

R2

P2
O
P1
B

R1 Q2

slika 348
556. Ako neki krug k sece dva kruga k1 i k2 , prvi u tackama A i B, a drugi
u tackama C i D, dokazati da se prave AB i CD seku na radikalnoj osi s dvaju
krugova k1 i k2 ili su uporedni s pravom s.
S obzirom da su prave s, AB, CD radikalne ose krugova k1 i k2, k i k1, ki
i k2, prema zadatku 551, one se seku u jednoj tacki, ili su medu sobom uporedne.
557. Dokazati da se prave odredena dvema antihomolognim tetivama A1 B1
i A2 B2 krugova k1 i k2 seku na radikalnoj osi s tih krugova.
Prema zadatku 424, tacke A1 , A2 , B1 , B2 pripadaju izvesnom krugu k. Pri
tome su prave s, A1 A2 i A2 B2 radikalne ose krugova k1 i k2 , k1 i k, i k2 i k
respektivno, pa se prema zadatku 551, prave A1 B1 i A2 B2 seku u nekoj tacki
P koja se nalazi na radikalnoj osi s krugova k1 i k2 .

375

Q
N

R
M

slika 349
558. Dokazati da se dirke dvaju krugova u antihomolognim tackama seku
na radikalnoj osi tih krugova, ili su uporedne sa radikalnom osom tih krugova.
Neka su A1 i A2 antihomologne tacke dvaju krugova k1 i k2 , a a1 i a2 njihove
tangente u tackama A1 i A2 . S obzirom na to da su A1 i A2 antihomologne tacke
krugova k1 i k2 , prave odredene poluprecnicima O1 A1 i O2 A2 se ili seku ili su
istovetne. Ako se seku u nekoj tacki O, i prave a1 i a2 upravne na njima seku
se u nekoj tacki P . U trouglu OA1 A2 uglovi kod temena A1 i A2 su jednaki,
pa su i duzi OA1 i OA2 medusobno jednake. Odatle sledi da postoji krug k
sa sredistem u tacki O, koji sadrzi tacke A1 i A2 . Taj krug dodiruje krugove
k1 i k2 u tackama A1 i A2 , pa su prave a1 i a2 radikalne ose krugova k1 , k i
k2 , k. Stoga se, prema zadatku 551, presecna tacka P pravih a1 i a2 nalazi na
radikalnoj osi s krugova k1 i k2 . Ako su prave O1 A1 i O2 A2 istovetne, onda su
prave a1 i a2 upravne na pravoj O1 O2 , pa su, prema tome, paralelne radikalnoj
osi s tih krugova.

376

Q
C

A
B

slika 350
559. Ako se dirke dvaju krugova k1 i k2 seku na radikalnoj osi tih krugova,
dokazati da su dodirne tacke tih dirki antihomologne.
Neka se tangente na krugove k1 i k2 u tackama A1 i A2 seku u tacki P , koja
se nalazi na radikalnoj osi s krugova k1 i k2 . Kako se prave P A1 i P A2 seku,
i upravne A1 O1 i A2 O2 na tim pravama u tackama A1 i A2 takode se seku u
nekoj tacki O. Iz podudarnosti trouglova P OA1 i P OA2 sledi da su duzi OA1
i OA2 medusobno jednake. Stoga postoji krug k sa sredistem O koji u tackama
A1 i A2 dodiruje krugove k1 i k2 . Odatle su, prema zadatku 525, tacke A1 i A2
antihomologne.
560. Ako su AA , BB , CC visine trougla ABC, dokazati da se presecne
tacke P , Q, R pravih BC i B C , CA i C A , AB i A B nalaze na radikalnoj
osi krugova l i l opisanih oko trouglova ABC i A B C .
Posto je cetvorougao BCC B tetivan, njegov opisani krug sece krugove l i
l u tackama B, C i B , C pa je, prema zadatku 551, presecna tacka P pravih
BC i B C na radikalnoj osi krugova l i l . Na isti nacin dokazuje se da i tacke
Q i R pripadaju radikalnoj osi krugova l i l (slika 351).

377

C
C

slika 351
561. Ako su Sa , Sb , Sc sredista spolja upisanih krugova trougla ABC, dokazati da se presecne tacke P , Q, R pravih BC i Sb Sc , CA i Sc Sa , AB i Sa Sb ,
nalaze na radikalnoj osi krugova l i l opisanih oko trouglova ABC i Sa Sb Sc .

Cetvorougao
BCSb Sc je tetivan, pa je P B P C = P Sb P Sc i, prema tome,
tacka P na radikalnoj osi krugova opisanih oko trouglova ABC i Sa Sb Sc . Istim postupkom dokazuje se da i tacke Q i R pripadaju radikalnoj osi krugova
opisanih oko trouglova ABC i Sa Sb Sc (slika 352).

378

D
C
B
A
S

slika 352
562. Ako su S, Sb , Sc srediste upisanog kruga i srediste spolja upisanih krugova trougla ABC, dokazati da se presecne tacke P , Q, R pravih BC i Sb Sc , CA
i SSc , AB i SSb nalaze na radikalnoj osi krugova l i l opisanih oko trouglova
ABC i SSb Sc .

Cetvorougao
BCSb Sc je tetivan, pa je P B P C = P Sb P Sc i, prema tome,
tacka P na radikalnoj osi krugova opisanih oko trouglova ABC i SSb Sc . Istim
postupkom dokazuje se da i tacke Q i R pripadaju radikalnoj osi tih krugova
(slika 353).

379

P6

A5

P5
A6

P4
P3

A4

A3
P2

A1

A2

A1

P1

A2

A6 A4

A5A3

slika 353
563. Dokazati da je prava odredena visinom AA trougla ABC radikalna osa
krugova ciji su precnici tezisne linije BB1 i CC1 tog trougla.
Obelezimo sa B i C podnozja visina iz temena B i C,a sa C1 i B1 sredista
stranica redom BA i CA trougla ABC . S obzirom da su uglovi BB Ba i CC C1
pravi , tacke B i C su na krugovima k1 i k2 kojima su precnici BB1 i CC1
. Tacke B1 , C1 , B , C pripadaju Ojlerovom krugu trougla ABC , pa je
AB AB1 = AC AC1 .Stoga su potencije tacke A u odnosu na krugove k1 i k2
medu sobom jednake,pa je tacka A na radikalnoj osi krugova k1 i k2 .Prava AA
upravna je na pravoj BC,dakle i na pravoj koja je odredena sredistem krugova
k1 i k2 ,pa je prava AA radikalna osa krugova k1 i k2 .

380

B
A

Q
P

R
C
A

Sl. 354
564. Ako su A1 , B1 , C1 sredista stranica BC, CA, AB trougla ABC, dokazati da su simetrale unutrasnjih uglova trougla A1 B1 C1 radikalne ose upisanih
krugova k, ka , kb , kc trougla ABC.
Obelezimo sa P ,Q,R tacke u kojima upisani krug k dodiruje stranice BC,CA,
AB trougla ABC,a sa Pa ,Qa ,Ra tacke u kojima upisani krug ka dodiruje stranicu BC i produzenja stranica AC i BC.Prema zadatku 550,sredista duzi P Pa ,
QQa ,RRa su na radikalnoj osi sa krugova k i ka .Srediste A1 stranice BC poklapa se sa sredistem duzi P Pa ,prema tome,prava sa sadrzi tacku A1 trougla
A1 B1 C1 .S obzirom da prava sa sadrzi srediste duzi QQa i RRa ,ona je upravna
na simetrali unutrasnjeg ugla kod temena A trougla ABC,dakle i simetrala unutrasnjeg ugla A1 trougla A1 B1 C1 .Otuda sleduje da je radikalna osa sa krugova
k i ka simetrala spoljasnjeg ugla A1 trougla A1 B1 C1 .Istim postupkom dokazuje
se da je radikalna osa krugova kb i kc simetrala unutrasnjeg ugla A1 trougla
A1 B1 C1 .

381

A
R

Q
O
B

Sl. 355
565. Dokazati da je ortocentar trougla radikalno srediste triju krugova ciji
su precnici stranice tog trougla.
Neka su AA ,BB ,CC visine,a la ,lb ,lc krugovi ciji su precnici stranice BC,CA,
AB trougla ABC.Krugovi lb i lc seku se u tackama A i A ,pa je prava AA njihova radikalna osa.Isto tako prava BB radikalna osa krugova lc i la ,a prava
CC radikalna osa krugova la i lb .Stoga je ortocentar H trougla ABC radikalno
srediste krugova la ,lb ,lc .

382

R
Q
A
B

Sl. 356
566. Ako je M tacka u ravni trougla ABC takva da prave M A, M B, M C
seku prave BC, CA, AB u tackama P , Q, R, dokazati da je ortocentar H trougla ABC radikalno srediste krugova sa dijametrima AP , BQ, CR.
Visine AA ,BB ,CC trougla ABC su tetive krugova la ,lb ,lc kojima su duzi
AP ,BQ,CR dijametri.No prave AA ,BB ,CC seku se u tacki H takvoj da je
HA HA = HB HB = HC HC ,pa u potencije tacke H u odnosu na krugove
la ,lb ,lc medu sobom jednake.Stoga je tacka H radikalno srediste krugova la ,lb ,lc .
567.Neka su O1 i O2 sredista,r1 i r2 poluprecnici i s radikalna osa krugova k1 i
k2 ,zatim p1 i p2 potencije tacke P u odnosu na krugove k1 i k2 ,Q i R podnozja
upravnih iz tacke P na pravama s i O1 O2 ,a S presek pravih s i O1 O2 .Pri tome je
p1 p2 = (O1 P 2 r12 )(O22 r22 ) = (O1 P 2 O2 P 2 )(r12 r22 ) = (O1 R2 O2 R2 )
(O1 S 2 O2 S 2 ) = (O1 R + RO2 ) (O1 R RO2 ) (O1 S + SO2 ) (O1 S SO2 ) =
O1 O2 (O1 RRO2 )O1 O2 (O1 S SO2 ) = O1 O2 (O1 R O1 S) + (SO2 RO2 ) =
O1 O2 (SR + SR) = 2 O1 O2 SR = 2 O1 O2 P Q.

383

G
R
P
B

Sl. 357
568. Dokazati da je skup svih tacaka kojima je razlika potencija u odnosu
na dva kruga k1 i k2 stalna, prava uporedna ili istovetna s radikalnom osom s
krugova k1 i k2 .
Prema prethodnom zadatku,razlika potencija naka tacke P u odnosu na dva
kruga k1 i k2 je funkcija odstojanja te tacke od radikalne ose s krugova k1 i
k2 .Stoga je skup svih tacaka kojima je razlika potencija u odnosu na krugove
k1 i k2 stalna,prava uporedna ili istovremena s radikalnom osom s tih dvaju
krugova.
569. Ako su O1 i O2 sredista, a r1 i r2 poluprecnici dvaju krugova k1 i k2 i
O srediste duzi O1 O2 , dokazati da je
2O1 O2 OS = r12 r22 .
S obzirom da je tacka S na radikalnoj osi s krugova k1 i k2 , imamo da je
O1 S 2 O2 S 2 = r12 r22 . Otuda je (O1 S + SO2 ) (O1 S SO2 ) = r12 r22 , i prema
tome
2 O1 O2 OS = r12 r22
.
570. Ako su O i r srediste i poluprecnik opisanog kruga trougla ABC, H
ortocentar tog trougla i X podnozje normale iz tacke O na ortickoj pravoj tog
trougla, dokazati da je
3r2 + OH 2
.
OX =
4OH
384

Prema zadatku 356 prave odredene stranicama trougla ABC seku prave
odredene odgovarajucim stranicama njegovog ortickog trougla A B C u tackama
koje pripadaju jednoj pravoj, ortickoj pravoj trougla ABC. Prema zadatku 560,
ta prava predstavlja radikalnu osu krugova l i l opisanih oko trouglova ABC i
A B C . Stoga je ta orticka prava upravna u tacki X na pravoj koja je odredena
sredistima O i O krugova l i l , tj. na Ojlerovoj pravoj OH trougla ABC. Ako
obelezimo sa O srediste duzi OO , prema zadatku 569, imamo da je
2 OO O X = r2

r2
4

te je


3
1
OH OX OH = r2 ,
4
4
i prema tome
4 OH OX = 3r2 + OH 2 .
Ovim je stav dokazan.
571. Ako je l(O, r) opisani i k(S,) upisani krug trougla ABC, a X tacka
u kojoj prava OS sece pravu s na kojoj se nalaze preseci simetrala spoljasnjih
uglova sa produzenjima naspramnih stranica tog trougla, dokazati da je
OX 2 = (r + q)2

r
.
r 2q

Prema zadatku 337 tacke u kojima simetrale spoljasnjih uglova seku prave
odredene naspramnim stranicama trougla ABC pripadaju jednoj pravoj s. Ako
obelezimo sa Sa , Sb , Sc tacke u kojima se seku simetrale spoljasnjih uglova, tj.
sredista spolja upisanih krugova trougla ABC, prema zadatku 516, bice prava s
radikalna osa krugova l(O, r) i l (O , r ) opisanih oko trouglova ABC i Sa Sb Sc .
Stoga je prava s upravna na pravoj OO tj. pravoj OS u tacki X. Prema zadatku
152 krug l je Ojlerov krug trougla Sa Sb Sc , te se njegovo srediste O poklapa sa
sredistem duzi SO , dok je r = 2r. Ako sad obelezimo sa O srediste duzi OO ,
prema zadatku 569, bice
2 OO O X = 3r2 ,
te je


1
2 OS OX + OS = 3r2 .
2
Otuda i iz relacije
OS 2 = r(r 2)
nalazimo da je
OX 2 = (r + )2
Ovim je stav dokazan.

r
r 2

572. Ako je l(O, r) opisani i ka (Sa , a ) spolja upisani krug trougla ABC a
Xa tacka u kojoj prava OSa sece pravu sa na kojoj se nalaze preseci simetrale

385

spoljasnjeg ugla A i simetrala unutrasnjih uglova B i C sa pravama odredenim


naspramnim stranicama trougla ABC, dokazati da je
OXa2 = (r a )2

r
.
r + 2a

Prema zadatku 336 tacke u kojima simetrale spoljasnjeg ugla A i unutrasnjih


uglova B i C seku prave odredene naspramnim stranicama trougla ABC pripadaju jednoj pravoj sa . Ako obelezimo sa S, Sb , Sc preseke pomenutih simetrala,
prema zadatku 562 bice prava sa radikalna osa krugova l(O, r) i la (Oa , ra ) opisanih oko trouglova ABC i SSb Sc . Stoga je prava sa upravna na pravoj OOa u
tacki X. S obzirom da krug l sadrzi podnozja visina trougla SSb Sc , krug l je
Ojlerov krug trougla SSb Sc , pa se njegovo srediste O poklapa sa sredistem duzi
koja je odredena sredistem Oa opisanog kruga i ortocentrom Sa trougla SSb Sc ,
dok je ra = 2r. Ako sad obelezimo sa O srediste duzi OOa , prema zadatku
569, imamo da je
2 OOa O X = 3r2 ,
te je
2 OSa

1
OSa OX
2

= 3r2 .

Otuda i iz relacije
OSa2 = r(r + 2a )
nalazimo da je
OX 2 = (r a )2

r
.
r + 2a

Ovim je stav dokazan.


573. Neka su s1 , s2 , s3 radikalne ose krugova k2 i k3 , k3 i k1 , k1 i k2 . Ako
prave s1 i s2 sadrze sredista O1 i O2 krugova k1 i k2 , dokazati da i prava s3
sadrzi srediste O3 kruga k3 .
Ako su r1 , r2 , r3 poluprecnici krugova k1 , k2 , k3 imamo da je
O1 O22 r22 = O1 O32 r32
i
O2 O1 r12 = O2 O32 r32 ,
pa je
O1 O32 O2 O32 = r12 r32 .
Otuda sleduje da radikalna osa s3 krugova k1 i k2 sadrzi srediste O3 kruga k3
(slika 358).

386

O3

k3
s2

s1

k1

O1

O2

k2

s3

Slika 358
574. Dokazati da je skup sredista svih krugova koje seku dati krugovi k1 i
k2 u dijametralno suprotnim tackama, odsecak na radikalnoj osi krugova k1 i
k2 koji se nalazi u krugovima k1 i k2 .
Obelezimo sa O1 i O2 sredista krugova k1 i k2 , sa r1 i r2 poluprecnike tih
krugova, a sa O srediste i r poluprecnik bilo kojeg kruga k kojeg sece krug k1 u
dijametralno suprotnim tackama A1 i B1 , a krug k2 u dijametralno suprotnim
tackama A2 i B2 . Iz pravouglih trouglova OO1 A1 i OO2 A2 s jednakim katetama
OA1 i OA2 nalazimo da je
O1 A21 O1 O2 = O2 A22 O2 O2 ,
tj. da je
O1 O2 O2 O2 = r12 r22 .
Otuda sledi da sredista svih krugova koje seku krugovi k1 i k2 u dijametralno
suprotnim tackama pripadaju radikalnoj osi s krugova k1 i k2 . S obzirom da je
tacka O na tetivama A1 B1 i A2 B2 krugova k1 i k2 , tacka O je u krugovima k1 i
k2 . Obratno, dokazimo sad da je svaka tacka O radikalne ose s krugova k1 i k2 ,
koja se nalazi u krugovima k1 i k2 , srediste izvesnog kruga kojeg krugovi k1 i k2
seku u dijametralno suprotnim tackama. S obzirom da je tacka O u krugovima
k1 i k2 , upravna u tacki O na pravoj O1 O sece krug k1 u tackama, npr. A1
i B1 , a upravna u tacki O na pravoj O2 O sece krug k2 u tackama, npr. A2 i
B2 . Po pretpostavci, tacka O je na radikalnoj osi s krugova k1 i k2 , pa je
O1 O2 O2 O2 = r12 r22 .
Iz pravouglih trouglova O1 O A1 i O2 O A2 nalazimo da je
O1 O2 = r12 O A2
1
387

i
O2 O2 = r22 O A2
2,

pa je O A1 = O A2 . No O A1 = O B1 i O A2 = O B2 , pa su tacke A1 , B1 , A2 , B2
na jednom krugu, kojeg seku krugovi k1 i k2 u dijametralno suprotnim tackama
(slika 359).

A1
A2

O
O1

O2
B2

B1

Slika 359

6.8

Pseudoradikalna osa dvaju krugova.


Pseudoradikalno sredi
ste triju krugova.

Definicija 6.9. Pravu simetricnu sa radikalnom osom dvaju krugova u odnosu


na srediste duzi koja spaja centre tih krugova nazivamo pseudoradikalnom ili
antiradikalnom osom tih dvaju krugova.
S obzirom da za svaku tacku P radikalne ose s dvaju krugova k1 (O1 , r1 ) i
k2 (O2 , r2 ) imamo da je
O1 P 2 = O2 P 2 = r12 r22 ,
za svaku tacku P pseudoradikalne ose s tih dvaju krugova bice
O1 P 2 O2 P 2 = r22 r12 .
Definicija 6.10. Tacku S simetricnu sa radikalnim sredistem S triju krugova
k1 , k2 , k3 u odnosu na srediste O kruga koji je odreden sredistima tih triju krugova nazivamo pseudoradikalnim ili antiradikalnim sredistem krugova k1 , k2 , k3 .
575. Dokazati da je skup sredista svih krugova koji seku dva data kruga k1
i k2 u dijametralno suprotnim tackama pseudoradikalna osa krugova k1 i k2 .
Resenje: Obelezimo sa O1 i O2 sredista krugova k1 i k2 , sa r1 i r2 poluprecnike tih krugova i sa O srediste bilo kojeg kruga k koji sece krug k1 u
dijametralno suprotnim tackama A1 i B1 , a krug k2 u dijametralno suprotnim
tackama A2 i B2 . Iz pravouglih trouglova OO1 A1 i OO2 A2 sa jednakim hipotenuzama OA1 i OA2 , nalazimo da je
OO12 + O1 A21 = OO22 + O2 A22 ,

388

tj. da je
OO12 OO22 = O2 A22 O1 A21 = r22 r12 .

Odatle sledi da sredista svih krugova koji seku krugove k1 i k2 u dijametralno


suprotnim tackama pripadaju pseudoradikalnoj osi krugova k1 i k2 .
Obratno, svaka tacka O pseudoradikalne ose s krugova k1 i k2 je srediste
jednog kruga k koji sece krugove k1 i k2 u dijametralno suprotnim tackama.
Zaista, neka su A1 B1 i A2 B2 precnici krugova k1 i k2 upravni na duzima O1 O
i O2 O . S obzirom da je
O1 O2 O2 O2 = r22 r12 ,
bice
2
2
2
2
2
(OA2
1 r1 ) (OA2 r2 ) = r2 r1 ,

pa je OA1 = OA2 . No OA1 = OB1 i OA2 = OB2 , pa su tacke A1 , B1 , A2 , B2


na jednom krugu kome je srediste O , a koji sece svaki od krugova k1 i k2 u
dijametralno suprotnim tackama.(slika 360)

A1
A2

O1

O2
B2

k1

k2

B1

Slika 360
576. Dokazati da se pseudoradikalne ose triju krugova seku u jednoj tacki,
ili su medu sobom uporedne, ili su pak istovetne.
Resenje: Neka su O1 , O2 , O3 sredista triju krugova k1 , k2 , k3 , a s1 , s2 , s3 radikalne ose i s1 , s2 , s3 pseudoradikalne ose krugova k2 , k3 i k3 , k1 i k1 , k2 .
Ako tacke O1 , O2 , O3 nisu na jednoj pravoj, prave s1 , s2 , s3 seku se u jednoj
tacki, radikalnom sredistu S tih krugova. Tacka S simetricna sa tackom S u
odnosu na srediste O kruga opisanog oko trougla O1 O2 O3 pripada pseudoradikalnim osama s1 , s2 , s3 tih krugova, prema tome, preudoradikalne ose krugova
k1 , k2 , k3 seku se u jednoj tacki.
Ako su tacke O1 , O2 , O3 na jednoj pravoj, pseudoradikalne ose s1 , s2 , s3 datih
krugova upravne su na toj pravoj u trima raznim tackama, ili pak u istoj tacki,
prema tome, one su medu sobom uporedne ili su, pak, istovetne.
389

6.9

Elipti
cki, paraboli
cki i hiperboli
cki pramenovi
krugova

Definicija 6.11. Skup svih krugova jedne ravni od kojih svaka dva imaju za
radikalnu osu istu pravu s nazivamo sistemom koaksijalnih krugova ili pramena
krugova, a pravu s radikalnom osom tog pramena krugova.
Iz ove definicije neposredno slede ove osobine. Ako se u jednom pramenu
krugova dva kruga seku u tackama, npr. A i B, tada se svaka dva kruga tog
pramena seku u istim tackama A i B; ako se u jednom pramenu krugova dva
kruga dodiruju u nekoj tacki S, tada se svaka dva kruga tog pramena dodiruju u
tacki S; ako u jednom pramenu krugova dva kruga nemaju zajednickih tacaka,
tada nikoja dva kruga tog pramena nemaju zajednickih tacaka. S obzirom na
takav uzajamni polozaj krugova jednog pramena, razlikujemo sledece tri vrste
pramenova krugova:
(a) Elipticki pramen krugova - kod kojeg se svi krugovi seku u dvema istim
tackama, osnovnim tackama tog pramena krugova. (slika a)

Slika a
(b) Parabolicki pramen krugova - kod kojeg se svi krugovi medu sobom
dodiruju u istoj tacki, osnovnoj tacki tog pramena krugova. (slika b)

390

Slika b
(c) Hiperbolicki pramen krugova - kod kojeg nikoja dva kruga nemaju zajednickih tacaka. (slika c)

Slika c
Pored navedene tri vrste pramenova krugova, koje smatramo nedegenerisanim, postoje jos dve vrste degenerisanih pramenova krugova. To je pramen

391

koncentricnih krugova koji za radikalnu osu imaju beskrajno daleku pravu i pramen pravih tj. krugova beskrajno velikih poluprecnika (pravih koje se seku u
jednoj tacki ili koje su medusobom uporedne). U ovom poglavlju proucavacemo
samo osobine nedegenerisanih pramenova krugova.
Nedegenerisani pramen krugova potpuno je odreden ako su data dva njegova kruga, ili, pak, ako su dati njegova radikalna osa i jedan njegov krug. Nije
tesko dokazati da se skup sredista svih krugova bilo kojeg nedegenerisanog pramena krugova nalazi na jednoj pravoj. Zaista, ako su O1 , O2 , O3 ... sredista
krugova k1 , k2 , k3 ... nekog pramena, sve prave O1 O2 , O1 O3 ,... sadrze tacku O1 ,
a upravne su na radikalnu osu tog pramena krugova, prema tome, one su istovetne. Prava koje sadrzi stediste krugova nekog pramena naziva se sredisnja ili
centralna prava tog pramena krugova, a tacka u kojoj ta prava sece radikalnu
osu naziva se srediste ili centar tog pramena krugova.
Prirodno se namece i pitanje da li je svaka tacka sredisnje prave, nekog
pramena krugova, srediste izvesnog kruga koji pripada tom pramenu krugova.
Odgovor na ovo pitanje dajemo pojedinacno za pojedine vrste pramena krugova.
Ako je pramen krugova elipticki, tacke A i B u kojima se seku svi njegovi
krugovi, simetricne su medu sobom u odnosu na sredisnju pravu p tog pramena,
pa je svaka tacka prave p srediste izvesnog kruga koji sadrzi tacke A i B i, prema
tome, pripada tom pramenu.
Ako je pramen krugova parabolicki, tacka C u kojoj se dodiruju svi njegovi
krugovi je na sredisnjoj pravoj p tog pramena. Stoga je svaka tacka prave p,
izuzev tacke C, stediste izvesnog kruga koji u tacki C dodiruje sve krugove tog
pramena i, prema tome, pripada tom pramenu.
Ako je pramen krugova hiperbolicki, njegovo srediste C, kao tacka radikalne
ose, ima jednake potencije u odnosu na sve krugove tog pramena, i to pozitivne
jer se tacka C nalazi izvan svih tih krugova. Stoga je za svaki krug k(O, r) tog
pramena CO2 r2 = c2 , i prema tome je CO > c. Odatle sledi, da samo one
tacke O sredisnje prave za koje je CO > c predstavljaju sredista krugova koji
pripadaju tom pramenu. Stoga, tacke C1 i C2 sredisnje prave koje se nalaze sa
raznih strana tacke C takve da je CC1 = CC2 = c nazivamo granicnim tackama
tog hiperbolickog pramena krugova.
577. Dokazati da su potencije proizvoljne tacke radikalne ose nekog pramena
krugova u odnosu na sve krugove tog pramena medu sobom jednake.
Resenje: Radikalna osa s nekog pramena krugova je radikalna osa bilo koja
dva kruga tog pramena, pa su potencije bilo koje tacke P prave s u odnosu na
sve krugove tog pramena medu sobom jednake.
578. Dokazati da se tacka S jednakim potencijama u odnosu na sve krugove
nekog pramena krugova nalazi na radikalnoj osi tog pramena krugova.
Resenje: Ako su potencijali tacke S u odnosu na sve krugove nekog pramena
medju sobom jednake, tacka S je na radikalnoj osi bilo koja dva kruga tog
pramena, dakle i na radikalnoj osi tog pramena krugova.
579. Ako je srediste kruga l na radikalnoj osi nekog pramena krugova i ako
je krug l ortogonalan na jednom krugu k tog pramena, dokazati da je krug l
ortogonalan na svim krugovima tog pramena krugova.
Resenje: S obzirom da je krug l ortogonalan na krugu k, potencija njegovog
stedista O u odnosu na krug k jednaka je kvadratu poluprecnika r kruga l. Po
pretpostavci, tacka O je na radikalnoj osi datog pramena, kome pripada i krug k,
pa su potencije tacke O u odnosu na sve krugove tog pramena jednake kvadratu
duzi r. Stoga je krug k ortogonalan na svim krugovima datog pramena krugova.
392

580. Ako je krug l ortogonalan na dvama krugovima k1 i k2 nekog pramena


krugova, dokazati da je krug l ortogonalan na svim krugovima tog pramena
krugova.
Resenje: S obzirom da je krug l ortogonalan na krugovima k1 i k2 , njegovo
srediste S je na radikalnoj osi krugova k1 i k2 , tj. na radikalnoj osi datog
pramena krugova. S toga su potencije tacke S u odnosu na sve krugove tog
pramena jednake kvadratu poluprecnika kruga l, pa je krug l ortogonalan na
svim krugovima datog pramena krugova.
581. Dokazati da je svaka tacka radikalne ose nekog pramena krugova koja
se nalazi izvan tih krugova srediste jednog kruga koji je ortogonalan na svim
krugovima tog pramena krugova.
Resenje: Ako je S proizvoljna tacka radikalne ose s nekog pramena krugova,
koja se nalazi izvan krugova tog pramena, bice potencije tacke S u odnosu na
krugove tog pramena jednake, pa su i odsecci na dirkama tih krugova kroz tacku
S takode jednake. Stoga postoji krug l, kome je srediste S, koji je ortogonalan
na svim krugovima datog pramena.
582. Dokazati da krugovi ortogonalni na svim krugovima nekog pramena
krugova takode obrazuju pramen krugova.
Resenje: Dokazimo najpre da svi krugovi ortogonalni na krugovima nekog
pramena {k} pripadaju izvesnom pramenu {l}. Obelezimo sa k1 i k2 dva proizvoljna kruga pramena {k}, a sa l1 i l2 dva kruga ortogonalna na svim krugovima
pramena {k}. S obzirom da su krugovi k1 i k2 ortogonalni na krugovima l1 i l2 ,
sredista O1 i O2 krugova k1 i k2 su na radikalnoj osi krugova l1 i l2 . Stoga svaka
dva kruga ortogonalna na krugovima pramena {k} imaju za radikalnu osu istu
pravu O1 O2 , prema tome, svi krugovi ortogonalni na krugovima pramena {k}
pripadaju izvesnom pramenu {l}. Dokazimo da svaki krug pramena {l} ortogonalan na svim krugovima pramena {k}. S obzirom sa je krug k1 ortogomalan
na krugovima l1 i l2 pramena {l}, krug k1 je ortogonalan na svim krugovima
pramena {l}, pa je i svaki krug pramena {l} ortogonalan na svim krugovima
pramena {k}. Stoga krugovi ortogonalni na svim krugovima nekog pramena
krugova obrazuju takode pramen krugova.
583. Dokazati da krugovi ortogonalni na svim krugovima eliptickog pramena
obrazuju hiperbolicki pramen.
Resenje: Prema prethodnom zadatku, krugovi ortogonalni na svim krugovima eliptickog pramena {k} obrazuju neki pramen {l}. S obzirom da je pramen
{k} elipticki, svaka tacka njegove sredisnje prave c koje srediste nekog kruga koji
je ortogonalan na svim krugovima pramena {l} pa su sve tacke c izvan svih krugova pramena {l}. Otuda sleduje da prava c, tj. radikalna osa pramena {l} nema
s krugovima tog pramen zajednickih tacaka, pa je pramen {l} hiperbolicki.
584. Dokazati da krugovi ortogonalni na svim krugovima hiperbolickog pramena obrazuju elipticki pramen.
Resenje: Prema zadatku 582, krugovi ortogonalni na svim krugovima hiperbilickog pramena {k} obrazuju neki pramen {l}. S obzirom da je pramen {k}
hiperbolick, njegovi krugovi nemaju s radikalnom osom s zajednickih tacaka,
pa su sve tacke prave s izvan krugova pramena {k}. Stoga je svaka tacka prave
s srediste izvesnog kruga koji je ortogonalan na svim kugovima pramena {k}.
Neka je O tacka u kojoj sredisnja prava s pramena {k} sece radikalnu osu s.
Krug pramena {l} kome je srediste O sece sredisnu pravu c pramena {k}, tj.
radikalnu osu pramena {l} u dvema tackama koje pripadaju svim krugovima
pramena {l} pa je pramen {l} elipticki.
393

585. Dokazati da krugovi ortogonalni na krugovima parabolickog pramena


obrazuju parabolicki pramen.
Resenje: Prema zadatku 582, krugovi ortogonalni na krugovima pramena
{k} obrazuju neki pramen {l}. S obzirom da je pramen {k} parabolicki, svi
njegovi krugovi dodiruju radikalnu osu s tog pramena u istoj tacki O. Krugovi
pramena {l} ortogonalni su na krugovima pramena {k}, a sredista su im na
pravoj s, pa se ovi krugovi pramena {l} dodiruju medu sobom u tacki O, stoga
je pramen {l} parabolicki.
586. Dokazati da postoji jedan i samo jedan krug koji pripada datom pramenu krugova, a koji sadrzi datu tacku izvan radikalne ose tog pramena krugova.
Resenje: Neka je k, bili koji krug nekog pramena {k}, s radikalna osa tog
pramena i A tacka izvan prave s. Obelezimo, sa P i Q dve proizvoljne tacke
prave s, a sa B i C tacke pravih P A i QA takve da su proizvodi P A P B i
QA QC jednaka potencijama tacaka P i Q u odnosu na krug k1 . Tacke A, B, C
odreduju neki krug k2 . Pri tome je prava s radikalna osa krugova k1 i k2 , pa je i
k2 krug pramena {k}. Krug k2 je jedini krug pramena {k} koji sadrzi tacku A.
Zaista, ako bi postojao jos neki krug k2 , bar jedna od tacaka B i C, npr. C ne
bi bilo na krugu k2 . U tom slucaju, na pravoj QA postojala bi tacka C razlicita
od C takode da je QA QC = QA QC = 0, je nemoguce. Dakle, postoji jedan
i samo jedan krug koji sadrzi tacku A, a pripada pramenu {k}.
587. Dokazati da je svaki krug koji sadrzi granicne tacke hiperbolickog
pramena krugova ortogonalan na svim krugovima tog pramena krugova.
Resenje: Neka je l(S, ) krug koji sadrzi granicne tacke C1 i C2 hiperbilickog
pramena, k(O, r) bilo koji krug tog pramena i C srediste tog pramena. Saglasno
definiciji granicnih tacaka hiperbolickog pramena krugova, imamo da je CO2
r2 = CC12 . Kako je tacka S na simetrali duzi C1 C2 duz SC1 je hipotenuza
pravouglog trougla SCC1 , pa je CC12 = SC12 SC 2 . Iz dobijenih dveju jednakisti
sledi da je CO2 + SC 2 = r2 + SC12 , tj. da je SO2 = r2 + 2 . Stoga su krugovi
k i l ortogonalni.
589.Ako je C jedna od granicnih tacaka hiperbolickog pramena krugova i D
tacka u kojoj prava kroz C dodiruje neki krug tog pramena krugova, dokazati
da su tacke C i D harmonijski spregnute sa tackama A i B u kojima ta dirka
sece bilo koji drugi krug tog pramena krugova.
Resenje:Neka su C1 i C2 granicne tacke hiperbolickog pramena krugova, a D1
i D2 tacke u kojima sredisnja prava C1 C2 sece proizvoljan krug k tog pramena
krugova. Prema prethodnom zadatku proizvoljan krug l kroz tacke C1 i C2
ortogonalan je na svim krugovima, dakle i na krug k tog hiperbolickog pramena
krugova, pa je prema zadatku 474. H(C1 , C2 ; D1 , D2 ).
590.Ako su k1 , k2 , k3 krugovi jednog pramena, A i B tacke u kojima jedna
prava dodiruje krugove k1 i k2 , a C i D tackE u kojima ta ista prava sece krug
k3 , dokazati da su tacke C i D harmonijski spregnute s tackama A i B.
Resenje: Srediste S duzi AB je na radikalnoj osi pramena k kome pripadaju
krugovi k1 ,k2 ,k3 , pa je
SA2 = SB 2 = SC SD.
Stoga su tacke C i D harmonijski spregnute s pravom, koja prolazi kroz tacke
A i B.
591.Ako su sredista svih krugova k1 , k2 , . . . na jednoj pravoj i ako postoji
tacka P koja ima jednake potencije u odnosu na sve te krugove, dokazati da ti
krugovi pripadaju izvesnom pramenu krugova.

394

Resenje:S obzirom da tacka P ima jednake potencije u odnosu na krugove


k1 , k2 ,..., tacka P se nalazi na radikalnoj osi bilo koja dva kruga iz tog skupa
krugova. Sve te radikalne ose sadrze tacku P , a upravne su na istoj pravoj koja
sadrzi sredista datih krugova, prema tome, one su istovetne. Odatle sledi da
krugovi k1 , k2 , ... pripadaju izvesnom pramenu krugova.
592.Ako postoje dve tacke P i Q od kojih svaka ima jednake potencije u
odnosu na krugove k1 , k2 , . . ., dokazati da pomenuti krugovi pripadaju izvesnom
pramenu krugova.
Resenje:Prava P Q je radikalna osa bilo koja dva kruga k1 , k2 , ..., prema tome,
pomenuti krugovi pripadaju izvesnom pramenu krugova.
593.Dokazati da su krugovi koji su ortogonalni na istom krugu i kojima se
sredista nalaze na jednoj pravoj obrazuju pramen krugova.
Resenje:Neka su k1 , k2 , ..., krugovi koji su ortogonalni na nekom krugu l,
i kojima se sredista nalaze na jednoj pravoj o. Potencije sredista S kruga l
jednake su u odnosu na sve krugove k1 , k2 , ..., pa prema tome, radikalne ose
parova tih krugova sadrze tacku S, a upravne su na pravoj o, dakle, poklapaju
se. Stoga pomenuti krugovi obrazuju pramen krugova.
594.Dokazati da se polare jedne tacke u odnosu na krugove bilo kojeg pramena seku u jednoj tacki ili su medusobom uporedne.
Resenje:Neka su k1 , k2 , k3 bilo koja tri kruga nekog pramena {k}, a p1 , p2 , p3
polare proizvoljne tacke P u odnosu na krugove k1 , k2 , k3 . Ako se prave p1 i
p2 seku u nekoj tacki Q, tacke P i Q su spregnute u odnosu na oba kruga k1 i
k2 , pa je, prema zadatku 498., krug l kome je duz P Q precnik ortogonalan na
krugove k1 i k2 . S obzirom da je krug l ortogonalan na dva kruga pramena {k},
on je ortogonalan na svim krugovima tog pramena, dakle, i na krug k3 . Prema
zadatku 499., dijametralno suprotne tacke P i Q kruga l, koji je ortogonalan
na krug k3 , spregnute su u odnosu na krug k3 , prema tome, polara p3 tacke P
u odnosu na krug k3 sadrzi tacku Q. Odatle, ako se polare tacke P u odnosu
na dva kruga pramena {k} seku u tacki Q, polare tacke P u odnosu na sve
krugove pramena {k} seku se u tacki Q. Indirektnim postupkom neposredno se
dokazuju da su u slucaju uporednosti dvaju od pomenu- tih polara sve te polare
medusobom uporedne.
595. Ako se polare neke tacke P u odnosu na krugove k1 , k2 , . . . kojima su
sredista na jednoj pravoj, seku u jednoj tacki Q, dokazati da pomenuti krugovi
pripadaju izvesnom pramenu krugova.
Resenje:Tacke P i Q su spregnute u odnosu na svaki od pomenutih krugova k1 , k2 , ..., pa je krug l, kome je duz P Q precnik, ortogonalan na svim tim
krugovima. Stoga, prema zadatku 593., krugovi k1 , k2 , ... pripadaju izvesnom
pramenu krugova.
596. Ako su tacke A i B spregnute u odnosu na neki krug k, dokazati da je
kvadrat duzi AB jednak zbiru potencija tacaka A i B u odnosu na krug k.
Resenje:Analizirajmo, najpre, slucaj kada prava AB sece krug k, neka su C
i D presecne tacke. Ako sa O obelezimo srediste duzi AB, bi`ce
AC AD = (AO + OC)(AO + OD) = AO2 + AO(OC + OD) + OC OD
i
BC BD = (BO + OC)(BO + OD) = BO2 + BO(OC + OD) + OC OD,
pa je
AC AD + BC BD = 2AO2 + 2OC OD.
395

Prema zadatku 500., tacke C i D su harmonijski spregnute sa tackama A i B,


pa je OC OD = AO2 . Odatle je
AC AD + BC BD = 4AO2 = AB 2 .
Analizirajmo, zatim, slucaj kada prava AB ne sece krug k. U tom slucaju
obe tacke A i B nalaze se izvan kruga k. Stoga, postoje krugovi l1 i l2 sa
sredistima A i B koji su ortogonalni na krugu k. S obzirom da su tacke A i B
spregnute u odnosu na krug k, krug l3 , kome je duz AB precnik, ortogonalan je
na krug k. Krugovi l1 , l2 , l3 , s kolinearnim sredistima, ortogonalni su na krug k,
te prema zadatku 593., pripadaju izvesnom pramenu {l} . Centralna prava AB
tog pramena krugova je radikalna osa njemu spregnutog pramena kome pripada
i krug k. S obzirom da krug k tog spregnutog pramena nema sa radikalnom osom
AB tog pramena zajednickih tacaka, taj pramen krugova je hiperbolicki. Odatle
sledi da je pramen {l} elipticki, pa se krugovi l1 , l2 , l3 seku u dvema tackama P
i Q. U pravouglom trouglu ABP katete AP i BP jednake su poluprecnicima
r1 i r2 krugova l1 il2 , pa je
AB 2 = AP 2 + BP 2 = r12 + r22 .
S toga je kvadrat duzi AB jednak zbiru potencije tacaka A i B u odnosu na
krug k.
597. Ako je kvadrat duzi AB jednak zbiru potencija tacaka A i B u odnosu
na krug k, dokazati da su tacke A i B spregnute u odnosu na taj krug.
Resenje:Analizirajmo najpre slucaj kad prava AB sece krug k, neka su C
i D presecne tacke. Ako sa O obelezimo srediste duzi AB, kao u prethodnom
zadatku, imamo da je AC AD + BC BD = 2AO2 + 2OC OD. Po pretpostavci
je AC AD + BC BD = AB 2 , pa je AB 2 = 2AO2 + 2OC OD, i prema tome
AO2 = AC AD. Stoga su tacke C i D harmonijski spregnute sa tackama A i
B, pa su tacke A i B spregnute u odnosu na krug k.
Sad analizirajmo slucaj kad prava AB ne sece krug k. U tom slucaju tacke
A i B su izvan kruga k, te postoje krugovi l1 i l2 sa sredistima A i B koji su
ortogonalni na krugu k. Ako sa r1 i r2 obelezimo poluprecnike krugova l1 i l2 ,
bice AB 2 = r12 + r22 . Otuda sleduje da se krugovi l1 i l2 seku, i da krug l3 kome
je duz AB precnik sadrzi presecne tacke. Stoga krugovi k1 , k2 i k3 pripadaju
izvesnom eliptickom pramenu l. Kako je krug k orogonalan na krugovima l1 i
l2 tog pramena, krug k je ortogonalan na svim krugovima tog pramena, dakle i
na krugu k3 . Otuda sleduje da su dijametralno spregnute tacke A i B kruga l3
spregnute u odnosu na krug k.
598.Ako su k1 , k2 , k3 medusobom ortogonalni krugovi, dokazati da su sredista
bilo koja dva od tih krugova spregnuta u odnosu na treci krug.
Resenje: Neka su O1 i O2 sredista, a r1 i r2 poluprecnici krugova k1 i k2 . S
obzirom da su ti krugovi k1 i k2 medusobom ortogonalni imamo da je O1 O2 2 =
r12 + r22 . Sem toga, krugovi k1 i k2 ortogonalni su i na krugu k3 , pa su kvadrati
poluprecnika r1 i r2 tih krugova jednaki potencijama tacaka O1 i O2 u odnosu
na krug k3 . Stoga je kvadrat duzi O1 O2 jednak zbiru potencija tacaka O1 i O2 u
odnosu na krug k3 , pa su prema zadatku 597, tacke O1 i O2 spregnute u odnosu
na krug k3
599.Ako su A , B , C tacke u kojima dirke kruga opisanog oko trougla ABC
konstruisane u temenima A, B, C seku prave BC, CA, AB, dokazati da krugovi
k1 (A , A A), k2 (B , B B), k3 (C , C C) pripadaju eliptickom pramenu krugova.
396

Resenje: Prema zadatku 338 tacke A ,B ,C pripadaju jednoj pravoj. S


obzirom da su krugovi k1 , k2 , k3 sa kolinearnim sredistima ortogonalni krugu
l koji je opisan oko trougla ABC, prema zadatku 593, oni pripadaju izvesnom
pramenu krugova. Dokazimo da je taj pramen elipticki. Prema zadatku 88 krug
k1 sadrzi tacke E i F u kojima simetrale unutrasnjeg i spoljasnjeg ugla A trougla
ABC seku pravu BC, pa je jedna od tacaka B i C, npr. tacka B u krugu k1 , a
druga izvan njega. Neka je G tacka u kojoj simetrala ugla C sece stranicu AB.
Kako je tacka A na krugu k1 , a tacka B u krugu k1 , sve unutrasnje tacke duzi
AB, dakle i tacka G, nalaze se u krugu k1 . Krug k3 sadrzi tacku G koja je u
krugu k1 i tacku C koja je izvan kruga k1 , prema tome, krugovi k1 i k3 se seku
u dvema tackama M i N . Stoga se svaka dva kruga dobijenog pramena seku u
tackama M i N , pa je on elipticki.
600.Ako su A , B , C tacke u kojima dirke kruga opisanog oko trougla ABC
konstruisane u temenima A, B, C seku prave BC, CA, AB, dokazati da krugovi
k1 , k2 , k3 kojima su duzi AA , BB , CC precnici pripadaju izvesnom pramenu
krugova kome je radikalna osa Ojlerova prava trougla ABC. Dokazati zatim da
je dobijeni pramen krugova elipticki, parabolicki ili hiperbolicki zavisno od toga
da li je trougao ABC ostrougli, pravougli ili tupougli.
Resenje: Prema zadatku 338 tacke A ,B ,C pripadaju jednoj pravoj. Pri
tome prave AB,BA ,A B ,B A obrazuju cetvorostranik kome su duzi AA ,BB
dijagonale. Prema zadatku 606 sredista tih dijagonala, tj. sredista krugova k1 ,
k2 , k3 pripadaju jednoj pravoj, Gausovoj pravoj tog cetvorostranika. S obzirom
da su krugovi k1 , k2 , k3 sa kolinearnim sredistima ortogonalni na krugu l koji
opisan oko trougla ABC, prema zadatku 593, oni pripadaju izvesnom pramenu
{k}. Kako je l ortogonalan na krugovima tog pramena, njegovo srediste je na
radikalnoj osi tog pramena. Visine AA, BB, CC trougla ABC su tetive
krugova t1 , t2 , t3 , a tacke H u kojoj se seku prave AA, BB, CC takva da
je HA HA = HB HB = HC HC.Stoga su potencije tacke H u odnosu
na krugove k1 , k2 , k3 jednake, te i ona pripada radikalnoj osi pramena {k}.
Otuda sleduje da je Ojlerova prava OH trougla ABC radikalna osa pramena
{k}. Drugi deo zadatka neka citalac dokaze sam.
601. Ako je l krug koji pripada pramenu {k}, dokazati da se radikalna
osa svih parova krugova, koji su obrazovani od krugova l i krugova pramena
{k}, seku u jednoj tacki na radikalnoj osi s pramena {k}, ili su uporedne sa
radikalnom osom tog pramena.
Resenje: Ako su O, O1 , O2 sredista triju krugova k, k1 , k2 bilo kojeg pramena
krugova, dokazati da su potencije svake tacke kruga k u odnosu na krugove k1 i
k2 srazmerne duzima O1 O i O2 O. Neka su k1 , k2 , k3 proizvoljni krugovi pramena
{k}, s1 , s2 , s3 radikalne ose krugova l i k1 , l i k2 ,l i k3 . Prema zadatku 551,
prave s, s1 , s2 seku se u jednoj tacki, ili su medjusobno upravne. Ako se prave
s, s1 , s2 seku u nekoj tacki S, prema istom stavu, u tacki S seku se i prave s, s1 ,
s3 pa se radikalne ose svih pomenutih prava krugova seku u jednoj tacki koja
se nalazi na radikalnoj osi pramena {k}. Ako su prave s, s1 , s2 uporedne bice
i prave s, s1 , s3 uporedne. Pa su radikalne ose svih pomenutih prava krugova
uporedne s radikalnom osom s pramena {k}.
602. Ako su O, O1 , O2 sredista triju krugova k, k1 , k2 bilo kojeg pramena
krugova, dokazati da su potencije svake tacke kruga k u odnosu na krugove k1
i k2 srazmerne duzima O1 O i O2 O.
Resenje: Ako obelezimo sa p1 i p2 potencije proizvoljne tacke P kruga k u
odnosu na krugove k1 i k2 , sa s radikalnu osu pramena kome pripadaju krugovi
397

k1 , k2 a sa Q podnozje upravne iz tacke P na pravoj s, prema zadatku 567, bice


p1 = 2O1 O P Q i p2 = 2O2 O P Q. Otuda je p1 : p2 = O1 O : O2 O (sl. 361)

B
A
C

R
S
C
X

A
B

P
Q

slika 361
603. Dokazati da je skup svih tacaka, cije su potencije u odnosu na dva data
kruga k1 i k2 srazmerne datim duzima m i n, krug pramena kojeg odreduju
krugovi k1 i k2 .
Resenje: Neka je P bilo koja tacka kojoj je potencija u odnosu na krugove
k1 i k2 srazmerna duzima m i n, a k krug koji sadrzi P a pripada pramenu
koji je odreden s krugovima k1 i k2 . Prema prethodnom zadatku, potencije
svake tacke kruga k u odnosu na krugove k1 i k2 srazmerne su duzima m i n.
Indirektnim postupkom dokazuje se da u krugu k i izvan toga kruga ne postoji
tacka koja ima tu istu osobinu. Stoga je skup svih tacaka cije su potencije u
odnosu na krugove k1 i k2 srazmerne duzima m i n, krug koji pripada pramenu
kojeg odreduju krugovi k1 i k2 .
604.Dokazati da krug k slicnosti dvaju krugova k1 i k2 pripada pramenu
kojeg odreduju krugovi k1 i k2 .
Resenje: Ako su O1 i O2 sredista, a r1 i r2 poluprecnici krugova k1 i k2 , zatim
P proizvoljna tacka kruga k, bice O1 P : O2 P = r1 : r2 tj. O1 P 2 : O2 P 2 = r12 :
r22 , pa je (O1 P 2 r12 ) : (O2 P 2 r22 ) = r12 : r22 . Stoga su potencije proizvoljne
tacke P kruga k u odnosu na krugove k1 i k2 srazmerne kvadratima duzi r1 i r2
te prema prethodnom zadatku krug k pripada pramenu kojeg odreduju krugovi
k1 i k2 .
605.Dokazati da krugovi kojima su dijametri dijagonale bilo kojeg cetvorostranika,
pripadaju izvesnom pramenu krugova (Teorema K.F. Gausa i Bodenmilera).
398

Resenje: Obelezimo sa abcd bilo koji cetvorostranik, sa A, B, C, D, E, F


tacke u kojima se seku prave d, a; a, b; b, c; c, d; a, c; d, b a sa k1 , k2 , k3 krugove kojima su dijagonale AC, BD, EF precnici. Ako su AA , BB , F F visine
trougla ABF i sa H1 obelezimo ortocentar ovog trougla, tada su tacke A , B , F
na krugovima k1 , k2 , k3 i H1 AH1 A = H1 B H1 B = H1 F H1 F pa su potencije
tacke H1 u odnosu na krugove k1 , k2 , k3 medusobom jednake. Istim postupkom
dokazuje se da ortocentar H2 trougla ADE ima jednake potencije u odnosu na
krugove k1 , k2 , k3 . Stoga krugovi k1 , k2 , k3 imaju za radikalnu osu istu pravu
H1 H2 , prema tome oni pripadaju izvesnom pramenu krugova (sl.362).
F

D
c
C
d
b

Slika 362
606. Dokazati da sredista dijagonala cetvorostranika pripadaju jednoj pravoj, Gausovoj pravoj cetvorostranika.
Resenje: Prema prethodnom zadatku, krugovi koji su dijametri dijagonale
nekog cetvorostranika pripadaju jednom pramenu, prema tome, sredista tih
krugova, tj. sredista dijagonala tog cetvorostranika pripadaju jednoj pravoj,
koju nazivamo Gausovom pravom tog cetvorostranika.
607.Dokazati da ortocentri cetiri trougla, koji su odredeni stranicama nekog
cetvorostranika, pripadaju jednoj pravoj Oberovoj pravoj tog cetvorostranika.
Resenje: Obelezimo sa abcd bilo koji etvorostranik a sa A, B, C, D, E, F njihova temena, tj. presecne tacke pravih d, a; a, b; b, c; c, d; a, c; d, b. Prema
prethodnom zadatku, ortocentri trouglova ADE i ABF odreduju radikalnu osu
pramena kome pripadaju krugovi kojima su precnici dijagonale AC, BD, EF .
Na isti nacin dokazuje se da radikalnu osu tog pramena odreduju i ortocentri
trouglova BCE, CDF . Stoga ortocentri cetiri trougla koji su odredeni stranicama cetvorostranika abcd pripadaju jednoj pravoj, koju nazivamo Oberovom
pravom tog cetvorostranika.
608.Dokazati da su Gausova prava i Oberova prava istog cetvorostranika
upravne medusobom.
Resenje: Prema zadatku 605, krugovi kojima su precnici dijagonale nekog cetvorostranika pripadaju izvesnom pramenu krugova kome su centralna
prava i radikalna osa respektivna. Stoga su Gausova prava i Oberova prava tog
cetvorostranika (v. z. 606 i 607) medusobno upravne prave.
609. Dokazati da se srediste kruga opisanog oko dijagonalnog trougla bilo
kojeg cetvorostranika nalazi na Oberovoj pravoj tog cetvorostranika.
399

Resenje: Prema zadatku 605, krugovi l1 , l2 , l3 kojima su precnici dijagonale nekog cetvorostranika abcd pripadaju izvesnom pramenu l krugova kome je
Oberova prava radikalna osa. Kako je, prema zadatku 475, opisani krug k oko
dijagonalnog trougla tog cetvorostranika, ortogonalan na krugovima l1 , l2 , l3 ,
srediste S kruga k je na radikalnoj osi pramena l, tj. na Oberovoj pravoj
cetvorostranika abcd.
610. Ako su P , Q, R tacke u kojima neka prava sece prave odredene stranicama BC, CA, AB trougla ABC, dokazati da krugovi k1 , k2 , k3 kojima su
duzi AP , BQ, CR precnici, pripadaju izvesnom pramenu krugova.
Resenje: Prave BC, CR, RQ, QB odreduju cetvorostranik kome su duzi
AP, BQ, CR dijagonale, te prema zadatku 605, krugovi k1 , k2 , k3 kojima su
duzi AP, BQ, CR precnici pripadaju jednom trapezu.
611.Ako su dva para suprotnih temena cetvorostranika spregnuta u odnosu
na neki krug, dokazati da je i tre`ci par suprotnih temena spregnut u odnosu na
taj krug.
Resenje: Neka su A i B, C i D, E i F parovi suprotnih temena nekog
cetvorostranika. Prema zadatku 605, krugovi k1 , k2 , k3 kojima su dijagonale
AC, BD, EF precnici pripadaju jednom pramenu k. Ako su parovi tacaka
A, C i B, D spregnuti u odnosu na neki krug l, krugovi k1 , k2 su ortogonalni na
krugu l, pa je svaki krug pramena k, dakle i krug k3 ortogonalan na krugu l.
Otuda sleduje da je par dijametralno suprotnih tacaka E, F kruga k3 spregnut
u odnosu na krug k.
612. Ako je ABCD tetivan cetvorougao kome su produzene naspramne
stranice AB i CD seku u tacki E, a produzene naspramne stranice BC i AD
seku u tacki F , dokazati da je krug k3 kome je precnik EF krug slicnosti krugova
k1 i k2 kojima su precnici AC i BD.
Resenje: Sredista O1 i O2 krugova k1 i k2 su sredista dijagonala AC i BD, pa
su duzi F O1 i F O2 tezisne linije slicnih trouglova F AC i F BD. Otuda je
F O1 : F O2 = AO1 : BO2 ,
pa je
F O12 : F O22 = AO12 : BO22 ,
i prema tome je
(F O12 AO12 ) : (F O22 BO22 ) = AO12 : BO22 .
Prva dva clana u ovoj proporciji su jednaka potencijama tacke F u odnosu na
krugove k1 i k2 , pa je tacka F na krugu slicnosti krugova k1 i k2 . S obzirom
da krug k3 , koji sadrzi tacku F trapeza, pripada pramenu, koji je odreden
krugovima k1 i k2 prema zadatku 603., potencije svake tacke P kruga k3 u
odnosu na krugove k1 i k2 srazmerne su kvadratima poluprecnika krugova k1 i
k2 , pa je k3 krug slicnosti krugova k1 i k2 .
613. Dokazati da krugovi slicnosti triju datih krugova k1 , k2 , k3 pripadaju
izvesnom pramenu krugova.
Resenje: Obelezimo sa l krug ortogonalan na krugove k1 , k2 , k3 , sa k krug
odreden sredistima krugova k1 , k2 , k3 i sa l1 , l2 , l3 krugove slicnosti parova
krugova k2 i k3 , k3 i k1 , k1 i k2 . Prema zadatku 604., krug l1 pripada pramenu,
koji je odreden krugovima k2 i k3 . Krug l je ortogonalan na krugove k2 i k3 , pa
je ortogonalan na sve krugove njihovog pramena, dakle i na krug l1 . Isto tako,
400

krug l je ortogonalan na krugove l2 i l3 . Sredista O2 i O3 krugova k2 i k3 su


tacke harmonijski spregnute sa tackama M i N u kojima prava O2 O3 sece krug
l1 , pa je, prema zadatku 475., krug k, koji sadrzi tacke O2 i O3 , ortogonalan na
krug l1 . Isto tako, krug k je ortogonalan na krugove l2 i l3 . S obzirom da su
krugovi l1 , l2 , l3 ortogonalni na dvama krugovima k i l, oni pripadaju izvesnom
pramenu krugova.
614. Ako je l krug ortogonalan na krugovima k1 , k2 , k3 i k krug odredjen
sredistima krugova k1 , k2 , k3 , dokazati da se sredista krugova slicnosti datih
krugova k1 , k2 , k3 nalaze na radikalnoj osi krugova k i l.
Resenje: Prema prethodnom zadatku, krugovi slicnosti datih krugova k1 , k2 ,
k3 ortogonalni su na krugove k i l, pa im se sredista nalaze na radikalnoj osi
krugova k i l.
615. Ako su T i H teziste i ortocentar trougla ABC, dokazati da opisani
krug l tog trougla, Ojlerov krug l1 tog trougla i krug l2 kome je duz T H precnik,
pripadaju hiperbolickom, parabolickom ili eliptickom pramenu krugova, prema
tome da li je trougao ABC ostrougli, pravougli ili tupougli.
Resenje: Tacke T i H su, prema zadatku 529., sredista slicnosti krugova l i l1 ,
pa je l2 krug slicnosti krugova l i l1 . S toga, prema zadatku 604., krugovi l,
l1 , l2 pripadaju izvesnom pramenu krugova. Ako je trougao ABC ostrougli,
ortocentar H, tj. spoljasnje srediste slicnosti krugova l i l1 , u krugu l je. Prema
tome, krugovi l i l1 nemaju zajednickih tacaka, pa je pramen hiperbolicki. Ako
je trougao ABC pravougli, krugovi l, l1 i l2 se dodiruju u temenu pravog ugla,
pa je pramen parabolicki. Ako je trougao ABC tupougli, krug l2 sadrzi tacku
H, koja je izvan kruga l, i tacku T , koja je u krugu l, pa se krugovi l i l2 seku,
pa je pramen elipticki.
616. Ako proizvoljna prava sece dva kruga k1 i k2 u tackama A, B i C, D
a ne sadrzi nijedno srediste slicnosti tih krugova, i ako su a, b i c, d tangente u
tackama A, B i C, D na krugovima k1 i k2 , dokazati da tacke u kojima prave
a i b seku prave c i d pripadaju jednom krugu, a zatim da taj krug pripada
pramenu koji je odredjen krugovima k1 i k2 .
Resenje: Neka su P i Q tacke u kojima prava a sece prave c i d, a R i S tacke u
kojima prava b sece prave c i d. Kod trouglova P AC, QAD, RBC, SBD imamo
da je
P A : P C = QA : QD = RB : RC = SB : SD, tj.
P A2 : P C 2 = QA2 : QD2 = RB 2 : RC 2 = SB 2 : SD2 .
Otuda sleduje da su potencije svake od tacaka P , Q, R, S srazmerne istim
duzima, pa se, prema tome, one nalaze na jednom krugu, koji pripada pramenu,
kojeg odreduju krugovi k1 i k2 .
617. Ako su AA , BB , CC visine trougla ABC, a P proizvoljna tacka njegove ravni, dokazati da krugovi k1 , k2 , k3 opisani oko trouglova P AA , P BB , P CC
pripadaju izvesnom pramen krugova.
Resenje: Neka je H ortocentar trougla ABC i Q druga presecna tacka prave
P H sa krugom k1 . Pri tome je
HP : HQ = HA : HA = HB : HB = HC : HC .
Otuda sleduje da je tacka Q i na krugovima k2 , k3 . S toga se krugovi k1 , k2 , k3
seku u dvema tackama P i Q, pa, prema tome, pripadaju izvesnom eliptickom
pramenu krugova.
401

618. Dve prave l i l kroz zajednicku tacku P svih krugova eliptickog pramena
seku tri kruga tog pramena u tackama A, B, C i A , B , C . Dokazati da je:
AB : BC = A B : B C .
Resenje: Ako obelezimo sa Q drugu zajednicku tacku svih krugova eliptickog
pramena, bice QAB QA B i QBC QB C , pa je
AB : A B = QB : QB i BC : B C = QB : QB .
Otuda je AB : A B = BC : B C , i prema tome AB : BC = A B : B C .
619. Ako su O1 , O2 , O3 sredista i r1 , r2 , r3 poluprecnici triju krugova istog
pramena, dokazati da je:
r12 O2 O3 + r22 O3 O1 + r32 O1 O2 + O1 O2 O2 O3 O3 O1 = 0.
Resenje: Neka je P bilo koja tacka radikalne ose datog pramena, koja se pretpostavimo nalazi izvan tih krugova. Prema Stjuartovoj teoremi, imamo da je
P O12 O2 O3 + P O22 O3 O1 + P O32 O1 O2 + O1 O2 O2 O3 O3 O1 = 0.
Ako sa d obelezimo odsecak na tangenti kroz tacku P na bilo kojem krugu tog
pramena, bice P O12 = d2 + r12 , P O22 = d2 + r22 , P O32 = d2 + r32 , pa je
r12 O2 O3 + r22 O3 O1 + r32 O1 O2 + O1 O2 O2 O3 O3 O1 = 0.
620. Ako su O1 , O2 , O3 sredista krugova k1 , k2 , k3 nekog pramena, a d1 ,
d2 , d3 odsecci na dirkama krugova k1 , k2 , k3 kroz proizvoljnu tacku P koja se
nalazi izvan njih, dokazati da je:
d21 O2 O3 + d22 O3 O1 + d23 O1 O2 = 0.
Resenje: Prema Stjuartovoj teoremi, imamo da je
P O12 O2 O3 + P O22 O3 O1 + P O32 O1 O2 + O1 O2 O2 O3 O3 O1 = 0.
Ako sa r1 , r2 , r3 obelezimo poluprecnike krugova k1 , k2 , k3 , bice P O12 = d21 + r12 ,
P O22 = d22 + r22 , P O32 = d23 + r32 , pa se, primenom prethodnog zadatka, dobija
da je
d21 O2 O3 + d22 O3 O1 + d23 O1 O2 = 0.

7
7.1

INVERZIJA
Inverzne ta
cke

Definicija 7.1. Ako je P proizvoljna tacka ravni nekog kruga k(O, r) i P


tacka poluprave OP takva da je OP OP = r2 , tada kazemo da je tacka P
402

inverzna sa tackom P u odnosu na krug k. Iz definicije neposredno sledi da je


i tacka P inverzna sa tackom P u odnosu na isti krug k. Stoga se takodje kaze
da su tacke P i P medjusobno inverzne u odnosu na krug k. Krug k nazivom
krugom inverzije, tacku O sredistem inverzije, a duz r poluprecnikom inverzije.
621. Dokazati da je svaki krug koji sadrzi par tacaka P , P inverznih u
odnosu na neki krug k, ortogonalan na krugu k.

T
k

O P

Slika 363
Resenje: Prava P P sadrzi srediste kruga k, prema tome, sece krug u dijametralno suprotnim tackama, recimo A i B. Pri tome su tacke P i P harmonijski
spregnute s dijametralno suprotnim tackama A i B kruga k, pa je prema zadatku 386, svaki krug koji sadrzi tacke P i P ortogonalan na krugu k (slika
363).
622. Ako su l i k dva ortogonalna kruga, dokazati da su tacke u kojima
proizvoljna prava kroz srediste kruga k sece krug l, inverzne u odnosu na krug
k.
Resenje: Neka su O i r srediste i poluprecnik kruga k, a P i P tacke u kojima
proizvoljna prava kroz O sece krug l. S obzirom da su krugovi l i k ortogonalni,
oni se seku; neka je T bilo koja od presecnih tacaka. Pri tome je duz OT odsecak
kruga k, pa je OP OP = OT 2 = r2 . Stoga su tacke P i P inverzne u odnosu
na krug k.
623. Dokazati da je par tacaka P , P inverznih u odnosu na krug k harmonijski spregnut s parom tacaka u kojima prava P P sece krug k.

403

O P

Slika 364
Resenje: Prava odredena inverznim tackama P i P u odnosu na krug k, sadrzi
srediste O tog kruga, pa prema tome, sece k u dijametralno suprotnim tackama
A i B. Pri tome je OP OP = OB 2 , pa su prema zadatku 402, tacke P i P
harmonijski spregnute sa tackama A i B(slika 364).
624. Ako su A i B dijametralno suprotne tacke kruga k, a P i P tacke
harmonijski spregnute sa tackama A i B, dokazati da su tacke P i P inverzne
u odnosu na krug k.
Resenje: Srediste O kruga k je srediste duzi AB, pa je prema zadatku 402,
OP OP = OB 2 . Odatle sledi da su tacke P i P inverzne u odnosu na krug k.
625. Ako je A, A par inverznih tacaka u odnosu na krug k kome je srediste
O, a M proizvoljna tacka kruga k, dokazati da je AOM M OA .

M
k

Slika 365
Resenje: S obzirom da je par tacaka A, A inverzan u odnosu na krug k, imamo
da je OA OA = OM 2 , pa je OA : OM = OM : OA . Sem toga je AOM =
M OA , pa je AOM M OA (slika 365).
626. Ako P, P i Q, Q dva para nekolinearnih tacaka, inverznih u odnosu na
isti krug k sa sredistem O, dokazati da su trouglovi OP Q i OQ P inverzno
slicni.

404

k
P
O Q

Slika 366
Resenje: S obzirom da su P , P i Q, Q parovi inverznih tacaka u odnosu na
krug k, bice OP OP = r2 i OQOQ = r2 , gde je r poluprecnik kruga k. Otuda
je OP OP = OQ OQ , i prema tome OP : OQ = OQ : OP . Sem toga, tacke
P i P su kolinearne s tackom O i nalaze se s iste strane od O. Takode tacke
Q i Q su kolinearne s tackom O i nalaze se sa iste strane od O. Iz ovoga sledi
da su uglovi P OQ i Q OP jednaki i suprotno usmereni. Odatle sledi da su
trouglovi OP Q i OQ P inverzno slicni(slika 366).
627. Dokazati da svaka dva para nekolinearnih tacaka P, P i Q, Q inverznih
u odnosu na isti krug k pripadaju jednom krugu l koji je ortogonalan na krugu
k.
Resenje: Ako obelezimo sa O srediste kruga k, prema zadatku 626, trouglovi
OP Q i OQ P slicni, pa su uglovi OP Q i OQ P jednaki. Odatle sledi
da tacke P , P , Q, Q pripadaju jednom krugu l. S obzirom da taj krug sadrzi
par tacaka P , P inverznih u odnosu na krug k, prema zadatku 621, krug l je
ortogonalan na krug k.
628. Ako su P i P inverzne tacke u odnosu na krug k(O, r), a Q proizvoljna
tacka kruga k, dokazati da je
OP : OP = P Q2 : P Q2 .

405

M
k

O P

Slika 367
Resenje: Prema zadatku 623, tacke P i P su harmonijski spregnute sa tackama
A i B, koje su tacke preseka prave P P i kruga k. Zbog toga je prema zadatku
413 OP : OP = P B 2 : P B 2 , a odatle sledi da je
OP : OP = P Q2 : P Q2 .
629. Prava odredena inverznim tackama P i P u odnosu na krug k, sadrzi
srediste O tog kruga. Iz toga sledi da ta prava sece krug k u dijametralno
suprotnim tackama, nazovimo ih A i B. Tacke A i B su harmonijski spregnute
sa tackama P i P , pa su prave M A i M B harmonijski spregnute sa pravama M P
i M P . Osim toga ugao AM B je prav, pa su prave M A i M B simetrale uglova
kod temena M trougla M P P . Odatle sledi da je M P : M P = P A : P A i
M P : M P = P B : P B(slika 367).
629. Ako je P, P par inverznih tacaka u odnosu na krug k, dokazati da su
duzi koje spajaju proizvoljnu tacku M toga kruga s tackama P i P srazmerne
odseccima na koje krug k deli duz P P .

M
k

O P

Slika 367
Resenje: Prava odredena inverznim tackama P i P u odnosu na krug k, sadrzi
srediste O tog kruga. Iz toga sledi da ta prava sece krug k u dijametralno suprotnim tackama, nazovimo ih A i B. Tacke A i B su harmonijski spregnute sa
tackama P i P , pa su prave M A i M B harmonijski spregnute sa pravama M P i
M P . Osim toga ugao AM B je prav, pa su prave M A i M B simetrale uglova
406

kod temena M trougla M P P . Odatle sledi da je M P : M P = P A : P A i


M P : M P = P B : P B(slika 367).
630. Dokazati da simetrala stranice BC trougla ABC sece prave AB i AC
u tackama P i Q inverznim u odnosu na opisani krug tog trougla.

M
A
k

O P

Slika 368
Resenje: Simetrala stranice BC sece opisani krug trougla ABC u dvema
tackama M i N takvim da su prava AM i AN simetrale unutrasnjeg i spoljasnjeg
ugla kod temena A trougla ABC. Zato su prave AM i AN harmonijski
spregnute s pravama AB i AC, pa su i tacke M i N harmonijski spregnute s
tackama P i Q. Srediste O duzi M N je srediste kruga opisanog oko trougla
ABC, pa je OP OQ = OM 2 . Odatle sledi da su tacke P i Q inverzne
u odnosu na opisani krug trougla ABC. 631. Simetrala tetive AB sadrzi
srediste O kruga k, prema tome, ona sece krug k u dijametralno suprotnim
tackama C i D. Pri tome su tacke C i D sredista lukova AB kruga k, pa su
prave M C i M D simetrale uglova kod temena M trougla M AB, i prema tome,
harmonijski spregnute sa pravama M A i M B. Zato su tacke C i D harmonijski
spregnute s tackama P i P , pa vazi OP OP = OC 2 , a odatle sledi da je par
tacaka P, P inverzan u odnosu na krug k(slika 368).
631. Ako je AB proizvoljna tetiva i M proizvoljna tacka kruga k, dokazati
da prave M A i M B seku simetralu duzi AB u dvema inverznim tackama.
632. Ako su P , P par inverznih tacaka u odnosu na krug k i M proizvoljna
tacka tog kruga, dokazati da prave M P i M P seku krug k u tackama A i B
takvim da je tetiva AB upravna na pravoj P P .

407

M
k

Slika uz zadatak 632.


Resenje: S obzirom da su tacke P i P inverzne u odnosu na krug k, prava P P
sadrzi srediste O toga kruga, i prema tome sece ga u dijametralno suprotnim
tackama C i D. Pri tome su tacke C i D harmonijski spregnute s tackama P
i P , pa su i prave M C i M D harmonijski spregnute s pravama M P i M P .
Sem toga, ugao CM D je prav, pa su M C i M D simetrale uglova kod temena
M trougla M AB. Stoga su tacke C i D sredista lukova AB kruga k, pa je
AB CD, tj. AB P P .

7.2

Inverzni likovi

Definicija 7.2. Ako je proizvoljan lik koji se nalazi u ravni nekog kruga,
lik koji se sastoji iz tacaka inverznih s tackama lika u odnosu na krug k,
tada kazemo da je lik inverzan s likom u odnosu na krug k.
Iz ove definicije neposredno sleduje da je i lik inverzan s likom u odnosu
na krug k, pa se kaze da su likovi i inverzni medju sobom u odnosu na krug
k.
633. Ako su M i M inverzne tacke u odnosu na krug k koji je opisan oko
trougla ABC, a P , Q, R i P , Q , R podnozja upravnih iz tacaka M i M na
pravama BC, CA, AB, dokazati da su trouglovi P QR i P Q R slicni.

408

M
O

Slika 369
Resenje: Neka su S i S sredista duzi AM i AM , a l i l krugovi kojima su duzi
AM i AM precnici. S obzirom da su uglovi ARM , AQM i AR M , AQ M
pravi, tacke R i Q su na krugu l, a tacke R i Q na krugu l . Iz jednakosti uglova
RM Q i R M Q sledi da su uglovi RSQ i R S Q jednaki, pa su jednakokraki
trouglovi RSQ i R S Q slicni. Otuda je QR : Q R = QS : Q S = AM : AM .
Isto tako je RP : R P = BM : BM i P Q : P Q = CM : CM . Prema
zadatku 625, imamo da je AM : AM = BM : BM = CM : CM , pa je
QR : Q R = RP : R P = P Q : P Q , i prema tome, P QR P Q R
(sl. 369).
634. Ako su M i M dve tacke u ravni trougla ABC, a P , Q, R i P , Q ,
R , podnozja upravnih iz tacaka M i M na pravama BC, CA, AB pri cemu su
trouglovi P QR i P Q R slicni, dokazati da su tacke M i M inverzne u odnosu
na krug k koji je opisan oko trougla ABC.

R
M
O

Slika uz zadatak 634.


Resenje: Kao u prethodnom zadatku imamo da je QR : Q R = AM : AM ,
RP : R P = BM : BM , P Q : P Q = CM : CM . Iz slicnosti trouglova
409

P QR i P Q R , sledi da je QR : Q R = RP : R P = P Q : P Q , pa je i
AM : AM = BM : BM = CM : CM . Otud sleduje da su tacke M i M
inverzne u odnosu na krug k.
635. Dokazati da su granicne tacke hiperbolickog pramena krugova inverzne
u odnosu na svaki krug tog pramena krugova.
Resenje: Obelezimo sa C1 i C2 granicne tacke hiperbolickog pramena krugova,
a sa A i B tacke u kojima prava C1 C2 sece proizvoljni krug k(O, r) tog pramena
krugova. Prema zadatku 588 tacke A i B su harmonijski spregnute sa tackama
C1 i C2 , pa je OC1 OC2 = OA2 = r2 . Stoga su tacke C1 i C2 inverzne medu
sobom u odnosu na krug k.
636. Ako su dve tacke P i P inverzne medju sobom u odnosu na krugove
k1 , . . . , kn , dokazati da pomenuti krugovi pripadaju jednom pramenu.
Resenje: Prema definiciji inverznih tacaka, sredista krugova k1 , . . . ,kn su na
pravoj P P . Ako obelezimo sa O srediste duzi P P , a sa A i B tacke u kojima
prava P P sece krug ki (i = 1, . . . , n), imamo da je OA OB = OP 2 . S toga
tacka O ima jednake potencije u odnosu na krugove k1 , . . . ,kn , pa je prava s
upravna u tacki O na pravoj P P zajednicka radikalna osa svih tih krugova.
Otuda sledi da krugovi k1 , . . . ,kn pripadaju jednom pramenu.
637. Ako su A i B dve proizvoljne tacke u ravni kruga k(O, r), a A i B
njima inverzne tacke u odnosu na krug k, dokazati da je
A B = AB

r2
.
OA OB

A
O

B
B

Slika uz zadatak 637.


Resenje: Iz jednakosti uglova AOB i B OA , i jednakosti OA OA = OB OB ,
tj. OA : OB = OB : OA sledi da je AOB B OA . Stoga je
OB
r2
AB : A B = OA : OB , i prema tome A B = AB
= AB
.
OA
OA OB

638. Ako su A, B, C, D cetiri tacke neke prave p i A , B C , D njima inverzne tacke u odnosu na neki krug kome se srediste nalazi na pravoj p, dokazati
da je
R(A, B; C, D) = R(A , B ; C , D ).
410

Resenje: Ako obelezimo sa O i r srediste i poluprecnik kruga inverzije, prema


zadatku 637 imamo da je
A C = AC
A D = AD
Otuda je

r2
r2
, B C = BC
,
OA OC
OB OC

r2
r2
, B D = BD
.
OA OD
OB OD

AC OB
A D
AD OB
A C
=

,i =

BC
BC OA
BD
BD OA
i prema tome

A C A D
AC AD
:
=
:
,
B C B D
BC BD
tj. R(A, B; C, D) = R(A , B ; C , D ).
639. Ako su O, A, B, C tacke neke prave p, pri cemu je B srediste duzi AC,
a A , B , C tacke inverzne s tackama A, B, C u odnosu na bilo koji krug sa
sredistem O, dokazati da su tacke H(O, A , B , C ) harmonijske.
Resenje: Ako obelezimo sa r poluprecnik inverzije, bice OA OA = r2 ,
OC OC = r2 . S obzirom da je tacka B srediste duzi AC, imamo da je
1
2
1
+
=
, i prema tome H(O, A ; B , C ).
OA + OC = 2OB, pa je
OA
OC
OB
640. Neka su k1 , . . . , kn krugovi nekog pramena krugova. Ako je A proizvoljna tacka van sredisnje prave tog pramena, A1 tacka inverzna sa tackom A
u odnosu na krug k1 , A2 tacka inverzna sa tackom A1 u odnosu na krug k2 , itd,
dokazati da tacke A, A1 , . . . , An pripadaju jednom krugu koji je ortogonalan na
svim krugovima pomenutog pramena krugova.

A1
A3
A2
O1

O2

O3

k2
k1

k3

Slika 370
411

Resenje: S obzirom da su tacke A i A1 inverzne u odnosu na krug k1 i tacke A1 i


A2 inverzne u odnosu na krug k2 , krug l odreden tackama A, A1 , A2 ortogonalan je na krugovima k1 i k2 , dakle i na krugovima k3 , . . . , kn . Iz ortogonalnosti
l i k3 sleduje da prava kroz srediste kruga k3 sece krug l u tackama inverznim
u odnosu na krug k3 , pa je i tacka A3 na krugu l. Istim postupkom dokazuje se
da i tacke A4 , . . . , An pripadaju krugu l koji je ortogonalan na svim krugovima
pomenutog pramena krugova (sl. 370).
641. Dokazati da je inverzan lik prave l u odnosu na krug k takodje prava
ili krug, prema tome da li prava l sadrzi ili ne sadrzi srediste kruga k.
l
P
k

Slika 371
Resenje: Analizirajmo najpre slucaj kada prava l sadrzi srediste O kruga k.
Tada svakoj tacki prave l odgovara tacka koja je takode na pravoj, i obrnuto,
svaka tacka prave l je inverzna s nekom tackom te iste prave, pa je inverzan lik
prave l u odnosu na krug k ta ista prava.
Sad pretpostavimo da prava l ne sadrzi srediste O kruga k (v.sl. 371). Neka
je A podnozje upravne kroz tacku O na pravoj l i A njena inverzna tacka u
odnosu na krug k; zatim P proizvoljna tacka prave l i P njena inverzna tacka
u odnosu na isti krug k. Prema tome je OA OA = OP OP , pa je
OA : OP = OP : OA . Sem toga je AOP = P OA, pa su trouglovi OAP i
OP A slicni, te je OAP = OP A . No OAP je prav, pa je i njemu jednak
OP A takode prav. Stoga je tacka P na krugu l kome je duz OA precnik.
Obrnuto, ako je Q bilo koja tacka l , a Q tacka u kojoj prava OQ sece pravu
l, bice OAQ = OQ A i AOQ = Q OA , pa su trouglovi OAQ i OQ A
slicni. Otuda je OA : OQ = OQ : OA , tj. OA OA = OQ OQ , pa je tacka
Q inverzna s tackom Q u odnosu na krug k. Prema tome, inverzni lik prave l
koja ne sadrzi srediste inverzije je krug l kome je duz OA precnik.
643. Ako je krug l ortogonalan na krug k, dokazati da je krug l inverzan
samom sebi u odnosu na krug k, i obrnuto, ako je krug l inverzan samom sebi
u odnosu na krug k, dokazati da su krugovi l i k medu sobom ortogonalni.
Resenje: Ako su O i r srediste i poluprecnik kruga k, a P i P tacke u kojima
proizvoljna prava kroz tacku O sece krug l, bice OP OP = r2 , pa su tacke P i
412

P inverzne u odnosu na krug k. Stoga svakoj tacki P krug l odgovara na tom


istom krugu l tacka P inverzna je u odnosu na krug k, pa je krug l inverzan
samom sebi u odnosu na krug k. Ovim je dokazan prvi deo zadatka ( sl.374).

M
P
P
O

Sl. 374
Da bismo dokazali drugi deo, obelezimo sa O i r srediste i poluprecnik kruga
k, a sa P i P tacke u kojima neka prava kroz tacku O sece krug l. S obzirom
da je krug l inverzan samom sebi u odnosu na krug k, bice OP OP = r2 . Pri
tome krug l sadrzi tacke P i P inverzne u odnosu na krug k, pa sledi da su
krugovi l i k ortogonalni.
644. Ako su krug l i prava p inverzni likovi u odnosu na krug k, dokazati da
je prava p radikalna osa krugova l i k.
Resenje: S obzirom da je prava p inverzan lik kruga l u odnosu na krug k,
srediste O kruga k je na krugu l. Neka je OA precnik kruga l, a A tacka inverzna
s tackom A u odnosu na krug k. Pri tome je OA AA = OA (OA OA) =
OA2 OA OA = OA2 r2 , gde je r poluprecnik kruga k. Stoga su potencije
tacke A u odnosu na krugove l i k jednake, pa je tacka A na radikalnoj osi
krugova l i k, i prema tome prava p radikalna osa krugova li k ( sl.375).

l
k

Sl. 375
645. Ako su l i l inverzni krugovi u odnosu na krug k, dokazati da krugovi
k, l, l pripadaju jednom pramenu.
Resenje: Ako se krugovi k i l seku, i krug l sadrzi presecne tacke, prema
tome, krugovi k, l, l pripadaju emptickom pramenu. Ako se krugovi k i l
dodiruju, u istoj tacki i krug l dodiruje krugove k i l, pa su k, l, l krugovi egarabolickog pramena. Ako krugovi k i l nemaju zajednickih tacaka, oni odreduju
izvestan hiperbolicki pramen krugova, neka su C1 i C2 granicne tacke tog pramena. Tacke C1 i C2 su inverzni medu sobom u odnosu na krugove k i l, pa

413

su inverzni u odnosu na krug l . Stoga krug l pripada pramenu koji je odreden


krugovima k i l ( sl.376).

C2

l
k

Sl. 376
646. Dokazati da opisani krug l1 , Ojlerov krug l2 , polarni krug l3 trougla
ABC i krug l4 opisan oko njegovog potencijalnog trougla A1 B1 C1 pripadaju
jednom pramenu.
Resenje: Ako su AA , BB , CC visine trougla ABC, bice tacke A i A ,
BiB , C i C inverzne u odnosu na krug l3 , pa su krug l1 kome pripadaju tacke
A, B, i C i krug l2 kome pripadaju tacke A , B , C inverzni medu sobom u
odnosu na krug l3 . Stoga, prema prethodnom zadatku, krug l3 pripada pramenu
krugova koji je odreden krugovima l1 i l2 . Temena A1 , B1 , C1 tangencijalnog
trougla su polovi pravih BC, CA, AB u odnosu na krug l1 . Ako obelezimo sa
A2 , B2 , C2 sredista stranica BC, CA, AB, bice tacke A1 i A2 , B1 i B2 , C1 i
C2 inverzne u odnosu na krug l1 , pa su krug l2 kome pripadaju tacke A2 , B2 ,
C2 i krugovi kome pripadaju tacke A1 , B1 , C1 inverzni medu sobom u odnosu
na krug l1 . Stoga prema prethodnom zadatku, krug l2 , pripada pramenu koji
je odreden krugovima l1 i l2 .
647. Dokazati da se srediste kruga opisanog oko potencijalnog trougla nalazi
na Ojlerovoj pravoj datog trougla.
Resenje: Prema prethodnom zadatku opisani krug l1 trougla ABC, Ojlerov
krug l2 trougla ABC i krug l4 opisan oko njegovog potencijalnog trougla pripadaju jednom pramenu, pa im se sredista nalaze na jednoj pravoj. Kako su
sredista krugova l1 i l2 na Ojlerovoj pravoj trougla ABC, i srediste kruga l4 je
na Ojlerovoj pravoj trougla ABC.
648. Ako krugovi k1 , k2 , k3 , k4 imaju takav polozaj u ravni da krug k1
dodiruje krug k2 u tacki A, krug k2 dodiruje krug k3 u tacki B, krug k3 dodiruje
krug k4 u tacki C, krug k4 dodiruje krug k1 u tacki D. Dokazati da dodirne tacke
A, B, C, D pripadaju jednom krugu ili jednoj pravoj.
Resenje: Obelezimo sa k proizvoljan krug kome je srediste tacka A, a sa k1 ,

k2 , k3 , k4 linije inverzne s linijama k1 , k2 , k3 , k4 , u odnosu na krug k. S obzirom


da se krugovi k1 i k2 dodiruju medu sobom u sredistu inverzije, linije k1 i k2 su
uporedne prave. Krugovi k3 i k4 ne sadrze srediste inverzije, te su inverzne linije
k3 i k4 krugovi. Pri tome, prava k1 dodiruje krug k2 u tacki B koja je inverzna
s tackom B, krug k2 dodiruje krug k3 u tacki C koja je inverzna s tackom C,
krug k3 dodiruje pravu k4 u tacki D koja je inverzna s tackom D. Dokazimo da
tacke B , C , D pripadaju jednoj pravoj. Ako su O3 i O4 sredista krugova k3 i
k4 , bice O3 B C O4 D C , pa je O3 C B = O4 C D . No iako su tacke
414

O3 ,O4 ,C na jednoj pravoj, a uglovi O3 C B i O4 C D jednaki i tacke B ,C ,D


pripadaju izvesnoj pravoj l . Inverzan lik l s tom pravom je krug ili prava; u
oba slucaja taj lik sadrzi tacke A, B, C, D (sl. 377).
k2

k4
k2

O4

B
k1

k4

k3

O3

k3

k1

slika 377
649. Ako su w i w dva lika inverzna medu sobom u odnosu na neki krug l,
a w1 i w1 njima inverzni likovi u odnosu na neki krug l1 , dokazati da su i likovi
w1 i w1 inverzni medu sobom u odnosu na neki krug l2 .
Resenje: Neka su O i S sredista krugova l i l1 , M i M bilo koje dve odgovarajuce tacke likova i , a M1 i M1 njima odgovarajuce tacke na likovima 1
i 1 . S obzirom na krug k odredene s tackama S, M, M sadrzi tacke M i M
inverzne medu sobom u odnosu na krug l, krug k je ortogonalan na krugu l, te
tri inverzije u odnosu na krug l odgovara samom sebi. Otuda sleduje da i tacka
S inverzna s tackom S u odnosu na krug l pripada krugu k. Posto se tacke
S, M, M nalaze na jednom krugu koji sadrzi srediste S kruga l1 , njima inverzne
tacke S1 , M1 , M1 u odnosu na krug l1 pripadaju jednoj pravoj. Stoga su sve
prave kroz odgovarajuce tacke likova 1 i 1 seku u jednoj tacki S1 . Dokazimo
sad da je proizvod drugi S1 M1 i S1 M1 stalan. Ako sa i 1 obelezimo koeficijente inverzija u odnosu na krugove l i l1 , prema poznatom stavu imamo da
je
SM
SM
S M

S M =
,
S
M
=

, S1 M1 =
,
S
M
=

1
1
1

OS

OM
OS

OM
SM

SS
S M
,
1
SM SS
zamenjujuci odsecke S M i S M iz prve dve jednakosti u druge dve, zatim
mnozeci dobijene jednakosti, imamo da je
21
2 21
=
,
S1 M1 S1 M1 =

2
2
2
OM OM OS SS
OS SS 2
pa je proizvod pomenutih duzi S1 M1 i S1 M1 stalan. Otuda sleduje da su
tacke M1 i M1 , prema tome, i likovi 1 i 1 inverzni medu sobom u odnosu na
izvestan krug L2 kome je srediste tacka s1 .
650. Ako krug odreden bilo kojim trima tackama nekog konacnog skupa
tacaka sadrzi najmanje jos jednu tacku tog skupa, dokazati da sve tacke tog
skupa pripadaju jednom krugu.
Resenje: Neka je A1 , ...An konacan skup tacaka takvih da krug odreden bilo
kojim trima tackama tog skupa sadrzi bar jos jednu tacku tog skupa. Ako
415

obelezimo sa l proizvoljan krug kome je srediste jedna od tacaka tog skupa,


npr. tacka A1 , a sa A2 , ...., An tacke inverzne s tackama A2 , ...., An u odnosu
na krug l, bice krugovima koji su odredeni tackom A1 i jos dvema tackama
skupa A1 , ...An inverzne prave odredene dvama tackama skupa A2 , ...An . Po
pretpostavci svaki od tih krugova sadrzi, pored tacaka kojima je on odreden,
bar jos jednu tacku skupa A1 , ...An , te i svaka od inverznih pravih sadrzi pored
tacaka kojim je ona odredena bar jos jednu tacku skupa A2 , ...An . S obzirom da
prava odredena bilo kojim dvema tackama skupa A2 , ...An sadrzi bar jos jednu
tacku tog skupa, prema Silvesterovoj teoremi, sve tacke tog skupa pripadaju
jednoj pravoj. Stoga i sve tacke skupa A1 , ...An pripadaju jednom krugu.
651. Dokazati da je ugao pod kojim se seku dve linije u nekoj tacki jednak s
uglom pod kojim se seku u odgovarajucoj tacki njima inverzne linije u odnosu
na neki krug k.
Resenje: Obelezimo sa m i n dve linije koje se seku u nekoj tacki P , a sa m i

n njima inverzne linije u odnosu na neki krug k(O, r). S obzirom da se linije m
i n seku u tacki P , i njima inverzne linije m i n seku se u odgovarajucoj tacki
P . Neka proizvoljna prava kroz tacku O , razlicita od prave Op, sece linije m i
n u tackama M i N , i prema tome linije m i n u odgovarajucim tackama M
i N . Prema zadatku 533 bice cetvorouglovi P P M M i P P N N tetivni, te je
OP M = OM P i OP N = ON P . Iz jednakosti OM.OM = ON.ON
sleduje da su tacke N i N izmedu tacaka M i M , ili su tacke M i M izmedu
tacaka N i N . Neka su na primer tacke N i N izmedu tacaka M i M . Pri
tome je
M P N = OP N = OP M = ON P = OM P = M P N .
Ako tacke M i N linija m i n teze ka tacki P i njima odgovarajuce tacke
M i N linija m i n teze ka tacki P . U granicnom polozaju secice M P , N P i
M P , N P su tangente u tackama P i P na linijama m, n i m , n . Stoga su
uglovi izmedu tangenti tih linija u tackama P i P jednaki, te su i uglovi pod
kojim se seku linije m i n odnosno linije m i n medu sobom jednaki (sl. 378).

n m

M
N
n
m

k
N
M

slika 378
652. Ako se dve linije l i l inverzne medu sobom u odnosu na neki krug
k seku u tacki P , dokazati da krug k sece linije l i l u tacki P pod jednakim
uglovima.
Resenje: S obzirom da su linije l i l inverzne medu sobom, njihova presecna
tacka P odgovara samoj sebi, te se nalazi na krugu k. U pomenutoj inverziji,
krugu k odgovara taj isti krug, te je prema prethodnom zadatku ugao pod kojim
se seku linije l i k u tacki P jednak s uglom pod kojim se seku linije l i k u istoj
tacki P .
416

653. Ako su A, B, C, D tacke takve da je krug OAB ortogonalan na


krugu OCD i krug OAC ortogonalan na krugu OBD, dokazati da je krug OBC
ortogonalan na krugu OAD.
Resenje: Obelezimo sa k bilo koji krug kome je srediste O, a sa A , B , C , D
tacke inverzne tackama A, B, C, D u odnosu na krug k. U tom preslikavanju,
ortogonalnim krugovima OAB i OCD odgovaraju upravne prave A B i C D ,
a ortogonalnim krugovima OAC i OBD odgovaraju upravne prave A C iB D .
Stoga je tacka D ortogonalan trougla A B C , pa je prava B C upravna na
pravoj A D , i prema tome krug OBC ortogonalan na krugu OAD.
654. Ako su A, B, C, D proizvoljne tacke jedne ravni, dokazati da je ugao
pod kojim se seku krugovi ABC i ABD jednak uglu pod kojim se seku krugovi
ACD i BCD.
Resenje: Obelezimo sa k bilo koji krug kome je srediste A, a sa B , C , D
tacke inverzne tackama B, C, D u odnosu na k u tom preslikavanju. Krugovima ABC i ABD odgovaraju prave B C i B D , a krugovima ACD i BCD
odgovaraju prava C D i krug B C D . Pri tome je ugao pod kojim se seku
krugovi ABC i ABD jednak uglu koji odreduju prave B C i D , a ugao pod
kojim se seku krugovi ACD i BCD jednak je uglu pod kojim se seku C D i
krug B C D . S obzirom da je ugao pod kojim se seku prave B C i B D jednak
uglu pod kojim se seku C D i krug C D , bice i ugao pod kojim se seku krugovi
ABC i ABD jednak uglu pod kojim se seku krugovi ACD i BCD.
655. Ako je P jedna od presecnih tacaka dvaju krugova k1 i k2 , koji dodiruju
jednu pravu u tackama A i B, a drugu pravu u tackama C i D, dokazati da se
krugovi l1 i l2 opisani oko trouglova P AB i P CD dodiruju u tacki P .
Resenje: Obelezimo sa k bilo koji krug kome je srediste P, sa k1 i k2 likove
inverzne krugovima k1 i k2 u odnosu na krug k. S obzirom da krugovi k1 i k2
sadrze tacku P njima inverzni likovi k1 i k2 su prave. Ako su tacke A , B ,
C , D inverzne tackama A, B, C, D u odnosu na krug k, bice krugovi OA B
i OC D inverzni pravama AB i CD. S obzirom da krugovi k1 i k2 dodiruju
pravu AB u tackama A i B, prave k1 i k2 dodirivace krug P A B u tackama
A i B . Isto tako prave k1 i k2 dodiruju krug P C D u tackama C i D . Pri
tom su prave A B i C D medu sobom uporedne, pa se krugovi OAB i P CD
tj. krugovi l1 i l2 inverzni pravama A B i C D u odnosu na krug k dodiruju u
zajednickoj tacki P . (sl.379)

A
B
P

Q
D

sl.379
656. Ako su l1 , l2 , l3 , tri medu sobom ortogonalna kruga kojima su za417

jednicke tetive AB, CD, EF , dokazati da se krugovi ACE i ADF dodiruju u


tacki A.
Resenje: Neka je k krug kome je srediste A, poluprecnik jednak proizvoljnoj
duzi r. Pri inverznom projektovanju u odnosu na krug k, ortogonalnim krugovima l1 i l2 koji se seku u tackama A i B odgovaraju upravne prave l1 i l2
koje se seku u tacki B inverznoj tacki B u odnosu na krug k. Krugu l3 u tom
preslikavanju odgovara takodje krug l3 koji je ortogonalan na pravama l1 i l2 te
mu je srediste tacka B . Tacke C , D i E , F u kojima krug l3 sece prave l1 i l2
inverzne su tackama C, D i E, F a prave C E i D F krugovima ACE i ADF u
odnosu na krug k. S obzirom da su prave C E i D F uporedne, krugovi ACE
i ADF se dodiruju u tacki A.
657. Neka su l i l inverzni krugovi, a P i P , Q i Q inverzne tacke u odnosu
na isti krug k. Ako su pri tome tacke P i O inverzne u odnosu na krug l,
dokazati da su tacke P i O inverzne u odnosu na krug l .
Resenje: S obzirom da su tacke P i Q inverzne u odnosu na krug l, bilo
koja dva kruga m i n kroz tacke P i Q ortogonalni su na krugu l, pa su i njima
inverzni krugovi m i n u odnosu na krug k ortogonalni na krugu l . Pri tome
krugovi m i n sadrze tacke P i Q , pa je prava P Q njihova radikalna osa.
Stoga se srediste S kruga l , koji je ortogonalna na krugovima m i n nalazi na
pravoj P Q , pa su tacke P i Q inverzne u odnosu na krug l . (sl.380)
P

P
Q
Q
O
S

sl.380
658. Ako su P , Q, R tacke u kojima upisani krug k dodiruje stranice BC,
CA, AB trougla ABC, dokazati da je Ojlerov krug l trougla P QR inverzan
s opisanim krugom l trougla ABC u odnosu na krug k.
Resenje: Obelezimo sa S srediste kruga k. Duzi SA, SB, SC seku stranice
QR, RP , P Q trougla P QR u tackama P , Q , R , pri cemu je SA SP =
SB SQ = SC SR , pa su tacke P , Q , R inverzne s tackama A, B, C
u odnosu na krug k. No, tacke P , Q , R su sredista stranica QR, RP , P Q
trougla P QR, te pripadaju Ojlerovom krugu trougla P QR. Stoga je krug l u
odnosu na krug k. (sl.381)

418

A
Q
P
R
S

Q
B

sl.381
659. Dokazati da Ojlerov krug trougla dodiruje sva cetiri upisana kruga tog
trougla (Fojerbahova teorema).
Resenje: Neka su k, ka , kb , kc upisani krugovi u l, Ojlerov krug trougla
ABC. Obelezimo sa S i Sa sredista krugova k i ka , sa P i Pa tacke u kojima
krugovi k i ka dodiruju stranicu BC, sa D podnozje visine iz temena A, sa E
tacku u kojoj simetrala ASa ugla A sece stranicu BC, sa A1 srediste stranice
BC, tu duzi P Pa , sa H ortocentar trougla ABC i sa K srediste duzi AH. Pri
tome su tacke S i Sa harmonijski spregnute s tackama A i E, pa su i njihove
uspravne projekcije na pravoj BC, dakle tacke P i Pa harmonijski spregnute s
tackama D i E. Stoga je A1 D A1 EA1 P 2 = A1 P a2 , pa su tacke D i E inverzne
medju sobom u odnosu na krug la kome je srediste A1 , a poluprecnik duz A1 P .
S obzirom da su krugovi k i ka ortogonalni na krugu la , u pomenutoj inverziji
svaki od krugova k i ka odgovara samom sebi. Dokazimo da u tom preslikavanju
Ojlerovom krugu l trougla ABC odgovara zajednicka unutrasnja dirka krugova
k i ka koja ne sadrzi stranicu BC, a dodiruje krugove k i ka u tackama recimo
Q i Qa . Krug l sadrzi srediste A1 inverzije i tacku D, pa je njegov inverzan
lik prava koja sadrzi tacku E, a upravna je na precniku A1 K kruga l. Kako se
sredista K i B1 duzi AH i AC na istom luku kruga l, bice
DKA1 = DB1 A1 = DB1 C A1 B1 C = 2DAC BAC =
= 2DAC 2BAC = 2DAE = 2P SE = P SQ = QA1 E.
Stoga je prava SQ, tj. prava QQa upravna na pravoj A1 K, i prema tome
inverzna krugu l u odnosu na krug la . S obzirom da prava QQa dodiruje krug
k u tacki Q i njima inverzni likovi, dakle krugovi k i l se dodiruju u tacki Q u
odnosu na krug la . Analogno se dokazuje da krug l dodiruje i krugove ka , kb ,
kc . (sl.382)
Napomena: Tacke u kojima Ojlerov krug dodiruje upisane krugove trougla
ABC nazivamo Fojerbakovim tackama tog trougla.
419

A
C1

B1
H S

C
D P

A1 P a

Qc

ka

Sa

Pa

sl.382
660. (Ptolemejeva teorema) Ako je ABCD tetivan cetvorougao, dokazati da
je
AB CD + BC DA = AC BD.
Resenje: Neka je k krug kome je srediste D a poluprecnik jednak proizvoljnoj
duzi r. Pri inverznom preslikavanju u odnosu na krug k tackama A, B , C koje
se nalaze na krugu l koji sadrzi srediste D inverzije odgovaraju tacke A , B ,
C koje se nalaze na jednoj pravoj l . Stoga je A B + B C = A C . Prema
zadatku 659 imamo da je
A B = AB

r2
r2
r2
, BC =
, C A =
DA DB
DB DC
DC DA

pa je s obzirom na prethodnu jednakost


AB
BC
CA
+
=
.
DA DB
DB DC
DC DA
Mnozenjem obeju strana jednakosti sa DA DB DC, dobijamo da je AB CD +
BC DA = AC BD.
661. Ako su d1 , d2 , ..., dn rastojanja temena A1 , A2 , ..., An pravilnog n-tougla
A1 A2 ...An od proizvoljne tacke P koja se nalazi na manjem luku A1 An kruga
opisanog oko tog n-tougla, dokazati da je
1
1
1
1
1
+
+
+ ... +
=
.
d1 d2
d2 d3
d3 d4
dn1 dn
d1 dn

420

r
An

An
P
A1

d2
A2
d3

A1

A3

sl.661

Obelezimo sa k krug kome je srediste P , a poluprecnik proizvoljna duz r, a


sa A1 , . . . , An tacke inverzne tackama A1 , . . . , An u odnosu na krug k. Prema
zadatku 543, imamo da je
A1 A2 = A1 A2

r2
r2
r2
, A2 A3 =
, . . . , A1 An = A1 An
OA1 OA2
OA2 OA3
OA1 OAn

S obzirom da je A1 A2 + A2 A3 + . . . + An1 An = A1 An i A1 A2 = . . . = An A1
bice
1
1
1
1
1
+
+
+ ... +
=
.
d1 d2
d2 d3
d3 d4
dn1 dn
d1 dn
662. Ako su d1 , d2 , ..., d2n+1 rastojanja temena pravilnog poligona A1 A2 ...A2n+1
sa neparnim brojem stranica od proizvoljne tacke P koja se nalazi na manjem
luku A1 A2n+1 kruga opisanog oko poligona, dokazati da je
d1 + d3 + . . . + d2n+1 = d2 + d4 + . . . + d2n .

421

A2 n + 1

A2 n + 1
k

d3

A3

d2
A1

b
A2

A1

sl.662

Kao u prethodnom primeru, obelezimo sa k krug kome je srediste P , a poluprecnik jednak proizvoljnoj duzi r, a sa A1 , . . . , A2n+1 tacke inverzne tackama
A1 , . . . , A2n+1 u odnosu na krug k. Ako sa a obelezimo stranice, a sa b tetive
jednake dijagonali A1 A3 tog pravilnog poligona, prema zadatku 543 imamo da
je
r2
r2
r2
, A2 A3 = a
, . . . , A1 An = a
,
A1 A2 = a
d1 d2
d2 d3
d1 dn
A1 A3 = b

r2
,
d1 d3

A2 A4 = b

Iz ovih jednakosti i iz jednakosti


A1 A2 +A2 A3 = A1 A3 ,

r2
r2
, . . . , A2n+1 A2 = a
.
d2 d4
d2n+1 d2

A2 A3 +A3 A4 = A2 A4 , . . . , A2n+1 A2 +A2 A1 = A2n+1 A1

nalazimo da je
a d1 + a d3 = b d2 ,

a d2 + a d4 = b d3 , . . . , a d1 + a d2n+1 = b d2

te se sabiranjem odgovarajucih strana dobija da je


d1 + d3 + . . . + d2n+1 = d2 + d4 + . . . + d2n .

422

663. Ako su p1 , p2 , . . . , p2n odstojanja proizvoljne tacke P kruga od pravih


koje sadrze stranice A1 A2 , A2 A3 , . . . , A2n A1 poligona A1 A2 . . . A2n upisanog u
krug, dokazati da je
p1 p3 . . . p2n1 = p2 p4 . . . p2n .

A2 n

A2 n
p1
A1

P
p
r

A2

p2

l
A2
A1

A3

sl.663

Obelezimo sa k krug kome je srediste P , a poluprecnik jednak proizvoljnoj


duzi r. S obzirom da se tacke A1 , . . . , A2n nalaze na krugu l koji sadrzi srediste
P kruga k, tacke A1 , . . . , A2n inverzne tackama A1 , . . . , A2n u odnosu na krug
k pripadaju jednoj pravoj. Neka je p odstojanje tacke P od te prave. Iz slicnih
trouglova P A1 A2 i P A2 A1 imamo da je
P A1
p1
=
p
P A2
ili

p21
P A1 P A2
=
2
p
P A1 P A2

Na isti nacin dobijamo da je

p22
P A2 P A3
p22n
P A2n P A1
=
,
.
.
.
,
=
.
p2
P A2 P A3
p2
P A2n P A1
423

Otuda je


p1 p3 . . . p2n1
pn

pa je

p2 p4 . . . p2n
pn

2

2

P A1 P A2 . . . P A2n
P A1 P A2 . . . P A2n

P A1 P A2 . . . P A2n
,
P A1 P A2 . . . P A2n

p1 p3 . . . p2n1 = p2 p4 . . . p2n .
664. Ako su p1 , p2 , . . . , pn odstojanja proizvoljne tacke P kruga l od pravih koje
sadrze stranice A1 A2 , A2 A3 , . . . , An A1 n-tougla A1 A2 . . . An upisanog u krug l,
a q1 , q2 , . . . , qn odstojanja tacke P od dirki kruga l u tackama A1 , A2 , . . . , An ,
dokazati da je
p1 p2 . . . pn = q1 q2 . . . qn .
Ovaj stav moze se smatrati granicnim slucajem prethodnog stava. Naime,
moze se pretpostaviti da je u krug l upisan poligon A1 A1 A2 A2 . . . An An koji
ima 2n temena, a kod kojeg se temena A poklapaju sa temenima A , pa se
mogu smatrati granicnim slucajem priblizavanja temena A ka temenima A .
Stoga je prema prethodnom zadatku
p1 p2 . . . pn = q1 q2 . . . qn .

665. Ako su a1 , a2 , . . . , an duzi jednake stranicama A1 A2 , A2 A3 , . . . , An A1 ntougla upisanog u krug l, a p1 , p2 , . . . , pn odstojanja proizvoljne tacke P luka
An A1 kruga l od pravih koje sadrze stranice A1 A2 , A2 A3 , . . . , An A1 ,dokazati da
je
a1
a2
an1
an
+
+ ...+
=
.
p1
p2
pn1
pn
Obelezimo sa k krug kome je srediste P , a poluprecnik jednak proizvoljnoj
duzi r. S obzirom da se tacke A1 , ..., A2n nalaze na krugu l koji sadrzi srediste
P kruga k, tacke A1 , . . . , A2n inverzne tackama A1 , ..., A2n u odnosu na krug k
pripadaju jednoj pravoj l . Pri tome je
A1 A2 + A2 A3 + . . . + An1 An = A1 An .
Neka je p odstojanje tacke P od prave l . Kako su P A1 A2 i P A2 A1 , . . . , P An A1
i P A1 An slicni, imamo da je A1 A2 : A1 A2 = p1 : p, . . . , An A1 : An A1 = pn : p,
an
a1
.
tj. da je A1 A2 = , . . . , An A1 =
p1
pn
Otuda je
a2
an1
an
a1
+
+ ...+
=
.
p1
p2
pn1
pn
424

666. Ako su a1 , a2 , . . . , an odstojanja proizvoljne tacke P manjeg luka An A1


kruga l opisanog oko pravilnog poligona A1 A2 . . . An od pravih koje sadrze stranice A1 A2 , A2 A3 , . . . , An A1 , dokazati da je
1
1
1
1
+
+ ...+
=
.
p1
p2
pn1
pn

Ovaj stav je specijalan slucaj prethodnog, jer je a1 = a2 = . . . = an .

425

INVOLUCIJA

Definicija 8.1. Jednoznacno preslikavanje f tacaka A,B,C, ... neke prave p


u tacke A,B,C,... te iste prave p nazivamo involucionim ili kratko involucijom
ako na toj pravoj p postoji tacka O takva da je
OA OA = OB OB = OC OB = ... =
Parove odgovarajucih tacaka involucionog preslikavanja nazivamo spregnutim ili
konjugovanim, pravu p na kojoj se nalaze te tacke nazivamo osnovom, a tacku
O sredistem involucionog preslikavanja.
S obzirom na znak konstante razlikujemo dve vrste involucionog preslikavanja
i to hiperbolicko i elipticko involuciono preslikavanje. Involuciono preslikavanje
nazivamo hiperbolickim ili eliptickim prema tome da li je > 0 ili < 0.
Iz ove definicije neposredno sleduje da su u hiperbolickom involucionom preslikavanju odgovarajuce tacke s iste strane sredista O, a u eliptickom sa raznih
strana tacke O tog involucionog preslikavanja. Stoga u hiperbolickom involucionom preslikavanju, postoje na osnovi p s raznih strana od O dve tacke X iY tako
da je OX 2 = OY 2 = . To su tacke koje u pomenutom preslikavanju odgovaruju
same sebi, te ih nazivamo dvojnim ili invarijantnim tackama tog hiperbolickog
involucionog preslikavanja. Elipticko involuciono preslikavanje nema invarijantnih tacaka.

8.1 Zadaci
667. Dokazati da su presecne tacke jedne prave sa krugovima nekog pramena
odgovarajuce tacke nekog involucionog preslikavanja.

A o

Sl. 383
Ako obelezimo sa A i A , B i B , C i C ... parove tacaka u kojima neka
prava p sece krugove a, b, c,... nekog pramena krugova, a sa O tacku u kojoj
prava p sece radikalnu osu o tog pramena krugova, bice
OA OA = OB OB = OC OC = ... = l.
Stoga su presecne tacke prave p sa bilo kojim krugom tog pramena krugova
odgovarajuce tacke jednog involucionog preslikavanja (sl. 383)
426

668. Dokazati da krugovi odredjeni odgovarajucim tackama nekog involucionog preslikavanja i jednom fiksiranom tackom koja se nalazi izvan osnove tog
preslikavanja obrazuju pramen krugova.
P

o
A B

Sl. 384
Neka su A i A , B i B , C i C , ... odgovarajuce tacke nekog involucionog
preslikavanja sa sredistem O, a P fiksirana tacka koja se nalazi izvan osnove p tog
preslikavanja. Obelezimo sa a, b, c,... krugove odredene trojkama nekolinearnih
tacaka P , A, A ; P , B, B ; P , C, C ;.... Pri tome svaka od tacaka O i P ima
jednake potencije u odnosu na sve krugove a, b, c,... te je prava OP radikalna
osa bilo koja dva od tih krugova. Stoga krugovi a, b, c, ... obrazuju pramen
krugova. (sl.384).
669. Dokazati da je involuciono preslikavanje jednoznacno odredjeno sa dva
para odgovarajucih tacaka.

Q
P
O

A BM

M B

Sl. 385
Prvi na
cin. Neka su A, A i B, B dva para tacaka neke prave p. Ako
obelezimo sa k1 proizvoljni krug koji sadrzi tacke A i A , sa k2 proizvoljan krug
427

koji sadrzi tacke B i B , a sa O tacku u kojoj radikalna osa tih krugova sece
pravu p, bice OA OA = OB OB . Stoga je tacka O srediste involucionog
preslikavanja u kojiem su A, A i B, B parovi odgovarajucih tacaka. Za bilo
koja druga dva kruga l1 i l2 kroz tacke A, A i B, B dobija se ista tacka O,
koja je prema tome jednoznacno odredena sa dva para odgovarajucih tacaka (sl.
385).
R
Q

Sl. 386
Drugi na
cin. Neka su A, A i B, B dva para tacaka neke prave p. Ako
obelezimo sa P bilo koju tacku koja se nalazi izvan prave p, sa Q tacku takvu
da je AP k B Q i BP k A Q, a sa O tacku u kojoj se seku prave p i P Q, bice
OAP OB Q i OBP OA Q , pa je OA : OB = OP : OQ i OB :
OA = OP : OQ. Otuda je OA : OB = OB : OA , tj. OA OA = OB OB ,
pa je tacka O srediste involucije kojoj su A, A i B, B parovi odgovarajucih
tacaka. S obzirom da su trouglovi P AB i QB A slicni i u slicnom polozaju u
odnosu na tacku O, bice AO : B O = AB : B A . Duzi koje figurisu na desnoj
strani ove proporcije su poznate, pa je tacka O jednoznacno odredena (sl. 386).
Napomena. Na slikama 385 i 386 izvedena je i konstrukcija tacke M koja u
tom involucionom preslikavanju odgovara proizvoljnoj tacki M prave p.

670. Dokazati da su odgovarajuce tacke hiperbolickog involucionog preslikavanja harmonijski spregnute sa dvojnim tackama tog preslikavanja.

Ako obelezimo sa X i Y dvojne tacke a sa P i P bilo koje dve odgovarajuce


tacke hiperbolickog involucionog preslikavanja, bice OP OP = OX 2 = OY 2 ,
te je prema zadatku 402 H(X, Y ; P, P ).

671. Dokazati da su parovi kolinearnih tacaka spregnutih u odnosu na neki krug


u involuciji u kojoj se dvojne tacke nalaze na tom krugu.

428

p
p

Sl. 387
Ako obelezimo sa M i N bilo koji par tacaka jedne prave p spregnutih u
odnosu na neki krug k i sa P pol prave p u odnosu na krug k, bice trougao P M N
autopolaran u odnosu na krug k, te mu se prema zadatku 402 ortocentar poklapa
sa sredistem O kruga k. S obzirom da je prava OP upravna na pravoj p, ona je
sece u nekoj tacki S. Pri tome je OSM N SP , pa je OS : SM = N S : SP ,
i prema tome SM SN = OS SP . Proizvod na desnoj strani u dobijenoj relaciji
je konstantan, pa su tacke P i Q odgovarajuce u involuciji kojoj je srediste S
i koeficijent OS SP . Taj koeficijent OS SP bice negativan ili pozitivan, a
involucija elipticka ili hiperbolicka, u zavisnosti od toga da li prava p sece ili
pak ne sece krug k (sl. 387).
Ako prava p sece krug k u tackama npr. X i Y , bice SX 2 = SY 2 = OS SP ,
pa su tacke X i Y dvojne u toj hiperbolickoj involuciji.

678.Ako su A i A ,B i B ,C i C tri para odgovarajucih tacaka involucije,


dokazati da je
AB BC CA + A B B C C A = 0.
Prema zadatku 675, imamo da je
R(A, B; A, C ) = R(A , B ; A, C),
tj.

Otuda je

i prema tome

A A A C
AA AC
:
=
:
.
BA BC
BA BC
AA BC
A A B C
=
,

BA AC
B A A C
AB BC CA = 0.

429

679.Ako su A,A ,B,B ,C,C tacke jedne prave takve da je AB BC CA +


A B B C C A = 0. dokazati da su A i A ,B i B ,C i C parovi odgovarajucih
tacaka involucije.

Dokaz se izvodi reverzibilnim postupkom. Iz


AB BC CA + A B B C C A = 0
nalazimo da je

A A B C
AA BC
=
,
BA AC
B A A C

pa je

a a ac
a a a c
: = : ,

ba bc
ba bc

i prema tome
(a, b; a , c ) = (a , b ; a, c).

stoga su, prema zadatku 676, parovi tacaka a i a , b i b , c i c u involuciji.

680.Dokazati da su parovi tacaka P i P ,Q i Q ,R i R u kojima neka prava


sece prave odredjene naspramnim stranicama AB i CD, BC i DA, AC i BD
cetvorotemenika ABCD u involuciji.
A

B
D
C
R
Q
P

F
G

Sl. 392
Neka je E tacka u kojoj se seku naspramne stranice AB i CD. Pri tome je
R(P, Q; R, P ) = R(CP, CQ; CR, CP ) = R(P, B; A, E) =
= R(DP, DB; DA, DE) = R(P, R ; Q , P ) = (p , q ; r , p).

Stoga su, prema zadatku 676, parovi tacaka P i P , Q i Q , R i R u involuciji.(sl.392)


Napomena: Ova teorema omogucuje da se u involuciji koja je data sa dva
para odgovarajucih tacaka pomocu cetvorotemenika,konstrise tacka M koja u
toj involuciji odgovara proizvoljnoj tacki M .
430

681.Ako je S tacka ravni trougla ABC, a s prava koja sece prave BC , CA ,


AB u tackama P , Q , R i prave SA , SB , SC u tackama P , Q i R dokazati
da su P i P , Q i Q , R i R odgovarajuce tacke involucionog preslikavanja.
R

P
S
Q

Q
B

Sl. 393
Tacke P i P , Q i Q , R i R su preseci prave s sa naspramnim stranicama
cetvorotemenika SABC, pa su, prema zadatku 680, te tacke u involuciji. (sl.393)

682.Ako neka prava s sece prave odredjene stranicama BC , CA , AB trotemenika ABC u tackama P , Q , R i ako su P , Q i R tacke prave s koje u nekoj
involuciji odgovaraju tackama P , Q , R, dokazati da se prave AP , BQ i CR
seku u jednoj tacki.
Neka je S tacka u kojoj se seku prave AS i BQ , a R tacka u kojoj se seku
prave SC i s. S obzirom da su tacke P i P , Q i Q , R i R preseci prave s sa
naspramnim stranicama cetvorotemenika SABC, prema zadatku 680, one su u
involuciji, pa je
R(P, Q; R, P ) = R(P , Q ; R , P ).
Po pretpostavci, tacke P i P , Q i Q , R i R su u involuciji, pa je
R(P, Q; R, P ) = R(P , Q ; R , P ).
Stoga je
R(P , Q ; R , P ) = R(P , Q ; R , P ),

pa su tacke R i R istovetne, i prema tome prave AP , BQ , CR su konkurentne.

431

683.Ako su P ,P i Q,Q parovi tacaka u kojima neka prava s sece naspramne


stranice AB,CD i BC,AD cetvorotemenika ABCD upisanog u krug k, dokazati
da tacke X i X u kojima prava s sece krug k odgovaraju jedna drugoj u involuciji
koja je odredjena parovima tacaka P ,P i Q,Q .

Q
Q

C
X

Sl. 394
prema zadatku 466 imamo da je
(p, q; x, x ) = (bp, bq; bx, bx ) = (ba, bc; bx, bx ) =
= (da, dc; dx, dx ) = (q , p ; x, x ) = (p , q ; x , x)

Stoga su, prema zadatku 675, tacke X i X odgovarajuce u involuciji koja je


odredjena parovima P ,P i Q,Q .(sl.394)

684.(Karnoova teorema) Ako neki krug k sece prave odredjene stranicama


BC, CA,AB trougla ABC u tackama P i P , Q i Q , R i R , dokazati da je
BP CQ AR BP CQ AR

= 1.
P C QA RB P C Q A R B

A
Q
R

R
S

S
B

Sl. 395
432

Neka prave QR i Q R seku pravu BC u tackama S i S . Primenom Menelajeve teoreme na trougao ABC nalazimo da je (sl.395)
BS CQ AR

= 1
SC QA RB

(1)

BS CQ AR

= 1
S C Q A R B

(2)

S obzirom da prava BC sece naspramne stranice QR, Q R i QQ , RR cetvorotemenika


QRR Q u tackama S, S , C i B a opisani krug k oko tog cetvorotemenika u
tackama P i P , bice prema zadatku 680, parovi tacaka B, C; S, S ; P, P u involuciji. Stoga je
R(B, C; P , S ) = R(C, B; P, S) = R(B, C; S, P )
pa je

BP BS
BS BP
:
=
:
P C S C
SC P C

tj.

BP BP
BS BS
=

PC P C
SC S C

(3)

Iz jednakosti (1), (2), (3) sledi da je


BP CQ AR BP CQ AR

= 1.
P C QA RB P C Q A R B

685.Ako je O srediste involucije odredjene sa dva para odgovarajucih tacaka


A,A i B,B , dokazati da je
a)

AB
OA
.
=
OB
BA

b)

OA
AB AB
=
.
OA
A B A B

a) Saglasno definiciji involucije imamo da je


OA OA = OB OB ,
tj.
OA : AB = OB : OA
pa je
OA : OB = (OA + AB ) : (OB + BA ),
i prema tome
OA : OB = AB : BA .

433

b) Iz
OA OA = OB OB
sledi da je
OA : OA = (OB : OA ) : (OB : OA ).
Medjutim, prema izvedenom delu ovog zadatka imamo da je
OB : OA = OA : OB = AB : BA .
S druge strane, iz
OA : OB = OB : CA
sledi da je
(OA OB) : OB = (OB OA ) : OA
pa je
OB : OR = AB : A B .
Stoga je i
OA : OA = (AB : A B )(AB : A B ).

686.Dokazati da su granicne tacke hiperbolickog pramena krugova dvojne tacke


involucije odredjene parovima tacaka u kojima sredisnja prava sece krugove tog
pramena .
Ako obelezimo sa O srediste i sa X i Y granicne tacke hiperbolicnog pramena
krugova, a sa A, A i B, B parove tacaka u kojima sredisnja prava sece bilo koja
dva kruga tog pramena, bice
OA OA = OB OB = OX 2 = OY 2 ,
pa su X i Y dvojne tacke i O srediste involucije odredjene parovima tacaka
A, A i B, B .

687.Ako su X i Y granicne tacke hiperbolickog pramena krugova, a X1 i Y1


dvojne tacke involucije odredjene parovima tacaka u kojima neka prava sece
krugove tog pramena , dokazati da tacke X,Y ,X1 ,Y1 pripadaju jednom krugu.

Ako je O1 tacka u kojoj prava X1 Y1 sece radikalnu osu datog pramena krugova,
bice O1 X = O1 Y = O1 X1 , pa su tacke X, X1 , Y , Y1 na jednom krugu.

688.Ako su X i Y dvojne tacke involucije odredjene parovima odgovarajucih


tacaka A,A i B,B , dokazati da su tacke X i Y odgovarajuce u involuciji koja
je odredjena parovima tacaka A,B i A ,B .

434

Prema zadatku 670, imamo da je R(X, Y ; A, A ) = 1 i R(Y, X; B, B ) = 1,


pa je
R(X, Y ; A, A ) = R(Y, X; B, B ).
Stoga su, prema zadatku 676, tacke X i Y u involuciji koja je odredjena parovima tacaka A, B i A , B .

689.Ako su A i A , B i B ,C i C parovi odgovarajucih tacaka neke involucije


a D i D tacke takve da je R(A, B; C, D) = R(A , B ; C , D ), dokazati da su
tacke D i D odgovarajuce u toj involuciji. Ako su A i A , B i B , C i C , D i
D parovi odgovarajucih tacaka neke involucije, dokazati da je R(A, B; C, D) =
R(A , B ; C , D ).
po pretpostavci je
tj.

a c a d
ac ad
:
= :
bc bd
bc bd

(1)

(a, b; c, a ) = (a , b ; c , a) tj.

ac aa
a c a a
: = :
bc ba
bc ba

(2)

(a, b; c, d) = (a , b ; c , d )
a prema zadatku 675,

iz dobijenih jednakosti (1) i (2) nalazimo da je


a d a a
ad aa
: = :
bd ba
bd bd
tj. da je
(a, b; d, a ) = (a , b ; d , a).

stoga su, prema zadatku 676, tacke d i d odgovarajuce u involuciji koja je


odredjena parovima tacaka a, a i b, b .

690.Ako su A i A , B i B , C i C , D i D parovi odgovarajucih tacaka neke


involucije, dokazati da je R(A, B; C, D) = R(A , B ; C , D ).

Dokaz se izvodi indirektno koristeci prethodni zadatak.

691.Ako su P , Q, R projekcije temena A, B, C iz proizvoljne tacke S na naspramnim stranicama trotemenika ABC, a P , Q , R tacke u kojima proizvoljna
prava s sece stranice BC, CA, AB tog trotemenika, dokazati da je
R(B, C; P, P ) R(Q, A; Q, Q ) R(A, B; R, R ) = 1.
435

A
Q
C
B
P

R
P

P
R

Sl. 396
Ako obelezimo sa P , Q , R projekcije tacaka P, Q, R iz tacke S na pravu
s (sl.396), bice
(b, c; p, p ) (q, a; q, q ) (a, b; r, r ) =
= (q , r ; p , p ) (r , p ; q , q ) (p , q ; r , r )
=(

q p q p
r q
r q
p r p r
: ) ( :
) ( : )

r p
p p
p q
p+q
q r
q r
=

p q q r r p
q p r q p r

parovi tacaka p i p , q i q , r i r su presecne tacke prave s sa naspramnim


stranicama cetvorotemenika sabc, pa su, prema zadatku 680, ti parovi tacaka u
involuciji. stoga je, prema zadatku 678,
p q q r r p
= 1
q p r q p r
pa je prema tome
(b, c; p, p ) (q, a; q, q ) (a, b; r, r ) = 1.

692.Ako su A i A , B i B , C i C parovi naspramnih temena nekog cetvorougaonika


i O proizvoljna tacka u njegovoj ravni, dokazati da su parovi pravih OA i OA ,
OB i OB , OC i OC u involuciji.

436

B
D

Sl. 397
Ako sa D i D obelezimo tacke u kojima prava OC sece prave AB i A B ,
imamo da je
(oa, ob; oc, oc ) = (a, b; c, d) = (b , a ; c, d ) =
= (ob , oa ; oc, od ) = (ob , oa ; oc, oc ) = (oa , ob ; oc , oc)

Stoga su parovi pravih OA i OA , OB i OB , OC i OC u involuciji. (sl.397)

693.Ako su A i A , B i B , C i C tacke u kojima tri prave kroz istu tacku seku


neki krug k i ako je S proizvoljna tacka kruga k, dokazati da su parovi pravih
SA i SA , SB i SB , SC i SC u involuciji.
S

A
A
D
B

O
B
C

slika 398
Ako obelezimo sa O tacku u kojoj se seku prave AA , BB ,CC i sa D tacku
u kojoj se seku prave AA i CB , imamo da je
R(A, O; D, A ) = R(A , D; O, A).
Otuda je
R(B A, B O; B D, B A ) = R(CA , CD; CO, CA)
437

i prema tome
R(B A, B B; B C, B A ) = R(CA , CB ; CC , CA).
Iz ove jednakosti i zadatka 466. sledi da je
R(SA, SB; SC, SA ) = R(SA , SB ; SC , CA)
pa su parovi pravih SA i SA , SB i SB , SC i SC u involuciji. Nije tesko
utvrditi da je involucija elipticka kada se tacka O nalazi u krugu k, a hiperbolicka
kada se tacka O nalazi izvan kruga k.

438

ZNACAJNE
TACKE
I LINIJE U GEOME
TRIJI TROUGLA I CETVOROUGLA

U geometriji trougla i cetvorougla vec smo upoznali niz znacajnih tacaka


i linija kao sto su u geometriji trougla: teziste, ortocentar, srediste opisanog

kruga, srediste upisanog kruga, Zergonova


tacka, Nagelova tacka, Fojerbahove
tacke, opisani krug, polarni krug, itd, a u geometriji cetvorougla teziste, Gausova
prava, Oberova prava, itd. U ovom clanu upoznacemo jos neke znacajne tacke
i linije u geometriji trougla i cetvorougla.

9.1

Simsonova prava trougla i


cetvorougla

694.Dokazati da podnozja upravnih iz bilo koje tacke P kruga opisanog oko


bilo kojeg trougla ABC na pravama koje su odredjene stranicama tog trougla,
pripadaju jednoj pravoj koju nazivamo Simsonovom pravom tacke P u odnosu
na trougao ABC.
Uputstvo v.z. 138.
695.Ako se podnozje upravnih iz neke tacke P na pravama koje su odredjene
stranicama jednog trougla nalaze na jednoj pravoj, dokazati da je ta tacka P
na krugu koji je opisan oko tog trougla.

Koristeci oznake uvedena pri rasavanju zadatka 138. i pretpostavljajuci da je


npr:[AB C] i [A B C ], imamo da su uglovi C B A i A B C jednaki i istosmerni.
No kod tetivnih cetvorouglova P C AB i P B A C uglovi C B A i A B C jednaki
su i istosmerni sa uglovima C P A i A P C pa su i uglovi C P A i AP C jednaki
i istosmerni. Pri tome je kod konveksnog cetvorougla C BA P ugao C P A
suplementan sa uglom C BA pa je i kod konveksnog cetvorougla ABCP ugao
AP C suplementan sa uglom ABC. Stoga je tacka P na krugu koji je opisan
oko trougla ABC.

696.Ako su AA , BB , CC visine trougla ABC, dokazati da podnozja P , Q,


R, S upravnih iz tacke A na pravama AB, AC, BB , CC pripadaju jednoj
pravoj.

439

B
Q

C
S
R
P
B

slika 399
Tacka A je na krugu koji je opisan oko trougla ABB , te se prema Simsonovoj
teoremi podnozja P , Q, R normala iz tacke A na pravama AB, AB , BB nalaze
na jednoj pravoj. Isto tako, tacka A je na krugu koji je opisan oko trougla
ACC , te se prema Simsonovoj teoremi podnozja P , Q, S upravnih iz tacke
A na pravama AC , AC, CC nalaze na jednoj pravoj. Otuda sledi da tacke
P, Q, R, S pripadaju jednoj pravoj.

697.Ako tri razne tetive istog kruga imaju jedan zajednicki kraj, dokazati da
se krugovi kojima su te tetive precnici seku u tackama koje pripadaju jednoj
pravoj (Salmonova teorema).

slika 400
440

Presecne tacke triju krugova kojima su precnici tetive P A, P B, P C istog


kruga k su podnozja upravnih iz P na pravama BC, CA, AB, te prema Simsonovoj teoremi pripadaju jednoj pravoj.
672.Dokazati da su parovi pravih jednog pramena koje su spregnute u odnosu na neki krug u involuciji kojoj su dvojne prave dirke toga kruga.
Promenljivi par pravih m,n jednog pramena {P } spregnutih u odnosu na neki
krug k seku polaru p tacke P u odnosu na krug k u promenljivom paru tacaka
M ,N koje su takode spregnute u odnosu na krug k.Prema prethodnom zadatku
ti parovi tacaka na pravoj p su u involuciji koja je elipticka ako prava p ne
sece krug k i hiperblicka ako prava p sece krug k.Stoga je i odgovarajuce prave
pramena{p} u involuciji koja je elipticka ako je tacka P u krugu k i hiperbolicka
ako je tacka P izvan kruga k.Kada prava p sece krug k,dvojne tacke pomenute
involucije na pavoj p su presecne tacke te prave sa krugom k,te su i dvojne prave
pomenute involucije pramena {P } dirke kruga k.Slika 388.
m

M
M

x
X
n
O

p
p N

Slika 388a.
Slika 388b.
673.Dokazati da su parovi tacaka u kojima neka prava sece prave odgovarajucih pravih neke involucije takode u involuciji.
Ako su A,A i B,B dva para odgovarajucih tacaka eliptickog involucionog preslikavanja bice jedna od tacaka B i B izmedu tacaka A i A ,a druga nije izmedu
njih,te se krugovi kojima su duzi AA i BB precnici seku u dvema tackama P i
Q koje su simetricne medu sobom u odnosu na osnovu tog preslikavanja.Ako je
O tacka u kojoj duz P Q sece osnovu tog preslikavanja iz pravouglih trouglova
AP A i BP B nalazimo da je
OA OA = OB OB = OP 2 ,
pa je tacka O srediste tog preslikavanja.Stoga je takode
OM OM = OP 2 ,
441

pa je ugao M P M prav.Iz simetricnosti tacaka P i Q u odnosu na pravu M M


sledi da je i ugao M QM prav.Slika 389.

Slika 389.
674.Dokazati da u eliptickom involucionom preslikavanju prave na samu sebe
postoje dve tacke P i Q simetricne medu sobom u odnosu na tu pravu takve da
su za svaki par odgovarajucih tacaka M i M tog preslikavanja uglovi M P M i
M QM pravi.
Ako su a i a ,b i b ,c i c ,. . . parovi odgovarajucih pravih neke involucije I pramena pravih,tada po definiciji postoji prava koja sece te prave redom u parovima
odgovarajucih tacaka A i A ,B i B ,C i C . . . neke involucije I .Obelezimo sa
O srediste involucije I ,sa P srediste datog pramena pravih - sa Q tacku prave
OP takvu da je OP OQ = OA OA .Simetrala s duzi P Q sece pravu p u nekoj
tacki S ili je uporedna s pravom p.U prvom slucaju,krug k kome je srediste
S i poluprecnik SP = SQ sece pravu p u dvema tackama M i M ,takvim da
je OM OM = OP OQ = OA OA i P M P M ,te su prave m = P M i
m = P M odgovarajuce u involuciji I i uprave medu sobom.Ako je skp, tada u
involuciji I pravoj P O odgovara prava koje je uporedna s pravom p,dakle prava
upravna na pravoj P O.Ovim je dokazan prvi deo stava.
Ako involucija pramena pravih ima dva para odgovarajuacih medu sobom upravnih pravih npr. P M ,P M i P N ,P N , krugovi kojima su duzi M M i N N
precnici odreduju elipticki pramen krugova koji se seku u tacki P i njoj asimetricnoj tacki Q u odnosu na pravu p.S obzirom da svaki krug kroz taku P
i par odgovarajucih tacaka X,X involucije I mora da pripada tom pramenu
442

krugova,on sadrzi i tacku Q,te mu se srediste nalazi na pravoj p.Stoga je ugao


XP X prav,te su svake dve odgovarajuce prave P X i P X involucije upravne
medu sobom.Slika 390.

P
s

Slika 390.
675.Dokazati da u svakoj involuciji pramena pravih postoji jedan par odgovarajucih pravih koje su medu sobom upravne;ako takvih pravih ima vise,dokazati
da su svake dve odgovarajuce prave te involucije upravne medu sobom.
Ako obelezimo sa O srediste involucije ,bice OA OA = OC OC = 12 .S
toga je

AC AA
AC BA
(OC OA) (OA OB)
=
=
:
=

BC BA
BC AA
(OC OB) (OA OA)



1
1
1
1

(OA OC ) (OB OA)


OC
OA
OA OB

=
=
= 
1
1
1
1
(OB OC ) (OA OA)

OC
OB
OA OA

R(A, B; C, A ) =

A C AB
A C A A
= : = R(A , B; C , A).

B C AA
BC BA

676.Ako su A i A ,B i B ,C i C tri proizvoljna para odgovarajucih tacaka


jedne involucije,dokazati da je
R(A, B; C, A ) = R(A , B ; C , A).
443

Prema zadatku 669 parovi tacaka A,A i B,B odreduju jedno involuciono preslikavanje,neka je C tacka koja u tom preslikavanju odgovara tacki C.Saglasno
prethodnom zadatku imamo da R(A, B; C, A ) = R(A , B ; C , A).Iz jednakosti
R(A, B; C, A ) = R(A , B ; C , A) = R(A, B; C, A ) = R(A , B ; C , A)
sledi da je R(A, B ; C , A) = R(A , B ; C , A) te su tacke C i C istovetne,i
prema tome tacke C i C odgovarajuce u pomenutoj involuciji.
677.Ako su A, A , B, B , C, C tacke jedne prave takve da je R(A, B; C, A ) =
R(A , B ; C , A) dokazati da su A i A ,B i B ,C i C parovi odgovarajucih tacaka
izvesne involucije.
Ako su a i a ,b i b ,c i c tri para odgovarajucih pravih neke involucije,prema
definiciji postoji prava p koja ih sece takode u parovima odgovarajucih tacaka
A i A ,B i B ,C i C neke involucije.Stoga je prema zadatku 675
R(A, B; C, A ) = R(A , B ; C , A).
Ako obelezimo sa A1 , A1 , B1 , B1 , C1 , C1 racke u kojima neka druga prava p1 sece
prave a, a , b, b , c, c datog pramena bice R(A, B; C, A ) = R(A1 , B1 ; C1 , A1 ) i
R(A , B ; C , A) = R(A1 , B1 ; C1 , A1 ).Otuda je
R(A1 , B1 ; C1 , A1 ) = R(A1 , B1 ; C1 , A1 ),
te su prema zadatku 676 parovi tacaka A1 i A1 ,B1 i B1 ,C1 i C1 takode u involuciji.Slika 391.

p1
A1
B1

C1
C1

B1

A1

Slika 391.
698.Ako se tri kruga s precnicima P A, P B, P C seku u tackama koje pripadaju jednoj pravoj, dokazati da je tacka P na krugu koji je opisan oko trougla
ABC.
444

Resenje:
Po dva od triju krugova kojima su precnici P A, P B, P C sadrze podnozje
upravnih iz tacke P na pravama BC, CA, AB. S obzirom da su podnozja tih
upravnih na jednoj pravoj, prema zadatku 695, tacka P je na krugu opisanom
oko trougla ABC.
699.Ako se tri kruga s precnicima P A, P B, P C seku u tackama koje pripadaju jednoj pravoj, dokazati da je tacka P na krugu koji je opisan oko trougla
ABC.
Resenje:
S obzirom da se tacka P nalazi na krugu koji je opisan oko trougla ABC,
prema zadatku 694 podnozja A , B , C upravnih iz tacke P na pravama BC,
CA, AB pripadaju jednoj pravoj, Simsonovoj pravoj tacke P u odnosu na trougao ABC. Pri tome je A AC = A P C = A P C = A B C, pa je
AA k A B . Analogno se dokazuje da je BB k A B i CC k A B (sl. 401).
C
P
C
A
B

A
B

Slika 401
700.Ako su A , B , C tacke kruga opisanog oko trougla ABC takve da
je AA , BB , CC , dokazati da se prave kroz tacke A , B , C upravne na
pravama BC, CA, AB seku u izvesnoj tachki P koja se nalazi na krugu, zatim
da se podnozja A , B , C tih normala nalaze na jednoj pravoj, Simsonovoj
pravoj tacke P u odnosu na trougao ABC.
Resenje: Prave kroz tacke A i B upravne na pravama BC i CA seku se
u nekoj tacki P , pri cemu je A P B = BCA = BA A = A AB , pa je
tacka P na krugu l. Isto tako, prave kroz tacke A i C upravne na pravama
BC i AB seku se u nekoj tacki P koja se nalazi na krugu l. Stoga su tacke
P i P istovetne. S obzirom da se prave kroz A , B , C upravne na pravama
BC, CA, AB seku u tacki P koja se nalazi na krugu l, podnozja A , B , C tih
normala su na jednoj pravoj, Simsonovoj pravoj tacke P u odnosu na trougao
ABC.
445

701. Dokazati da Simsonova prava tacke P u odnosu na trougao ABC sadrzi


srediste duzi koja spaja tu tacku P sa ortocentrom H trougla ABC.
Resenje:
Pored oznaka uvedenih pri resavanju zadatka 699, obelezimo sa K tacku
u kojoj prava odredena visinom BB1 sece krug l, a sa L tacku duzi P B takvu da je HL k BB . S obzirom da je cetvorougao BB LH paralelogram, a
cetvorougao BB P K jednakokraki trapez, bice BB =HL i BB =KP , pa je
trapez HLP K takode jedankokrak. Prema poznatom stavu, tacka K je simetricna sa tackom H udnosu na pravu AC, pa je prava AC simetrala duzi HK,
dakle i duzi LP . Iz BB k HL i BB k A B sledi da je HL k A B . No B je
srediste duzi LP , pa je i presek S duzi HP i prave A B srediste duzi HP .
702.Ako tri trougla ABC, A B C , A B C upisana u istom krugu k imaju
zajednicko teziste, dokazati da se Simsonove prave proizvoljne tacke M kruga k
u odnosu na te trouglove seku u jednoj tacki.
Resenje:
Trouglovi ABC, A B C , A B C imaju zajednicko srediste O opisanog
kruga, zajednicko teziste T , dakle i zajednicki ortocentar H. Prema prethodnom zadatku, Simsonove prave tacke M u odnosu na trouglove ABC, A B C ,
A B C sadrze srediste duzi M H, te se iste seku u jednoj tacki.
703. Ako je O srediste kruga opisanog oko trougla ABC, dokazati da je
jedan od uglova koje odredjuju Simsonove prave dveju tacaka P1 i P2 kruga u
odnosu na trougao ABCjednak polovini ugla P1 OP2 .
Resenje:
Obelezimo sa A1 i A2 , B1 i B2 , C1 i C2 , podnozja upravnih iz tacaka P1 i P2
na pravama BC, CA, AB. Prema zadatku 694 tacke A1 , B1 , C1 pripdaju jednoj
pravoj p1 , Simsonovoj pravoj tacke P1 u odnosu na trougao ABC, a tacke A2 ,
B2 , C2 pripadaju pravoj p2 , Simsonovoj pravoj tacke P2 u odnosu na trougao
ABC. Ako su D1 i D2 tacke u kojima prave P1 A1 i P2 A2 seku krug l, prema
zadatku 699, bice AD1 k p1 i AD2 k p2 . Stoga je jedan od uglova odredenih
1
s pravama p1 i p2 jednak s uglom D1 AD2 . No, D1 AD2 = D1 OD2 =
2
1
P1 OP2 , pa je jedan od uglova odredenih s pravama p1 i p2 jednak polovini
2
ugla P1 OP2 (sl. 402).

446

C2
C1
P1
P2
A
B1

B2

A1
B

A2
C

D2
D1

p1

p2

Slika 402
704.Ako su p1 i p2 Simsonove prave dveju tacka P1 i P2 u odnosu na trougao
ABC, dokazati da se prave n1 i n2 kroz tacke P1 i P2 upravne na pravama p1 i
p2 seku na krugu koji je opisan oko trougla ABC
Resenje:
Ako su tacke P1 i P2 razlicite, prave p1 i p2 se seku u nekoj tacki S, te se
i prave n1 i n2 seku u nekoj tacki N . Prema tome je (p1 , p2 ) = (n1 , n2 ) =
1
1
P1 N P2 i (p1 , p2 ) = P1 OP2 pa je P1 N P2 = P1 OP2 , gde smo sa O
2
2
obelezili srediste kruga l opisanog oko trougla ABC. Stoga je tacka N na krugu
l (sl. 402).
705. Dokazati da se Simsonove prave p1 i p2 dveju dijametralno suprotnih
tacaka P1 i P2 u odnosu na isti trougao ABC seku pod pravim uglom u tacki
koja se nalazi na Ojlerovom krugu tog trougla.
Re:senje:
Ako je O srediste kruga opisanog oko trougla ABC i S tacka u kojoj se seku
1
prave p1 , p2 , imamo da je (p1 , p2 ) = P1 OP2 , pa su prave p1 i p2 medu
2
sobom upravne. Da bismo dokazali da je tacka S na Ojlerovom krugu trougla
ABC, obelezimo sa Q1 i Q2 tacke u kojima duzi HP1 i HP2 seku prave p1
i p2 . Prema zadatku 701, tacke Q1 i Q2 su srediste duzi HP1 i HP2 , pa je
duz Q1 Q2 precnik Ojlerovog kruga trougla ABC. S obzirom da je duz Q1 Q2
precnik Ojlerovog kruga trougla ABC, a ugao Q1 SQ2 prav, tacka S je takode
na Ojlerovom krugu tog trougla (sl. 402).
706.Ako je ABCD tetivan cetvorougao, dokazati da se Simsonove prave temena A, B, C, D u odnosu na trouglove BCD, CDA, DAB, ABC seku u
jednoj tacki.
Resenje:

447

Prema zadatku 701, Simsonove prave a, b, c, d tacaka A, B, C, D u odnosu


na trouglove BCD, CDA, DAB, ABC sadrze sredista duzi koje spajaju tacke
A, B, C, D s ortocentrima HA , HB , HC , HD navedenih trouglova. No duzi
AHA , BHB , CHC , DHD imaju zajednicko srediste, prema tome, prave a, b, c,
d seku se u jednoj tacki (sl. 403).

A
B
Slika 403
707.Ako je P proizvoljna tacka kruga opisanog oko tetivnog cetvorougla
ABCD, dokazati da podnozja upravnih iz tacke P na Simsonovim pravama te
iste tacke u odnosu na trouglove BCD, CDA, DAB, ABC pripadaju jednoj
pravoj, koju nazivamo Simsonovom pravom tacke P u odnosu na cetvorougao
ABCD.

9.2

Mikelova ta
cka trougla i
cetvorougla

Definicija 9.2. Ako su P , Q i R tacke stranica BC, CA i AB trougla ABC,


tada se krugovi opisani oko trouglova AQR, BRP , CP Q seku u izvesnoj tacki
M . Krugove opisane oko trouglova AQR, BRP , CP Q nazivamo Mikelovim
krugovima, a tacku M Mikelovom tackom trougla ABC za trojku tacaka P , Q
i R.
Resenje:
Simsonove prave pa , pb , pc tacke P u odnosu na trouglove BCD, CDA,
DAB obrazuju izvestan trougao cija su temena na krugu kome je duz P D
precnik, pa su podnozja upravnih iz tacke P na pravama pa , pb , pc na izvesnoj
pravoj p. Istim postupkom dokazije se da podnozja upravne iz tacke P na
Simsonovoj pravoj pa te tacke u odnosu na trougao ABC pripada pravoj koja
448

je odredena podnozjima upravnih iz tacke P na pravama pb i pc , tj. pravoj p.


Ovim je stav dokazan (sl. 404).

D
D1

A1
P
C1

B1

slika 404
708.Ako su P , Q, R proizvoljne tacke stranica BC, CA i AB trougla ABC,
dokazati da se krugovi opisani oko trouglova AQR, BRP , CP Q seku u jednoj
tacki.
Resenje:
Krugovi opisani oko trouglova BRP i CP Q dodiruju se u tacki P ,ili se seku
u tacki P i jos u nekoj tacki M . U prvom slucaju, uglovi P QC i P RB su
pravi, pa je cetvorougao ARP Q tetivan. Stoga tacka P pripada i krugu koji
je opisan oko trougla AQR. U drugom slucaju tacka M je u trouglu ABC
ili izvan njega. Ako je tacka M u trouglu ABC, bice susedni uglovi RM P i
RM Q suplementni uglovima B i C, pa je i ugao QM R suplementan sa uglom
A. Dakle cetvorougao M QAR je tetivan, pa je tacka M na krugu koji je opisan
oko trougla AQR. Ako je tacka M izvan trougla ABC, npr. u uglu A, bice
BRM = BP M i M P C = M QC. No uglovi M P C i M QC su naporedni
sa uglovima BP M i AQM , pa je BRM = AQM . S toga je cetvorougao
M QAR tetivan, pa je i tacka M na krugu koji je opisan oko trougla AQR (sl.
405).

449

R
Q

slika 405
709. Ako su P , Q, R tacke stranica BC, CA i AB, a M Mikelova tacka
tog trougla za trojku P , Q, R, dokazati da duzi M P , M Q, M R zahvataju s
odgovarajucim stranicama tog trougla jednake uglove, zatim da je BM C =
BAC + RP Q.
Resenje:
Tacka M je na trouglu ABC, u njemu, ili izvan njega. Ako je na trouglu
ABC, ona se poklapa sa jednom od tacaka P, Q, R npr. sa P , pa je ARM =
CQM ; ako je tacka M u trouglu ABC bice ARM = BP M = CQM ,
a ako je izvan trougla ABC, a u uglu npr. A, bice ARM = CP M =
CQM . Dakle, u svakom slucaju uglovi koje odreduju duzi M P, M Q, M R sa
odgovarajucim stranicama su medu sobom jednaki. Sada pokazimo drugi deo
stava. Ako je tacka M na trouglu ABC, dokaz je jednostavan; ako je u trouglu
ABC, bice BM C = BM P + P M C = BRP + P QC = (RAP +
RP A) + (P AQ + AP Q) = BAC + RP Q. Analogno se izvodi dokaz i u
slucaju kad je tacka M izvan trougla ABC.
710.Ako su P , Q, R proizvoljne tacke stranica BC, CA i AB trougla ABC, a
A , B , C sredista krugova opisanih oko trouglova AQR, BRP , CP Q, dokazati
da je ABC A B C .
Resenje:
Prema Minelovoj teoremi tj. zadatku 708., krugovi opisani oko trouglova
AQR, BRP, CP Q seku se u izvesnoj tacki M . Ako je tacka M u trouglu ABC,
bice B A C = 1/2RA Q = RAQ = BAC i A B C = 1/2RB P =
RBP = ABC, pa je ABC A B C . Analogno se izvodi dokaz i u
slucaju kada je tacka M na ili izvan trougla ABC.
450

711. Ako su P1 i P2 , Q1 i Q2 , R1 i R2 proizvoljne tacke stranica BC, CA


i AB trougla ABC, zatim A1 , B1 , C1 sredista krugova opisanih oko trouglova
AQ1 R1 , BR1 P1 , CP1 Q1 , i A2 , B2 , C2 sredista krugova opisanih oko trouglova
AQ2 R2 , BR2 P2 , CP2 Q2 , dokazati da je A1 B1 C1 A2 B2 C2 .
Resenje:
Prema prethodnom zadatku imamo da je ABC A1 B1 C1 i ABC
A2 B2 C2 pa je A1 B1 C1 A2 B2 C2 .
712.Dokazati da se krugovi opisani oko cetiri trougla koji su odredjeni sa
cetiri prave seku u jednoj tacki, Mikelovoj tacki tog cetvorougla.
Resenje:
Obelezimo sa p, q, r, s cetiri proizvoljne prave, a sa A, B, C, D, E, F tacke u
kojima se seku prave s i p, p i q, q i r, r i s, p i r, q i s. S obzirom da prave
q i s nisu uporedne, saglasni uglovi ADE i BCE su nejednaki, pa su i dirke
u tacki E na krugovima opisanim oko trouglova ADE i BCE razlicite. Stoga
se ti krugovi seku u tacki E i jos u nekoj tacki M . Prema Simsonovoj teoremi
podnozja P, Q, R, S upravnih iz tacke M na pravama p, q, r, s pripadaju jednoj
pravoj. Stoga tacka M pripada i krugovima koji su opisani oko trouglova ABF
i CDF (sl.406).

F
Q
C

S
D
R

P
E

slika 406
713. Dokazati da se sredista krugova opisanih oko trouglova koji su odredjeni
sa cetiri prave i Mikelova tacka cetvorougla koji je odredjen tim pravama nalaze
na jednom krugu.
Resenje:

451

Pored oznaka uvedenih u predhodnom zadatku, obelezimo sa M M1 , M M2 , M M3 , M M4


precnike krugova opisanih oko trouglova ADE, BCE, ABF, CDF . Prema zadatku 698., cetvorke tacaka M, M1 , M2 , M3 i M, M1 , M2 , M4 su kociklicne, prema
tome svih pet tacaka M, M1 , M2 , M3 , M4 pripadaju jednom krugu. Stoga tacka
M i sredista duzi M M1 , M M2 , M M3 , M M4 , tj. krugova opisanih oko trouglova
ADE, BCE, ABF, CDF pripadaju takode jednom krugu.
714.Dokazati da su ortocentri trouglova koji su odredjeni sa cetiri prave
kolinearni.

9.3

Ojlerova prava trougla i


cetvorougla

Definicija 9.2. Pravu koja je odredjena sredistem opisanog kruga i ortocentrom trougla nazivamo Ojlerovom pravom tog trougla.
Resenje:
Pored oznaka uvedenih pri resavanju zadatka 712., obelezimo sa Hp , Hq ,
Hr , Hs ortocentre trouglova CDE, ADE, ABF, BCE. Primenom Simsonove
teoreme, nalazimo neposredno da podnozja P, Q, R, S upravnih iz tacke M na
pravama p, q, r, s pripadaju jednoj pravoj m, zajednickoj Simsonovoj pravoj
tacke M u odnosu na pomenute trouglove. Prema zadatku 701., sredista duzi
M Hp , M Hq , M Hr , M Hs su na pravoj m, prema tome i tacke Hp , Hq , Hr , Hs
pripadaju jednoj pravoj.
715.Dokazati da se teziste T trougla ABC nalazi na Ojlerovoj pravoj tog
trougla, tj. pravoj koja je odredjena ortocentrom H i sredishtem O opisanog
kruga tog trougla, pri cemu HT : T O = 2 : 1.
Resenje:
Videti zadatak 145.
716. Ako su A1 , B1 , C1 sredista stranica BC, CA, AB trougla ABC, dokazati da se Ojlerova prava trougla A1 B1 C1 poklapa sa Ojlerovom pravom trougla
ABC.
Resenje:
Srediste O kruga opisanog oko trougla ABC je ortocentar trougla A1 B1 C1 ,
a teziste T trougla ABC istovetno s tezistem trougla A1 B1 C1 , pa se Ojlerova
prava trougla A1 B1 C1 poklapa s Ojlerovom pravom trougla ABC.
717.Ako je O srediste i r poluprecnik opisanog kruga, a S srediste i poluprecnik upisanog kruga trougla ABC, zatim P , Q, R tacke u kojima upisani
krug dodiruje stranice BC, CA, AB i H ortocentar trougla P QR, dokazati da
tacke O, S, H pripadaju jednoj pravoj, Ojlerovoj pravoj trougla P QR, pri cemu
je tacka H iza S u odnosu na O takva da je OS : SH = r : .
Resenje:
Proucimo najpre opsti slucaj, tj. slucaj kada je trougao ABC nejednakostranican. Ako stranice AB i AC nisu jednake, tacka P je izvan prave AS, pa su
prave P H i AS razlicite. S obzirom da su te dve prave P H i AS razlicite medu
sobom i upravne na istoj pravoj QR, one su medu sobom uporedne. Simetrala
AS ugla A sece pravu ON stranice BC u izvesnoj tacki N koja se nalazi na
krugu l opisanom oko trougla ABC, prema tome i njoj uporedna prava P H
sece prave OS i ON u nekim tackama H i M. Kako su tacke O i P sa raznih
strana prave AS, tacka H je iza S u odnosu na O. Primenom glavnog stava

452

slicnosti, nalazimo da je OS : SH = ON : N M. Kako je ON = r, a kod paralelograma SP M N je N M = SP = , bice OS : SH = r : . Istim postupkom


dokazujemo da i prave kroz tacke Q i R uporedne s pravama BS i CS seku
pravu OS u tacki H . Otuda je H = H , te su tacke O, S, H na jednoj pravoj
i pri tome tacka H iza S u odnosu na O takva da je OS : SH = r : (sl. 407).
A

Q
R
H S O

Slika 407
718. Dokazati da se prave kroz sredista stranica tetivnog cetvorougla upravne
na pravama koje su odredjene naspramnim stranicama seku u jednoj tacki koja
je simetricna sa sredistem opisanog kruga u odnosu na tezishte tog cetvorougla.
Tu tacku nazivamo Kantorovom tackom ili ortotezstem tetivnog cetvorougla.
Resenje:
Obelezimo sa P1 , P2 , P3 , P4 sredista stranica A1 A2 , A2 A3 , A3 A4 , A4 A1 tetivnog cetvorougla A1 A2 A3 A4 , sa O srediste opisanog kruga i sa T teziste tog
cetvorougla. Prava kroz tacku P3 upravna na pravoj A1 A2 sece pravu OT u
izvesnoj tacki K. S obzirom da je tacka T srediste duzi P1 P3 i OP1 k KP3 , bice
tacka T srediste duzi OK, i prema tome tacka K simetricna sa tackom O u
odnosu na tacku T. Istim postupkom dokazuje se da prave kroz sredista ostalih stranica tog cetvorougla upravne na pravama koje su odredene naspramnim
stranicama seku pravu OT u tacki koja je simetricna sa tackom O u odnosu na
tacku T. Ovim je stav dokazan (sl. 408).

453

A4
P3
A3

K
P4
T

P2

A1

P1

A2

Slika 408
719.Neka je A1 A2 A3 A4 cetvorougao upisan u krug l(O, r). Ako su Hij tacke
simetricne sa tackom O u odnosu na prave Ak Al za razlicite vrednosti indeksa
i, j, k, l = 1, 2, 3, 4, dokazati da se prave hij kroz tacke Hij upravne na pravama
Ai Aj seku u jednoj tacki, ortocentru H tog cetvorougla, pri chemu je tacka H
na Ojlerovoj pravoj OT tog cetvorougla takva da je HT : T O = 3 : 1.
Resenje:
Obelezimo sa Tij sredista duzi OHij , a sa tij prave kroz tacke Tij upravne
na pravama Ai Aj . Prema prethodnom zadatku prave tij seku se u izvesnoj tacki
K koja je simetricna s tackom O u odnosu na teziste T tog cetvorougla, pa se
i prave hij seku u izvesnoj tacki H koja je simetricna sa tackom O u odnosu
na tacku K. Otuda sleduje da se tacka H nalazi na Ojlerovoj pravoj OT datog
cetvorougla, pri cemu je HT : T O = 3 : 1 (sl. 409).

454

H12

A4

A3
H

H23
K

H41

T
O

A1

A2

H34

Slika 409
720. Ako je H ortocentar cetvorougla A1 A2 A3 A4 upisanog u krug l(O, r),
dokazati da su ortocentri H1 , H2 , H3 , H4 trouglova A2 A3 A4 , A3 A4 A1 , A4 A1 A2 ,
A1 A2 A3 simetricni sa temenima A1 , A2 , A3 , A4 u odnosu na sredishte S duzi
OH, i prema tome da tacke H1 , H2 , H3 , H4 pripadaju krugu l (H, r) koji je
simetrican sa krugom l u odnosu na tacku S. Tacku S nazivamo sredistem ili
centrom, a pravu s koja je u tacki S upravna na pravoj OH nazivamo sredisnom
ili centralnom pravom tog cetvorougla.
Resenje:
S obzirom da je teziste T1 trougla A2 A3 A4 tacka duzi OH1 takva da je
H1 T1 : T1 O = 2 : 1, a tacka T na duzi A1 T1 takva da je A1 T : T T1 = 3 : 1,
primenom Menelajeve teoreme na trougao A1 H1 O nalazimo da prava OT sadrzi
srediste S1 duzi A1 H1 . Primenom Menelajeve teoreme na trougao OS1 H1 nalazimo da je tacka T srediste duzi OS1 , pa je tacka S1 srediste duzi OH, i prema
tome istovetna sa tackom S. Stoga je tacka H1 simetricna sa temenom A1 u
odnosu na tacku S. Istim postupkom dokazuje se da su i tacke H2 , H3 , H4 simetricne sa temenima A2 , A3 , A4 u odnosu na tacku S.
S obzirom da tacke A1 , A2 , A3 , A4 pripadaju krugu l, i njima simetricne tacke
H1 , H2 , H3 , H4 u odnosu na tacku S pripadaju izvesnom krugu l koji je simetrican sa krugom l u odnosu na tacku S (sl. 410).

455

H12

H2

H1
A4

A3

H
T1
S

H41

T
O
H3
H4
A1

A2

Slika 410
721.Dokazati da se srediste S tetivnog cetvorougla A1 A2 A3 A4 sa upravnim
dijagonalama A1 A3 i A2 A4 poklapa sa presekom dijagonala tog cetvorougla.
Resenje:
Ortocentri H1 i H2 trouglova A2 A3 A4 i A3 A4 A1 su na pravama A1 A3 i A2 A4 ,
pa se presek dijagonala A1 A3 i A2 A4 poklapa s presekom pravih A1 H1 i A2 H2 .
Prema prethodnom zadatku, prave A1 H1 i A2 H2 seku se u tacki S, pa se srediste
S cetvorougla A1 A2 A3 A4 poklapa s presekom dijagonala tog cetvorougla (sl.
411).

456

A3

A4

S
T

Q
O

A1

A2

Slika 411

9.4

Ojlerov krug trougla i


cetvorougla

722. Dokazati da sredista stranica, podnozja visina i sredista duzi koje


spajaju ortocentar sa temenima bilo kojeg trougla pripadaju jednom krugu, Ojlerovom krugu tog trougla.
Uputstvo: Vidi zadatak 152.
723. Dokazati da se srediste Ojlerovog kruga bilo kojeg trougla poklapa sa
sredistem duzi koja spaja ortocentar sa sredistem opisanog kruga tog trougla,
zatim da je poluprecnik tog kruga jednak polovini poluprecnika opisanog kruga.
Uputstvo: Vidi zadatak 153.
724. Ako su Sa , Sb , Sc sredista spolja upisanih krugova trougla ABC, dokazati da je opisani krug trougla ABC Ojlerov krug trougla Sa Sb Sc .
Uputstvo: Tacke A, B, C su podnozja visina trougla Sa Sb Sc , pa je krug koji
sadrzi tacke A, B, C Ojlerov krug trougla Sa Sb Sc .
725. Ako je H ortocentar, T teziste, O srediste opisanog kruga i O srediste
Ojlerovog kruga trougla ABC, dokazati da su tacke T i H harmonijski spregnute sa tackama O i O .
Uputstvo: S obzirom da su tacke T i O izmedu tacaka O i H takve da je
HT : T O = 2 : 1 i HO = O O, bice tacka T izmedu tacaka O iO , a tacka H
iza tacke O u odnosu na O, pri cemu je OT : T O = OH : HO = 2 : 1, pa su
tacke T i H harmonijski spregnute s tackama O i O .
457

726. Ako je H ortocentar i T teziste trougla ABC, a l opisani krug i l Ojlerov krug tog trougla,dokazati da je tacka T unutrasnje, a tacka H spoljasnje
srediste slicnosti krugova l i l .
Uputstvo: S obzirom da su tacke A, B, C na krugu l, a sredista A , B , C duzi
HA, HB, HC na krugu l , tacka H je spoljasnje srediste slicnosti krugova l i l .
No tacke H i T su harmonijski spregnute sa sredistima O i O krugova l i l , pa
je T unutrasnje srediste slicnosti tih dvaju krugova.
727. Ako su A , B , C sredista stranica BC, CA, AB i S, Sa , Sb , Sc sredista
upisanih krugova trougla ABC, dokazati da se sredista Ojlerovih krugova trouglova SBC i Sa BC nalaze na simetrali unutrasnjeg ugla A , a sredista Ojlerovih
krugova trougla Sb BC i Sc BC na simetrali spoljasnjeg ugla A trougla A B C .
Uputstvo: Srediste opisanog kruga trougla SBC je tacka N u kojoj simetrala
unutrasnjeg ugla A sece opisani krug trougla ABC. Ako obelezimo sa H ortocentar trougla SBC i sa K srediste duzi SH, prema poznatom stavu bice N A
i SK jednake i istosmerne, pa je cetvorougao SN A K paralelogram, i prema
tome A K k SN . S obzirom da je tacka K srediste stranice SH, a prava KA
uporedna sa stranicom SN trougla HSN bice srediste E duzi HNna pravoj
A K, tj. strediste E Ojlerovog kruga trougla SBC na simetrali A K unutrasnjeg
ugla A trougla A B C . Analognim postupkom dokazuju se ostali delovi ovog
stava (sl.412).

A
H

K
C

B
S

Sa

sl.412

458

728. Ako je H ortocentar, O srediste Ojlerovog kruga i r poluprecnik opisanog kruga trougla ABC, dokazati da je
O A2 + O B 2 + O C 2 + O H 2 = 3r2 .
Uputstvo:Ovaj zadatak vec smo resavali na jedan nacin primenom Lajbnicove
teoreme (vidi zadatak 312), ovde navodimo drugi nacin. Ako su A , B , C podnozja visina trougla ABC, bice tacka O srediste opisanog kruga, tacka H
srediste upisanog kruga, dok su tacke A, B, C sredista spolja upisanih krugova
trougla A B C , pa je prema zadacima 271. i 153.
O A2 + O B 2 + O C 2 + O H 2 = 3r2 .

729. Ako su a, b, c stranice trougla ABC, a p(M ) potencija tacke M u


odnosu na Ojlerov krug tog trougla, dokazati da je
p(A) + p(B) + p(C) =

a2 + b 2 + c2
.
4

730. Ako su H i S ortocentar i srediste cetvorougla A1 A2 A3 A4 upisanog u


krug (O, r), dokazati da sredista S1 , S2 , S3 , S4 duzi koje spajaju tacku O sa
ortocentrima H1 , H2 , H3 , H4 trouglova A2 A3 A4 , A3 A4 A1 , A4 A1 A2 , A1 A2 A3 ,
podnozja B1 , B2 , B3 , B4 upravnih iz temena A1 , A2 , A3 , A4 sredisnjim pravama
tih trouglova i sredista C1 , C2 , C3 , C4 duzi HA1 , HA2 , HA3 , HA4 pripadaju
krugu k(S, r2 ), Ojlerovom krugu tetivnog cetvorougla A1 A2 A3 A4 .
9.5. Nagelova tacka trougla
Ako su Pa , Qb , Rc tacke u kojima spolja upisani krugovi dodiruju stranice
BC, CA, AB trougla ABC, tada se, prema zadatku ..., duzi APa , BQb , CRc
seku u jednoj tacki koju nazivamo Nagelovom tackom trougla.
Uputstvo: Prema zadatku 720, tacke H1 , H2 , H3 , H4 pripadaju krugu k (H, r),
te i sredista S1 , S2 , S3 , S4 duzi OH1 , OH2 , OH3 , OH4 pripadaju izvesnom krugu
k kome je srediste istovetno sa sredistem duzi OH, tj. sa tackom S, a poluprecnik jednak polovini duzi r. S obzirom da tacke S1 , S2 , S3 , S4 pripadaju krugu k, i njima simetricne tacke C1 , C2 , C3 , C4 u odnosu na srediste S
toga kruga pripadaju tome krugu. Prave A1 B1 , A2 B2 , A3 B3 , A4 B4 simetricne
su sa pravama OH1 , OH2 , OH3 , OH4 u odnosu na tacku S te sadrze tacke
C1 , C2 , C3 , C4 . S obzirom da su duzi S1 C1 , S2 C2 , S3 C3 , S4 C4 precnici kruga
k, a uglovi S1 B1 C1 ,
S2 B2 C2 ,S3 B3 C3 ,S4 B4 C4 pravi, tacke B1 , B2 , B3 , B4 takode pripadaju krugu k.
Ovim je stav dokazan (sl 413).

459

H2

H1

A4
A3

B2
C4
B1
C
S2
S1 3
H
S

B3
C1
H3

B4

S3

C2
S4
H4

A1

A2

sl.413

9.5

Nagelova ta
cka trougla

Ako su Pa , Qb , Rc tacke u kojimaa spolja upisani krugovi dodiruju stranice


BC, CA, AB, tada se duzi APa , BQb , CRc seku u jednoj tacki koju nazivamo
Nagelova tacka trougla.
731. Ako su A , B , C sredista stranica BC, CA, AB trougla ABC, dokazati
da je srediste kruga upisanog u trougao ABC Nagelova tacka trougla A B C .
Uputstvo: Obelezimo sa S srediste upisanog kruga k, sa Sa srediste spolja
upisanog kruga ka koji odgovara temenu A, sa P i Pa tacke u kojima krugovi
k i ka dodiruju stranicu BC, sa D tacku u kojoj simetrala AS ugla BAC sece
stranicu BC i sa Q tacku u kojoj prava P S sece pravu APa . Pri tome je
ASa Pa ASQ i DSa Pa DSP , pa je Sa A : SA = Sa Pa : SQ i Sa D :
SD = Sa Pa : SP . Prema poznatom stavu imamo da je Sa A : SA = Sa D : SD,
pa je Sa Pa : SQ = Sa Pa : SP i prema tome SP = SQ. S obzirom da je Q iza
S u odnosu na P i SP = SQ, tacka S je srediste duzi P Q, a duz P Q precnik
kruga k. Kako je tacka S srediste duzi P Q, a prema zadatku 154, tacka A
srediste duzi P Pa , bice QPa k A S i prema tome APa k A S.
Otud sledi da u homotetijskom preslikavanju trouglova ABC i A B C , pravoj
APa odgovara prava A S. Istim postupkom dokazuje se da i prave B S i C S

460

u tom preslikavanju odgovaraju analognim pravama kroz temena B i C, prema


tome, tacka S je Nagelova tacka trougla A B C (sl. 414).
A

Q
C

Pa
B

Sa

sl.414

732. Dokazati da Nagelova tacka N , teziste T i srediste S upisanog kruga


trougla ABC pripadaju jednoj pravoj, pri cemu je tacka T izmedu tacaka N i
S takva da je N T : T S = 2 : 1.
Uputstvo: Obelezimo sa A1 srediste stranice BC, sa D podnozje visine iz temena
A, sa P tacku u kojoj upisani krug dodiruje stranicu BC i sa Pa tacku u kojoj
spolja upisani krug dodiruje stranicu BC trougla ABC. Pri tome trouglovi
ADPa i SP A1 imaju odgovaraju`ce paralelne stranice, dakle slicni su, pa je
2p
a i
APa : SA1 = AD : SP . Iz ove relacije i ovih relacija AD : SP =
:
p
APa = AN (v.z.375) sledi da je AN :SA1 =2:1. S obzirom da je tacka T
a
izmedju tacaka A i A1 takve da je AT : T A1 =2:1, a N tacka s one strane prave
AA1 s koje nije tacka S takva da je AN ||SA1 i AN : SA1 =2:1, bi`ce tacka T
izmedu tacaka N i S takva da je N T : T S=2:1(sl.415).

461

sl. 415
733. Ako je O srediste opisanog kruga, S srediste upisanog kruga, H ortocentar i N Nagelova tacka trougla ABC, dokazati da je HN kOS i HN = 2OS.
B
C
caka
Uputstvo: Prema zadacima 145.
i 732. tezisDte TPtrougla
A1 ABC
P a je izmedu ta
O i H izmedu tacaka N i S pri cemu je HT : T O=2:1 i N T : T S=2:1, pa je
HN ||OS i HN : OS=2:1.

462

9.6

Spikerov krug trougla

Definicija 9.6. Krug opisan u medijalni trougao A1 B1 C1 trougla ABC nazivamo Spikerovim krugom trougla ABC.
734. Dokazati da se srediste Spikerovog kruga trougla ABC poklapa sa
sredistem duzi koja spaja srediste S upisanog kruga s Nagelovom tackom N tog
trougla.
Uputstvo: Obelezavamo sa A1 , B1 , C1 sredista stranice BC, CA, AB, sa T teziste
trougla ABC i sa S1 srediste duzi SN . S obzirom da su tacke T i S1 izmedu
tacaka N i S takve da je N T : T S=2:1 i tacke S i S1 takveda je ST : T S1 =2:1.
Tacka T je takode izmedu tacaka A i A1 takve da je AT : T A1 =2:1, pa su
duzi AS i A1 S1 madu sobom uporedne. Stoga je prava A1 S1 simetrala ugla A1
trougla A1 B1 C1 . Isto tako, prava B1 S1 je simetrala ugla B1 trougla A1 B1 C1 ,
pa je tacka S1 srediste upisanog kruga trougla A1 B1 C1 , tj. srediste Spikerovog
kruga trougla ABC (sl. 416).

463

C1

B1
S

sl. 416

S1
N

735. Ako je S srediste upisanog kruga k, S1 srediste Spikerovog kruga k1 , T


teziste i N Nagelova tacka trougla ABC, dokazati da su tacke T i N harmonijski
spregnute s tackama S i S1 , B
zatim da su tacke T i N sredista slicnosti krugova
C
P
A1
Pa
k i k1 .
Uputstvo: Prema zadacima 732 i 734 tacka T je izmedu tacaka S i S1 takva da
je ST : T S1 =2:1, a tacka N iza S1 u odnosu na S takav da je SN : S1 N =2:1.
Stoga su tacke T i N harmoniski spregnute sa tackama S i S1 . Ako su A1 B1 C1
srediste stranica BC, CA, AB trougla ABC, tacka T je srediste slicnosti trouglova ABC i A1 B1 C1 , dakle i krugova k i k1 upisanim u tim touglovima.
Sobzirom da su tacke T i N harmoniski spregnute sa tackama S i S1 , a tacka
464

T unutrasnje srediste slicnosti trouglova k i k1 , je spoljasnje srediste slicnosti


krugovak i k1 .

9.7

Furmanov krug trougla

Definicija 9.7. Krug kome je precnik duz odredena ortocentrom H i Nagelovom tackom N trougla ABC nazivamo Furmanovim krugom trougla ABC.
736. Ako su X, Y , Z tacke u kojima simetrale unutrasnjih uglova A, B,
C seku opisani krug trougla ABC, dokazati da se tacke X , Y , Z simetricne
s tackama X, Y , Z u odnosu na prave BC, CA, AB nalaze na Furmanovom
krugu trougla ABC.
Uputstvo: Obelezavamo sa H ortocentar, sa N Nagelovu tacku i sa k Furmanovkrug trougla ABC. Ako je M tacka u kojoj prava XX sece opisani
krug trougla ABC, bice cetvorougao AHX M paralelogram, pa je AM ||HX .
Obelezimo sa S srediste opisanog kruga trougla ABC, sa A1 srediste stranice
BC i sa N tacku u kojoj se seku prave AS i A1 N . Prema zadacima 145 i 732
imamo da je AN ||A1 S i AN = 2SA1 pa je tacka A1 srediste duzi N N , i prema
tomeXN ||N X . S obzirom da je AM ||HX i HN ||N X , a AM XN bice
XH N X pa je tacka X na krugu kome je duz HN precnik , tj. na Furmanovom krugu trougla ABC. Na isti nacin dokazuje se da i tacke Y i Z pripadaju
tom krugu(sl. 417).

H
S

Qa

N
A
B

A1

Pa

sl. 417
737. Ako su X, Y , Z tacke u kojima simetrale unutrasnjih uglova A, B, C
seku opisani krug trougla ABC, a X , Y , Z njima simetricne tacke u odnosu
na prave BC, CA, AB, dokazati da su trouglovi XY Z i X Y Z inverzno slicni.
Uputstvo: Prema prethodnom zadatku tacke x , y , z i ortocentar H trougla
465

ABC pripadaju jednom krugu . Pri tome je npr. y x z = y hz . No Hy ||xz


i HZ ||XY , pa je Y X Z = XY Z, i td.
738. Dokazati da Furmanov krug trougla ABC sece prave odredene visinama
AA , BB , CC u tackama A , B , C takvim da je AA = BB = CC = 2,
gde je poluprecnik upisanog kruga tog trougla.
Uputstvo: Duz odredena ortocentrom H i Nagelovom tackom N je precnik Furmanovog kruga trougla ABC, pa je ugao HA N prav, i prema tome N A ||BC.
Ako je Pa tacka u kojoj prava AN sece stranicu BC, bice AA : AA = AN :
APa . No AN : APa = a : p, pa je AA : AA = a : p. S druge strane imamo
2p
...(2). Iz jednakosti (1) i (2) sledi da je
da je a AA = 2p, pa je AA =
a
2p
a
a
AA = AA =
= 2.
p
p a
739. Ako su A , B , C tacke u kojima Furmanov krug sece prave odredene
visinama AA , BB , CC trougla ABC, dokazati da su trouglovi ABC i A B C
inverzno slicni.
Uputstvo: S obzirom da je Nagelova tacka N na Furmanovom krugu trougla
ABC, bice npr. ugao B C jednak uglu B N C . No N B ||AC i N C ||AB,
pa je ugao B A C jednak uglu BAC. itd.

9.8

Izometri
cke ta
cke u odnosu na du
z i na trougao

Definicija 9.7. Dve tacke P i P prave koja sadrzi neku duz AB, simetricne
medu sobom u odnosu na srediste O te duzi nazivamo izotomicki spregnutim ili
samo izotomickim tackama u odnosu na tu duz.
740. Ako su P , Q, R tacke pravih koje su odredene stranicama BC, CA,
AB trougla ABC, a P , Q , R njima izometricki spregnute tacke u odnosu na
odgovarajuce stranice tog trougla, i ako se pri tome prave AP , BQ, CR seku
u jednoj tacki O, dokazati da se i prave AP , BQ , CR takode seku u nekoj
tacki O . Tacke O i O nazivamo izotomicki spregnutim, ili samo izotomickim u
odnosu na trougao ABC.

Uputstvo:Primeniti Cevinu
teoremu.
741. Ako su P , Q, R tacke u kojima neka prava sece prave odredene stranicama BC, CA, AB trougla ABC, dokazati da i njima izotomicki spregnute
tacke u odnosu na odgovarajuce stranice takode pripadaju jednoj pravoj.
Uputstvo:Primeniti Menelajevu teoremu.
742. Prave kroz temena trougla ABC uporedne s naspramnim stranicama
odreduju izvestan trougao A B C . Ako je K srediste kruga upisanog u trougao
A B C , a K tacka izotomicki spregnuta s tackom K u odnosu na trougao
A B C , dokazati da trougao ABC odseca na pravama, koje sadrze tacku K, a
uporedne su sa stranicama tog trougla, jednake odsecke.
Uputstvo: Tacku D iD ukojima prave A K i A K seku stranicu B C trougla
A B C su izotomicke u odnosu natu stranicu. Ako obelezimo sa L tacku u kojoj
prava A K sece stranicu BC trougla ABC, bice BL : LC = C D : DB = B D :
D C = A B : A C = A C : A B = AB : AC,pa je L tacka u kojoj simetrala
466

ugla A sace stranicu BC trougla ABC. Neka su M i N tacka stranica AB i AC


takve da je cetvorougao AM LN paralelogram. S obzirom da diagonala AL toga
paralelograma razlaze ugao A na dva jednaka ugla, bice to romb, LM = LN .
Ako su prave kroz tacku K uporedne sa stranicama BC, CA, AB seku druge dve
stranice trougla ABC, respektivno u tackama P1 i Q1 ,P2 i Q2 ,P3 i Q3 imamo
da je P2 Q2 : LM = BQ2 : BM = A K : A L = CP3 : CN = P3 Q3 : LN . No
LM = LN , pa je i P2 Q2 = P3 Q3 . Isto tako je i P3 Q3 = P1 Q1 , pa su odsecci P1
i Q1 P1 , Q2 P2 i P3 Q3 medusobnom jednaki. (sl. 418)

D A

P3
N

Q2
P1
C

K
Q3

L P2

sl. 418
A

467

Q1
B

9.9

Izogonalne prave u odnosu na ugao. Izogonalne ta


cke
u odnosu na trougao.

Definicija 9.8. Dve prave kroz teme nekog ugla simetricne medu sobom u
odnosu na simetralu tog ugla nazivamo izogonalno spregnutim pravama, ili samo
izogonalnim pravama u odnosu na taj ugao. Tako je npr. prava koja sadrzi visinu
AA1 trougla ABC izogonalna s pravom koja sadrzi poluprecnik OA opisanog
kruga tog trougla u odnosu na ugao A. Simetrala jednog ugla izogonalna je
samoj sebi u odnosu na taj ugao.
743. Ako su m i n dve izogonalne prave u odnosu na ugao P OQ, a A i
B podnozja upravnih iz proizvoljne tacke M prave m na pravama OP i OQ,
dokazati da je n AB.
Uputstvo: Obelezimo sa K tacku u kojoj prava n sece pravu P Q. S obzirom
da je cetvorougao OAM B tetivan, imamo da je AM B = AOM = KOB.
Otuda i iz M B OB sledi da je OK AB, pa je n AB (sl.419).
O

N
D

B
A
M

sl.419
744. Ako su M i N proizvoljne tacke dveju pravih izogonalnih u odnosu na
ugao P OQ, dokazati da su odstojanja tacke M od pravih OP i OQ obrnuto
proporcionalna odstojanjima tacke N od pravih OP i OQ.
Uputstvo: Obelezimo sa A i B podnozja upravnih iz tacke M na pravama OP i
OQ, a sa C i D podnozja upravnih iz tacke N na pravama OQ i OP . Pri tome
je OAM OCN i OBM ODN , pa je M A : N C = OM : ON =
M B : N D i prema tome M A : M B = M C : N D.
745. Ako su M i N tacke u ravni ugla P OQ takve da su odstojanja tacke
M od pravih OP i OQ obrnuto proporcionalna odstojanjima tacke N od pravih
OP i OQ, dokazati da su prave OM i ON izogonalne u odnosu na ugao P OQ.

468

Uputstvo: Koristeci oznake uvedene pri resavanju predhodnog zadatka, imamo


da je M A : M B = N C : N D i AM B = CN D, pa je AM B CN D, i
prema tome ABM = N DC. No cetvorougli OAM B i OCN D su tetivni ,
pa je ABM = AOM i N DC = N OC. Stoga je AOM = N OC, pa su
prave OM i ON izogonalne u odnosu na P OQ.
746. Ako su M i N proizvoljne tacke dveju pravih koje su izogonalne u odnosu na ugao P OQ, dokazati da podnozja upravnih iz tacaka M i N na pravama OP i OQ pripadaju jednom krugu, kome se srediste poklapa sa sredistem
duzi M N .
Uputstvo: Koristeci oznake pri resavanju prethodnog zadatka 743(sl. 419),
imamo da je OAM OCN i OBM ODN , pa je OA : OC = OM :
ON = OB : OD. Stoga je OA OD = OB OC, pa su tackeA, B, C, D na
jednom krugu. Simetrale tetiva AD i BC toga kruga seku se u sredistu duzi
M N , koja je dakle i srediste toga kruga.
747. Ako je O proizvoljna tacka u ravni trougla ABC, dokazati da se prave
izogonalne s pravama OA, OB, OC respektivno u odnosu na A, B, C, seku u
jednoj tacki O , ili su medu sobom uporedne. Tacke O i O nazivamo izogonalno
spregnutim ili samo izogonalnim u odnosu na trougao ABC.
Uputstvo: Predpostavimo najpre da se prave izogonalne pravama OAi OB u
odnosu na uglove A i B seku u nekoj tacki O . Ako su P , Q, R podnozja
upravnih iz tacke O , a P , Q , R podnozja upravnih iz tacke O na pravama
BC, CA, AB, prema zadatku 744, imamo da je OQ : OR = O R : O Q
i OR : OP = O P : O R . Mnozenjem odgovarajucih strana ovih proporcija,
nalazimo da je OQ : OP = O P : O Q . Stoga su prema zadatku 745, prave OC
i O C izogonalne u odnosu na ugao C. Ako bi prave izogonalne s pravama OA
i OH u odnosu na uglove A i B bile uporedno bice i prava izogonalna s pravom
OC u odnosu na ugao C njima uporedna. Dokaz se izvodi indirektno(sl. 420).
Ortocentar i sredista opisanog kruga trougla predstavljaju primer izogonalnih
tacaka u odnosu na taj trougao.

469

R
O
B

sl.420
748. Ako su O i O dve tacke izogonalne u odnosu na trougao ABC, dokazati da podnozja upravnih iz tacaka O i O na pravama BC, CA, AB pripadaju
jednom krugu, kome se srediste poklapa sa sredistem duzi OO . Taj krug nazivamo pedalnim krugom dveju tacaka izogonalnih u odnosu na trougao ABC.
Uputstvo: Koristeci oznake uvedene pri resavanju zadataka 747 (sl. 420), prema
zadatku 746, sleduje da tacke Q, Q , R, R , pripadaju jednom krugu kome se
srediste poklapa sa sredistem duzi OO . Isto tako tacke R, R , P , P pripadaju
jednom krugu kome se srediste takodje poklapa sa sredistem duzi OO . Stoga
svih sest tacaka P , P , Q, Q , R, R pripadaju jednom krugu kome se srediste
poklapa sa sredistem duzi OO .
749. Ako neki krug k sece stranice BC, CA, AB trougla ABC u tackama
P i P , Q i Q , R i R , pri cemu se normale u tackama P , Q, R na stranicama
BC, CA, AB seku u jednoj tacki O, dokazati da se i normale u tackama P , Q ,
R takode seku u jednoj tacki O , zatim da su tacke O i O izogonalne u odnosu
na trougao ABC.
Uputstvo: Obelezimo sa S srediste kruga k. Normale u tackama P , Q , R
respektivno sa stranicama BC, CA, AB seku pravu OS u tacki O koja je simetricna s tackom O u odnosu na tacku S, prema tome, one se seku i medju sobom
u toj tacki. Indirektnim postupkom dokazuje se da su tecke O i O izogonalne
u odnosu na trougao ABC.
750. Ako su O i O dve tacke izogonalne u odnosu na trougao ABC, a A ,
B , C , tacke simetricne s tackom O u odnosu na prave BC, CA, AB, dokazati
da je tacka O srediste kruga opisanog oko trougla A B C .
Uputstvo: Obelezimo sa P , Q, R podnozja upravnih iz tecke O na pravama
BC, CA, AB i sa S srediste duzi OO . Prema zadatku 748, teke P , Q, R su
na krugu kome je srediste S. Iz trougla OO A , OO B , OO C , nalazimo da je
O A = 2SP , O B = 2SQ, O C = 2SR. Otuda i iz jednakosti SP = SQ = SR

470

sledi da je O A = O B = O C , pa je tacka O srediste kruga opisanog oko


trougla A B C .

9.10

Lemoanova ta
cka i Lemoanova prava trougla

Definicija 9.9. Ako je duz AA medijana iz temena A trougla ABC, a A


tacka u kojoj prava simetricna s pravom AA u odnosu na simetralu unutrasnjeg
ugla A sece stranicu BC, kazemo da je duz AA unutrasnja simedijana ili samo
simedijana iz temena A trougla ABC. Ako je T tacka u kojoj dirka kroz tacku
A kruga opisanog oko trougla ABC sece pravu BC, kazemo da je duz AT spoljasnja simedijana iz temena A trougla ABC. Specijalno, ako je ugao A trougla ABC prav, simedijana iz temena A poklapa se s visinom iz tog istog temena.
751. Dokazati da prava odredena simedijanom AA trougla ABC sadrzi pol
A0 prave BC u odnosu na krug opisan oko trougla ABC.
Uputstvo: Ako su M i N tacke u kojima simetrale spoljasnjeg i unutrasnjeg
ugla A seku opisan krug trougala ABC, bice prave AM i AN simetrale i uglova
koje odredjuju prave AA i AA , pa su prave AM i AN harmonijski spregnute
s pravama AA i AA . S druge strane, tacke M i N su harmonjiski spregnute
sa A i A0 . S druge strane, tacke M i N su harmonijski spregnute sa tackama
A i AA0 , pa su i prave AM i AN harmonijski spregnute s pravama AA i
AA0 . Stoga su prave AA i AA0 istovetne, pa je tacka A0 na pravoj AA .
Napominjemo da se ovaj stav moze iskazati i na sledeci nacin. Dokazati da se
prave odredjene spoljasnjim simedijanama iz temena B i C i prava odredjena
untrasnjom simedijanom iz temena A trougla ABC, seku u jednoj tacki (sl.421).
M

A D

A0

sl.421

471

752. Ako su AA i AT unutrasnja i spoljasnja simedijana trougla ABC,


dokazati da su prave AA i AT harmonijski spregnute s pravama AB i AC.
Uputstvo: Prema predhodnom zadatku, prava AA sadrzi pol A0 prave BC u
odnosu na krug ABC , te i prava BC sadrzi pol prave AA . Kako se taj pol nalazi i na pravoj koja u tacki A dodiruje krug ABC, pol prave AA u odnosu na
krug ABC je tacka T . Stoga su tacke A i T harmonijske spregnute s tackama
B i C, pa su i prave AA i AT harmonijski spregnute s pravama AB i AC.
753. Ako je AA simedijana iz temena A trougla ABC, dokazati da je
BA : A C = AB 2 : AC 2 .
Uputstvo: Ako je AT spoljasnja simedijana iz temena A trougla ABC, prema
predhodnom zadatku, imamo da su tacke A i T harmonijski spregnute s tackama
B i C. Prilikom resavanja zadatka 338 imali smo da je BT : T C = AB 2 : AC 2 ,
pa je BA : A C = AB 2 : AC 2 .
754. Dokazati da se simedijane AA , BB , CC trougla ABC seku u jednoj
tacki simedijalnoj ili Lemoanovoj tacki trougla ABC.

Slika 754-757
Prema prethodnom zadatku, imamo da je
BA : A C = AB 2 : AC 2 , CB : B A = BC 2 : AB 2 , AC : C B = AC 2 : BC 2 .
BA CB AC

teoremi prave AA , BB ,
Stoga je = 1, pa se saglasno Cevinoj
A
C
B
A
C
B
CC seku u jednoj tacki, koju nazivamo simedijalnom ili Lemoanovom tackom
trougla ABC.

472

755. Ako je L Lemoanova tacka trougla ABC, tj. tacka u kojoj se seku
simedijane AA , BB , CC , dokazati da je:
AL : LA = (AB 2 + AC 2 ) : BC 2
.
Prema Van Obelovoj teoremi i zadatku 753 nalazimo da je
AC
AB 2
AB 2 + AC 2
AB
AC 2
AL
=
+
=
.(Sl.754-757).
+
=
LA
C B
BC
BC 2
BC 2
BC 2
756. Dokazati da su odstojanja Lemoanove tacke trougla od pravih koje
su odredjene stranicama tog trougla srazmerne njegovim odgovarajucim stranicama.
Obelezimo sa AA simedijanu trougla ABC, a sa M i N podnozja upravnih
i tacke A na pravama AB i AC. Pri tome je AB : M A = AC : N A = BA :
CA = AB 2 : AC 2 , pa je M A : N A = AB : AC. S obzirom da svaka tacka
prave AA ima takvu osobinu, i odstojanja Lemoanove tacke L trougla ABC
od pravih AB i AC srazmerna su stranicama AB i AC. Analogno se dokazuje
da su i odstojanja tacke L od pravih AB i BC srazmerna stranicama AB i AC
(Sl.754-757).
757. Ako su odstojanja tacke L od pravih koje su odredjene stranicama trougla ABC srazmerna odgovarajucim stranicama, dokazati da je L Lemoanova
tacka tog trougla.
Ako je A tacka u kojoj se seku prave AL i BC, bice odstojanja tacke L od
pravih AB i AC srazmerna odstojanjima tacke A od pravih AB i AC. Stoga je
AA simedijana iz temena A trougla ABC. Isto tako je tacka L na pravoj koja
sadrzi simedijanu iz temena B , te je L Lemoanova tacka trougla ABC(Sl.754757).
758. Ako su P , Q, R tacke u kojima upisani krug dodiruje stranice BC, CA,

AB trougla ABC, dokazati da je Zergonova


tacka G trougla ABC Lemoanova
tacka trougla P QR.

473

R
G

Slika 758
Prava P G sadrzi teme P i pol A prave koja je odredena stranicom QR u odnosu
na opisani krug trougla P QR. Stoga, prava P Q sadrzi simedijanu iz temena
P trougla P QR. Isto tako prava QG sadrzi simedijanu iz temena Q, pa je G
Lemoanova tacka trougla P QR.
759. Dokazati da se Lemoanova tacka pravouglog trougla poklapa sa sredistem
visine koja odgovara hipotenuzi tog trougla.

Slika 759
Ako je ugao kod temena A trougla ABC prav, simedijana AA iz temena A
poklapa se sa visinom iz istog temena. Oznacimo li sa L Lemoanovu tacku tog
474

trougla, prema zadatku 755, imamo da je AL : LA = (AB 2 + AC 2 ) : BC 2 = 1,


pa je AL = LA .
760. Ako su P , Q, R podnozja upravnih iz Lemoanove tacke L na stranicama BC, CA, AB trougla ABC, dokazati da je tacka L teziste trougla P QR.

A
P
Q
R
L

Slika 760
Ako obelezimo sa P tacku u kojoj prava kroz tacku Q uporedno sa pravom LR sece pravu LP , bice odgovarajuce stranice trouglova ABC i QP L
proporcionalne. Stoga je ABC QP L,pa je LQ : AC = QP : AB. No
kako je LQ : AC = LR : AB, to je QP : AB = LR : AB i, prema tome,
QP = LR. Otuda sleduje da je cetvorougao LQP R paralelogram, te prava
P L sadrzi srediste duzi QR. Isto tako, prava LQ sardzi srediste duzi RP , pa je
tacka L teziste trougla P QR.
761. Dokazati da je Lemoanova tacka L trougla ABC pol Lemoanove prave
s(v.3...) u odnosu na opisani krug tog trougla.
Obelezimo sa AA , BB i CC unutrasnje simedijane i sa ATA , BTB i CTC
spoljasnje simedijane trougla ABC. Prema zadatku 338, tacke TA , TB i TC pripadaju jednoj pravoj , Lemoanovoj pravoj trougla ABC. Pri resavanju zadatka
751 dokazali smo da je tacka TA pol prave AA u odnosu na krug ABC. Isto
tako, tacka TB je pol prave BB , a tacka TC pol prave CC . Stoga je tacka L
u kojoj se seku prave AA , BB i CC pol prave s koja sadrzi tacke TA , TB i TC .
762. Ako je L Lemoanova tacka i O srediste opisanog kruga trougla ABC,
dokazati da je prava OL upravna na Lemoanovoj pravoj s tog trougla.

475

Prema prethodnom zadatku, tacka L je pol prave s u odnosu na opisani krug


trougla ABC, pa je OLs.
763. Ako su AA i AA medijana i simedijana iz temena A trougla ABC.
Ako su Q i R podnozja upravnih iz proizvoljne tacke P medijane AA na stranicama AB i AC, dokazati da je QR AA , i obrnuto, ako su Q i R podnozja
upravnih iz proizvoljne tacke P simedijane AA na stranicama AB i AC, dokazati da je Q R AA . Najzad, dokazati da je P QR P R Q .

A
R
Q
P
R
Q

Slika 763
S obzirom da su naspramni uglovi Q i R cetvorougla AQP R pravi, taj
cetvorougao je tetivan, pa je ugao P QR jednak i istosmeran sa uglom P AR.
No ugao P AR je jednak i istosmeran sa uglom BAA , te su uglovi P QR i
BAA jednaki i istosmerni. Otuda iz QP AB sledi da je QRAA . Na isti
nacin dokazuje se da je Q R AA . Kod trouglova P QR i P Q R uglovi P i P
su suplementni sa uglom A trougla ABC, dakle, medusobno jednaki. Sem toga
je P QR = P AR = Q AP = Q R P . Stoga je P QR P R Q .
764. Ako simedijana AA koja odgovara temenu A trougla ABC sadrzi
sredishte duzi M N kojoj su krajevi na stranicama AB i AC, dokazati da je
cetvorougao BCM N tetivan.

476

A
N

Slika 764
Ako je AT prava uporedna s pravom M N , prave AA i AT bice harmonijski
spregnute sa pravama AB i AC, pa je prema prethodnoj teoremi prava AT dirka
u tacki A kruga opisanog oko trougla ABC. Pretpostavimo li da je tacka T s
one strane prave AB sa koje nisu tacke O i N , imamo da je T AB = AM N
i T AB = ACB. Otuda je AM N = ACB, pa je cetvorougao BCN M
tetivan, a prave BC i M N antiparalelne u odnosu na prave AB i AC.
765. Ako su M i N tacke stranica AB i AC trougla ABC takve da je
cetvorougao BCN M tetivan, dokazati da se srediste K duzi M N nalazi na simedijani koja odgovara temenu A trougla ABC.

477

A
N
K
M

Slika 765
Duz M N uporedna je s pravom AT koja u temenu A dodiruje krug opisan
oko trougla ABC. S obzirom da su prava AT i prava koja sadrzi simedijanu
AA harmonijski spregnute s pravama AB i AC, prava AA sadrzi srediste K
duzi M N.
766. Neka su P i Q tacke stranica AB i BC, a R i S tacke stranica AC i
BC takve da su cetvorouglovi AP QC i ARSB tetivni. Ako su pri tome duzi
P Q i RS jednake, dokazati da se prave P Q i RS seku na pravoj koja sadrzi
simedijanu iz temena A trougla ABC, ili su uporedne s tom pravom.

A
R

P
B

C
Q

Slika 766-767
Ako pretpostavimo da se prave P Q i RS seku u nekoj tacki V , bice [P QV ]
i [RSV ], ili [P V Q] i [RV S], ili [V P Q] i [V RS]. U svakom od tih slucajeva, bice
V QC = V SQ, pa je V Q = V S. Stoga je i V P = V R. Ako je V tacka u
478

kojoj prava AV sece pravu BC, P tacka prave AB takva da je V P k V P i R


tacka prave AC takva da je V R = V R , bice
V P : V P = AV : AV = V R : V R pa je V P : V R = V P : V R .
No V P = V R, pa je V P = V R . S obzirom da je V BP ABC i
V R C ABC, bice BV : V P = AB : AC i V R : V C = AB : AC.
Mnozenjem odgovarajucih strana ovih dveju jednakosti i koristeci jednakost
V P = V R nalazimo da je BV : V C = AB 2 : AC 2 , pa je AV simedijana
trougla ABC. Prema tome presecna tacka V pravih P Q i RS je na pravoj koja
sadrzi simedijanu iz temena A trougla ABC. Ako su prave P Q i RS medu sobom uporedne, one su upravne na pravoj BC. U tom slucaju ugao A trougla
ABC je prav, pa se simedijana iz temena A poklapa s visinom iz tog temena.
Stoga je simedijana iz temena A uporedna s duzima P Q i RS
767. Ako su P i Q tacke stranica AB i BC, a R i S tacke stranica AC i BC
takve da su cetvorouglovi AP QC i ARSB tetivni. Ako se pri tome prave P Q i
RS seku na pravoj, koja sadrzi simedijanu iz temena A trougla ABC. Dokazati
da je P Q = RS.
Neka je presecna tacka V pravih P Q i RS na pravoj koja sadrzi simedijanu AV trougla ABC. Ako cu P i R tacke pravih AB i AC takve da je
V P k V P i V R k V R bice V BP ABC i V R C ABC,pa je
BV : V P = AB : AC i V R : V C = AB : AC.
Mnozenjem odgovarajucih strana ovih jednakosti, nalazimo da je za
(BV : V C) : (V P : V R ) = AB 2 : AC 2 . No BV : V C = AB 2 : AC 2 , pa je
V P : V R = 1, tj. V P = V R . S obzirom da je V P : V P = AV : AV =
V R : V R , bice V P : V R = V P : V R . No V P = V R , pa je V P = V R.
Kod trougla V QS uglovi Q i S su jednaki medu sobom, pa je V Q = V S. Kako je
[P QS] i [RSV ], ili [P V Q] i [RV S], [V P Q] i [V RS], zatim V P = V R i V Q = V S
bice P Q = RS.(Sl.766-767).

9.11

Lemoanovi krugovi trougla

768. Dokazati da prave kroz Lemoanovu tacku trougla uporedne sa stranicama seku taj trougao u tackama koje pripadaju jednom krugu, prvom Lemoanovom krugu tog trougla.

479

A
P3
K

Q2
P1

O1

Q1
O

C
Q3

P2

Slika 768-772
Neka prava kroz Lemoanovu tacku L uporedne sa stranicama trougla ABC
seku taj trougao u tackama P1 , Q1 ; P2 , Q2 ; P3 , Q3 . S obzirom da je cetvorougao
AQ2 LP3 paralelogram, prava AL odredena simedijanom iz temena A sadrzi
srediste duzi Q2 P3 kojoj su krajevi na stranicama AB i AC, pa je prema zadatku
655, cetvorougao BCP3 Q2 , dakle i cetvorugao P1 Q1 P3 Q2 tetivan. Na isti nacin
dokazuje se da je i cetvorougao P2 Q2 P1 Q3 tetivan. Pri tome su uglovi AQ2 P3
i BP3 Q3 jednaki s uglom C dakle i medu sobom. Stoga je trapez P3 Q3 P1 Q2
jednakokrak, pa se oko njega moze opisati krug. S obzirom da su cetvorouglovi
P1 Q1 P3 Q2 , P2 Q2 P1 Q3 , P3 Q3 P1 Q2 tetivni sest tacaka P1 , Q1 , P2 , Q2 , P3 i Q3
pripadaju jednom krugu prvom Lemoanovom krugu tog trougla.
769. Dokazati da se srediste prvog Lemoanovog kruga trougla poklapa sa
sredistem duzi koja spaja Lemoanovu tacku sa sredistem opisanog kruga tog
trougla.
Sem oznaka uvedenih u predhodnom zadatku, obelezimo sa O srediste opisanog kruga trougla ABC, sa O1 srediste duzi OL i sa K tacku u kojoj se seku
duzi AL i Q2 P3 . S obzirom da je cetvorougao AQ2 LP3 paralelogram, tacka K
je srediste duzi AL, pa je O1 K k OA. No duz OA je upravna na tangenti AT
u tacki A na opisani krug trougla ABC, a prava AT uporedna sa duzi Q2 P3 ,
pa je O1 KQ2 P3 . Stoga je prava O1 K simetrala tetive Q2 P3 prvog Lemoanovog kruga l1 trougla ABC. Istim postupkom dokazuje se da je tacka O1 na
simetrali tetive P1 Q3 kruga l1 , pa je tacka O1 srediste kruga l1 . Prema tome,
srediste prvog Lemoanovog kruga je srediste duzi koja spaja Lemoanovu tacku
sa sredistem opisanog kruga trougla ABC.(Sl.768-772).
770. Dokazati da sest tacaka u kojima prvi Lemoanov krug sece trougao
odredjuju dva medju sobom podudarna trougla.
Zadrzavajuci oznake uvedene pri resavanju zadatka 768, imamo da je P3 P1 P2 =
P3 Q3 P2 = B i P1 P2 P3 = P1 Q2 P3 = C, pa je P3 P1 P2 ABC. Isto
tako je Q2 Q3 Q1 ABC, pa je P3 P1 P2 Q2 Q3 Q1 . S obzirom da su
slicni trouglovi P3 P1 P2 i Q2 Q3 Q1 upisani u istom krugu, oni su podudarni(Sl.768772).

480

771. Ako su Q3 i P2 tacke u kojima prvi Lemoanov krug sece stranicu BC


trougla ABC, dokazati da je:
BQ3 : Q3 P2 : P2 C = AB 2 : BC 2 : CA2
.
Koristeci oznake uvedene pri resavanju zadatka 768,imamo da je BQ3 :
Q3 P2 = Q2 L : LP2 = Q2 B 2 : BP22 = AB 2 : AC 2 i BQ3 : P2 C = P1 L :
LQ1 = AP12 : AQ21 = AB 2 : AC 2 . Otuda je BQ3 : Q3 P2 : P2 C = AB 2 : BC 2 :
CA2 .(Sl.768-772).
772. Ako su Q2 P1 , Q3 P2 , Q1 P3 duzi koje prvi Lemoanov krug odseca od
stranica AB, BC, CA trougla ABC. Dokazati da je:
Q2 P1 : Q3 P2 : Q1 P3 = AB 3 : BC 3 : CA3
.
Pored oznaka uvedenih pri resavanju zadatka 768, obelezimo sa A i L podnozja upravnih iz tacaka A i L na pravoj BC, a sa C i L podnozja upravnih
iz tacaka C i L na pravoj AB. Pri tome je Q3 P2 : LL = BC : AA , Q2 P1 =
LL = AB : CC , BC : AA = AB : CC , BC : AA = AB : CC , LL : LL =
BC : AB. Iz ovih jednakosti nalazimo da je Q2 P1 : Q3 P2 = AB 3 : BC 3 . Isto
tako je Q3 P2 : Q1 P3 = BC 3 : CA3 .(Sl.768-772).
773. Dokazati da prave kroz Lemoanovu tacku trougla, od kojih je svaka
antiparalelna s jednom stranicom tog trougla u odnosu na ostale dve stranice,
seku taj trougao u tackama koje pripadaju jednom krugu, drugom Lemoanovom
krugu tog trougla. Srediste tog kruga je Leomanova tacka tog trougla.
A

P3

Q2

P1

Q1

P2

Q3

Slika 773
Neka su P1 ,Q1 ;P2 ,Q2 ;P3 ,Q3 tacke u kojima prave kroz Lemoanovu tacku L
trougla ABC antiparalelne sa stranicama u odnosu na ostale dve seku taj trougao. Prema zadatku 767, duzi P1 Q1 ,P2 Q2 i P3 Q3 su medu sobom jednake, a
481

prema zadatku 765 tacka L je srediste svake od tih duzi, prema tome, svih sest
tacaka P1 , P2 , P3 , Q1 ,Q2 , Q3 pripadaju jednom krugu, kome je srediste tacka
L. Krug kome pripadaju navedene tacke zove se drugi Lemoanov krug, ili pak
kosinusni krug trougla ABC.
774. Ako su r1 i r2 poluprecnici prvog i drugog Lemoanovog kruga i r poluprecnik opisanog kruga trougla ABC, dokazati da je:
4r12 r22 = r2
.
Sem oznaka uvedenih pri resavanju zadatka 768,obelezimo sa P1 i Q1 tacke
u kojima prave kroz Lemoanovu tacku L antiparalelne sa stranicom BC u odnosu na ostale dve stranice, sece stranice AB i AC tog trougla. Pri tome je
duz P1 Q1 precnik, a duz LQ1 poluprecnik drugog Lemoanovog kruga; sem toga
1
je KP3 kLQ1 i KP3 = LQ1 . Kod pravouglog trougla O1 KP3 imamo da je
2
1
1
1
O1 P32 KP32 = O1 K 2 . No O1 P3 = r1 , KP3 = LQ2 = r2 i O1 K = OA =
2
2
2
1
r, pa je 4r12 r22 = r2 (sl.768).
2
775. Dokazati da kod trougla prvi Lemoanov krug sece drugi Lemoanov krug
u dijametralno suprotnim tackama.
Ako su P1 i Q1 tacke u kojima prava kroz Lemoanovu tacku L trougla ABC

uporedna sa stranicom BC sece ostale dve stranice AB i AC, a P1 i Q1 tacke


u kojima prava kroz tacku L antiparalelna sa stranicom BC u odnosu na ostale

dve stranice tog trougla sece AB i AC,bice tacke P1 , P1 , Q1 , Q1 na jednom


krugu, pa je LP1 LQ1 = LP1 LQ1 . Otuda sleduje da je tacka L na radikalnoj osi dvaju Lemoanovih krugova l1 i l2 trougla ABC. No tacka L je srediste
drugog Lemoanovog kruga l2 , prema tome, prvi Lemoanov krug l1 sece drugi
Lemoanov krug l2 u dijametralno suprotnim tackama.

9.12

Tikerovi krugovi trougla

776. Ako su A , B , C tacke duzi LA, LB, LC koje spajaju Lemoanovu tacku
L s temenima trougla ABC takve da je LA : LA = LC : LC , zatim P1 i Q1
tacke u kojima prava B C sece stranice AB i AC, P2 i Q2 tacke u kojima prava
C A sece stranice BC i AB, P3 i Q3 tacke u kojima prava A B sece stranice
AC i BC, dokazati da sest tacaka P1 , Q1 , P2 , Q2 , P3 , Q3 pripadaju jednim krugu,
Tikerovom krugu trougla ABC.

482

Q2

P3
K

P1

Q1

P2

Q3

Slika 776-777
S obzirom da je LA : LA = LB : LB = LC : LC , imamo da je
AB k A B ,BC k B C , CA k C A . Stoga je cetvorougao AQ2 A P3 paralelogram, pa je srediste K duzi Q2 P3 na pravoj AA , tj. pravoj AL. Otuda sleduje
da je duz Q2 P3 antiparalelna sa stranicom AB i AC, prema tome, cetvorougao
P1 Q1 P3 Q2 je tetivan. Isto tako i cetvougao P2 Q2 P1 Q3 je tetivan. Uglovi AQ2 P3
i BP1 Q3 jednaki su s uglom C, dakle i medu sobom, pa je trapez P1 Q3 P3 Q2
jednakokrak i prema tome cetvorougao P1 Q3 P3 Q2 je tetivan. S obzirom da su
cetvorougli P1 Q1 P3 Q2 , P2 Q2 P1 Q3 , P1 Q3 P3 Q2 tetivni, sest tacaka P1 , Q1 , P2 ,
Q2 , P3 , Q3 pripadaju jednom krugu, koji nazivamo Tikerov krug trougla ABC
777. Dokazati da se sredicta Tikerovih krugova trougla nalaze na pravoj
koja je odredjena Lemoanovom tackom i sredistem opisanog kruga tog trougla.
Sem oznaka uvedenih pri resavanju prethodnog zadatka, obelezimo sa U
tacku duzi OL takvu da je OA k U K. S obzirom da je OA P3 Q2 i OA k U K
bice U K P3 Q2 . No tacka K je srediste te duzi P3 Q2 , pa je prava U K simetrala tetive P3 Q2 Tikerovog kruga k trougla ABC. Na isti nacin dokazuje se
da se tacka U nalazi i na simetrali tetive P1 Q3 kruga k, pa je tacka U srediste
toga kruga. Prema tome, srediste U Tikerovog kruga k je na pravoj OL. S
obzirom da su trouglovi ABC i A B C slicni i u slicnom polozaju u odnosu na
tacku L, srediste O kruga opisanog oko trougla A B C je na pravoj OL, pri
cemu je OA k O A . Tacka K je srediste AA , pa je i tacka U srediste duzi OO
(sl.776-777).

9.12. Tajlorov krug trougla


778. Dokazati da upravne projekcije podnozja svih visina trougla na pravama koje sadrze neodgovarajuce stranice tog trougla, pripadaju jednom krugu,
483

Tajlorovom krugu tog trougla.

A
B2

C1
B
A2

A1

C2

B1

Slika 778
Prvi nacin. Neka su AA , BB , CC visine trougla ABC, A1 i A2 upravne
projekcije tacke A na pravama AB i AC, B1 i B2 upravne projekcije tacke
B na pravama BC i BA, C1 i C2 upravne projekcije tacke C na pravama
CA i CB. S obzirom da je cetvorougao A1 C2 A C tetivan i duz A C uporedne s duzi B1 B2 , cetvorougao A1 C2 B1 B2 je tetivan, pa su tacke A1 , C2 ,
B1 , B2 na istom krugu k. Istim postupkom dokazuje se da je i cetvorougao
C2 B1 A2 C1 tetivan pa su i tacke C2 , B1 , A2 , C1 na izvesnom krugu k1 . Kako
je A1 C2 B = A1 C A = BCA i A2 B1 C = A2 B A = ABC, bice
A1 C2 kAC i B1 A2 kAB. Sem toga je AA2 A1 = AB C = ABC. Stoga
je A1 A2 B1 = A1 C2 B2 , pa je cetvorougao C2 B1 A2 A1 takode tetivan, pa su
tacke C2 , B1 , A2 , A1 na izvesnom krugu k2 . Krugovi k i k2 imaju tri zajednicke
tacke dakle istovetni su; isto tako krugovi k2 i k1 imaju tri zajednicke tacke
pa su takode istovetni. Stoga tacke A1 , A2 , B1 , B2 , C1 , C2 pripadaju jednom
krugu.
Drugi nacin. S obzirom da je C1 B2 k BC, A1 C2 k CA, B1 A2 k AB, prave
C1 B2 , A1 C2 , B1 A2 odreduju izvestan trougao A3 B3 C3 koji je homotetican s trouglom ABC u odnosu na izvesnu tacku L. Pri tome je cetvorougao AA1 A3 A2
paralelogram, te duz AA3 , dakle i prava AL sadrzi srediste duzi A1 A2 koja je
antiparalelna sa stranicom BC. Stoga prava AL sadrzi simedijanu iz temena A
trougla ABC. Isto tako prave BL i CL sadrze simedijane iz temena B i C, pa
je tacka L Lemanova tacka trougla ABC. Otuda sleduje da tacke A1 , A2 , B1 ,
B2 , C1 , C2 pripadaju jednom krugu. Na taj nacin dokazano je istovremeno da
je Tajlorov krug trougla specijalan Tikerov krug tog istog trougla.
779. Dokazati da se srediste Tejlorovog kruga trougla nalazi na pravoj koja
je odredena Lemoanovom tackom i sredistem opisanog kruga tog trougla.
484

Re
senje: S obzirom da je Tejlorov krug trougla specijalan Tikerov krug
istog trougla, prema zadatku 777, srediste Tejlorovog kruga je na pravoj koja je
odredena Lemoanovom tackom i sredistem kruga tog trougla.

9.13

Brokarove ta
cke trougla

Sledecim zadatkom bice dokazano da u ravni proizvoljnog trougla ABC postoji jedna i samo jedna tacka X, takva da je XAB = XCA, zatim da postoji
jedna i samo jedna tacka X , takva da je X AC = X BA = X CB. Tacku
X zvacemo prvom, a tacku X drugom Brokarovom tackom trougla ABC. Sem
toga, poluprave AX, BX, CX zvacemo prvim, a poluprave AX , BX , CX drugim Brokarovim polupravama trougla ABC.
780. Dokazati da u ravni trougla ABC postoji jedna i samo jedna tacka X,
takva da je XAB = XBC = XCA, zatim da postoji jedna i samo jedna tacka
X , takva da je X AC = X BA = X CB.

X
C
B

Slika 432.
Re
senje: Ako bi takva tacka X postojala, iz jednakosti XAB = XBC,
XBC = XCA , XCA = XAB sledilo bi da krug AXB dodiruje pravu BC u
tacki B, krug BXC dodiruje pravu CA u tacki C, krug CXA dodiruje pravu
AB u tacki A.
781. Dokazati da su Brokarove tacke trougla izogonalno spregnute u odnosu
na taj trougao.

Re
senje: Neka su X i X Brokarove tacke trougla ABC, tj. tacke takve

da je XAB = XBC = XCA i X AC = X BA = X CB. Ako je X


tacka izogonalno spregnuta u odnosu na trougao ABC, imamo da je XAB =

X AC, XBC = X BA, XCA = X CB, pa je saglasno prethodnoj

teoremi X X .
782. Ako je ABC proizvoljan trougao, k krug koji sadrzi teme B i dodiruje
pravu AC u tacki C, a D tacka u kojoj prava kroz teme C, uporedna sa stranicom AB, sece krug k, dokazati da je tacka X u kojoj prava AD sece krug k
prva Brokarova tacka trougla ABC.
Re
senje: S obzirom da je XDC = XAB,XDC = XBC, XDC =
XCA, bice XAB = XBC = XCA, pa je tacka X prva Brokarova tacka

trougla ABC. Ako bi k bio krug koji sadrzi tacku C i dodiruje pravu AB u

tacki B, a D bila tacka u kojoj prava kroz teme B uporedna sa stranicom AC


485

sece krug k , presecna tacka X kruga k s pravom AD bila bi druga Brokarova


tacka trougla ABC.
Ovaj zadatak ukazuje i na veoma jednostavan postupak za konstrukciju Brokarovih tacaka trougla.
A

E
B

X
Xa

F
D

Slika 433.
Napomena 1.
Prava AB je tetiva kruga opisanog oko trougla ABC, jer je DBC = DXC
= XAC + XCA = XAC + XAB = BAC.
Napomena 2.
Ako bismo obelezili sa E i F podnozja upravnih iz tacaka C i D na pravoj AB
i sa F AD Brokarov ugao trougla ABC, imali bismo da je AF = AE + EB +
AE
EB
BF
AE
EB
BF
AF
=
+
+
=
+
+
, i prema tome
BF , pa je
DF
DF
DF
DF
CE
CF
FD
coth = coth A + coth B + coth C.
783. Ako su X i X prva i druga Brokarova tacka trougla ABC, a Xa i Xa
tacke u kojima prave AX i AX seku pravu BC, dokazati da je
BXa
BXa
c2
a2
(a)
= 2,
= 2;

Xa C
a
Xa C
b
AX
a2 b 2 + b 2 c2
AX
b 2 c2 + a2 c2
(b)
=
,
=
.
2
2

XXa
a c
X Xa
a2 b 2
Re
senje: (a) Koristeci oznake uvedene pri resavanju prethodnog zadatka,
imamo da je ABXa DCXa , pa je BXa : Xa C = AB : CD. No ABC
BCD, pa je AB : BC = BC : CD, te je CD = BC 2 : AB, i prema tome

BXa : Xa C = AB 2 : BC 2 = c2 : a2 . Isto tako je BXa : X C = a2 : b2 .


(b) Ako obelezimo sa Xb i Xc tacke u kojima prave BX i CX seku
prave CA i AB, prema dokazanom delu ovog zadatka i Van Obelovoj teoremi,

nalazimo da je AX : XXa = (a2 b2 + b2 c2 ) : a2 c2 . Isto tako je AX : X Xa =


(b2 c2 + a2 c2 ) : a2 b2 .
784. Ako je X prva Brokarova tacka trougla ABC i ako su A , B , C tacke
u kojima prave CX, AX, BX seku opisani krug trougla ABC, dokazati da su
trouglovi ABC i A B C podudarni, zatim da je tacka X druga Brokarova tacka
trougla A B C .

Re
senje: Trouglovi ABC i A B C su slicni, jer je B A C = B A C +

CA C = B AC + CBC = B AC + BAB = BAC i na isti nacin

A B C = ABC. S obzirom da su trouglovi ABC i A B C slicni i upisani u

isti krug, oni su podudarni. Tacka X je druga Brokarova tacka trougla A B C ,

486

jer je XA C = CA C = CBC = , XB A = AB A =ACA =

, XC B = BC B = BAB = . Analogna osobina izvodi se i za drugu


Brokarovu tacku trougla ABC (sl. 434).
A

C
A
X
B

Slika 434.
Napomena: Ako je O srediste opisanog kruga trougla ABC, imamo da je

AOA = BOB = COC = 2.


785. Ako su X i X Brokarove tacke i O srediste opisanog kruga trougla
ABC, dokazati da je OX = OX .

Re
senje: Ako su A , B , C tacke u kojima prave CX, AX, BX seku
opisani krug trougla ABC (videti sliku 434), prema prethodnom zadatku 784,

bice kod podudarnih trouglova ABC i A B C , koji su upisani u istom krugu,

tacke X i X prve Brokarove tacke, pa je OX = OX .


786. Ako su P,Q,R podnozja upravnih iz bilo koje Brokarove tacke X na
stranicama BC, CA, AB, dokazati da je ABC RP Q.
Re
senje: Pretpostavimo da je X prva Brokarova tacka trougla ABC. Iz
tetivnih cetvorouglova CQAR i XRBP nalazimo da je
A = XAR + XAQ = XBP + XAQ = XRP + XQR = QRP
Isto tako je B = P, pa je ABC RP Q.

P
Q
X
C
B

Slika 435.
487

787. Ako su P, Q, R podnozja upravnih iz prve Brokarove tacke, a P, R,


Q podnozja upravnih iz druge Brokarove tacke na stranicama BC, CA, AB
trougla ABC, dokazati da je P QR R P Q .
Re
senje: Prema prethodnom zadatku, trouglovi RPQ i QRP slicni su s
trouglom ABC, dakle i medu sobom. Obzirom da su tacke X i X izogonalne,
tacke P, Q, R, P, Q, R su na jednom krugu pa su slicni trouglovi RPC i
QRP upisani u istom krugu, prema tome, oni su podudarni.

9.14

Brokarov krug trougla

Definicija 9.14. Krug kome je precnik duz odredena sredistem opisanog


kruga i Lemoanovom tackom trougla nazivamo Brokarovim krugom tog trougla. Iz ove definicije neposredno sleduje da su Brokarov krug i prvi Lemoanov
krug koncentricni. Simetrale stranica BC,CA,AB seku Brokarov krug trougla
ABC u tacki O, sredistu opisanog kruga, i u tackama A1 , B1 , C1 koje obrazuju
prvi Brokarov trougao A1 B1 C1 datog trougla ABC. Prave koje sadrze simedijane iz temena A,B,C seku Brokarov krug trougla ABC u Lemoanovoj tacki
L, i u tackama A2 , B2 , C2 koje odreduju drugi Brokarov trougao A2 B2 C2 datog
trougla ABC.
788. Dokazati da se Brokarove tacke trougla nalaze na Brokarovom krugu
tog trougla.
Re
senje: Obelezimo sa O srediste opisanog kruga, sa L Lemoanovu tacku,
sa k Brokarov krug i sa A1 B1 C1 prvi Brokarov trougao proizvoljnog trougla
ABC. Obzirom da je OA1 L prav i OA1 BC, bice LA1 k BC.
Isto tako je LB1 k CA, LC1 k AB. Stoga su odstojanja A1 A , B1 B , C1 C
tacaka A1 , B1 , C1 od pravih BC, CA, AB jednaka odstojanju tacke L od istih
pravih, pa je A1 A : BC = B1 B : CA = C1 C : AB. Iz ovih proporcija sleduje
da su jednakokraki trouglovi ABC1 i BCA1 slicni, pa je XA1 O = BA1 A =
AC1 C = XC1 O, gde je X tacka u kojoj se seku prave AC1 i BA1 . Otuda
sleduje da je tacka X na krugu koji sadrzi tacke O,A1 ,C1 , tj. na krugu k. Na
isti nacin dokazuje se da je presecna tacka pravih BA1 i CB1 na krugu k, te
se prave AC1 , BA1 i CB1 seku u istoj tacki X koja je na krugu k. Iz slicnosti
jednakokrakih trouglova A1 BC, B1 CA i C1 AB sledi da je XAB = XBC =
XCA, pa je X prva Brokarova tacka trougla ABC. Analognim postupkom se
dokazuje da se prave AB1 , BC1 i CA1 seku u drugoj Brokarovoj tacki X koja
se takode nalazi na krugu k (slika 436).

488

A1

C1
X
B

B1
kO

Slika 436.
789. Dokazati da je Brokarov prvi trougao inverzno slican s datim trouglom.
Re
senje: Koristeci oznake uvedene u resavanju zadatka 788. imamo da
je BAC = C1 LB1 = C1 B1 A1 . Isto tako je ABC = C1 B1 A1 , pa su
trouglovi ABC i A1 B1 C1 inverzno slicni.
790. Dokazati da se prave kroz temena trougla, uporedne s odgovarajucim
stranicama prvog Brokarovog trougla, seku u jednoj tacki, koja se nalazi na

opisanom krugu datog trougla. Tu tacku nazivamo Stajnerovom


tackom datog
trougla.
Re
senje: Ako obelezimo sa A1 B1 C1 Brokarov prvi trougao proizvoljnog
trougla ABC i sa S tacku u kojoj se seku prave kroz temena B i C uporedne sa
stranicama A1 C1 i A1 B1 (videti sliku 436), imamo da je BSC = C1 B1 A1 =
BAC. No tacke A i S su sa iste strane prave BC, pa je tacka S na krugu koji je
opisan oko trougla ABC. Analognim postupkom dokazuje se da se i prave kroz
temena A i B, uporedne sa stranicama B1 C1 i C1 A1 , takode seku na opisanom
krugu trougla ABC, koja se prema tome poklapa sa tackom S.
791. Dokazati da se prave kroz temena trougla, upravne na odgovarajucim
stranicama Brokarovog prvog trougla, seku u jednoj tacki, koja se nalazi na opisanom krugu datog trougla. Tu tacku nazivamo Tarijevom tackom tog trougla.
Re
senje: Koristeci oznake uvedene pri resavanju prethodnog zadatka, nalazimo da su normale u tackama A, B, C datog trougla na stranicama B1 C1 ,
C1 A1 i A1 B1 Brokarovog prvog trougla upravne i na pravama SA, SB i SC,
prema tome, one seku opisani krug trougla ABC u tacki T koja je dijametralno
suprotna s tackom S.
792. Dokazati da su temena drugog Brokarovog trougla sredista duzi koja
odseca opisani krug datog trougla na pravama odredenim simedijanama tog
trougla.
Re
senje: Obelezimo sa L Lemoanovu tacku i sa O srediste kruga opisanog
oko trougla ABC, a sa A2 ,B2 , C2 tacke u kojima Brokarov krug trougla ABC,
tj. krug kome je duz OL precnik, sece prave AL,BL, CL. Pri tome su uglovi
489

LA2 O, LB2 O i LC2 O pravi, pa su temena A2 , B2 , C2 drugog Brokarovog trougla


sredista duzi koje odseca opisani krug trougla ABC na pravama AL, BL, CL.
793. Ako je ABC proizvoljan trougao, dokazati da se krugovi k1 i k2 , od
kojih prvi sadrzi teme B i dodiruje stranicu AC u tacki A, a drugi sadrzi teme
C i dodiruje stranicu AC u tacki A, seku sem u tacki A u izvesnoj tacki A2 ,
koja predstavlja teme drugog Brokarovog trougla datog trougla ABC.
Re
senje: Obzirom da se tetive AC i AB na krugovima k1 i k2 u njihovoj
zajednickoj tacki A seku, i krugovi k1 i k2 se seku u tacki A i jos nekoj tacki A2 .
Neka su D i E tacke u kojima prave AA2 i BA2 seku opisani krug l trougla ABC.
Pri tome je CAA2 = ABA2 = ABE = ADE i ACA2 = BAA2 =
BAD = BED. Iz jednakosti uglova CAD i ADE sleduje da su tetive AE i
DC jednake, pa su kod jednakokrakog trapeza ADCE jednake i dijagonale AC
i DE. Stoga je ACA2 DEA2 , pa je AA2 = A2 D. No iz slicnih trouglova
BAA2 i ACA2 sleduje da je visina iz temena A2 tih trouglova proporcionalna
sa stranicama AB i AC, prema tome, prava AA2 sadrzi simedijanu iz temena A
trougla ABC. Otuda sleduje da je tacka A2 teme drugog Brokarovog trougla
datog trougla ABC (sl. 437).

k1

k2
A2
B

C
l

Slika 437.

9.15

Apolonijevi krugovi i izodinami


cke ta
cke trougla

Definicija 9.15. Neka su E i F tacke u kojima simetrale unutrasnjeg i spoljasnjeg ugla A, trougla ABC, seku pravu BC. Krug ka kome je duz EF precnik
nazivamo Apolonijevim krugom koji odgovara temenu A ili stranici BC trougla
ABC. Analogno se konstruisu i Apolonijevi krugovi kb i kc koji odgovaraju
temenima B i C tog trougla. S obzirom da je ugao EAF prav, teme A je na
krugu ka . Isto tako je teme B na krugu kb , a teme C na krugu kc .
794. Dokazati da je opisani krug trougla ortogonalan na Apolonijevim krugovima tog trougla.

490

Re
senje: Tacke E i F u kojima simetrala unutrasnjeg ugla A i simetrala
spoljasnjeg ugla A trougla ABC seku pravu BC su harmonijski spregnute sa
tackama B i C. Stoga su tacke B i C inverzne medu sobom u odnosu na krug
la kome je duz EF precnik, pa je i opisani krug l trougla ABC ortogonalan
na Apolonijevom krugu la tog trougla. Na isti nacin dokazuje se da je krug l
ortogonalan na Apolonijevim krugovima lb i lc (sl.438).

la

l
A

Oa

Slika 438.
795.Dokazati da je Brokarov krug trougla ortogonalan na Apolonijevim krugovima tog istog trougla.
Re
senje: Lemoanova tacka L i srediste O opisanog kruga oko trougla ABC
su spregnute u odnosu na svaki Apolonijev krug tog trougla, pa je Brokarov
krug, tj. krug kome je duz OL precnik, ortogonalan na Apolonijevim krugovima tog trougla.
796.Dokazati da je Lemoanova prava trougla radikalna osa Brokarovog kruga
i opisanog kruga istog trougla.
Re
senje: S obzirom da su Brokarov krug i opisani krug l trougla ABC ortogonalni na Apolonijevim krugovima istog trougla (vidi zadatke 794. i 795.), oni
odreduju jedan pramen krugova koji je spregnut s pramenom kome pripadaju
Apolonojevi krugovi tog trougla. Stoga je Lemoanova prava na kojima se nalaze
sredista Apolonijevih krugova trougla, radikalna osa krugova l i k.
797.Dokazati da Apolonijevi krugovi trougla pripadaju eliptickom pramenu
krugova. Tacke u kojima se seku ti krugovi nazivamo izodinamickim tackama
datog trougla.
491

Re
senje: Pretpostavimo da je kod trugla ABC npr. AC > AB > BC. Ako
su E i F tacke u kojima simetrala unutrasnjeg ugla A i simetrala spoljasnjeg
ugla A seku pravu BC, tacka E bice izmedu tacaka B i C, a tacka F iza tacke B
u odnosu na tacku C. Stoga je tacka B u, a tacka C izvan Apolonijevog kruga
la trougla ABC. S obzirom da je tacka A na krugu la , a tacka B u tom krugu,
tacka D, u kojoj simetrala ugla C sece stranicu AB, bice u krugu la . Otuda
sleduje da Apolonijev krug lc sadrzi tacku C koja je izvan kruga la i tacku D
koja je u krugu la , pa se krugovi la i lc seku u tackama, npr. X i Y . Pritom je
BX : CX = AB : AC i AX : BX = AC : BC, pa je i AX : CX = AB : BC.
Stoga je X tacka i Apolonijevog kruga lb . Isto tako je i Y tacka kruga lb , pa
se Apolonijevi krugovi la , lb i lc trougla ABC seku u dvema tackama X i Y , te
pripadaju eliptickom pramenu krugova.
798. Dokazati da se izodinamicke tacke trougla nalaze na pravoj koja je
odredena sredistem opisanog kruga i Lemoanovom tackom tog trougla.
Re
senje: Prema zadatku 794, opisani krug l trougla ABC ortogonalan je
na Apolonijevim krugovima tog trougla, pa se njegovo srediste O nalazi na
zajednickoj radikalnoj osi tih krugova, tj. na pravoj koja je odredena izodinamickim tackama X i Y tog trougla. S obzirom da je prava XY koja sadrzi
tacku O upravna na pravoj koja sadrzi sredista Apolonijevih krugova, tj. Lemoanovoj pravoj trougla ABC, a prema zadatku 762. prava odredena tackom O
i Lemanovom tackom L takode je upravna na Lemoanovoj pravoj tog trougla,
bice da su prave XY i OL istovetne, i prema tome O, L, X i Y kolinearne.
799. Dokazati da je srediste bilo kojeg Apolonijevog kruga trougla srediste
slicnosti druga dva Apolonijeva kruga tog trougla.
Re
senje: Obelezimo sa Oa , Ob i Oc sredista Apolonijevih krugova la , lb i lc
trougla ABC, a sa X i Y izodinamicke tacke tog trougla. U inverziji u odnosu
na jedan od krugova la , lb i lc , npr. la , tackama X, Y i B kruga lb odgovaraju
tacke X, Y i C kruga lc , pa su krugovi lb i lc inverzni medu sobom u odnosu na
krug la . Stoga je tacka Oa srediste slicnosti krugova lb i lc .
800. Dokazati da prava odredena presecnim tackama opisanog kruga s Apolonijevim krugom, koji odgovara jednom temenu trougla, sadrzi simedijanu iz
istog temena tog trougla.
Re
senje: Opisani krug l i Apolonijev krug, npr. la trougla ABC seku se
u tacki A i jos jednoj tacki, recimo A . Prema zadatku 794, krugovi l i la su
ortogonalni, pa je prava AA polara sredista Oa kruga la u odnosu na krug l.
Stoga prava AA sadrzi simedijanu AA trougla ABC.
801. Dokazati da je prava koja sadrzi simedijanu iz jednog temena trougla,
polara sredista opisanog kruga u odnosu na Apolonijev krug koji odgovara istom
temenu tog trougla.
Re
senje: Prema prethodnom zadatku, prava odredena simedijanom, npr.
AA trougla ABC, sadrzi tacke u kojima opisani krug l sece Apolonijev krug
la tog trougla. S obzirom da su krugovi l i la ortogonalni, prava AA je polara
sredista O kruga l u odnosu na krug la .

492

9.16

Droz-Farnijevi krugovi trougla

Definicija 9.16. Krugove kojima se sredista poklapaju sa ortocentrom nekog


trougla nazivamo Droz-Farnijevim krugovima tog trougla. Polarni krug trougla
je specijalan Droz-Farnijev krug tog trougla.
802. Ako su A1 , B1 , C1 sredista stranica BC, CA, AB trougla ABC, a
ta , tb , tc krugovi jednakih poluprecnika, kojima su sredista A, B, C, dokazati
da tacke P1 i P2 , Q1 i Q2 , R1 i R2 u kojima prave B1 C1 , C1 A1 , A1 B1 seku
respektivno krugove ta , tb , tc pripadaju jednom krugu, kome se srediste poklapa
s ortocentrom H tog trougla.
Re
senje: Ako obelezimo sa A , B , C podnozja visina iz temena A, B, C
trougla ABC i sa K tacku u kojoj visina AA sece pravu B1 C1 , kod trougla
AHP1 nalazimo da je AP12 = AH 2 + HP12 + 2AH HK, pa je
HP12 = AP12 AH 2 2AH HK = AP12 AH(AH +2HK) = AP12 2AH HA .
Isto tako je
HP22 = AP22 2AH HA , HQ21 = BQ21 2BH HB , HQ22 = BQ22 2BH HB ,
HR12 = CR12 2CH HC , HR22 = CR22 2CH HC .
No AP1 = AP2 = BQ1 = BQ2 = CR1 = CR2 i AH HA = BH HB =
CH HC pa je i HP1 = HP2 = HQ1 = HQ2 = HR1 = HR2 . Stoga tacke
P1 , P2 , Q1 , Q2 , R1 , R2 pripadaju jednom krugu kome se srediste poklapa s
ortocentrom H trougla ABC (slika 439).

Q2

R2
C1
P1

B1
K

P2

C
H

A1

Q1

Slika 439.
493

R1

803. Ako su poluprecnici krugova ta , tb , tc , navedenih u prethodnom zadatku


jednaki duzi 0 , a poluprecnici njima odgovarajuceg Droz-Farnijevog kruga i
opisanog kruga trougla ABC jednaki duzima r0 i r, dokazati da je
1
r02 = 4r2 + 20 (a2 + b2 + c2 ).
2
Re
senje: Pri resavanju prethodnog zadatka, dobili smo da je r02 = HP12 =
1
AP12 2AH HA . Prema zadacima 246 i 244 imamo da je AH AA = (b2 +
2
c2 a2 ) i AH 2 = 4r2 a2 , pa je AH HA = AH(AA AH) = AH HA
1
1
AH 2 = (b2 + c2 a2 ) + a2 4r2 = (a2 + b2 + c2 ) 4r2 . Otuda je r02 =
2
2
1
4r2 + 20 (a2 + b2 + c2 ). Specijalno, ako je 0 = r, imamo da je
2
1
r02 = 5r2 (a2 + b2 + c2 ).
2
804. Ako su A , B , C podnozja visina iz temena A, B, C i O srediste
opisanog kruga trougla ABC, dokazati da tacke P1 i P2 , Q1 i Q2 , R1 i R2
u kojima krugovi (A , A O), (B , B O), (C , C O) seku respektivno prave BC,
CA, AB pripadaju jednom krugu kome se srediste poklapa s ortocentrom H
tog trougla.
Re
senje: Ako obelezimo sa O srediste duzi OH, tj. srediste Ojlerovog
kruga trougla ABC, imamo da je HP12 = HP22 = HA2 + A O2 = 2A O2
1
1
OH 2 . Isto tako HQ21 = HQ22 = 2B O OH 2 , HR12 = HR22 = 2C O2
2
2
1
OH 2 . No duzi O A , O B , O C kao poluprecnici Ojlerovog kruga trougla
2
ABC jednake su medu sobom, pa je HP1 = HP2 = HQ1 = HQ2 = HR1 =
HR2 . Stoga tacke P1 , P2 , Q1 , Q2 , R1 , R2 pripadaju jednom krugu kome se
srediste poklapa s ortocentrom H trougla ABC (sl. 440).

494

R1

Q2

C
Q1

O
H

B
P1 R
2

C
P2

Slika 440.
805. Ako su A1 , B1 , C1 sredista stranica BC, CA, AB i H ortocentar trougla
ABC, dokazati da tacke P1 i P2 , Q1 i Q2 , R1 i R2 u kojima krugovi (A1 , A1 H),
(B1 , B1 H), (C1 , C1 H) seku respektivno prave BC, CA, AB pripadaju jednom
krugu kome se srediste poklapa sa sredistem O opisanog kruga trougla ABC.
Re
senje: Ako obelezimo sa O srediste duzi OH, imamo da je OP12 =
1
OP22 = OA21 + A1 H 2 = 2A1 O2 + OH 2 . Isto tako je OQ21 = OQ21 = 2B1 O2 +
2
1
1
2
2
2
2
OH i OR1 = OR1 = 2C1 O + OH 2 . No duzi O A1 , O B1 , O C1 su
2
2
poluprecnici Ojlerovog kruga trougla ABC, dakle medu sobom jednake, pa su
i duzi OP1 , OP2 , OQ1 , OQ2 , OR1 , OR2 medu sobom jednake. Stoga tacke
P1 , P2 , Q1 , Q2 , R1 , R2 pripadaju jednom krugu kome se srediste poklapa sa
sredistem opisanog kruga trougla ABC. S obzirom da je tacka O ortocentar
trougla A1 B1 C1 , dobijeni krug je Droz-Farnijev krug trougla A1 B1 C1 (sl. 441).

495

A
Q2
R1

B1

C1
O

H
R2
Q1

P1

A1

P2

Slika 441.

9.17

Adamsovi krugovi trougla

806. Dokazati da tacke u kojima prave kroz Zargonovu


tacku trougla upravne
na simetralama njegovih unutrasnjih uglova seku taj trougao, pripadaju jednom
krugu, Adamsovom krugu tog trougla.
Re
senje: Obelezimo sa S srediste kruga k upisanog u trougao ABC, sa P ,

Q, R tacke u kojima taj krug dodiruje stranice BC, CA, AB, sa G Zargonovu
tacku tog trougla, a sa P1 i Q1 , P2 i Q2 , P3 i Q3 tacke u kojima prave kroz
tacku G upravne na simetralama unutrasnjih uglova A, B, C tog trougla sece
prave AB i AC, AB i BC, BC i AC (sl. 442). Duzi RQ i P1 Q1 upravne su na
simetrali unutrasnjeg ugla A, dakle medu sobom uporedne, te je RP1 = QQ1 .
Isto tako je RQ2 = P P2 i P Q3 = QP3 . Trouglovi P QR i GP3 Q2 su perspektivni
u odnosu na tacku A, a stranice P Q i P R uporedne sa stranicama GP3 i GQ2 ,
pa je RQ k Q2 P3 . Stoga je i QP3 = RQ2 . Isto tako je i RP1 = P Q3 i
P P2 = QQ1 . Na taj nacin, svih sest odsecaka P P2 i P Q3 , QQ1 i QP3 i P P1
i P Q2 na tangentama kruga k jednaki su medu sobom, prema tome, svih sest
tacaka P1 , Q1 , P2 , Q2 , P3 , Q3 pripadaju jednom krugu, Adamsovom krugu
trougla ABC.

496

P3
Q2
Q
R
Q1

P1

Q3

P2

Slika 442.
807. Dokazati da se srediste Adamsovog kruga trougla poklapa sa sredistem
upisanog kruga tog trougla.
Re
senje: Tacke u kojima Adamsov krug sece stranice trougla jednako su
udaljene od sredista upisanog kruga tog trougla, pa se srediste Adamsovog kruga
poklapa sa sredistem upisanog kruga istog trougla.

tacka koja odgovara spolja upisanom krugu ki (i =


808. Ako je Gi Zargonova
a, b, c) trougla ABC, a Si srediste kruga ki , dokazati da tacke u kojima prave
kroz tacku Gi upravne na pravama ASi , BSi , CSi seku respektivno prave AB
i BC, BC i BA, CA i CB, pripadaju jednom krugu, Adamsovom krugu koji
odgovara spolja upisanom krugu ki trougla ABC.
Re
senje: Dokaz se izvodi analogno kao u zadatku 806.
809. Dokazati da se srediste Adamsovog kruga koji odgovara spolja upisanom
krugu ki trougla ABC poklapa sa sredistem Si kruga ki trougla ABC.
Dokaz se izvodi analogno kao u zadatku 806.

9.18

Ortopol prave u odnosu na trougao

810. Ako su A , B , C upravne projekcije temena A, B, C trougla ABC na


nekoj pravoj s, dokazati da se prave kroz tacke A , B , C , upravne na pravama
BC, CA, AB seku u izvesnoj tacki S, ortopolu prave s u odnosu na trougao
ABC.
Ovaj stav vec smo dokazali na jedan nacin (vidi zadatak 321), ovde navodimo
drugi nacin. Obelezimo sa A , B , C podnozja upravnih iz tacaka A , B , C
na pravama BC, CA, AB sa S i S tacke u kojima prave B B i C C seku
pravu A A , a sa K tacku u kojoj se seku prave A A i BC. Pri tome su
odgovarajuce stranice trouglova ABK i C S A , kao i trouglova ACK i
B SA medu sobom upravne, pa je AK : BK = A C : A S i CK : AK =
497

A S : A B . Iz uporednosti pravih AA , BB , CC sledi da je BK : CK = A B :


A C . Mnozenjem odgovarajucih strana dobijenih triju jednakosti, nalazimo
da je A S = A S , pa su tacke S i S istovetne, a prave A A , B B , C C
konkurentne. Tacku S nazivamo ortocentrom prave s u odnosu na trougao
ABC(slika 442).
A
C
B
S
S
A

B
s

Slika 442
811. Dokazati da ortopolovi S1 i S2 dveju uporednih pravih s1 i s2 u odnosu
na isti trougao ABC odreduju duz koja je upravna na pravama s1 i s2 i jednaka
medusobnom odstojanju pravih s1 i s2 .
Obelezimo sa A1 , B1 , C1 podnozja upravnih iz temena A, B, C na pravoj
s1 , a sa A2 , B2 , C2 podnozja upravnih iz temena A, B, C na pravoj s2 . Pri
tome su trouglovi S1 B1 C1 i S2 B2 C2 pomerno podudarni, pa je S1 S2 k B1 B2
i S1 S2 = B1 B2 . Duz B1 B2 je upravna na pravama s1 i s2 , pa je i duz S1 S2
upravna na pravama s1 i s2 , i jednaka odstojanju izmedu tih pravih(slika 443).
A
C
C
B
S1
B

A A
S2

s1

B1

A1

C1

s2

B2

A2

C2

Slika 443
812. Dokazati da se ortopol prave koja sadrzi srediste opisanog kruga trougla
u odnosu na taj trougao, nalazi na Ojlerovom krugu tog trougla.
Obelezimo sa s proizvoljnu pravu koja sadrzi srediste O kruga l opisanog
oko trougla ABC, sa A , B , C podnozja upravnih iz tacaka A, B, C na
498

pravoj s, a sa A , B , C tacke u kojima uporedne AA , BB , CC , seku krug


l. Prema zadatku 700, prave kroz tacke A , B , C upravne na pravama BC,
CA, AB seku se u jednoj tacki P koja se nalazi na krugu l. Kako je tacka
A srediste duzi AA prava kroz tacku A upravna na pravoj BC, uporedna s
visinom AA1 i s pravom P A, te sadrzi srediste S duzi koja je odredena tackom
P i ortocentrom H tog trougla. Isto tako, i prave kroz tacke B i C upravne
na pravama CA i AB sadrzi tacku S. Stoga je tacka S ortopa prave s u odnosu
na trougao ABC. S obzirom da je tacka S srediste duzi P H, tacka S je na
Ojlerovom krugu trougla ABC(slika 444).
t

A
T
D

Slika 444
813. Ako su P i Q tacke u kojima neka prava s sece opisani krug trougla
ABC, dokazati da se Simsonove prave tacaka P i Q u odnosu na trougao ABC
seku u ortopolu prave s u odnosu na trougao ABC.
Obelezimo sa A podnozje upravne iz temena A na pravoj s, sa A tacku u
kojoj prava AA sece krug l, sa A1 podnozje upravne iz tacke P na pravoj BC,
sa A2 tacku u kojoj prava P A1 sece krug l i sa K tacku prave P A1 takvu da je
A K k BC. Iz tetivnih cetvorouglova P AA A2 nalazimo da je ugao
A A K = A P K = A P A2 = A AA2 ,
pa je A K k AA2 . Prema zadatku 699, Simsonova prava p tacke P u odnosu na
trougao ABC uporedna je s pravom AA2 , pa je prava p uporedna i s pravom
A K. Stoga prava p sece pravu kroz A upravnu na pravoj BC u nekoj tacki
S . Pri tome je cetvorougao A KA1 S paralelogram, pa su duzi S A i A1 K
jednake i istosmerne. Neka je tacka M presek kruga l i prave normalne na BC
koja sadrzi tacku A . Ako je N tacka u kojoj prava A M sece pravu BC,
bice cetvorougao A KA1 N paralelogram, pa su i duzi A1 K i N A jednake i
istosmetne. Stoga su i duzi S A i N A jednake i istosmerne. S druge strane,
Simsonova prava m tacke M u odnosu na trougao ABC, tj. prava koja sadrzi
tacku N a uporedna je s pravom AA (vidi zadatak 699) sece pravu A S u
izvesnoj tacki, ortopolu S prave s u odnosu na trougao ABC. Pri tome je
cetvorougao A A N S paralelogram, pa su duzi SA i S A jednake i istismerne,
pa su tacke S i S istovetne. Prema tome, Simsonova prava p tacke P u odnosu
na trougao ABC sadrzi ortopol S prave s u odnosu na trougao ABC. Na
499

isti nacin dokazuje se da i Simsonova prava q tacke Q sadrzi tacku S(slika 445).
p

A
S

M
s

A
O

N
B

A1

A2
K A

Slika 445

500

10
10.1

GEOMETRIJA POLIGONA
Op
sti poligoni

814. Ako je {A1 , . . . , An } konacan skup od n tacaka, dokazati da se tezisne


linije koje spajaju tacke Ai tog skupa sa tezistima Ti podskupova koji se sastoje
iz preostalih n 1 tacaka seku u jednoj tacki T , tezistu tog skupa tacaka, pri
cemu je
Ai T : T Ti = (n 1) : 1.

Za n=3 dokaz je jednostavan. Da bismo dokazali teoremu u opstem slucaju


primenimo princip matematicke indukcije. Stoga pretpostavimo da je teorema
istinita za n=k-1, a dokazimo da je istinita i za n=k. Ako obelezimo sa T1 , . . . , Tk
tezista skupova A2 , . . . , Ak ,. . . , {A1 , . . . , Ak1 }, tada se svake dve tezisne linije Ai Ti i Aj Tj za i,j=i,. . . ,k, i6=j, seku. Zaista, ako obelezimo sa Tij teziste
skupa tacaka {A1 , . . . , Aj1 , Ai11 , . . . , Aj1 , Aj+1 , . . . , Ak } bice tacke Tj i Ti
na duzima Ai Tij i Aj Tij takve da je Ai Tj : Tj Tij = (k 2) : 1 i Aj T2 : Ti Tij
= (k-2):1. Stoga se duzi Ai Ti i Aj Tj seku u nekoj tacki T , dok je Ti Ti k Ai Aj i
prema tome (v. sl. 446)
Ai T : T Ti = Aj T : T Tj = Ai Aj : Ti Tj = (k 1) : 1.

sl. 446
S obzirom da je na svakoj tezisnoj liniji A1 T1 , . . . , Ak Tk tacka T jednoznacno
odredena, sve te tezisne linije seku se u istoj tacki T koju nazivamo tezistem
skupa
{A1 , . . . , Ak }.

Ovim je teorema dokazana.


815. Ako je konacan skup {A1 , . . . , An } od n tacaka razlozen na dva podskupa S1 i S2 , od kojih prvi sadrzi p, a drugi preostalih n p tacaka, dokazati
da se teziste T datog skupa nalazi izmedu tezista T1 i T2 podskupova S1 i S2 ,
pri cemu je
T T1 : T T2 = (n p) : p.
501

A4

T2

T1

T2

A3
T1

Primenimo princip matematicke indukcije. Ako je p=3 stav se svodi na prethodni. Sad pretpostavljajuci da je stav istinit za p=k, dokazimo da je on istinit i
za p=k+2. U tom cilju obelezimo sa T1 i T2 tezista podskupova {A1 , . . . , Ak+1 } i
{Ak+1 , . . . , An }, a sa M1 i N1 tezista podskupova {A2 , . . . , Ak+1 } i {A2 , . . . , An }.
Saglasno pretpostavci tacka N1 je izmedu tacaka M1 i T2 takva da je
M1 N1 : N1 T2 = (n k 1) : k,
a prema prethodnom zadatku tacka T1 je izmedu tacaka A1 i M1 takva da je
A1 T1 : T1 N1 = k : 1
i tacka T izmedu tacaka A1 i N1 takva da je
A1 T : T N1 = (n 1) : 1.
Primenom Menelajeve teoreme na trougao A1 M1 N1 nalazimo da tacke T, T1 , T2
pripadaju jednoj pravoj. Ponovnom primenom Menelajeve teoreme na trougao
M1 T1 T2 i secicu A1 N1 nalazimo da je tacka T izmedu tacaka T1 i T2 takva da
je
T1 T : T T2 = (n k 1) : (k + 1).
Na taj nacin mi smo dokazali da je stav tacan i kada je p=k+1. Time je teorema
dokazana (sl. 447).
T2

A1

T1

Ni

M1

sl. 447
816. Ako su P1 , . . . , Pn tacke stranica A1 A2 , . . . , An A1 n-tougla A1 , . . . An
takve da je A1 P1 : P1 A2 = . . . = An Pn : Pn A1, dokazati da se tezista poligona
A1 , . . . An i P1 , . . . Pn poklapaju.
Primenimo princip matematicke indukcije. Zadatkom...stav je dokazan za
slucaj kada je n=3; sad pretpostavljajuci da je stav istinit za n=k, dokazimo
da je on istinit za n=k+1. Neka su A1 i P1 tezista poligona A2 , . . . , Ak+1 i
P2 , . . . , Pk+1 . Ako je M tacka dijagonale A2 Ak+1 takva da je
Ak+1 M : M A2 = Ak+1 Pk+1 : Pk+1 A1 ,
bice cetvorougao A1 P1 M Pk+1 paralelogram, pa je A1 P1 k Pk+1 M i A1 P1 =
Pk+1 M. Saglasno pretpostavci, tezista poligona A2 . . . Ak+1 = P2 . . . Pk M se
poklapaju. Ako obelezimo sa N teziste poligona P2 . . . Pk , bice tacke P1 i A1 na
duzima Pk+1 N i M N takve da je
Pk+1 P1 : P1 N = (k 1) : 1
502

i
pa je Pk+1 M k P1 A1 i

M A1 : A1 N = (k 1) : 1,
Pk+1 M : P1 A1 = k : 1.

Stoga se duzi A1 A1 i P1 P1 seku u izvesnoj tacki T pri cemu je


A1 T : T A1 = k : 1
i
P1 T : T P1 = k : 1.
Otuda sleduje da je tacka T zajednicko teziste poligona A1 . . . Ak+1 i P1 . . . Pk+1 .
Dakle, stav je istinit i za n=k+1. Ovim je teorema dokazana.

sl. 448
817. Ako je T teziste proizvoljnog skupa {A1 , . . . , An } od n tacaka jedne
ravni, i ako su T , A1 , . . . , An } uporedne projekcije tacaka T, A1 , . . . , An na nekoj
pravoj s koja je takode u toj ravni, dokazati da je
TT =

1
(A1 A1 + . . . + An An ).
n

Prema zadatku 177, za n=2 neposredno nalazimo da je


TT =

1
(A1 A1 + A2 A2 ).
2

Da bismo dokaz
stava
izveli A3
za n2, primenimo matematicku indukciju.
A4 ovog P
3
Stoga pretpostavimo da je N
stav istinit za n-1 tacaka, a dokazimo da je on istinit i
P 1

P2

A1
T

P4
A1

503
M

P1

A2

za n tacaka. Obelezimo sa Tn teziste podskupa {A1 , . . . , An1 } a sa Tn uporednu


projekciju tacke Tn na pravoj s. Prema zadatku 210, tacka T je izmedu tacaka
An i Tn takva da je
An T : T Tn = (n 1) : 1,
pa je prema zadatku 177
TT =

1
[(n 1)Tn Tn + An An ].
n

S obzirom da je po pretpostavci
Tn Tn =

1
(A1 A1 + . . . + An1 An1 ),
n1

bice

1
(A1 A1 + . . . + An An ).
n
818. (Lajbnicova teorema) Ako je T teziste konacnog skupa od n tacaka
{A1 , . . . , An } i P bilo koja tacka, dokazati da je:
TT =

n
X

P A2i =

n
X

P A2i + nP T 2 .

i=1

i=1

Primenimo metodu matematicke indukcije, stoga pretpostavimo da je teorema tacna kada skup ima n 1 tacaka. Ako obelezimo sa Tn teziste podskupa
{A1 , . . . , An1 }, saglasno pretpostavci, imamo da je
n1
X

P Ai 2 =

Tn Ai 2 + (n 1)P Tn 2 . . .

(3)

n1
X

Tn Ai 2 + (n 1)T Tn 2 . . .

(4)

i=1

i=1

n1
X

n1
X

T Ai 2 =

i=1

i=1

Prema poznatoj teoremi, tacka T je izmedu tacaka An i Tn , takva da je An T :


T Tn = (n 1) : 1. Otuda, primenom Apolonijeve teoreme na trougao P An Tn i
tacku T koja se nalazi na stranici An Tn tog trougla, nalazimo da je
P An 2 + (n 1)P Tn 2 = T An 2 + (n 1)T Tn 2 + nP T 2

(5)

Iz jednakosti (3), (4), (5) sledi da je


n
X
i=1

P Ai =

n
X

T Ai 2 + nP T 2

i=1

Zadatkom . . . dokazali smo da je ova teorema tacna kada je n = 3, prema tome,


ona je tacna i kada je n > 3.
819. Ako je {A1 , . . . , An } proizvoljan skup od n tacaka, T teziste tog skupa
i Ti teziste podskupa {A1 , . . . , Ai1 , Ai+1 , . . . , An }, dokazati da je:
(a) Ai Ti2 =

n
n
X
X
1
2
Aj A2k ];
A
A

[n
i
j
(n 1)2 j=1
j,k=1

504

n
X

n
X
n
Ai T =
(b)
Ai A2j ;
(n 1)2 i,j=1
i=1

(v)

n
X

(g)

n
X

n
X
1
Ai A2j , i < j;
n(n 1)2 i,j=1

T Ti2 =

i=1

Ai T 2 =

i=1

n
1 X
Ai A2j .
n i,j=1

Ovaj stav poznat je u geometriji trougla, stoga metodom matematicke indukcije dokazimo da je on istinit i za n > 3. U tom cilju, pretpostavimo da
je stav istinit za slucaj kada se skup sastoji od n 1 tacaka. (a) Primenom
Lajbnicove teoreme, nalazimo da je

n
n
X
1 X

Ai Ti2 =
Ti Aj 2 =
Ai Aj 2

n 1 j=1
i=1
j6=i

n
X

1
1

Ai Aj 2
n 1 j=1
n1

j,k=1
j,k6=i, j<k

n
X

1
(n 1)
Ai Aj 2
2
(n 1)
j=1

n
X

1
n
Ai Aj 2
2
(n 1) j=1

n
X

j,k=1
j<k

Aj Ak 2
=

n
X

j,k=1
j,k6=i, j<k

Aj Ak 2
=

Aj Ak
.

(b) Prema izvedenom delu ove teoreme imamo da je

n
n
n
n
X
X
X
X
1
n
Aj Ak 2 =
Ai Aj 2
Ai Ti 2 =
2
(n

1)
j=1
i=1
i=1
j,k=1

n
n
X
X

1
2
2n
Ai Aj 2 n

n
A
A
i j
=

2
(n 1)
i,j=1
i,j=1
i<j

n
(n 1)2

n
X

Ai Aj 2

i,j=1
i<j

505

i<j

(v) S obzirom da je Ai T =
n
X

Ai T 2

i=1

n1
Ai Ti bice
n

n
n
X
(n 1)2 X
(n 1)2
n
2
=
A
T
Ai Aj 2
i i
2
2
n2
n
(n

1)
i=1
i,j=1
i<j

n
1 X
Ai Aj 2
n i,j=1
i<j

(g) Isto tako, koristeci jednakost T Ti =


n
X

T Ti

i=1

1
Ai Ti , nalazimo da je
n

n
n
n
X
X
1
n
1
1 X
2
2
Ai Ti = 2
Ai Aj =
Ai Aj 2
= 2
n i=1
n (n 1)2 i,j=1
n(n 1)2 i,j=1
i<j

i<j

820. Ako su P i Q tezista dvaju konacnih skupova tacaka A1 , . . . , Am i


B1 , . . . , Bn , dokazati da je
m n

m
n
X
1 XX
m X
2
2
2
PQ =
Ai Bj
Ai Ak
Bj Bl .
mn i=1 j=1
n
2

i,k=1

j,l=1

Prema Lajbnicovoj teoremi i teoremi . . . imamo da je


n
X

P Bj 2 =

j=1

m
X
i=1

Ai Bj 2 =

n
X

QBj 2 + nP Q2 =

j=1

m
X

n
1 X
Bj Bl 2 + nP Q2
n
j,l=1

P Ai 2 + mBj P 2 =

i=1

m
1 X
Ai Al 2 + mBj P 2
m
i,l=1

Ako prvu od ovih jednakosti pomnozimo sa m, a drugu sumiramo po indeksu


j, dobijamo dve jednakosti iz kojih sleduje trazena relacija.
821. Ako je T teziste proizvoljnog skupa A1 , . . . , An od n tacaka i ako su P
i Q dve bilo koje tacke, dokazati da je
(P A21 + . . . + P A2n ) (QA21 + . . . + QA2n ) = n((P T 2 QT 2 ).
Prema Lajbnicovoj teoremi imamo da je
P A1 2 + . . . + P An 2 = T A1 2 + . . . + T An 2 + nP T 2 ,
QA1 2 + . . . + QAn 2 = T A1 2 + . . . + T An 2 + nQT 2
Otuda je
(P A1 2 + . . . + P An 2 ) (QA1 2 + . . . + QAn 2 ) = n(P T 2 QT 2 )
822. Odrediti skup svih tacaka kojima je zbir kvadrata rastojanja od n datih
tacaka A1 , . . . , An jednak kvadratu date duzi l.

506

Ako obelezimo sa T teziste datog skupa tacaka i sa P proizvoljnu tacku


trazenog skupa, prema Lajbnicovoj teoremi imamo da je
P A1 2 + . . . + P An 2 = T A1 2 + . . . + T An 2 + nT P 2 ,
tj. da je 12 = d2 + nT P 2 , gde je d2 = T A1 2 + . . . + T An 2 . Stoga je
1
T P 2 = (12 d2 ), pa se tacka P nalazi na krugu k kome je srediste T a
n
1
poluprecnik r = (12 d2 ). Ako je pak Q proizvoljna tacka kruga k, prema
n
Lajbnicovoj teoremi imamo da je
QA1 2 + . . . + QAn 2 = T A1 2 + . . . + T An 2 + nT Q2 = d2 + nr2 = 12
Pa je Q tacka trazenog skupa. Stoga, skup svih tacaka kojima je zbir kvadrata
rastojanja od n datih tacaka A1 , . . . , An predstavlja krug k.

507

10.2

Tetivni i tangentni poligoni

823. Dokazati da je kod prostog tetivnog poligona A1 , . . . , A2n s parnim brojem stranica zbir unutrasnjih uglova kod temena s neparnim indeksima jednak
zbiru unutrasnjih uglova kod temena s parnim indeksima.
Obelezimo sa O srediste kruga opisanog oko tetivnog poligona A1 , . . . , An .
S obzirom da je poligon A1 , . . . , A2n prost i tetivan, poluprava OA2 i OAi+1
odreduju dva ugla pri cemu su sva temena poligona, sem temena Ai i Ai+1
koji se nalaze na njegovim kracima, u jednom od tih uglova saglasimo se da
pod uglom Ai OAi+1 podrazumevamo onaj ugao u kome nisu ostala temena tog
poligona. Pri tome je
A1 =

1
(A2 OA3 + . . . + A2n1 OA2n )
2

A2 =

1
(A3 OA4 + . . . + A2n OA1 )
2
.........

1
(A1 OA2 + . . . + A2n2 OA2n1 )
2
Otuda je A1 + A3 + . . . + A2n1 = (n 1)2R i A2 + A4 + . . . + A2n =
(n 1)2R, pa je A1 + A3 + . . . + A2n1 = A2 + A4 + . . . + A2n , gde
je R prav ugao (sl. 449).
A2n =

A4

An

A3

A1

A2

sl. 449
824. Ako dva poligona A1 , . . . , A2n i B1 , . . . , B2n upisani u isti krug k imaju
2n 1 uporednih odgovarajucih stranica, dokazati da su i preostale dve odgovarajuce stranice tih poligona medu sobom uporedne.
Neka je A1 A2 k B1 B2 , A2 A3 k B2 B3 , . . . , A2n1 A2n k B2n1 B2n . Ako pod
lukom Ai Bi podrazumevamo onaj luk kruga k koji se nalazi izmedju uporednih
pravih Ai Ai+1 i Bi Bi+1 , bice luci A1 B1 i B2 A2 jednaki i istosmerni, luci B2 A2 i
A3 B3 jednaki i istosmerni, . . . , luci A2n1 B2n1 i B2n A2n jednaki i istosmerni.
Otud sleduje da su i luci B2n A2n i A1 B1 jednaki i istosmerni, pa je A2n A1 k
B2n B1 (sl. 450).

508

B2 n

B3

A2 n

A3

A1

A2
B1

B2

sl. 450
825. Ako dva poligona A1 , . . . , A2n+1 i B1 , . . . , B2n+1 upisani u isti krug
k imaju 2n uporednih odgovarajucih stranica, dokazati da su preostale dve
odgovarajuce stranice tih poligona medu sobom jednake.
Neka je A1 A2 k B1 B2 , A2 A3 k B2 B3 , . . . , A2n A2n+1 k B2n B2n+1 . Kao u
prethodnom primeru saglasimo se da pod lukom Ai Bi podrazumevamo onaj luk
kruga k koji se nalazi izmedju uporednih pravih Ai Ai+1 i Bi Bi+1 . Pri tome su
luci A1 B1 i B2 A2 jednaki i istosmerni, luci B2 A2 i A3 B3 jednaki i istosmerni,
. . . , luci B2n A2n i A2n+1 B2n+1 jednaki i istosmerni. Otud sleduje da su i luci
A2n+1 B2n+1 i A1 B1 jednaki i istosmerni, pa su i tetive A2n+1 A1 i B2n+1 B1
medju sobom jednake (sl. 451).

B3
A3

A2 n + 1
B2 n + 1

A1
B1

A2
B2

sl. 451
826. Ako su unutrasnji uglovi tetivnog poligona A1 , . . . , A2n+1 s neparnim
brojem stranica medu sobom jednaki, dokazati da je taj poligon pravilan.
Da bismo dokazali da je poligon A1 . . . A2n+1 pravilan, saglasno definiciji,
mi treba da dokazemo da su i stranice tog poligona medju sobom jednake. Iz
podudarnosti trouglova A1 A2 A3 i A4 A3 A2 sledi da je A1 A2 = A3 A4 . Isto tako
je A3 A4 = A5 A6 = . . . = A2n+1 A1 = A2 A3 = . . . = A2n A2n+1 . Dakle, sve
stranice poligona A1 . . . A2n+1 su medju sobom jednake, pa je stav dokazan (sl.
452.).

509

A2 n + 1

A3

A1

A2

sl. 452
827. Ako je P tacka kruga opisanog oko tetivnog n-tougla A1 , . . . , An , dokazati da podnozja upravnih iz tacke P na Simsonovim pravama te iste tacke u
odnosu na (n 1)-touglove A1 . . . Ai1 Ai+1 . . . An za i = 1, . . . , n pripadaju jednoj pravoj koju nazivamo Simsonovom pravom tacke P u odnosu na n-tougao
A1 , . . . , An .
Zadacima 138 i 707 dokazali smo ovaj stav za n = 3 i n = 4. Primenom
matematicke indukcije dokazimo da stav vazi i za n > 4. U tom cilju pretpostavimo da stav vazi za tetivne poligone od n 1 stranica. Obelezimo sa pi , pj , pk
Simsonove prave tacke P u odnosu da (n 1)-touglove A1 . . . Ai1 Ai+1 . . . An ,
A1 . . . Aj1 Aj+1 . . . An , A1 . . . Ak1 Ak+1 . . . An , gde medju sobom razliciti indeksi i,j,k uzimaju vrednosti 1, . . . , n. S obzirom da je n > 4 navedeni (n 1)tougli imaju najmanje jedno zajednicko teme Ap . Podnozje upravnih iz tacke
P na pravama koje spajaju teme Ap sa ostalim temenima A1 . . . An pripadaju
krugu kome je duz P Ap precnik. Prave odredjene parovima tih n 1 dobijenih
tacaka predstavljaju Simsonove prave tacke P u odnosu na trouglove kojima je
teme Ap zajednicko, dok se ostala temena poklapaju sa ostalim temenima poligona A1 . . . An . Ako pomenuti (n 1)-touglovi imaju sem Ap jos zajednickih
temena, neka je Aq bilo koje od njih, a Apq podnozje upravne iz tacke P na
pravoj Ap Aq . Tacka Apq je jedna od n 1 dobijenih tacaka. Podnozja upravnih iz tacke P na pravama koje spajaju tacku Apq sa ostalim tackama iz tog
skupa takodje pripadaju jednom krugu kome je duz P Apq precnik. Prave odredjene parovima tih n 2 tacaka predstavljaju Simsonove prave cetvorouglova
kojima su dva temena Ap i Aq zajednicka, dok se ostala temena poklapaju sa
ostalim temenima poligona A1 . . . An . Ako pomenuti (n 1)-touglovi imaju
sem Ap i Aq jos zajednickih temena, neka je An bilo koje od njih. Simsonova
prava tacke P u odnosu na trougao Ap Aq Ar sadrzi tacku Apq i jos jednu od
ranije dobijenih n 1 tacaka, pa se podnozje Apqr upravne iz tacke P na njoj
poklapa s jednom od dobijenih n 2 tacaka. Podnozja upravnih iz tacke P
na pravama koje sadrze tacku Apqr sa ostalim tackama iz tog skupa takodje
pripadaju jednom krugu kome je duz P Apqr precnik, a prave odredjene parovima tih n 3 tacaka predstavljaju Simsonove prave petouglova kojima su tri
temena Ap , Aq , Ar zajednicka, dok se ostala temena poklapaju sa ostalim temenima poligona A1 . . . An . Nastavljajuci ovaj postupak dobijamo najzad samo
tri tacke koje odredjuju jedan krug na kome se nalazi tacka P . Te tri tacke su
preseci pravih pi , pj , pk pa se podnozja Pi Pj Pk upravnih iz tacke P na pravama
pi , pj , pk nalaze na jednoj pravoj p. Analognim postupkom dokazujemo da se
510

podnozje P1 upravne iz tacke P na Simsonovoj pravoj p1 te tacke u odnosu na


(n 1)-tougao A1 . . . Ai1 Ai+1 . . . An nalazi na pravoj Pi Pj , tj. pravoj p. Ovim
je stav dokazan.
828. Ako je T teziste proizvoljnog skupa A1 , . . . , An od n tacaka nekog kruga
l(O, r), dokazati da se prave kroz tezista podskupova
A1 , . . . , Ai1 , Ai+1 , . . . , Aj1 , Aj+1 , . . . , An upravne na pravama Ai Aj (i, j =
1, . . . , n; i 6= j) seku u jednoj tacki, ortotezistu K datog skupa tacaka. Dokazati
zatim da je tacka K na Ojlerovoj pravoj OT tog skupa tacaka, pri cemu je
OT : T K = (n 2) : 2.
Obelezimo sa Tij i Tij tezista podskupova {Ai , Aj } i { A1 , . . . , Ai1 , Ai+1 ,
Aj1 , Aj+1 , . . . , An } a sa tij i tij prave kroz tacke Tij i Tij upravne na pravoj
Ai Aj . S obzirom da se duzi Tij Tij za i, j = 1, . . . , n, i 6= j seku u jednoj tacki,
tezistu T datog skupa tacaka, pri cemu je Tij T : T Tij = (n 2) : 2, a prave
tij seku u jednoj tacki, sredistu O kruga k, i njima upravne prave tij se seku u
izvesnoj tacki K koja se nalazi na Ojlerovoj pravoj OT datog skupa tacaka, pri
cemu je OT : T K = Tij T : T Tij = (n 2) : 2. Prave tij nazivamo ortotezisnim
pravama, a tacku K ortotezistem ili Kantorovom tackom datog skupa tacaka.
829. Ako je T teziste proizvoljnog skupa {A1 , . . . , An } od n tacaka nekog
kruga l(O, r), metodom matematicke indukcije dokazati da se prave kroz ortocentre podskupova {A1 , . . . , Ai1 , Ai+1 , . . . , Aj1 , Aj+1 , . . . , An } upravne na
pravama Ai Aj seku u jednoj tacki, ortocentru H datog skupa tacaka. Dokazati
zatim da je tacka H na Ojlerovoj pravoj OT datog skupa tacaka, pri cemu je
HT : T O = (n 1) : 1.
Koristeci vec uvedene pojmove za ortocentar skupa {A1 , A2 , A3 } od tri tacke
i skupa {A1 , A2 , A3 , A4 } od cetiri tacke nekog kruga i metodom matematicke
indukcije izvedimo i pojam ortocentra proizvoljnog skupa {A1 , ..., An } od n
tacaka nekog kruga l. Zato pretpostavimo da je za n 2 tacaka stav tacan,
dokazimo da je on tacan i za n-tacaka. Obelezimo sa Hij i Tij ortocentre i
tezista podskupova {A1,...,Ai1 ,Ai+1 , ..., Aj1 , Aj+1 , ..., An } , a sa hij i tij prave
kroz tacke Hij i Tij upravne na pravoj Ai Aj . S obzirom da se tacke O,Hij ,Tij
nalaze na jednoj pravoj pri cemu je Hij Tij : Tij O = (n 3) : 1, a prave tij
seku u jednoj tacki, ortotezistu datog skupa tacaka, i njima upravne prave hij
seku se u izvesnoj tacki, ortocentru H datog skupa tacaka. Pri tome je tacka H
na Ojlerovoj pravoj OT datog skupa tacaka takva da je HK : KO = Hij Tij :
Tij O = (n 3) : l. No KT : T O = 2 : (n 2), pa je HT : T O = (n 1) : l (sl.
453).
Ovako definisan ortocentar proizvoljnog skupa tacaka nekog kruga moze se
izvesti polazeci od pojma ortocentra skupa koji se sastoji iz samo jedne ili dveju
tacaka nekog kruga l. U tom slucaju bismo ortocentrom jedne tacke nekog
kruga nazivali samu tu tacku, a ortocentar dveju tacaka nekog kruga nazivali
tacku koja je simetricna sa sredistem tog kruga u odnosu na pravu odredjenu
tim dvema tackama.

511

A4
T 1.2

A5

A3

A1

A2

sl. 453
830. Ako je H ortocentar proizvoljnog skupa {A1 , . . . , An } od n tacaka nekog
kruga l(O, r), dokazati da su ortocentri Hi podskupova
{A1 , . . . , Ai1 , Ai+1 , . . . , An } simetricni sa tackama Ai u odnosu na srediste
S duzi OH. Tacku S nazivamo sredistem ili centrom, a pravu s koja je u tacki S
upravna na Ojlerovoj pravoj, nazivamo sredisnjom ili centralnom pravom datog
skupa tacaka.
Tezista T1 i T2 podskupova {A2 , A3 , ..., An } i {A1 , A3 , ..., An } su tacke duzi
OH1 i OH2 takve da H1 T1 : T1 O = H2 T2 : T2 O = (n 2) : l, pa je H1 H2 :
T1 T2 = H1 O : T1 O = (n 1) : l. Pri tome su duzi H1 H2 i T1 T2 istosmerne.Duzi
A1 T1 i A2 T2 seku se u tacki T1 tezistu datog skupa tacaka, pri cemu je A1 T :
T T = A2 T : T T2 = (n1) : l,a duz A1 A2 suprotno usmerena s duzi T1 T2 . Otuda
sledi da su duzi A1 A2 i H1 H2 jednake i suprotno usmerene pa je cetvorougao
A1 A2 H1 H2 paralelogram kome se dijagonale A1 H1 i A2 H2 seku u nekoj tacki S
koja je srediste svake od duzi A1 H1 i A2 H2 .Na taj nacin sve duzi Ai Hi seku se
u istoj tacki S koja je zajednicko srediste svih tih duzi.Primenom Menelajeve
teoreme na trougao Ai Hi Ti , nalazimo da je tacka S na pravoj OT . S obzirom
da je tacka T na tezisnoj liniji OS trougla OAi Hi , prava OAi u nekoj tacki Ui ,
pri cemu je Ai Ui : U2 O = Hi Ti : Ti O = (n 2) : l. Stoga se primenom teoreme
Van Obela, dobija da je OT : T S = 2 : (n 2). No OT : T S = 2 : (n 2),pa
su tacke S i S istovetne(sl.454).

512

Hi
H

S
Ti

Ai

Ui

sl. 454
831. Ako je skup {A1 , . . . , An } od n tacaka nekog kruga l(O, r) razlozen na
proizvoljan nacin na dva podskupa S1 i S2 tacaka, dokazati da su ortocentri tih
podskupova simetricni medu sobom u odnosu na srediste S datog skupa tacaka.
Pretpostavimo da pravih p tacka datog skupa pripadaju u podskupu S1 , a
ostalih n p tacaka pripadaju podskupu S2 . Ako dati skup tacaka nije tako
razlozen na komplementarne skupove S1 i S2 , prenumeracijom indeksa to uvek
mozemo postici.
Prema definiciji, ortocentar jedne tacke, npr.A1 je sama ta tacka, a ortocentar H1 podskupa {A2 , ..., An } je prema zadatku 830 tacka simetricna s tackom
A1 u odnosu na sredista S datog skupa tacaka. Prema tome, ortocentri podskupova {A1 } i {A2 , ..., An } simetricni su medu sobom u odnosu na tacku S.
Ortocentri H1 i H12 podskupova {A2 , ..., An } i {A3 , ..., An } simetricni su
s tackama O i a2 u odnosu na srediste skupa {A2 , . . . , An }, a tacke O i A2

simetricne su s ortocentrima H12


i H1 podskupova {A1 , A2 } i {A1 } u odnosu

na srediste podskupova {A3 , ..., An }, pa su duzi H1 H12 i H1 H12


jednake i su
protno usmerene. Otuda sledi da se srediste duzi H12 H12 poklapa sa sredistem
S duzi H1 H1 , pa su ortocentri podskupova {A1 A2 } i {A3 , ..., An } simetricni
medu sobom u odnosu na tacku S.
Ortocentri H12 i H123 podskupova {A3 , ..., An } i {A4 , ..., An } simetricni su
stackama O i A3 u odnosu na srediste podskupa {A3 , ..., An }, a tacke O i A3

simetricne s ortocentrima H123


i H12
podskupova {A4 , ..., An } i {A3 , ..., An } u

odnosu na srediste podskupa {A4 , ..., An } pa su duzi H12 H123 i H12


H123
jednake

i suprotne. Otuda sledi da se srediste duzi H123 H123 poklapa sa sredistem S duzi

H12 H12
, pa su ortocentri H123
i H123 podskupova {A1 , A2 , A3 } i {A4 , ..., An }
513

simetricni medu sobom u odnosu na tacku S.


Nastavljajuci taj postupak, dokazuje se da su ortocentri bilo kojih dveju
komplementarnih podskupova {A1 , ..., An } i {Ap+1 , ..., An } simetricni medu sobom u odnosu na tacku S.
832. Ako je S srediste, a H ortocentar proizvoljnog skupa {A1 , . . . , An } od
n tacaka nekog kruga l(O, r), dokazati da se Hi podskupova
{A1 , . . . , Ai1 , Ai+1 , . . . , An } nalaze na izvesnom krugu l kome je srediste
H, a poluprecnik jednak duzi r.
S obzirom da su tacke Ai na krugu 1, a prema zadatku 830, tacke Hi simetricne sa tackama Ai u odnosu na tacku S, bice i tacke Hi na izvesnom krugu
1 . Iz simetrije neposredno sledi da se srediste kruga 1 poklapa s ortocentrom
H datog skupa tacaka i da je poluprecnik kruga 1 jednak poluprecniku kruga
1, tj. duzi r.
833. Ako je {A1 , . . . , An } proizvoljan skup od n tacaka nekog kruga l(O, r),
dokazati da se prave ni kroz tacke Ai upravne na centralnim pravama si podskupova {A1 , . . . , Ai1 , Ai+1 , . . . , An } seku u ortocentru H datog skupa tacaka.

H1

H3
A3

H2

A4
H
O

A2

A1

slika 455
Prema zadatku 830, ortocentri Hi podskupova
A1 , ..., Ai1 , Ai+1 , ..., An
514

simetricne su s tackama Ai u odnosu na antisrediste S datog skupa tacaka, a


prava OHi simetricne s pravama ni u odnosu na istu tacku S, prema tome, sve
praveni seku se u ortocentru H datog skupa tacaka(sl.455).
834. Ako je S centar i H ortocentar proizvoljnog skupa {A1 , . . . , An } sa n
tacaka nekog kruga l(O, r), dokazati da ortocentri Si podskupova
{A1 , . . . , Ai1 , Ai+1 , . . . , An }, podnozje Bi upravnih iz tacaka Ai na centralnim pravama si podskupova {A1 , . . . , Ai1 , Ai+1 , . . . , An } i sredista Ci duzi
HAi pripadaju jednom krugu k, kome je srediste S, a poluprecnik jednak polovini duzi r. Krug k nazivamo Ojlerovim karakteristicnim krugom datog skupa
tacaka.

H1
A3

A4

S1

O
A2

A5

A1

slika 456
Prema zadatku 832, ortocentri Hi podskupova
A1 , ..., Ai1 , Ai+1 , ..., An
pripadaju krugu k (H, r), pa prema zadatku 720, tacke Si su sredista duzi OHi ,
prema tome, tacke Si pripadaju izvesnom krugu k, kome se srediste poklapa sa
sredistem duzi OH, tj. sa sredistem S datog skupa tacaka, i kome je poluprecnik
jednak polovini duzi r. S obzirom da tacke Si pripadaju krugu k, i njima simetricne tacke Ci u odnosu na tacku S takode pripadaju krugu k. Prema zadatku
732, upravne Ai Bi na pravama s2 seku u tacki H, pa su tacke Ci na pravama
Ai Bi . Takode su tacke Si na pravama OHi , pa su uglovi Si Bi Ci pravi. Kako
515

su duzi Si Ci prave, tacke Bi takode pripadaju krugu k(sl.456).


835. Dokazati da se ortopolovi Pijk centralnih pravih pijk podskupova
{A1 , . . . , Ai1 , Ai+1 , . . . , Aj1 , Aj+1 , Ak1 , Ak+1 , . . . , An }
u odnosu na trouglove odredene tackama Ai , Aj , Ak datog skupa {A1 , . . . , An }
od n tacaka nekog kruga l(O, r), nalaze na Ojlerovom karakteristicnom krugu
datog skupa tacaka.

Obelezimo sa Pijk
prave kroz srediste O kruga i uporedne s pravama Pijk ,

a sa Pijk ortopolove prave Pijk


u odnosu na trouglove Ai Aj Ak . S obzirom da

prave pijk sadrze srediste O kruga opisanog oko trouglova Ai Aj Ak , tacke Pijk
su na Ojlerovim karakteristicnim krugovima. Obelezimo sa S srediste datog

skupa tacaka, a sa Sijk i Sijk


sredista podskupova

A1 , ..., Ai1 , Ai+2 , ..., Aj2 , Aj+1 , ..., Ak Ak1 An


i
Ai , Aj , Ak .

Prena pomenutim stavovima, duziPijk


Pijk su istosmerni i jednake sa duzima

OSijk , a duzi OSijk su istosmerne i jednake sa duzima Sijk


s pa su duzi Pijk
Pijk

istosmerne i jednake s duzima Sijk . Otuda sledi da su duzi SPijk jednake s

duzima Sijk
Pijk
, pa su trouglovi Pijk pravih pijk u odnosu na trouglove Ai Aj Ak
tacke karakteristicnog kruga datog skupa tacaka.

836. Ako je H ortocentar i K ortoteziste proizvoljnog skupa {A1 , . . . , An } od


n tacaka nekog kruga l(O, r), dokazati da tezista Ti (i = 1, . . . , n) podskupova
{A1 , . . . , Ai1 , Ai+1 , . . . , An } i tacke Ei duzi KAi takve da je Ai Ei : Ei K =
(n 2) : 1, pripadaju jednom krugu l1 , kome je srediste O1 tacka duzi OH
takva da je HO1 : O1 O = (n 2) : 1, a poluprecnik r1 duz takva da je r : r1 =
(n 1) : 1.Krug l1 nazivamo Ojlerovim centralnim krugom datog skupa tacaka.

516

Ai

Ei
O1

H
K

Ti

slika 457
S obzirom da su tacke Ai na jednom krugu, a tacke Ei na duzima KAi
takode Ai K : Ei K = (n 1) i tacke Ei su na izvesnom krugu l1 kome je srediste
O1 tacka duzi OK takva da je
OK : O1 K = (n 1) : 1,
tj.
OO1 : O1 K = (n 2) : 1.
Dokazimo da tacke Ti pripadaju krugu l1 . S obzirom da je
OT : T K = (n 1) : 2,
tj.
OT =
i
O1 K =

n2
(T O1 + O1 K)
2
1
(OT + T O1 )
(n 2)

sledi da je OT : T O1 = (n 1) : 1. Najzad, iz proporcija Ai : T Ti = (n 1) : 1 i


OT : T O1 = (n 1) : 1
sledi da je Ai T : T Ti = OT : T O1 . No uglovi OT Ai iO1 T Ti su unakrsni, dakle
jednaki, pa su trouglovi OT Ai i O1 T Ti slicni. S toga je
OA1 : O1 Ti = Ai T : T Ti = (n 1) : 1.
517

Iz jednakosti OA1 : O1 Ti = (n 1) : 1 i OAi : O1 Ei = (n 1) : 1 sledi da je


O1 Ti = O1 Ei , pa su tacke Ti na krugu l1 . S obzirom da su tacke O, H, O1 , K
na jednoj pravoj, pri cemu je
KO1 : O1 O = 1 : (n 2)
i
HK : KO = (n 3) : 1,
bice HO1 : O1 O = (n 2) : 1.Iz proporcije OAi : O1 Ei = (n 1) : 1 sledi da
je r : r1 = (n 1) : 1. Iz izvedenih osobina sleduje da su tacke Ti Ei simetricne
medu sobom u odnosu na tacku O1 , pa su duzi Ti Ei precnici kruga l1 .
Ojlerov centroidni krug trougla poklapa se s Ojlerovim karakteristicnim krugom tog trougla, s toga nije potrebno ni isticati da je taj krug karakteristicni ili
centroidni krug(sl. 457).
837. Ako su K, T i O1 ortoteziste, teziste i srediste Ojlerovog centralnog
kruga l1 proizvoljnog skupa {A1 , . . . , An } od n tacaka nekog kruga l(O, r), dokazati da su tacke T i K harmonijski spregnute sa tackama O i O1 , sta vise da
su tacke T i K sredista slicnosti krugova l i l1 .
Pri resavanju zadatka 836 imali smo da je
OT : T O1 = (n 1) : 1
i
OK : KO1 = (n 1) : 1,
pa je
OT : T O1 = OK : KO1 .
Pri tome je tacka T izmedu tacaka O i O1 , a tacka K na produzenju duzi OO1 ,
pa su tacke T i K harmonijski spregnute s tackama O i O1 . S obzirom da su
poluprecnici r i r1 krugova l i l1 takvi da je
r : r1 = (n 1) : 1,
bice
OT : T O1 = OK : KO1 = r : r1 ,
pa su tacke T i K sredista slicnosti krugova l i l1 .
838. Ako je T teziste i K ortoteziste proizvoljnog skupa {A1 , . . . , An } od n
tacaka nekog kruga l(O, r), dokazati da je
n
1 X
Ai Aj ;
(a) OT 2 = r2 2
n i,j=1
(b) OK 2 =
(v) KT 2 =

n
X
1
2 2
Ai Aj 2 ];

[n
r

(n 2)2
i,j=1

n
1 X
4
2
Ai Aj 2 ], i < j.

[r

(n 2)2
n2 i,j=1

518

(a) Prema Lajbnicovoj teoremi i zadatku 819, imamo da je


OT 2 =

n
n
n
1X
1X
1 X
OA2i
T A2i = r2 2
Ai A2j .
n i=1
n i=1
n i,j=1

(b) Tacka T je na duzi OK takva da je


OK : OT = n : (n 2),
pa je
OK 2 =

n
X
1
n2
2
2 2
OT
=
[n
r

Ai A2j ].
(n 2)2
(n 2)2
i,j=1,i<j

(v) S obzirom da je
OT : T K = (n 2) : 2,
bice
KT 2 = (

2
n
1
OT 2 =
[r2 2
(n 2)2
(n 2)2
n

n
X

Ai A2j .

i,j=1,i<j

839. Ako je H ortocentar, K srediste i S srediste proizvoljnog skupa {A1 , . . . , An }


od n tacaka nekog kruga l(O, r), dokazati da je
n
n
X
X
2
2
2
Ai Aj 2 ;
HAi = n (n 1) r (n 2)
(a)
i=1

i=1

(b)

n
X

KA2i

i=1

(v)

n
X

n
X
4n
n4
2
=
Ai A2j , i < j;

r
+

(n 2)2
(n 2)2 i,j=1

SA2i =

i=1

n
n(n 2)2 2 n 4 X
Ai A2j , i < j.
r

4
4
i,j=1

(a) Teziste T datog skupa tacaka je tacka duzi OH takva da je


HT = (n 1)OT,
pa se primenom Lajbnicove teoreme i zadataka 838 i 819, dobija da je
n
X

HA2i = nHT 2 +

n
X
i=1

i=1

n(n 1)2 (r2

1
n2

T A2i = n(n 1)2 OT 2 +

n
X

Ai A2j ) +

i,j=1,i<j

n(n 1)2 r2 (n 2)

n
X

T A2i =

i=1

n
1 X
Ai A2j =
n i,j=1,i<j

n
X

Ai A2j .

i,j=1,i<j

(b) Teziste T datog skupa tacaka je tacka duzi OK takva da je


KT : T O = 2 : (n 2),

519

pa se primenom Lajbnicove teoreme i zadataka 838 i 819, dobija da je


KA2i = nKT 2 + T A2i =

4n
OT 2 + T A2 =
(n 2)2

1
1
n4
4n
4n
(r2 2 Ai A2j ) + Ai A2j =
r2 +
Ai A2j .
2
2
(n 2)
n
n
(n 2)
(n 2)2

(v) Teziste T datog skupa tacaka je tacka duzi OS takva da je


ST : T O = (n 2) : 2

pa se primenom Lajbnicove teoreme i zadataka 838 i 819, dobija da je


SA2i = nST 2 + T A2i =

n(n 2)2
OT 2 + T A2i =
4

n(n 2)2 2
1
1
n(n 2)2 2 n 4
(r 2 Ai A2j ) + Ai A2j =
r
Ai A2j .
4
n
n
4
2
840. Ako je H ortocentar, T teziste i K ortoteziste proizvoljnog skupa
{A1 , . . . , An } od n tacaka nekog kruga l(O, r), dokazati da je
n
X
Ai A2j , i < j;
(a) HO2 = n2 r2
i,j=1

(b) HT 2 = (n 1)2 r2
(v) HK 2 =

n
(n 1)2 X
Ai A2j , i < j;
n2
i,j=1

n
n2 (n 3)2 2
1 X
Ai A2j ), i < j.
(r

(n 2)2
n2 i,j=1

(a) S obzirom da je tacka T izmedu tacaka O i H takva da je


HO = mT O,
primenom zadatka 838, nalazimo da je
HO2 = n2 T O2 = n2 (r2

1
n2

n
X

i,j=1,i<j

Ai A2j = n2 r2

n
X

Ai A2j .

i,j=1,i<j

(b) Isto tako je


HT = (n 1)T O.
pa je
1
HT = (n 1) T O = (n 1) (r 2
n
2

(n 1)2 r2

(n 1)2
n2

n
X

i,j=1,i<j

(v) Iz jednakosti
HT = (n 1)T O
520

n
X

i,j=1,i<j

Ai A2j .

Ai A2j =

i
KT =
nalazimo da je
HK =
pa je
HK 2 =

2
OT
n2

n(n 3)
OT,
n2

n2 (n 3)2
n2 (n 3)2 2
1
2
OT
=
(r 2
2
2
(n 2)
(n 2)
n

n
X

Ai A2j ).

i,j=1,i<j

841. Ako je S centar i H ortocentar proizvoljnog skupa {A1 , . . . , An } od n


tacaka nekog kruga l(O, r), dokazati da je
n
X
i=1

SA2i (n 4)SH 2 = nr2 .

S obzirom da je tacka S srediste duzi OH,primenom zadataka 839 i 840,


nalazimo da je
n
X
i=1

SA2i (n 4)SH 2 =

n
X
i=1

SA2i

n4
OH 2 =
4

n
n
(n 4)n2 2 n 4 X
n(n 2)2 2 n 4 X
Ai A2j
Ai A2j ) =
r
r +
4
4 i,j=1,i<j
4
4 i,j=1,i<j

n(n 2)2
(n 4)n2 2

]r = nr2 .
4
4

842. Ako je K ortoteziste proizvoljnog skupa {A1 , . . . , An } od n tacaka nekog


kruga l(O, r), a Ti teziste podskupa {A1 , . . . , Ai1 , Ai+1 , . . . , An } dokazati da
je
(a)
KA2i =

n
n
X
X
1
2
2
A
A

Aj A2k ], j < k;
[4r
+
(n

2)
i
j
(n 2)2
j=1
j,k=1

(b)
KTi2 =

n
n
X
X
1
2
2 2
A
A

Aj A2k ], j < k.
[4(n

1)
r

(n

2)
i j
(n 1)2 (n 2)2
j=1
j,k=1

(a) Teziste T datog skupa tacaka je tacka duzi OK takva da je


OT : T K = (n 2) : 2,
521

pa se primenom Stjuartove teoreme i zadataka 819 i 838, dobija da je


KA2i =

2
2n
n
OT 2
T A2i +
OA2i =
n2
(n 2)2
n2

1
1
2n
2n
[r2 2 Aj A2k
[nAi A2j Aj A2k ] +
=
n(n 2)
(n 2)2
n
n2
1
[4r2 + (n 2)Ai A2j Aj A2k ].
(n 2)2

(b) Teziste T datog skupa tacaka je tacka duzi Ai Ti takva da je


Ai T : T Ti = (n 1) : 1,
pa se primenom Stjuartove teoreme i zadataka 838 i 819, dobija da je
KTi2 =

1
n1
[nKT 2 +
Ai Ti2 KA2i ] =
n1
n
n
X

1
4n
1
(r2 2
[
2
n 1 (n 2)
n

j,k=1,j<k

n
X
1
Ai A2j
(n
n(n 1) j=1

n
X

n
X

Aj A2k )

j,k=1,j<k

1
Ai A2j
[4r2 + (m 2)
(n 2)2
j=1
=

Aj A2k )+

n
X

Aj A2k ]]

j,k=1,j<k

n
X

1
Ai A2j
[4r2 (n 1)2
(n 1)2 (n 2)2
j=1

n
X

Aj A2k ].

j,k=1,j<k

843. Ako je T teziste poligona A1 . . . An upisanog u krug k kome je srediste


O i poluprecnik r, i ako je d duz odredena tackama O i T , dokazati da je
T A2 + . . . + T A2n = n(r2 d2 ).

Prema Lajbnicovoj teoremi, imamo da je



T A21 + . . . + T A2n = OA21 + . . . + OA2n nOT 2 = nr2 nd2 = n r2 d2 .
844. Ako je T teziste poligona A1 . . . An upisanog u krug k i ako su B1 , . . . , Bn
tacke u kojima prave A1 T, . . . , An T seku krug k, dokazati da je
An T
A1 T
+ ...+
= n.
T B1
T Bn

522

Ako obelezimo sa O i r srediste i poluprecnik kruga k, sa P i Q tacke u


kojima prava OT sece krug k i sa d duz odredenu tackama O i T , imamo da je
A1 T 2
A1 T
= 2
, te
A1 T T B1 = P T T Q = (r + d)(r d) = r2 d2 . Stoga je
T B1
r d2
koristeci prethodi zadatak, nalazimo da je
A1 T
An T
+ ...+
= n.
T B1
T Bn

845. Ako su p1 , . . . , p2n odstojanja proizvoljne tacke P kruga l od pravih


koje sadrze stranice A1 A2 , . . . , A2n A1 poligona A1 . . . A2n upisanog u krug l,
dokazati da je
p1 p3 p5 . . . pn = p2 p4 . . . p2n .
V. Z. ...
846. Ako su p1 , . . . , pn odstojanja proizvoljne tacke P kruga l od pravih koje
sadrze stranice n-tougla A1 . . . An upisanog u krug l, a q1 , . . . , qn odstojanja
tacke P od dirki kruga l u tackama A1 , . . . , An , dokazati da je
p1 p2 . . . pn = q1 q2 . . . qn .

V. Z. ...

10.3

Pravilni poligoni

Definicija 10.1. Prost ravan poligon kome su jednake sve stranice i svi unutrasnji uglovi nazivamo pravilnim. Prost ravan poligon kome je zadovoljen samo
jedan od pomenutih dvaju uslova nazivamo polupravilnim. Stoga razlikujemo
dve vrste polupravilnih poligona i to polupravilne jednakostranicne i polupravilne jednakougaone poligone.
U ovom clanu proucavace se pravilni poligoni.
847. Ako su a1 , . . . , an duzi jednake stranicama A1 A2 , . . . , An A1 n-tougla
A1 . . . An upisanog u krug l, a p1 , . . . , pn odstojanja proizvoljne tacke P luka
An A1 kruga l od pravih koje sadrze stranice A1 A2 , . . . , An A1 , dokazati da je
a2
an1
an
a1
+
+ ...+
=
.
p1
p2
pn1
pn
V. Z. ...
848. Ako je r poluprecnik i P proizvoljna tacka kruga opisanog oko pravilnog
poligona A1 . . . An , dokazati da je
P A21 + . . . + P A2n = 2nr2 .
523

S obzirom da se teziste T pravilnog poligona A1 . . . An poklapa sa sredistem


O opisanog kruga, prema Lajbnicovoj teoremi imamo da je
P A21 + . . . + P A2n = OA21 + . . . + OA2n + nOP 2 = nr2 + nr2 = 2nr2 .

849. Ako su M1 , . . . , Mn sredista stranica A1 A2 , . . . , An A1 pravilnog poligona A1 . . . An , a P proizvoljna tacka kruga k(O, r) opisanog oko tog poligona,
i a stranica tog poligona, dokazati da je
1
P M12 + . . . + P Mn2 = 2nr2 na2 .
4

Duzi P 
M1 , . . . ,P Mn su tezisne
 linije trouglova P A1 A2 , . . . ,P An A1 pa je
1
1
1
1
2
2
2
2
P M1 =
P A1 + P A2 A1 A2 , . . . ,P Mn2 =
P A21 An A21 . Sabira2
2
2
2
njem odgovarajucih strana ovih jednakosti i primenom zadatka ..., nalazimo da
je
1
1
P M12 + . . . + P Mn2 = P A21 + . . . + P A2n a2 = 2nr2 a2 .
4
4
850. Ako je r poluprecnik opisanog kruga i poluprecnik upisanog kruga
pravilnog poligona A1 . . . An , zatim P proizvoljna tacka upisanog kruga, dokazati da je
P A21 + . . . + P A2n = n(r2 + 2 ).

S obzirom da se teziste tog poligona poklapa sa sredistem O opisanog kruga


i sredistem upisanog kruga, prema Lajbnicovoj teoremi, imamo da je

P A21 + . . . + P A2n = OA21 + . . . + OA2n + nOP 2 = nr2 + n2 = n r2 + 2 .
851. Ako su A1 , . . . , An i B1 , . . . , Bn dva pravilna poligona s jednakim brojem stranica, upisana u isti krug k, i ako je P proizvoljna tacka u ravni tog
kruga, dokazati da je
P A21 + . . . + P A2n = P B12 + . . . + P Bn2 .

Teziste pravilnih poligona A1 . . . An i B1 . . . Bn poklapaju se sa sredistem O


kruga k, pa je prema Lajbnicovoj teoremi
P A21 + . . . + P A2n = nr2 + nOP 2

i
524

P B12 + . . . + P Bn2 = nr2 + nOP 2 ,

gde je r poluprecnik kruga k. Otuda je


P A21 + . . . + P A2n = P B12 + . . . + P Bn2 .

852. Ako je A1 . . . A2n pravilan poligon s parnim brojem stranica i P bilo


koja tacka njegove ravni, dokazati da je
P A21 + P A2n+1 = P A22 + P A2n+2 = . . . = P A2n + P A22n .

Ako obelezimo sa O srediste tog poligona, bice duz P O zajednicka tezisna


linija trouglova P A1 An+1 , P A2 An+2 , . . . , P An A2n , pa je
 1
1
P A21 + P A2n+1 A1 A2n+1 ,
2
4
 1
1
P O2 =
P A22 + P A2n+2 A2 A2n+2 ,
2
4

 1
1
P A2n + P A22n An A22n .
P O2 =
2
4

P O2 =

S obzirom da su leve strane ovih jednakosti jednake, a na desnim stranama


poslednji clanovi takode jednaki jer je
A1 An+1 = A2 An+2 = . . . = An A2n ,
imamo da je
P A21 + P A2n+1 = P A22 + P A2n+2 = . . . = P A2n + P A22n .

853. Ako su d1 , . . . , d2n+1 rastojanja temena A1 , . . . , An pravilnog n-tougla


A1 . . . An od proizvoljne tacke P koja se nalazi na manjem luku A1 An kruga
opisanog oko tog n-tougla, dokazati da je
1
1
1
1
1
+
+
+ ... +
=
.
d1 d2
d2 d3
d3 d4
dn1 dn
d1 dn

V. Z. ...
854. Ako su d1 , . . . , dn rastojanja temena A1 , . . . , An pravilnog n-tougla
A1 . . . An od proizvoljne tacke P koja se nalazi na manjem luku A1 An kruga
opisanog oko tog n-tougla, dokazati da je
1
1
1
1
1
+
+
+ ... +
=
.
d1 d2
d2 d3
d3 d4
dn1 dn
d1 dn

525

V. Z. ...
855. Ako su p1 , . . . , pn odstojanja proizvoljne tacke P manjeg luka An A1
kruga l opisanog oko pravilnog poligona A1 . . . An od pravih koje sadrze stranice
A1 A2 , . . . , An A1 , dokazati da je
1
1
1
1
+
+ ...+
=
.
p1
p2
pn1
pn

V. Z. ...
856. Ako su an i bn stranice dvaju pravilnih konveksnih n-touglova od kojih je prvi opisan a drugi upisan u krugu poluprecnika r, zatim d2n stranica
pravilnog konveksnog 2n-tougla opisanog oko tog istog kruga, dokazati da je
(a)
1
1
1
=
+ ;
a2n
an
bn
(b)


1
1
2
.

a2n = 4r
bn
an

Bn
B3

C2n1

C4

An
A2

C2n

M
N

B1
C1

C3
B2
C2

A1

Slika 458

A3

An
Bn

B3
O

C2n

A1

B1

C4

N B2
M

C1

C2

Slika 459
526

C3

A2

(a) Obelezimo sa A1 . . . An pravilan konveksan poligon opisan oko kruga


k(O, r), sa B1 . . . Bn pravilan konveksan poligon upisan u krugu k pri cemu se
njegova temena B1 . . . Bn poklapaju sa tackama u kojima stranice An A1 ,. . .,
An1 An dodiruju krug k, a sa C1 . . . C2n pravilan konveksan poligon opisan oko
kruga k kome se naizmenicne stranice
C2n C1 , C2 C3 , . . ., C2n2 C2n1 nalaze na stranicama An A1 , . . . , An1 An . Ako
sem toga obelezimo sa M i N tacke u kojima duz OA1 sece stranice B1 B2 i
C1 C2 , bice A1 C1 N A1 B1 M , pa je A1 C1 : A1 B1 = C1 N : B1 M . Otuda je (sl.
458) (an a2n ) : an = a2n : bn , i prema tome
1
1
1
=
+ .
a2n
an
bn

(b) Obelezimo sa A1 . . . An pravilan konveksan poligon opisan oko kruga


k(O, r), sa B1 . . . Bn pravilan konveksan poligon koji je upisan u krugu k i
kome su stranice paralelne sa odgovarajucim stranicama poligona A1 . . . An , a
sa C1 . . . Cn pravilan konveksan poligon koji je opisan oko kruga k i kome su
naizmenicne stranice C1 C2 , C3 C4 , . . . ,C2n1 C2n na stranicama A1 A2 , A2 A3 ,
. . . ,An A1 . Ako sem toga obelezimo sa M tacku u kojoj stranica A1 A2 dodiruje
krug k, sa N tacku u kojoj i prava OM sece stranice B1 B2 i sa K tacku prave
B1 B2 takvu da je C1 K = OM , bice B1 C1 K OBM1 , pa je B1 C1 : OB1 =
C1 K : B1 K, tj. O2n : 2r = 2M N : bn (1). No OM : ON = A1 M : B1 N , pa je
(OM ON ) : OM = (A1 M B1 N ) : A1 M , tj. M N : r = (an bn ) : a . . . (2).


1
1
2
.

a2n = 4r
bn
an
857. Ako je an stranica pravilnog konveksnog n-tougla upisanog u krug
poluprecnika r, a an stranica pravilnog konveksnog n-tougla opisanog oko tog
istog kruga, dokazati da je
2ran
an = 2
.
4r a2n

527

A1
A1

A2
A2

Slika 460
Neka je A1 ,...,An pravilan konveksan poligon upisan u krug k poluprecnika
r, a A1 ,...,An pravilan konveksan poligon opisan oko kruga k, zatim O srediste
kruga k, a D i D podnozja upravnih kroz O na stranicama A1 A2 i A1 A2 . Iz
slicnih trouglova OA1 D i OA1 D nalazimo da je
A1 D : A1 D = OD : OD
(sl.460).
S obzirom da je A1 D = 12 an , A1 D = 21 an , OD = r i

bice an : an = r :

1
2

1
1p 2
4r a2n ,
OD2 = r2 a2n =
4
2

p
4r2 a2n pa je

2ran
an = p
.
4r2 a2n

858. Ako je an stranica pravilnog konveksnog n-tougla upisanog u krug


poluprecnika r, a a2n stranica pravilnog konveksnog 2n-tougla upisanog u taj
isti krug, dokazati da je
q
p
a2n = 2r2 r 4r2 a2n .

528

S
A

B
C

Slika 461
Ako je AB stranica pravilnog konveksnog n-tougla upisanog u krug k poluprecnika r, a C srediste kruznog luka AB na kome nisu ostala temena tog
poligona, bice tetiva AC stranica pravilnog 2n-tougla upisanog u isti krug k.
Neka je CD precnik i O srediste kruga k, a S presek tetiva AB i CD. Kod
trougla ACD ugao A je prav, a tacka S podnozje visine iz temena A, pa je
AC 2 = CD CS (sl.461).
S obzirom da je AC = a2n , CD = 2r i
r
1p 2
1
4r a2n
CS = OC OS = r r2 a2n = r
4
2
p
bice a22n = 2r(r 21 4r2 a2n ) pa je
a22n = 2r2 r

4r2 a2n .

859. Ako je an stranica pravilnog konveksnog n-tougla upisanog u krug


poluprecnika r, a a3n stranica pravilnog 3n-tougla upisanog u taj isti krug,
dokazati da je
a33n 3r2 a3n + r2 an = 0.

529

Slika 462
Ako je AB stranica pravilnog konveksnog n-tougla upisanog u krug k poluprecnika r, a C i D tacke koje dele na tri jednaka dela onaj luk AB kruga k
na kome nisu ostala temena tog n-tougla, bice tetive AC, CD i DB stranice
pravilnog konveksnog 3n-tougla upisanog u taj isti krug (sl.462).
primenom ptolemejeve teoreme na cetvorougao acdb nalazimo da je
ad bc = ab cd + ac bd. s obzirom da je ad = bc = d, bice
d2 = an a3n + a23n .

(6)

kako je tacka d sredi


ste kruznog luka bc, prema prethodnom zadatku, imamo da
je a23n = 2r2 r 4r2 d2 , odakle dobijamo da je
d2 = 4a23n

a43n
.
r2

(7)

Iz jednakosti (1) i (2) nalazimo da je


a33n 3r2 a3n + r2 an = 0.
860. Dva pravilna poligona A1 . . . An i A1 . . . A2n imaju jednake obime. Ako
je r poluprecnik opisanog i poluprecnik upisanog kruga poligona A1 . . . An ,
a r poluprecnik opisanog i poluprecnik upisanog kruga poligona A1 . . . A2n ,
dokazati da je
p
1
= (r + ),
r = r .
2

530

A3

B5

B4

C3

C2

O
B6

B3

A1

B1

A2

B2

C1

Slika 463
Obelezimo sa l krug opisan oko poligona A1 . . . An i sa C1 . . . Cn sredista
manjih lukova kruga l koji odgovaraju stranicama A1 A2 . . . An A1 . Poligon
A1 C1 . . . An Cn je pravilan, pa su tacke B1 . . . B2n , u kojima njegov upisani krug
l dodiruje stranice A1 C1 . . . Cn A1 , takode temena pravilnog poligona B1 . . . B2n .
Stranica B1 B2 poligona B1 . . . B2n jednaka je polovini stranice A1 A2 poligona
A1 . . . An , te su obimi tih pravilnih poligona medu sobom jednaki. Ako su M
i N sredista stranica A1 A2 i B1 B2 , a O zajednicko srediste tih dvaju poligona
(sl.463), imamo da je OA1 = r, OB1 = r , OM = , ON = .

No ON = 21 (OC1 +OM ) i OB12 = OC1 ON , pa je = 12 (r+) i r = r .


861. Ako je pn obim pravilnog n-tougla opisanog oko kruga k, qn obim
pravilnog n-tougla upisanog u tom krugu, p2n obim pravilnog 2n-tougla oko
kruga k i q2n obim pravilnog 2n-tougla upisanog u tom krugu, dokazati da je:
1
1
1
1
= (
+ ),
p2n
2 pn
qn
r
1
1
1
=

.
q2n
p2n q2n

531

F
C

slika 464
Obelezimo sa O srediste kruga k, sa AB stranicu pravilnog n-tougla upisanog
u krug k, a sa AC stranicu pravilnog 2n-tougla upisanog u taj isti krug (v.sl.464).
S obzirom da su obimi dvaju pravilnih poligona s jednakim brojem stranica,
srazmerni poluprecnicima upisanih krugova imamo da je
AO
OC
pn
=
=
qn
OD
OD
i
p2n
OC
OE
=
=
.
q2n
OE
OF
Otuda je

OD
OC
=
1
1
pn
qn
i

OE
OF
=
.
1
1
p2n
q2n
Iz slicnih trouglova OEC i ACD nalazimo da je
OC
AC
q2n
,
=
=
OE
AD
qn
tj .da je

OC
OE
=
,
1
1
qn
q2n
532

pa je

OC
OF
OE
OD
=
=
=
.
1
1
1
1
pn
qn
p2n
q2n

Otuda iz jednakosti
OF =

1
(OC + CD)
2
i

OE =
sledi da je

q2n
,
OC OF

1
1
1
1
= (
+ )
p2n
2 pn
qn
i
1
=
q2n

1
1

.
p2n q2n

862. Ako je A1 ...A7 pravilan sedmougao dokazati da je:


1
1
1
=
+
.
A1 A2
A1 A3
A1 A4
Primenom Ptolomejeve teoreme na cetvorougao A1 A3 A4 A5 nalazimo da je
A1 A3 A4 A5 + A3 A4 A5 A1 = A1 A4 A3 A5 .
Deljenjem obe strane ove jednakosti sa A1 A2 A1 A3 A1 A4 i koristeci jednakosti
A1 A2 = A3 A4 = A4 A5 , A1 A3 = A3 A5 , A1 A4 = A1 A5
dobijamo da je

1
1
1
=
+
.
A1 A2
A1 A3
A1 A4

863. Ako ie O prasecna tacka dijagonala AC i BD pravilnog konveksnog


petougla ABCDE dokazati da je:
AO2 = AC OC.
E

A
D
S
O
B

Slika 465
533

Iz podudarnosti trouglova ABC i DCB (sl.465) sledi da je BCA = CDB,


tj. da je BCD = CBO, i prema tome, OC = OB. Jednakokraki trouglovi
ABC i BOC imaju zajednicki ugao kod temena C, dakle slicni su, pa je
BC : AC = OC : BC
i prema tome
BC 2 = AC OC.
S obzirom da su kod trougla AOB uglovi O i B jednaki, jednake su i duzi
AB i AO. Stoga je BC = AB = AO, pa je i
AO2 = AC OC.

864. Dokazati da je stranica pravilnog konveksnog desetougla upisanog u


krug jednaka vecem odsecku poluprecnika podeljenog zlatnim presekom.

L
A
B

Slika 466
Neka je AB stranica pravilnog konveksnog desetougla upisanog u krug k sa
sredistem O i L tacka u kojoj simetrala ugla OAB sece duz OB(sl.466). S
obzirom da je
4R
AOB =
,
10
gde je R prav ugao, bice
OAB =

1
4R
8R
(2 R
)=
.
2
10
10

4R
Otuda sledi da je OAL =
, pa je u trouglu OAL OAL = AOL,
10
i prema tome OL = AL. U trouglu ABL takode je ABL = ALB, pa je
AL = AB. S obzirom da je L tacka u kojoj simetrala ugla A trougla OAB sece
stranicu OB, imamo da je
OL : LB = OA : AB,
534

pa je
OL : LB = OB : OL,
i prema tome
OL2 = OB LB.
865. Ako je a10 stranica pravilnog konveksnog desetougla, a b10 stranica
pravilnog konveksnog desetougla, koji su upisani u isti krug poluprecnika r,
dokazati da je:
(a)
b10 a10 = r,
(b)

a10 b10 = r2 .
(a)
A2
A3
A1

M
L

A4

A10
O

A5

A9
A6
A8
A7

Slika 467
Ako je A1 . . . A10 pravilan konveksan desetougao upisan u krug k, njegova dijagonala A1 A4 bice stranica pravilnog konveksnog (zvezdastog) desetougla upisanog
u taj isti krug. Dokazimo da je A1 A4 A1 A2 = r. U tom cilju, obelezimo sa
O srediste kruga k i sa L presek duzi OA2 i A1 A4 (sl.467). Ako sa R obelezimo
prav ugao, imamo da je
A1 OA2 =

4R
2R
=
,
10
5

pa je u jednokrakom trouglu OA1 A2


OA1 A2 =

4R
4R
1
(2 R
)=
.
2
10
5

Posto je OA1 A2 A6 A1 A2 a poluprava A2 A4 simetrala ugla A6 A1 A2 , to


je
2R
1
.
OA1 L OA1 A2 =
2
5
535

Kako je A1 LA2 spoljasnji ugao trougla OA1 L, imamo da je


A1 LA2 = LOA1 + OA1 L =

4R
.
5

Stoga je A1 LA2 = A1 A2 L, i prema tome A1 A2 = A1 L. U trouglu OLA4 je


OLA4 = A1 LA2 =

4R
5

4R
,
5
pa je OLA4 = LOA4 i prema tome OA4 = LA4 . S obzirom da je A1 A4
A1 L = LA4 , bice A1 A4 A1 A2 = OA4 , pa je b10 a10 = r.
LOA4 =

866. Ako je a10 stranica pravilnog konveksnog desetougla upisanog u krug


poluprecnika r, a b10 stranica pravilnog zvezdastog desetougla upisanog u taj
isti krug, dokazati da je:
(a)
r
a10 = ( 5 1),
2
(b)
r
b10 = ( 5 + 1).
2
(a)
Ako je OA poluprecnik kruga, K tacka tog poluprecnika takva da je OK 2 =
OA KA, prema zadatku 864. bice a10 = OK. Otuda je
a210 = r(r a10 ),
i prema tome
a10 =

r
( 5 1).
2

(b)
Prema zadatku 865. imamo da je b10 a10 = r, pa je
b10 = a10 + r,
r
i prema tome b10 = ( 5 + 1).
2
867. Ako je a5 stranica pravilnog konveksnog petougla upisanog u krug k
poluprecnika r, a b5 stranica pravilnog zvezdastog petougla upisanog u taj isti
krug, dokazati da je:
(a)
q

r
102 2 5,
a5 =
2
(b)
r
b5 =
2

(10 + 2 5).
536

(a) Stranica a5 pravilnog konveksnog petougla upisanog u krug k poluprecnika r jednaka je dijagonali A1 A3 pravilnog konveksnog desetougla A1 ... A10
upisanog u taj isti krug. Ako obelezimo sa O srediste kruga k, dijagonala A1 A3
bice jednaka dvostrukoj visini A2 M trougla OA1 A2 . Tu visinu, prema tome i
stranicu a5 mozemo odrediti u funkciji poluprecnika r, jer su i stranice trougla poznate u funkciji poluprecnika r. Medutim, visinu A1 M trougla OA1 A2
mozemo odrediti na jos jedna nacin. U tom cilju, obelezimo sa L tacku u kojoj
se seku duzi OA2 i OA1 A4 . Visina A1 M trougla OA1 A2 poklapa se sa visinom
temena A1 trougla OA1 L. Prema zadatku 865, trougao OA1 L je jednakokrak
(OL = A1 L), pa je visina iz temena A1 trougla OA1 L jednaka visini ON iz
temena O tog istog trougla. Visina ON iz temena O trougla OA1 L poklapa se
sa visinom iz temena O trougla OA1 A4 , pa je:

a5 =

r
1
1
2
A1 A3 = 2A1 M = 2ON = 2
A1 A4 = 2 r2 b210 =
4
4
q

r
10 2 5.
=
2
OA22

(b) Stranica b5 pravilnog zvezdastog petougla upisanog u krug poluprecnika


r, jednaka je dijagonali A1 A5 pravilnog konveksnog desetougla A1 ...A10 upisanog u taj isti krug. Dijagonala A1 A5 je kateta pravouglog trougla A1 A5 A6 , pa je:

b5

r
q
q
r2
=
(A1 A26 A5 A26 ) = (4r2 a210 ) = 4r2
( 5 1) =
4
q

r
10 + 2 5.
=
2

868. Ako je a8 stranica pravilnog konveksnog osmougla upisanog u krug k


poluprecnika r, a b8 stranica pravilnog zvezdastog osmougla upisanog i taj isti
krug, dokazati da je:
(a)
q

a8 = r 2 2
(b)
b8 = r

2+

537

2.

A5
A6

A4

A7

A3
O

A8

A2
A1

slika 468
Ako je A1 ... A8 pravilan konveksan osmougao upisan u krug k, bice A1 A2 =
a8 i A2 A5 = b8 . S obzirom da je dijagonal A1 A5 precnik kruga k, a ugao A2
trougla A1 A2 A5 je prav. Ako obelezimo sa O srediste kruga k i sa D podnozje
visine iz temena A2 trougla A1 A2 A5 , bice DA1 = r OD i DA5 = r + OD
r
2. Otuda je
(sl. 468). Pravougli trougao DOA2 je jednokrak, pa je OD =
2

r
DA1 = r
2 i DA5 = r + r2 2. Iz pravouglog trougla A1 A2 A5 nalazimo
2
2
da je A1 A2 = 2r DA1 i A2 A25 = 2r DA5 , pa je :
r
q

r
2r (r ) 2 = r 2 2
2
r
q
p

r
b8 = A2 A5 = 2r DA5 = 2r (r + ) 2 = r 2 + 2.
2
a8 = A1 A2 =

p
2r DA1 =

(1)
(2)

869. Ako je a12 stranica pravilnog konveksnog dvanaestougla, a b12 stranica


pravilnog zvezdastog dvanaestougla i a4 stranica kvadrata, koji su svi upisani u
isti krug, dokazati da je:
b12 a12 = a4 .
A7

A8

A6

A9

A5

A10

A4

A11

K
A12

A3
A2

A1

538

slika 469
Ako je A1 ... A12 pravilan konveksan dvanaestougao bice
A1 A2 = a12 , A1 A6 = b12 , A2 A5 = a4 .
Obelezimo sa K tacku u kojoj prava kroz teme A2 uporedna sa stranicom A5 A6
sece dijagonalu A1 A6 . Unutrasnji uglovi trougla KA1 A2 su jednaki, pa je on
jednakostranican. Otuda je:
A1 A6 A1 K = KA6 , tj.
A1 A6 A1 A2 = A2 A5
i prema tome: b12 a12 = a4 (sl. 469).
869/2. Ako je a12 stranica pravilnog konveksnog dvanaestougla A1 ...A12 , a
a4 duz jednaka dijagonali A1 A4 i rho poluprecnik upisanog kruga, dokazati da
je:
a12 + a4 = 2.
Prvi nacin: Prema prethodnom zadatku imamo da je a12 + a4 = b12 , no
b1 2 = 2, pa je a12 + a4 = 2.

Drugi nacin: Cetvorougao


A1 A2 A3 A4 je jedakokraki trapez kome su stranice A1 A2 , A2 A3 , A3 A4 medu sobom jednake, a ugao A4 A1 A2 jednak trecini

pravog ugla. Ako obelezimo sa A2 i A3 upravne projekcije temena A2 i A3 na


dijagonale A1 A4 i sa R prav ugao, bice:
A1 A4

=
=

A1 A2 + A2 A3 + A3 A4 = A2 A3 + 2A1 A2 =

a12 + a12 cos = a12 (1 + 3).


3

Otuda je:
A1 A2 + A1 A4 = a12 (2 +

3).

Medutim,
a12 = 2 tg

2
.
= 2 (2 3) =
6
2+ 3

870. Ako je a12 stranica pravilnog konveksnog dvanaestougla upisanog u


krug k polupecnika r, a b12 stranica pravilnog zvezdastog dvanaestougla upisanog u taj isti krug, dokazati da je:
(a)
q

a12 = r 2 3;
(b)

b12 = r

2+

539

3.

A7

D
A12

A2

A1

slika 470
Ako je A1 ...A12 pravilan konveksan dvanaestougao upisan u krug k poluprecnika
r, bice A1 A2 = a12 i A2 A7 = b12 . S obzirom da je dijagonala A1 A7 precnik
kruga k, ugao A2 trougla A1 A2 A7 je prav. Ako obelezimo sa O srediste kruga
k i sa D podnozje visine iz temena A2 trougla A1 A2 A7 , bice DA1 = r OD i
DA7 = r + OD (sl. 470). Duz OD je visina jednakostranog trougla OA2 A12 , pa
r
r
3, i prema tome DA1 = r
3, Iz pravouglog trougla A1 A2 A7
je OD =
2
2
2
2
nalazimo da je A1 A2 = 2r.DA1 i A2 A7 = 2rDA, pa je:
(a)
r
q
p

r
3) = r 2 3;
a12 = A1 A2 = 2r.DA1 = 2r(r
2
(b)

b12 = A2 A7 =

p
2r.DA7 =

r
q

r
2r(r +
3) = r 2 + 3.
2

871. Ako je a15 stranica pravilnog konveksnog petnaestougla upisanog u krug


(1) (2) (3)
poluprecnika r, a b15 , b15 , b15 stranice pravilnih zvezdastih petnaestouglova
upisanih takode u krug poluprecnika r, dokazati da je:
(a)
q

r
a15 = [ 10 + 2 5 3( 5 1)];
4
(b)
q

r
(1)
b15 = [ 3( 5 + 1) 10 2 5];
4
(v)
q

r
(2)
b15 = [ 3( 5 1) + 10 2 5];
4
540

(g)
(3)

b15 =

r
[ 3( 5 + 1) +
4

10 2 5].

M
N
A

slika 471
(a) Obelezimo sa k krug kome je srediste O i poluprecnik r, a sa A,B,C tacke
toga kruga takve da su konveksni uglovi AOC i BOC istosmerni, a tetive AC
i BC jednake sranicama pravilnog sestougla i pravilnog konveksnog desetougla
koji su upisani u taj isti krug. Pri tome, ako obelezimo sa R prav ugao, imacemo
da je (sl. 471):
AOB = AOC BOC =

4R 4R
4R

=
5
10
15

, pa je tetiva AB stranica pravilnog konveksnog petnaestougla upisanog u taj


isti krug. Odredimo tu stranicu AB u funkciji p poluprecnika r kruga K. Neka
je D podnozje visine iz temena C trougla ABC. S obzirom da je ugao B trougla
ABC tup, bice D iza B u odnosu na A, pa je AB = ADBD. Da bismo odredili
tetive AD i BD, obelezimo sa M i N srediste tetiva AC i BC. Iz podudarnosti
trouglova ACD i OCN nalazimo da je
r
1
AD = ON = r2 a210 ,
4
a iz slicnih trouglova BCD i COM , nalazimo da je CD =
BD =

1
BC, pa je
2

1
a210 a210 .
4

Otuda, primenom zadatka ...., nalazimo da je


r
r
1 2
1
2
a15 = AB = AD BD = r a10 a210 a210 =
4
4
541


r
[ 10 + 2 5 3( 5 1)].
4
U ostala tri dela ovog zadatka primenjuje se analogan postupak. Razlika je
samo u tome sto tacke A,B,C kruga k zadovoljavaju sledece uslove. Kod prvog
zvezdastog petnaestougla moraju uglovi AOC i BOC da budu istosmerni, a
tetive AC i BC da budu jednake stranici pravilnog zvezdastog desetougla i
stranici pravilnog sestougla, jer je u tom slucaju
=

AOB = AOC BOC = 3

4R
4R 4R

=2 .
10
6
15

Kod drugog zvezdastog petnaestougla, konveksni uglovi AOC i BOC moraju da budu suprotno usmereni, a tetive AC i BC jednke stranici pravilnog
konveksnog desetougla i stranici pravilnog sestougla, jer je u tom slucaju
AOB = AOC + BOC =

4R 4R
4R
+
=4
.
10
6
15

Kod treceg zvezdastog petnaestougla, konveksni uglovi AOC i BOC moraju da budu suprotno usmereni, a tetive AC i BC jednake stranici pravilnog
zvezdastog desetougla i stranici pravilnog sestougla, jer je u tom slucaju
AOB = AOC + BOC = 3

4R 4R
4R
+
=7 .
10
6
15

872. Ako su a5 , a6 , a10 stranice pravilnog konveksnog petougla, sestougla,


desetougla koji su upisani u isti krug k, dokazati da je
a26 + a210 = a25 .
Ako je r poluprecnik kruga k, prema zadacima 866 i 867 nalazimo da je
a26 + a210 = r2 +

r2
r2
( 5 1)2 = (10 2 5)2 = a25 .
4
4

873. Ako su a6 , b5 , b10 bilo stranice pravilnog sestougla, pravilnog zvezdastog


petougla i pravilnog zvezdastog desetougla koji su upisani u isti krug k, dokazati
da je
a26 + b210 = b25 .
Ako je r poluprecnik kruga k, prema zadacima 866 i 867 nalazimo da je
a26 + b210 = r2 +

r2
r2 4( 5 + 1)2 = (10 + 2 5) = b25 .
4

874. Ako su a3 , a10 , b10 bilo stranice pravilnog trougla, pravilnog konveksnog desetougla i pravilnog zvezdastog desetougla koji su upisani u isti krug k,
dokazati da je
a210 + b210 = a23 .
542

Ako je r poluprecnik kruga k, prema zadatku 865 bice


a10 .b10 = r2 , b10 a10 = r
pa je

a210 + b210 = (a10 b10 )2 + 2a10 b10 = r2 + 2r2 = 3r2 = (r 3)2 = a23 .

11
11.1

I ODREDIVANJE
RAZLAGANJE POVRSI

POVRSINA
Razlaganje povr
si

875. Dokazati da se ravan moze razloziti na pravilne, podudarne poligonske


povrsi na pet razlicitih nacina.

Slika 472 a)

Slika 472 b)

Slika 472 v)
543

Slika 472 g)

Slika 472 d)
Treba razlikovati slucaj kada se sve stranice poligona poklapaju sa stranicama susednih poligona i slucaj kada se izvesne stranice samo delimicno poklapaju sa stranicama susednih poligona.
U prvom slucaju, u svakom temenu sustice se k pravilnih podudarnih ntouglova, pa je
(n 2) 2R
4
= 4R, tj. k = 2 +
,
n
n2
gde je R prav ugao. Da bi k bio ceo broj, mora biti broj 4 deljiv sa n 2, a to ce
biti samo ako je n = 3, n = 4 ili n = 6. Odatle sledi da se u prvom slucaju ravan
moze razloziti na podudarne jednakostranicne trougaone povrsi [ sl. 472 a) ], na
podudarne kvadratne povrsi [ sl. 472 b) ] i na podudarne pravilne sestougaone
povrsi [ sl. 472 v) ].
U drugom slucaju, k pravilnih podudarnih n-touglova imaju za zajednicko
teme unutrasnju tacku stranice njima susednog poligona, pa je
k

(n 2) 2R
2
= 2R, tj. k = 1 +
,
n
n2
gde je R prav ugao. Da bi k bio ceo broj, mora biti broj 2 deljiv sa n 2, a to
ce biti samo ako je n = 3 ili n = 4. Odatle sledi da se u drugom slucaju ravan
moze razloziti na podudarne jednakostranicne trougaone povrsi [ sl. 472 g) ] i
podudarne kvadratne povrsi [ sl. 472 d) ].
876. Dokazati da se ravan moze razloziti:
(a) na trougaone povrsi koje su podudarne s proizvoljnom trougaonom povrsi;
(b) na cetvorougaone povrsi koje su podudarne s proizvoljnom
cetvorougaonom povrsi;
(v) na sestougaone povrsi koje su podudarne s proizvoljnom centralno
simetricnom sestougaonom povrsi.
k

544

Slika 473 a)
A1
C

D1

B
A

C1

A
B

Slika 473 b)

C
F
B

Slika 473 v)
a) S obzirom da se od dveju podudarnih trougaonih povrsi moze obrazovati paralelogramska povrs, a ravan moze razloziti na takve paralelogramske
povrsi, ravan se moze razloziti i na trougaone povrsi podudarne s proizvoljnom
trougaonom povrsi [ sl. 473 a) ].
b) Neka je (ABCD) bilo koja cetvorougaona povrs. Konstruisimo njoj
simetricnu povrs (BAC D ) u odnosu na srediste stranice AB, zatim povrs
545

(B A D C ) simetricnu povrsi (BAC D ) u odnosu na srediste stranice C D , i


tako dalje.
Neka je povrs (CBD1 A1 ) simetricna povrsi (ABCD) u odnosu na srediste
stranice BC. Uglovi D BD1 i A1 D1 B su medju sobom podudarni i naizmenicni,
a duzi BD i D1 A1 podudarne pa je tacka D simetricna tacki A1 u odnosu na
srediste stranice BD1 . Ako je C1 tacka simetricna tacki C u odnosu na srediste
duzi BD1 , bice povrs (D1 BD C1 ) simetricna povrsi (BD1 A1 C) u odnosu na
srediste duzi BD1 .
Postupak mozemo neograniceno nastaviti, razlazuci ravan na cetvorougaone
povrsi koje su simetricne susednim, prema tome i podudarne povrsi (ABCD) [
sl. 473 b) ].
v) S obzirom da se dijagonalom AD centralno simetricna sestougaona povrs
(ABCDEF ) razlaze na dve podudarne cetvorougaone povrsi, medju sobom simetricne u odnosu na srediste te dijagonale,kao u delu b) ovog zadatka, ravan
se moze razloziti na takve cetvorougaone povrsi. Ako se u tako dobijenoj mrezi
izostave sve duzi koje u simetrijama odgovaraju pomenutoj dijagonali, dobija se
mreza koja razlaze ravan na sestougaone povrsi podudarne povrsi (ABCDEF )
[ sl. 473 v) ].
877. Dokazati da se ravan moze razloziti na trougaone povrsi tako da se u
svakom temenu sustizu:
(a) po tri trougaone povrsi;
(b) po cetiri trougaone povrsi;
(v) po pet trougaonih povrsi;
pri cemu ni jedno teme tih povrsi nije unutrasnja tacka stranice susedne
povrsi.

D
A

B
C

546

slika 474 a)

3
1

slika 474 b)

547

P1
13

12
5

11

14
4

3
1
10

7
P2

15

P3

16

slika 474 v)
(a) Ako bi se ravan mogla razloziti na trougaone povrsi tako da se u svakom
temenu susticu po tri povrsi, bilo koje dve susedne od tih povrsi, npr. povrsi
1 = (ABC) i 2 = (ACD) obrazuju cetvorougaonu povrs (ABCD). S obzirom
da je zbir unutrasnjih uglova cetvorougaone povrsi pun ugao, ne mogu dva
unutrasnja ugla te povrsi biti konkavna, prema tome, bar jedan od unutrasnjih
uglova A i O povrsi (ABCD) je konveksna. Ugao trece trougaone povrsi 3 kod
tog temena bice konkavan ugao, sto je nemoguce. Otud sleduje da se ni ravan
ne moze razloziti na trougaone povrsi tako da se u svakom temenu susticu po
tri povrsi (sl.474a).
(b) Pretpostavimo da se ravan moze razloziti na trougaone povrsi tako da
se u svakom temenu susticu po cetiri povrsi. Obelezimo sa 1 bilo koju od njih,
a sa 2 , 3 , 4 njoj susedne povrsi (sl.474b).
Te cetiri povrsi obrazuju sestougaonu povrs u cijim se trima nesusednim
temenima susticu po tri povrsi, a u ostalim temenima po jedna povrs. Unutrasnji
uglovi kod temena u kojima se susticu tri povrsi su konkavni, jer uglovi cetvrtih
povrsi koje se susticu u tim temenima moraju biti konveksni. Otud sleduje da
te cetvrte povrsi sa povrsima 1 , 2 , 3 , 4 obrazuju trougonu povrs u cijim
ses temenima susticu po tri povrsi. Otud sleduje da uglovi cetvrtih povrsi kod
temena povrsi moraju biti konkavni, sto je nemoguce. Prema tome, ne moze
se ni ravan razloziti na trougaone povrsi tako da se u svakom temenu susticu
po cetiri povrsi.
(v) Pretpostavimo da se ravan moze razloziti na takvo mnostvo trougaonih
povrsi. Obelezimo sa 1 bilo koju od tih povrsi, 2 , 3 , 4 njoj susedne povrsi
548

(sl.474v).
U svakom temenu povrsi 1 susticu se po tri pljosni, u njima se moraju susticati jos po dve, obelezimo ih sa 5 i 6 , 7 i 8 , 9 i 10 . Tih deset trougaonih
povrsi obrazuju sestougaonu povrs kod koje se u trima neuzastopnih temenima
P1 , P2 , P3 susticu po dve trougaone povrsi, a u ostalim temenima Q1 , Q2 , Q3
po tri trougaone povrsi. Stoga se u temenima Q1 , Q2 , Q3 moraju susticati jos
po dve trougaone povrsi, obelezimo ih sa 11 , 12 , 13 14 , 15 , 16 . Zajedno
sa njima, u temenima P1 , P2 , P3 susticu se po cetiri trougaone povrsi, kojima je
zbir uglova u svakom temenu konkavan ugao, zato sto uglovi petih povrsi 17 ,
18 , 19 koje se susticu u tim temenima moraju biti konveksni. Otud sleduje
da povrsi 1 , ... , 19 obrazuju jednu trougaonu povrs u cijim se temenima
susticu po cetiri povrsi, pa ce uglovi kod tih temena petih trougaonih povrsi biti
konkavni, sto je nemoguce. Stoga se ni ravan ne moze razloziti na trougaone
povrsi tako da se u svakom temenu sustice po pet povrsi.
878.Dokazati da se konveksna poligonska povrs s neparnim brojem stranica
ne moze razloziti na paralelogramske povrsi.
N

M
k

2
D1

C1
1

slika 475
Pretpostavimo da je konveksna poligonska povrs s neparnim brojem stranica razlozena na paralelogramske povrsi. Proizvoljna stranica AB povrsi
sadrzi stranicu A1 B1 jedne od paralelogramskih povrsi, npr. 1 . Naspramna
stranica C1 D1 povrsi 1 sadrzi celu ili samo deo stranice A2 B2 susedne paralelogramske povrsi 2 . Naspramna stranica C2 D2 povrsi 2 sadrzi celu ili samo deo
stranice A3 B3 susedne paralelogramske povrsi 3 , itd. Na taj nacin dolazimo
549

do paralelogramske povrsi k kojoj se stranica Ck Dk nalazi na jednoj stranici


M N povrsi .Otud sleduje da su stranice AB i M N poligonske povrsi medju
sobom uporedne. S obzirom da je povrs konveksna samo dve njene stranice
mogu biti uporedne medju sobom, prema tome, svakoj stranici povrsi odgovara jedna njoj uporedna stranica te iste povrsi . Stoga su stranice povrsi
dve po dve medju sobom uporedne sto ne moze biti jer je broj stranica povrsi
omega neparan. Otud sleduje da se i konveksna poligonska povrs s neparnim
brojem stranica ne moze razloziti na paralelogramske povrsi (sl.475).
879. Dokazati da svaka konveksna centralno simetricna poligonska povrs se
moze razloziti na paralelogramske povrsi.

Ak
A2k1
Ak1
A2k
A2

A1

Slika 476
Neka je (A1 , . . . , An ) proizvoljna centralno simetricna poligonska povrs.S obzirom da svaka centralno simetricna poligonska povrs ima paran broj temena,
bice n = 2k. Ako obelezimo sa A1 A1 , A2 A2 , . . . , Ak2 Ak2 duzi jednake i istosmerne sa duzi A2k A2k1 , dakle i duzi Ak1 Ak , bice poligonska linija
A2k1 A1 . . . Ak2 Ak u poligonu A1 . . . A2k . Zaista, dijagonala Ak A2k1 razlaze
konveksnu poligonsku povrs (A1 . . . A2k ) na dve takode konveksne poligonske
povrsi (A1 . . . Ak A2k1 A2k ) i (Ak . . . A2k1 ). Prava Ai Ai za i = 1, . . . , k 2
nalazi se izmedu paralelnih pravih A2k A2k1 i Ak1 Ak te sece duz Ak A2k1
u nekoj tacki Bi . Pri tome je tacka Ai izmedu tacaka Ai i Bi , te se nalazi u
konveksnom poligonu A1 . . . Ak A2k1 A2k , dakle i u poligonu A1 . . . A2k . Stoga
je duzima Ai Ai za i = 1, . . . , k 2 i poligonskom linijom A2k1 A1 . . . Ak2 Ak
poligonska povrs (A1 . . . A2k ) razlaze na k 1 paralelogramskih povrsi i poligonsku povrs A1 . . . Ak2 Ak Ak+1 . . . A2k1 kojoj je broj stranica za dva manji
od broja stranica date poligonske povrsi.
Dokazimo da je dobijena poligonska povrs (A1 . . . Ak2 Ak Ak+1 . . . A2k1 )
takode konveksna i centralno simetricna. Prava odredena bilo kojom stranicom
te poligonske povrsi predstavlja pravu oslonca te povrsi, pa je ta poligonska
povrs konveksna.
Ako je O srediste simetrije povrsi (A1 . . . A2n ) i O tacka takva da je duz OO
istosmerna sa duzi Ak1 Ak i jednaka njenoj polovini, bice tacke Ak+1 , . . . , A2k1
simetricne sa tackama A1 , . . . , Ak2 , Ak u odnosu na tacku O , te je poligonska
povrs (A1 . . . Ak2 Ak . . . A2k1 ) centralno simetricna.
Ako na ovu poligonsku povrs primenimo isti postupak, razlozicemo istu na
k 2 paralelogramskih povrsi i jednu poligonsku povrs kojoj je broj stranica
manji opet za dva. Nastavljajuci ovaj proces dobicemo na kraju centralno simetricnu cetvorougaonu povrs, koja je dakle paralelogramska.
550

Na taj nacin poligonska povrs (A1 . . . A2n ) je razlozena na


(k 1) + (k 2) + . . . + 2 + 1 =

k(k 1)
2

paralelogramskih povrsi (sl. 476).


880. Ako svakoj stranici konveksne poligonske povrsi odgovara njoj jednaka paralelna stranica te povrsi, dokazati da je ta poligonska povrs centralno
simetricna, i da se prema tome moze razloziti na paralelogramske povrsi.

Aj+2

s
N

Aj+1
Aj

Aj1
Ai1

Ai+2

Ai
Ai+1

Slika 477

Slika 478
Neka je (A1 . . . An ) konveksna poligonska povrs takva da svakoj njenoj stranici odgovara jedna njoj jednaka i paralelna stranica te iste povrsi. Ako u toj
korespodenciji stranici ai Ai Ai+1 odgovara stranica aj Aj Aj+1 , dokazimo
najpre da susednoj stranici ai+1 Ai+1 Ai+2 odgovara susedna stranica aj+1
Aj+1 Aj+2 . Pretpostavimo li da stranice ai+1 i aj+1 nisu odgovarajuce, one ne
mogu biti paralelne, prema tome ni uglovi Ai Ai+1 Ai+2 i Aj Aj+1 Aj+2 ne mogu
biti jednaki. Neka je npr. Ai Ai+1 Ai+2 > Aj Aj+1 Aj+2 . U tom slucaju,
ako obelezimo sa s pravu kroz tacku Aj+1 uporednu sa stranicom Ai+1 Ai+2 ,
bice obe tacke Aj i Aj+2 s iste strane prave s. Stoga na stranicama Aj Aj+1
i Aj+1 Aj+2 postoje tacke M i N takve da je M N k Ai+1 Ai+2 k s. Pri tome
duz M N razlaze konveksnu poligonsku povrs (A1 . . . An ) na trougaonu povrs
(M N Aj+1 ) i konveksnu poligonsku povrs (A1 . . . Aj M N Aj+2 . . . An ) koja ima
tri paralelne stranice sto je nemoguce. Otuda sleduje da stranici ai+1 odgovara
stranica aj+1 .
551

S obzirom da u pomenutoj korespodenciji stranicama Ai Ai+1 , Ai+1 Ai+2 , . . .,


odgovaraju redom stranice Aj Aj+1 , Aj+1 Aj+2 , . . ., bice tacke Aj ,Aj+1, Aj+2 , . . .
simetricne sa tackama Ai , Ai+1 , Ai+2 , . . . u odnosu na istu tacku. Stoga je poligonska povrs (A1 . . . An ) centralno simetricna (sl. 477).
Napominjemo da ova osobina moze ali ne mora da vazi za konkavne poligonske povrsi. Tvrdenje ovo ilustrujemo poligonom predstavljenim na sl. 478.
881. Ako se konveksna poligonska povrs moze razloziti na konacan broj
centralno simetricnih poligonskih povrsi, dokazati da je i ta konveksna povrs
centralno simetricna.
Resenje: Neka je poligonska povrs razlozena na konacan broj centralno
simetricnih poligonskih povrsi 1 , . . . n . Temenima poligonskih povrsi i (i =
1, . . . , n) koja se nalazi na proizvoljnoj stranici AB povrsi razlozena je ta stranica na izvestan broj duzi od kojih svaka predstavlja stranicu neke od povrsi
1 .
S obzirom da su povrsi i centralno simetricne svim tim duzima kao stranicama nekih od povrsi i odgovaraju na tim istim povrsima jednake i paralelne
stranice. Sve tako dobijene stranice pripadaju nekoj drugoj stranici povrsi
, ili pak predstavljaju stranice susednih poligonskih povrsi iz skupa i . Ovim
stranicama odgovaraju na tim susednim povrsima jednake i paralelne stranice.
Nastavljajuci ovaj postupak i koristeci cinjenicu da konveksna poligonska povrs
moze da ima samo dve paralelne stranice zakljucujemo na kraju da sve na taj
nacin dobijene duzi pripadaju nekoj stranici A B povrsi koja je paralelna i
jednaka sa stranicom AB. Stoga je povrs centralno simetricna (sl. 479).

Slika 479
882. Ako je fn broj poligonskih povrsi koje se dobijaju razlaganjem konveksne poligonske povrsi (A1 . . . An njenim dijagonalama, pri cemu se nikoje tri i
vise dijagonala ne seku u jednoj tacki, dokazati da je
fn =

1
(n 1)(n 2)(n2 3n + 12).
24

552

11.2. Odredivanje povr


sina
Resenje: Ovaj stav vec smo dokazali na jedan nacin (v.z....), ovom prilikom dokaz izvodimo drugacije. Ako obelezimo sa s3 , s4 , . . . , sk broj trougaonih,
cetvorougaonih, . . . , k-ugaonih povrsi koje se dobijaju razlaganjem poligonske
povrsi (A1 . . . An ) njenim dijagonalama, imamo da je sn = s3 + s4 + . . . + sk .
Zbir temena svih tih poligonskih povrsi iznosi 3s3 + 4s4 + . . . + ksk . Svako teme
tih poligonskih povrsi koje se nalaze u poligonu A1 . . . An predstavlja presek
dveju dijagonala tog poligona, prema tome, ona je zajednicko teme cetiri poligonske povrsi. Prema zadatku . . . broj takvih temena, tj. tacaka u kojima se
seku dijagonale poligona A1 . . . An iznosi
n(n 1)(n 2)(n 3)
.
24
Svako teme poligonske povrsi (A1 . . . An ) zajednicko je teme n 2 poligonskih
povrsi koje se dobijaju razlaganjem njenim dijagonalama. Stoga je
3s3 + 4s4 + . . . + ksk =

1
n(n 1)(n 2)(n 3) + n(n 2)
6

(1)

Zbir unutrasnjih uglova svih poligonskih povrsi na koje je razlozena povrs


(A1 . . . An ) njenim dijagonalama iznosi
[s3 + 2s4 + 3s5 + . . . + (k 2)sk ]2R,
gde je R prav ugao. S obzirom da je zbir uglova kod svakog temena koje prestavlja presek dveju dijagonala poligona A1 . . . An pun ugao, a zbir uglova kod
svih temena koja se poklapaju s temenima poligona A1 . . . An jednak (n 2)2R,
bice
[s3 + 2s4 + 3s5 + . . . + (k 2)sk ]2R =

1
n(n 1)(n 2)(n 3)4R + (n 2)2R.
24

Otuda je
s3 + 2s4 + 3s5 + . . . + (k 2)sk =

1
n(n 1)(n 2)(n 3) + (n 2)
12

Oduzimanjem jednakosti (2) od jednakosti (1) nalazimo da je


2(s3 + s4 + . . . + sk ) =

1
n(n 1)(n 2)(n 3) + (n 1)(n 2),
12

i prema tome da je
sn = s3 + s4 + . . . + sk =
=

1
1
n(n 1)(n 2)(n 3) + (n 1)(n 2)
24
2

1
(n 1)(n 2)(n3 3n + 12).
24

553

(2)

11.2

Odredjivanje povr
sina

883. Dokazati da razlozivo jednake poligonske povrsi imaju jednake povrsine.


Resenje: Neka su i dve razlozivo jednake poligonske povrsi. Saglasno
definiciji razlozive jednakosti poligonska povrs moze se razloziti na konacan
niz trougaonih povrsi 1 , . . . , n a poligonska povrs na konacan niz trougaonih povrsi 1 , . . . , n pri cemu je i
= i za i = 1, . . . , n. Stoga je prema
uslovima navedenim u definicije povrsine povrsi S() = S(1 ) + . . . + S(i ) =
S(1 ) + . . . + S(i ) = S( ).
884. Dokazati da dopunski jednake poligonske povrsi imaju jednake povrsine.
Resenje: Neka su i dve dopunski jednake poligonske povrsi. Saglasno definiciji dopunski jednakih povrsi postoje trougaone povrsi 1 , . . . , n koje
mozemo dodati poligonskoj povrsi i trougaone povrsi 1 , . . . , n koje mozemo
dodati poligonskoj povrsi pri cemu je i
= i za i = 1, . . . , n, a povrs
sastavljena iz povrsi , 1 , . . . , n razlozivo jednaka sa povrsi sastavljene od
povrsi , 1 , . . . , n . Stoga je S(i ) = S(i ), a prema ptrethodnom zadatuku
S() = S( ).
S obzirom da su poligonske povrsi i razlozivo jednake, povrsi i mozemo
razloziti na parove podudarnih povrsi. Tim razlaganjem razlozene su povrsi
i na trougaone povrsi j i j , a povrsi i i i na trougaone povrsi ik i ik .
Stoga je
XX
X
X
S(i ),
S(j ) +
S(ik ) = S() +
S() =
i

S( ) =

X
j

S(j ) +

XX
i

S(ik ) = S( ) +

S(i )

iz ovih jednakosti i jednakosti


S() = S( ), S(i ) = S(i ) sledi da je S() = S( ).
885. Dokazati da su pravougaone povrsi sa jednakim povrsinama razlozivo
jednake.
Resenje: Neka je AB < 2A B . S obzirom da je AB > A B i AD <
A D ,postoji izmedu tacaka A i B tacka B1 takva da je AD1 = A D . Pri tome
su pravougaone povrsi (AB1 C1 D1 ) i (A B C D ) podudarne,te imaju jednake
povrsine. Stoga i pravougaone povrsi (ABCD) i (AB1 C1 D1 ) imaju jednake
povrsine. Duz B1 D razlaze povrs (ABCD) na povrsi (AB1 D) i (B1 BCD), a
povrs (AB1 C1 D1 ) na povrsi (AB1 D) i (B1 C1 D1 D). Ako obelezimo sa L tacku
u kojoj se seku prave B1 D i C1 D1 , a sa M i N tacke polupravih B1 D i B1 C1
takve da je B1 M = DL i B1 N = DD1 , bice povrs (B1 BCD) podudarna sa
povrsi (M N C1 L), a povrs (M N C1 L) razlozivo jednaka sa povrsi (B1 C1 D1 D).
Stoga su povrsi (ABCD) i (AB1 C1 D1 ),dakle i povrsi (ABCD) i (A B C D )
razlozivo jednake (Slika 480).

554

Slika 480
Neka je AB = 2A B . Ako obelezimo sa B1 srediste stranice AB, sa D1
tacku simetricnu sa A u odnosu na D, sa C1 tacku takvu da je cetvorougao
AB1 C1 D1 paralelogram i sa K presek duzi B1 C1 , bice sa duzi B1 K povrs
(ABCD) razlozena na povrsi (AB1 KD) i (B1 BCK), a sa KD povrs (AB1 C1 D1 )
L
razlozena na povrsi (AB1 KD) i D1
(DKC1 D1 ). Pri tomeC1
su povrsi (B1 BCK) i
(DKC1 D1 ) podudarne, te su i povrsi (ABCD) i (AB1 C1 D1 ), dakle i povrsi
D
C
(ABCD) i (A B C D ) razlozivo jednake(Slika
481).

N
A

B1

555

Slika 481
Ako je AB > 2A B , bice i AD < 2A D . U tom slucaju postoje na duzi
AB redom tacke B1 , . . . , Bk takve da je AB1 = B1 B2 = . . . = Bk1 Bk = A B
i A B < Bk B 2A B , a na duzi A D redom tacke D1 , . . . , Dk takve da je

AD1 = D1 D2 = . . . = Dk1
Dk = AD i AD < Dk D 2AD. Pri tome su
pravougaone povrsi (AB1 C1 D1 ),(B1 B2 C2 C1 ),. . .,(Bk1 Bk Ck Ck1 ) podudarne
D1
C1

sa prvougaonim povrsima (A B C1 D1 ),(D1 C1 C2 D2 ),. . .,(Dk1


Ck1
Ck Dk ),

a prema predhodnim delovima ovog zadatka povrsi (Bk BCCk ) i (Dk Ck C D ) su
D
razlozivo jednake, te su i pravougaone povrsi (ABCD)
i (A B C D )KrazlozCivo
jednake (Slika 482).

556

B1

C1

C2

B1

B2

D2

C2

D1

C1

Slika 482
886. Dokazati da su trougaone povrsi sa jednakim povrsinama razlozivo
jednake.
A

557

Resenje: Neka dve trougaone povrsi (ABC) i (A B C ) imaju jednake povrsineAko


obelezimo sa M i N sredista stranica AB i AC, a sa D i E podnozja upravnih
iz tacaka C i B na pravoj M N , bice trougaona povrs (ABC) razlozivo jednaka
sa pravougaonom povrsi (BCDE), dakle i S(ABC) = S(BCDE). Ako zatim
obelezimo sa M i N sredista stranica A B i A C , a sa D i E podnozja
upravnih iz tacaka C i B na pravoj M N , bice trougaona povrs (A B C )
razlozivo jednaka sa pravougaonom povrsi (B C D E ), dakle i
S(A B C ) = S(B C D E ). No S(ABC) = S(A B C ), pa je i
S(BCDE) = S(B C D E ). Prema prethodnom zadatku pravougaone povrsi
(BCDE) i (B C D E ) su razlozivo jednake, te su i trougaone povrsi (ABC) i
(A B C ) razlozivo jednake (Slika 483).

Slika 483
887. (Teorema F. Boljaja-Gervina). Dokazati da su poligonske povrsi sa
jednakim povrsinama razlozivo jednake.
Resenje: Neka dve poligonske povrsi () i ( ) imaju jednake povrsine.
Prema poznatom stavu postoje trougaone povrsi (ABC) i (A B C ) razlozivo
jednake sa povrsima () i ( ). Sobzirom da razlozivo jednake povrsi imaju
jednake povrsine, bice S() = S(ABC) i S() = S(A B C ). No S( ) =
S( ), pa je i S(ABC) = S(A B C ). Prema prethodnom zadatku trougaone
A
povrsi (ABC) i (A B C ) su razlozivo jednake, te su poligonske povrsi () i ( )
razlozivo jednake.
E
888. (Paposova teorema).
Ako su ABKL i ACMDN dva paralelograma
konstruisana nad stranicama AB i ACF bilo kojeg trougla ABC, s onih strana
pravih AB i AC s kojih nije taj trougao i ako je D presek pravih KL i M N ,
zatim BCP Q paralelogram s bilo koje strane prave BC takav da je BQkAD i
BQ = AD, dokazati da je B
C
S(BCP Q) = S(ABKL) + S(ACM N ).
Resenje: Sobzirom da se oba paralelograma ABKL i ACM N nalaze sonih
strana pravih AB i AC skoje nije trougao ABC ili sonih strana pravih AB i AC
558

skojih se nalazi taj trougao, tacka D je u uglu koji je unakrsan suglom BAC
ili pak u uglu BAC, te prava AD sece stranice BC i P Q paralelograma BCP Q
u tackama, obelezimo ih sa E i F . Ako su su zatim G i H tacke u kojima prave
BQ i CP seku prave KL i M N , bice S(BCP Q) = S(BEF Q) + S(ECP F ) =
S(ABGD) + S(ACHD) = S(ABKL) + S(ACM N ). Specijalno, ako je ugao A
trougla ABC prav i ako su paralelogrami ABKL i ACM N kvadrati, dokazana
Paposova teorema svodi se na poznatu Pitagorinu teoremu (Slika 484).

559

N
A
H

Slika 484
M

K
E
B

560
Q

889. Ako neka prava paralelna sa stranicom AB paralelograma ABCD sece


stranicu BC u tacki P i dijagonalu AC u tacki Q, dokazati da je
S(ABP ) = S(AQD).
Resenje: S obzirom da je QP k AB, bice (sl. 485)
S(ABP )
BP
AQ
S(AQD)
S(AQD)
=
=
=
=
S(ABC)
BC
AC
S(ACD)
S(ABC)
Otuda je S(ABP ) = S(AQD).

slika 485
D

890. Ako je P proizvoljna tacka u paralelogramu ABCD, dokazati da je


S(P AB) + S(P CD) = S(P BC) + S(P AD).
Resenje: Ako obelezimo sa K i M tacke u kojima prava kroz P uporedna sa
P
AD sece AB i CD, a sa L i N tacke u Q
kojima prava kroz P uporedna
sa AB
sece BC i AD, bice (sl. 486)
S(P AB) + S(P CD) = S(P AK) + S(P KB) + S(P M C) + S(P M D) =
A

561

= S(P AN ) + S(P BL) + S(P CL) + S(P DN ) = S(P BC) + S(P AD)

slika 486

891. Ako je P proizvoljna tacka ravni paralelograma ABCD, dokazati da je


S(P AC) = S(P AB) = S(P AD).
N
L B i D
Resenje: Obele
zimo sa B i D tacke uP kojima prave kroz temena
uporedne s pravom AP seku pravu AC. Iz podudarnosti trouglova BB C i
DD A sledi da je B C = AD . Ako je tacka B izmedu tacaka A i C, bice (sl.
487)
A

562

slika 487
D

S(P AC) = S(P AB ) + S(P B C) = S(P AB ) + S(P AD ) = S(P AB) +

S(P AD). Analogan dokaz izvodi se i u slucaju kada tacka


B B nije izmedu
tacaka A i C.
563
D
A

892. Ako su M i N tacke stranice BC trougla ABC takve da je BM =


M N = N C, P proizvoljna tacka duzi M N , a R i Q tacke u kojima prave kroz
M i N uporedne sa AP seku stranice AB i AC, dokazati da je
S(ARP Q) = S(BP R) = S(CP Q).
Resenje: Duzi AM i AN razlazu trougaonu povrs (ABC) na tri trougaone
povrsi (ABM ), (AM N ), (AN C) koje imaju jednake osnovice i njima odgova1
rajuce visine, pa je S(ABM ) = S(AM N ) = S(AN C) = S(ABC). S obzirom
3
da je S(AM B) = S(RBM ) + S(ARM ) = S(RBM ) + S(P RM ) = S(BP R),
1
1
bice S(ABM ) = S(ABC). Isto tako je S(CP Q) = S(ABC), pa je i
3
3
1
S(ARP Q) = S(ABC) (sl. 488).
3

564
R

slika 488
893. Ako su P, Q, R tacke u kojima tri uporedne prave kroz temena A, B, C
seku prave odredene naspramnim stranicama trougla ABC, dokazati da je
S(P QR) = 2S(ABS).
Resenje: Jedna od triju uporednih pravih AP , BQ, CR je izmedu ostalih
dveju, neka je npr. prava AP izmedu pravih BQ i CR. Pri tome duz AP
razlaze trougaonu povrs (ABC) na pravougaone povrsi (ABP ) i (ACP ). Prava
AP sece duz QR u nekoj tacki V , te duz P V razlaze trougaonu povrs (P QR)
na trougaone povrsi (P V Q) i (P V R). Sobzirom da su P i V tacke u kojima
prava kroz presek A dijagonala BR i CQ uporedna sa osnovicama BQ i CR
sece krake BC i QR trapeza BCRQ prema zadatku . . . tacka A je srediste
duzi P V , pa je P V = 2AP . Otuda je S(P QR) = S(P QV ) + S(P RV ) =
2S(AQP ) + 2S(ARP ) = 2S(ABP ) + 2S(ACP ) = 2S(ABC). (sl489)

slika 489
894. Neka je ABC proizvoljan trougao i P proizvoljna tacka. Ako su A ,
B , C tezista trouglova P BC, P CA, P AB, dokazati da je

S(ABC) = 9S(A B R
C ).
V

Resenje: Ako obelezimo sa A1 , B1 , C1 sredista stranica BC, CA, AB trougla


ABC, bice BC = 2B1 C1 =A 3B C , CA = 2C1 A1 = 3C A , AB = 2A1 B1 =
3A B , pa je (sl. 490) S(ABC) = 9S(A B C ).
B

565

C1

B1

A1

566
A

slika 490
895. Ako su P , Q , R upravne projekcije proizvoljne tacke O koja se nalazi
u trouglu ABC na pravama BC, CA, AB a P, Q, R tacke polupravih OP , OQ ,
OR takve da je OP = BC, OQ = CA, OR = AB, dokazati da je
S(P QR) = 3S(ABC).
Resenje: S obzirom da je tacka O u trouglu ABC, ona je u uglovima B
i C trougla ABC, pa su tacke Q i R , prema tome i tacke Q i R s raznih
strana od praveOP . Stoga su konveksni uglovi P OQ i P OR susedni. Ti susedni
uglovi su suplementni sa uglovima B i C trougla ABC, pa je njihova unija
konveksan ugao. Otuda sleduje da prava OP sece duz QR u nekoj tacki S
pri cemu je tacka O izmedu temena P i unutrasnje tacke S stranice QR, pa
je prema definiciji tacka O u trouglu P QR. Stoga duzi OP , OQ, OR razlazu
povrs (P QR) na povrsi (P OQ), (QOP ) i (ROP ). Dokazimo da je svaka od njih
jednaka povrsi (ABC). Stranice OQ i OR trougla OQR jednake su stranicama
AC i AB trougla ABC, a uglovi QOR i CAB zahvaceni navedenim stranicama
suplementni, pa je S(OQR) = S(ABC). Isto tako je S(ORP ) = S(ABC) i
S(OP Q) = S(ABC). SToga je (sl. 491) S(P QR) = 3S(ABC).

567

S
Q

A
Q

R
O

slika 491
896. Ako obelezimo sa O tacku koja se nalazi u konveksnom cetvorouglu
ABCD, sa K , L , M , N podnozja upravnih iz tacke O na pravama AB, BC,
P

568

CD, DA i sa K, L, M , N tacke polupravih OK , OL , OM , ON takve da


je OK = AB, OL = BC, OM = CD, ON = DA, dokazati da duzi KL, LM ,
M N , N K obrazuju izvesnu povrs takvu da je S() = 2S(ABCD).
Resenje: S obzirom da je tacka O u konveksnom cetvorouglu ABCD, tacka
O je u konveksnim uglovima DAB i ABC, te su tacke N i L , dakle i tacke N i
L sa raznih strana prave OK. Stoga su konveksni uglovi N OK i KOL susedni.
Isto tako susedni su i konveksni uglovi KOL i DOM , DOM i M ON , M ON
i N OK. Otuda sleduje da duzi KL, IM , M N , N K odreduju izvesnu povrs
pri cemu duzi OK, OL, OM , ON razlazu istu na trougaone povrsi (KOL),
(DOM ), (M ON ), (N OK). Kako je cetvorougaona povrs (ABCD) konveksna,
dijagonala AC razlaze istu na trougaone povrsi (ABC) i (ACD), a dijagonala
BD na trougaone povrsi (DAB) i (BCD).

C
L

N
O

K
B

Sl. 492
S obzirom da su duzi OK i OL jednake sa duzima AB i BC, a uglovi KOL i
ABC suplementni, bi`ce S(KOL) = S(ABC). Isto tako je S(LOM ) = S(BCD),
S(M ON ) = S(CDA), S(N OK) = S(DAB).
Stoga je (sl. 492) S() = S(KOL) + S(LOM ) + S(M ON ) + S(N OK) =
= S(ABC) + S(BCD) + S(V DA) + S(DAB) = 2S(ABCD).
897. Kroz srediste svake dijagonale konveksnog cetvorougla ABCD konstruisana je prava uporedna s drugom dijagonalom. Dokazati da duzi koje spajaju
presek O tih pravih sa sredistima stranica razlazu cetvorougaonu povrs (ABCD)
na cetiri ekvivalentne cetvorougaone povrsi.
Resenje: Neka su K, L, M , N sredista stranica AB, BC, CD, DA a P i
Q sredista dijagonale AC i BD cetvorougla ABCD. Najpre dokazimo da je
tacka O u tom cetvorouglu. Kako je cetvorougao ABCD konveksan, njegove
dijagonale se seku u nekoj tacki O . Ako je tacka O na nekoj dijagonali ona
je u tom cetvorouglu; ako nije na dijagonali, ona je u jednom od uglova koja
odredjuju dijagonale; n pr. uglu AO B. U tom slucaju bi`ce tacke P i Q na
1
1
duzima O A i O B, pa je O P < O A i O Q < O B. Stoga je teme O
2
2
569

paralelograma P O QO u trouglu AO B, dakle i u cetvorouglu ABCD. Otuda


sleduje da duzi OK, OL, OM , ON razlazu cetvorougaonu povrs (ABCD) na
cetvorugaone povrsi (AKON ), (BLOK), (CM OL), (DN OM ).
C

O
N

Sl. 493
Kako je tacka P srediste dijagonale AC, duzi BP i DP razlazu povrs (ABCD)
na dve jednake povrsi (DABP ) i (BCDP ). Duz AP razlaze povrs (DABP ) na
dve trougaone povrsi (ABP ) i (ADP ), a duzi KP i N P razlazu svaku od ovih
trougaonih povrsi na po dve jednake trougaone povrsi. Stoga je S(AKP N ) =
1
S(ABCD).
4
S obzirom da je P O||BD||KN , bi`ce S(P N K) = S(ON K), i prema tome
1
S(AKON ) = S(ABCD).
4
1
1
Isto tako je S(BLOK) = S(ABCD), S(CM OL) = S(ABCD),
4
4
1
S(DN OM ) = S(ABCD), pa je S(AKOM ) = S(BLOK) = S(CM OL) =
4
S(DN OM )
898. Ako je ABCD konveksan cetvorougao i ako su A , B , C , D tacke
simetricne s tackama A, B, C, D respektivno u odnosu na tacke B, C, D, A,
dokazati da je
S(A B C D ) = 5S(ABCD).
Resenje: S obzirom da su tacke A i B sredista stranica DD i AA trouglova
BDD i D AA , imamo da je S(ABD) = S(ABD ) i S(ABD ) = S(BA D ).
Otuda je S(AA D ) = 2S(ABD).
Isto tako je S(BB A ) = 2S(BCD), S(CC B ) = 2S(BCD), S(DD C ) =
2S(ABC).
Stoga je S(A B C D ) = S(ABCD) + S(AA D ) + S(BB A ) + S(CC B ) +
S(DD C ) = 5S(ABCD) (sl. 494).

570

A
B

Sl. 494
899. Ako je ABCD proizvoljan konveksan cetvorougao. Ako su K i L tacke
stranice AB takve da je AK = KL = LB, a M i N tacke stranice CD takve
da je CM = M N = N D, dokazati da je
S(KLM N ) =

1
S(ABCD).
3

Resenje: S obzirom da je AK = KL = LB i CM = M N = N D, bi`ce


1
2
1
S(BLC) = S(ABC) i S(AN D) = S(ACD), pa je S(ALCN ) = S(ABCD).
3
3
3
Zatim je S(AKN ) = S(KLN ) i S(LOM ) = S(LM N ), pa je S(ALCN ) =
1
2S(KLM N ). Otuda je S(KLM N ) = S(ABCD) (v. sl. 495).
3

Sl. 495
900. Ako su M i N sredista stranica AB i CD konveksnog cetvorougla
ABCD, a P i Q tacke u kojima duzi AN i BN seku duzi DM i CM , dokazati
da je
S(P M QN ) = S(DAP ) + S(BCQ).
571

Resenje: S obzirom da su tacke M i N sredista stranica AB i CD, imamo


1
1
da je (sl. 496) S(AM C) = S(ABC), S(AN C) = S(ACD) S(BM D) =
2
2
1
1
S(ABD), S(BN D) = S(BCD). Sabiranjem odgovaraju`cih strana, nalazimo
2
2
da je S(AM C) + S(AN C) + S(BM D) + S(BN D) = S(ABCD). Pored toga
je S(AM C) + S(AN C) + S(BM D) + S(BN D) = S(ABCD) + S(P M QN )
S(DAP ) S(BCQ). Otuda je S(P M QN ) = S(DAP ) + S(BCQ).
C

Sl. 496
901. Ako obelezimo sa P i Q sredista dijagonala AC i BD prostog cetvorougla
ABCD i sa R tacku u kojoj se seku prave odredene naspramnim stranicama
AB i CD, dokazati da je
S(P QR) =

1
S(ABCD).
4

Resenje: Ako obelezimo sa E srediste stranice BC, bi`ce (sl. 497)


S(P QR) = S(P BRCQ) S(P BR) S(CQR) =
S(P BRCQ) S(EBR) S(CER) = S(P BCQ) = S(P BC) + S(P CQ) =
1
1
1
1
= S(ABC) + S(ACQ) = S(ABC) + (S(ACD) S(AQD) S(QCD)) =
2
2
2
2
1
1
1
S(ABC) + (S(ACD) S(ABCD)) =
2
2
2
1
1
1
= S(ABC) + S(ACD) S(ABCD) =
2
2
4
1
1
1
= S(ABCD) S(ABCD) = S(ABCD)
2
4
4

572

Sl. 497
902. Ako obelezimo sa P i Q sredista dijagonala AC i BD prostog cetvorougla
ABCD kome se prave odredene naspramnim stranicama AB i CD seku u jednoj
tacki E, a prave odredene naspramnim stranicama BC i AD seku u nekoj tacki
F , dokazati da je
S(P QE) = S(P QF ).
Resenje: Uputstvo. Koristiti prethodni zadatak.
903. Neka su AB = a i CD = b osnovice trapeza ABCD kome se dijagonale
seku u tacki O. Ako je P proizvoljna tacka stranice AB, Q tacka u kojoj se
seku duzi P D i AC, dokazati da je
S(P QR) =

a+b
S(OQR).
b

Resenje: Tacke M i N u kojima prava P O sece duzi QR i CD su harmonijski


spregnute s tackama O i P , pa je
P M : M O = P N : N O = (a + b) : b.
Stoga je (sl. 498)
S(P QR) =

a+b
S(OQR).
b

573

sl. 498

904. Ako su E i F tacke u kojima simetrale unutrasnjeg i spoljasnjeg ugla


O : AC = m : n,
A seku pravu odredenu stranicom BC trougla ABC i ako je AB
dokazati da je
M Q
R
S(ABC)
m2 n 2
=
.
S(AEF )
2mn
A

Resenje: Trougaone povrsi (ABC) i (AEF ) imaju jednake visine iz temena


A, pa je
S(ABC) : S(AEF ) = BC : EF.
Prema zadatku . . . imamo da je
BC : EF = (m2 n2 ) : 2mn,
pa je
S(ABC) : S(AEF ) = (m2 n2 ) : 2mn.
905. Ako su P i Q tacke paralelnih stranica AB i CD konveksnog trapeza
ABCD takve da je AB = kP B i DC = kQC, dokazati da je
S(ABCD) = nS(P BCQ).
Resenje: Prema poznatom stavu prave AD, BC, P Q seku se u jednoj tacki
O pri cemu je [ADO] ili [DAO]. Ako je npr. [ADO], bice
S(ABCD) = S(OAB) S(ODC)
i
S(P BCQ) = S(OP B) S(OQC).
Pri tome je S(OAB) = n S(OP B) i S(ODC) = n S(OQC) , pa je
S(ABCD) = n S(P BCQ).
Ako je [DAO], dokaz je analogan (sl. 499).
574

sl. 499
906. Ako su P1 , . . . , Pn paralelne projekcije temena A1 , . . . , An poligonske
povrsi (A1 . . . An ) na nekoj pravoj p koja sa tom poligonskom povrsi nema
zajednickih tacaka, a B1 , . . . , Bn , redom, tacke na duzima A1 P1 , . . . , An Pn takve
da je A1 P1 = kB1 P1 , . . . , An Pn = kBn Pn , dokazati da je
S(A1 . . . An ) = kS(B1 . . . Bn ).
Resenje: Jedna od paralelnih pravih Ai Pi za i = 1, . . . , n ima takvu osobinu
da su sve ostale prave iz tog skupa pravih s iste strane od nje, sem izvesnih koje
se poklapaju sa njom. Neka je to prava A1 P1 . Ako se ona poklapa sa jos nekim
pravama iz pomenutog skupa, ona sadrzi vise stepena poligona A1 . . . An , neka
se sva ta temena nalaze na duzi A1 P1 . Teme An je u uglu P1 A1 A2 ili izvan
njega. Ako je teme An u tom uglu, bice
S(A1 . . . An ) = S(A1 A2 P2 P1 ) S(A2 A3 P3 P2 ) . . . S(An A1 P1 Pn ),
S(B1 . . . Bn ) = S(B1 B2 P2 P1 ) S(B2 B3 P3 P2 ) . . . S(Bn B1 P1 Pn ).
Pri tome u prvoj od ovih jednakosti treba uzeti znak plus ili minus zavisno od
toga da li odgovarajuca povrs ima isti ili suprotan smer od povrsi (A1 A2 P2 P1 ),
a u drugoj jednakosti treba uzeti znak plus ili minus zavisno od toga da li
odgovarajuca povrs ima isti ili suprotan smer od povrsi (B1 B2 P2 P1 ). S obzirom
da odgovarajuce povrsi u dobijenim jednakostima imaju isti smer, one imaju i
isti znak i s obzirom da je prema prethodnom zadatku
S(A1 A2 P2 P1 ) = S(B1 B2 P2 P1 ), . . . , S(An A1 P1 Pn ) = S(Bn B1 P1 Pn ),
bice (sl.500)
S(A1 . . . An ) = S(B1 . . . Bn ).

575

A3

A2

A4

An
A1

B3

B2

B4

Bn
B1

P1

Pn P2

P3

P4

sl. 500
907. Ako dve poligonske povrsi (A1 . . . A2n ) i (B1 . . . B2n ) imaju zajednicka
sredista odgovarajucih stranica, dokazati da je
S(A1 . . . A2n ) = S(B1 . . . B2n ).
Resenje: Neka su O1 , . . . , O2n zajednicka sredista odgovarajucih stranica
A1 A2 i B1 B2 ,. . . , A2n A1 i B2n B1 . Duzi A1 B1 i A2 B2 simetricne su medu
sobom u odnosu na tacku O1 , te su prave A1 B1 i A2 B2 istovetne ili uporedne.
Isto tako, prave A2 B2 i A3 B3 su istovetne ili uporedne, i t.d.
Obelezimo sa p proizvoljnu pravu, koja sa poligonskim povrsima (A1 . . . A2n )
i (B1 . . . B2n ) nema zajednickih tacaka, a upravna je na pravoj A1 B1 , dakle i na
pravim A2 B2 , . . . , A2n B2n , od tacaka na pr. C1 , . . . , C2n . Jedna od uporednih
pravih A1 B1 , . . . , A2n B2n ; recimo prava A1 B1 ima takvu osobinu da su sve
ostale prave iz tog mnostva pravih s iste strane od nje, sem mozda nekih koje se
poklapaju sa njom. Ako tacka A2n pripada uglu C1 A1 A2 , i tacka B2n pripada
uglu C1 B1 B2 , bice
S(A1 . . . A2n ) = S(A1 C1 C2 A2 ) S(A2 C2 C3 A3 ) . . . S(A2n C2n C1 A1 ) . . . (1)

S(B1 . . . B2n ) = S(B1 C1 C2 B2 ) S(B2 C2 C3 B3 ) . . . S(B2n C2n C1 B1 ) . . . (2)


pri cemu u jednakosti (1) treba uzeti znak plus ili minus prema tome da li odgovarajuca povrs ima isti ili suprotan smer od smera povrsi (A1 C1 C2 A2 ), a u
576

jednakosti (2) treba uzeti znak plus ili minus prema tome da li odgovarajuca
povrs ima isti ili suprotan smer od povrsi (B1 C1 C2 B2 ). S obzirom da su odgovarajuce povrsi na desnoj strani u jednakostima (1) i (2) jednake i istosmerne ,
bice
S(A1 . . . A2n ) = S(B1 . . . B2n )
(sl. 501).

A2 n
B1
A4
B2 n

B3

A1
O3

O1
A2

O2

B4

A3

B2

C1

C2 n

C2

C3

C4

sl. 501
908. Odrediti potreban i dovoljan uslov pod kojim prava s uporedna sa
stranicom BC trougla ABC sece stranice AB i AC u tackama M i N takvim
da je
S(BCN M ) : S(AM N ) = m : n.
Resenje: dredimo najpre potreban uslov. Iz S(BCN M ) : S(AM N ) = m : n
sledi da je S(ABC) : S(AM N ) = (m + n) : n. Ako sa M Obelezimo tacku
stranice AB takvu da je BM : AM = m : n bice AB : AN = (m + n) : n, pa
je i
s(abc) : s(amn) = ab : am
(3)
Iz slicnosti trouglova ABC i AM N imamo da je
s(abc) : s(amn) = ab2 : am2

(4)

pa iz propozicija (1) i (2) nalazimo da je AB 2 : AM 2 = AB : AM tj. da je


AM 2 = AB AM .
577

N
M

slika 502
Dokazimo sad da je taj uslov i dovoljan, naime iz AM 2 = AB AM sledi
S(BCN M ) : S(AM N ) = m : n. Zaista, imamo da je
S(BCN M ) : S(AM N ) = S(ABC) S(AM N ) : S(AM N ) =

S(ABC)
1=
S(AM N )

AB 2
AB 2
AB
AM + BM
BM
m
1=
1=
1=
1=
=
2

AM
AB AM
AM
AM
AM
n
Prema tome, da bi prava s uporedna sa stranicom BC trougla ABC sekla
stranice AB i AC u tackama M i N takvim da je S(BCN M ) : S(AM N ) = m :
n, potrebno je i dovoljno da bude AM 2 = AB AM , gde je M tacka stranice
AB takva da je BM : AM = m : n (slika 502).
=

909. Odrediti potreban i dovoljan uslov pod kojim prava s uporedna sa


osnovicama sece krake AD i BC trapeza ABCD u tackama M i N takvim da
je
S(ABN M ) : S(M N CD) = m : n.
Resenje: Odredimo najpre potreban uslov. Ako su uporedne stranice AB i
CD jednake, zadatak je jednostavan, pretpostavimo da one nisu jednake. Ako je
AB > CD, prave AD i BC se seku u nekoj tacki O takvoj da je A M D O
i B N C O. U tom slucaju, bice (vidi sliku 503).

578

O
D

M
D

C
D
M
s

N
M

slika 503

S(ABN M ) : S(M N CD) = [S(OAB) S(OM N )] : [S(OM N ) S(OCD)] =


=[

s(oab)
s(ocd)
oa2
od2
)
1] : [1
]=(
1) : (1
s(omn)
s(omn
om2
om2

(5)

Obelezimo sa D tacku duzi OA takvu da je OD2 = OA OD , a sa M


tacku duzi AD takvu da je AM : M D = m : n. Kako je po pretpostavci
S(ABN M ) : S(M N CD) = m : n, bice
s(abnm) : s(mncd) = am : m d = (oa om ) : (om od ) =
=(

od
oa2
od2
oa

1)
:
(1

)
=
(

1)
:
(1

)
om
om
oa om
oa om

(6)

Iz (3) i (4) sledi da je OM 2 = OA OM .


Dokazimo sad da je taj uslov i dovoljan, naime da iz OM 2 = OA OM sledi
S(ABN M ) : S(M N CD) = m : n. Zaista, imamo da je
S(ABN M ) : S(M N CD) = [S(OAB) S(OM N )] : [S(OM N ) S(OCD)] =
=[

S(OAB)
S(OCD)
OA2
OD2
)=
1] : [1
]=(
1) : (1
S(OM N )
S(OM N )
OM 2
OM 2

=(

OA OD
OA
OD
OA2

1)
:
(1

)
=
(

1)
:
(1

)=
OA OM
OA OM
OM
OM
579

= (OA OM ) : (OM OD ) = AM : M D = m : n.
Prema tome, da bi prava s uporedna sa osnovicama sekla krake AD i BC trapeza ABCD u tackama M i N takvim da je S(ABN M ) : S(M N CD) = m : n,
potrebno je i dovoljno da bude OM 2 = OA OM , gde je M tacka duzi AD takva da je AM : M D = m : n, a D tacka duzi OA takva da je OD2 = OAOD.
910. Odrediti potreban i dovoljan uslov pod kojim prava s uporedna sa
datom pravom p razlaze datu trougaonu povrs (ABC) na dve povrsi 1 i 2
takve da je
S(1 ) : S(2 ) = m : n.
Resenje: Ako je prava p uporedna s nekom stranicom trougla ABC, zadatak
se svodi na 908. Analizirajmo slucaj kad prava p nije uporedna ni sa jednom
stranicom trougla ABC. Neka su AA , BB i CC prave uporedne s pravom p
koje sadrze temena, redom, A, B i C i koje seku naspramne stranice u tackama
A , B i C . Jedna od njih npr. AA je izmedu drugih dveju; ona sece stranicu
BC u nekoj tacki A i razlaze trougaonu povrs (ABC) na trougaone povrsi
(AA B) i (AA C). Pri tome je S(AA B) : S(AA C) = BA : A C, i prema tome
je
S(ABC) : S(AA C) = bc : a c
(7)

s
A

M
B

slika 504
obelezimo sa m tacku duzi bc takvu da je
bm : m c = m : n

580

(8)

tacka m je istovetna sa a , ili je izmedu tacaka B i A , ili je izmedu tacaka A i C.


U tom slucaju je BA : A C = m : n, i pri tome je S(AA B) : S(AA C) = m : n.
Otud sledi da se prava s nalazi s one strane prave AA s koje je C, tj. da prava
s sece stranice A C i AC trougla AA C u tackama m i n, takvim da je
s(abmn) : s(cmn) = m : n

(9)

iz proporcije (6) nalazimo da je BC : M C = (m + n) : n, pa je


s(abc) : s(cmn) = bc : m c

(10)

Najzad, iz proporcija (5) i (8) sledi da je S(AA C) : S(CM N ) = A C : M C. Na


taj nacin zadatak se svodi na odredivanje uslova pod kojim prava s uporedna sa
stranicom AA sece straniceA C i AC trougla AA C u tackama M i N takvim da
je S(AA M N ) : S(CM N ) = A M : M C, tj na zadatak ...... Otuda nalazimo
da je CM 2 = CM CA , sto predstavlja potreban uslov da prava s razlaze
trougaonu povrs ABC na povrsi 1 = (ABM N ) i 2 = (CM N ) takve da je
S(1 ) : S(2 ) = m : n. Dokazimo da je taj uslov i dovoljan. Kako je CM 2 =
CM CA , prema zadatku ....... , bice S(AA M N ) : S(CM N ) = A M : M C,
i prema tome je S(AA C) : S(CM N ) = A C : M C (slika 504).
911.Odrediti potreban i dovoljan uslov pod kojim prava s,uporedna s datom
pravom p razlaze datu konveksnu poligonsku povrs na dve poligonske povrsi 1
i 2 takve da je
S(1 ) : S(2 ) = m : n.
Uputstvo: Prave kroz temena poligonske povrsi = (A1 , A2 , . . . , An ) uporedne sa pravom p, u opstem slucaju, razlazu povrs na n 1 povrsi, obelezimo
ih redom sa 1 , 2 , . . . , n1 . Povrsi 1 i n1 su trougaone, a ostale povrsi
2 , 3 , . . . , n2 su cetvorougaone, ogranicene trapeznim. Neka su L, L1 , L2 , . . . , Ln1
tacke na nekoj pravoj l takve da je [LL1 L2 L3 . . . Ln1 ] i
S(1 ) : S(2 ) : . . . : S(n1 ) = LL1 : L1 L2 : . . . : Ln2 Ln1
a K tacke duzi LLn2 takve da je LK : KLn1 = m : n. Ako je tacka
K izmedju tacaka L1 i L tada prava s uporedna sa p razlaze povrsi na
dve povrsi i takve da je S( ) : S( ) = L1 K : KL . Zaista, ako
sa 1 obelezimo povrs iz S(1 ) : S(2 ) = m : n = LK : KLn1 sledi da je
S(i ) : S( i ) = L1 K : KL . Na taj nacin zadatak se svodi na 909. ili 910.,
prema tome da li je povrs trougaona ili cetvorougaona (sl. 505).
P

a1 2
i

Li1

L1
L

L2

n1

Ln1

Li
K

Ln2

Slika 505
581

912.Odrediti u ravni cetvorougla ABCD skup svih tacaka X takvih da je


S(AXB) + S(CXD) = S(BXC) + S(DXA)
.
Uputstvo:Odredimo najpre potreban uslov. Ako obolezimo sa X proizvoljnu
tacku koja zadovoljava datu relaciju i sa P srediste dijagonale AC, bice npr.
(sl.506).

Q
P

Slika 506
S(AXB) = S(AP B) + S(BP X) S(AP X),
S(CXD) = S(CP D) + S(CP X) S(DP X),
S(BXC) = S(BP C) S(BP X) S(CP X),
S(DXA) = S(AP D) + S(AP X) + S(DP X).
Zamenom ovih jednakosti u relaciju
S(AXB) + S(CXD) = S(BXC) + S(DXA),
a zatim primenom jedakosti
S(AP X) = S(CP X) ,

S(AP B) = S(BP C) i S(AP D) = S(CP D),

nalazimo da je S(BP X) = S(DP X). Iz ove jednakosti sledi da trouglovi BP X


i DP X imaju jednake visine iz temena B i D, te prave P X sadrzi srediste
Q dijagonale BD. Drugim recima, tacka X mora da pripada pravoj koja je
odredjena sredistima P i Q dijagonala AC i BD.
Dokazimo sad da je taj uslov i dovoljan, naime da svaka tacka X prave P Q
zadovoljava relaciju. Posto je
S(AXB) = S(AP B) + S(BP X) S(AP X),
S(CXD) = S(CP D) + S(CP X) S(DP X),
S(BXC) = S(BP C) S(DP X) S(CP X),
S(DXA) = S(AP D) + S(AP X) + S(DP X).
Zatim,
S(AP B) = S(BP C),

S(CP D) = S(ADP ),
582

S(AXP ) = S(CXP )

S(BXP ) = S(DXP ),

bice
S(AKB) + S(CXD) = S(BXC) + S(DXA).
Prema tome, skup svih tacaka koje zadovoljavaju dati uslov je prava odredena
sredistima dijagonala AC i BD cetvorougla ABCD.
913.Ako su P i Q sredista dijagonala AC i BD tangentnog cetvorougla
ABCD, a O srediste kruga upisanoga u tom cetvorouglu, dokazati da tacke
P , Q, O pripadaju jednoj pravoj. (Njutnova teorema.)
Uputstvo: S obzirom da je kod tangentnog cetvorougla ABCD zbir dveju
naspramnih stranica jednak zbiru drugih dvaju stranica, tj. AB + CD = BC +
AD, odstojanja tacke O od stranica AB, BC, CD, DA medu sobom jednaka,
bice S(AOB) + S(COD) = S(BOC) + S(DOA), (sl.507).

P O Q

Slika 507
Stoga prema prethodnom zadatku tacka O pripada pravoj koja je odredena
sredistima P i Q dijagonala AC i BD.
914.Ako je ABCD proizvoljan cetvorougao, E tacka u kojoj se seku prave
AB i CD, F tacka u kojoj se seku BC i DA, dokazati da se sredista P , Q, R
duzi AC, BD, EF nalaze na jednoj pravoj (Gausova teorema).
Uputsvo: Prvi nacin: Obelezimo sa M , N i P sredista duzi AC, BD i EF, redom, a A1 , B1 i F1 , redom, sredista strana BF, F A i AB trougla ABF (sl.508.a.).
A

F1

M
D

B1

C
A1
E
P
F

Slika 508.a.
583

Tada je M p(F1 , B1 ), N p(A1 , F1 ) i P p(A1 , B1 ). Pokazimo da vazi:



A1 P B1 M F1 N
= 1 (1)
P B1 M F1 N A1
Kako je p(B1 , A1 ) k p(A, B), sledi da je

A1 P
BE
=
P B1
EA

(2)

Analogno se dobija:

FC
B1 M
=

M F1
CB

(3)

F1 N
AD
=

N A1
DF

(4)

Iz (2), (3) i (4) sledi da je


BE
A1 P B1 M F1 N

P B1 M F1 N A1
EA

AD

DF

FC

CB

Kako su tacke A, C i D kolinearne proizvod na desnoj strani jednak je 1.


Prema tome, vazi (1) i na osnovu Menelajeve teoreme, tacke M, N i P su
kolinearne.
Napomena. Tacke A1 , B1 i
pripadaju pravama odredjenim stranama
BC, CA i AB, redom, trougla ABC. Tada su tacke A1 , B1 i C1 kolinearne akko je

A1 P B1 M F1 N
= 1
P B1 M F1 N A1
(Menelajeva teorema)
Drugi nacin:
Lema 1. Ako se tetive AB i CD kruznice k seku u tacki S tada je
| SA | | SB |=| CD | | SD |
Lema 2. Ako je H ortocentar trougla ABC, a A , B
iz temena A, B i C redom. Tada je

i C podnozja visina

| HA | | HA |=| HB | | HB |=| HC | | HC |
Dokaz leme 2. Treba iskoristiti cinjenicu da su cetvorouglovi ABA B ,
BCB C i CAC A tetivni u lemi 1.
Obelezimo sa A1 ortocentar trougla ABF i posmatrajmo kruznice k1 , k2 i k3
ciji su precnici redom duzi AC, BD i EF. Kruznica k1 prolazi kroz tacke A
i A kruznica k2 kroz tacke B i B , a kruznica k3 kroz F i F . (sl.508.b.)

584

F
D

A C

Slika 508.b.
Na osnovu leme 2, sledi
| HA | | HA |=| HB | | HB |=| HF | | HF |,
sto znaci da tacka H ima istu potenciju u odnosu na sve tri kruznice (Napomena
1). Analogno se pokazuje da i ortocentar H2 trougla ADE ima istu potenciju u
odnosu na kruznice k1 , k2 i k3 . Svaka tacka prave p(H1 , H2 ) ima istu potenciju
u odnosu na kruznice k1 , k2 , i k3 sto znaci da te kruznice pripadaju istom
pramenu. Medjutim, ako su kruznice elementi istog pramena, njihovi centri, a
to su sredista duzi AC, BD, i EF , su kolinearne tacke.
Napomena 1. Stalnu vrednost proizvoda P A P B, gde je P data tacka i
kruznica k a tacke A i B su tacke preseka proizvoljne prave kroz tacku P sa
kruznicom k, naziva se potencija tacke P u odnosu na kruznicu k.
915.Ako su a, b, c stranice i ha , hb , hc visine trougla ABC, r poluprecnik
opisanog kruga i S povrsina trougaone povrsi (ABC), dokazati da je :
abc
a. S =
4r
r
b. S 2 = ha hb hc . Uputstvo: (a) Ako je D podnozje visine iz temena A, a AE
2
precnik kruga opisanog oko trougla ABC, bice ABD AEC, pa je AB : AD =
bc
2S
2S
bc
AE : AC, tj. c : ha = 2r : b. Otuda je ha =
. No ha =
, pa je
=
i
2r
a
a
2r
abc
. (sl. 509)
prema tome S =
4r

O
B

D
F

sl. 509

585

(b) Koristeci izvedeni deo ovog zadatka nalazimo da je


S2 =

1
1 abc 2S 2S 2S

= rha hb hc .
2 4S a
b
c
2

916.Ako su , a , b , c poluprecnici upisanih krugova trougla ABC, a p


poluobim tog trougla, dokazati da je:
a. S(ABC) = p, S(ABC) = (p a)a ; b. S 2 (ABC) = p(p a)(p b)(p c);
c. S 2 = a b c . Uputstvo: (a) Ako sa S obelezimo srediste upisanog kruga i
sa Sa srediste kruga koji dodiruje stranicu BC i produzenja stranica AB i AC
bice
S(ABC) = S(ASB) + S(BSC) + S(CSA) =

(AB + BC + CA) = p
2

a
(AB + AC BC) = (p a)a
2
. (b) Ako sa P i Pa obelezimo upravne projekcije tacaka S i Sa na pravoj BC,
bice BSP Sa BPa , pa je

S(ABC) = S(ASa B)+ S(ASa C) S(BSa C) =

a = (p b)(p c)
iz prethodnog dela ovog zadatka nalazimo da je
S 2 (ABC) = a p(p a)
, pa je
S 2 (ABC) = p(p a)(p b)(p c)
. (c) Iz prethodna dva dela, nalazimo da je
S 4 (ABC) = p(p a)(p b)(p c)a b c = S 2 (ABC)
pa je
S 2 (ABC) = a b c
917.Ako su b i c katete, a hipotenuza, p poluobim i S povrsina povrsi (ABC)
kojoj je ugao A prav, dokazati da je
S = p(p a) i S = (p b)(p c).
Uputstvo: Ako je poluprecnik upisanog kruga, kod pravouglog trougla ABC
bice p a = , pa je S = p(p a). Otuda i iz obrasca S 2 = p(p a)(p b)(p c)
nalazimo da je S = (p b)(p c).
918.Ako su A , B , C podnozja visina ostrouglog trougla ABC, a r poluprecnik opisanog kruga i p poluobim trougla A B C , dokazati da je
S(ABC) = rp .
Resenje: Ako Obelezimo sa O srediste opisanog kruga i sa A1 , B1 , C1 sredista
stranica BC, CA, AB trougao ABC, prema zadatku...., imamo da je BC :
B C = r : OA1 , CA : C A = r : OB1 , AB : A B = r : OC1 . Otuda je
B C + C A + A B =

1
(BC OA1 + CA OB1 + AB OC1 ).
r
586

Kako je trougao ABC ostrougli, srediste O opisanog kruga je u tom trouglu, pa


je
BC OA1 + CA OB1 + AB OC1 = 2S(ABC).

Stoga je S(ABC) = rp .
919.Ako su Sa , Sb , Sc sredista spolja upisanih krugova ostrouglog trougla
ABC, a p poluobim tog trougla i r poluprecnik opisanog kruga, dokazati da je
S(Sa Sb Sc ) = 2pr.
Uputstvo: Tacke A, B, C su podnozja visina trougla Sa Sb Sc , pa je opisani krug
l trougla ABC Ojlerov krug trougla Sa Sb Sc . No, poluprecnik Ojlerovog kruga
jednak je polovini poluprecnika opisanog kruga, pa je prema prethodnom zadatku S(Sa Sb Sc ) = 2pr.
920. Ako su na , nb , nc odsecci koje odreduje trougao ABC na pravama koje
sadrze srediste S upisanog kruga, a paralelne su respektivno na stranicama BC,
CA, AB, zatim ha , hb , hc visine iz temena A, B, C tog trougla, dokazati da je
S=

1
(na ha + nb hb + nc hc ).
4

Uputstvo: Ako obelezimo sa a, b, c stranice naspram temena A, B, C i sa


poluprecnik upisanog kruga trougla ABC, imamo da je
na
ha nb
hb nc
hc
,
,
=
=
=
a
ha
b
hb
c
hc
otuda je
na ha + nb hb + nc hc =
= a(ha ) + b(hb ) + c(hc ) =
= 6S (a + b + c) = 6S 2S = 4S
.

921.Ako su la i la simetrale unutrasnjeg i spoljasnjeg ugla A trougla ABC


kome su stranice AB i AC srazmerne datim duzima m i n, dokazati da je
S(ABC) =

m2 n 2
la la .
4mn

Uputstvo: Obelezimo sa E i F tacke u kojima simetrale unutrasnjeg i spoljasnjeg


ugla A trougla ABC seku pravu BC. Trouglovi ABC i AEF imaju zajednicku
visinu iz temena A, pa je S(ABC) : S(AEF ) = BC : EF . Ugao A trougla
1
1
AEF je prav, pa je S(AEF ) = AE AF = la la . S obzirom da su tacke E i
2
2
F harmonijski spregnute s tackama B i C, pri cemu je BE : EC = BF : EF =
AB : AC = m : n, prema zadatku ...., bice BC : EF = (m2 n2 ) : 2mn. Otuda
m2 n 2
BC
S(AEF ) =
la la .
je S(ABC) =
EF
4mn
922.Ako su ma , mb , mc tezisne linije trougla ABC, a m poluzbir tih tezisnih
linija, dokazati da je
S(ABC) =

4
m(m ma )(m mb )(m mc ).
3
587

Uputstvo: Ako su A , B , C sredista stranica BC, CA, AB trougla ABC, a T


teziste tog trougla i T tacka simetricna sa T u odnosu na A , bice BT T trougao
2
2
2
kome je T T = ma , BT = mb , BT = mc (sl.510).
3
3
3
4
S obzirom da je S(ABC) = 3S(BT T ), a prema zadatku... S(BT T ) = m(m
9
ma )(m mb )(m mc ), bice
S(ABC) =

4
m(m ma )(m mb )(m mc )
3

(sl. 441).

C
T
B

A
T

sl. 510
923.Ako su ha , hb , hc visine trougla ABC, a h poluzbir tih visina, dokazati
da je
1
= 4h(h ha )(h hb )(h hc ).
S(ABC)
Uputstvo: Ako je A B C trougao cije su stranice B C , C A , A B jednake
duzima ha , hb , hc bice
S(ABC) : S(A B C ) = a2 :
pa je

1
= a2 h2a = 4S 2 (ABC),
h2a

1
= 4S(A B C ) i prema tome
S(ABC)
1
= 4h(h ha )(h hb )(h hc)
S(ABC)

.
924.Ako su duzi a, b, c, d jednake stranicama AB, BC, CD, DA konveksnog
cetvorougla ABCD upisanog u krug k, a p poluobim tog cetvorougla, dokazati
da je
S(ABCD) = (p a)(p b)(p c)(p d).
Uputstvo: Prave odredene naspramnim stranicama cetvorougla ABCD se seku
ili su uporedne. Ako su uporedne, cetvorougao ABCD je pravougaonik, te se
dokaz izvodi neposredno. Zato pretpostavimo da se prave odredene bar jednim parom naspramnih stranica, npr. BC i AD, seku u nekoj tacki P . Kako
je cetvorougao ABCD konveksan, presecna tacka P pravih BC i AD je na
produzenjima tih stranica; Neka je P s one strane prave CD s koje nije stranica
(sl. 511).
588

sl. 511
Iz slicnosti trouglova OV D i P AB sledi da je S(P OD) : S(P AB) = c2 : a2 ,
a iz ove proporcije sledi da je
[S(P AB) S(P CD)] : S(P AB) = (a2 c2 ) : a2 ,
i prema tome S(ABCD) =
i BP, imamo da je
S(P AB) =

a2 c2
S(P AB). Ako sa m i n obelezimo duzi AP
a2

1
(a + m + n)(a + m + n)(a m + n)(a + m n).
4

S obzirom da je trougao P AB podudaran trouglu P CD imamo da je P A :


AB = P C : CD i P B : AB = P D : CD, tj. da je m : a = (n b) : c i
n : a = (m d) : c. Sabiranjem ili oduzimanjem odgovarajucih strana ovih
a(d b)
a(b + d)
i mn =
, i prema tome
proporcija, nalazimo da je m + n =
ac
a+c
da je
a
2a
a+m+n=
(a + b c + d) =
(p c),
ac
ac
2a
a
(a + b + c + d) =
(p a),
a + m + n =
ac
ac
a
2a
am+n=
(a + b + c d) =
(p d),
a+c
a+c
a
2a
a+m+n=
(a b + c + d) =
(p b).
a+c
a+c
Otuda je
a2
(p a)(p b)(p c)(p d)
S(P AB) = 2
a c2
a prema tome
S(ABCD) = (p a)(p b)(p c)(p d).

589

925. Ako su a, b, c, d stranice tetivnog i tangentnog cetvorougla ABCD,


dokazati da je
S 2 (ABCD) = abcd.
Uputstvo: Upisani krug cetvorougla ABCD je u ili izvan tog cetvorougla. U
prvom slucaju imamo da je a + c = b + d, pa je p a = c, p b = d, p c =
a, p d = b i prema tome
S 2 (ABCD) = (p a)(p b)(p c)(p d) = abcd.
Analogan postupak primenjuje se i u drugom slucaju.
926.Ako je ABCD konveksan cetvorougao upisan u krug k poluprecnika r,
E tacka u kojoj prava kroz C uporedna sa BD sece k i ako je AC = e, BD = f ,
AE = g; dokazati da je
ef g
.
S(ABCD) =
4r
Uputstvo: Dijagonala AC razlaze cetvorougaonu povrs (ABCD) na trougaone
povrsi (ABC) i (CDA), pa je S(ABCD) = S(ABC)+ S(CDA). Ako sa a, b, c, d
obelezimo stranice AB, BC, CD, DA, prema zadatku... imamo da je S(ABC) =
cde
e
abc
i S(CDE) =
, pa je S(ABCD) = (ab + cd). Primenom Ptolomejeve
4r
4r
4r
teoreme na cetvorougao ABED nalazimo da je ab + cd = f g, pa je
S(ABCD) =

ef g
.
4r

sl. 512
927.Ako je kod prostog cetvorougla ABCD zbir duzi koje spajaju sredista
naspramnih stranica jednak duzi l i ako je AC = BD = d, dokazati da je
S(ABCD) =

1 2
(l d2 ).
2

Uputstvo: S obzirom da su dijagonale cetvorougla ABCD jednake, sredista


K, L, M, N stranice AB, BC, CD, DA su temena romba KLM N . Stranice tog
romba jednake su polovini duzi d, a zbir njegovih dijagonala m i n jednak je
1
duzi l. Iz jednakosti m + n = 1 i m2 + n2 = d2 nalazimo da je mn = (l2 d2 ).
2
Stoga je
1
mn
= (l2 d2 ).
S(ABCD) = 2S(KLM N ) = 2
2
2
590

928.Ako su a i b stranice, d1 i d2 dijagonale i S povrsina paralelogramske


povrsi, dokazati da je
1
S 2 = a2 b2 (d21 d22 )2 .
16
Uputstvo: Ako su C i D podnozja upravnih iz temena C i D paralelograma
ABCD na pravoj AB, bice AD = BC . Pretpostavimo li da je ugao A ostar,
bice ugao B tup,pa je kod trouglova ABC i ABD AC 2 = AB 2 +BC 2 +2ABBC
i BD2 = AB 2 + AD2 2AB AD . Otuda je (sl.512)
AC 2 BD2 = 4AB AD = 4AB
AD2 DD2 = 4 AB 2 AD2 AB 2 DD2 = 4AB 2 AD2 S 2 , i prema tome
1
1
S 2 = AB 2 AD2 (AC 2 BD2 )2 , tj. S 2 = a2 b2 (d21 d22 )2 .
16
16

C
E
D

sl. 513
929. Ako obelezimo sa ABCD romb, a sa r1 i r2 poluprecnike krugova
opisanih oko trouglova ABC i ABD, dokazati da je
S(ABCD) =

8r12 r22
.
(r12 + r22 )2

Uputstvo:Ako je O tacka u kojoj se seku dijagonale AC i BD, E tacka u kojoj


krug oko ABD sece pravu AC i F tacka u kojoj krug ABC sece pravu BD,
bice (sl. 513)AO OE = OB OD i AO OC = DO OF. Stavimo li u ovim
jednakostima da je AO = m i BO = n dobijamo da je
m(2r1 m) = n2
i
n(2r2 n) = m2 ,
pa je
2r1 m = m2 + n2
i
2r2 n = m2 + n2 ,

591

i prema tome

n
m
= .
r2
r1
Ako vrednosti dobijenih razmera obelezimo sa k, bice m = kr2 , n = kr1 ,
2kr1 r2 = k 2 (r12 + r22 ). Otuda je
k=

2r1 r2
+ r22 )2

(r12

i prema tome

8r13 r23
.
+ r22 )2
930. Ako su a i b stranice i S povrsina paralelogramske povrsi (ABCD), zatim na , nb , nc , nd rastojanja proizvoljne tacke N od tacaka A, B, C, D, dokazati
da je
1
S 2 = a2 b2 (n2A n2B + n2C n2D )2 .
4
Uputstvo:Ako dijagonale AC i BD date paralelogramske povrsi obelezimo sa d1
1
i d2 , prema prethodnom zadatku imamo da je S 2 = a2 b2 (d1 2 d2 2 )2 .
16
No duz koja spaja tacku N s presekom O dijagonalom AC i BD je zajednicka
tezisna linija trougla N AC i N BD, pa je 4N O2 = 2nA 2 + 2nC 2 d1 2 i 4N O2 =
2nB 2 + 2nD 2 d2 2 . Iz ovih dveju jednakosti nalazimo da je
d1 2 d2 2 = 2(nA 2 nB 2 + nC 2 nD 2 ), pa je
1
S 2 = a2 b2 (nA 2 nB 2 + nC 2 nD 2 )2 (sl. 514).
4
S(ABCD) = 2mn =

(r12

Slika 514
931.Ako su a, b, c, d stranice, a e, f dijagonale i S povrsina bilo koje
cetvorougaone povrsi (ABCD), dokazati da je
1
(4e2 f 2 (a2 b2 + c2 d2 )2 ).
16
Uputstvo: Ako su E i F tacke takve da su cetvorouglovi ABEC i ACF D paralelogrami, bice i cetvorougao BEF D paralelogram. Sem toga je S(ABCD) =
1
S(BEF D), a prema prethodnom zadatku
2
1
S 2 (BEF D) = e2 f 2 (a2 b2 + c2 d2 ), te je
4
1
2
2 2
2
S =
[4e f (a b2 + c2 d2 )2 ] (sl. 515).
16
S2 =

592

Slika 515
932.Ako je P proizvoljna tacka u ili na pravilnom n-touglu A1 . . . An i
poluprecnik upisanog kruga tog poligona, zatim d1 , . . . , dn odstojanja tacke P
od pravih A1 A2 , . . . , An A1 , dokazati da je
d1 + . . . + dn = n.
Uputstvo: S obzirom da je tacka P u ili na poligonu A1 ...An , bice
S(A1 A2 P ) + ... + S(An A1 P ) = S(A1 ...An ). Ako s A obelezimo stranicu tog praa
a
vilnog poligona, imamo da je (a1 + ... + an ) = . Otuda je a1 + ... + an = n.
2
2
Tako je specijalno kod jednakostranog trougla taj zbir jednak visini trougla,
kod kvadrata jednak dvostrukoj stranici, itd. Ako bi tacka P bila izvan poligona A1 ...An , teorema ostaje tacna samo je u tom slucaju rec o algebarskom
zbiru odstojanja tacke P od pravih A1 A2 , ..., An A1 .
933.Ako su a1 , . . . , an odstojanja temena A1 , . . . , An pravilnog n-tougla A1 . . . An
proizvoljne dirke t kruga k opisanog oko tog n-tougla i ako je r poluprecnik kruga
k, dokazati da je
a1 + . . . + an = nr.
Uputstvo: Prvi nacin: Obelezimo sa t1 , ..., tn dirke kruga k u temenima
A1 , ..., An poligona A1 ...An , sa P dodirnu tacku dirke t i sa a1 , ..., an odstojanja
tacke P od pravih t1 , ..., tn . S obzirom da je odstojanje tacke A1 od prave t
jednako odstojanju tacke P od prave ti , bice a1 + ... + an = a1 + ... + an . Dirke
t1 , ..., tn kruga k obrazuju takode pravilan n-tougao B1 ...Bn kome je poluprecnik
upisanog kruga jednak duzi r. Tacka P je u ili na pravilnom poligonu B1 ...Bn ,
pa je prema prethodnom zadatku a1 + ... + an = nr. Otuda je a1 + ... + an = nr.
Drugi nacin. S obzirom da je poligon A1 ...An pravilan njegovo teziste T poklapa se sa sredistem O kruga k, a odstojanje tacke T od prave s jednako je
poluprecniku r kruga k, pa je prema zadatku ..., a1 + ... + an = nr.
934.Nad stranicama trougla ABC konstruisani su spolja jednakokraki trouglovi BCA , CAB , ABC kod kojih su unutrasnji uglovi pri vrhovima A , B ,
C medu sobom jednaki. Dokazati da se prave AA , BB , CC seku u jednoj
tacki.
593

Uputstvo: Ako sa P, Q, R obelezimo tacke u kojima prave AA , BB , CC


seku prave BC, CA, AB bice
S(BAA )
BP
=
,
PC
S(CAA )

CQ
S(CBB )
=
,
QA
S(ABB )

AR
S(ACC )
=
.
RB
S(BCC )

Iz slicnih trouglova ACB i ABC nalazimo da je AB AB = AC AC , pa


je s obzirom na jednakost uglova BAB i CAC S(ABB ) = S(ACC ). Istim
postupkom dokazuje se da je S(BAA ) = S(BCC ) i S(CAA ) = S(CBB otuda
BP CQ AR

= 1, pa se prema Cevijevoj
teoremi prave AP, BQ, CR prema
je
P C QA RB

tome i prave AA , BB , CC seku u jednoj tacki (sl. 516).

A
B

Slika 516
935. Ako su ha , hb , hc visine i a , b , c poluprecnici spolja upisanih krugova
trougla ABC, dokazati da je
hb + hc
hc + ha
ha + hb
+
+
= 6.
a
b
c
Uputstvo:Ako obelezimo sa a, b, c stranice, sa p poluobim i sa S povrsinu
trougane povrsi (ABC), bice
1
1
1
= (p a) =
(b + c a),
a
S
2S
1
1
1
= (p b) =
(c + a b),
b
S
2S
1
1
1
= (p c) =
(a + b c).
b
S
2S
Otuda je

hb + hc
hc + ha
ha + hb
+
+
=
a
b
c

594

1
(hb + hc )(b + c a) + (hc + ha )(c + a b) + (ha + hb )(a + b c) = 6.
2S
936.Ako je O tacka u ravni trougla ABC i ako su A , B , C tacke u kojima
prave AO, BO, CO seku respektivno prave BC, CA, AB, dokazati da je:
OB
OC
OA
+
+
= 1;
a.
AA
BB
CC
AO
BO
CO
b.
+
+
= 1.
AA
BB
CC
Uputstvo: a) Primenom poznatog stava nalazimo da je
S(OBC)
OA
=
,

AA
S(ABC)

S(OCA
OB
=
,

BB
S(ABC)

S(OAB)
OC
=
.

CC
S(ABC)

Sabiranjem odgovarajucih strana ovih jednakosti dobijamo da je


OA
OB
OC
S(OBC) + S(OCA) + S(OAB)
S(ABC)
+
+
=
=
=1

AA
BB
CC
S(ABC)
S(ABC)
b) S obzirom da je
A0 = AA OA ,
BO = BB OB ,

CO = CC OC , bice

OA
BO
OB
OC
CO
AO
=
1

,
=
1

=
1

.
,
AA
AA
BB
BB
CC
CC
Sabiranjem odgovarajucih strana ovih jednakosti i primenom prethodnog dela
ovog zadatka, nalazimo da je
BO
CO
OA
OB
OC
AO
+
+
=3(
+
+
)=2

AA
BB
CC
AA
BB
CC
937. Ako su AA , BB , CC visine trougla ABC, a A , B , C tacke u
kojima prave AA , BB , CC seku krug opisan oko trougla ABC, dokazati da
je
BB
CC
AA
+
+
= 4.
AA
BB
CC
Resenje: S obzirom da je AA = AA + A A , BB = BB + B B , CC =
CC + C C i da su tacke A , B , C simetricne s ortocentrom H trougla ABC
u odnosu na prave BC, CA, AB bice:
AA
BB
CC
AA + A A
BB + B B
CC + C C
+
+
=
+
+
=
AA
BB
CC
AA
BB
CC
=3+

B B
C C
S(HBC) + S(HCA) + S(HAB)
A A
+
+
=3+
=

AA
BB
CC
S(ABC)
S(ABC)
=3+
= 4.
S(ABC)

938. Ako je O tacka u trouglu ABC i ako su A , B , C tacke u kojima prave


AO, BO, CO seku odgovarajuce stranice BC, CA, AB, dokazati da je
AO BO CO
AO
BO
CO

=
+
+
+ 2.
OA OB OC
OA
OB
OC
595

C
O

Slika 517.
Resenje: Ako obelezimo sa S, S1 , S2 , S3 povrsine trougaonih povrsi (ABC),
(OBC), (OCA), (OAB) bice (sl. 517):
S S1
AA OA
=

OA
S1
pa je

AO
S2 + S3
=
OA
S1
Analognim postupkom nalazimo
S3 + S1
BO
=

OB
S2

CO
S1 + S2
=
OC
S3
Mnozenjem odgovarajucih strana triju jednakosti nalazimo da je

(1)

(2)
(3)

S2 + S3 S3 + S1 S1 + S2
S2 + S3 S3 + S1 S1 + S2
AO BO CO

=
+
+
+2.

OA OB OC
S1
S2
S3
S1
S2
S3
Otuda, s obzirom na jednakost (1), (2), (3) dobijamo da je
AO BO CO
AO
BO
CO

=
+
+
+ 2.

OA OB OC
OA
OB
OC

939. Ako su AA , BB , CC visine trougla ABC, a AA , BB , CC podnozja upravnih iz proizvoljne tacke O na pravama BC, CA, AB, dokazati da
je
OB
OC
OA
+
+
= 1.

AA
BB
CC

596

Resenje: Prema poznatom stavu, imamo da je


OA
S(OBC)
=
,
AA
S(ABC)
S(OCA)
OB
=
,

BB
S(ABC)
S(OAB)
OC
=
.
CC
S(ABC)
Sabiranjem odgovarajucih strana ovih jednakosti nalazimo da je
OA
OB
OC
S(OBC) + S(OCA) + S(OAB)
S(ABC)
+
+
=
=
= 1.
AA
BB
CC
S(ABC)
S(ABC)

940. Ako su a, b, c prave kroz proizvoljnu tacku O uporedne sa stranicama


BC, CA, AB trougla ABC i ako prava a sece prave AB i AC u tackama B1 i
C1 , prava b sece prave BC i BA u tackama C2 i A2 , prava c sece prave CA i
CB u tackama A3 i B3 , dokazati da je
C2 A2
A3 B3
B1 C1
+
+
= 2.
BC
CA
AB
Resenje: Ako su A , B , C tacke u kojima prave AO, BO, CO seku prave
BC, CA, AB, bice:
BO
CO
C2 A2
A3 B3
AO
B1 C1
+
+
= 2.
+
+
=
BC
CA
AB
AA
BB
CC

941. Ako su A , B , C tacke u kojima prave odredene precnicima AA , BB ,


CC kruga opisanog oko trougla ABC seku prave BC, CA, AB, dokazati da je
A A
B B
C C
+
+
= 1.

AA
BB
CC

Resenje: S obzirom da je A A = AA AA , B B = BB BB , C C =
CC CC , bice:
B B
C C
AA AA
BB BB
CC CC
A A
+
+
=
+
+
=
AA
BB
CC
AA
BB
CC
2(

AO
BO
CO
+
+
)3=1
AA
BB
CC

942. Ako je O srediste i r poluprecnik kruga opisanog oko trougla ABC,


a A , B , C tacke u kojima prave AO, BO, CO seku prave BC, CA, AB,
dokazati da je
1
1
2
1
+
+
= .

AA
BB
CC
r
597

Resenje: Prema zadatku 936, imamo da je


BO
CO
r
r
r
AO
+
+
=
+
+
= 2,
AA
BB
CC
AA
BB
CC
gde je

1
1
1
2
+
+
= .
AA
BB
CC
r

943. Ako su A , B , C tacke na stranicama ili produzenjima stranica BC,


CA, AB trougla ABC takve da se prave AA , BB , CC seku u jednoj tacki O,
dokazati da je
AO
AC
AB
=
+
.
OA
CB
BC
Resenje: Primenom poznate teoreme, nalazimo da je
S(AOB)
S(AOC)
S(AOB)
S(AOC)
AB
AC
AO
=
=
=
+
=
+
.
OA
S(OA B)
S(OA C)
S(BOC) S(BOC)
BC
C B
944. Ako su P , Q, R tacke u kojima krug upisan u trougao ABC dodiruje

stranice BC, CA, AB i ako je G Zergonova


tacka trougla ABC, a r poluprecnik
opisanog i poluprecnik upisanog kruga, dokazati da je
4r
AG BG CG

= .
GP GQ GR

Resenje: Ako sa a, b, c obelezimo stranice BC, CA, AB i sa p poluobim


trougla ABC, prema teoremi Van Obela imamo da je
AG
a(p a)
BG
b(p b)
CG
c(p c)
=
,
=
,
=
.
GP
(p b)(p c) GQ
(p c)(p a) GR
(p a)(p b)
Otuda je
abcp
4rpS(ABC)
4r
AG BG CG

=
=
= .
GP GQ GR
p(p a)(p b)(p c)
S 2 (ABC)

945. Ako su kod trouglova ABC i A B C uglovi A i A jednaki ili suplementni, dokazati da je
AB AC
S(ABC)
=
.
S(A B C )
A B A C
Resenje: Analizirajmo najpre slucaj kada su uglovi A i A jednaki. Na
polupravama AB, AC odredimo tacke B1 i C1 takve da je AB1 = A B i AC1 =
598

A C . Pri tome je S(ABC) : S(AB1 C1 ) = AB : AB1 i S(AB1 C) : S(AB1 C1 ) =


AC : AC1 , pa je
S(ABC)
AB AC
.
=
S(AB1 C1 )
AB1 AC1
Otuda je i

S(ABC)
AB AC
.
=

S(A B C )
A B A C

Sad analizirajmo slucaj kada su uglovi A i A suplementni. Na polupravoj AB


odredimo tacku B1 takvu da je AB1 = A B , a na produzenju poluprave AC
odredimo tacku C1 takvu da je AC1 = A C . Pri tome je
S(ABC)
AC
=
S(ABC1 )
AC1
i

S(ABC1 )
AB
,
=
S(AB1 C1 )
AB1

pa je
AB AC
S(ABC)
.
=
S(AB1 C1 )
AB1 AC1

Otuda je

S(ABC)
AB AC
.
=
S(A B C )
A B A C
946. Ako su na stranicama BC, CA, AB trougla ABC tacke P i P , Q i Q ,
R i R simetricne medu sobom u odnosu na sredista tih stranica, dokazati da je
S(P QR) = S(P Q R ).

Slika 518.
599

Resenje: Ako sa T i T obelezimo tacke u kojima prave QR i Q R seku


pravu BC, bice:
S(BQR) : S(P QR) = BT : P T
i
S(CQ R ) : S(P Q R ) = CT : P T
S obzirom da je prema Menelajevoj teoremi
BT CQ AR

= 1
T C QA RB
i

BT CQ AR

= 1,
T C Q A R B
a po pretpostavci CQ = Q A , CQ = QA, AR = R B, AR = RB, nalazimo
da je BT = T C i P T = T P . Otuda je
S(BQR) : S(P QR) = S(CQ R ) : S(P Q R ).
Medutim, S(BQR) = S(R QA) = S(CQ R ), pa je i S(P QR) = S(P Q R ) (sl.
518).
947. Ako stranice BC, CA, AB trougla ABC dodiruju upisani krug u
tackama P , Q, R i spolja upisane krugove u tackama P , Q , R , dokazati da je
S(P QR) = S(P Q R ).

Resenje: Prema zadatku 154, tacke P i P su simetricne medu sobom u


odnosu na srediste BC, tacke Q i Q simetricne medju sobom u odnosu na
srediste stranica CA, a tacke R i R simetricne medu sobom u odnosu na srediste
stranice AB, pa je prema prethodnom zadatku
S(P QR) = S(P Q R ).

948. Ako su stranice trougla A B C jednake tezisnim linijama trougla ABC,


dokazati da je
S(A B C ) : S(ABC) = 3 : 4.

C1

B1

A2

A1

600

Slika 519.
Resenje: Ako obelezimo sa AA1 , BB1 , CC1 tezisne linije trougla ABC i sa
A2 tacku takvu da je cetvorougao BB1 A2 A1 paralelogram, bice da su stranice
trougla AA1 A2 takode jednake tezisnim linijama trougla ABC, a presecna tacka
D duzi A1 A2 i CB1 zajednicko srediste tih duzi. Stoga je (sl. 519)
S(A B C ) = S(AA1 A2 ) = 2S(AA1 D) = 2[S(AA1 C) S(A1 CD)] =

1
3
= 2S(AA1 C)[1 ] = S(ABC).
4
4
949. Ako su a , b , c bilo kakve slicne poligonske povrsi konstruisane na
stranicama pravouglog trougla ABC, pri cemu su hipotenuza BC i katete CA
i AB odgovarajuce duzi u tom preslikavanju, dokazati da je
S(a ) = S(b ) + S(c ).
Resenje: Ako stranice BC, CA, AB trougla ABC obelezimo sa a, b, c; prema
poznatom stavu, imamo da je
S(b ) : S(a ) = b2 : a2
i
S(c ) : S(a ) = c2 : a2 .
Otuda je

b 2 + c2
S(b ) + S(c )
.
=
S(a )
a2

S obzirom da je
a2 = b 2 + c2 ,
bice i
S(a ) = s(b ) + S(c ).
Napomena:Ovaj stav predstavlja jedan vid uopstenja Pitagorine teoreme,
dokazao ga je vec Euklid, pa se i u literaturi ovaj stav i naziva Euklidova teorema
(sl. 520).

Wb

A
Wc

C
Wa

601

Slika 520.
950. Ako su tacke A1 , B1 , C1 sredista stranica BC, CA, AB proizvoljnog
trougla ABC i M bilo koja tacka njegove ravni, dokazati da je
S(M AA1 ) = S(M BB1 ) = S(M CC1 ).
Resenje: Ako trougao ABC paralelno projektujemo u jednakostranican trougao A B C , tacke A1 , B1 , C1 se projektuju u sredistima A1 , B1 , C1 stranica
B C , C A , A B trougla A B C , a tacka M u neku tacku M . Pri tome je
prema poznatom stavu (sl. 521a i sl.521b)

B1

C1
T

A1

Slika 521a.
S(M AA1 )
S(M BB1 )
S(M CC1)
=
=
.
S(M A A1 )
S(M B B1 )
S(M C C1 )
Obelezimo sa P , Q , R podnozja upravnih iz tacke M na pravama A A1 , B B1 ,
C C1 . S obzirom da su prave A A1 , B B1 , C C1 konkurentne i da za svake dve
od njih zahtevaju medu sobom ugao jednak s uglom 2R
3 , gde je R prav ugao,
bice:
M P = |M Q M R |.

602

M
A

C1

B1
T

A1

Slika 521b.
Otuda iz jednakosti duzi A A1 , B B1 , C C1 sledi da je
S(M A A1 ) = |S(M B B1 ) S(M C C1 )|,
pa je s obzirom na prethodne jednakosti
S(M AA1 ) = |S(M BB1 ) S(M CC1 )|.
951. Ako su A1 , B1 , C1 tacke stranica BC, CA, AB trougla ABC takve da
je BA1 : A1 C = CB1 : B1 A = AC1 : C1 B = m : n, a A2 , B2 , C2 tacke u kojima
se seku prave BB1 i CC1 , CC1 i AA1 , AA1 i BB1 , dokazati da je za m < n
m2 + mn + n2
S(ABC)
.
=
S(A2 B2 C2 )
(m n)2
Resenje: Prema poznatom stavu, paralelnim projektovanjem trougao ABC
se moze preslikati u jednakostanican trougao A B C . Pri tom se tacke A1 , B1 ,
C1 preslikavaju u tacke A1 , B1 , C1 koje se nalaze na stranicama B C , C A ,
A B i koje zadovaljavaju relacije (sl. 522)

603

B1
C2

C1
A2
B2

A1

Slika 522a.
B A1 : A1 C = C B1 : B1 A = A C1 : C1 B = m : n,
dok se tacke A2 , B2 , C2 preslikavaju u tacke A2 , B2 , C2 u kojima se seku prave
B B1 i C C1 , C C1 i A A1 , A A1 i B B1 . Sem toga je
S(A B C )
S(ABC)
=
.
S(A2 , B2 , C2 )
S(A2 , B2 , C2 )
Kako su uglovi B i A1 trougla B A1 C2 jednaki sa uglovima A i A1 trougla
A A1 B , trouglovi B A1 C2 i A A1 B su slicni. Primenom Apolonijeve teoreme
nalazimo da je koeficijent slicnosti tih dvaju trouglova odredjen relacijom
B A1
m
= 2
.
A A1
m + mn + n2

B1
C2

C1
A2
B2
A1

Slika 522b.
604

Stoga je

S(B A1 C2 ) =

m2

m3
m2
S(A A1 B ) =
S(A B C ).
2
2
+ mn + n
m (m + mn + n2

S druge strane imamo da je

S(A B C) = S(A2 B2 C2 ) + 3S(A A1 B ) 3S(B A1 C2 ),


pa je s obzirom na prethodne relacije
S(ABC)
S(A B C )
m2 + mn + n2
.
=
=

S(A2 B2 C2 )
S(A2 B2 C2 )
(m n)2
952. Ako su A1 , B1 , C1 , D1 tacke stranica AB, BC, CD, DA paralelograma
ABCD takve da je AA1 : A1 B = BB1 : B1 C = CC1 : C1 D = DD1 : D1 A =
m : n a A2 , B2 , C2 , D2 tacke u kojima se seku prave DA1 i A1 B, AB1 i BC1 ,
BC1 i CD1 , CD1 i DA1 , dokazati da je
(m + n)2 + m2
S(ABCD)
.
=
S(A2 B2 C2 D2 )
m2

Resenje: Prema poznatom stavu, paralelnim projektovanjem paralelogram


ABCD moze se preslikati u kvadrat A B C D . Pri tome se tacke A1 , B1 , C1 ,
D1 preslikavaju redom u tacke A1 , B1 , C1 , D1 koje pripadaju stranicama A B ,
B C , C D , D A i zadovoljavaju relacije (sl. 523)
A A1 : A1 B = B B1 : B1 C = C C1 : C1 D = D D1 : D1 A = m : n,
dok se tacke A2 , B2 , C2 , D2 preslikavanje u tacke A2 , B2 , C2 , D2 u kojima se
seku prave D A1 i A B1 , A B1 i C1 B , C D1 i B C1 , D A1 i C D1 . Osim toga
je
S(ABCD)
S(A B C D )
=
.
S(A2 B2 C2 D2 )
S(A2 B2 C2 D2 )

605

C1

D
D1

C2

D2
A2
A

B2
B1

A1

C1

D2
D1

C
C2
B2

B1

A1

A2

Slika 523.

Kako su uglovi A i A1

A B1 B , trouglovi A A1 A2
odreden je relacijom

trougla A A1 A2 jednaki sa uglovima A i B1 trougla


i A B1 B su slicni. Koeficijent slicnosti tih trouglova
A A1
m
=
.
A B1
(m + n)2 + m2

Stoga je
m2
S(A B1 B ) =
(m + n)2 + n2
m3
=
s(a b c d ).
2(m + n)(2m2 + 2mn + n2 )

S(A A1 A2 ) =

S druge strane imamo da je


A(A B C D ) = S(A2 B2 C2 D2 ) + 4S(A B1 B ) 4S(A A1 A2 ),
pa je s obzirom na prethodne relacije
S(A B C D )
(m + n)2 + m2
S(ABCD)
.
=
=
S(A2 B2 C2 D2 )
S(A2 B2 C2 D2 )
n2
953. Ako su P , Q, R tacke stranica BC, CA, AB trougla ABC takve da je
BP : P C = CQ : QA = AR : RB = m : n, dokazati da je
(a)
S(ARQ) = S(BP R) = S(CQR) =

mn
S(ABC);
(m + n)2

(b)
S(P QR) =

m2 mn + n2
S(ABC).
(m + n)2
606

Resenje:
(a) Trouglovi ARQ i ABC imaju zajednicki ugao A, pa je (sl. 524)
AR AQ
m
n
mn
S(ARQ)
.
=

=
S(ABC)
AB AC
m+n m+n
(m + n)2
Analogno se izvode i ostali obrasci iz (a).

Q
R

Slika 524.

(b) Duzima P Q, QR, RP razlozena je povrs (ABC) na cetiri povrsi (ARQ),


(BP R), (CQR), (P QR), pa je

S(ABC) S(ARQ) S(BP R) S(CQR)


S(P QR)
=
=
S(ABC)
S(ABC)
3mn
m2 mn + n2
=1
=
.
(m + n)2
(m + n)2

954. Ako su P , Q, R tacke stranice BC, CA, AB trougla ABC takve da je


BP : P C = p1 : p2 , CQ : QA = q1 : q2 , AR : RC = r1 : r2 , dokazati da je
(a)
r1 q1
S(AQR)
=
;
S(ABC)
(r1 + r2 ) (q1 + q2 )

(b)

p1 q1 r1 + p2 q2 r2
S(P QR)
=
.
S(ABC)
(p1 + p2 ) (q1 + q2 ) (r1 + r2 )
Resenje:
(a) Trouglovi ARQ i ABC imaju zajednicki ugao A, pa je

607

AR AQ
r1 q1
S(ARQ)
=

=
.
S(ABC)
AB AC
(r1 + r2 ) (q1 + q2 )
(b) Duzima P Q, QR, RP razlozena je povrs (ABC) na cetiri povrsi (ARQ),
(BP R), (CQR), (P QR), pa je

S(P QR) S(ABC) S(ARQ) S(BP R) S(CQR)


=
=
S(ABC)
S(ABC)
p1 r2
r1 q2

=1
(q1 + q2 ) (r1 + r2 ) (p1 + p2 ) (r1 + r2 )
q1 p2
p1 q1 r1 p2 q2 r2

=
.
(p1 + p2 ) (q1 + q2 )
(p1 + p2 ) (q1 + q2 ) (r1 + r2 )
955. Ako su M i N tacke stranica AB i AC trougla ABC takve da je
AM N ABC, dokazati da je
S 2 (ABN ) = S(AM N ) S(ABC).

Resenje: Trouglovi AM N i ABN imaju zajednicku visinu iz temena N , a


trouglovi ABN i ABC imaju zajednicku visinu iz temena B, pa je
AM
S(AM N )
=
S(ABN )
AB

S(ABN )
AN
=
.
S(ABC)
AC

S obzirom da je AM N ABC, imamo da je


AM
AN
=
,
AB
AC
pa je
S(AM N )S(ABN ) =

S(ABN )
S(ABC)

i prema tome
S 2 (ABN ) = S(AM N ) S(ABC).
956. Ako je O presek dijagonala AC i BD trpeza ABCD kome je AB > CD,
dokazati da je
S(ABCD) = S(ABO) + S(CDO).

Resenje: Ako obelezimo sa E tacku u kojoj prava c uporedna sa dijagonalom


BD sece pravu AB, imamo da je S(ABCD) = S(AEC). S obzirom da je
AEC ABC i ABO CDO, bice:
S(AEC) : S(ABO) = AC : AO = (AO + OC) : AO = 1 + OC : AO =
= 1 + S(CDO) : S(ABO).
608

otuda je
S(ABCD) = S(ABO) + S(CDO).

957. Ako su P i Q tacke stranica AB i AC trougla ABC i R tacka prave


P Q takve da je BP : P A = AQ : QC = P R : RQ, dokazati da je
S(RBC) = 2S(AP Q).

Resenje: Iz BP : P A = AQ : QC = P R : RQ = k sledi da je
AB : AP = k + 1,

AC : AQ = (k + 1) : k,

P Q : RQ = k + 1,

P Q : P R = (k + 1) : k.

Otuda je
AB AC
(k + 1)2
S(ABC)
=
=
,
S(AP Q)
AP AQ
k

S(BP R)
BP P R
k2
=
=
,
S(AP Q)
AP P Q
k+1

S(CQR)
CQ RQ
1
=
=
.
S(AP Q)
AQ P Q
k(k + 1)

Iz ovih jednakosti nalazimo da je

S(AP Q) + S(BRC)
S(ABC) S(BP R) S(CQR)
=
= 3,
S(AP Q)
S(AP Q)
tj. da je
S(RBC)
= 2.
S(AP Q)

958. Ako su P i Q tacke stranica AB i AC trougla ABC i R tacka duzi P Q


takve da je BP : P A = AQ : QC = P R : RQ, dokazati da je
k 2 S(ABC) = (k + 1)2 S(BQR) + [k(k + 1)]2 S(CQR).

Resenje: Iz BP : P A = AQ : QC = P R : RQ = k, sledi da je
S(BP R) = kS(AP R),
S(AQR) = kS(CQR),
S(AP R) = kS(AQR).
Koristeci prethodni zadatak i ove jednakosti, nalazimo da je
1
S(ABC) = 3S(AP Q)+S(BP R)+S(CQR)= 3S(AP Q)+kS(AP R)+ S(AQR)
k
609

1
k2
S(AP Q) +
S(AP Q)
k+1
k(k + 1)
(k + 1)2
(k + 1)2
=
S(AP Q) =
(S(AP R) + S(AQR))
k
k
(k + 1)2 1
( S(BP R) + kS(CQR))
=
k
k
= 3S(AP Q) +

(k + 1)2
S(BP R) + (k + 1)2 S(CQR).
k2

Otuda je
k 2 S(ABC) = (k + 1)2 S(BP R) + [k(k + 1)]2 S(CQR).

959. Ako je O presek dijagonala konveksnog trapeza ABCD, dokazati da je


S 2 (BOC) = S(AOB) S(COD).
Resenje: Trouglovi AOB i BOC imaju zajednicku visinu koja odgovara
starnicama AO i OC, a trouglovi BOC i COD imaju zajednicku visinu koja
odgovara stranicama BO i OD, pa je
S(AOB) : S(BOC) = AO : OC
i
S(BOC) : S(COD) = BO : OD.
S obzirom da je AO : OC = BO : OD, imamo da je
S(AOB) : S(BOC) = S(BOC) : S(COD)
i prema tome
S 2 (BOC) = S(AOB) S(COD).
960. Ako se dijagonale AC i BD cetvorougla ABCD seku u tacki O pod
pravim uglom, dokazati da je
S(AOB) S(COD) = S(BOC) S(AOD).
Resenje: S obzirom da su trouglovi AOB, COD, BOC i AOD pravougli,
imamo da je
1
S(AOB) S(COD) = OA OB OC OD
4
i
1
S(BOC) S(AOD) = OA OB OC OD.
4
Otuda je
S(AOB) S(COD) = S(BOC) S(AOD).
610

961. Ako je ABC trougao, P proizvoljna tacka stranice BC, a Q i R tacke


stranica AC i AB takve da je P Q k AB i P R k AC, dokazati da je
S 2 (AQR) = S(BP R) S(CP Q).
Resenje: Trouglovi BP R i AQR imaju jednake visine koje odgovaraju stranicama BR i RA, a trouglovi AQR i CQP imaju jednake visine koje odgovaraju
stranicama AQ i QC, pa je
S(BP R)
BR
=
,
S(AQR)
RA
AQ
S(AQR)
=
.
S(CQP )
QC
S obzirom da je BP R P CQ, imamo da je BR : P Q = RP : QC, tj. da
je BR : RA = AQ : QC, i prema tome da je S 2 (AQR) = S(BP R) S(CQP ).
962. Ako su OA i OB dva upravna poluprecnika kruga k, P i Q tacke u
kojima proizvoljna dirka kruga k sece prave OA i OB, M dodirna tacka i N
podnozje upravne kroz M na pravoj OA, dokazati da je
S 2 (OAB) = S(OM N ) S(OP Q).
Q
B
M

Slika 525.
Resenje: Kod trouglova OM N i AOB uglovi N i O su pravi, dakle jednaki,
a kod trouglova AOB i P OQ uglovi O istovetni, pa je (sl. 525)
M N ON
M N ON
S(OM N )
=
=
S(OAB)
OA OB
OM 2
i

OA OB
OM 2
S(OAB)
=
=
.
S(OP Q)
OP OQ
OP OQ
611

S obzirom da je OP M OM N i QOM OM N , imamo da je OM :


OP = ON : OM i OM : OQ = M N : OM . Otuda je
M N ON
OM 2
,
=
OP OQ
OM 2
te je
S(OM N )
S(OAB)
=
S(OAB)
S(OP Q)
i prema tome
S 2 (OAB) = S(OM N ) S(OP Q).
963. Ako su P , Q, R podnozja upravnih iz neke tacke M na pravama koje
su odredene stranicama BC, CA, AB trougla ABC i ako je O srediste i r
pluprecnik kruga l opisanog oko trougla ABC, a d duz koja spaja tacke O i M ,
dokazati da je
S(P QR)
1
d2
= (1 2 ),
S(ABC)
4
r
pri cemu se uzima znak + ili prema tome da li je tacka M u ili izvan kruga l.
A
D

Slika 526.
Resenje: Ako se tacka M nalazi na krugu l koji je opisan oko trougla ABC,
prema Simsonovoj teoremi tacke P , Q, R pripadaju jednoj pravoj. Zato analiziramo samo slucaj kada se tacka M nalazi u i izvan kruga l. Ako je tacka
M u krugu l i ako je D tacka u kojoj prava BM sece krug l, bice da je
RP Q = RP M + M P Q = RBM + M CQ = ABD + M CA =
ACD + M CA = M CD. Otuda je (sl. 526)
1
1
P R P Q sin RP Q = P R P Q sin M CD.
2
2
S obzirom da su duzi BM i CM precnici krugova opisanih oko tetivnih cetvorouglova
BP M R i CQM P , imamo da je P R = M B sin B i P Q = M C sin C. Kod trougla
S(P QR) =

612

M CD nalazimo da je
M C sin M CD = M D sin M DC = M D sin A.
Stoga je
S(P QR) =

1
1
P R P Q sin M CD = M B M C sin M CD
2
2

1
M B M D sin A sin B sin C.
2
Analogan postupak primenjuje se u slucaju kada se tacka M nalazi izvan kruga
l. Ako sa X i Y obelezimo tacke u kojima OM sece krug l, bice
=

M B M D = M X M Y = (r d)(r + d) = (r2 d2 ),
pri cemu vazi znak plus kada je tacka M u krugu l, a znak minus kada je tacka
M izvan kruga l. Iz
S(ABC) =

1
1
bc sin A = (2rs sin B)(2r sin C) sin A = 2r2 sin A sin B sin C
2
2

nalazimo da je
sin A sin B sin C =
Otuda je

S(ABC)
.
2r2

1
d2
S(P QR) = (1 2 )S(ABC),
4
r

i prema tome

1
d2
S(P QR)
= (1 2 ).
S(ABC)
4
r

964. Ako su r i poluprecnici opisanog i upisanog kruga trougla ABC, a P ,


Q, R tacke u kojima upisani krug dodiruje stranice BC, CA, AB dokazati da
je

S(P QR)
= .
S(ABC)
2r
A

Q
R
K

Slika 527.
613

Resenje:
Prvi nacin. Ako obelezimo sa O srediste opisanog kruga, sa K srediste
upisanog kruga i sa d duz koja spaja tacke O i K, prema prethodnom zadatku
bice (sl. 527):
1

1
d2
r2 2r
S(P QR)
)= .
= (1 2 ) = (1
2
S(ABC)
4
r
4
r
2r
Drugi nacin. Uglovi QKR, RKP , P KQ suplementni su uglovima A, B, C
trougla ABC, pa je
S(QRK)
KQ KR
2
=
= ,
S(ABC)
AB AC
bc
S(RP K)
2
= ,
S(ABC)
ca

2
S(P QK)
= .
S(ABC)
ab
Sabiranjem odgovarajucih strana ovih jednakosti nalazimo da je
1
1
1
a+b+c
S(QRK) + S(RP K) + S(P QR)
=( +
+ )=
.
S(ABC)
ab bc ca
abc
Otuda je
S(P QR)

= .
S(ABC)
2r
972. Ako prave odredene dvema tetivama AB i CD nekog kruga k sadrze
srediste S tetive M N tog istog kruga dokazati da su tacke X i Y u kojima prava
M N sece prave AD i BC simetricne medu sobom u odnosu na tacku S.
Uputstvo: Ako obelezimo sa SA , SB , SC i SD povrsine trougaonih povrsi
SAX, SBY , SCY i SDX bice (slika 532) :

614

Y
N

C
A

slika 532
SA
AX AS SD
DY DS SA
SA SX SD
DS SX
=
=
=
=
,
,
,
SC
CY CS SB
BY BS SB
SY SB SC
SC SY
pa je
AX AS DX DS SA SD
SA SX D S D X
SA SD
=
i
=
SB SC
BY BS CY CS SB SC
SB SY SC SY
Iz ovih dveju relacija sledi da je
SX 2
AX DX
MX NX
=
=
2
SY
BY CY
MY NY
Ako stavimo da je M S = SN = 1, SX = x i SY = y bice
12 x2
x2
=
, pa je x = y, tj. SX = SY .
z2
1 2 y2
973. Ako su a, b, c stranice trougla ABC takve da je a b = b c = d i ako
je poluprecnik upisanog kruga, dokazati da je
b2 4d2 = 122 .
Uputstvo: Prema zadatku 916b, imamo da je S 2 (ABC) =
prema zadatku 916a, imamo da je S 2 (ABC) =

615

3b2 (b2 4d2 )


, a
16

9b2 2
. Uporedivanjem tih jed4

nakosti nalazimo da je b2 4d2 = 122 .


974. Ako su a, b, c stranice trougla ABC takve da je a b = b c i ako je
r poluprecnik opisanog kruga, a poluprecnik upisanog kruga, dokazati da je
ac = 6r.
3b
Uputstvo: Prema zadatku 916, imamo da je S(ABC) =
, a prema
2
abc
. Uporedivanjem tih jednakosti nazadatku 915, imamo da je S(ABC) =
4r
lazimo da je ac = 6r .
975. Ako je D tacka u kojoj raspolovnica ugla A sece stranicu BC trougla
ABC, a E i F tacke prave BC takve da je DAE = DAF , dokazati da je
BE CF
BE BF
AB 2
AE 2
=
=
i
.
2
2
AF
BF CF
AC
CE CF
Uputstvo: Trouglovi ABE i AF C imaju jednake uglove kod temena A i
jednake visine iz tog temena, pa je (vidi sliku 533) :

slika 533
S(ABF )
AB AE S(ABE)
BE
=
i
=
.
S(AF C)
AC AF S(AF C)
CF
otuda je

ab ae
be
=
ac af
cf
616

(1)

Isto tako trouglovi AEC i ABF imaju jednake uglove kod temena A i jedAC AE S(AEC)
CE
S(AEC)
=
i
=
.
nake visine iz tog temena, pa je
S(ABF )
AB AF S(ABF )
CF
otuda je

ac ae
ce
=
ab af
bf

(2)

Mnozenjem odgovarajucih strana jednakosti (1) i (2), a zatim deljenjem istih


nalazimo da je
BE CE AB 2
BE BF
AE 2
=
=
i
.
2
2
AF
BF CF AC
CE CF

12

NEJEDNAKOSTI U PLANIMETRIJI

976. Ako za svake tri prave proizvoljnog skupa pravih jedne ravni postoji
krug poluprecnika r koji sa svakom od tih triju pravih ima zajednickih tacaka,
dokazati da postoji krug poluprecnika r koji sa svakom pravom datog skupa ima
zajednickih tacaka.
Uputstvo: Neka je a1 , ..., an skup od n pravih jedne ravni, pri cemu sa
svake tri prave tog skupa postoji krug poluprecnika r koji sa svakom od tih
triju pravih ima zajednickih tacaka. Obelezimo sa i (i = 1, ..., n) skup svih
tacaka ravni kojima su odstojanja od prave ai manja ili jednaka duzi r. Skup
tacaka predstavlja deo ravni ogranicen dvema uporednim pravama koje su na
odstojanju r od prave ai , prema tome, i je konveksna povrs. Dakle u ravni
imamo skup 1 , ..., n od n konveksnih povrsi. S obzirom da za svake tri
prave datog skupa pravih postoji krug koji sa svakom od tih triju pravih ima
zajednickih tacaka, svake tri povrsi iz skupa 1 , ..., n sadrze srediste odgovarajuceg kruga, prema tome, imaju najmanje jednu zajednicku tacku. Stoga,
prema Halijevoj teoremi svih n povrsi 1 , ..., n imaju najmanje jednu zajednicku tacku, npr. O . Pri tome su odstojanja tacke O od pravih a1 , ..., an
manja ili jednaka duzi r, prema tome, krug k(O, r) ima sa svakom pravom datog
skupa pravih zajednickih tacaka .
977. Ako je d dijametar bilo kojeg ogranicenog skupa a1 , . . . , an pravih jedne
d
koji sa svakom pravom
ravni, dokazati da postoji krug poluprecnika r 2
3
datog skupa ima najmanje jednu zajednicku tacku.
Uputstvo: Bilo koje tri prave ai , aj , ak (i , j , k = 1, ..., n) datog skupa pravih
obrazuju trougao kome je obim 2p 3d. Krug najmanjeg poluprecnika koji
sa pravama ai , aj , ak ima zajednickih tacaka je upisani krug u trougao kojeg
obrazuju te tri prave. S obzirom da od svih trouglova opisanih oko nekog kruga
polupre
cnika r jednakostrani
can trougao ima najmanji obim, koji je jednak

6r 3, imamo da je 6r 3 2p 3d. Odatle sledi da postoji krug poluprecnika


d
r koji sece sve tri prave ai , aj , ak . Stoga, prema zadatku 976, postoji
2 3
d
krug poluprecnika r koji sece sve prave datog skupa pravih.
2 3
617

978. Ako je d dijametar bilo kojeg skupa tacaka neke ravni, dokazati da
postoji kruzna povrs poluprecnika r = d3 koja sadrzi sve tacke tog skupa
(Jungova teorema).
Uputstvo: Nijedna od stranica trougla Ai Aj Ak odredenog trima tackama
datog skupa A1 , A2 , ..., An tacaka nije veca od duzi d. Ako je taj trougao tupougli ili pravougli, sve njegove tacke pripadaju kruznoj povrsi ijk kojoj je
precnik najveca stranica tog trougla. Poluprecnik te kruzne povrsi nije veci od
d
d
, dakle manji od . Ako pretpostavimo da je trougao Ai Aj Ak ostrougli,
2
3
2
bar jedan njegov unutrasnji ugao, npr. A1 nije manji od R, gde je R prav
3

2
ugao. Iz Ai R sledi da je Aj Ak 3, gde je poluprecnik kruzne povrsi
3

opisane oko tog trougla. Pri tome je 3 Aj Ak d, pa je 3 d, i prema


d
tome . Stoga, ma kakav bio trougao Ai Aj Ak uvek postoji kruzna povrs
3
d
poluprecnika r = kojoj pripadaju tacke Ai , Aj , Ak . Na taj nacin mi smo
3
dokazali da ma koje tri od n tacaka A1 , A2 , ..., An jedne ravni pripadaju nekoj
d
kruznoj povrsi poluprecnika . Stoga, prema zadatku 976, postoji kruzna
3
d
povrs poluprecnika r = koja sadrzi sve tacke datog skupa tacaka.
3
979. Ako je d dijametar bilo kojeg ogranicenog skupatacaka u ravni, dokazati
da postoji pravilna trougaona povrs sa stranicama a 3d koja sadrzi sve tacke
tog skupa tacaka.
Uputstvo: Obelezimo sa ABC bilo koji jednakostranicni trougao opisan oko
datog skupa tacaka, a sa A B C njemu homotetican trougao koji je takode
opisan oko datog skupa tacaka i sa O proizvoljnu tacku koja se nalazi u svakom
od trouglova ABC i A B C . Neka su P , Q, R upravne projekcije tacke O na
stranicama BC, CA, AB trougla ABC, a P , Q , R upravne projekcije tacke O
na stranicama B C , C A , A B trougla A B C . Prema poznatom stavu imamo
da je zbir duzi OP , OQ, OR jednak visini h trougla ABC, a zbir duzi OP ,
OQ , OR jednak visini h trougla A B C . S obzirom da su prave odredene
odgovarajucim stranicama trouglova ABC i A B C uporedne i svaka od tih
pravih sadrzi najmanje po jednu tacku datog skupa tacaka kome je dijametar
d, imamo da je
OP + OP d,

OQ + OQ d,

OR + OR d.

Sabiranjem odgovarajucih strana nalazimo da je


OP + OA + OP + OQ + OR 3d
tj. da je
h + h 3d.

3
Odatle sledi da je jedna od visina h i h manja ili jednaka d, pa je i stranica
2

koja odgovara toj visini manja ili jednaka 3d.


980. Ako je a1 , . . . , an konacan skup od n duzi, dokazati da je
a2
an
a1
+
+ ... +
n.
a2
a3
a1
618

Uputstvo: Prema nejednakosti za aritmeticku i geometrijsku sredinu, imamo


da je
r
a2
an
an
a1
a1 a2
+
+
nn

a2
a3
a1
a2 a3
a1
pa je

a2
an
a1
+
+
n.
a2
a3
a1

981. Ako je a1 , . . . , an konacan skup od n duzi, dokazati da je


(a1 + . . . + an )(

1
1
+ ...+
) n2 .
a1
an

Uputstvo: Prema nejednakosti za aritmeticku i harmonijsku sredinu, imamo


da je
1
1
),
a1 + + an n2 ( + +
a1
an
pa je
1
1
(a1 + an )( + +
) n2 .
a1
an
Drugi nacin : S obzirom da je
(a1 + an )(
=n+(
i

1
1
+ ) =
a1
an

a1
a2
a1
an
a2
a3
an
an1
+ )+ + (
+
)+ ( + ) + + (
+
)
a2
a1
an
a1
a3
a2
an1
an
aj
ai
+
2
aj
ai

bice
(a1 + + an )(

za i, j = 1, ..., n

1
1
n(n 1)
+ +
)n+
2 = n2 .
a1
an
2

982. Ako je a1 , . . . , an konacan skup od n duzi, dokazati da je


1
(a1 + . . . + an )2 a21 + . . . + a2n (a1 + . . . + an )2 .
n
Uputstvo: S obzirom da je
(a1 + + an )2 = a21 + + a2n + 2(a1 a2 + + an1 an )
i
2ai aj a2i + a2j

za i, j = 1, ..., n

bice
(a1 + + an )2 a21 + + a2n (a1 + + an )2 .
983. Ako su An i Hn aritmeticka i harmonijska sredina duzi a1 , . . . , an
dokazati da je
(a) min(a1 , . . . , an ) Hn
(b) max(a1 , . . . , an ) An .
619

Uputstvo:
a) Neka je npr. a1 a2 an .
Iz ove pretpostavke sledi da je

a1
a1
a1
+
+ +
n,
a1
a2
an
pa je a1

n
, tj. da je min(a1 , ..., an ) Hn .
1
1
+ +
a1
an

b) Neka je npr. a1 a2 an . Iz ove pretpostavke sledi da je


a21 + + a2n na2n .
Prema zadatku 982, imamo da je

1 2
(a + + a2n ) a21 + + a2n pa je
n 1

1
(a1 + + an ) an ,
n
tj. da je max(a1 , ..., an ) An .
984. Ako su P i Q sredista stranica AD i BC cetvorougla ABCD, dokazati
da je
|AB CD| 2P Q AB + CD.
Uputstvo: Ako obelezimo sa R srediste dijagonale AC, imamo da je AB =
2 QR i CD = 2 P R (sl. 534). Oduzimanjem i sabiranjem odgovarajucih strana
ovih jednakosti nalazimo da je
|AB CD| = 2 |QR P R| 2 P Q
AB + CD = 2 (QR + P R) 2 P Q.

C
D
R

P
A

Sl. 534.
985. Ako je kod prostog cetvorougla ABCD A = B i C = D, dokazati
da je BC > AD.
Uputstvo: Vidi zadatak 50.

620

986. Ako je kod prostog cetvorougla ABCD A = B i BC > AD, dokazati


da je D > C.
Uputstvo: Vidi zadatak 51.
987. Ako je kod prostog cetvorougla ABCD AD = BC i A > B, dokazati
da je C > D.
Uputstvo: Vidi zadatak 52.
988. Dokazati da je kod svakog trougla ABC
h a l a ma .
Uputstvo: Vidi zadatak 41.
989. Dokazati da kod trougla vecoj stranici odgovara manja tezisna linija, i
obrnuto, da manjoj tezisnoj liniji odgovara veca stranica.
Uputstvo: Neka je kod trougla ABC AC > AB, a T tacka u kojoj se seku
tezisne linije AA , BB , CC . Iz trouglova AA C i AA B sledi da je AA C >
AA B, tj. T A C > T A B, a iz trouglova T A C i T A B da je CT > BT ,
pa je i BB < CC . Reverzibilnim postupkom dokazuje se drugi deo stava (sl.
535).

B
T

Sl. 535.
990. Ako je kod trougla ABC AC > AB, dokazati da je lb < lc .
Uputstvo: Prvi nacin.
Ako obelezimo sa E i F tacke u kojima simetrale uglova B i C seku naspramne
stranice trougla ABC i sa K tacku takvu da je cetvorougao BEKF paralelogram, bice (sl. 536) F KC = F KE + EKC i F CK = F CE + ECK.
S obzirom na pretpostavke, imamo da je
F KE = F BE =

1
1
B > C = F CE.
2
2

621

Sl. 536.
Iz jednakosti CE > BF sledi da je CE > KE, pa je kod trougla CKE
EKC > ECK. Stoga je F KC > F CK, pa je CF > F K, i prema
tome CF > BE.
Drugi nacin.
Razvijanjem obrasca za
lb2 = a c

b2 a c
,
(a + c)2

lc2 = a b

c2 a b
,
(a + b)2

pa je
lc2 lb2 = a (b c) + a b c
S obzirom da je b > c, bice


c
b
.

(a + c)2
(a + b)2

c
b

, pa je lc2 lb2 > 0, i prema


(a + c)2
(a + b)2

tome lb < l(c).


991. Ako je ABC jednakostranican trougao i P proizvoljna tacka njegove
ravni, dokazati da je
AP BP + CP.

Uputstvo: Ako obelezimo sa BAP trougao obrtno podudaran sa trouglom


BCP , tada je AP AP + P P . Medutim, P P = BP i AP = BP , pa otuda
je (sl. 537)
AP BP + CP
A
A
P

P
B
B

C
P

622

Sl. 537.
Znak jednakosti vazice samo u slucaju kada je tacka P na duzi AP . Taj slucaj
nastupa kad se tacka P nalazi na onom luku BC opisanog kruga na kome nije
teme A.
992. Ako je C srediste kruznog luka ALB i D bilo koja druga tacka tog
kruznog luka, dokazati da je
AC + BC > AD + BD.
Uputstvo: Obelezimo sa k krug kome je srediste C, a poluprecnik CA, odnosno CB. Pri tome je tacka D u krugu k, te prava AD sece krug k u tacki A
i jos nekoj tacki E koja se nalazi iza tacke D u odnosu na tacku A.

C
l
E

D
A
B

Sl. 538.
S obzirom da je ADB spoljasnji ugao trougla BDE, bice ADB = DEB +
DBE. Otuda i iz jednakosti ADB = ACB = 2 DEB sledi da je
DEB = DBE, pa je BD = DE. Stoga je (sl.538) AC + BC = AC + CE >
AE = AD + DE = AD + BD, pa je stav dokazan.
993. Ako su duzi a, b, c jednake
nekog trougla, dokazati da su

stranicama
za svaki prirodni broj n odsecci n a, n b, n c takode jednaki stranicama nekog
trougla.


n
n
Uputstvo: S obzirom da je b + c > a bice n b + n c = b + bn1 c +

. . . + c > ( n a) , pa je n b + n c > n a
994. Ako su b i c katete i a hipotenuza pravouglog trougla, a n prirodan broj
veci od 2, dokazati da je

(a) a 2 b + c,
(b) an > bn + cn .
Uputstvo: S obzirom da je a2 = b2 + c =

(b + c)2 , i prema tome a 2 b + c.

623

(b + c)2
(b c)2
+
, bice 2 a2
2
2

995. Ako je razlika kateta b i c pravouglog trougla jednaka simetrali pravog


ugla, dokazati da je pri b > c

2+ 6
b
=
.
c
2
Resenje: Ako je E tacka u kojoj simetrala pravog ugla A sece hipotenuzu BC
pravouglog trougla ABC i D tacka iza A u odnosu na C takva da je AB = AD,
bice
CDB CAE
pa je
DC : AC = DB : AE
i prema tome

(b + c) : b = c 2 : AE.

bc 2
bc 2
. Po predpostavci je AE = b c, pa je
= b c, tj.
Otuda je AE =
b+c
b+c

2+ 6
b
bc 2 = b2 c2 . Iz ove relacije nalazimo da je =
(sl.539).
c
2
D

Slika 539.
996. Ako su a, b, c hipotenuza i katete pravouglog trougla, ha visina koja
odgovara hipotenuzi i poluprecnik kruga upisanog u taj trougao, dokazati da
je
(a) b + c < a + ha ,
2
1
(b) ha < < ha .
5
2
Resenje:
(a) Ako je D podnozje visine iz temena A na hipotenuzu BC pravouglog trougla
ABC, bice ABC DAC, pa je AB : BC = AD : AC = k, tj. AB = kBC
i AD = kAC, pri cemu je k < 1. Otuda je AB AD = k(BC AC) pa je
AB AD < BC AC, i prema tome AB + AC < BC + AD, tj. b + c < a + ha .
(b) Ako je p poluobim pravouglog trougla ABC i R tacka u kojoj upisani krug
624

dodiruje stranicu AB, bice AR = p a. S obzirom da je ugao A prav imamo da


je AR = , pa je
1
= p a = (b + c a).
(1)
2
Ako je D podnozje visine iz temena A, bice ABC DBA, pa je
a : b = c : ha ,
i prema tome

bc
.
(2)
a

a
Iz jednakosti (1) i (2) sledi da je
=
(b + c a).
ha
2bc
2

2
> , dovoljno je dokazati da
Da bismo dokazali da je ha < , tj. da je
5
ha
5
a
2
2
je
(b + c a) > , tj. da je 5a(b + c) > 5a + 4bc. Iz ove nejednakosti i iz
2bc
5
2
a = b2 + c2 nalazimo posle kvadriranja da je 5b3 c 8b2 c2 + 5bc3 > 0. Oduzimanje od leve strane 2b2 c2 dobijamo nejednakost 5bc(b c)2 > 0, koja je tacna
2
2

> , pa je ha < .
jer je (b c)2 > 0. Prema tome, tacna je i nejednakost
ha
5
5
1

1
< , dovoljno je dokazati da
Da bi smo dokazali da je < ha , tj. da je
2
ha
2
a
1
2
je
(b + c a) < , tj. da je a(b + c) < a + bc. Iz ove nejednakosti i iz
2bc
2
2
2
a = b + c2 , dobijamo posle kvadriranja nejednakost b2 c2 > 0, koja je tacna pa
1
je i < ha .
2
ha =

997. Ako je ha visina koja odgovara hipotenuzi pravouglog trougla , r poluprecnik opisanog
i poluprecnik upisanog kruga tog trougla, dokazati da je

(a)ha (
2
+
1),

(b) r( 2 1).
Resenje:
(a) Obelezimo sa S srediste kruga upisanog u trougao ABC, sa Q tacku u kojoj
taj krug dodiruje stranicu AC, sa D podnozje visine iz tacke A i sa E podnozje
upravne kroz S na pravoj AD. Duz AS je hipotenuza pravouglog trougla SAE
ili se poklapa sa duzi AE, pa je AE AS. S obzirom da je tacka E izmedu
tacaka A i D, bice AE = AD DE, tj. AE = ha . Duz AS je hipotenuza

pravouglog trougla ASQ


cije su katete AQ i SQjednake duzi , pa je AS = 2.
Stoga je ha 2, i prema tome ha ( 2 + 1).
(b) Ako je R tacka u kojoj upisani krug dodiruje stranicu AB i p poluobim
trougla ABC, bice AR = p a, i prema tome
=
S obzirom da je 2bc a2 , imamo

a b+c
(
1).
2 a

a2+2bc
2bc
1,tj.
2. Otuda je
2
a
a2

b2 + 2bc + c2
2,
a2
625

(1)

i prema tome

Iz (1) i (2) sledi da je

b+c
2.
a

(2)

a
( 2 1) i prema tome r( 2 1).
2

998. Ako je r poluprecnik opisanog kruga, poluprecnik upisanog kruga i


ha visina iz temena A proizvoljnog trougla ABC, dokazati da je
(a)r 2.,
(b)ha > 2.
Resenje:
(a) Ako je O srediste opisanog kruga i S srediste upisanog kruga trougla ABC,
prema poznatoj Ojlerovoj teoremi (v. z. 271), bice OS 2 = r(r 2), pa je
r 2 O, i prema tome r 2. Znak jednakosti vazi samo u slucaju kada su
tacke O i S istovetne, tj. kada je trougao ABC jednakostranican.
(b) Ako obelezimo sa S srediste upisanog kruga trougla ABC, sa Sa srediste
spolja upisanog kruga koji dodiruje stranicu BC i sa E tacku kojoj
simetrala ugla A, tj. prava SSa , sece stranicu BC, bice AS : SE = ASa : Sa E.

Ako zatim obelezimo sa S i Sa upravne projekcije tacaka S i Sa na pravoj

koja sadrzi visinu AD, bice i AS : S E = ASa : Sa E. No ASa > Sa E, pa je i

AS > S E. Otuda je AD > 2S D, tj. ha > 2.


999. Ako su a, b, c stranice trougla pri cemu je a b i a c, zatim ha visina
kojoj odgovara stranica a a i a poluprecnici upisanih krugova, dokazati da je
(a) ha 3,
(b) ha a .
Resenje:
(a) Ako obelezimo sa D podnozje visine iz temena A trougla ABC, sa E tacku

kojoj simetrala ugla A sece stranicu BC, sa S srediste upisanog kruga k i sa S


podnozje upravne iz tacke S na pravoj AD, imamo da je

AS : SE = AS : S D.
Kako je
AS : SE = (b + c) : a,
pa je i

AS : S D = (b + c) : a
tj.
(ha ) : = (b + c) : a.
Iz b a i c a sledi da je (b+c) : a 2, pa je i (ha ) : 2. Otuda je ha 3.
(b) Ako obelezimo sa D podnozje visine iz temena A trougla ABC, sa E tacku
u kojoj simetrala ugla A sece stranicu BC, sa Sa srediste spolja upisanog

kruga ka i sa Sa podnozje upravne iz tacke Sa na pravoj AD, imamo da je

ASa : ESa = ASa : DSa .


626

Kako je
ASa : ESa = (b + c) : a,
pa je i

ASa : DSa = (b + c) : a,
tj.
(ha + a ) : = (b + c) : a.
Iz b a i c a sledi da je (b + c) : a 2, pa je i (ha + a ) : 2. Otuda je
h a a .
1000. Ako je r poluprecnik opisanog kruga i poluprecnik upisanog kruga
tetivnog i tangentnog cetvorougla, dokazati da je

r 2.
Resenje: Ako obelezimo sa d duz odredenu sredistima opisanog i upisanog kruga,
prema zadatku 304, bice
d2 = r2 + 2
Otuda je
pa je

p
4r2 + 2 .

p
r2 + 2 4r2 + 2 0,
r 2 + 2

p
4r2 + 2 .

Kvadriranjem obeju strana nalazimo da je

r4 + 2r2 2 + 4 4r2 2 + 4 ,
pa je
r4 2r2 2 ,
i prema tome

r 2.

1001. Ako je r poluprecnik opisanog kruga, poluprecnik upisanog kruga i


ha najveca visina netupouglog trougla ABC, dokazati da je
r + ha .
Resenje: S obzirom da trougao ABC nije tupougli, srediste O opisanog kruga je
u tom trouglu ili se poklapa sa sredistem neke njegove stranice. Ako sa A , B , C
obelezimo podnozja upravnih kroz O na stranicama BC, CA, AB prema Ptolemejevoj teoremi primenjenoj na tetivne cetvorouglove OB AC , OC BA , OA CB
nalazimo da je
b
c
OB + OC = r B C ,
2
2
627

a
c
OC + OA = r A C
2
2
a
b
OA + OB = r A B .
2
2
Sabiranjem odgovarajucih strana ovih jednakosti nalazimo da je
b
c
a
p (OA + OB + OC ) ( OA + OB + OC ) = rp.
2
2
2
Otuda je
p (OA + OB + OC ) S = rp,
gde je S povrsina trougaone povrsi (ABC). S obzirom da je S = p, bice
r + = OA + OB + OC . Iz proporcija
S(OBC) : S(ABC) = OA : ha
S(OCA) : S(ABC) = OB : hb
S(OAB) : S(ABC) = OC : hc
nalazimo da je OA =
je
r+=

S(OBC)
S(OCA)
S(OAB)
ha , OB =
hb , OC =
hc , pa
S(ABC)
S(ABC)
S(ABC)

1
[ha S(OBC) + hb S(OCA) + hc S(OAB)].
S(ABC)

Kako je hb ha i hc ha , imamo da je
r+

ha
[S(OBC) + S(OCA) + S(OAB)],
S(ABC)

i prema tome r + ha . Znak jednakosti vazice samo u slucaju kada je trougao


ABC jednakostranican. (sl.540)

628

B
C

slika 540
1002. Ako su a, b, c stranice netupouglog trougla pri cemu je a b i a c
zatim r poluprecnik opisanog kruga i q poluprecnik upisanog kruga, dokazati
da je
r
(b + c)
h
.

a
Resenje: Prema prethodnom zadatku imamo da je r + hha , pa je
r hha =

S
S
S
(b + c)
2S
=
(2p a) =
(b + c) =
.
a
p
ap
ap
a

Otuda je
r
(b + c)
h
.

a
Znak jednakosti vazice samo u slucaju kada je trougao jednakostranican.
1003. Ako su ha , hb , hc visine trougla ABC, dokazati da je
l
l
l
l
l

<
<
+ .
ha
hb
hc
ha
hb
Resenje: Ako sa a, b, c obelezimo stranice trougla ABC, prema poznatom stavu
bice aha = bhb = chc , odakle se deljenjem sa ha hb dobija da je
a : hb = b : ha = c : l

629

ha hb
. Neka je C tacka stranice AC takva da je AC = ha , a B
hc
tacka u kojoj prava kroz C uporedna sa BC sece AB. Iz slicnosti trouglova
ABC i AB C sledi da je
gde je l =

BC : B C = CA : C A = AB : AB ,
tj.
a : B C = b : ha = C : AB .
Iz prethodnih jednakosti sledi da je
B C = hb
i
AB = l.
Kod trougla AB C je
AC B C < AB < AC + B C ,
pa je
ha hb < l < ha + hb .

Stavljajuci da je

l=

ha hb
hc

i deljenjem sa ha hb nalazimo da je
l
l
l
l
l

<
<
+ .
ha
hb
hc
ha
hb

C
B

C
B

slika 541
630

1004. Ako je p poluobim trougla ABC, r poluprecnik opisanog kruga, q poluprecnik upisanog kruga i qa poluprecnik spolja upisanog kruga koji dodiruje
stranicu BC, dokazati da je
(a) p > 2r , ako je trougao ABC ostrougli,
(b) p > 4r a , ako je kod trougla ABC ugao A tup.
Resenje:
(a) Ako je O srediste opisanog kruga i A1 srediste stranice BC, bice
BA1 + A1 O > OB, pa je
a
a
+ (r
) > r,
2
2
i prema tome
a > a .
Isto tako je
b > b
i
c > c .
Otuda je
a + b + c > a + b + c 3
tj.
2p > 4r + 3
i prema tome je
p > 2r .
(b) Ako je ugao A trougla ABC tup, bice
a
a
+ (r
) > r,
2
2
tj.
a > b + c .
Pri tome je
b > b
i
c > c .
Otuda je
a + b + c > 2b + c
tj.
2p 2(4r a )
i prema tome je
p > 4r a .
1005. Ako je p poluobim trougla ABC i M bilo koja njegova unutrasnja
tacka, dokazati da je
p < M A + M B + M C < 2p.
631

Resenje se dobija slicno kao u prethodnim zadacima.


1006. Ako obelezimo sa H ortocentar, O srediste upisanog kruga i S srediste
opisanog kruga trougla, dokazati da je
OH < 3r.
Resenje: Teziste T trougla ABC nalazi se u tom trouglu, dakle i u krugu l(O, r)
koji je opisan oko tog trougla. Stoga je OT < r. Medutim, prema Ojlerovoj
teoremi imamo da je OH = 3OT , pa je OH < 3r.
1007. Ako je H ortocentar, O srediste upisanog kruga i S srediste opisanog
kruga trougla, dokazati da je

OH 2SH.
Resenje: Ako obelezimo sa p poluobim, sa r poluprecnik opisanog kruga i sa
poluprecnik upisanog kruga trougla, prema zadacima 309 i 311 imamo da je
OH 2 = 9r2 2p2 + 22 + 8r
i
SH 2 = 4r2 + 4r + 32 p2 ,
pa je
OH 2 2SH 2 = r2 42 42 42 = 0.
Otuda je
OH 2 SH 2 ,
i prema tome je
OH

2SH

1008. Dat je konveksan ugao M ON i u njemu tacka P . Odrediti na kracima


OM i ON ugla M ON tacke X i Y kolinearne s tackom P da obim trougla
OXY bude minimalan.
Resenje: Vidi zadatak 1711.
1009. Ako su a, b, c stranice i p poluobim trougla, dokazati da je
(a) 8(p a)(p b)(p c) 6 abc,
(b) a2 (p a) + b2 (p b) + c2 (p c) 6

3
abc,
2

(c) ab(a + b) + bc(b + c) + ca(c + a) > 6abc,


(d) ab(a + b) + bc(b + c) + ca(c + a) > 48(p a)(p b)(p c).
632

Resenje:
(a) Ako obelezimo sa r poluprecnik opisanog kruga i sa poluprecnik upisanog
kruga trougla,bice
abc
.
(p a)(p b)(p c) =
4r
Otuda i iz jednakosti
r > 2
sledi da je
8(p a)(p b)(p c) 6 abc.
(b) S obzirom da je
(a b)2 > 0,
(b c)2 > 0,
(c a)2 > 0

i
a + b c > 0,
b + c a > 0,
c + a b > 0,
imamo da je
(a b)2 (a + b c) > 0,
(b c)2 (b + c a) > 0,

(c a)2 (c + a b) > 0.
Sabiranjem odgovarajucih strana ovih nejednakosti nalazimo da je
6abc 2a2 (b + c a) 2b2 (c + a b) 2c2 (a + b c) > 0,
pa je
a2 (p a) + b2 (p b) + c2 (p c) 6
(c) S obzirom da je

3
abc.
2

a + b > 2 ab,

b + c > 2 bc,

c + a > 2 ca,

bice

ab(a + b) + bc(b + c) + ca(c + a) > 2[(ab) 2 + (bc) 2 + (ca) 2 ].


Prema nejednakosti za aritmeticku i geometrijsku sredinu, imamo da je
3

(ab) 2 + (bc) 2 + (ca) 2 > 3abc,


pa je
ab(a + b) + bc(b + c) + ca(c + a) > 6abc.
(d) Prema prethodnim delovima ovog zadatka imamo da je
ab(a + b) + bc(b + c) + ca(c + a) > 6abc
633

i
abc > 8(p a)(p b)(p c),

pa je

ab(a + b) + bc(b + c) + ca(c + a) > 48(p a)(p b)(p c).


1010. Ako su a, b, c stranice i p poluobim trougla, dokazati da je
1
1
9
1
+
+
>
,
a+b b+c c+a
4p
b
c
3
a
+
+
> ,
(b)
b+c c+a a+b
2
pa
pb pc
3
(c)
=
+
> ,
b+c
c+a a+b
4
p+a p+b p+c
15
(d)
+
+
>
.
b+c
c+a a+b
4
(a)

Resenje:
(a) Prema nejednakosti za harmonijsku i aritmeticku sredinu imamo da je
(
pa je

1
1
4p
1
+
+
)6
,
a+b b+c c+a
9
1
1
1
9
+
+
>
.
a+b b+c c+a
4p

(b) S obzirom da je
a
b
c
1
1
1
+
+
= 2p(
+
+
)3
b+c c+a a+b
b+c c+a a+b
i

1
1
1
9
+
+
>
b+c c+a a+b
4p

bice

a
b
c
3
+
+
> .
b+c c+a a+b
2

(c) S obzirom da je
1
a
b
c
pa pb pc
+
+
= [3 (
+
+
)]
b+c
c+a a+b
2
b+c c+a a+b
i

b
c
3
a
+
+
>
b+c c+a a+b
2

bice

pa
pb pc
3
=
+
> .
b+c
c+a a+b
4

(d) S obzirom da je
p+a p+b p+c
3
a
b
c
+
+
= (1 +
+
+
)
b+c
c+a a+b
2
b+c c+a a+b
634

b
c
3
a
+
+
> ,
b+c c+a a+b
2

bice

15
p+a p+b p+c
+
+
>
.
b+c
c+a a+b
4

1011. Ako su a, b, c stranice trougla, p njegov poluobim, r poluprecnik opisanog


kruga i poluprecnik upisanog kruga, dokazati da je
1
1
1 1 1
1
+
+
> 2( + + ),
pa pb pc
a b
c
1
1
1
9
(b)
+
+
> ,
pa pb pc
p

1
1
1
3
(c)
+
+
>
,
pa pb pc

1
1
2 3
1
+
+
>
.
(d)
pa pb pc
r
(a)

Resenje:
(a) Prema nejednakosti za aritmeticku i geometrijsku sredinu, imamo da je
1
1
4
+
>
,
pa pb
2p (a + b)
1
4
1
+
>
,
pb pc
2p (b + c)

1
4
1
+
>
.
pc pa
2p (c + a)

Sabiranjem odgovarajucih strana ovih triju nejednakosti, dobijamo da je


1
1
1
1 1 1
+
+
> 2( + + ).
pa pb pc
a b
c
(b) Prema nejednakosti za harmonijsku i aritmeticku sredinu, imamo da je
(

1
1
1 1
(p a) + (p b) + (p c)
+
+
) 6
,
pa pb pc
9

pa je

1
1
1
9
+
+
> .
pa pb pc
p

(c) Kako je

A
B
C

+
+
> tan + tan + tan ,
pa pb pc
2
2
2
a prema Jensenovoj nejednakosti imamo da je
tan

A
B
C
A+B+C
+ tan + tan > 3 tan
= 3,
2
2
2
6
635

bice

(d) Iz nejednakosti

1
1
3
1
+
+
>
.
pa pb pc

1
1
1
3
+
+
>
pa pb pc

i
6
sledi da je

r
2

1
1
1
2 3
+
+
>
.
pa pb pc
r

1012. Ako su a, b, c stranice trougla, dokazati da je


(a) a2 + b2 + c2 > ab + bc + ca,
1
(b) a2 + b2 + c2 > (a + b + c)2 ,
3
(c) a2 + b2 + c2 < 2(ab + bc + ca),
1
(d) a2 + b2 + c2 < (a + b + c)2 .
2
Resenje:
(a) S obzirom da je

a + b > 2 ab,

b + c > 2 bc,

c + a > 2 ca,

bice
a2 + b2 > 2ab,
b2 + c2 > 2bc,
c2 + a2 > 2ca.
Sabiranjem odgovarajucih strana ovih triju nejednakosti nalazimo da je
a2 + b2 + c2 > ab + bc + ca.
(b) S obzirom da je
(a + b + c)2 = a2 + b2 + c2 + 2(ab + bc + ca)
i
ab + bc + ca 6 a2 + b2 + c2 ,
bice
(a + b + c)2 6 3(a2 + b2 + c2 )
i prema tome
a2 + b 2 + c2 >

1
(a + b + c)2 .
3

636

(c) Iz nejednakosti
b c < a,
ca<b
i
ab<c
nalazimo da je
b2 + c2 a2 < 2bc,
c2 + a2 b2 < 2ca

i
a2 + b2 c2 < 2ab.
Sabiranjem odgovarajucih strana ovih triju nejednakosti dobijamo da je
a2 + b2 + c2 < 2(ab + bc + ca).
(d) Iz jednakosti
(a + b + c)2 = a2 + b2 + c2 + 2(ab + bc + ca)
i nejednakosti
2(ab + bc + ca) > a2 + b2 + c2 ,
sledi da je
(a + b + c)2 > 2(a2 + b2 + c2 ),
pa je

1
(a + b + c)2 .
2
1013. Ako su a, b, c stranice trougla, dokazati da je (a) ab + bc + ca
1
1
(a + b + c)2 ; (b) ab + bc + ca > (a + b + c)2 .
3
4
a2 + b 2 + c2 <

Uputstvo.
(a) Iz ab + bc + ca a2 + b2 + c2 = (a + b + c)2 2(ab + bc + ca) sledi da je
1
ab + bc + ca < (a + b + c)2 .
3
(b) Iz (a + b + c)2 = a2 + b2 + c2 = (a + b + c)2 + 2(ab + bc + ca) i
a2 + b2 + c2 < 2(ab + bc + ca) sledi da je (a + b + c)2 < 4(ab + bc + ca), i prema
1
tome ab + bc + ca > (a + b + c)2 .
4
1014. Ako je p poluobim trougla, r poluprecnik opisanog
kruga i polu

3
3 3
precnik upisanog kruga, dokazati da je (a) 3 3 p
(4r + )
r;
3
2
3r
27
r 3(4r + ) 16r 52 p2 4r2 + 4r + 32 (4r + ).
(b)
2
2
Uputstvo.

(a) Ako obelezimo sa a, b, c stranice trougla, prema


nejednakosti za aritmeticku i geometrijsku sredinu, imamo da je a+b+c 3 3 abc, pa je 8p3 27abc
637

. Otuda i iz jednakosti abc = 4rp (v.z. 264) nalazimo


da je 2p2 27r, pa se

primenom nejednakosti r 2 dobija da je p 3 3 .


3
(4r + ), obelezimo
Da bismo dokazali drugu nejednakost iz (a),tj. da je p
3
sa a , b , c poluprecnike spolja
upisanih
krugova.
Prema
zadacima
1021. i154.

imamo da je a +b + c p 3 i a + b + c = 4r + , pa je 4r + p 3, i
3
prema tome p
(4r + ) . Poslednja nejednakost iz (a) sledi neposredno iz
3
prethodne uzimajuci u obzir da je r 2 .
(b) Dovoljno je dokazati da je 16r 52 p2 4r2 + 4r + 32 , ostale
nejednakosti sleduju neposredno iz svih uzimajuci u obzir da je r 2 .
Da bismo dokazali da je 16r 52 p2 , obelezimo sa T teziste trougla i sa S
srediste upisanog kruga. Prema zadatku 310. imamo da je
9T S 2 = p2 + 52 16r, pa je p2 + 52 16r 0, i prema tome
p2 16r 52 .
Da bismo dokazali da je p2 4r2 + 4r + 32 , obelezimo sa H ortocentar
i sa S srediste upisanog kruga trougla. Prema zadatku 311. imamo da je
HS 2 = 4r2 + 4r + 32 p2 , pa je 4r2 + 4r + 32 p2 0, i prema tome
p2 4r2 + 4r + 32 .
1015. Ako su a, b, c stranice i p poluobim trougla, a r poluprecnik opisa9
1 1 1
;
nog kruga i poluprecnik upisanog kruga, dokazati da je (a) + +
a b
c
2p

1 1 1
3
3
1 1 1
(b) + +
; (c) + +
.
a b
c
r
a
b
c
2
Uputstvo.

(a) Prema nejednakosti za harmonijsku i aritmeticku sredinu, imamo da je


1 1 1 1
1
1 1 1
9
3( + + ) (a + b + c), pa je + +
.
a b
c
3
a b
c
2p

1
1
9
1
+ +

i 2p 3 3r, sledi da je
(b) Iz izvedenih nejednakosti
a
b
c
2p

1 1 1
3
+ +
.
a b
c
r
1
1 1 1
(ab + bc + ca), a prema zadacima 264. i 1013.
(c) Kako je + + =
a
b
c
abc
4
1 1 1
p
abc = 4pr i ab + bc + ca p2 , bice + +
. Otuda i iz nejednakosti
3
a
b
c
3r

1 1 1
3
3 3
r sledi da je + +
.
p
2
a b
c
2
1016. Ako su a, b, c stranice trougla, r poluprecnik opisanog kruga i
poluprecnik upisanog kruga, dokazati da je (a) ab + bc + ca 9r2 ; (b)
ab + bc + ca 18r; (c) ab + bc + ca 362 ; (d) ab + bc + ca 4(r + )2 ;
(e) ab + bc + ca 4(5r ); (f ) ab + bc + ca 4(4r + ).
Uputstvo.

638

(a) Prema zadacima 1013. i 1014. ab + bc + ca

ab + bc + ca 9r2 .

4 2
27 2
p i p2
r , pa je
3
4

1 1 1
9
+ +
i abc = 4pr.
a b c
2p
Mnozenjem odgovarajucih strana nalazimo da je ab + bc + ca 18r .
(b) Prema zadacima 1015. i 264, imamo da je

(c) Iz nejednakosti ab+bc+ca 18r i r 2 sledi da je ab+bc+ca 362 .


(d) Prema zadacima 263. i 1014, imamo da je ab + bc + ca = p2 + 2 + 4r i
p 4r2 + 4r + 32 , pa je ab + bc + ca 4(r + )2 .
2

(e) Prema zadacima 263. i 1014, imamo da je ab + bc + ca = p2 + 2 + 4r i


p 16r 52 pa je ab + bc + ca 4(5r ) .
2

(f) Prema zadacima 263. i 1014, imamo da je ab + bc + ca = p2 + 2 + 4r i


p 3(4r + ), pa je ab + bc + ca 4(4r + ) .
2

1017. Ako su a, b, c stranice trougla, r poluprecnik opisanog kruga i


poluprecnik upisanog kruga, dokazati da je (a) a2 + b2 + c2 362 ; (b)
a2 + b2 + c2 18r; (c) a2 + b2 + c2 9r2 ; (d) a2 + b2 + c2 4(2r2 + 2 ); (e)
a2 + b2 + c2 12(2r ); (f ) a2 + b2 + c2 4(4r + ).
Uputstvo.
(a) Prema zadacima 1012. i 1016. imamo da je a2 + b2 + c2 ab + bc + ca i
ab + bc + ca 362 , pa je a2 + b2 + c2 362 .
(b) Prema zadacima 1012. i 1016. imamo da je a2 + b2 + c2 ab + bc + ca i
ab + bc + ca 18r, pa je a2 + b2 + c2 18r .
(c) Ako obelezimo sa T teziste trougla, sa O srediste opisanog kruga i sa d
duz odredenu tackama O i T , prema zadatku 308, imamo da je
1
1
d2 = r2 (a2 + b2 + c2 ). Otuda je r2 (a2 + b2 + c2 ) 0, i prema tome
9
9
a2 + b2 + c2 9r2 .
(d) Prema zadacima 263. i 1014, imamo da je a2 + b2 + c2 = 2p2 22 8r
i p 4r2 + 4r + 32 , pa je a2 + b2 + c2 4(2r2 + 2 ) .
2

(e) Prema zadacima 263. i 1014, imamo da je a2 + b2 + c2 = 2p2 22 8r


i p 16r 52 , pa je a2 + b2 + c2 12(2r ).
2

(f) Prema zadacima 1012. i 1014, imamo da je a2 + b2 + c2

4 2
p i
3

p2 3(4r + ), pa je a2 + b2 + c2 4(4r + ) .
1018. Ako su ha , hb , hc visine i a , b , c poluprecnici spolja upisanih

639

krugova trougla ABC, dokazati da je


a
b
c
+
+
3.
ha
hb
hc

Ako obelezimo sa p poluobim i sa S povrsinu trougaone povrsi (ABC), bice


a =

S
,
pa

b =

S
,
pb

c =

S
pc

2S
2S
2S
, hb =
, hc =
.
a
b
c
Primenom nejednakosti za aritmeticku i geometrijsku sredinu i nejednakosti iz
zadatka 1009. dobija se
s
b
c
1 a
b
c
abc
a
+
+
= (
+
+
)33
3.
ha
hb
hc
2 pa pb pc
S(p a)(p b)(p c)
ha =

1019. Ako su ha , hb , hc visine trougla, p njegov poluobim, r poluprecnik opisanog kruga i poluprecnik upisanog kruga, dokazati da je : (a) ha + hb + hc

9
9; (b) ha + hb + hc 3p; (c) ha + hb + hc r; (d) ha + hb + hc 4r + ;
2
(e) ha + hb + hc 3(r + ); (f ) ha + hb + hc 2r + 5.
Uputstvo.

(a) Prema zadacima 256. i 1015. imamo da je


1 1 1
ha + hb + hc = 2p( + + )
a b
c

1 1 1
9
+ +
,
a b
c
2p

pa je ha + hb + hc 9.
Drugi nacin. Kako je
a+b+c a+b+c a+b+c
1 1 1
+
+
)=
ha + hb + hc = 2p( + + ) = (
a b
c
a
b
c
b c
a c
+ +1+ + +1+
a a
b b
Iz nejednakosti
a
b
+ 2,
b
a
bice ha + hb + hc 9.

= (1 +

a b
a b
b c
c a
+ ) = [3 + ( + ) + ( + ) + ( + )].
c c
b a
c b
a c
b c
+ 2,
c b

c
a
+ 2
a
c

Treci nacin. Prema nejednakosti za aritmeticku i harmonijsku sredinu nalazimo da je


1
1
1
+
+ )1
ha + hb + hc 9(
ha
hb
hc
640

Iz jednakosti

1
1
1
1
+
+
=
ha
hb
hc

sledi da je ha + hb + hc 9.
(b) Posto je
1 1 1
ha + hb + hc = 2p( + + ) i
a b
c

bice ha + hb + hc 3p.

1 1 1
3
+ +
,
a b
c
2

Drugi nacin. Kako je


ha + hb + hc =

1
4
(ab + bc + ca) i ab + bc + ca p2 ,
2r
3

bice
ha + hb + hc
Iz nejednakosti

2p2
.
3r

3 3
p
r
2

sledi da je ha + hb + hc 3p.
(c) Posto je
ha + hb + hc =

1
(ab + bc + ca)
2r

i ab + bc + ca 9r2 ,

bice ha + hb + hc 29 r.
(d) Iz
ha + hb + hc =

1
4
(ab + bc + ca) i ab + bc + ca p2 ,
2r
3

sledi da je
ha + hb + hc

2p2
.
3r

Posto je

3r
(4r + )
2
nalazimo da je ha + hb + hc 4r + .
p2

Drugu nacin. Prema zadacima 988. i 154. imamo da je


h a + h b + h c a + b + c

i a + b + c = 4r + ,

pa je ha + hb + hc 4r + .
(e) Iz
ha + hb + hc =

1
(ab + bc + ca) i ab + bc + ca 4(r + )2
2r

641

sledi da je
ha + hb + hc
Posto je

2
(r + )2 .
r

r
nalazimo da je
2

ha + hb + hc

2
2 3r
(r + )(r + )
(r + ) = 3(r + ).
r
r 2

(f) Prema zadacima 256. i 263. imamo da je


ha + hb + hc =

1
(ab + bc + ca) i ab + bc + ca = p2 + 2 + 4r
2r

pa je

1 2
(p + 2 + 4r).
2r
Ako sa d obelezimo duz koja spaja ortocentar trougla sa sredistem upisanog
kruga, prema zadatku 311 imamo da je p2 = 4r2 + 4r + 32 d2 . Stoga je
ha + hb + hc =

ha + hb + hc =

1
(4r2 + 8r + 42 d2 ),
2r

i prema tome

1
(4r2 + 8r + 42 ).
2r
Primenom nejednakosti 2 r, dobijamo da je
ha + hb + hc

ha + hb + hc

1
(4r2 + 10r),
2r

i prema tome ha + hb + hc 2r + 5.
Nejednakost 1019(f) predstavlja uopstenje nejednakosti 1019(b), 1019(d) i
1019(e). Primenom nejednakosti 2 r mogu se iz 1019(f) dobiti 1019(c),
1019(d) i 1019(e).
Uputstvo.

1020. Ako su la , lb , lc simetrale unutrasnjih uglova trougla, p njegov poluobim, r poluprecnik opisanog kruga i poluprecnik upisanog kruga, dokazati

9
da je (a) la + lb + lc 3p; (b) la + lb + lc r; (c) la + lb + lc 9; (d)
2
la + lb + lc 4r + ; (e) la + lb + lc 3(r + ).
(a) Posto je

bice la

2 bc p
la =
p(p a) i 2 bc b + c,
b+c

p
p(p a). Otuda je

1
3a
3la 3p(p a) [p + 3(p a)] = 2p ,
2
2
642

i prema tome

3
(4p 3a).
la
6

Isto tako je

3
3
(4p 3b) i lc
(4p 3c)
6
6
Sabiranjem odgovaraju
cih strana dobijenih triju nejednakosti, nalazimo da je

la + lb + lc p 3.

3 3
9
r sledi da je la + lb + lc r.
(b) Iz nejednakosti la + lb + lc p 3 i p
2
2
(c) Iz nejednakosti la + lb + lc ha + hb + hc i ha + hb + hc 9, sledi da
je la + lb + lc 9.

3
(d) Iz nejednakosti la +lb +lc p 3 i p
(4r +) sledi da je la +lb +lc
3
4r + .

(e) Iz nejednakosti la +lb +lc p 3 i p 3(r+) sledi la +lb +lc 3(r+).


Uputstvo.
lb

1021. Ako su a , b , c poluprecnici spolja upisanih krugova, r i poluprecnici opisanog i


upisanog kruga i p poluobim trougla ABC, dokazati da je
(a) a + b + c 3p; (b) a + b + c 9; (v) a + b + c 3(r + ); (g)
9
a + b + c r.
2
Uputstvo.
Prema zadacima 260. i 154. imamo da je a b + b c + c a = p2 i a + b +
c = 4r . Kako za ma koje tri duzi x, y, z vazi nejednakost x2 + y 2 + z 2
xy + yz + zx, jer je
x2 + y 2 + z 2 (xy + yz + zx) =

1
[(x y)2 + (y z)2 + (z x)2 ] 0,
2

bice
p 2 = a b + b c + c a =

1
[(a b + b c + c a ) + 2(a b + b c + c a )]
3

1 2
1
[a + 2b + 2c + 2(a b + b c + c a )] = (a + b + c )2 .
3
3

Otuda je a + b + c 3p
(b) Prema zadacima 154. i 998. imamo da je

a + b + c = 4r + ,

i r 2,

pa je a + b + c 9.
(c) Prema zadacima 154. i 998. imamo da je
a + b + c = 4r +
pa je a + b + c 3(r + ).
643

i r 2,

(d) Prema zadacima 154. i 988. imamao da je


a + b + c = 4r +

i r

r
,
2

9
pa je a + b + c r
2
1022. Ako su ma , mb , mc tezisne linije, r i poluprecnici opisanog i upisanog
3
kruga i p poluobim trougla ABC, dokazati da je (a) p < ma + mb + mc < 2p;
2
9
(b) 9 ma + mb + mc r.
2
Uputstvo.

Pod (a) i (b) se resava koristeci nejednakost ha + hb + hc la + lb + lc


ma + mb + mc .
1023.Ako su ha , hb , hc visine, la , lb , lc simetrale uglova, ma , mb , mc tezisne
linije, a , b , c poluprecnici spolja upisanih krugova trougla ABC, dokazati
da je (a) ha + hb + hc la + lb + lc ma + mb + mc ; (b) la + lb + lc

b c + c a + a b a + b + c ;
Uputstvo.

(a) Koristiti relacije ha la ma , hb lb mb , hc lc mc .


(b) Primenom Heronovog obrasca nalazimo da

l a b c , l b c a , l c a b ,
pa je
la + lb + lc

a b + b c + c a .

1024. Ako su a, b, c stranice i p poluobim trougla, a r poluprecnik opisanog i

8 3
poluprecnik upisanog kruga, dokazati da je (a) abc
p ; (b) abc 3 3r3 ;
27

3
2
2
;
(d)
abc

8p
;
(e)
abc

2pr
;
(f
) abc 6 3r2 ; (g)
(c) abc
24
3
2
abc 12 3r ;
Uputstvo.

(a) Prema nejednakosti za geometrijsku i aritmeticku sredinu imamo da je

1
3
abc (a + b + c),pa je

8 3
abc
p .
27

r
3 3
(b) Iz nejednakosti abc = 4rp i nejednakosti i p
r sledi da je
2
2
3
abc 3 3r .
644


(c) Izjednakosti abc = 4rp i nejednakosti p 3 3 i r 2 sledi da je
abc 24 33 .
(d) Iz abc = 4pr i r 2, sledi da je
abc 8p2 .
(e) Iz abc = 4pr i 2 r sledi da je
abc 2pr2 .

(f) Iz
= 4pr i 2p 3 3r, sledi da je
abc
abc 6 3r2 .

(g) Izabc = 4pr i p 3 3, sledi da je


abc 12 3r2

1025.Ako su a , b , c poluprecnici spolja upisanih krugova, poluprecnik


upisanog kruga, r poluprecnik opisanog kruga i p poluobim tog trougla, doka27 2
27 2
r ; (c) a b c
r ; (d)
zati da je (a) a b c 273 ; (b) a b c
2
4

3 3
27 3
1
a b c
r ; (e) a b c
p ; (f ) a b c
(4r + )3 .
8
9
27
Uputstvo.
(a) Prema zadacima 261 i 1014, imamo da je a b c = p2 i p2 272 , pa

je
a b c 273 .
Drugi nacin. Prema nejednakosti za geometrijsku iharmonijsku sredinu, imamo
da je
1
1
1

3
a b c 3( +
+ )1 .
a
b
c
Otuda i iz jednakosti
a b c 273 .

1
1
1
1
+
+
= sledi da je
a
b
c

27
rp, pa je
(b) Prema zadacima 261 i 1014, imamo da je a b c = p2 i p2
2
27 2
r .
a b c
2
27 2
(c) Prema zadacima 261 i 1014, imamo da je a b c = p2 i p2
r , pa je
4
27 2
a b c
r .
4
(d) Prema zadatku 261, imamo da je a b c = p2 . Otuda i iz nejednakosti
27 2
r
p2
r i sledi da je
4
2
27 3
a b c
r .
8
Drugi nacin. Prema nejednakosti za geometrijsku i aritmeticku sredinu
9r

imamo da je 3 3 a b c a + b + c . Otuda i iz nejednakosti a + b + c


2
645

sledi da je
27 3
a b c
r .
8

3
(e) Prema zadacima 261 i 1014, imamo da je a b c = p i
p, pa je
9

3 3
p .
a b c
9
2

(f) Prema nejednakosti za geometrijsku i aritmeticku sredinu, imamo da je

3 3 a b c a + b + c .
Otuda i iz jednakosti a + b + c = 4r + sledi da je
1
(4r + )3 .
a b c
27
1026.Ako su ha , hb , hc visine trougla, p njegov poluobim, r poluprecnik
opisanog kruga i poluprecnik upisanog kruga, dokazati da je : (a) ha hb hc

27 3
3 3
p ; (b) ha hb hc
r ; (c) ha hb hc 273 ; (d) ha hb hc p2 ; (e) ha hb hc
9
8
27 2
27 2
r ; (f ) ha hb hc
r .
2
4
Uputstvo.

da je

(a) Prema nejednakosti za geometrijsku i aritmeticku sredinu, imamo


3 3 ha hb hc ha + hb + hc . Otuda i iz nejednakosti ha + hb + hc p 3 sledi da
je

3 3
ha hb hc
p .
9
(b) Prema nejednakosti za geometrijsku i aritmeticku sredinu, imamo da je

9
3
3 ha hb hc ha + hb + hc . Otuda i iz nejednakosti ha + hb + hc r, sledi da je
2
27 3
ha hb hc
r
8

a2 b 2 c2
i abc 3 3r2 , sledi da je
Drugi nacin. Iz ha hb hc =
3
8r
27 3
ha hb hc
r .
8
(c) Prema nejednakosti za geometrijsku i harmonijsku sredinu, imamo da je
p
1
1
1
3
ha hb hc 3(
+
+ )1
ha
hb
hc

1
1
1
1
Otuda i iz nejednakosti
+
+
= sledi da je 3 ha hb hc 3 , pa je
ha
hb
hc

ha hb hc 273 .
Drugi nacin. Iz ha hb hc =
ha hb hc 273 .

8p3 3
8 3
i abc
p , sledi da je
abc
27

646

(d) Iz ha hb hc =
ha hb hc p2 .
(e) Iz ha hb hc =
27 2
ha hb hc
r .
2

2p2 2
i 2 r, sledi da je
r

2p2 2
3 3
ip
r, sledi da je
r
2

2p2 2
(f) Iz ha hb hc =
i 2 r, sledi da je ha hb hc p2 . Otuda i iz
r

3 3
r, nalazimo da je
p
2
27 2
ha hb hc
r .
4
1033. Ako su ha , hb , hc visine trougla, a p njegov poluobim, r poluprecnik
opisanog kruga i poluprecnik upisanog kruga, dokazati da je: (a) ha hb +hb hc +
27
27 2
hc ha 272 (b) ha hb + hb hc + hc ha
r (c) ha hb + hb hc + hc ha
r (d)
2
4
2
ha hb + hb hc + hc ha p .
Uputstvo.

(a) Prema zadacima 258. (b) i 1026. (c), imamo da je


1
1
1
1
+
+
=
i ha hb hc 273 .
ha
hb
hc

Mnozenjem odgovarajucih strana nalazimo da je


ha hb + hb hc + hc ha 272 .
(b) Prema zadacima 258. (b) i 1026. (e), imamo da je
1
1
1
1
27 2
+
+
=
i ha hb hc
r .
ha
hb
hc

2
Mnozenjem odgovarajucih strana nalazimo da je
ha hb + hb hc + hc ha

27
r.
2

(c) Prema zadacima 258. (b) i 1026. (f), imamo da je


1
1
1
1
27 2
+
+
=
i ha hb hc
r .
ha
hb
hc

4
Mnozenjem odgovarajucih strana nalazimo da je
ha hb + hb hc + hc ha

27 2
r .
4

(d) Prema zadacima 258. (b) i 1026. (d), imamo da je


1
1
1
1
+
+
=
i ha hb hc p2 .
ha
hb
hc

647

Mnozenjem odgovarajucih strana nalazimo da je


ha hb + hb hc + hc ha p 2 .
1034.Ako su a, b, c stranice, ha , hb , hc visine i a , b , c poluprecnici spolja
upisanih krugova, dokazati da je: (a) ha hb + hb hc + hc ha a b + b c + c a
3
3
(b) ha hb +hb hc +hc ha (a2 +b2 +c2 ) (c) ha hb +hb hc +hc ha (ab+bc+ca).
4
4
Uputstvo.

(a) Prema zadacima 1033. (d) i 260. (d), imamo da je


h a h b + h b h c + h c h a p 2 i a b + b c + c a = p 2 ,
pa je
h a h b + h b h c + h c h a a b + b c + c a .
(b) Prema zadacima 1033. (d) i 1012. (b), imamo da je
ha hb + hb hc + hc ha p 2 i p 2

3 2
(a + b2 + c2 ),
4

pa je

3 2
(a + b2 + c2 ).
4
(c) Prema zadacima 1033. (b) i 1016. (b), imamo da je
ha hb + hb hc + hc ha

ha hb + hb hc + hc ha

27
27
3
r i
r (ab + bc + ca),
2
2
4

pa je

3
(ab + bc + ca).
4
1035. Ako su ha , hb , hc visine, la , lb , lc simetrale uglova i ma , mb , mc tezisne
1
1
1
1
1
1
+
+

+
+

linije trougla ABC, dokazati da je: (a)


ha
hb
hc
la
lb
lc
1
1
1
+
+
(b) h2a + h2b + h2c la2 + lb2 + lc2 m2a + m2b + m2c
ma
mb
mc
Uputstvo.
ha hb + hb hc + hc ha

(a) Na osnovu zadatka 988., imamo da je ha la ma , pa vazi

1
1

ma
la

1
. Kako tvrdenje zadatka 988. vazi za svaku stranicu trougla, tj. za stranice
ha
1
1
1
1
1
1

. Sabiranjem odgovarajucih
b i c, dobijamo
mb
lb
h b mc
lc
hc
strana poslednje tri nejednacine, dobijamo
1
1
1
1
1
1
1
1
1
+
+

+ +
+
+
.
ha
hb
hc
la
lb
lc
ma
mb
mc
(b) Na osnovu zadatka 988., imamo da je ha la ma , pa vazi hb lb mb
648

i hc lc mc . Kvadriranjem poslednje tri nejednacine i sabiranjem odgovarajucih strana, dobijamo


h2a + h2b + h2c la2 + lb2 + lc2 m2a + m2b + m2c .
1036.Ako su ma , mb , mc tezisne linije i r poluprecnik kruga opisanog oko
trougla ABC, dokazati da je:
1
1
2
1
+
+
> .
ma
mb
mc
r
Uputstvo.
Neka je A tacka preseka stranice BC i prave AO. Jedan od uglova BA1 A i
CA1 A je tup, jer se tacke A i A , u slucaju da je jedan od njih prav, poklapaju,
1
1
1
2
pa vazi
+
+
= . Neka je CA1 A tup (analogno i za BA1 A).
ma
mb
mc
r
Onda AA A1 mora biti ostar, pa, kako je naspram veceg ugla veca stranica,
1
1
<
. Istim postupkom dobijamo da
imamo da je AA1 < AA , odnosno
AA
ma
1
1
1
1
<
i
<
. Sabiranjem odgovarajucih strana i na osnovu
vazi

BB
mb CC
mc
zadatka 942., dobijamo
1
1
1
2
+
+
> .
ma
mb
mc
r
1037.Ako je P srediste a p bilo koja druga tacka stranice BC trougla ABC,
zatim M i N podnozja upravnih iz P , a M i N podnozja upravnih iz P na
pravama AB i AC, dokazati da je:
P M P N > P M P N .
Uputstvo.
Ako su BB i CC visine iz temena B i C, bice M P B C CB i
N CP B CB, pa je P M : CC = BP : BC i P N : BB = P C : BC.
Otuda je
BB
CC
BP i P N =
P C,
PM =
BC
BC
i prema tome je
BB CC
BP P C.
PM PN =
BC 2
Na isti nacin se iz M P B C CB i N CP B CB dobija da je
P M P N =

BB CC
BP P C.
BC 2

Ako vazi raspored B P P C i kako je P C = BP , onda je BP > P C, pa,


ako pomnozimo poslednju nejednacinu sa P P i dodamo na obe strane BP P C,
a znamo da je P C = P P + P C i BP = BP + P P , dobijamo da je BP P C >
649

BP P C. Ako vazi raspored B P P C i kako je P C = BP , onda je


P C > BP , pa, ako pomnozimo poslednju nejednacinu sa P P i dodamo na obe
strane BP P C, a znamo da je BP = BP + P P i P C = P P + P C, dobijamo
da je BP P C > BP P C. Kako imamo da je BP P C > BP P C, onda je
P M P N > P M P N (sl. 542).
A

C
M
M

N
N

slika 542
1038. Ako je D podnozje visine iz temena A trougla ABC, a E tacka u
kojoj simetrala ugla A sece stranicu BC, dokazati da je:
|AC 2 AB 2 | 2|BC DE|.
Uputstvo.

Ako obelezimo sa A1 srediste stranice BC trougla ABC, primenom Apolonijeve teoreme, tj. prema zadatku 288. (a), imamo da je
1
AB 2 + AC 2 = 2AA21 + BC 2 .
2
A kako su ADA1 i ADB pravougli, imamo da je
1
AA21 = AD2 + DA21 = AB 2 BD2 + DA21 = AB 2 ( BC DA1 )2 + DA21 ,
2
pa je onda AC 2 AB 2 = 2BC DA1 . Prema zadatku 41., tacka E je izmedu
tacaka D i A1 ili se poklapa sa njima, te je DA1 DE. S toga:
1 ako je AC > AB, onda je AC 2 AB 2 > 2BC DE,
2 ako je AC = AB, onda je AC 2 AB 2 = 2BC DE,
3 ako je AC < AB, onda je AC 2 AB 2 < 2BC DE.
Dakle, u opstem slucaju bice |AC 2 AB 2 | 2|BC DE|.
1039.Ako su AB i CD dve tetive kruga k koje se seku u nekoj tacki S pod
pravim uglom i ako su O i r srediste i poluprecnik kruga k, a d duz odredena
tackama O i S, dokazati da je: (a) AB + CD 2r (b) AB CD 2(2r2 d2 ).
Uputstvo.
650

(a) Neka je AE precnik kruga k. S obzirom da su tetive BE i CD upravne


na pravoj AB, one su medu sobom uporedne ( paralelne ), pa je cetvorougao
CDBE jednakokraki trapez. S toga je BD = CE. Iz pravouglih trouglova
ASC i BSD nalazimo da je
AS 2 + CS 2 = AC 2 iBS 2 + DS 2 = BD2 = CE 2 .
Otuda je
AS 2 + BS 2 + CS 2 + DS 2 = AC 2 + CE 2 = AE 2 = 4r2 .
A kako znamo ( iz kvadrata binoma ) da je
(AS + BS + CS + DS)2 AS 2 + BS 2 + CS 2 + DS 2 ,
onda je (AB + CD)2 4r2 , i prema tome je AB + CD 2r. ( Iz AB 2r i
CD 2r, nalazimo da je i AB + CD 4r. )
1
(b) S obzirom da je AB CD (AB 2 + CD2 ), a, prema zadatku 236., vazi da
2
je AB 2 + CD2 = 4(2r2 d2 ), bice AB CD 2(2r2 d2 ).
1040. U ravni trougla ABC odrediti tacku X takvu da zbir duzi AX, BX,
CX bude minimalan
Uputstvo.

Najpre dokazimo da se trazena tacka X ne nalazi izvan trougla ABC. Ako


obelezimo sa Y bilo koju tacku izvan trougla ABC, ona pripada jednom od
unutrasnjih uglova trougla ABC ili jednom od uglova, koji su unakrsni sa njima.
Ako tacka Y pripada jednom od unutrasnjih uglova, npr. uglu A, ona je sa
one strane prave BC sa koje nije teme A, te duz AY sece duz BC u nekoj
tacki Y . Pri tome je BY + CY = BC < BY + CY i AY < AY , pa je
i AY + BY + CY < AY + BY + CY . Ako tacka Y pripada jednom od
uglova, koji je unakrsan sa nekim od uglova trougla ABC, npr. sa uglom A,
i ako teme tog ugla obelezimo sa Y , bice BY + CY < BY + CY , pa je i
AY + BY + CY < AY + BY + CY . Ovim smo dokazali da trazena tacka X
nije izvan trougla ABC.
Da bismo odredili polozaj tacke X, pretpostavimo da ugao C trougla ABC nije
najveci. Obelezimo sa Z promenljivu tacku trougaone povrsi (ABC), a sa ABB
i AZZ jednakostranicne trouglove, kojima su uglovi BAB i ZAZ istosmerni
sa uglom CAB. Pri tome je AZ = ZZ i BZ = Z B , pa je AZ + BZ + CZ =
CZ + ZZ + Z B . Iz istosmernosti konveksnih uglova CAB i BAB sleduje da
su tacke C i B sa raznih strana prave AB, te duz CB sece pravu AB u nekoj
tacki D. Razlikovacemo dva slucaja.
1 Ako se tacka D nalazi izmedu tacaka A i B, tada je CAB < 2R i CBB <
2R, gde je R prav ugao, pa je, s obzirom na jednakosti
CAB = CAB + BAB = CAB +

2R
,
3

CBB = CBA + ABB = CBA +

2R
,
3

651

4R
. U tom slucaju, na duzima CD i DB
3
postoje jedinstvene tacke X i X takve da je trougao AXX jednakostranican.
Pri tome je AXB = AXC = BXC. Sem toga je

svaki od uglova A i B manji od ugla

AX + BX + CX = XX + X B + CX = CB
i prema tome, za svaku tacku Z trougaone povrsi ABC razlicitu od tacke X
vazi
AX + BX + CX < AZ + BZ + CZ (slika543 (a)).
2 Ako tacka D nije izmedu tacaka A i B, ona se poklapa sa jednom od tih
dveju tacaka, ili se nalazi iza jedne od tih dveju tacaka u odnosu na drugu.
Ako je tacka D istovetna sa tackom A ili je iza tacke A u odnosu na B, bice
4R
. Ako zatim tacku A obelezimo sa X, tada za svaku tacku Z
BAC
3
trougaone povrsi ABC razlicitu od tacke X vazi
AX + BX + CX = CA + AB < CZ + ZZ + Z B = AZ + BZ + CZ
(slika 543 (b)).

A
B
X
Z
D
X
Z

slika 543(a)

Z
Z
B

slika 543(b)
652

1041. U ravni cetvorougla ABCD odrediti tacku X takvu da zbir duzi AX,
BX, CX, DX bude minimalan.
Uputstvo.

Ako je ABCD proizvoljan cetvorougao onda on moze biti prost ili mu se


stranice seku.Ako mu se stranice seku onda mozemo uociti cetvorougao koji je
prost i ima ista temena kao i pocetni cetvorougao.U oba slucaj zadatak se svodi
na zadatak 1042..
1042.U prostom cetvorouglu ABCD odrediti tacku X takvu da zbir duzi
AX, BX, CX, DX bude minimalan.
Uputstvo.

Neka je prost cetvorougao ABCD takav da je AB > CD(analogno se radi


ako je AB < CD), tada se prave AD i BC seku u nekoj tacki P koja je sa
iste strane prave AB kao i tacke C i D. Sada uocimo trougao ABP i na osnovi
zadatka 1040. mozemo naci tacku X takvu da je AX + BX + P X minimalan
zbir, a samim tim da je zbir AX + BX + CX + DX minimalan.
1043. Ako stranice AB, BC, CD, DA i dijagonale AC, BD cetvorougla
ABCD obelezimo respektivno sa a, b, c, d, e, f dokazati da je
e2 + f 2 b2 + d2 + 2ac.
Uputstvo.

Ako obelezimo sa M i N sredista stranica AD i BC cetvorougla ABCD,


1
prema zadacima 289. i 984. imamo da je M N 2 = (a2 + c2 b2 d2 + e2 + f 2 )
4
1
i M N 2 (a + c)2 , pa je
4
a2 + c2 b2 d2 + e2 + f 2 a2 + 2ac + c2 ,
i prema tome e2 + f 2 b2 + d2 + 2ac.
1044. Ako su a, b, c, d duzi jednake stranicama AB, BC, CD, DA i e, f
duzi jednake dijagonalama AC, BD proizvoljnog cetvorougla ABCD, dokazati
da je
(a + b + c + d)(e + f ) > 2(e2 + f 2 ).
Uputstvo.

S obzirom da je kod svakog trougla zbir dveju stranica veci od trece stranice,
imamo da je a + b > e, c + d > e, a + d > f , b + c > f . Sabiranjem odgovarajucih strana prvih dveju nejednakosti, zatim odgovarajucih strana ostalih
653

dveju nejednakosti, nalazimo da je a + b + c + d > 2e i a + b + c + d > 2f . Otuda


je je (a + b + c + d)e > 2e2 i (a + b + c + d)f > 2f 2 , i prema tome
(a + b + c + d)(e + f ) > 2(e2 + f 2 ).

1045.Ako su A, B, C, D cetiri proizvoljne tacke jedne ravni, dokazati da je


AC BD AB CD + BC AD.
Uputstvo.
Ako obelezimo sa C tacku takvu da je trougao DBC obrtno slican s
trouglom DAC , bice AC : AD = BC : BD pa je (slika 544 a)
AC =

BC AD
.
BD

S obzirom da je i DC C DAB, bice C C : CD = AB : BD pa je i


C C =

AB CD
.
BD

Iz ove dve jednakosti i iz nejednakosti AC AC + C C sledi da je AC BD


BC AD + AB CD. Znak jednakosti vazice samo ako je AC = AC + C C
(slika 544 b), a to biti samo kada je tacka C izmedu tacaka A i C. Pri tome
je AC D = BCD i DC C = DAB. Pritom su uglovi AC D i DC C
naporedni, dakle suplementni, pa su i uglovi BAD i BCD suplementni. Sem
toga su tacke A i C sa raznih strana prave BD, pa su tacke A, B, C, D na
izvesnom krugu l i pritom tacke A i C su na raznim lucima BD tog kruga(vidi
Ptolomejevu teoremu, zadatak 378.).
B
D

A
C
l
C

Slika 544a

Slika 544b

1046.Ako je M proizvoljna tacka ravni trougla ABC kome su stranice BC,


CA, AB srazmerne datim brojevima p, q, r, dokazati da je
pAM qBM + rCM.
654

Uputstvo.

Prema prethodnom zadatku je BC AM CA BM + AB CM , pa je s


obzirom na srazmeru BC : CA : AB = p : q : r, pAM qBM + rCM . Znak
jednakosti vazice samo u slucaju ako je tacka M na luku BC kruga l opisanog
oko trougla ABC na kom nije tacka A (slika 545).
A

C
B

Slika 545
1047. U ravni trougla ABC odrediti tacku X takvu da zbir
pAX + qBX + rCX,
gde su p, q, r dati pozitivni brojevi, bude minimalan.
Uputstvo.

Analizirajmo najpre slucaj kada je jedan od brojeva p, q, r nije manji od


zbira ostala dva broja; neka je npr. p q + r. U tom slucaju za svaku tacku X
ravni ABC imamo
pAX + qBX + rCX (q + r)AX + qBX + rCX = q(AX + BX) + r(AX + CX).
Medutim, kako je AX + BX AB i AX + CX AC, bice
pAX + qBX + rCX qAB + rAC.
Odatle sledi da je zbir pAX + qBX = rCX minimalan kada se tacka X poklapa
sa temenom A trougla ABC.
Analizirajmo sada slucaj kada je najveci od brojeva p, q, r manji od zbira ostala
dva broja. U tom slucaju postoji trougao A BC takav da je BC : CA : A B =
p : q : r. Pri tome je (slika 546) AA AM + A M gde znak jednakosti vazi
samo u slucaju kada tacka M pripada duzi AA . S druge strane prema zadatku
1046. je pA M qBM + rCM , gde znak jednakosti vazi samo u slucaju kada
je tacka M na luku BC kruga opisanog oko trougla A BC na kome nije tacka
A . Ako obe strane nejednakosti AA AM + A M pomnozimo sa p, tada na
osnovu nejednakosti pA M qBM + rCM vazi pAA pAM + qBM + rCM ,
gde znak jednakosti vazi samo u slucaju kada se tacka M poklapa sa tackom X
655

u kojoj duz AA sece luk BC kruga opisanog oko trougla A BC na kome nije
tacka A . Zbog toga je
pAX + qBX + rCX pAM + qBM + rCM.
U ekonomskoj teoriji se ovaj problem prestavlja u drugom obliku. Neka su A,
B, C tri punkta u kojima su otkrivena nalazista iste rude i neka su bogatstva
te rude u punktovima A, B, C proporcionalna s brojevima p, q, r. Gde treba
izgraditi fabriku za preradu te rude tako da transportni troskovi budu svedeni
na minimum.
M

Slika 546
1048. Ako obelezimo sa M proizvoljnu tacku iz unutrasnjosti trougla ABC,
sa xa , xb , xc rastojanja tacke M od temena A, B, C i sa ya , yb , yc odstojanja tacke M od pravih BC, CA, AB, dokazati da je (a) xa + xb + xc
1
1
1
1 1
1
1
2(ya + yb + yc ); (b)
+
+
( +
+ ); (c) xa xb xc 8ya yb yc ;
xa
xb
xc
2 ya
yb
yc
(d) xa xb xc (yb + yc )(yc + ya )(ya + yb ).

(a) Prvi nacin: Ako obelezimo sa M tacku simetricnu sa tackom M u od


nosu na simetralu ugla A, bice odstojanje tacke M od pravih AB i AC jednaka
duzini yb i yc . S obzirom da je tacka M u trouglu ABC, tacka M je u uglu

A, pa je tacka M u uglu A. Stoga prava AM sece stranicu BC u nekoj

tacki A . Ako zatim obelezimo sa B1 i C1 podnozja upravnih iz temena B i

C na pravoj AM , bice (sl.547) BC=BA +A M BB1 + CC1 . Mnozenjem


obeju strana ove nejednakosti sa xa , nalazimo da je BCxa BBxa + CC1 xa =

2S(M AB) + 2S(M AC) = Cyb + byc .

656

A
Q
B1
M

R
M
B

C
C1

Iz ove i analognim nejednakosti sledi da je


xa

c
b
a
c
b
c
yb + yc , xb yc + ya , xc ya + yb .
a
a
b
b
c
a

Sabiranjem odgovarajucih strana ovih triju nejednakosti nalazimo da je


c
a
a
b
b c
xa + xb + xc ( + )ya + ( + )yb + ( + )yc 2(ya + yb + yc )
c b
a
c
b
a
Drugi nacin. Ako obelezimo sa P, Q, R podnozja upravnih iz tacke M na pravama BC, CA, AB; sa P1 i P2 podnozja upravnih iz tacaka R i Q na pravoj
BC; sa Q1 i Q2 podnozja upravnih iz tacaka P i R na pravoj CA; sa R1 i R2
podnozja upravnih iz tacaka Q i P na pravoj AB, tada je (sl.548).
QR P1 P2 , RP Q1 Q2 , P Q R1 R2
i prema tome

R1
A Q2
Q

Q1

R
R2
B

P1P

MA + MB + MC MA

P2

P1 P2
Q1 Q2
R1 R2
+ MB
+ MC
.
QR
RP
PQ

Medjutim, kako je
P1 P2 = P1 P + P P2 , Q1 Q2 = Q1 Q + QQ2 , R1 R2 = R1 R + RR2
Zatim
P1 P = RP

MQ
MR
, P P2 = P Q
,
MB
MC
657

MR
MP
, QQ2 = QR
,
MQ
MA
MQ
MP
R1 R = QR
, RR2 = RP
,
MA
MB

Q1 Q = P Q

bice
M A + M B + M C 2(M P + M Q + M R).

(b) Ako obelezimo sa P, Q, R podnozja upravnih iz tacke M na pravama


BC, CA, AB i sa A , B , C tacke polupravih M P, M Q, M R takve da je (sl.549)
M A =

1
1
1
1
1
1
= , M B =
= , M C =
=
,
MP
ya
MQ
yb
MR
yc ,

bice tacke A , B , C nekolinearne, te odredjuju izvestan trougao A , B , C . Sem


toga, tacka M je u trouglu A B C . Ako zatim obelezimo sa P , Q , R tacke
polupravih M A, M B, M C takve da je
MP =

1
1
1
1
1
1
, M Q =
=
= , M R =
=
,
MA
xa
MB
xb
MC
xc ,

bice tacke P , Q , R podnozja upravnih iz tacke M na pravama B C , C A , A B .


Zaista, kako je M zajednicki ugao trouglova AM Q i B M P , zatim M A : M Q =
xa : yb = M B : M P , bice AM B B M P , pa je AQM = B P M.
No AQM je prav, pa je B P M prav. Istim postupkom dokazuje se da je i
C P M prav. Stoga je tacka P podnozje upravne iz tacke M na pravoj B C .
Isto tako su tacke Q i R podnozja upravnih iz tacke M na pravama C A i
A B . Prema prethodnom delu ovog zadatka imamo da je
1
(M A + M B + M C ),
2

M P + M Q + M R
pa je

1
1
1
1 1
1
1
+
+
( +
+ ).
xa
xb
xc
2 ya
yb
yc

Znak jednakosti vazice samo u slucaju kada je trougao A B C jednakostranican,


a tacka M njegovo srediste. U tom slucaju bice i trougao ABC jednakostranican,
a tacka M njegovo srediste.
A

B
Q

R
M
Q
B

R
P

Sl. 549.
658

(c)Ako obelezimo sa A1 tacku u kojoj prava AM sece stranicu BC , a sa B i


C podnozja upravnih iz tacaka B i C na pravoj AM , bice (sl.550) BC = BA1 +
A1 C BB + CC . Mnozenjem obeju strana nejednakosti sa AM , nalazimo da
je

BC AM BB AM + CC AM = 2S(M AB) + 2S(M AC) = AByc + ACyb .


Iz ove i analognih nejednakosti sledi da je
axa bab + cyc , bxb cyc + aya , cxc aya + byb .
Mnozenjem odgovarajucih strana dobijenih triju nejednakosti, nalazimo da je
abcxa xb xc (byb + cyc )(cyc + aya )(aya + byb ).
Medjutim, iz nejednakosti
p
p

( byb cyc )2 0, ( cyc aya )2 0, ( aya byb )2 0

sledi da je

byb + cyc 2
Stoga je

p
p

byb cyc , cyc + aya 2 cyc aya , aya + byb 2 aya byb .

abcxa xb xc 8
i prema tome

p
p

byb cyc cyc aya aya byb = 8abcya yb yc


xa xb xc 8ya yb yc .
A

M
B
P

A1

Sl. 550.
(d) Prilikom dokazivanja treceg i prvog dela ovog zadatka imali smo da je
axa byb + cyc iaxa byb + cyb
Sabiranjem odgovarajucih strana ovih nejednakosti, nalazimo da je
2axa byb + cyc + byc + cyb = (b + c)(yb + yc ).
Istim postupkom dobijamo i nejednakosti
2bxb (c + a)(yc + ya )i2cxc (a + b)(ya + yb ).
659

Mnozenjem odgovarajucih strana poslednjih triju nejednakosti, nalazimo da je


8abcxa xb xc (b + c)(c + a)(a + b)(yb + yc )(yc + ya )(ya + yb )
Medjutim, kako je

b + c 2 bc, c + a 2 ca, a + b 2 ab
bice tim pre

8abcxa xb xc 8 bc ca ab(yb + yc )(yc + ya )(ya + yb ),
i pri tome
xa xb xc (yb + yc )(yc + ya )(ya + yb ).
1049.Ako obelezimo sa M bilo koju tacku iz unutrasnjosti trougla ABC, sa
xa , xb , xc rastojanja tacke M od temena A, B, C i sa ya , yb , yc odstojanja tacke
M od pravih BC, CA, AB,
da je (a) x2a + x2b + x2c > 2(ya2 + yb2 + yc2 );
dokazati

(b) xa + xb + xc 2( ya + yb + yc ).
(a) Prilikom dokazivanja teoreme iz 1048. zadatka imali smo da je
xa

1
1
1
(byc + cyb ), xb (cya + ayc ), xc (ayb + bya ).
a
b
c

x2a >

b 2 2 c2 2 2
c2 2 a2 2 2
a2 2 b 2 2
y
+
y
,
x
>
y
+
y
,
x
>
y + y
a2 c a2 b b
b2 a b2 c c
c2 b c2 a

Otuda je

i prema tome
x2a + x2b + x2c > (

b2
c2 2
c2
a2 2
a2
b2 2
+
)y
+
(
+
)y
+
(
+
)y .
c2
b2 a
a2
c2 b
b2
a2 c

Iz ove nejednakosti i iz relacije


c2
c2
a2
a2
b2
b2
+ 2 2, 2 + 2 2, 2 + 2 2,
2
c
b
a
b
b
a
dobijena se trazena nejednakost.
(b) Kao u prethodnom delu, polazimo od relacije
xa

1
1
1
(byc + cyb ), xb (cya + ayc ), xc (ayb + byb ).
a
b
c

Iz prve od tih relacija i nejednakosti


byc + cyb 2

xa

1
1
(byc + cyb )
a
2a

p
byc cyb

q
p
1 p

byc + cyb + 2 byc cyb = ( byc + cyb )


2a
660

Analogno dobijamo i relacije


p

1
1

xb ( cya + ayb ), xc ( ayb + bya ).


2c
2b

Sabiranjem odgovarajucih strana dobijenih triju nejednakosti nalazimo da je

1
b
c
c
a
a
b
xa + xb + xc [( + ) ya + ( + ) yb + ( + ) yc ]
c
a
c
a
2
b
b
Iz ove nejednakosti i iz relacije

b
c
c
a
a
b
+ 2, + 2, + 2
c
a
c
a
b
b
dobijamo da je

xa + xb + xc 2( ya + yb + yc ).
Znak nejednakosti vazice samo u slucaju kada je

b
c
c
a
a
b
+ = 2, + = 2, + = 2,
c
a
c
a
b
b
tj. ako je a = b = c, a tacka M srediste trougla ABC.
1050. Ako obelezimo sa M bilo koju tacku iz unutrasnjosti trougla ABC, sa
xa , xb , xc rastojanja tacke M od temena A, B, C i sa ya , yb , yc odstojanja tacke
1
1
1
M od pravih BC, CA, AB, dokazati da je (a) (xa + xb + xc )( + + ) 18.
ya yb yc
1
1
1
(b) (x2a + x2b + x2c )( 2 + 2 + 2 ) 36.
ya
yb
yc
(a) Prema zadatku 1048. imamo da je
xa + xb + xc 2(ya + yb + yc )

18
.
1
1
1
+
+
ya
yb
yc

Iz ovih relacija neposredno dobijamo trazenu nejednakost. Znak jednakosti


vazi samo u slucaju kada je trougao ABC jednakostranican, a tacka M njegovo
srediste.
(b) S obzirom da je
(x2a + x2b + x2c )(
(

1
1
2
ya

1
yb2

1
yc2

)(

xa
xb
xc
+
+ )2 =
ya
yb
yc

xb
xc
xc
xa 2
xc
xb
)+(
) + ( )2
yc
yb
ya
yc
yb
ya

bice
(x2a + x2b + x2c )(

xb
xc
1
1
xa
1
+
+ )2 0,
+ 2 + 2) (
2
ya
yb
yc
yb
yb
yc
661

pa je
(x2a + x2b + x2c )(

1
1
xa
xb
xc
1
+ 2 + 2) (
+
+ )2
2
ya
yb
yc
yb
yb
yc

Medjutim prema zadatku 1048. imamo da je


r
xb
xc
xa xb xc
xa
+
+
33
yb
yb
yc
ya yb yc
i

xa xb xc
8,
ya yb yc

pa je
(x2a + x2b + x2c )(

r
xa xb xc 2
1
1
1
3
(
+
)

9
) 36
+
2
2
2
ya
yb
yc
ya yb yc

Znak jednakosti vazi samo u slucaju kada je trougao ABC jednakostranican, a


tacka M njegovo srediste.
1051. Ako obelezimo sa M proizvoljnu tacku iz unutrasnjosti trougla ABC,
sa xa , xb , xc rastojanje tacke M od temena A, B, C i sa ya , yb , yc odstojanje
tacke M od pravih BC, CA, AB, dokazati da je
axa + bxb + cxc 2(aya + byb + cyc ).

S obzirom daje tacka M u trouglu ABC, bice


aya + byb + cyc = aha = bhb = chc
i prema tome
a(ha ya ) = byb + cyc , b(hb yb ) = cyc + aya , c(hc yc ) = aya + byb
Iz ovih jednakosti i iz nejednakosti
ha ya xa , hb yb xb , hc yc xc
sledi da je
axa byb + cyc , bxb cyc + aya , cxc aya + byb
Sabiranjem odgovarajucih strana ovih triju nejednakosti dobijamo trazenu
nejednakost. Znak jednakosti vazice samo u slucaju kada je
ha ya = xa , hb yb = xb , hc yc = xc
tj. u slucaju kada je tacka M ortocentar trougla ABC.
1052. Ako obelezimo sa M proizvoljnu tacku iz unutrasnjosti trougla ABC,
sa xa , xb , xc rastojanje tacke M od temena A, B, C i sa ya , yb , yc odstojanje
tacke M od pravih BC, CA, AB, dokazati da je (a) xa ya + xb yb + xc yc
2(yb yc + yc ya + ya yb ); (b) xb xc + xc xa + xa xb 2(xa ya + xb yb + xc yc ); (c)
662

xb xc + xc xb + xa xb 2(yb yc + yc ya + ya yb ).
(a)Prilikom dokazivanja zadataka 1051. imali smo da je
axa byb + cyc , bxb cyc + aya , cxc ayc + byb .
Ove nejednakosti mozemo napisati u obliku
xa ya

c
b
yb yc + ya yc ,
a
a

c
a
yb yc + ya yb ,
b
b
a
b
xc yc ya yc + yb yc
c
c
Sabiranjem odgovarajucih strana ovih nejednakosti, nalazimo da je
xb yb

c
a
a
b
b c
xa ya + xb yb + xc yc ( + )yb yc + ( + )yc ya + ( + )ya yb
c b
a
c
b
a
Iz ove i iz nejednakosti
c
a
a
b
b c
( + ) 2, ( + ) 2, ( + ) 2,
c b
a
c
b
a
dobija se trazena nejednakost. Znak jednakosti vazice samo u slucaju kada je
trougao ABC jednakostranican, a tacka M njegovo srediste.
(b) Ako obelezimo sa A , B , C tacke polupravih M A, M B, M C takve da je
M A =

1
1
1
, M B =
, M C =
MA
MB
MC

bice te tri tacke nekolinearne, a tacka M u trouglu A B C . Ako zatim obelezimo


sa xa , xb , xc rastojanja tacke M od tacaka A , B , C i sa ya , yb , yc odstojanja
tacke M od pravih B C , C A , A B , tada je prema Erdesevoj teoremi
xa + xb + xc 2(ya + yb + yc ).
Pri tome je
xa =

xc xa
xa xb
xb xc
, xb
, xc =
xa xb xc
xa xb xc
xa xb xc

S obzirom da su trouglovi BM C, CM A, AM B slicni s trouglovima


C M B ,A M B ,B M A , bice i
ya =

xa ya
xb yb
xc yc
, yb =
, yc =
xa xb xc
xa xb xc
xa xb xc

Iz ovih nejednakosti sleduje trazena relacija. Znak jednakosti vazi samo u


slucaju kada je trougao A B C pravilan, tacka M njegovo srediste; tj. kada je
trougao ABC pravilan, a tacka M njegovo srediste.
c) Dokaz ovog dela zadatka sleduje nepodredno iz prethodna dva dela.
663

1053. Ako obelezimo sa M tacku iz unutrasnjosti trougla ABC, sa xa , xb , xc


rastojanje tacke M od temena A, B, C i sa ya , yb , yc odstojanje tacke M od pra1
1
1
1
1
1
vih BC, CA, AB, dokazati da je (a)
+
+
2(
+
+
);
yb yc yc ya ya yb
xa ya xb yb xc yc
1
1
1
1
1
1
1
1
1
+
+
2(
+
+
); (c)
+
+

(b)
xa ya
xb yb
xc yc
xb xc
xc xa
xa xb
yb yc
yc ya
ya yb
1
1
1
4(
+
+
).
xb xc
xc xa
xa xb
(a) Ako obelezimo sa A , B , C tacke polupravih M A, M B, M C takve da je
M A =

1
1
1
, M B =
, M C =
,
MA
MB
MC

tacke A , B , C ce biti nekolinearne, a tacka M u trouglu A B C . Ako zatim


obelezimo sa xa , xb , xc , rastojanja tacke M od tacaka A , B , C i sa ya , yb , yc
odstojanja tacke M od pravih B C , C A , A B prama zadatku 1052. imamo da
je
xa ya + xb yb + xc yc 2(yb yc + yc ya + ya , yb ).
Ako vrednosti za xa , xb , xc i ya , yb , yc koje smo dobili prilikom dokazivanja teorema..... zamenimo u ovoj nejednakosti dobijamo da je
yb
yc
yb yc
yc ya
ya yb
ya
+
+
2( 2
+
+
).
xa xb xc
xa xb xc
xa xb xc
xa xb xc
xa x2b xc
xa bxb x2c
xa xb xc
dobijamo trazenu nejedya yb yc
nakost. Znak jednakosti vazi samo u slucaju kada je trougao ABC jednakostranican, a tacka M njegovo srediste.
(b) Ako obelezimo sa P, Q, R podnozja upravnih iz tacke M na pravama
BC, CA, AB i sa A , B , C tacke polupravih M P, M Q, M R takve da je
Mnozenjem obeju strana ove nejednakosti sa

M A =

1
1
1
1
1
1
= , M B =
= , M C =
= ,
MP
ya
MQ
yb
MR
yc

tacke A , B , C ce biti nekolinearne, a tacka M u trouglu A , B , C . Ako zatim


obelezimo sa P , Q , R tacke polupravih M A, M B, M C takve da je
MP =

1
1
1
1
1
1
, M Q =
=
= , M R =
=
MA
xa
MB
xb
MC
xc

tacke P , Q , R bice podnozja upravnih iz tacke M na pravama B C , C A , A B .


Najzad, ako obelezimo sa xa , xb , xc rastojanja tacke M od tacaka A, B, C i sa
ya , yb , yc odstojanje tacke M od pravih B C , C A , A B bice prama zadatku
1052.
xa ya + xb yb + xc yc 2(yb yc + yc ya + ya , yb ),
pa je i
1
1
1
1
1
1
+
+
2(
+
+
).
xa ya
xb yb
xc yc
xb xc
xc xa
xa xb

664

Znak jednakosti vazi samo u slucaju kada je trougao ABC jednakostranican i


tacka M njegovo srediste.
(c) Dokaz ovog dela neposredno sleduje iz prethodna dva dela.

1054.Ako je S srediste i poluprecnik kruga upisanog u trouglu ABC, dokazati da je (a) SA + SB + SC 6; (b) SA SB SC 83 .
Ako obelezimo sa P, Q, R tacke u kojima upisani krug dodiruje stranice BC, CA, AB
trougla ABC, prema zadatku 1048. bice
(a) SA + SB + SC 2(SP + SQ + SR) = 6
(b) (SA SB SC) (SQ + SR)(SR + SP )(SP + SQ) = 83 .
1055. Ako su d1 , . . . , dn rastojanja proizvoljne tacke kruga opisanog oko pravilnog poligona koji ima n stranica od njegovih

temena i r poluprecnik opisanog


kruga, dokazati da je (a) d1 + . . . + dn r 2n; (b) d1 + . . . + dn nr 2.
(a) Prema zadacima 982.i 848. imamo da je
(d1 + + dn )2 (d21 + + d2n ) i d21 + + d2n = 2nr2
pa je

d1 + + dn r 2n.
2
2
2
(b)Prema zadacima 982. i 848. imamo da je (d
1 + +dn ) n(d1 + +dn )
2
2
2
i (d1 + + dn ) = 2nr , pa je d1 + + dn nr 2.

1056. Ako je T teziste poligona A1 . . . An upisanog u krug poluprecnika r,


dokazati da je
T A21 + . . . + T A2n nr2 .

Ako obelezimo sa O srediste kruga opisanog oko poligona A1 An i sa d duz

odredenu
tackama O i T , prema zadatku 850. imamo da je
T A21 + + T A2n = n(r2 + d2 ), pa je T A21 + + T A2n nr2 .
1057.Ako je T teziste konacnog skupa od n tacaka A1 , . . . , An i P proizvoljna
tacka, dokazati da je
T A21 + . . . + T A2n P A21 + . . . + P A2n .

665

Primeniti Lajbnicovu teoremu.

1058.Ako je T teziste konacnog skupa od n tacaka A1 , . . . , An , zatim P


proizvoljna tacka neke prave p i Q podnozje upravne iz tacke T na pravoj p,
dokazati da je
P A21 + . . . + P A2n QA21 + . . . + QA2n .

Primeniti Lajbnicovu teoremu.

1059. Ako obelezimo sa M proizvoljnu tacku iz unutrasnjosti trougla ABC,


sa xa , xb , xc rastojanja tacke M od temena A, B, C i sa poluprecnik upisanog
kruga tog trougla, dokazati da je (a) xa + xb + xc 6; (b) x2a + x2b + x2c 122 .
(a) Ako obelezimo sa ya , yb , yc odstojanje tacke M od pravih BC, CA, AB i
sa ha , hb , hc visine iz temena A, B, C trougla ABC, imamo da je xa + ya ha ,
xb + yb hb , xc + yc hc . Sabiranjem odgovarajucih strana ovih nejedna
kosti, nalazimo sa je xa + xb + xc + ya + yb + yc ha + hb + hc . Medutim
prema zadacima 1048. i 1019. imamo da je xa + xb + xc 2(ya + yb + yc ) i
1
ha + hb + hc 9, pa je stoga xa + xb + xc + (xa + xb + xc ) 9 i prema tome
2
xa + xb + xc 6. Znak jednakosti vazi samo u slucaju kada je trougao ABC
jednakostranican a tacka M njegovo srediste.
(b)Ako obekezimo sa T teziste trougla ABC, prema zadatku 1057.imamo
da je AM 2 + BM 2 + CM 2 AT 2 + BT 2 + CT 2 . Obelezimo li sa ma , mb , mc
tezisne linije iz temena A, B, C , tada koristeci zadatke 1031. i 1017. nalazimo
da je
x2a + x2b + x2c

1
4 2
(ma + m2b + m2c ) = (a2 + b2 + c2 ) 122 .
9
3

Znak jednakosti vazi samo u slucaju kada je trougao ABC jednakostranican, a


tacka M njegovo srediste.
1061. Dokazati da od svih trouglova upisanih u dati ostrougli trougao najmanji obim ima trougao koji je odreden podnozjima visina datog trougla.
Obelezimo sa A , B , C podnozja visina iz temena A, B, C trougla ABC i sa
A , B , C proizvoljne tacke stranica BC, CAiAB tog trougla. Sem toga, obelezimo
sa M i N tacke simetricne s tackom A u odnosu na prave AB i AC, a
sa M i N tacke simetricne s tackom A u odnosu na prave AB i AC. Pri
tome su tacke B i C izmedu tacaka M i N takve da je M C B N
,M C = A C i B N = A B , pa je obim A B C jednak duzi M N . Iz
jednakosti A C = M C i A B = B N sledi da je obim A B C jednak
zbiru duzi M C , C B , B N , pa je duz M N jednaka ili manja od obima
A B C . Kod AM N M AN je dva puta veci od ugla A ABC, a stranice AM i AN jednake duzi AA , dakle jednake i medu sobom. Isto tako, kod
666

AM N M AN takode je dva puta veci od ugla kod temena A ABC, a


stranice AM i AN jednake duzi AA , dakle jednake i medju sobom. S obzirom da je AA < AA , kod jednakokrakih trouglova AM N i AM N kojima
su uglovi pri vrhu A jednaki bice duz M N manja od duzi M N , pa je obim
A B C manji od duzi M N , dakle i od obima A B C (sl. 551).
N
A

N
B

C
C

M
M

C
A

Sl. 551
1062.Ako su ha , hb , hc visine trougla ABC i ya , yb , yc odstojanja proizvoljne
tacke iz unutrasnjosti trougla ABC od pravih BC, CA, AB, dokazati da je
min(ha , hb , hc ) ya + yb + yc max(ha , hb , hc ).

Ako obelezimo sa a, b, c duzine stranica BC, CA, AB i sa S povrsinu trougaone povrsi (ABC), bice
aya + byb + cyc = 2S.
Ako pretpostavimo da je stranica BC najveca, bice aya + ayb + ayc 2S, tj.
2S
ya + yb + yc
, i prema tome ya + yb + yc ha . Ako pretpostavimo da je
a
2S
i prema
stranica BC najmanja, bice aya + ayb + ayc 2S, tj. ya + yb + yc
a
tome ya + yb + yc ha . Stoga je
min(ha , hb , hc ) ya + yb + yc max(ha , hb , hc ).

1063. Ako su ha , hb , hc visine trougla ABC i poluprecnik upisanog kruga,


dokazati da je
min(ha , hb , hc ) 3 max(ha , hb , hc ).

667

Ako je S srediste kruga upisanog u ABC, bice odstojanja ya , yb , yc tacke


S od stranica BC, CA, AB jednaka duzi , pa je prema prethodnom zadatku
min(ha , hb , hc ) 3 max(ha , hb , hc ).

1064.Neka je M proizvoljna tacka iz unutrasnjosti trougla ABC i M Lemoanova tacka tog trougla. Ako su P i P , Q i Q , R i R podnozja upravnih iz
tacke M i M na pravama BC, CA, AB dokazati da je
M P 2 + M Q2 + M R2 M P 2 + M Q2 + M R2 .

1065.Dokazati da od svih trouglova upisanih u dati trougao najmanji zbir


kvadrata stranica ima trougao kome su temena podnozja upravnih iz Lemoanove tacke na stranicama datog trougla.

Obelezimo sa P, Q, R podnozja upravnih iz Lemoanove tacke L na stranicama


datog trougla ABC,a sa P , Q , R bilo koje tri tacke stranica BC, CA, AB.
Prema zadatku 760, tacka L je teziste P QR. Neka je T teziste P Q R .
Ako su P , Q , R podnozja upravnih iz tacke T na pravama BC, CA, AB,
prema zadaku.... imamo da je (sl. 552)
LP 2 + LQ2 + LR2 < T P 2 + T Q2 + T R2 < T P 2 + T Q2 + T R2 .
No
LP 2 + LQ2 + LR2 =
i
T P 2 + P Q2 + T R2 =

1
(P Q2 + QR2 + RP 2 )
3
1 2
(P Q + Q R2 + R P 2 ),
3

pa je
P Q2 + QR2 + RA2 < P Q2 + Q R2 + R P 2 .

A
R
R
R

Q
Q

L
T

C
P

Sl. 552

668

1066.Ako je O proizvoljna tacka iz unutrasnjosti trougla ABC i ako su


A , B , C tacke u kojima prave AO, BO, CO seku respektivno stranice BC, CA, AB,
BB
CC
BB
CC
9
AA
AA
+
+
9.
+
+
; (b)
dokazati da je (a)

AO
BO
CO
2
OA
OB
OC

(a) Prema zadatku ...... imamo da je


(

AO
BO
CO AA
BB
CC
+
+
)(
+
+
) 9.
AA
BB
CC AO
BO
CO

A prema zadatku 936. imamo da je


BO
CO
AO
+
+
= 2,
AA
BB
CC
pa je

AA
BB
CC
9
+
+
.
AO
BO
CO
2
(b)Prema zadatku ..... imamo da je
(

OA
OB
OC AA
BB
CC
+
+
)(
+
+
) 9,
AA
BB
CC OA
OB
OC

a prema zadatku 936. da je


OB
OC
OA
+
+
= 1,
AA
BB
CC
pa je

BB
CC
AA
+
+
9.
OA
OB
OC

1067. Ako je O proizvoljna tacka iz unutrasnjosti trougla ABC i ako su


A , B , C tacke u kojima prave AO, BO, CO seku respektivno stranice BC, CA, AB,
8
1
OA OB CO
AO BO CO

;
(b)
.
dokazati da je (a)
AA BB CC
27
AA BB CC
27
(a)Prema nejednakosti za geometrijsku i aritmeticku sredinu i zadatku 936.
imamo da je
AO BO CO
1 AO
BO
CO 3

+
+
)
(

AA BB CC
27 AA
BB
CC
i
pa je

AO
BO
CO
+
+
=2

AA
BB
CC
AO BO CO
8

AA BB CC
27

669

(b)Prema nejednakosti za geometrijsku i aritmeticku sredinu i zadatku 936.


imamo da je
1 OA
OB
OC 3
OA OB OC

+
+
)
(
AA BB CC
27 AA
BB
CC
i

OA
OB
OC
+
+
= 1,
AA
BB
CC

pa je

OA OB OC
1

AA BB CC
27
1068. Ako je O proizvoljna tacka iz unutrasnjosti trougla ABC i ako su A ,

B , C tacke u kojima prave AO, BO, CO seku respektivno stranice BC, CA,
AB, dokazati da je
(a)
BO
CO
AO
+
+
6,
OA
OB
OC
(b)
AO
BO
CO
+
+
8.
OA
OB
OC
(a) Prema Van Obelovoj teoremi, imamo da je
AB
AC
AO
=
+
,
OA
BC
CB
BC
BA
BO
=
+
,
OB
CA
A C
CO
CA
CB
=
+
.
OC
A B
BA
Sabiranjem odgovarajucih strana i primenom nejednakosti .... nalazimo da je
BO
CO
AC
BA
CB
CB
A C
BA
AO
+
+
=
(
)
+
(
)
+
(
) 6.
+
+
+
OA
OB
OC
C B
AC
A C
BA
B A CB
Znak jednakosti nastupa samo u slucaju kada je tacka O teziste trougla ABC.
Drugi nacin. Ako sa S1 , S2 , S3 obelezimo povrsine trougaonih povrsi (BOC),
(COA), (AOB), bice
S2 + S3
AO
=
,
OA
S1
S3 + S1
BO
=
,

OB
S2
CO
S1 + S2
=
,

OC
S3

670

pa se sabiranjem odgovarajucih strana i primenom nejednakosti .... dobija da


je
BO
CO
S1
S2
S2
S3
S3
S1
AO
+
+
=(
+
)+(
+
)+(
+
6.

OA
OB
OC
S2
S1
S3
S2
S1
S3
Znak jednakosti nastupa samo u slucaju kada je S1 = S2 = S3 , tj. kada je tacka
O teziste trougla ABC.
(b) Prema Van Obelovoj teoremi imamo da je
AB
AC
AO
= + ,

OA
BC
CB
BO
BC
BA
= + ,

OB
CA
AC

CA
CB
CO
= + .

OC
AB
BA
Mnozenjem odgovarajucih strana ovih jednakosti i primenom zadataka .... i ....,
nalazimo da je
AC
BA
CB
CB
A C
BA
AO BO CO

=
(
)
+
(
)
+
(
)+
+
+
+
OA OB OC
C B
AC
A C
BA
B A CB

AB CA BC
AC BA CB

) 8.
C B A C B A B C A B C A

Drugi nacin. Mnozenjem idgovarajucih strana jednakosti


AO
S2 + S3
=
,
OA
S1
S3 + S1
BO
=
,
OB
S2
CO
S1 + S2
=
,

OC
S3
i primenom nejednakosti za aritmeticku i geometrijsku sredinu, nalazimo da je

(S1 + S2 )(S2 + S3 )(S3 + S1 )


8 S1 S2 S2 S3 S3 S1
AO BO CO

= 8.
OA OB OC
S1 S2 S3
S1 S2 S3

1069. Ako se dijagonale AC i BD cetvorougla ABCD seku u tacki O i ako


prava kroz O uporedna sa AB sece stranice AD i BC u tackama A1 i B1 , prava
kroz O uporedna sa BC sece stranice AB i CD u tackama B2 i C2 , prava kroz
O uporedna sa AD sece stranice CD i AB u tackama D1 i A2 , dokazati da je
A1 B2 B1 C2 C1 D2 D1 A2

1.
AB
BC
CD
DA
671

Koristeci zavisnost koja postoji izmedu geometrijske i aritmeticke sredine,


nalazimo da je
r
4

A1 B2 B1 C2 C1 D2 D1 A2
1 A1 B2
B1 C2
C1 D2
D1 A2

(
+
+
+
).
AB
BC
CD
DA
4 AB
BC
CD
DA

Prema zadatku .... imamo da je


B1 C2
C1 D2
D1 A2
A1 B2
+
+
+
= 4,
AB
BC
CD
DA
pa je i

A1 B2 B1 C2 C1 D2 D1 A2

1.
AB
BC
CD
DA

1070. Ako su a, b, c stranice i S povrsina trougaone povrsi, dokazati da je


(a)
S

1 2
(a ab + b2 );
2

(b)
S
(v)
S
(g)

1
(a + b)2 ;
8

1 2
(a + B 2 );
4

1 2
(a + b2 + c2 ).
6
Ako obelezimo sa ha visinu koja odgovara stranici a, bice
S<

(a)

S=

aha
ab
ab + (a b)2
a2 ab + b2

=
2
2
2
2

(b)

S=

ab
4ab + (a b)2
(a + b)2
aha

=
2
2
8
8

(v)

S=

ab
2ab + (a b)2
a2 + b 2
aha

=
2
2
4
4

(g) Prema prethodnom delu zadatka, imamo da je


S

a2 + b 2
b 2 + c2
c2 + a2
,S
,S
,
4
4
4

pri cemu znak jednakosti moze da vazi samo u jednoj od tih relacija. Sabiranjem
odgovarajucih strana nalazimo da je
S<

1 2
(a + b2 + c2 ).
6
672

1071. Ako su a, b, c stranice i S povrsina trougaone povrsi, dokazati da je


(a)
S

3
(a + b + c)2 ;
36

(b)

3 2
S
(a + b2 + c2 );
12

(v)

3
S
(ab + bc + ca);
12

(g)

3 2
[a + b2 + c2 (a b)2 (b c)2 (c a)2 ].
S
12

p2
p
(a) Prema zadatku 1014, imamo da je , pa je p , i prema
3 3
3 3

3 2
3
tome S
p . Otuda je S
(a + b + c)2 .
9
36

3 2
p . Otuda i
(b) Prema izvedenom delu ovog zadatka, imamo da je S
9

3 2
3
iz nejednakosti p2 (a2 + b2 + c2 ) sledi da je S
(a + b2 + c2 ).
4
12

3 3
3 3
r, pa je p
r, i prema
(v) Prema zadatku 1014 imamo da je p
2
2

3 3
1
tome S
r .Otuda i iz nejednakosti r
(ab + bc + ca) sledi da je
2
18

3
S
(ab + bc + ca).
18
(g) Ako obelezimo sa p poluobim i sa , a , b , c , poluprecnike
upisanih krugova, prema zadatku
1027,
imamo
da
je

p
3, pa je
a
b
c

4(a + b + c ) 4S 3. Primenom stava .... , nalazimo da je


4(a + b + c ) = 4[(p b)(p c) + (p c)(p a) + (p a)(p b)]

= 4(ab+bc+ca)4p2 = 4(ab+bc+ca)(a+b+c)2 = 2(ab+bc+ca)(a2 +b2 +c2 )

= a2 + b2 + c2 (a b)2 (b c)2 (c a)2 4S 3. Otuda je


p
3 2
[a + b2 + c2 (a b)2 (b c)2 (c a)2 ].
S
12

Nejednakost 1071(g) je poznata nejednakost Finslera i Hadvigera koja je


objavljena 1938. godine. Ona predstavlja poostrenje nejednakosti 1071(b) i
1071(v).

673

1072. Ako je S povrsina i p poluobim neke trougaone povrsi, a r poluprecnik


opisanog kruga i poluprecnik upisanog kruga, dokazati da je
(a)

3 2
p ;
S
9
(b)

S 3 32 ;
(v)

3 3
r;
S
2

(g)

3 3 2
S
;
4

(d)

3 2
S
(2r + 2 );
3

(d)

3
S
(4r + )2 ;
27

(e)
S

3
(r + )2 .
3

p2
p
(a) Prema zadatku 1014, imamo da je , pa je p , i prema
3 3
3 3

3 2
tome S
p .
9

(b) Prema
1014. imamo da je p 3 3, pa je p 3 32 , i prema
zadatku
tome S 3 32 .

3 3
3 3
(v) Prema zadatku 1014. imamo da je p
r, pa je p
r, i
2
2

3 3
r.
prema tome S
2

3
abc
3 3 2
(g) Prema zadatku 1024. imamo da je abc 3 3r , pa je

r ,i
4r
3

3 3 2
prema tome S
r .
4

r
3 3 2
3 3
r i sledi da je S
r .
Drugi nacin. Iz jednakosti S
2
2
4

3 2
(d) Prema zadatku 1071. i 1017. imamo da je S
(a + b2 + c2 ) i
12

3 2
(2r + 2 ).
(a2 + b2 + c2 ) 4(2r2 + 2 ) pa je S

3 2
3
3
2
(d) Iz nejednakosti S
p ip
(4r+) sledi da je S
(4r+2 ).
9
3
24

3
(e) Iz nejednakosti S
(ab + bc + ca) i ba + bc + ca 4(r + )2 sledi da
12

3
(r + )2 .
je S
3
674

2p
Drugi nacin. Prema zadatku 1014. imamo da je r, tj.
3 3
1 p
p
( + + 3) r + .
3 3
3
Primenom nejednakosti za aritmeticku i geometrijsku sredinu, nalazimo da je
p
p
p
1 p
p
3
p2 ( + + 3) pa je 3 p2 r + . Otuda je 3 Sp r + , tj.
3 3
3
p

3
S 2 p2 (r + )2 . Iz ove nejednakosti i iz 3 3S p2 sledi da je
q
3
( 3S)3 (r + )2 ,

i prema tome

3
S
(r + )2
3

.
1073. Ako su r i poluprecnici opisanog i upisanog kruga trougla ABC,
dokazati da je

S(ABC) > 2 r.
Ako su a, b, c stranice i ha , hb , hc visine trougla ABC, prema zadatku 998.
bice hi > 2, tj. 2rhi > 4r, gde je i = a, b, c. Iz tih nejednakosti i iz 2rha = bc,
2rhb = ca, 2rhc = ab sledi da je bc > 4r, ca > 4r, ab > 4r. Stoga je
abc

> 2 r, i prema tome


abc > 8r r, tj.
4r

S(ABC) > 2 r
.
1074. Ako su a, b, c stranice i S povrsina neke trougaone povrsi, dokazati da
je
(a)

3
3
a2 b 2 c2 ;
S
4

(b)
S
(v)

1p 2 2
a b + b 2 c2 + c2 a2 ;
4

1p 4
a + b 4 + c4 .
4
(a) Prema zadacima 1024. i 1072. imamo da je

abc
3
3 3 abc
3 3
abc
3
=
S=
a2 b 2 c2 .


3
4r
4
2p
4
4
3 abc
S

(b) Prema nejednakosti za geometrijsku i aritmeticku sredinu, imamo da je

3
3 a4 b4 c4 a2 b2 + b2 c2 + c2 a2 . Otuda i iz prethodne nejednakosti sledi da je
S

1p 2 2
a b + b 2 c2 + c2 a2
4
675

.
(v) Prema nejednakosti za geometrijsku i aritmeticku sredinu, imamo da je

3
3 a4 b4 c4 a4 + b4 + c4 . Otuda i iz nejednakosti pod (a) sledi da je
S

1 4 4 4
a b c .
4

1075. Ako su a, b, c stranice, ha , hb , hc visine, a , b , c poluprecnici spolja


upisanih krugova i S povrsina neke trougaone povrsi, dokazati da je
(a)

3
3
S
a2 b 2 c2 ;
4
(b)
q
33 2 2 2
a b c ;
S
3
(v)
q
33
S
ha2a ha2b ha2c ;
3
(g)

3 2
(a + b2 + c2 );
S
12
(d)

3 2 2 2
a b c .
S
9

3
abc
3
(a) Prema zadacima 1024. i 1072. imamo da je S =
abc, pa
ir
4r
3

3 3 2 2 2
je S
a b c .
4

(b) Prema zadacima 1021. i 1025.imamo da je S = a b c i S

3p
1
3
3 pa je S
2a 2b 2c .
a b c
3
3
r
3
ha hb hc i r
(v) Prema zadacima 1026. i 1033. imamo da je S =
2

2
3p
3
3
ha hb hc , pa je S
h2a h2b h2c .
3
3

3
3
(g) S obzirom da je S
a2 b2 c2 , a prema nejednakosti za geometrijsku
4

1 2
3 2
3
2
2
2
2
2
(a + b2 + c2 ).
i aritmeticku sredinu a b c (a + b + c ), bice S
3
12

3p
3
(d) S obzirom da je S
2a 2b 2c , a prema nejednakosti za geometrij3

p
3 2
1
sku i aritmeticku sredinu 3 2a 2b 2c (2a +2b +2c ), bice S
( +2 +2c ).
3
9 a b
1076. Ako je r poluprecnik opisanog kruga, poluprecnik upisanog kruga i
S povrsina povrsi pravouglog trougla, dokazati da je
S

1
(r + )2 .
2
676

Ako obelezimo sa a, b, c hipotenuzu i katete pravouglog trougla i sa p njegov

a S
a
bc
b+c
poluobim, bice r + = + = +
=
bc = 2S, pa je
2
p
2 a+b+c
2
S

1
(r + )2 .
2

1077. Ako su a,b,c,d duzi jednake stranicama AB, BC, CD, DA i e i f duzi
jednake dijagonalama AC i BD prostog cetvorougla ABCD, dokazati da je
(a)
1
S(ABCD) (a2 + b2 + c2 + d2 );
4
(b)
1
S(ABCD) (e2 + f 2 ).
4
(a) S obzirom da je cetvorougao ABCD prost, bar jedna njegova dijagonala,
npr. AC je unutrasnja. Ta dijagonala razlaze cetvorougaonu povrs (ABCD) na
dve trougaone povrsi (ABC) pa je prema tome
S(ABCD) = S(ABC) + S(ACD)

1 2
(a + b2 + c2 + d2 ).
4

(b) Prave odredene dijagonalama AC i BD prostog cetvorougla ABCD seku


se u nekoj tacki O. Jedna od dijagonala tog cetvorougla, npr. dijagonala AC
je unutrasnja, prema tome, ona razlaze cetvorougaonu povrs (ABCD) na dve
trougaone povrsi (ABC) i (ACD). Ako obelezimo sa h1 i h2 visine iz temena
B i D trouglova ABC i ACD, a sa f1 i f2 duzi jednake odseccima OB i OD, bice
S(ABCD) = S(ABC) + S(ACD) =
e(f1 + f2 )
ef
2ef + (e f )2
e2 + f 2
e(h1 + h2 )

=
=
=
.
2
2
2
4
4
1078. Ako su a, b, c, d duzi jednake stranicama AB, BC, CD, DA i e i f
duzi jednake dijagonalama AC i BD konveksnog cetvorougla ABCD, dokazati
da je

3 2
a + b2 + c2 + d2 + e2 + f 2 .
S(ABCD) <
12
Kako je cetvorougao ABCD konveksan, obe njegove dijagonale su unutrasnje, prema tome, svaka od njih razlaze cetvorougaonu povrs (ABCD) na
dve trougaone povrsi. Pa otuda, primenom zadatka 1071b, nalazimo da
je
1
1 3 2
S(ABCD) = [S(ABC) + S(ACD) + S(ABD) + S(BCD)] < [
(a +
2 12

2
3 2
3 2
3 2
(c + d2 + e2 ) +
(a + d2 + f 2 ) +
(b + c2 + f 2 )] =
b2 + e2 ) +
12
12
12

3 2
(a + b2 + c2 + d2 + e2 + f 2 ).
12
1079. Ako su a, b, c, d stranice konveksnog cetvorougla ABCD, dokazati da
je
S(ABCD)

1
(a + c)(b + d).
4

677

S obzirom da je cetvorougao ABCD konveksan, obe njegove dijagonale


su unutrasnje. Stoga, svaka od tih dijagonala razlaze cetvorougaonu povrs
(ABCD) na dve trougaone, pa je
1
S(ABCD) = [S(ABC) + S(BCD) + S(CDA) + S(DAB)]
2
1
1
(ab + bc + cd + da) = (a + c)(b + d).
4
4
1080. Ako je p poluobim i S povrsina tangentne i tetivne cetvorougaone
povrsi ABCD, dokazati da je
1
S p2 .
4
Ako obelezimo sa
a, b, c, d stranice tangentne i tetivne cetvorougaone
povrsi

ABCD, bice 2p 4 4 abcd, a prema zadatku 925 da je S = abcd. Stoga je

1
2p 4 S , i prema tome S p2 .
4
1081. Neka je S tacka u konveksnom uglu AOB i neka su s i s dve razne
prave kroz S, od kojih prva sece krake OA i OB ugla AOB u tackama P i Q, a
druga sece krake OA i OB u tackama P i Q .Ako je pri tome tacka S srediste
duzi P Q, dokazati da je
S(OP Q) < S(OP Q ).
Postoje dva slucaja: ili je [OPP] ili je [OPP]. Ako je [OPP] bice O-Q-Q, pa
je S(OP Q) = S(OP M Q ) + S(M QQ ) i S(OP Q ) = S(OP M Q ) + S(M P P ).
Prava kroz tacku P uporedna sa OQ sece pravu s u nekoj tacki Q, koja
je izmedu M i P pa je S(M P P ) = S(M P Q) + S(P P Q ). S obzirom da
su trouglovi M QQ i M P Q podudarni , bice S(M QQ ) = S(M P Q), pa je
S(OP Q) < S(OP Q ). Ako je [OPP] postupak pri dokazivanju ostaje isti.

Q
S
Q

678

Sl. 553
1082. Ako je k krug upisan u trougao ABC, a A B C jednakostranican
trougao upisan u krug k, dokazati da je
S(ABC) 4S(A B C ).
Obelezimo sa poluprecnik kruga k upisanog u trougao ABC, sa poluprecnik kruga k upisanog u trougao A B C , sa p poluobim trougla ABC i sa
p poluobim trougla A B C . S obzirom da je trougao A B C jednakostrani
can,
3
3

3
i p = A B =
.
a poluprecnik njegovog opisanog kruga, bice =
2
2
2

3 3 2
Stoga je S(ABC) = p i S(A B C ) =
. Prema zadatku 1033, imamo da
2 4

je p 3 3, pa je i S(ABC) 3 3 , odnosno S(ABC) 4S(A B C ).


1083. Ako su temena P , Q, R trougla P QR na stranicama BC, CA, AB
trougla ABC, dokazati da je povrsina bar jedne od trougaonih povrsi (AOR),
(BRP ), (CP Q) manja ili jednaka od povrsine trougaone povrsi (P QR).
Obelezimo sa A , B , C sredista stranica BC, CA, AB trougla ABC. Ako
se tacke P , Q, R poklapaju s tackama A , B , C ili se samo dve tacke, npr. Q
i R poklapaju s tackama B i C , bice S(P QR) = S(A B C ) = S(AB C ) =
S(AQR), pa je u tom slucaju tvrdenje dokazano. Ako nije taj slucaj, jedna
od duzi QR, RP , P Q nema s trouglom A B C zajednickih tacaka, ili pak sve
te duzi imaju s trouglom A B C zajednickih tacaka. Ako npr. duz QR nema
s trouglom A B C zajednickih tacaka, duz QR je u trouglu A B C . U tom
slucaju duz AP sece duz QR u nekoj tacki K, pri cemu je P K > AK. Stoga
je S(AKR) < S(P KR) i S(AKQ) < S(P KQ), te je S(AKR) + S(AKQ) <
S(P KR) + S(P KQ), i prema tome S(AQR) < S(P QR).
Ako sve duzi QR, RP , P Q imaju s trouglom A B C zajednickih tacaka, tada
se odgovarajuce stranice trouglova P QR i A B C seku. U tom slucaju je npr.
A P C, i prema tome B Q A i C R B. Stoga je S(P QR) >
1
S(A QR) > S(A QC) = S(A B C ) = S(ABC). Iz dobijene najednakosti
4
1
3
S(P QR) > S(ABC) sledi da je S(AQR) + S(BRP ) + S(CP Q) < S(ABC),
4
4
1
pa je povrsina bar jedne od povrsi (AQR), (BRP ), (CP Q) manja od S(ABC),
4
i prema tome manja od S(P QR). Time je tvrdenje dokazano.

679

R
C

R
B

Sl. 554
1084. Ako su ABC i A1 B1 C1 dva trougla od kojih je BC = B1 C1 , A =
A1 , a razlika stranica AB i AC manja od razlike stranica A1 B1 i A1 C1 , dokazati da je
S(ABC) > S(A1 B1 C1 ).
Pretpostavimo da je AB > AC i A1 B1 > A1 C1 . S obzirom da je BC = B1 C1
s one strane prave BC s koje je A postoji tacka A takva da je A BC
A1 B1 C1 . Iz jednakosti BAC = B1 A1 C1 i B1 A1 C1 = BA C, sledi da je
BAC = BA C, pa je tacka A na kruznom luku BAC. Kako je AB > AC i
A B > A C, na duzinama AB i A B postoje tacke D i D takve da je AD = AC
i A D = A C. Pri tome su tacke D i D s iste strane prave BC, a uglovi
BDC i BD C jednaki, pa su tacke B, C, D, D na istom krugu. S obzirom
da je razlika stranica AB i AC trougla ABC manja od razlike stranica A B i
A C trougla A BC, bice duz BD manja od duzi BD , pa je BCD < BCD
i prema tome CBD > CBD , tj. CBA > CBA . Neka je M srediste
luka BAC. Iz AB > AC i A B > A C sledi da su tacke A i A na istom luku
CM . Otuda je M BA < M BA , pa je M A < M A , i prema tome odstojanje
tacke A od prave BC vece od odstojanja tacke A od prave BC. Dakle, imamo
da je S(ABC) > S(A BC), i prema tome S(ABC) > S(A BC), i prema tome
S(ABC) > S(A1 B1 C1 ).

680

A
D

Sl. 555
1085. Ako su ABC i A1 B1 C1 dva trougla od kojih je BC = B1 C1 , A =
A1 , a razlika uglova B i C manja od razlike uglova B1 i C1 , dokazati da je
S(ABC) > S(A1 B1 C1 ).
S obzirom da su duzi BC i B1 C1 jednake, s one strane prave BC s koje je
A postoji tacka A takva da je A BC
= A1 B1 C1 (sl. 556).
M

Sl. 556
Iz jednakosti uglova BAC i B1 A1 C1 , zatim uglova B1 A1 C1 i BA C sledi da
su uglovi BAC i BA C medu sobom jednaki; prema tome tacka A pripada
kruznom luku BAC. Ako je M srediste kruznog luka BAC, ugao M BA jednak
je polurazlici uglova B i C trougla ABC, a ugao M BA jednak je polurazlici
uglova B i C trougla A BC. Po pretpostavci je razlika uglova B i C trougla
ABC manja od razlike uglova B i C trougla A BC, pa je M BA < M BA , i
prema tome M A < M A . Otud sleduje da je odstojanje tacke A od prave BC
vece od odstojanja tacke A od prave BC, pa je S(ABC) > S(A BC), i prema
tome
S(ABC) > S(A1 B1 C1 ).
1086. Ako dva trougla ABC i A1 B1 C1 imaju jednake obime i jednake stranice BC i B1 C1 , i ako je razlika uglova B i C manja od razlike uglova B1 i C1 ,
681

dokazati da je
S(ABC) > S(A1 B1 C1 ).
S obzirom da su duzi BC i B1 C1 medu sobom jednake, s one strane prave
BC s koje je A postoji tacka A takva da je A BC
= A1 B1 C1 (sl. 557).
A

F
A
D

Sl. 557
Pretpostavimo da je B < C i B1 < C1 . U tom slucaju bice i B1 <
B. Zaista, ako tako ne bi bilo, iz nejednakosti C B < C1 B1 imali bismo
da je C < C1 . Iz nejednakosti ABC A BC i ACB < A CB sledilo bi
da se tacka A nalazi u trouglu A BC ili na stranici A B, sto je nemoguce, jer
trouglovi ABC i A BC imaju jednake obime. Dakle poluprava BA je u uglu
ABC, prema tome, ona sece stranicu AC u nekoj tacki D. Tacka A nije izmedu
tacaka B i D niti je istovetna s D, jer bi tada obim trougla A BC, dakle i obim
trougla A1 B1 C1 , bio manji od obima trougla ABC, sto je suprotno pretpostavci.
Stoga je tacka A iza D u odnosu na B. U trouglu DBC je B < C, pa je
DC < DB, te izmedu tacaka B i D postoji tacka E takva da je CD = DE. Ako
je F tacka poluprave DA takva da je DA = DF , bice tacka F izmedu tacaka
D i A. Zaista, ako bi tacka F bila iza A u odnosu na D, ili istovetna sa A, imali
bismo, s obzirom na jednakost AB+CDAF = BE+ED+DA +A C. Otuda i
iz CD = DE, DA = DF , CA = EF nalazimo da je AB = AF + F E + EB, sto
je nemoguce. Stoga je S(ABC) = S(BCD)+S(DEF )+S(ABEF ) i S(A BC) =
S(BCD) + S(A CD), te s obzirom na jednakost S(DEF ) = S(A CD) nalazimo
da je S(ABC) > S(A BC), i prema tome da je
S(ABC) > S(A1 B1 C1 ).
1087. Dokazati da od svih trougaonih povrsi koje imaju jednak po jedan
ugao i jednake zbirove stranica koje zahvataju te uglove, najvecu povrsinu ima
ona trougaona povrs kojoj su te dve stranice medu sobom jednake.
Neka je ABC trougao kod koga je AB = AC i A1 B1 C1 trougao kod koga je
A1 B1 > A1 C1 i prema tome A = A1 , a AB + AC = A1 B1 + A1 C1 . Dokazimo
da je S(ABC) > S(A1 B1 C1 ). Iz navedenih pretpostavki neposredno sleduje da
je A1 B1 > AB i A1 C1 < AC. Ako su B i C tacke polupravih AB i AC takve
da je AB = A1 B1 i AC = A1 C1 , bice iza B u odnosu na A, a C izmedu
682

A i C, te se duzi BC i B C seku u nekoj tacki D. Pri tome je S(ABC) =


S(ABDC ) + S(CC D) i S(AB C ) = S(ABDC ) + S(BB D). Kod trouglova
BB D i CC D jednake su stranice BB i CC i naspramni uglovi BDB i CDC .
Sem toga je DBB > DCC i DB B < DC C, pa je razlika uglova C i C
trougla CC D manja od razlike uglova B i B trougla BB D. Stoga je prema
zadatku ... S(CC D) > S(BB D), pa je i S(ABC) > S(AB C ), i prema tome
S(ABC) > S(A1 B1 C1 ). Ovim je stav dokazan (sl. 558).

A1

C1

B1

Sl. 558
1088. Dokazati da od svih trapeza koji imaju jednake obime i jednake odgovarajuce uporedne stranice, najvecu povrsinu ima povrs jednakokrakog trapeza.
Da bi pri zadatim osnovicama povrsina trapezne povrsi (ABCD) bila najveca
potrebno je da njena visina bude najveca. Neka je osnovica AB veca od osnovice
CD, a E tacka stranice AB takva da je AE = CD. Visina CC trapeza ABCD
bice najveca ako je trougao BCE jednakokrak, tj. BC = EC, pa je povrsina
trapeza povrsi (ABCD) najveca kada su kraci BC i AD medu sobom jednaki,
tj. kada je trapez ABCD jednakokrak (sl. 559).

Sl. 559
1089. Dokazati da od svih paralelogramskih povrsi koje imaju jednak po
jedan ugao i koje imaju jednake obime, najvecu povrsinu ima povrs romba.

683

Neka je ABCD romb i A1 B1 C1 D1 bilo koji paralelogram kome je A1 = A


i obim je jednak sa obimom romba (sl. 560).Iz ovih pretpostavki sledi da je
kod trouglova ABC i A1 B1 C1 A = A1 , AB + AD = A1 B1 + A1 D1 zatim
AB = AD i A1 B1 6= A1 D1 , pa je prema zadatku 1087 S(ABD) > S(A1 B1 D1 ).
Pa odatle sledi da je S(ABCD) > S(A1 B1 C1 D1 ).

D1

A1

C1

B1

sl. 560
1090. Dokazati da od svih trougaonih povrsi s jednakim obimima, najvecu
povrsinu ima povrs jednakostranicnog trougla.
Neka je ABC nejednakostranican trougao, a BC njegova najveca stranica,
zatim A1 BC jednakokraki trougao (A1 B = A1 C) kome je obim jednak obimu
trougla ABC. Prema zadatku 1089, bice S(A1 BC) > S(ABC). Obelezimo
sa C1 tacku poluprave BC takvu da je duz BC1 jednaka jednoj trecini obima
trougla A1 BC, a sa A2 tacku poluprave BA1 takvu da je obim trougla A1 BC
jednak obimu trougla A2 BC1 . S obzirom da je duz BC najveca stranica trougla
ABC, a obim trougla ABC jednak obimu jednakokrakog trougla A1 BC, bice
duz BC najveca stranica i trougla A1 BC, pa je ona veca i od jedne trecine obima
tog trougla. Stoga je duz BC veca od duzi BC1 , i prema tome tacka C1 je izmedu
tacaka B i C. Tacka A2 je iza A1 u odnosu na B. Zaista, ako bi tacka A2 bila
izmedu tacaka B i A1 , bilo bi A2 C1 = A2 A1 + A1 C = CC1 , sto je nemoguce,
ako bi tacka A2 bila istovetna sa A1 , bilo bi A2 C1 = A2 C + CC1 , sto je takode
ne moguce. Odatle sledi da su uglovi A1 C1 C i C1 A1 A2 naporedni unutrasnjim
uglovima C1 i A1 trougla A1 C1 B. S obzirom da je duz BC najveca stranica
trougla A1 BC, kome su druge dve stranice jednake, pace stranica A1 B manja
od jedne trecine obima trougla A1 BC, tj. A1 B < BC1 . Stoga je A1 C1 B >
C1 A1 C, zaista ako bi bilo A1 C1 C2 < C1 A1 C, u trouglu A1 C1 C postojala
bi tacka A1 takva da je C1 A1 A2
= A1 C1 A2 , sto je nemoguce, jer ti trouglovi
imaju jednake obime. Istim postupkom iskljucuje se i mogucnost da su uglovi
A1 C1 A2 i C1 A1 C medusobom jednaki. Sobzirom da trouglovi A1 C1 C i C1 A1 A2
imaju zajednicku stranicu A1 C1 , jednake obime, i pri tome razlika uglova C1 i
A1 trougla A1 C1 C veca od razlike uglova A1 C1 trougla C1 A1 A2 prema zadatku
1089, bice S(C1 A1 A2 ) > S(A1 C1 C). Otuda je S(A2 BC1 ) > S(A1 BC1 ). Najzad
neka je A3 BC1 jednako stranican trougao, obim tog trougla jednaka je obimu
trougla A2 BC1 , pa je prema zadatku 1089, S(A3 BC1 ) > S(A2 BC1 ). Otuda
sledi da od trougaonih povrsi s jednakim obimima najvecu povrsinu ima povrs
684

jednakostranicnog trougla (sl. 561).

A3
A2
A1
A

sl. 561
1091. Dokazati da od svih cetvorougaonih povrsi s jednakim obimima,
najvecu povrsinu ima kvadratna povrs.
B

C1

Najmanje jedna od diagonala proizvoljne diagonalne povrsi ABCD je unutrasnja, neka je to diagonala AC (sl. 562). Ta diagonala razlaze diagonalnu
povrs ABCD na dve trougone povrsi ABC i ACD. Neka je B1 tacka sa one
prave AC sa koje je B takva da je AB1 = CB1 i pri tome je obim trougla ABC
jednak obimu trougla AB1 C, a D1 sa one strane prave AC s koje je D takva da
685

je AD1 = CD1 i pri tome je obim trougla ACD jednak obimu trougla ACD1 .
Prema zadatku 1089, bice S(AB1 C) > S(ABC) i S(ABC) > S(ADC). Stoga
je S(AB1 CD1 ) > S(ABCD). Diagonala B1 D1 cetvorougaone povrsi AB1 CD1
je unutrasnja prema tome ona razlaze tu povrs na dve trougaone povrsi AB1 D1 i
CB1 D1 . Neka je A1 tacka sa one strane prave B1 D1 = A1 D1 i pri tome obim trougla AB1 D1 jednak obimu trougla A1 B1 C1 , a C1 tacka sa one strane prave B1 D1
s koje je C takva da je B1 C1 = D1 C1 i pri tome obim trougla CB1 D1 jednaka
obimu trougla C1 B1 D1 . Prema zadatku 1089, bice S(A1 B1 D) > S(AB1 D1 )
i S(C1 B1 D1 ) > S(CB1 D1 ) pa je S(A1 B1 C1 D1 ) > S(AB1 CD1 ) cetvorougao
A1 B1 C1 D1 , je romb kome je obim jednak obimu cetvorougla ABCD. Neka
je A2 B2 C2 D2 kvadrat kome je obim jednak obimu romba A1 B1 C1 D1 . Prema
zadatku 1089, bice S(A1 B1 C1 D1 ) > S(A1 B1 C1 D1 ). Otud sledi da od svih
cetvorougaonih povrsi sa jednakim obimima najvecu povrs ima kvadratna povrs.

D1

A1

C1
B

B1

sl. 562

13
13.1

KONSTRUKTIVNI ZADACI
Metoda geometrijskih mesta

1213.Date su tri prave a, b, c i na pravama a, b tacke A, B. Odrediti na


pravama a i b tacke X i Y takve da je AX = BY , a srediste Z duzi XY na
pravoj c.
Uputstvo. Koristiti prethodni zadatak.
1214. Konstruisati skup temena pravih uglova svih pravouglih trouglova koji
su podudarni s nekim datim pravouglim trouglom Ao Bo Co a kojima ostala dva
686

temena pripadaju raznim kracima datog pravog ugla XOY .


Resenje:
Ako obelezimo sa ABC trougao podudaran s trouglom A0 B0 C0 , kome
temena ostrih uglova A i B pripadaju kracima OX i OY pravog ugla XOY ,
bice kod cetvorougla OACB naspramni uglovi O i C pravi, pa je isti upisan
u nekom krugu k. Pri tom su tacke O i C na raznim lucima AB kruga ili pak
na jednom od tih dvaju lukova.
Ako su tacke O i C na raznim lucima AB kruga k, bice BOC = BAC
= B0 A0 C0 , Pa je tacka C na fiksiranoj polupravi OM koja se nalazi u uglu
XOY i koja sa polupravom OY zahvata konstantan ugao.
S obzirom da je duz OC tetiva kruga k, ona ne moze biti veca od precnika
AB toga kruga, pa je OC ON = AB = A0 B0 . Ako je AC BC i pri tome
tacka A poklapa s tackom O, tacka C se poklapa s nekom tackom M pomenute
poluprave, pa je OC OM = AC. Na taj nacin, skup tacaka C u analiziranom
slucaju pripada duzi MN.
Obratno, ako je C bilo koja tacka duzi MN, odstojanje tacke C od kraka OX
manja je ili jednaka duzi A0 C0 . Stoga krug l(C, C0 A0 ) sece pravu OX u dvema
tackama. Kako je COX ostar i OC A0 C0 , obe presecene tacke pripadaju
polupravoj OX, neka je A ona od tih tacaka koja je bliza tacki O. Ako je B
tacka sa one strane prave AC s koje je tacka Y takva da je ABC podudaran
A0 B0 C0 , bice zbog jednakosti uglova AOC i ABC cetvorougao ACBO
tetivan a duz AB precnik opisanog kruga, pa je i ugao AOB prav. Stoga je
tacka B na polupravoj OY, te je ABC trougao koji zadovoljava date uslove.
Stoga skup svih tacaka C u analiziranom slucaju predstavlja duz MN (sl. 608).

N
X
C

Y
O

Slika 608
Slucaj kada su obe tacke O i C na istom luku AB kruga k ostavljamo da
citalac analizira sam.
1215.Date su tri prave a, b, c od kojih su prve dve upravne medju sobom.
Odrediti na pravama a, b, c tacke A, B, C takve da trougao ABC bude podudaran s datim trouglom Ao Bo Co kome je ugao Co prav.
687

Uputstvo. Koristiti prethodni zadatak.


1216. Dat je ugao AOB i duz l. Ako su P i Q promenljive tacke polupravih
OA i OB takve da je P Q = l, konstruisati skup svih tachaka R u kojima se
seku upravne na polupravama OA i OB u tackama P i Q.
Uputstvo. S obzirom da su uglovi OP R i OQR pravi, cetvorougao OPRQ
je tetivan. Sem toga, ugao P OQ tog cetvorougla je fiksiran, a duz PQ jednaka
konstantnoj duzi l, pa je precnik OR kruga kruga opisanog oko tog promenjivog
cetvorougla konstantan. Stoga tacka R pripada izvesnom krugu kome je srediste
C, a poluprecnik jednak precniku kruga opisanog oko bilo kojeg cetvorougla
OPRQ. (sl. 609)

B
Q
R

Slika 609
1217.Date su tri razne prave a, b, c i duz l. Odrediti na pravoj c tacku C
takvu da podnozja A i B upravnih iz tacke na pravama a i b odredjuju duz
jednaku duzi l.
Uputstvo. Koristiti prethodni zadatak.
1218. Dati su krug l i na njemu dve razne tacke B, C. Konstruisati skup
ortocentara svih trouglova kojima su dva temena tacke B i C, a trece teme
promenljiva tacka A kruga l.
Uputstvo. Ako je A proizvoljna tacka kruga l razlicita od tacaka B i C,
H ortocentar trougla ABC i H tacka u kojoj prava AH sece krug l, prema
zadatku bice tacka H simetricna s tackom H u odnosu na pravu BC. Stoga
tacka H pripada krugu l koji je simetrican s krugom l u odnosu na pravu BC.
Obratno, ako je H proizvoljna tacka kruga l , H njena simetricna tacka u
odnosu na pravu BC i A tacka razlicita od tacke H u kojoj prava HH sece
krug l, bice zbog simetricnosti krugova l i l u odnosu na pravu BC tacka H
na krugu l. Stoga je tecka H ortocentar trougla ABC.
Otuda sleduje da skup ortocentara svih trouglova, kojima su dva temena
tacke B i C a trece teme promenljiva tacka A kruga l,predstavlja krug l koji
je simetrican s krugom l u odnosu na pravu BC. (sl. 610).

688

A
l

H
B

H
l

O1

Slika 610
1219.Dati su krug l, prava p i na krugu k dve razne tacke B, C. Odrediti na
krugu l tacku A takvu da ortocentar trougla ABC bude na pravoj p.
Uputstvo. Koristiti zadatak.
1222. Dati su krug k, na njemu dve fiksirane tacke A, B i promenljiva tacka
P . Konstruisati skup podnozja svih upravnih iz sredista duzi AP na pravoj
BP .
Uputstvo. Ako obelezimo sa Q srediste duzi AP , sa R podnozje upravne iz
tacke Q na pravoj BP , sa C tacku kruga k dijametralno suprotno s tackom B i
sa D tacku u kojoj prava QR sece pravu AC, bice QR||CP , a QD srednja linija
trougla ACP . Kako je tacka C fiksirana i tacka D je fiksirana, te krak RQ u
uglu BRQ sadrzi tacku D. Stoga tacka R pripada krugu l kome je duz BD
precnik. (sl. 611)
P
R
k
C

Slika 611
689

1223. Date su u jednoj ravni dve prave l1 i l2 koje nisu paralelne niti upravne
medju sobom i izvan tih pravih tacka H. Konstruisati skup svih tacaka P kojima
simetricne tacke P 1 i P 2 u odnosu na prave l1 i l2 odredjuju pravu koja sadrzi
tacku H.
Uputstvo:Obelezimo sa B tacku u kojoj se seku prave l1 i l2 , sa H2 tacku
simetricnu sa tackom H u odnosu na pravu l2 , sa A tacku u kojoj prava HH2 sece
pravu l1 , zatim dokazimo da tacke A, B, H2 , P pripadaju jednom krugu. Tacke
P i H mogu imati razlicite polozaje u odnosu na prave l1 i l2 . Analizirajmo
slucaj kad su obe te tacke P i H u istom uglu koji je odreden pravama l1
i l2 . Sem toga pretpostavimo da se podnozja Q1 i Q2 upravnih iz tacke na
pravama l1 i l2 nalaze na kracima toga ugla. Pri uvedenim pretpostavkama
bice cetvorougao P Q1 BQ2 tetivan zatim P1 P2 k Q1 Q2 i P P2 k HH2 , pa je
ABP = Q1 BP = Q1 Q2 P = P1 P2 P = P2 HH2 = HH2 P = AH2 P .
Stoga su tacke A, B, H2 , P na izvesnom krugu l. Analogno se dokazuju i ostali
slucajevi. S obzirom da su tacke A, b, H2 fiksirane, skup svih tacaka P pripada
krugu l koji je odreden tim trima tackama. Ovim je izveden potreban uslov.
Dovoljan uslov ostavljamo da citalac izvede sam(sl. 612).

P1
l1
A Q1

Q2 C

l2

H2

P2

sl. 612
1224.Dati su krug k i prava s koja sadrzi njegovo srediste O. Konstruisati
skup svih tacaka u kojima tangente kruga k dodiruju promenljivi krug l koji
sadrzi tacku O, a srediste mu se nalazi na pravoj s.
Uputstvo. Obelezimo sa K i L tacke u kojima jedna prava dodiruje krugove
k i l, i sa P podnozje upravne iz tacke L na pravoj s. S obzirom da prava s
sadrzi srediste S kruga l, ona ima s krugom l sem tacke O jos jednu zajednicku
690

tacku R. Pri tome su uslovi KLS i OLR jednaki i istosmerni, te su i uglovi


KLO i SLR jednaki. Trougao SLR je jednakokrak pa su uglovi SLR i LRS
nalegli na njegovoj osnovici RL medu sobom jednaki. Kod trouglova RLO i
LP O je RLO = LP O i ROL = LOP , te je i LRO = OLP , tj.
LOS = OLP . Stoga je KLO = OLP , te je prava LP simetricna sa
pravom LK u odnosu na pravu LO. Po pretpostavci je prava LK dirka kruga
k, pa je i prava LP dirka kruga. S obzirom da je prava LP dirka kruga k, a ugao
LP O prav, dodirna tacka prave LP sa krugom k je tacka. Ovim smo dokazali
da se tacke L nalaze na dirkama p i q kruga k upravnim na pravoj s. Obratno,
ako je L proizvoljna tacka jedne od pravih p i q, npr. prave p, i P tacka u kojoj
se seku prave s i p bice ugao LOP ostar, te normala u tacki L na kraku OL sece
drugi krak OP tog ostrog ugla u nekoj tacki R. S obzirom da je ugao OLR prav,
srediste S kruga l opisanog oko trougla OLR je na pravoj OR, tj. pravoj s. Ako
je d druga dirka k kroz tacku L i K njena dodirna tacka, bice KLO = OLP ,
zatim je OLP = LRS, LRS = SLR, te je KLO = SLR. Ti uglovi su
i istosmerni, pa je KLS = OLR. No ugao OLR je prav, te je i ugao KLS
prav. Stoga prava KL dodiruje i krug l u tacki L. Ovim je stav dokazan(sl.
613).

L
K

k
l
q

sl. 613
1225.Data su dva komplanarna kruga k i l koji se seku u dvema tackama
A i B. Konstruisati skup ortocentara svih trouglova P QR kojima je teme P
promenljiva tacka kruga k, dok su temena Q i R tacke u kojima prave P A i P B
seku krug l.

691

Uputstvo. Razmotrimo najpre trouglove P QR kojima je teme P izvan kruga


l. Ako obelezimo sa P1 , Q1 , R1 podnozja visina trouglova P QR, sa P2 tacku u
kojoj prava P P1 sece krug k, sa Q2 i R2 tacke u kojima prave QQ1 i RR1
seku krug l i sa T prav ugao, bice P AP2 = 2 T P P2 A AP P2 =
2T P BA(T P QR) = 2T P QRT +P QR = T . Stoga je duzP P2
precnik kruga k, te sve prave P P1 sadrze fiksiranu tacku, srediste O kruga k.
Iz jednakosti P QQ2 = P QQ1 = P RR1 = P RR2 = T QP R = const
sledi da prave QQ1 sadrze fiksiranu tacku Q2 , a prave RR1 sadrze fiksiranu
tacku R2 . Dokazimo zatim da se prave AR2 i BQ2 seku u tackiO. Kako je
OAB = T AP B = P RR2 a cetvorougao ABRR2 tetivan, bice ugao OAB
spoljasnji kod temena A, i preme tome tacka O na pravoj AR2 . Isto tako tacka
O je na pravoj BQ2 . S obzirom da su prave AP2 i R2 H upravne na istoj pravoj
P A, one su paralelne, pa je OR2 H = OAP2 = OP A2 = OHR2 , i prema
tome OH = OR2 . Isto tako je OH = OQ2 . Iz dobijenih dveju jednakosti sleduje
da tacke H, Q2 , R2 pripadaju izvesnom krugu k koji je koncentrican sa krugom
k. Analognim postupkom dokzuje se da ortocentri trouglova P QR kojima je
teme P u krugu l(sl. 614)

P
k

R2

P2

Q2

H
Q

P1

sl. 614
1226. Neka je A fiksirana tacka nekog kruga k, B promenljiva tacka tog
kruga i ABCD kvadrat na duzi AB.
(a) Konstruisati skup temena C svih kvadrata ABCD;
(b) Konstruisati skup temena D svih kvadrata ABCD.
Uputstvo. (a) Skup temena C svih kvadrata ABCD predstavljaju dva kruga
c i c koji sadrze tacku A, sredista im se nalaze na krugu k, a poluprecnici su
692

im jednaki stranici kvadrata upisanog


u krugu
k. Krugovi c i c predstavljaju


likove koji u obrnutoj slicnosti RS A, , 2 odgovaraju krugu k.
4
(b) Skup temena D svih kvadrata ABCD predstavljaju dva kruga d i d koji
u tacki A ortogonalno seku krug k, a poluprecnici su im jednaki
polupre
cnikom
 s


kruga k. Krugovi d i d predstavljaju likove koji u rotaciji R A,


odgovaraju
2
krugu k.
1323. Date su tri nekolinearne tacke Sa , Sb , Sc . Konstruisati trougao ABC
kome su tacke Sa , Sb , Sc sredista spolja upisanih krugova.
Uputstvo. Podnozja visina trougla Sa Sb Sc su temena trougla ABC.
1324.Date su tri nekolinearne tacke S, Sb , Sc . Konstruisati trougao ABC
kome je tacka S srediste upisanog kruga, tacka Sb srediste spolja upisanog kruga
koji dodiruje stanicu AC, a tacka Sc srediste spolja upisanog kruga koji dodiruje
stranicu AB.
Uputstvo. Podnozja visina trougla SSb Sc su temena trougla ABC.
1325.Konstruisati trougao ABC kada su date tacke K, L, N , u kojima
njegov opisani krug sece prave odredjene visinom AD, tezisnom linijom AA1 i
simetralom AE unutrasnjeg ugla A.
Uputstvo. Ako obelezimo sa O srediste kruga opisanog oko trougla ABC,
bice ON k AK i A1 presek pravih AL i ON (sl. 615).
M

E
K

A1

EA1

Sl. 616.

Sl. 615.

1326.Konstruisati trougao ABC kada su date tacke K, L, N , u kojima


njegov opisani krug sece prave odredjene visinom AD, tezisnom linijom AA1 i
simetralom AF spoljasnjeg ugla A.
Uputstvo. Ako obelezimo sa O srediste kruga opisanog oko trougla ABC,
bice OM k AK i A1 presecna tacka pravih OM i AL.
1327. Date su tri kolinearne tacke D, E, A1 . Konstruisati trougao ABC
kome je tacka D podnozje visine iz temena A, tacka E presek simetrale ugla A
sa stranicom BC, A1 srediste stranice BC i kome je duz koja spaja teme A sa
ortocentrom H jednaka datoj duzi d.
Uputstvo. Ako je O srediste kruga opisanog oko trougla ABC, tada je duz
OA1 istosmerna i jednaka polovini duzi AH, a prava AE simetrala ugla OAH.
Stoga mozemo konstruisati najpre tacku O, zatim tangentu OA iz tacke O na
krugu m(E, ED) i njen presek A sa pravom koja je u tacki D upravna na pravoj
DE (sl. 616).
1328.Date su tri razne tacke D, E, O. Konstruisati trougao ABC kome
je tacka D podnozje visine iz temena A, tacka E presek simetrale ugla A sa

693

stranicom BC i tacka O srediste opisanog kruga.


Uputstvo. Ako obelezimo sa N tacku u kojoj raspolovnica ugla A sece opisani krug, bice podnozje K upravne iz tacke O na pravoj AE srediste duzi AN .
Stoga se tacka K nalazi na krugu k kome je duz OE precnik i na pravoj a koja
je jednako udaljena od paralelnih pravih AD i ON (sl. 617).
M
A

O
K

A1

s N

Sl. 617.
1329.Date su tri tacke A, A1 , H. Konstruisati tacke B i C takve da ABC
bude trougao kome je tacka H ortocentar, a tacka A1 srediste stranice BC.
Uputstvo. Ako obelezimo sa D podnozje visine iz temena A i sa O srediste
kruga opisanog oko trougla ABC, tada je AH DA1 , OA1 DA1 i OA1 =
1
AH. Stoga se mogu konstruisati najpre tacke D i O, zatim zajednicke tacke
2
prave DA1 sa krugom l(O, OA).
1330.Dat je krug l i u njegovoj ravni tacka H. U krug l upisati trougao ABC
kome je ortocentar tacka H, a stranica BC jednaka datoj duzi a.
Uputstvo. Duz koja spaja srediste O kruga l opisanog oko trougla ABC sa
sredistem A1 stranice BC istosmerna je sa duzi AH i jednaka njenoj polovini.
1331.Konstruisati trougao ABC kome je stranica BC jednaka datoj duzi a,
duz koja spaja teme A sa ortocentrom jednaka duzi d, a poluprecnik upisanog
kruga jednak datoj duzi .
Uputstvo. Ako je O srediste kruga opisanog oko trougla ABC, A1 srediste
stranice BC i H ortocentar tog trougla, tada je
OA1 =

1
AH
2

i prema tome

694

OA1 =

1
d.
2

S
A1
F

C
Pa

Sa

Sl. 618.
1332. Konstruisati trougao ABC kome je stranica BC jednaka datoj duzi
a,zbir ili razlika stranica AB i AC jednaka datoj duzi s, a duz koja spaja teme
A sa ortocentrom H jednaka datoj duzi d.
Uputstvo. Koristiti uputstvo iz prethodnog zadatka.
1333. Dat je krug l i u njegovoj ravni tacka T . Upisati u krug l trougao
ABC kome je tacka T teziste, a razlika uglova B i C jednaka datom uglu .
Uputstvo. Ako obelezimo sa O srediste kruga l opisanog oko trougla ABC i
sa H ortocentar tog trougla, tada su tacke O, T , H kolinearne i pri tome tacka
T izmedju tacaka O i H takva da je T H = 2OT . Sem toga je ugao OAH jednak
ili suplementan s uglom .
1334. Date su tri nekolinearne tacke A1 , S, E. Konstruisati trougao ABC
kome je tacka A1 srediste stranice BC, tacka S srediste upisanog kruga, a E
tacka u kojoj simetrala unutrasnjeg ugla A sece stranicu BC.
Uputstvo. Ako obelezimo sa P i Pa tacke u kojima upisani krug k i spolja
upisani ka trougla ABC dodiruju stranicu BC, tada je tacka A1 srediste duzi
P Pa i SA1 k APa (sl. 618).
1335.VIDI TEKST
Uputstvo. Ako obelezimo sa P i Pa tacke u kojima upisani krug k i spolja
upisani krug ka trougla ABC dodiruju stranicu BC, bice tacka A1 srediste duzi
P Pa i A1 Sa k AP .
1370. (*) Konstruisati skup sredista svih krugova koje sece svaki od dvaju
datih ekscentricnih krugova k1 i k2 u dijametralno suprotnim tackama.
Uputstvo. Ako bi postojao krug k(O, r) kojeg sece svaki od krugova k1 (O1 , r1 )
i k2 (O2 , r2 ) u dijametralno suprotnim tackama, njegovo srediste O mora biti
u svakom od tih krugova k1 i k2 . Sem toga, ako obelezimo sa A1 bilo koju

695

od presecnih tacaka krugova k i k1 , a sa A2 bilo koju od presecnih tacaka


krugova k i k2 bice kod pravouglih trouglova OO1 A1 i OO2 A2 katete OA1
i OA2 jednake, pa je O1 A1 2 OO1 2 = O2 A2 2 OO2 2 , pa je pri r1 > r2
OO1 2 OO2 2 = r1 2 r2 2 = l2 . Na taj nacin, tacka O pripada onom delu
radikalne ose krugova k1 i k2 koji se nalazi u tim krugovima k1 i k2 , tj. unutar
zajednicke tetive M N krugova k1 i k2 (Sl. 627).

k1

A1
k

A2

k2

O
O1

O2

B2

B1
N

Sl 627.
1371. (*) Konstruisati skup sredista svih krugova koji seku svaki od dvaju
datih ekscentricnih krugova k1 i k2 u dijametralno suprotnim tackama.
Uputstvo. Ako obelezimo sa k(O, r) proizvoljan krug koji sece svaki od krugova k1 (O1 , r1 ) i k2 (O2 , r2 ) u dijametralno suprotnim tackama, sa A1 bilo koju
od presecnih tacaka krugova k i k1 , a sa A2 bilo koju od presecnih tacaka krugova k i k2 bice kod pravouglih trouglova OO1 A1 i OO2 A2 hipotenuze OA1 i
OA2 jednake, pa je OO1 2 + O1 A1 2 = OO2 2 + O2 A2 2 ,i prema tome pri r1 > r2
OO2 2 OO1 2 = r1 2 r2 2 = l2 . Na taj nacin, tacka O pripada pseudoradikalnoj
osi krugova k1 i k2 , tj. pravoj koja je simetricna s radikalnom osom krugova k1
i k2 u odnosu na srediste duzi O1 O2 (Sl. 628).

696

O
A1
A2
k1

k2
O1

O2

B2
B1
p

Sl 628.
1372. (*) Konstruisati skup sredista svih krugova koji seku dati krug k1 pod
pravim uglovima i dati krug k2 u dijametralno suprotnim tackama.
Uputstvo. Ako obelezimo sa k(O, r) proizvoljan krug koji sece krug k1 (O1 , r1 )
pod pravim uglovima i krug k2 (O2 , r2 ) u dijametralno suprotnim tackama, njegovo srediste O je izvan kruga k1 . Sem toga, ako obelezimo sa A1 bilo koju od
tacaka u kojoj krug k sece krug k1 i sa A2 bilo koju od tacaka u kojoj krug k sece
krug k2 , bice kateta OA1 pravouglog trougla OO1 A1 jednaka sa hipotenuzom
OA2 pravouglog trougla OO2 A2 , pa je
OO1 2 O1 A1 2 = OO2 2 O2 A2 2 , i prema tome OO1 2 OO2 2 = r1 2 + r2 2 = l2 .
Na taj nacin, tacka O se nalazi izvan kruga k1 a na izvesnoj pravoj p koja je u
nekoj tacki P upravna na pravoj O1 O2 (Sl. 629).

697

O
A1
k1

A2
k2
O2

O1

B2

B1
s

Sl 629.
1373. (*) Konstruisati skup sredista svih krugova koje sece dati krug k1 pod
pravim uglovima i dati krug k2 u dijametralno suprotnim tackama.
Uputstvo. Ako obelezimo sa k(O, r) bilo koji krug kojeg sece krug k1 (O1 , r1 )
pod pravim uglovima i krug k2 (O2 , r2 ) u dijametralnim tackama, bice srediste O
tog kruga izvan kruga k1 i u krugu k2 . Sem toga, ako obelezimo sa A1 jednu od
tacaka u kojima krug k1 sece krug k i sa A2 jednu od tacaka u kojima krug k2 sece
krug k, bice katete OA1 i OA2 pravouglih trouglova OO1 A1 i OO2 A2 jednake, pa
2
je OO1 2 O1 A1 2 = O2 A2 2 OO2 , i prema tome OO1 2 +OO2 2 = r1 2 +r2 2 = l2 .
Stoga, prema zadatku 1346 , tacka O pripada izvesnom krugu
r p kome je srediste
1 2 1 2
istovetno sa sredistem duzi O1 O2 , a poluprecnik r =
(l d ), gde je
2
2
d = O1 O2 . Na taj nacin, srediste O kruga k pripada onom delu kruga p koji se
nalazi izvan kruga k1 i u krugu k2 (Sl. 630).

698

A2

k2

A1
O

k1
O1

O2
S
p
B1
B2

Sl 630.
1374. (*) Konstruisati skup sredista svih krugova koji seku dati krug k1 u
dijametralno suprotnim tackama i koje sece dati krug k2 u dijametralno suprotnim tackama.
Uputstvo. Ako obelezimo sa k(O, r) bilo koji krug koji sece krug k1 (O1 , r1 )
u dijametralno suprotnim tackama i kojeg sece krug k2 (O2 , r2 ) u dijametralno
suprotnim tackama, bice tacka O u krugu k2 . Sem toga, ako obelezimo sa A1
jednu od tacaka u kojima krug k sece krug k1 i sa A2 jednu od tacaka u kojima
krug k2 sece krug k, bice hipotenuza OA1 pravouglog trougla OO1 A1 jednaka
kateti OA2 pravouglog trougla OO2 A2 , pa je OO12 + O1 A21 = O2 A22 OO22 , i
prema tome za r2 > r1 , OO12 + OO22 = r22 r12 = l2 . Stoga, prema zadatku 1373,
tacka O pripada izvesnomskrugu p cije je srediste istovetno sa sredistem duzi


1
1
l2 d2 , gde je d = O1 O2 . Na taj nacin,
O1 O2 , a poluprecnik r =
2
2
srediste O kruga k pripada onom delu kruga p koji se nalazi u krugu k2 (slika
631).

699

A1 A2

Slika 631
Okrug
1
k1
1381.(*) Konstruisati
kojiOima polupre
cnik jednak duzi r i koji pripada
O2
pramenu krugova odredenim sa dva data kruga k1 i k2 .
B1
Uputstvo. Ako je P proizvoljna
tacka radikalne ose datog pramena krugova,
potencija tacke P u odnosu na krug
k bice jednaka potenciji tacke P u odnosu
B2
na bilo koji od krugova k1 i k2 . S obzirom da su
cke P u
k2 poznati potencija ta
odnosu na krug k i poluprecnik kruga k, poznata je duz koja spaja tacku P sa
sredistem S kruga k.
1382.(*) Konstruisati krug k koji pripada pramenu krugova odredenim sa
dva data kruga k1 , k2 i koji dodiruje treci dati krug l.

Uputstvo. Obelezimo sa P tacku u kojoj radikalna osa kruga l i nekog kruga


datog pramena sece radikalnu osu datog pramena i sa M , M dodirne tacke tangenti iz P na krugu l. Ako prave odredene poluprecnicima OM i OM kruga l
seku pravu odredenu sredistima krugova k1 i k2 u tackama S i S , bice k(S, SM )
i k (S , S M ) trazeni krugovi.
1383.(*) Konstruisati krug k koji pripada pramenu krugova odredenim sa
dva data kruga k1 i k2 , a ortogonalan je na datom krugu l koji ne pripada tom
pramenu krugova.
700

Uputstvo. Ako obelezimo sa l bilo koji krug ortogonalan na krugovima k1 i


k2 , tacka S u kojoj radikalna osa krugova l i l sece pravu odredenu sredistima
S1 i S2 krugova k1 i k2 bice srediste trazenog trougla k. Ako krug l ne pripada pramenu krugova koji je spregnut sa datim pramenom, problem ima jedno
resenje ili uopste nema resenja zavisno od toga da li je tacka S izvan kruga l ili
nije. Ako krug l pripada pramenu krugova koji je spregnut sa datim pramenom,
tada je svaki krug datog pramena ortogonalan na krugu l.
1384.(*) Konstruisati skup svih tacaka u kojima je razlika potencija u odnosu na dva data ekscentricna kruga k1 i k2 jednaka kvadratu date duzi l.
Uputstvo. Obelezimo sa S1 i S2 sredista, sa r1 i r2 poluprecnike i sa s radikalnu osu krugova k1 i k2 . Ako zatim obelezimo sa P bilo koju tacku trazenog
skupa, sa Q i R podnozja upravnih iz tacke P na pravama s i S1 S2 , sa p1 i p2
potencije tacke P u odnosu na krugove k1 i k2 , a sa S presek pravih s i S1 S2 ,
bice
p1 p2

(S1 P 2 r12 ) (S2 P 2 r22 ) = (S1 P 2 S2 P 2 ) (r12 r22 )

(S1 R2 S2 R2 ) (S1 S 2 S2 S 2 )
(S1 R + RS2 )(S1 R RS2 ) (S1 S + SS2 )(S1 S SS2 )

=
=
=
=

S1 S2 (S1 R RS2 ) S1 S2 (S1 S SS2 )


S1 S2 (S1 R S1 S) + S1 S2 (SS2 RS2 ) = S1 S2 (SR + SR)

2 S1 S2 SR = 2 S1 S2 P Q = l2 .

Stoga tacka P pripada izvesnoj pravoj p koja je paralelna s pravom s i na


odstojanju
l2
od nje.
PQ =
2 S1 S2
Ako izvedemo konstrukciju prave p, mozemo reverzibilnim postupkom dokazati da svaka njena tacka pripada trazenom skupu. Ovaj deo dokaza ostavljamo
da citalac izvede sam (slika 632).

k1

k2

S1

S2

Slika 632
1385. Na datoj pravoj p odrediti tacku X kojoj je razlika potencija u odnosu
na dva data kruga k1 i k2 kvadratu date duzi a.

701

Uputstvo. Koristiti prethodni zadatak.


1389. a, a , a : .
1390. b, c, a : .
1391. b c, ha , a : .
1392. b c, la , a : .
1393. b c, ma , a : .
1394. b c, B C, a : .
1395. a, la , b : c .
1396. b, c, b : c .
1397. b + c, ha , b : c .
1398. b + c, la , b : c .
1399. b + c, ma , b : c .
1400. b + c, B C, b : c .
13891400. Uputstvo. Svaki od ovih zadataka resava se neposrednom primenom
zadataka 12261322.
1401. la , B C, b : c.
1402. ha , B C, b : c.
1403. ha , la , b : c.
1404. ha , a, a : .
1405. ha , p, a : .
1406. ha , b + c, a : .
1407. ha , ma , a : .
1408. ha , la , a : .
1409. ha , r, a : .
1410. ha , a, b : c .
1411. ha , p, b : c .
1412. ha , b c, b : c .
1413. ha , ma , b : c .
1414. ha , la , b : c .
1415. ha , r, b : c .
14011415. Uputstvo. Svaki od ovih zadataka moze se resiti najpre primenom zadataka 12261322, zatim zadataka 13891400. Medutim, prva tri od
ovih zadataka, dakle zadaci 1401, 1402, 1403 mogu se resiti koristeci sledeca
svojstva:
Ako kod trougla ABC obelezimo sa D podnozje visine iz temena A, sa E
tacku u kojoj simetrala ugla A sece stranisu BC, sa X tacku u kojoj prava kroz
B paralelna sa AC sece AB, a sa Y i Z tacke u kojima prava kroz X paralelna
sa BC sece AC i AB, bice
AX = BX = CY

AZ
AY
AY
b
=
=
= .
ZB
YC
AX
c

Stoga se u svakom od pomenuta tri zadatka posle konstrukcije trougla ADE


moze konstruisati tacka X, zatim trougao ABC.
1416.(*) Date su na nekoj pravoj p tri razne tacke A, B, C. Odrediti na
pravoj p tacku D takvu da je H(A, B; C, D).
Uputstvo. Ako obelezimo sa M i N tacke u kojima dve paralelne prave kroz
tacke A i B seku proizvoljnu pravu koja sadrzi tacku C, sa N tacku simetricnu

702

s tackom N u odnosu na tacku B i sa D tacku u kojoj se seku prave M N i p,


bice H(A, B; C, D).

M
N

Slika 633
A

Konstruisati trougao ABC kada znamo:


N

1417.(*) ha , , a.

Resenje. Pretpostavimo da postoji ABC kome je stranica BC jednaka datoj


duzi a, visina AD iz temena A jednaka duzi ha , a poluprecnik upisanog kruga
k jednak datoj duzi .

703

A1

M1

Q
R

S1

N1
D1

L1

E
D

Pa

Qa
Ra
Sa

Sa

ka
S2

Slika 634
Ako obelezimo sa E tacku u kojoj simetrale ugla A cece stranice BC, da S i
Sa sredista upisanih krugova k i ka tog trougla, sa S i Sa podnozja upravnih iz
tacaka S i Sa na pravoj AD a sa R i Ra tacke u kojima krugovi k i ka dodiruju
pravu AB, bice H(A, E; S, Sa ) i RRa = a.
Prva od ovih dveju osobina sleduje otuda sto su A, B, S , Sa upravne projekcije na pravoj AD tacaka A, E, S, Sa za koje vazi relacija H(A, D; S , Sa ).
Da bismo dokazali drugu od navedenih dveju osobina, obelezimo sa P i
Pa tacke u kojima krugovi k i ka dodiruju stranicu BC, sa Q i Qa tacke u
kojima krugovi k i ka dodiruju pravu AC. Pri tome je RRa = ARa AR i
QQa = AQa AQ, zatim ARa = AQa i AR = AQ, pa je RRa = QQa . Stoga je
RRa = 12 (RRa + QQa ) = 12 (RB + BRa + QC + CQa ) = 12 (BR + BPa + P C +
Pa C) = BC = a.
Navedene osobine omogucavaju da iz datih elemenata konstkuisemo trougao
ABC.
Obelezimo sa A1 i D1 tacke proizvoljne prave p1 takve da je A1 D1 = ha i sa
S1 tacku poluprave D1 A1 takvu da je D1 S1 = . Ako pretpostavimo da su tacke
A1 i S1 razlicite, tada na pravoj P1 postoji jedna i samo jedna tacka S2 takva da
je H(A1 , D1 ; S1 , S2 ). Pri tome je jedna od tacaka S1 i S2 izmedu tacaka A1 i D1 ,
dok je druga od tih tacaka iza tacke D1 u odnosu na tacku A1 ili iza tacke A1

704

u odnosu na tacku D1 . Neka je tacka S1 izmedu tacaka A1 i D1 , a tacka S2 iza


tacke D1 u odnosu na tacku A1 . Neka su R i Ra tacke proizvoljne prave m takve
da je RRa = a, zatim RS i Ra Sa prave upravne na pravoj m, a S i Sa tacke na
tim pravama s iste strane od prave m takve da je RS = D1 S1 i Ra Sa = D1 S2 .
Iz ovih pretpostavki neposredno sleduje da je prava m zajednicka spoljasnja
tangenta krugova k(S, SR) i ka (Sa , Sa Ra). S obzirom da je H(A1 , D1 ; S1 , S2 )
bic A1 S1 : S1 D1 = A1 S2 : S2 D1 , pa je i S2 D1 : D1 S1 = S2 A1 + A1 S1 . No
S2 A1 > S1 A1 , pa je i S2 D1 > S1 D1 . Otuda je i Sa Ra > SR, te prava SSa
cece pravu m u nekoj tacki A koja se nalazi iza tacke R u odnosu na tacku
Ra . Tacka A je pri tome spoljasnje srediste slicnosti krugova k i ka , te i druga
zajednicka spoljasnja tangenta tih krugova sadrzi tacku A. Obelezimo sa Q i
Qa dodirne tacke te druge spoljasnje tangente sa krugovima k i ka . Krugovi
k i ka mogu imati ili nemati zajednickih unutrasnjih tangenta. Pretpostavimo
da krugovi k i ka imaju zajednickih unutrasnjih tangenti, neka je a bilo koja
od njih. Obelezimo sa P i Pa tacke u kojima ta tangenta dodiruje krugove
k i ka , a sa B i C tacke u kojima ona sece duzi RRa i QQa . S obzirom da
je tacka A spoljasnje srediste slicnosti krugova k i ka , a prava BC zajednicka
unutrasnja tangenta tih krugova, tacka A je van prave BC, pa su tacke A, B, C
temena izvesnog trogula ABC. Dokazimo da taj trougao predstavlja resenje
postavljenog problema.
Istim postupkom kao u analizi ovog problema dokazuje se da je BC = RRa .
No RRa = a, pa je BC = a.
S obzirom da krug k(S, SR) dodiruje sve tri stranice trougla ABC, on je
upisan u taj trougao. Pri tome je SR = S1 D1 i S1 D1 = , pa je i poluprecnik
SR kruga k jednak duzi .
Da bi smo dokazali da je visina AD trougla ABC jednaka duzi ha , obelezimo
sa E tacku u kojoj simetrala SSa ugla A sece stranicu BC, a S i Sa podnozja
upravnih iz tacaka S i Sa na pravoj AD. Kako su S i Sa sredista upisanih
krugova k i ka trougla ABC, bice H(A, E; S, Sa ). No tacke A, D, S , Sa su
upravne projekcije tacaka A, E, S, Sa na pravoj AD, pa je i H(A, D; S , Sa ). S
obzirom da su A1 , D1 , S1 , S2 i A, D, S , Sa dve cetvorke harmonijskih spregnutih
i na isti nacin rasporedenih tacaka, zatim da je S1 D1 = SR = SP = S D i
D1 S2 = Ra Sa = DSa , bice i A1 D1 = AD. No A1 D1 = ha , pa je i AD = ha .
Pretpostavljajuci da je tacka S2 iza tacke D1 u odnosu na tacku A, tj. da je
< 12 ha , problem ima dva jedno i nijedno resenje zavisno od toga da li krugovi
k i ka imaju dve, jednu ili nemaju zajednickih unutrasnjih tangenta.
1450. Date su dve razne tacke A, B i duz l. Odrediti na pravoj AB tacke
C i D takve da je H(A, B : C, D) i CD = l.
Uputstvo.
Prvi nacin. Obelezimo sa k proizvoljan krug koji sadrzi tacke A, B i sa
P , Q sredista lukova AB kruga k. Kroz jednu od tacaka P i Q, npr. tacku
Q, konstruisimo pravu paralelnu sa pravom AB i na njoj s bilo koje strane od
tacke Q odredimo tacku R takvu da je QR = l. Pri tome su tacke P, Q, R
nekolinearne, te odredjuju izvestan krug k . Taj krug sadrzi tacke P i Q koje se
nalaze sa raznih strana prave AB, te sece pravu AB u dvema tackama; Neka je
D bilo koja od njih. Pri tome prava RD sece pravu AB, te i njoj paralelna prava
kroz tacku Q sece pravu AB u nekoj tacki C. Dokazimo da je H(A, B; C, D) i
CD = l.
S obzirom da je P QAB i QR k AB, bice P QR prav, pa je duz P R precnik
705

kruga k . Tacka D je dakle na krugu k kome je duz P R precnik, pa je RDP D,


i prema tome QCP D u nekoj tacki K. Kako je duz P Q precnik kruga k, a
ugao P KQ prav, tacka K je na krugu k. Pri tome je jedna od pravih KP i KQ
simetrala unutrasnjeg ugla K, a druga od tih pravih simetrala spoljasnjeg ugla
A trougla KAB, pa je H(KA, KB; KC, KD) i prema tome H(A, B; C, D).
Iz QR k CD i QC k RD sledi da je cetvorougao QRCD paralelogram, te je
CD = QR. No QR = l, pa je i CD = l.
Polazeci od druge presecne tacke kruga k sa pravom AB dobijamo jos
jedno resenje problema. Indirektnim postupkom dokazuje se da su dobijena
dva resenja jedina (slika 1450a).

Slika 1450a
Drugi nacin. Navodimo samo uputstvo. Ako obelezimo sa k proizvoljan krug
koji sadrzi tacke A, B i sa O srediste duzi CD, bice OA OB = OC 2 , i prema
1
tome OA OB = l2 .Na taj nacin poznata je potencija tacke O u odnosu
4
na krug k. S toga se moze konstruisati skup svih tacaka kojima su potencije u
1
odnosu na krug k jednake sa l2 , i prema tome tacka O. (slika 1450b).
4

S
C
A

slika 1450b
Konstruisati trougao kada znamo:
1451. A, a, la .

706

Uputstvo. Neka je ABC trougao kome je ugao A jednak datom uglu ,


stranica BC jednaka datoj duzi a, a simetrala AE ugla A jednaka datoj duzi
la . Ako obelezimo sa R i Ra tacke u kojima upisani krugovi k(S, ) i ka (Sa , a )
dodiruju pravu AB i sa F podnozje upravne iz tacke E na pravoj AB, bice
H(A, F ; R, Ra ) i RRa = a. Prva od ovih dveju osobina sleduje neposredno otuda
sto su tacke A, F, R, Ra upravne projekcije na pravoj AB tacaka A, E, S, Sa
za koje znamo da je H(A, E; S, Sa ). Da bismo dokazali drugu od navedenih
osobina, obelezimo sa P i Pa tacke u kojima krugovi k i ka dodiruju stranicu
BC a R i Ra tacke u kojima krugovi k i ka dodiruju pravu AC. S obzirom da
je RRa = ARa AR i QQa = AQa AQ, zatim ARa = AQa i AR = AQ, bice
RRa = QQa . Otuda je
1
1
1
RRa = (RRa + QQa ) = (RB + BRa + QC + CQa ) = (BP + P Pa +
2
2
2
P C + Pa C) = BC = a.
Navedene dve osobine omogucuju da iz datih elemenata konstruisemo trougao ABC.
U tom cilju konstruisimo ugao XAY = i na njegovoj raspolovici AZ
odredimo tacku E takvu da je AE = la . Ako pretpostavimo da je ugao konveksan, bice i ugao XAY konveksan, pa je ugao ZAX ostar. S toga se podnozje
F upravne iz tacke E na pravoj AX nalazi na polupravoj AX. Odredimo na
polupravoj AX tacke R i Ra takve da je H(A, S; R, Ra ) i RRa = a. U tom
cilju kroz tacke A i F konstruisemo proizvoljan krug l, zatim obelezimo sa M
i N sredista lukova AF kruga l. Kroz jednu od tacaka M i N , npr. tacku
M konstrusimo pravu paralelnu sa pravom AF i na njoj s bilo koje strane od
prave M N , npr. s one strane s koje je tacka F , odredimo tacku L takvu da je
M L = a. Pri tome su tacke L, M, N nekolinearne, te odredjuju izvestan krug
l . Taj krug sadrzi tacke M i N koje se nalaze s raznih strana prave AF , te sece
pravu AF u dvema tackama koje pripadaju raznim lucima M N kruga l. Neka
je Ra ona od tih presecnih tacaka koja se nalazi na kruznom luku M LN . Pri
tome prava LRa sece pravu AF , te i njoj paralelna prava kroz tacku M sece
pravu AF u nekoj tacki R. Dokazimo da je H(A, F ; R, Ra ) i RRa = a.
A

M
R
F
B

S
E

Ra
Sa
X

slika 1451
S obzirom da je M N AF i M L k AF , bice LM N prav, pa je duz LN
precnik kruga l . Tacka Ra je na krugu l kome je duz LN precnik, pa je
707

LRa N Ra , i prema tome M RN Ra u nekoj tacki K. Kako je duz M N


precnik kruga l, a ugao M KN prav, tacka K je na krugu l. Pri tome su
prave KM i KN simetrale unutrasnjeg i spoljasnjeg ugla K trougla KAF ,
pa je H(KA, KF ; KM, KN ) i prema tome H(A, F ; R, Ra ). Iz M L k RRa i
M R k LRa sledi da je cetvorougao LM RRa paralelogram, pa je RRa = M L.
No M L = a, pa je i RRa = a.
Kako je Ra zajednicka tacka prave AF s kruznim lukom M LN , ona se
nalazi izvan kruga l i s one strane prave M N s koje nije tacka A, tacka Ra
je na polupravoj AF iza tacke F u odnosu na tacku A. Otuda iz relacije
H(A, F ; R, Ra ) sledi da je tacka R izmedju tacaka A i F . Stoga su obe tacke R
i Ra na kraku AX ostrog ugla XAZ, te prave upravne u tackama R i Ra na
kraku AX seku krak AZ u dvema tackama, obelezimo ih sa S i Sa . Konstruisimo
zatim krugove k(S, SR) i ka (Sa , Sa Ra ). S obzirom da su poluprecnici SR i Sa Ra
krugova k i ka upravni na pravoj AF u tackama R i Ra , krugovi k i ka dodiruju
pravu AF u tackama R i Ra .
S obzirom da krugovi k i ka kojima se sredista nalaze na raspolovici AZ ugla
XAY dodiruju polupravu AX, oni dodiruju i polupravu AY , obelezimo dodirne
tacke sa Q i Qa .
Krugovi k i ka imaju ili nemaju zajednickih unutrasnjih tacaka. Pretpostavimo da krugovi k i ka imaju zajednickih unutrasnjih tangenata, neka je t
bilo koja od njih. Obelezimo sa P i Pa tacke u kojima prava t dodiruje krugove k i ka a sa B i C tacke u kojima prava t sece spoljasnje tangente RRa i
QQa . Kako je tacka A spoljasnje srediste slicnosti krugova k i ka , a prava BC
zajednicka unutrasnja tangenta tih krugova, tacka A je van prave BC, pa su
A, B, C temena izvesnog trougla ABC. Dokazimo da taj trougao predstavlja
resenje datog problema.
S obzirom da su tacke B i C unutrasnje tacke duzi RRa i QQa , one su
na polupravama AX i AY , pa je BAC = XAY . No XAY = , pa je i
BAC = .
Na potpuno isti nacin kao u analizi ovog problema dokazuje se da je BC =
RRa . No RRa = a, pa je i BC = a.
Da bismo dokazali da je kod trougla ABC simetrala ugla A jednaka duzi la ,
neophodno je dokazati da prava SSa sece pravu t u tacki E. Ako prava SSa
ne bi sekla pravu t u tacki E vec u nekoj drugoj tacki E , imali bismo da je
H(A, E ; S, Sa ). S druge strane tacke A, E, S, Sa su paralelne projekcije tacaka
A, F, R, Ra , te iz H(A, F ; R, Ra ) sledi da je H(A, E; S, Sa ). U tom bi slucaju
za tri razne kolinearne tacke A, S, Sa postojale dve razne cetvrte harmonijske
tacke E i E , sto je nemoguce. Stoga prava SSa sece pravu t u tacki E, pa je
duz AB simetrala ugla A. No AE = la pa je i simetrala ugla A trougla ABC
jednaka duzi la .
Zavisno od toga da li krugovi k i ka imaju dve, jednu ili nemaju zajednickih
unutrasnjih tangenata, problem ima dva, jedno ili nema resenja.
1452. A, a, la .
1453. A, r, la .
1454. a, r, la .
1455. ha , la , a.
1456. la , la , a.
1457. B C, ha , a.
1458. B C, ha , a.
708

1459.
1460.
1461.
1462.
1463.
1464.
1465.
1466.
1467.
1468.
1469.
1470.
1471.
1473.
1474.
1475.
1476.
1477.
1478.
1479.
1480.
1481.
p tacke C

B C, Ha , b c.
B C, la , b c.
ha , la , b c.
la , la , b c.
A, ha , b c.
ha , + a, b c.
ha , + a , a.
ha , + a , p.
ha , + a , r.
h a , + a , m a .
h a , + a , l a .
ha , + a , A.
ha , + a , B C.
ha , b c , b c.
ha , b c , p.
ha , b c , r.
ha , b c , .
h a , b c , a .
h a , b c , l a .
ha , b c , A.
ha , b c , B C.
Date su na nekoj pravoj p tri razne tacke A, B, O. Odrediti na pravoj
i D takve da je H(A, B : C, D) i O srediste duzi CD.

Uputstvo. Prvi nacin. Ako na pravoj p postoje tacke C i D takve da


je H(A, B; C, D) i O srediste duzi CD, bice OA OB = OC 2 a tacka O na
produzenju duzi AB. S toga je poznata potencija tacke O u odnosu na proizvoljan krug k koji sadrzi tacke A i B. Pri tome je tacka O izvan kruga k, pa se
moze konstruisati tangenta OT toga kruga, zatim tacke C i D u kojima krug
l(O, OT ) sece pravu p. Resenje postoji samo jedno i to u slucaju kada je tacka
O na produzenju duzi AB (slika 1481a).

sl. 1481a
Drugi nacbin. Ako postoje tacke C i D koje zadovoljavaju postavljene uslove,
bice OA OB = OC 2 i tacka O na produzenju duzi AB. Da bismo konstruisali
tacke C i D, obelezimo sa B tacku simetricnu sa tackom B u odnosu na tacku
O i sa k krug kome je duz AB precnik. Pri tome je tacka O u krugu k, te prava
n upravna na pravoj p u tacki O sece krug k, neka je P bilo koja od presecnih
tacaka. Prava p sadrzi srediste O kruga l(O, OP ) te ga sece u dvema tackama,
709

obelezimo ih sa C i D. S obzirom da je duz OP visina koja odgovara hipotenuzi


AB pravouglog trougla AP B , bice OA OB = OP 2 , pa je i OA OB = OC 2 .
Stoga je H(A, B; C, D) (slika 1481b).
T

O B

Slika 1481b
1482. A, la , b + c.
Uputstvo.
Pretpostavimo da postoji ABC kome je ugao A jednak datom uglu ,
simetrala AE ugla jednaka datoj duzi la , a zbir stranica AB i AC jednak datoj
duzi d. Ako obelezimo sa S srediste upisanog kruga k, sa Sa srediste spolja
upisanog kruga ka koji dodiruje stranicu BC, sa N tacku u kojoj raspolovica
ugla A sece opisani krug l i sa N podnozje upravne iz tacke N na pravoj AB,
1
bice H(A, E; S, Sa ), tacka N srediste duzi SSa i AN = d. Prva od ovih triju
2
osobina sleduje neposredno, jer su S i Sa tacke u kojima simetrale unutrasnjeg i
spoljasnjeg ugla B seku pravu AE. Druga od navedenih triju osobina izvodi se
takode jednostavno jer je kod trougla N SB S = B, pa je N S = N B, a kod
trougla N BSa B = Sa , pa je N B = N Sa . Otuda je N S = N Sa , pa je tacka
N srediste duzi SSa . Da bismo dokazali trecu osobinu obelezimo sa P, Q, R
tacke u kojima upisani krug k dodiruje stranice BC, CA, AB i sa Pa , Qa , Ra
tacke u kojima spolja upisani krug ka dodiruje prave BC, CA, AB. Pri tome je
1
1
AN = ARa N Ra . No ARa = (ARa + AQa ) = (AB + BRa + AC + CQa )
2
2
1
1
= (AB + BPa + AC + Pa C) = (AB + BC + CA) i
2
2
1
1
1
1
N Ra = RRa = (RRa + QQa ) = (RB + BRa + QC + CQa ) = (BP +
2
4
4
4
1
1
BPa + P C + Pa C) = BC, pa je AN = (AB + AC). Navedene tri osobine
2
2
omogucuju da se iz datih elemenata konstruise trougao ABC.

710

l
Q

R
T
S
B
N

k
E
Pa

N
Qa

Ra
l2
Sa

Y
ka

l1

Slika 1482
U tom cilju konstruisemo XAY = , na njegovoj raspolovici AZ odredimo tacku E takvu da je AE = la , a na kraku AX tacku N takvu da je
1
AN = d. Ako pretpostavimo da je ugao konveksan, bice ugao XAZ ostar,
2
te prava upravna u tacki N na polupravoj AX sece polupravu AZ u nekoj
tacki N . Neka je pri tome tacka N iza tacke E u odnosu na tacku A. Konstruisemo sad na polupravoj AE tacke S i Sa takve da je H(A, E; S, Sa ) a tacka
N srediste duzi SSa . Neka je l1 proizvoljan krug koji sadrzi tacke A i E. S
obzirom da je tacka N iza tacke E u odnosu na tacku A, tacka N je van kruga
l1 , te postoje dve prave koje sadrze tacku N i dodiruju krug l1 . Obelezimo
sa T dodirnu tacku bilo koje od njih i sa l2 krug kome je srediste tacka N a
poluprecnik duz N T . Prava AE sadrzi srediste N kruga l2 , te sece taj krug
u dvema tackama, obelezimo ih sa S i Sa . Pri tome je jedna od tih tacaka
S i Sa , npr. tacka S izmedu tacaka A i E, dok je druga iza tacke N u odnosu na tacku S. Stoga su obe tacke S i Sa na polupravoj AE. Iz jednakosti
N A N E = N T 2 = N S 2 = N Sa2 sledi da je H(A, E; S, Sa ). Zaista, uzimajuci u
obzir orijentaciju duzi, iz jednakosti N S 2 = N A N E, tj. N A : N S = N S : N E
nalazimo da je (N A + N S) : (N A N S) = (N S + N E) : (N S N E). Otuda
je Sa A : SA = Sa E : SE, pa je AS : SE = ASa : Sa E, i prema tome
H(A, E; S, Sa ).
S obzirom da su tacke S i Sa na raspolovici AZ konveksnog ugla XAY , postoje
srugovi k i ka sa sredistima S i Sa koji dodiruju krake ugla XAY . Neka krug k
dodiruje poluprave AX i AY u tackama R i Q, a krug ka dodiruje poluprave AX
i AY u tackama Ra i Qa . Pri tome krugovi k i ka imaju ili nemaju zajednickih
unutrasnjih tangenata. Ako krugovi k i ka imaju zajednickih unutrasnjih tan711

genata, neka je t bilo koja od njih. Obelezimo sa P i Pa tacke u kojima prava


t dodiruje krugove k i ka , a sa B i C tacke u kojima prava t sece spoljasnje
tangente RRa i QQa . Kako je tacka A presek spoljasnjih tangenata krugova k
i ka , a prava BC unutrasnja tangenta tih krugova, tacka A je van prave BC.
Stoga tacke A, B, C odreduju izvestan trougao ABC. Dokazimo da taj trougao
predstavlja resenje datog problema.
Kako su tacke B i C unutrasnje tacke duzi RRa i QQa koje pripadaju polupravama AX i AY , tacke B i C su takode na polupravama AX i AY , pa je
BAC = XAY . No XAY = , pa je BAC = .
Na potpuno isti nacin kao u analizi ovog problema dokazuje se da je
1
1
(AB + AC) = AN . No AN = d, pa je AB + AC = d.
2
2
Da bismo dokazali da je kod trougla ABC simetrala ugla A jednaka duzi la ,
neophodno je dokazati da prava SSa sece duz BC u tacki E. Ako simetrala
ugla A ne bi sekla stranicu BC u tacki E, vec u nekoj drugoj tacki E , imali
bismo da je H(A, E; S, Sa ) i H(A, E ; S, Sa ), pa bi za tri razne kolinearne tacke
A, S, Sa postojale dve cetvrte harmonijske tacke E i E , sto je nemoguce. Stoga
raspolovica ugla A sece stranicu BC u tacki E. Kako je AE = la i simetrala
ugla A trougla ABC jednaka je duzi la .
Zavisno od toga da li krugovi k i ka imaju dve, jednu ili nemaju zajednickih
unutrasnjih tangenata, problem ima dva, jedno ili nema resenja.

1483. ha , a , b c.
1484. ha , a , p.
1485. ha , a , la .
1486. ha , a , hb .
1487. ha , a , A.
1488. ha , a , b.
1489. ha , a , B C.
1490. ha , a , b c.
1491. A, la , b c.
1492. ha , b + c , b c.
1493. ha , b + c , p.
1494. ha , b + c , la .
1495. ha , b + c , hb .

712

1496. ha , b + c , A.
1497. ha , b + c , B.
1498. ha , b + c , B C.
1499. ha , b + c , a + .
1500. la , bc, a.
Uputstvo.
Predpostavimo najpre da postoji trougao ABC kome je simetrala AE ugla
A jednaka datoj duzi la , stranica BC jednaka datoj duzi a, a proizvod stranica
AB i AC jednak kvadratu date duzi d. Ako obelezimo sa S i Sa sredista upisanih
krugova k i ka tog trougla i sa N tacku u kojoj simetrala AE ugla A sece opisani
krug l trougla ABC, bice AE AN = d2 , H(A, E; S, Sa ) tacka N srediste duzi
SSa , a tacke B i C na krugu l kome je duz SSa precnik.
Da bismo izveli prvu od navedenih osobina obelezimo sa D podnozije visine
iz temena A i sa AL i M N precnike kruga l. Pri tome je ABD ALC i
ADE N AM , pa je AB : AL = AD : AC i AD : AE = M N : AN , i
prema tome AB AC = AD AL i AE AN = AD M N . Iz ovih dveju jednakosti
i iz jednakosti AL = M N sledi da je AE AN = AB AC = d2 .
Druga od navedenih osobina dokazuje se neposredno, jer su S i Sa tacke u
kojima simetrala unutrasnjeg i simetrala spoljasnjeg ugla B trougla ABE seku
pravu AE.
Trecu osobinu dokazujemo koristeci trouglove SN B i Sa N B. S obzirom da
je kod trougla SN B, S = B i kod trougla Sa N B, Sa = B, bice
N S = N B i N Sa = N B, pa je N S = N Sa , i prema tome tacka N je srediste
duzi SSa .

713

T
D

C
N L
B

Sa

Slika 1500.
Poslednja od navedenih osobina sleduje neposredno iz cinjenice prema kojoj
su uglovi SBSa i SCSa pravi.
Izvedene osobine omogucuju da pristupimo konstrukciji trougla ABC.
Na proizvoljnoj polupravoj AX odredimo tacku E takvu da je AE = la ,
zatim tacku N inverznu s tackom E u odnosu na krug l1 (A, d). Predpostavimo
pri tome da je tacka N iza tacke E u odnosu na tacku A, zatim na polupravoj
AE odredimo tacke S i Sa takve da je H(A, E : S, Sa ), a tacka N srediste
duzi SSa . U tom cilju konstruisemo proizvoljan krug l2 koji sadrzi tacke A
i E. S obzirom da je tacka N iza tacke E u odnosu na tacku A, tacka N
je van kruga l2 , te postoje dve prave koje sadrze tacku N i dodiruju krug l2 ,
neka je T dodirna tacka bilo koja od njih. Prava AE sadrzi srediste kruga
l (N, N T ), prema tome sece taj krug u dvema tackama, obelezimo ih sa S i
Sa . Pri tome je jedna od tih presecnih tacaka, na primer tacka S, izmedu
tacaka A i E, dok je druga od njih, dakle tacka Sa , iza tacke E u odnosu na
tacku A. Stoga se obe tacke S i Sa nalaze na polupravoj AE. Iz jednakosti
N A N E = N S 2 = N Sa 2 sledi da je H(A, E; S, Sa ). Zaista, uzimajuci u obzir
orijentisanu duz, iz jednakosti N S 2 = N A N E, tj. N A : N S = N S : N E
nalazimo da je (N A + N S) : (N A N S) = (N S + N E) : (N S N E). Otuda
je Sa A : SA = Sa E : SE, pa je AS : SE = ASa : Sa E, i prema tome
H(A, E; S, Sa ). Duz a je manja od precnika SSa kruga l ili ne. Ako je a < SSa ,
tada na krugu la postoje dve tacke B i C takve da je B C = a. Pri tome
tetiva B C kruga la ne sadrzi tacku N , te postoji krug l3 koji je koncentrican
sa krugom la a dodiruje tetivu B C . Kroz tacku E postoji tangenta na krugu
l3 ili ne. Ako postoji, ona sadrzi tacku E koja je izmedu tacaka S i Sa , dakle u
714

krugu la , te sece taj krug u dvema tackama, obelezimo ih sa B i C. S obzirom


da prave AE i BC imaju zajednicku tacku E i da prava od njih sece a druga
dodiruje krug l3 , prave AE i BC se seku u tacki E, pa je tacka A van prave
BC. Stoga tacke A, B, C odreduju izvestan trougao ABC. Dokazimo da taj
trougao predstavlja resenje datog problema.
Prema konstrukciji tetive BC i B C kruga l dodiruju krug l3 koji je koncentrican s krugom l pa je BC = B C . No B C = a, pa je i BC = a.
S obzirom da su tacke S i Sa na polupravoj AE takve da je H(A, E; S, Sa ),
a tacke B i C na krugu l kome je duz SSa precnig, bice AB : BE = AS : SE
i AC : CE = AS : SE, pa je tacka S na simetrali ugla B i simetrali ugla C
trougla ABC. Stoga je tacka S srediste kruga upisanog u trougao ABC, i prema
tome duz AE simetrala ugla A trougla ABC. No duz AE jednaka je duzi la pa
je i simetrala ugla A trougla ABC jednaka duzi la .
Istim postupkom kao u analizi ovog problema dokazuje se da je AB AC =
AE AN . No tacke E i N su inverzne medu sobom u odnosu na krug l1 (A, d),
pa je AE AN = d2 . Stoga je i AB AC = d2 .
Predpostavljajuci da je tacka N iza tacke E u odnosu na tacku A, tj. da je
la < d, zatim da je a < SSa , postojece dva, jedno ili nijedno resenje problema
zavisno od toga da li je tacka E izvan kruga l , na tome krugu ili u njemu. Pri
la d ili a SSa problem nema resenja.

13.2

Metoda transformacije

1611. Date su dve prave a, b izvan tih pravih tacka O. Konstruisati pravu s
koja sadrzi tacku O i sece prave a ib u tackama A iB takvim da duzi OA i OB
budu srazmerne dvema datim duzima m i n.
Resenje. Ako bi postojala prava s koja sadrzi tacku O i sece prave a,b u
tackama A,B takvim da je OA : OB = m : n, tada bi tacka B pripadala
pravoj a koja u homotetiji H(O, m : n) odgovara pravoj a. Na taj nacin, B je
zajednicka tacka pravih a i b. Ako obelezimo sa S proizvoljnu tacku prave a
(npr. presecnu tacku pravih a i b ukoliko se one seku) i za S tacku prave OS
takvu da je OS : OS = m : n tada prava a kroz tacku S uporedna s pravom
a u pomenutoj homotetiji odgovara pravoj a. Prave a i b imaju zajednickih
tacaka ili ne. Ako te prave imaju zajednickih tacaka, obelezimo sa B bilo koju
od njih. S obzirom da s odredena tackama O i B sece pravu a , ona sece i
njoj paralelnu pravu a u nekoj tacki A. Pri tome je OAS OBS , pa je
OA : OB = OS : OS . No OS : OS = m : n, pa je i OA : OB = m : n. Zavisno
od toga da li su prave a i b istovetne, konkurentne ili paralelne, problem ima
neograniceno mnogo resenja, jedno resenje ili pak nema resenja.

715

b
S

Slika 669
1612. Data su dva kruga a i b i tacka O. Odrediti na krugovima a ib tacke A
i B kolinearne s tackom O pri cemu su duzi OA i OB srazmerne dvama datim
duzima m i n.
Resenje. Ako bi na krugovima a i b postojale tacke A i B koinearne s
tackom O pri cemu je OA : OB = m : n, tada bi tacka B pripadala krugu a
koji u homotetiji H(O, m : n) odgovara krugu a. Na taj nacin B je jedna od
zajednickih tacaka krugova a i b.
b

B
O

Slika 670
Ako obelezimo sa S srediste, i sa P proizvoljnu tacku kruga a, a sa S i P tacke
koje u homotetiji H(O, m : n) odgovaraju tackama S i P , tada u toj homotetiji
krugu a(S, SP ) odgovara krug a (S , S P ). Krugovi a i b imaju zajednickih
tacaka ili ne. Ako krugovi a i b imaju zajednickih tacaka, obelezimo sa B
bilo koju od njih. S obzirom da se tacka B nalazi na krugu a , ona u homotetiji
H(O, m : n) odgovara nekoj tacki A kruga a, pa su A i B trazene tacke. Zavisno
od toga da li se krugovi a i b poklapaju, seku, dodoruju ili nemaju zajednickih
tacaka, problem ima beskonacno mnogo resenja, dva, jedno ili nemaju resenja.
1637.Konstruisati trougao ABC kome je simetrala AE unutrasnjeg ugla A
jednaka datoj duzi a1 dok su poluprecnici krugova opisanih oko trouglova ABE
i ACE jednaki datim duzima r1 i r2 .
Uputstvo. Ako obelezimo sa O1 i O2 sredista krugova opisanih oko trougova
ABE i ACE, bice ABO1 ACO2 , pa je AB : AC = AO1 : AO2 = r1 : r2 .
No AB : AC = BE : EC, pa je i BE : EC = r1 : r2 .
1638.Konstruisati trougao ABC kome je ugao A jednak datom uglu , dok
su poluprecnici krugova opisanih oko trouglova ABE i ACE gde je E tacka u
kojoj simetrala ugla A sece stranicu BC, jednaki datim duzima r1 i r2 .
716

Uputstvo. Ako obelezimo sa O1 i O2 sredista krugova opisanih oko trougova


ABE i ACE, bice O1 AO2 = BAC.
1642.Data su dva kruga k1 , k2 i tacka S. Konstruisati dve paralelne prave
t1 i t2 od kojih prva dodiruje krug k1 a druga krug k2 tako da odstojanja tacke
S od tih pravih t1 i t2 budu srazmerna dvema datim duzima m i n.
Uputstvo. Ako obelezimo sa k1 krug koji u homotetiji H(S, m : n) odgovara
krugu k1 , bice prava t2 zajednicka tangenta krugova k1 i k2 .
1644.Data su tri koncentricna kruga a, b, c i tacka P . Konstruisati pravu s
koja sadrzi tacku P i sece date krugove redom u tackama A, B, C, takvim da
duzi AB i BC budu srazmerne dvema datim duzima m i n.

Uputstvo.Ako
obelezimo

 sa A proizvoljnu tacku kruga a, sa c krug koji u
m
+
n
homotetiji H A ,
odgovara krugu c, sa B jednu od zajednickih tacaka
m
krugova b i c , i sa C tacku kruga c kojoj u pomenutoj homotetiji odgovara
tacka B , bice A C : A B = (m + n) : n, pa je i A B : B C = m : n. Ako
zatim obelezimo sa l krug koncentrican s datim krugovima a koji dodiruje pravu
s odredenu tackama A i B , bice trazena prava s tangenta kruga l.

B
C

E
O
s
O
s
A

Slika 671
1645.Konstruisati pravu s koja sadrzi tacku P i sece cetiri data koncentricna
kruga a, b, c, d redom u tackama A, B, C, D takvim da je duz AB jednaka duzi
CD.
Uputstvo. Ako postoji prava s koja zadovoljava uslove navedene u formulaciji zadatka i ako obelezimo sa A , B ,C ,D druge presecne tacke prave s sa
krugovima a, b, c, d, a sa ra , rb , rc , rd pouprecnike tih krugova,

717

D C B A

A B C D
s
O
d
c
b
a

Slika 672

bice AD = BC i prema tome


AD
(ra rd )(ra + rd )
AD AD
r2 rd2
=
.
=
= a2

BC
BC BC
(rb rc )(ra + rb )
rb rc2
AB
AD
=2
+ 1.
Medutim, kako je AD = AB + BC + CD = 2AB + BC, bice
BC
BC
Otuda je
 r2 rb2 + rc2 rd2
AB
1 AD
=
1 = a
,
BC
2 BC
2(rb2 rc2 )
i prema tome
AC
AB + BC
BC
r2 + rb2 rc2 rd2
=
=1+
= a2
.
AB
AB
AB
ra rb2 + rc2 rd2
Na taj nacin problem se svodi na prethodni.
1722.U dati trougao ABC upisati pravougaonik P QRS kome su temena P
i O na stranici BC, temena R i S na stranicama CA i AB, a dijagonala P R
paralelna s datom pravom l.
Upustvo. Ako obelezimo sa DBCF pravougaonik homotetican s trazenim
pravougaonikom P QRS, bice
CDkP Rkl
(sl.673).
A

Slika 673
718

1723.Dat je trougao ABC i na stranici BC tacka P . Odrediti na stranicama


BC, CA, AB tacke Q, R, S takve da cetvorougao P QRS bude paralelogram
kome su susedne stranice P Q i QR srazmerne datim duzima m i n.
Uputstvo. Najpre konstuisemo paralelogram DECB homotetican o trazenim
paralelogramom P QRS u odnosu na tacku A. U tom cilju konstuisemo na pravoj AP tacku D takvu da je
BC : BD = m : n,
zatim tacku E takvu da je cetvorougao DECB paralelogram. Sad se moze
konstuisati i paralelogram P QRS jer je
DBkP S
(sl. 674).
A

Slika 674
1724.Na stranicama AB, BC, CD, CA datog cetvorougla ABCD odrediti
tacke P, Q, R, S takve da cetvorougao P QRS bude romb kome su stranice paralelne s dijagonalama AC i BD.
Uputstvo. Najpre konstuisati romb P Q R S homotetican s trazenim rombom P QRS u odnosu na jedno teme npr. A cetvorougla ABCD (sl. 675).
D
R
C

R
Q
Q

Slika 675
719

1725.Konstruisati trapez ABCD kome su uglovi A i B na osnovici jednaki


datim uglovima i , a dijagonale AC i BD jednake datim duzima d1 i d2 .
Uputstvo. Ako obelezimo sa B , C , D tacke polupravih AB, AC, AD takve
da je
B C kBC
i
C D kCD,

a sa S i S tacke u kojima se seku dijagonale trapeza ABCD i AB C D , bice


(sl. 676)
AA : B S = AO : B D = AC : BD = d1 : d2.

S
A

Slika 676
Sem toga, ako obelezimo sa O tacku u kojoj se seku prave AD ,B C i sa M
tacku u kojoj se seku prave O S i AB , bice tacka M srediste duzi AB . Stoga
mozemo konstuisati najpre trougao AB C , zatim tacku S , te trapez AB C D ,
i najzad trapez ABCD.
1726.Konstruisati pravu S koja je paralelna s datom pravom p i koja sece
stranice AB i AC datog trougla ABC u tackama X i Y takvim da su duzi BX
i CY srazmerne dvema datim duzima m i n.
Uputstvo. Ako najpre konstuisemo cetvorougao BX Y C koji je homotetican s cetvorouglom BXY C u odnosu na tacku B, bice Y tacka u kojoj
prava BY sece stranicu AC (sl. 677).

720

A
X
X
Y
Y

Slika 677
1727.Na stranicama AB i AC datog trougla ABC odrediti tacke X i Y takve
da je AX = XY = Y C.
Uputstvo. Ako najpre konstuisemo izlomljenu liniju AX Y C homoteticnu
s izlomljenom linijom AXY C u odnosu na tacku A, bice
X C kXC
(sl. 678).

Y
C
Y

Slika 678
1728.Na stranicama AB i AC datog trougla ABC odrediti tacke X i Y takve
da je BX = XY = Y C.
Uputstvo. Koristiti uputstvo iz zadatka 1726.
1729.Na stranicama AB i AC datog trougla ABC odrediti tacke X i Y takve
da duz XY bude jednaka datoj duzi d, a duzi BX i CY srazmerne dvema datim
duzima m i n.
Uputstvo. Koristiti uputstvo iz zadatka 1726.
1730.Na stranicama AB i AC datog trougla ABC odrediti tacke X i Y takve
da duzi BX i CY budu srazmerne dvema dati duzima m i n, a duz XY jednaka
zbiru ili razlici duzi BX i CY .
721

Uputstvo. Koristiti uputstvo iz zadatka 1726.


1734.Date su tri konkurentne prave a, b, c i van njih tacka S. Konstruisati
pravu s koja sadrzi tacku S i sece prave a, b, c respektivno u tackama A, B, C
takvim da duzi AB i BC budu srazmerne datim duzima m i n.
Uputstvo. Obelezimo sa O tacku u kojoj se seku prave a,b,c i sa s proizvoljnu
pravu koje ne sadrzi tacku O a paralelna s pravom s. Pri tom prava s sece prave
a,b,c u tackama A ,B ,C , takvim da je
A B : B C = AB : BC = m : n.
Medjutim, ako za A uzmemo proizvoljnu tacku prave razlicitu od O, tada se
moze konstuisati prava s , zatim prava s (sl. 679).
A

s
B

B
S

C
C
b

Slika 679
1735.Date su cetiri prave a, b, c, d. Konstruisati pravu p koja je uporedna
pravoj d i sece prave a, b, c u tackama A, B, C takvim da duzi AB i BC budu
srazmerne dvema datim duzima m i n.
Uputstvo. Ako je p prava koja je paralelna s pravom d i koja sece prave a i
b u raznim tackama A i B , zatim C tacka prave p takva da je
A B : B C = m : n,
tada prava odredena tackom C i tackom O u kojoj se seku prave a i b sece
pravu c u tacki C. (sl. 680).

722

a
p

A
d

A
O

C
c
C

Slika 680
1736.Date su komplanarne prave a, b, c, d, e i dve duzi m, n. Konstruisati
pravu s koja je paralelna s pravom e i sece prave a, b, c, d u tackama A, B, C, D
takvim da je
AB : CD = m : n.
Uputstvo. Analizirajmo opsti slucaj. Ako obelezimo sa O tacku u kojoj se
seku prave a i b, sa S tacku u kojoj prava kroz O paralelna s pravom l sece
pravu c, sa A , B , C tacke u kojima neka druga prava, s paralelna s pravom
l sece prave a, b, c i sa D tacku te prave s bilo koje strane od tacke C , takvu
da je
A B : C D = m : n,
bice D tacka u kojoj se seku prave SD i d (sl. 681).

D
D

d
C

S
C

c
b

B
O

s
A

Slika 681
1737.Konstruisati pravu koja je paralelna sa datom pravom p i koja sece
stranice AB i CD datog cetvorougla ABCD u tackama M i N takvim da je
AM : M B = DN : N C.
723

Uputstvo. Ako obelezimo sa C tacku takvu da je CC k p i DC k AB, bice


tacka O u kojoj se seku prave AD i BC na pravoj s (sl. 682)

O
p
C

D
N
A

Slika 682
1740.Date su dve uporedne prave p, q i izvan njih dve tacke A i B. Konstruisati dve prave od kojih jedna sadrzi tacku A i sece prave p i q u tackama X i
Y , a druga sadrzi tacku B i sece prave p i q u tackama Z i Y takvim da je (a)
XY = Y Z; (b) XZ = Y Z.
Uputstvo
(a) Ako je C tacka u kojoj prava kroz tacku A uporedna s pravom p sece
pravu Y Z, bice trougao XY Z homotetican s trouglom AY C, pa je AC = AY.
Sad se zadatak svodi na problem...(a).
(b) Kao u prethodnom delu, ako je C tacka u kojoj prava kroz tacku A
uporedna s pravom p sece pravu Y Z, bice trougao XY Z homotetican s trouglom
AY C, pa je AC = Y C. Stoga se zadatak svodi na problem...(b). (Sl. 683 i 684)

X
Z

Y
B

Slika 683

724

q
Y

Slika 684
1741.Na stranicama AB i AC datog trougla ABC odrediti tacke X i Y takve
da je XY k BC i XY = BX CY .
Uputstvo. X i Y su tacke u kojima prava kroz srediste S upisanog ili srediste
Sb spolja upisanog kruga uporedna sa stranicom BC sece stranice AB i AC.
1742.Na stranicama AB i AC datog trougla ABC odrediti tacke X i Y takve
da je XY k BC i XY 2 = AX XB.
Uputstvo. Ako obelezimo sa X i Y proizvoljne tace duzi AB i AC takve
da je X Y k BC, sa Z tacku takvu da je AZ B prav, bice
X Y 2 = X Z 2 = AX X B .
Prava BY je uporedna s pravom B Y (Sl.685)

725

A
Z
X

Y
Y

B
B

Slika 685
1744.Konstruisati trougao ABC kada znamo: A, a + b, a + c.
Uputstvo. Ako obelezimo sa M tacku iza tacke B u odnosu na tacku A
takvu da je BO = BM i sa N tacku iza tacke C u odnosu na tacku A takvu
da je BO = CN, tada se moze konstruisati trougao AM N , a problem svesti na
zadatak 1728z.
1745.Konstruisati trougao ABC kada znamo: A, a b, a c.
Uputstvo. Ako obelezimo sa M i N tacke polupravih BA i CA takve da je
BC = BM i BO = CN, tada se moze konstruisati trougao AM N, a problem
svesti na zadatak 1728.
1746.Konstruisati trougao ABC kada znamo: A, a + b, b + c.
Uputstvo. Ako obelezimo sa D tacku iza tacke A u odnosu na tacku B
tako da je AC = AD i sa E tacku iza tacke C u odnosu na tacku A takvu da
je BC = CE, tada se moze konstruisati najpre trougao DA C homotetican s
trouglom DAC u odnosu na tacku D, zatim na polupravama DA i A C tacke
B i E takve da je
DB = a + c
i
A E = a + b,
najzad prava l,koja sadrzi tacku E a uporedna je s pravom A D. Iz jednakosti
i istosmernosti duzi AE i A E sleduje da i tacka E pripada pravoj l.Ako zatim
obelezimo sa S tacku u kojoj se seku prave DC i l, a sa B bilo koju od tacaka
u kojima prava SB sece krug k(C , C E ), bice BC k B C .
1747.Konstruisati trougao ABC kada znamo: A, a b, b c.
726

Uputstvo. Koristiti postupak iz prethodnog zadatka.


1748.Konstruisati trougao ABC kada znamo: A, a, b + 2c.
Uputstvo. Ako obelezimo sa D tacku iza tacke A u odnosu na tacku B takvu
da je AD = 2AC, bice kod trougla ACD poznat ugao A i odnos stranica AD i
AC, pa se moze konstruisati njemu slican trougao DA C . Ako zatim obelezimo
sa B tacku poluprave DA takvu da je
BD = b + 2c
i sa C jednu od tacaka u kojima poluprava DC sece krug k(B, a), bice A tacka
duzi BD takva da je AC k A C .
1749.Konstruisati trougao ABC kada znamo: ma a, (ma , a), b.
Uputstvo. Ako obelezimo sa D srediste stranice BC i sa E tacku iza tacke
D u odnosu na tacku A takvu da je DE = BC, bice
AE = ma + a,
a kod trougla CDE poznat ugao D i odnos stranica DC i DE. Stoga se moze
konstruisati trougao C D E slican s trouglom CDE, na polupravoj ED odrediti
tacka A takva da je
XA = ma + a,
zatim tacka C u kojoj poluprava EC sece krug k(A, b).
1750.Konstruisati trougao ABC kada znamo: ma a, (ma , a), b : c.
Uputstvo. Koristiti uputstvo iz prethodnog zadatka.
1751.Konstruisati trougao ABC kada znamo: A, mb , b c.
Uputstvo. Ako obelezimo sa D srediste stranice AC i sa E tacku simetricnu sa tackom D u odnosu na tacku A, tada se kao u problemu 1749 moze
konstruisati trougao BDE ,zatim trougao ABC.
1752.Konstruisati trougao ABC kada znamo: B, a, b ha .
Uputstvo. Ako obelezimo sa D podnozje visine iz temena A i sa E tacku
poluprave AD takvu da je AE = AC, bice DE = b ha . Ako zatim obelezimo
sa S tacku prave AB takvu da je SE k BC, sa A proizvoljnu tacku poluprave
SA, sa E podnozje upravne iz tacke A na pravoj SE i sa C jednu od tacaka
u kojima poluprava SC sece krug k(A , A E ) bice trougao A E C homotetican
s trouglom AEC na tacku S.
1753.Konstruisati trougao ABC kada znamo: ha , hb , hc .
Uputstvo. Prema poznatom stavu imamo da je aha = bhb = chc . Deljenjem
sa ha hb nalazimo da je a : hb = b : ha = c : (ha hb : hc ). Stoga je trougao ABC
slican sa trouglom AB C kome je B C = hb , AC = ha , AB = ha hb : c. Iz
ovih jednakosti sleduje da se moze konstruisati najpre trougao AB C , zatim
trougao ABC.
1754.Konstruisati jednakokraki trougao kome je krak jednak datoj duzi b, a
zbir ili razlika osnovice i njoj odgovarajuce visine jednaka datoj duzi l.
Uputstvo. Koristiti uputstvo iz zadatka 1749.
1755.Konstruisati trougao ABC kada znamo A, a, nc gde je n dati broj.
Uputstvo. Koristiti uputstvo iz zadatka 1748.
1756.Konstruisati trougao ABC kome upisani krug sece tezisnu liniju AA1
u tackama M i N takvim da je AM = M N = N A1 i kome je poznat jedan od
elemenata: a, b + c, p, ha , ma , la , r, q.
Uputstvo. Ako kod trougla ABC upisani krug sece tezisnu liniju AA1 u
tackama M i N takvim da je AM M N = N A, tada je prema zadatku . . .
727

AB : BC : CA = 5 : 10 : 13, pa se moze konstruisati najpre trougao slican s


trouglom ABC, zatim koriscenjem poznatog elementa i sam trougao ABC.
1757.Na stranici BC datog trougla ABC odrediti tacku D takvu da poluprecnici krugova upisanih u trouglove ABD i ACD budu medusobno jednaki.
Uputstvo. Ako obelezimo sa S1 i S2 sredista krugova upisanih u trouglove
1
ABD i ACD, bice S1 S2 k BC, S1 AS2 = BAC, a srediste duzi S1 S2 na
2
pravoj AD. Stoga mozemo najpre konstruisati trougao A S1 S2 homotetican s
trouglom AS1 S2 u odnosu na srediste S upisanog kruga, zatim trougao AS1 S2 ,
i najzad tacku D (sl. 686).

S
S1
S1

S2
S2

Slika 686
1758.Na stranici BC datog trougla ABC odrediti tacku D tako da poluprecnici spolja upisanih krugova trougla ABD i ACD koji odgovaraju stranicama BD i CD budu medusobno jednaki.
Uputstvo. Postupiti kao u zadatku 1757.
1759.Na stranici BC datog trougla ABC odrediti tacku D tako da poluprecnici spolja upisanih krugova trouglova ABD i ACD koji odgovaraju stranicama AB i AC budu medusobno jednaki.
Uputstvo. Postupiti kao u zadatku 1757.
1760.Dat je trougao ABC i na pravoj BC data je tacka P . Konstruisati
pravu s koja sadrzi tacku P i sece prave AB i AC u tackama X i Y takvim da
je BX = CY .
BP CY AX
Uputstvo. Prema Menelajevoj teoremi imamo da je

XB =
PC Y A P
1. Otuda se iz jednakosti BX = CY sledi da je AX : AY = CP : P B.
728

Stoga treba na pravama AB i AC najpre odrediti tacke X i Y takve da je


AX : AY = CP : P B, a zatim kroz tacku P konstruisati pravu s paralelnu s
pravom X Y (sl. 687).

X
X
Y
Y
C

Slika 687
1761.Data su dva kruga k1 I k2 i tacka A. Konstruisati pravu koja sadrzi
tacku A, a sece krug k1 u tackama X1 , Y1 i krug k2 u X2 , Y2 takvim da tetive
X1 Y1 i X2 Y2 budu srazmerne poluprecnicima krugova k1 i k2 .
Uputstvo. Trazena prava sadrzi tacku A i najmanje jedno srediste slicnosti
krugova k1 i k2 .
1762.Konstruisati krug koji dodiruje dva data kruga k1 i k2 pri cemu prava
kroz dodirne tacke sadrzi datu tacku A.
Uputstvo. Prava kroz dodirne tacke sadrzi najmanje jedno srediste slicnosti
krugova k1 i k2 .
1763.Konstruisati krug koji dodiruje dva data kruga k1 i k2 pri cemu prava
kroz dodirne tacke mora biti paralelna datoj pravoj p.
Uputstvo. Koristiti uputstvo iz predthodnog zadatka.
1764.Konstruisati krug k koji sadrzi dva date tacke A, B i dodiruje datu
pravu p.
Uputstvo. Ako bi postojao krug k(S, r) koji sadrzi tacke A, B i dodiruje
pravu p, imali bismo da je SA = SB, pa bi se tacka S nalazila na simetrali
s duzi AB. Pri tome bi prava s bila konkurentna, paralelna ili istovetna s
pravom p. Pretpostavimo li da je prava s konkurentna s pravom p, naime da
prava s sece pravu p u nekoj tacki O, bice svaki krug k (S , r ) kome se srediste
nalazi na pravoj s i koji dodiruje pravu p homotetican s krugom k u odnosu
729

na tacku O. Zaista, ako obelezimo sa P i P tacke u kojima krugovi k i k


dodiruju pravu p, sa M proizvoljnu tacku kruga k i sa M tacku orijentisane
prave OM da je OS : OS = OM : OM , bice OS : OS = SP : S P i
OS : OS = SM : S M , pa je SP : S P = SM : S M . Otuda i iz jednakosti
SP = SM sledi da je S P = S M , pa je tacka M na krugu k . Obratno,
ako obelezimo sa M proizvoljnu tacku kruga k i sa M tacku orijentisane prave
OM takvu da je OS : OS = OM : OM , bice takode OS : P S = SP : S P i
OS : OS = SM : SM , pa je SP : S P = SM : S M . Otuda i iz jednakosti
S P = S M sledi da je SP = SM , pa je tacka M na krugu k. Stoga je krug
k homotetican s krugom k u odnosu na tacku O.

S
k
S

Slika 688
Izvedena osobina omogucuje da pristupimo konstrukciji kruga k. U tom
cilju konstruisimo najpre proizvoljan krug k (S , r ) kome se srediste nalazi na
pravoj s i koji dodiruje pravu p. Pri tome prava OA ima s krugom k zajednickih
tacaka ili ne. Pretpostavimo da ima, neka je A bilo koja od njih. U homotetiji
(O, OA : OA ) tacka S odgovara nekoj tacki S. S obzirom da su tacke S i A
razlicite, i njima homoteticne tacke S i A su razlicite, te postoji krug k kome je
srediste tacka S a poluprecnik duz SA. Dokazimo da je k trazeni krug.
S obzirom da je srediste S kruga k na simetrali OS duzi AB , bice SA = SB.
No duz SA je poluprecnik kruga k, pa je i duz SB poluprecnik kruga k. Stoga
krug k sadrzi tacke A i B. Da bismo dokazali da krug k dodiruje pravu p,
obelezimo sa P i P podnozja upravnih iz tacaka S i S na pravoj p. Pri tome je
OS : OS = SP : SP i OS : OS = SA : SA pa je SP : SP = SA : SA . No
krug k dodiruje pravu p u tacki P , pa je S P = S A , i prema tome SP = SA.
Stoga i krug k dodiruje pravu p u tackiP .
Zavisno od toga da li prava OA ima s krugom k dve, jednu ili nijednu
zajednicku tacku, problem ima dva, jedno ili nijedno resenje. Slucaj kada je
730

s k p i s p neka citalac analizira sam.


1765.Konstruisati krug k koji sadrzi datu tacku A i dodiruje dve date prave
p i q.
Resenje. Prave p i q se seku ili su paralelne. Pretpostavimo najpre da se
prave p i q seku u nekoj tacki O. Tacka A je van pravih p i q, na jednoj od njih
ili na obema pravama p i q. Analizirajmo svaki od tih slucajeva.
Ako je tacka A van pravih p i q, ona je u jednom od konveksnih uglova koji
su odredeni pravama p i q, obelezimo ga sa .
Ako bi postojao krug k(S, r) koji sadrzi tacku A i odreduje prave p i q, on

bi pripadao uglu i svaki krug k (S , r ) upisan u uglu bio bi homotetican s


krugom k u odnosu na tacku O.
Prva od ovih dveju osobina sleduje neposredno otuda sto krug k nema tacaka
koje se nalaze s one strane prave p s koje nije tacka A i s one strane prave p s
koje nije tacka A i s one strane prave q s koje nije tacka A. Da bismo dokazali

drugu od navedenih dveju osobina, obelezimo sa P i P tacke u kojima krugovi

k i k dodiruju pravu p, sa M proizvoljnu tacku kruga k i sa M tacku poluprave

OM takvu da je OS : OS = OM : OM . Pri tome je OS : OS = SP : S P

i OS : OS = SM : S M , pa je SP : S P = SM : S M . Otuda i iz SP =

SM sledi da je S P = S M , pa je tacka M na krugu k . Obratno, ako je M

proizvoljna tacka kruga k i M tacka poluprave OM takva da je OS : OS =

OM : OM , bice OS : OS = SP : S P i OS : OS = SM : S M , pa je SP :

S P = SM : S M . Otuda i iz S P = S M sledi da je SP = SM , pa je tacka

M na krugu k. Stoga je krug k homotetican s krugom k u odnosu na tacku O.


Izvedene osobine omogucuju da pristupimo konstrukciji kruga k, u tom cilju

konstruisimo najpre proizvoljan krug k (S , r ) upisan u uglu . S obzirom da

je krug k upisan u uglu , a tacka A je u uglu , poluprava OA sece krug k ,

neka je A bilo koja od presecnih tacaka. Pri tome, u homotetiji (O, OA : OA )

tacka S odgovara nekoj tacki S. Kako su tacke S i A razlicite, razlicite su i


tacke S i A, te postoji krug k kome je srediste tacka S, a poluprecnik duzi SA.
Dokazimo da je k trazeni krug (sl. 689).
q
Q
Q

A
A

P
P

Slika 689
Prema konstrukciji krug k sadrzi tacku A. Da bismo dokazali da krug k

dodiruje pravu p, obelezimo sa P i P podnozja upravnih iz tacaka S i S na

pravoj p. Pri tome je OS : OS = SP : S P i OS : OS = SA : SA , pa

je SP : S P = SA : S A . No krug k dodiruje pravu p u tacki P , pa je


731

S P = S A , i prema tome SP = SA. Stoga i krug k dodiruje pravu p. Istim


postupkom dokazuje se da krug k dodiruje i pravu q.

S obzirom da poluprava OA sece krug k u dvema tackama, problem u ovom


opstem slucju ima dva resenja. Ako se prave p i q seku, a tacka A nalazi na
jednoj od pravih p i q, npr. pravoj p, trazeni krug k dodirivace pravu p u tacki
A, te se njegovo srediste nalazi na pravoj n koja je u tacki A upravna na pravoj
p. No krug k dodiruje obe prave p i q, pa se njegovo srediste nalazi na osi
simetrije pravih p i q. Ako bi S bila zajednicka tacka prave n i neke od dveju
osa simetrije pravih p i q, tada krug k(S, SA) sadrzi tacku A i dodiruje prave
p i q. S obzirom da prava n sece svaku od pomenutih osa, i u ovom slucaju
problem ima dva resenja. Ako tacka A pripada obema pravama p i q, problem
nema resenja, jer bi u protivnom dve razne prave p i q, dodirivale krug k u istoj
tacki A, sto je nemoguce. Slucaj kada je p k q neka citalac analizira sam.
1766.Konstruisati krug k koji dodiruje dve date prave p i q i dati krug k.
Uputstvo. Ako bi k(S, r) bio krug koji dodiruje prave p i q i krug l(L, ),

tada krug k (S, r ) sadrzi tacku L i dodiruje izvesne dve prave p , q koje su
paralelne s pravama p, q i na rastojanju od njih. Pri tome znak plus treba uzeti
u slucaju kada se krugovi k i l dodiruju spolja, a znak minus kada se krugovi k
i l dodiruju iznutra. Stoga, kao u prethodnom problemu, treba najpre konstrui
sati krug k , zatim krug k. Problem u opstem slucaju ima osam resenja (sl. 690).

O
O

q
L

R
k

P
p

Sl. 690
1767.Konstruisati krug k koji sadrzi datu tacku A i dodiruje date pravu p i
dati krug l.
Uputstvo. Obelezimo sa O i S sredista krugova l i k, sa M N precnik kruga
l upravan na pravoj p, sa K tacku u kojoj prava M N sece pravu p, a sa P i L
tacke u kojima krug k dodiruje linije p i l. Pri tome je tacka L srediste slicnosti
krugova l i k, a tacka P odgovarajuca jednoj od tacaka M i N , npr. tacki M .
Sada su kod cetvorougla KP LN naspramni uglovi K i L pravi, pa je on tetivan.
Ako zatim obelezimo sa B drugu tacku prave AM i kruga k, bice (v. sl. 691).

732

M
l
O

k
A

Sl. 691
MA . MB = MP . ML = MK . MN.
Stoga se na pravoj AM moze konstruisati tacka B takva da je M A . M B
= M K . M N . Na taj nacin postavljeni problem svodimo na problem 1764.
1768.Konstruisati krug koji dodiruje dva data kruga O1 i O2 i datu pravu p.
Uputstvo. Ako bi k(S, r) bio krug koji dodiruje krugove l1 (O1 , 1 ), l2 (O2 , 2 )
i pravu p, tada pri npr. 1 > 2 krug k (S , r2 ) sadrzi tacku O2 , dodiruje krug
l1 (O1 , 1 2 ) i izvesnu pravu p koja je paralelna sa pravom p i na rastojanju
2 od nje. Pri tome za poluprecnik kruga k treba uzeti duz r 2 zavisno od
toga da li se krugovi k i l2 dodiruju spolja ili iznutra, a za poluprecnik kruga l
treba uzeti duz 1 2 zavisno od toga da li se krugovi k i l1 dodiruju iznutra
ili spolja. Stoga, kao u prethodnom problemu, treba najpre konstruisati krug
k , zatim krug k. Problem u opstem slucaju ima osam resenja.
l1
l1

l2

O2
O1
L2

L1
L1

Slika 692
1769.Konstruisati krug k koji sadrzi datu tacku A i dodiruje dva data kruga
l1 i l2 .
Tacke L1 i L2 u kojima krug k dodiruje krugove l1 i l2 su sredista slicnosti
kruga k sa krugovima l1 i l2 , te prava odredena tackama L1 i L2 sadrzi i srediste
733

O slicnosti krugova l1 i l2 . Ako obelezimo sa B drugu presecnu tacku prave


OA sa krugom k, bice OA OB = OL1 OL2 (v. sl. 693). Primenom ove
jednakosti moze se konstruisati tacka B. Na taj nacin postavljeni problem svodi
se na konstrukciju kruga k koji sadrzi dve poznate tacke A i B i dodiruje jedan
od datih krugova (v. zad. 1764).

A
k

S
B
L1
L2

O1

O2

l2
l1

Slika 693
1770.Konstruisati krug k koji dodiruje tri data kruga l1 , l2 i l3 .
Uputstvo. Ako bi k(S, r) bio krug koji dodiruje tri data kruga l1 (O1 , 1 ),
l2 (O2 , 2 ), l3 (O3 , 3 ) i duz 3 najmanja medju duzima 1 , 2 , 3 , tada krug
k (S, r 3 ) sadrzi tacku O3 i dodiruje krugove l1 (O1 , 1 3 ) i l2 (O2 , 2 3 ).
Pri tome za poluprecnik kruga k treba uzeti duz r 3 zavisno od toga da li se
krugovi k i l3 dodiruju spolja ili iznutra, za poluprecnik kruga l1 treba uzeti duz
1 3 zavisno od toga da li se krugovi k i l1 dodiruju iznutra ili spolja, a za
poluprecnik kruga l2 treba uzeti duz 2 3 zavisno od toga da li se krugovi k
i l2 dodiruju iznutra ili spolja. Stoga, kao u prethodnom zadatku, treba najpre
konstruisati krug k , zatim krug k. Problem u opstem slucaju ima osam resenja.

734

l3
O3
k
P3

k
P1
l1

S
P2

O1

O2

l2

Slika 694
1771.Dat je konveksan ugao XAY i u njemu tacka P . Konstruisati pravu
koja sadrzi tacku P i sece krake AX i AY tog ugla u tackama B i C takvim da
obim trougla ABC bude minimalan.
Uputstvo. Ako obelezimo sa B i C tacke u kojima proizvoljna prava kroz
tacku P sece poluprave AX i AY , sa k spolja upisani krug trougla AB C
koji odgovara stranici B C i sa P , Q , R tacke u kojima taj krug dodiruje
prave B C , C A, AB , bice duz AQ jednaka poluobimu trougla AB C , jer je
1
1
1
AQ = (AQ + AR ) = (AC + C Q + AB + B R ) = (AB + AC + B P +
2
2
2
1
P C ) = (AB + AC + B C ) (v. sl. 695). Stoga ce trougao AB C imati
2
minimalan obim ako je duz AQ minimalna. No duz AQ bice minimalna ako
spolja upisani krug k trougla AB C sadrzi tacku P . Na taj nacin, problem se
svodi na problem iz zadatka 1765.

735

R
B
R

C
Q
Q

P
k

Slika 695
1772.Date su tri nekolinearne tacke A, B, C. Na pravoj AB odrediti tacku
P takvu da je AB BP = CP 2 .
Uputstvo. Ako dirka u tacki C kruga l opisanog oko trougla ABC sece pravu
AB u tacki P , bice AP BP = CP 2 . Ovom konstrukcijom dobijamo jedno
resenje. Medutim, problem ima jos resenja. Da bismo ih odredili, obelezimo
sa D tacku simetricnu sa tackom C u odnosu na tacku A. Prava d koja sadrzi
tacku D i paralelna je sa pravom AB sece krug l, dodiruje ga ili sa njim nema
zajednickih tacaka. Ako prava A sece krug l u tackama npr. D1 i D2 , bice
sredista P1 i P2 tetiva CD1 i CD2 na pravoj AB i pri tome AP1 BP1 =
CP1 D1 P1 = CP12 i AP2 BP2 = CP2 D2 P2 = CP22 . Ako prava d dodiruje
krug l u nekoj tacki D1 bice takode srediste P1 tetive CD1 na pravoj AB pri
cemu je AP1 BP1 = CP1 D1 P1 = CP12 . Stoga problem ima tri, dva, jedno ili
pak nema resenja.

736

P1

D1

P2

D2

Slika 696

13.3

Inverzija

1774. Dati su krug k i tacka A, prava a i krug l.


(a)Konstruisati tacku A inverznu s tackom A u odnosu na krug k.
(b)Konstruisati pravu a inverznu s pravom a u odnosu na krug k.
(c)Konstruisati krug l inverzan s krugom l u odnosu na krug k.
Resenje:
(a) Obelezimo sa A tacku inverznu s tackom A u odnosu na krug k, zatim
razmotrimo tri mogucnosti s obzirom na polozaj tacke A u odnosu na krug k.
Ako je tacka A na krugu k, tada iz OA OA = r2 i OA = r sledi da je
OA = r tada je A A.
Ako je tacka A u krugu k (slika 1), razlicita od tacke O, tada iz OAOA = r2
i OA < r sledi da je OA > r pa je tacka A izvan kruga k. Stoga postoje dve
prave koje sadrze tacku A i dodiruju krug k. Neka je D dodirna tacka bilo koje
od njih. Iz OAOA = OD2 , tj. OA : OD = OD : OA i AOD = DOA sledi
da je AOD DOA . No ODA je prav, pa je i OAD prav. Otuda sledi
da je D jedna od tacaka u kojima prava kroz tacku A upravna na pravoj OA
sece krug k. Zato da bi smo konstruisali tacku A konstruisemo najpre pravu
m koja je u tacki A upravna na pravoj OA. S obzirom da prava m sadrzi tacku
A koja se nalazi u krugu k, ona sece krug k, neka je D bilo koja od presecnih
tacaka. Pri tome je AOD ostar, te upravna n u tacki D na polupravoj OD
sece poupravu OA u nekoj tacki A . Iz AOD = DOA i OAD = ODA
sledi da je OAD DOA , pa je OA : OD = OD : OA , i prema tome
OA OA = OD2 = r2 . Stoga je tacka A inverzna s tackom A u odnosu na krug
k. Iz simetrije u odnosu na pravu OA sledi da se tacka A poklapa s tackom
koja se dobija polazeci od druge presecne tacke prave m s krugom k.

737

slika 1
Ako je tacka A izvan kruga k, tada iz OA OA = r2 i OA > r sledi da je
OA < r pa je tacka A u krugu k. Stoga prava kroz tacku A upravna na pravoj
OA sece krug k, neka je D bilo koja od presecnih tacaka. Iz OAOA = OD2 , tj.
OA : OD = OD : OA i AOD = DOA sledi da je AOD DOA . No
OA D je prav, pa je i ODA prav. Stoga je tacka D na krugu l koma je duz
OA precnik. Zato, da bi smo konstruisali tacku A , konstruisimo najpre krug l
kome je OA precnik. Taj krug sadrzi tacku O koja se nalazi u krugu k i tacku
A koja se nalazi izvan kruga k, te sece krug k, neka je D bilo koja od presecnih
tacaka i A podnozje upravne iz tacke D na pravoj OA. Iz AOD = DOA i
ADO = DA O sledi da je AOD DOA , pa je OA : OD = OD : OA ,
i prema tome OA OA = OD2 = r2 . Stoga je tacka inverzna s tackom A u
odnosu na krug k. Iz simetrije u odnosu na pravu OA sledi da se tacka A
poklapa sa tackom koja se dobija polazci ok druge presecne tacke krugova l i k.
Problem u svakom od razmatranih slucajeva ima jedno resenje.
(b) Ako prava a sadrzi srediste O inverzije, tada svakoj tacki P koja pripada
pravoj a i razlicita je od tacke O odgovara neka tacka P koja pripada pravoj
OP , tj. pravoj a, i obrnuto, svaka tacka P koja pripada pravoj a a i razlicita
je od tacke O je odgovarajuca nekoj tacki P prave OP , tj. prave a. Stoga u
inverziji u odnosu na krug k pravoj a odgovara ta ista prava.
Ako prava a ne sadrzi srediste O inverzije (slika 2), obelezimo sa A podnozje
upravne iz tacke O na pravoj a, na p bilo koju drugu tacku prave a, a sa A i P
tacke inverzne s tackama A i P u odnosu na krug k. Iz OA OA = OP OP , tj.
OA : OP = OP : OA i AOP = P OA sledi da je AOP P OA, pa
je OAP = OP A. No OAP je prav, pa je i OP A prav. Stoga je tacka
P na krugu a kome je duz OA precnik. Zato konstruisimo podnozje tacke A
upravne iz tacke O na pravoj a, zatim tacku A izverznu s tackom A u odnosu
na krug k, najzad krug a kome je duz OA precnik. Ako je P bilo koja tacka
kruga a razlicita od tacaka O i A , bice A OP ostar, te prava a upravna na
polupravoj OA sece polupravu OP u nekoj tacki P . Iz AOP = P OA i
OAP = OP A sledi da je OAP OP A , pa je OA : OP = OP : OA ,
i prema tome OP OP = OA OA = r2 . Stoga je tacka P inverzna s tackom
P u odnosu na krug k. Iz izvedenih osobina sledi da u inverziji u odnosu na
krug k pravoj a odgovara krug a .
738

D
P

P
k

slika 2
(c) Analizirajmo najpre slucaj kada krug l sadrzi srediste O inverzije. Ako
obelezimo sa A tacku kruga l dijametalno supratnu s tackom O, sa P bilo koju
tacku kruga l razlicitu od tacaka O i A, a sa A i P tacke inverzne s tackama A
i P u odnosu na krug k bice OA OA = OP OP , tj. OA : OP = OP : OA i
AOP = P OA , pa je OAP OP A , i prema tome OP A = OA P .
No OP A je prav, pa je i OA P prav. Stoga je tacka A na pravoj l koja
je u tacki A upravna na pravoj OA. Zato konstruisemo na krugu l tacku
A dijametralno suprotnu s tackom O, zatim tacku A inverznu s tackom A u
odnosu na krug k, najzad pravu l koja je u tacki A upravna na pravoj OA.
Ako je P proizvoljna tacka prave l razlicita od tacke A , bice AOP ostar,
te poluprava OP sece krug l u nekoj tacki P . Pri tome je AOP = P OA i
OP A = OA P , pa je OAP OP A . Stoga je OP : OA = OA : OP ,
tj. OP OP = OA OA = r2 , i prema tome tacka P je inverzna s tackom P u
odnosu na krug k. Na taj nacin, krugu l koji sadrzi srediste inverzije odgovara
neka prava l .
Analizirajmo sad slucaj kada krug l ne sadrzi srediste O inverzije (slika 3).
Ako obelezimo sa P proizvoljnu tacku kruga l, sa P njenu inverznu tacku u
odnosu na krug k i sa P drugu zajednicku tacku prave OP s krugom l, bice
OP OP = r2 i OP OP = t2 gde je t2 potencija tacke O u odnosu na krug l.
Stoga je OP : OP = r2 : t2 pa se tacka P nalazi na krugu l koji u homotetiji
(O, r2 : t2 ) odgovara krug l. Zato konstruisimo krug l koji u toj homotetiji
odgovara krugu l. Ako obelezimo sa P proizvoljnu tacku kruga l , sa P tacku
u kojoj u pomenutoj homotetiji odgovara tacka P i sa P drugu zajednicku
tacku prave OP sa krugom l, bice OP : OP = r2 : t2 i OP OP = t2, pa je
OP OP = r2 . Stoga je tacka P inverzna s tackom P u odnosu na krug k. Na
taj nacin, krugu l koji ne sadrzi srediste inverzije odgovara neki krug l .

739

P
P
O

S
l
l

slika 3
1775. Date su prave a, b i tacka O. Konstruisati pravu koja sadrzi tacku
O i sece prave a i b u tackama X i Y takvim da proizvod duzi OX i OY bude
jednak kvadratu date duzi r.
Resenje:
Uputstvo. Koristiti zadatak 1774(b).

1776. Data su dva kruga a, b i tacka O. Na krugovima a i b odrediti tacke


A i B kolinearne s tackom O takve da proizvod duz OA i OB bude jednak
kvadratu date duzi r.
Resenje:
Uputstvo. Koristiti zadatak 1774(c).

1777. Konstruisati trougao ABC kada znamo: A, a, b (b c).


1778. Konstruisati trougao ABC kada znamo: A, r, b (b c).
1779. Konstruisati trougao ABC kada znamo: a, r, b (b c).
1780. Konstruisati trougao ABC kome je stranica BC podudarna datoj
duzi a, a ugao A jednak datom uglu , a proizvod duzi BA i BD, gde je D
podnozje visine iz temena C, jednak kvadratu date duzi r.
Resenje:
Uputstvo. koristiti zadatak 1774(c)
740

1781. Odrediti srediste i koeficijent inverzije u kojoj trima datim tackama


A, B, C odgovaraju nekolinearne tacke A , B , C takve da trougao A B C
bude podudaran sa datim trouglom A1 B1 C1 .
Resenj:
Uputstvo. Ako je (O, k) trazena inverzija prema poznatom stavu, bice
BC =
tj.

kOB
OB OC

OB OC = k

OB
B1 C1

Iz ove i dveju analognih jednakosti sledi da je


B1 C1 C1 A1 A1 B1
:
:
BC
CA
AB
Stoga primenom zadatka mozemo konstruisati najpre tacko O, zatim odrediti
koeficjent k.
OA : OB : OC =

1782. Konstruisati srediste O inverzije proizvoljnog koeficijenta r u kojoj


trima datim kolinearnim tackama A, B, C odgovaraju kolinearne tacke A , B , C
takve da tacka C bude srediste duzi A B .
Resenje.
Ako bi postavljeni problem imao resenje. tacka O mora da pripada pravoj l
koja sadrzi tacke A, B, C; sem toga mora da vazi H (A, B; C, O). Prva od ovih
dveju osobina sledi neposredno, jer u svakom inverznom preslikavanju tackama
A, B, C prave l odgovaraju tacke A , B , C neke prave l ako i samo ako

srediste O takvog preslikavanja pripada pravoj l. Staci


se, pri tome je prava l
istovetna s pravom l. Da bi smo dokazali drugu od navedenih osobina koristimo cinjenicu prema kojoj u svakom tom inverznom preslikavanju sredistu O
prave l odgovara beskonacno daleka tacka O prave l . Kako je tacka C srediste
duziA B i O beskonacno daleka tacka prave A B , bice H (A , B ; C , O ), te je
i H (A, B; C, O).
Stoga na pravoj l konstruisimo tacku O takvu da je H (A, B; C, O). U inverznom preslikavanju u odnosu na krug k(O, r) raznim tackama A, B, C prave
l odgovaraju razne tacke A , B , C te iste prave l, dok tacki O odgovara beskonacno daleka tacka O prave l. Iz H (A, B; C, O) sledi da je H (A , B ; C , O ),
te je tacka C srediste duzi A B . Sobzirom da za tri razne kolinearne tacke
A, B, C postoji jedinstvena tacka O takva da je H (A, B; C, O), problem ima
jedinstveno resenje.

1783. Konstruisati srediste i koeficijent inverzije u kojoj svaki od triju datih


krugova k1 , k2 , k3 odgovara samom sebi, drugim recima, konstruisati krug koji
741

sece ortogonalno tri data kruga k1 , k2 , k3 .


Resenje:
Uputstvo. S obzirom da u trazenoj inverziji svaki od triju datih krugova treba
da odgovara samom sebi, srediste O te inverzije ima jednake potencije u odnosu
na svaki od tih krugova, te predstavlja radikalno srediste tih krugova.

1784. Konstruisati srediste i koeficijent inverzije u kojoj trima datim krugovima k1 , k2 , k3 na nekolinearnim sredistima S1 , S2 , S3 odgovaraju krugovi
k1 , k2 , k3 kojima se sredista S1 , S2 , S3 nalaze na datoj pravoj l .
Resenje:
Uputstvo. S obzirom da prava l treba da sadrzi sredista krugova k1 , k2 , k3 ,
prava l mora da bude ortogonalna na krugovima k1 , k2 , k3 . Stoga u trazenoj
inverziji pava l odgovara nekom krugu l koji je ortogonalan na krugovima k1 ,
k2 , k3 . Zato, kao i u predhodnom problemu treba konstruisati najpre krug l
ortogonalan na trima datim krugovima k1 , k2 , k3 , zatim na tom krugu l tacku
O takvu da u inverziji u odnosu na tu tacku krugu l odgovara prava l .

1785. Date su tri tacke A, B, C i duz r. Kroz tacku C konstruisati pravu s


takvu da podnozja X i Y upravnih iz tacaka A i B na toj pravoj s zadovoljavaju
relaciju:
(a) AX BY = r2 ,
(b) AX 2 BY 2 = r2 .
Resenje:
(a) Uputstvo. S obzirom da ugao AXC treba da bude prav (slika 4), tacka
X mora da pripada krugu k kome je duz AC precnik. Ako zatim obelezimo sa
AC1 Y1 pomereno podudaran s BCY , bice i AY1 C1 prav, pa je i tacka Y1
na krugu k1 kome je duz AC1 precnik. Sem toga, bice AX AY1 = AX BY = r2 .
Stoga treba konstruisati najpre tacku C1 takvu da duz AC1 bude jednaka i istosmerna sa duzi BC, zatim krugove k i k1 kojima su duzi AC i AC1 precnici i
na njima tacke X i Y1 inverzne medju sobom u odnosu na krug l(A, r).

742

k1

Y1
C1
B
Y

X
C

slika 4
(b) Uputstvo. Ako obelezimo sa A1 tacku simetricnu s tackom A u odnosu
na tacku C i sa D i D1 upravne projekcije tacaka A i A1 na pravoj BY , bice
tacke D i D1 na krugovima k i k1 kojima su duzi AB i A1 B precnici. Sem toga
bice
BD BD1 = (AX BY )(AX + BY ) = AX 2 BY 2 = r2
Stoga treba najpre konstruisati krugove k i k1 , zatim na njima odrediti tacke D
i D1 koje u inverziji (B, r2 ) odgovaraju jedna drugoj. Trazena prava s sadrzi
tacku C i srediste Y duzi DD1 .

1786. Konstruisati krug k koji sadrzi dve date tacke A, B i sece dati krug l
pod pravim uglovima.
Resenje:
pripada krugu
Uputstvo. Tacka A inverzna tacki A u odnosu na krug l takode
k.

13.4

Translacija

1808. Date su u ravni cetiri prave a, b, c, d pri cemu je a k b i c k d. Kroz


datu tacku M konstruisati pravu s koja sece date prave u tackama A, B, C, D
takvim da je AB = CD.
Uputstvo. Date prave a, b, c, d odreduju izvestan paralelogram P QRS, trazena
prava uporedna je sa bilo kojom dijagonalom tog paralelograma (sl. 717).

743

D
S

B
R
C
A

slika 717
1809. Date su tri prave a, b, c i na pravama a, b tacke A, B. Konstruisati
pravu s koja je paralelna s pravom c i koja sece prave a i b u tackama X i Y
takvim da su duzi AX i BY srazmerne datim duzima m i n.
Uputstvo. Ako bi postojala prava s koja je paralelna sa pravom c i koja sece
prave a i b u tackama X i Y takvim da je AX : BY = m : n, tada bi u translaciji
T (XY ) tacki B odgovarala izvesna tacka L. Pri tom bi cetvorougao BY XL bio
paralelogram, te bi bilo BL||c, BY ||b, i BY = XL. Ove osobine omogucuju da
se konstruise najpre tacka L, a zatim trazena prava s (sl. 718).
S

X
Y
M

L
B

slika 718
1810. Date su tri prave a, b, c i na pravama a, b tacke A i B. Konstruisati
pravu s koja je paralelna s pravom c i koja sece prave a i b u tackama X i Y
takvim da je zbir ili razlika duzi AX i BY jednaka datoj duzi l.
Uputstvo. Pretpostavimo da postoji prava a koja je paralelna sa pravom c i
koja sece prave a i b u tackama X i Y takvim da je zbir ili razlika duzi AX i BY
jednaka duzi l. Pri tome u translaciji T (XY ) tacki B odgovara tacka K takva
da je cetvorougao BY XK paralelogram. Ako zatim obelezimo sa L tacku prave
744

a takvu da je AL = l, bice BY = KX = XL, pa je trougao KLX jednakokrak.


Ugao pri vrhu X tog trougla jednak je izvesnom uglu kojeg odreduju prave a i
b, pa je ugao L na osnovici tog jednakokrakog trougla jednak polovini ugla koji
je suplementan sa tim uglom. Stoga je prava LK paralelna sa simetralom p tog
ugla. Dobijeni uslovi omogucavaju da se konstruise najpre tacka K, zatim tacka
X i najzad prava s (sl. 719).

S
L

X
Y

K
A

slika 719
1811. Date su dve prave a, b i na svakoj od njih po jedna tacka A, B. Konstruisati pravu s koja sece prave a i b u tackama X i Y takvim da je duz XY
jednaka datoj duzi d, a duzi AX i BY srazmerne dvema datim duzima m i n.
Uputstvo: Ako pretpostavimo da postoji prava s koja sece prave a i b u
tackama X i Y takvim da je XY = d i AX : BY = m : n, zatim obelezimo sa
K tacku koja u trnslaciji T (Y X) odgovara tacki B, bice cetvorougao BY XK
paralelogram, pa je BK = XY = d i XK||b i XK = Y B. Pri tome je kod
trougla AXK AX : XK = m : n i Y K||b, pa se moze konstruisati trougao
AM N homotetican sa trouglom AXK zatim odrediti tacka K kao presek prave
AN sa krugom k(B, d). (sl. 720).

745

X
Y
M

K
B

slika 720
1812. Date su dve prave a, b i na svakoj od tih pravih po jedna tacka A, B.
Konstruisati pravu s koja sece prave a i b u tackama X i Y takvim da je duz
XY jednaka datoj duzi d, a zbir ili razlika duzi AX i BY jednaka datoj duzi l.
Uputstvo: Prave a i b se seku ili su paralelne. Analizirajmo slucaj kada se
prave a i b seku u nekoj tacki O. Ako pretpostavimo da postoji prava s koja
sece prave a i b u tackama X i Y takvim da je duz XY jednaka duzi d, a zbir ili
razlika duzi AX i BY jednaka dizi l, zatim obelezimo sa B0 i Y0 tacke prave s
takve da je OB = OB0 i OY = OY0 , bice duz XY0 jednaka zbiru ili razlici duzi
AB0 i l. Stoga su u trouglu XY Y0 poznate stranice XY, XY0 i ugao XY 0Y, pa se
moze konstruisati trougao X1 Y1 B0 koji u translaciji T (XX1 ) odgovara trouglu
XY Y0 (sl. 721). Slucaj kada su prave a i b paralelne neka citalac analizira sam.

746

Bo

Yo

slika 721
1813. Date su prave a, b, c, d, p. Konstruisati pravu s koja je paralelna s
pravom p i koja sece prave a, b, c, d respektivno u tackama A, B, C, D takvim da
duzi AB i CD budu srazmerne dvema datim duzima m i n.
1814. Date su kolinearne prave a, b, c, d, p. Konstruisati pravu s koja je
paralelna s pravom p i koja sece prave a, b, c, d u tackama A, B, C, D takvim da
je zbir ili razlika duzi AB i CD jednaka datoj duzi l.
1815. Date su dve paralelne prave a, b i dve tacke M, N koje se nalaze s
raznih strana svake od tih pravih. Odrediti na pravama a i b tacke A i B
takve da duz AB bude paralelna s datom pravom p, a zbir duzi M A, AB, BN
minimalan.
1816. Date su cetiri prave a, b, c, d takve da je a k b i c k d, zatim dve tacke
M, N koje se nalaze s raznih strana svake od tih pravih. Odrediti na pravama
a, b, c, d tacke A, B, C, D takvim da duzi AB i CD budu respektivno paralelne
s dvema datim pravama p i q, a zbir duzi AB, BC, CD, DN minimalan.
Konstruisati trougao ABC kada znamo:
1817.
1818.
1819.
1820.
1821.
1822.
1823.
1824.
1825.
1826.
1827.
1828.
1829.

ma , mb , mc .
h a , ma , mb .
h a , ma , mb .
ma , h a , h b .
ma , h b , h c .
ma , b, c.
ma , ha , b : c.
ma , hb , b : c.
ma , mb , b : c.
A, mb , mc .
A, ma , mb .
A, ma , ha .
A, ma , hb .
747

1830. A, ma , b2 c2 .
1831. A, ma , b2 c2 .
Konstruisati cetvorougao ABCD kada znamo:
1832.
1833.
1834.
1835.
1836.
1837.
1838.
1839.
1840.
1841.

AB, CD, AC, BD, (AC, BD).


AB, BC, CD, DA, (AB, CD).
AC, BD, AB : CD, BC : AD, (AC, BD).
A, B, AC, BD, (AC, BD).
A, B, BC, CD, DA.
A, B, AB, CD, (AB, CD).
A, B, AB, CD, AD : BC.
A, B, AB, CD, AD BC.
A, B, AB, AD BC, (AB, CD).
A, B, C, AB, CD.

Konstruisati trapez ABCD sa paralelnim stranicama AB i CD kada znamo:


1842. AB, BC, CD, DA.
1843. AB, CD, AC, BD.
1844. AD, BC, AC, BD.
1845. AB, AC, BD, (AC, BD).
1846. A, B, AB, CD.
1847. A, B, AC, BD.
1848. A, B, AB : CD, AC.
1849. A, B, AB CD, AC : BD.
1850. Na katetama AB i AC datog pravouglog trougla ABC odrediti tacke
X i Y takve da duz XY bude jednaka datoj duzi d, a zbir kvadrata duzi BX i
CY jednak kvadratu date duzi l.
Uputstvo: Ako pretpostavimo da na katetama AB i AC pravouglog trougla
ABC postoje tacke X i Y takve da je XY = d i BX 2 + CY 2 = l2 , tada u
translaciji T (Y X) tacki C odgovara izvesna tacka C1 . Pri tome je cetvorougao
CY XC1 paralelogram, pa je CC1 = XY = d, a kod trougla BXC kome je ugao
X prav BC12 = BX 2 + XC12 = BX 2 + CY 2 = l2 . Stoga se moze konstruisati
najpre tacka C1 , a zatim tacke X i Y (sl. 722).

C1

slika 722

748

1851. Oko datog trougla P QR opisati trougao ABC kome su stranice


BC, CA, AB respektivno jednake datim duzima a, b, c.
1852. Oko datog trougla P QR opisati jednakostranican trougao ABC maksimalnog obima.
1853. Konstruisati pravougaonik ABCD takav da stranica AB bude jednaka
datoj duzi i da prave odredene stranicama AB, BC, CD, DA sadrze respektivno
date tacke P, Q, R, S.
1854. Konstruisati paralelogram kome znamo stranice i ugao izmedu dijagonala.
1855. Konstruisati trapez ABCD kome znamo dijagonale AC i BD, duz koja
spaja sredista dijagonala i duz koja spaja sredista dveju naspramnih stranica.
1856. U dati krug l upisati trapez ABCD kome znamo visinu i zbir ili razliku
paralelnih stranica AB i CD.
1857. Konstruisati cetvorougao ABCD kome znamo stranice AB, BC, CD, DA
i duz koja spaja sredista M i N naspramnih stranica AB i CD.
1858. Konstruisati cetvorougao ABCD kome su stranice AB, BC, CD, DA
jednake datim duzima a, b, c, d, a duz koja spaja sredista M i N naspramnih
stranica AB i CD jednaka datoj duzi l.
1859. Konstruisati cetvorougao ABCD kome su stranice AB, BC, CD, DA
jednake datim duzima a, b, c, d, a duz koja spaja sredista P i Q dijagonala AC
i BD.
1860. Konstruisati cetvorougao ABCD kome su stranice AB, BC, CD, DA
jednake datim duzima a, b, c, d, a duz M N jednaka datoj duzi l,gde su M i N
tacke stranica AB i CD takve da je AM : M B = m : n i DN : N C = m : n, a
m i n date duzi.
1861. Konstruisati cetvorougao ABCD kome su stranice AB, BC, CD, DA
jednake datim duzima a, b, c, d, a duz P Q jednaka datoj duzi l,gde su P i Q
tacke dijagonala AC i BD takve da je AP : P C = m : n i BQ : QD = m : n, a
m i n date duzi.
1862. U dati trougao ABC upisati pravougaonik P QRS kome su temena P
i Q na stranici BC, temena R i S na stranicama AC i AB, a dijagonala P R
jednaka datoj duzi d.
Uputstvo: Ako obelezimo sa A tacku takvu da je A BBC i A A k BC, a
sa R i S tacke u kojima prava RS sece stranice A C i A B trougla A BC, bice
(v. Sl. 723)
Sl. 723
SR : BC = AS : AB = A S : A B = S R : BC,
pa je SH = S R . Otuda je i P RS
= BR S , dakle i P R = BR .
1863. U dati trougao ABC upisati pravougaonik P QRS kome su temena P
i Q na stranici BC, temena R i S na stranicama AC i AB, a dijagonala P R
minimalne duzine.
Uputstvo: Ako obelezimo sa A tacku takvu da je A BBC i A A k BC, a
sa R i S tacke u kojima prava RS sece stranice A C trougla A BC bice (Sl.
724)
Sl. 724
749

SR : BC = AS : AB = A S : A B = S R : BC.
Otuda je SR = SR , pa je P RS
= BR S , dakle i BR = P R. Dijagonala
BR pravougaonika BQ R S upisanog u trougao A BC bice minimalna ako je
BR A C.
1864. U dati trougao ABC upisati pravougaonik P QRS datog obima 2p
tako da njegova dva temena P i Q budu na stranici BC, a ostala dva temena R
i S na stranicama AC i AB. Uputstvo: Ako obelezimo sa A tacku takvu da je
A BBC i A A k BC,a sa R i S tacke u kojima prava RS sece stranice A C i
A B trougla A BC, bice (v. Sl. 725)
Sl. 725
SR : BC = AS : AB = A S : A B = S R : BC,
pa je SR = S R . Stoga je zbir odstojanja tacke R od stranica A B i BC jednak
poluobimu pravougaonika P QRS, tj. duzi d.
1865. U dati trougao ABC upisati pravougaonik maksimalne povrsine tako
da njegova dva temena P i Q budu na stranici BC, a ostala dva temena R i
S na stranicama AC i AB. Uputstvo: Ako obelezimo sa A tacku takvu da
je A BBC i A A k BC, a sa R i S tacke u kojima prava RS sece stranice
A C i A B trougla A BC i sa Q podnozje upravne iz tacke R na stranici BC,
bice povrsine pravougaonih povrsi (P QRS) i (BQ R S ) jednake, stavise, bice i
povrsina pravougaone povrsi (BQ R S ), upisane u trougao A BC, maksimalna.
Da bi ta pravougaona povrs (BQ R S ) imala maksimalnu povrsinu, mora zbir
povrsina trougaonih povrsi (A R S ) i (R CQ ) biti maksimalan. No pravougaone povrsi (A R S ) i (R CQ ) su slicne, te su im povrsine proporcionalne
kvadratima hipotenuza. Na taj nacin problem se svodi na odredivanje tacke R
na stranici A C takve da zbir A R2 + R C 2 bude minimalan. Pri tome je (v.
Sl. 726)
Sl. 726

A R2 + R C 2 =

1
1
[(A R + R C)2 + (A R R C)2 ] = [A C 2 + (A R R C)2 ].
2
2

Iz ove jednakosti sleduje da ce zbir A R2 + R C 2 biti minimalan ako je |AR


R C| minimalna, tj. ako je tacka R srediste duzi A C.
1866. U dati trougao ABC upisati pravougaonik P QRS date povrsine k
tako da njegova dva temena P i Q budu na stranici BC, a ostala dva temena
R i S na stranicama AC i AB.
Uputstvo: Ako obelezimo sa A tacku takvu da je A BBC i A A k BC,
a sa R i S tacke u kojima prava RS sece stranice A C i A B trougla A BC i
sa Q podnozje upravne iz tacke R na stranici BC, bice povrsine pravougaonih

750

povrsi (P QRS) i (BQ R S ) jednake. Ako je zatim A tacka poluprave BA


takva da je BC = BA , bice
S(A BC) = S(A BC)

BC
A B
= S(A BC) .
A B
AB

Obelezimo zatim sa R tacku u kojoj prava Q R sece stranicu A C trougla


A BC i sa S tacku stranice A B takvu da je R S k BC. Pri tome je (v. Sl.
727)
Sl. 727

S(BQ R S ) = S(BQ R S )

BC
BC
BC
= S(P QRS) = k = k ,

AB
AB
AB

i prema tome
BQ Q R = AQ Q C = k .

Stoga mozemo konstruisati najpre tacku Q , zatim redom pravougaonike BQR S ,


BQ R S , P QRS.
1867. Dat je trougao ABC i u njemu tacka P . Odrediti na trouglu tacku Q
takvu da izlomljena linija AP Q razlozi trougaonu povrs (ABC) na dve jednake
povrsi.
1868. Kroz datu tacku P koja se nalazi na stranici AB date cetvorougaone
povrsi (ABCD) konstruisati pravu koja razlaze tu povrs na dve ekvivalentne
povrsi.
1869. Kroz teme A1 konveksne poligonske povrsi (A1 A2 . . . An ) konstruisati
k 1 pravih koje razlazu tu poligonsku povrs na k ekvivalentnih povrsi.

13.5

Simetrija

Simetrija u odnosu na ta
cku
1870. Date su dve prave a, b i van njih tacka S. Konstruisati pravu s koja
sadrzi tacku S i sece prave a i b u tackama A i B takvim da je tacka S srediste
duzi AB.
1871. Dat je konveksan ugao XAY i u njemu tacka P . Konstruisati pravu
koja sadrzi tacku P i sece krake AX i AY u tackama B i C takvim da povrsina
trougla ABC bude minimalna.
1872. Kroz presecnu tacku P datih krugova k1 i k2 konstruisati pravu p na
kojoj krugovi k1 i k2 odsecaju jednake tetive.
1873. Kroz presecnu tacku P dvaju datih krugova k1 i k2 konstruisati pravu
p koja sece krugove k1 i k2 jos u tackama Q i R takvim da je zbir ili razlika
tetiva P Q i P R jednaka datoj duzi l.
1874. Dati su krug k, na njemu dve tacke A i B, zatim prava p i na njoj
tacka P . Odrediti na krugu k tacku X takvu da prave AX i BX seku pravu p
u tackama X i Z koje su simetricne medu sobom u odnosu na tacku P .

751

1875. Data je prava p i s iste strane od te prave date su dve tacke B , C .


Konstruisati trougao ABC kome se stranica BC nalazi na pravoj p, i kome su
tacke B i C podnozja visina iz temena B i C.
1876. Dat je ugao XAY i u njegovoj ravni tacka S1 . Konstruisati trougao
ABC kome se srediste Ojlerovog kruga poklapa s tackom S1 , ortocentar H
nalazi na pravoj AX, a srediste upisanog kruga na polupravoj AY .
Konstruisati trougao ABC kada znamo:
1877. ma , mb , mc .
1878. ma , mb , gc .
1879. ma , hb , hc .
1880. ma , hb , b c.
1881. ma , ha , b2 c2 .
1882. ma , ha , b : c.
1883. A, ma , ha .
1884. A, ma , hb .
1885. A, ma , b?c.
1886. A, ma , b2 c2 .
1887. A, ma , b(b c).
1888. A, mb , mc .
1889. A, , b c.
1890. A, a , b c.
1891. A, b , b + c.
1892. A, , hb hc .
1893. A, a , hb hc .
1946. Na trima datim pravama a, b, c odrediti tacke P , Q, R takve da zbir
duzi P Q, QR, AB bude minimalan.
Resenje: Razmotricemo dva slucaja:

P
A
Q
R
c

C
P

Slika 748 a
1. slucaj - kada se prave a, b, c seku, tako da obrazuju ostougli trougao ABC
(slika 748a.). Prema zadatku 1944., podnozja visina trougla ABC odreduju
trougao P QR koji od svih trouglova upisanih u trougao ABC ima minimalan
obim. Dokazacemo sad da su i u ovom problemu P, Q, R trazene tacke. Ako sa
P obelezimo bilo koju tacku prave a koja ne pripada duzi BC, vec se nalazi ili
752

iza tacke B u odnosu na tacku C ili iza tacke C u odnosu na tacku B. Neka
je npr. tacka P iza tacke B u odnosu na tacku C. Sa Q i R obelezimo
promenljive tacke pravih b i c, a sa P podnozje upravno iz tacke P na pravu
b, bice:
P Q + QR + RP < 2BQ < 2P P < P Q + Q R + R P .
P1

Y
X

A
b

c
Z

P
B

B1 P

a
C1

C
Y

P2

Slika 748 b
2. slucaj - kada se prave a, b, c seku tako da obrazuju tupougli trougao ABC
sa tupim uglom kod temena A (slika 748b.). Prema zadatku 1945., svakoj tacki
P prave a odgovaraju tacke Q i R na pravama b i c, tako da je zbir duzi
P Q + Q R + R P minimalan. Odredimo onu tacku P na pravoj a za koju
ce odgovarajuci zbir P Q + Q R + R P biti minimalan. Ako bi tacka P bila
na produzenju duzi BC, npr. iza tacke B u odnosu na tacku C, onda bi ona
pripadala unutrasnjosti ostrog ugla koji zahvataju prave b i c. Stoga bi, prema
zadatku 1945, duz P1 P2 , pri cemu su tacke P1 i P2 simetricne sa tackom P u
odnosu na prave b i c, bila jednaka odgovarajucem zbiru P Q + Q R + R P ,
gde su Q i R tacke u kojima duz P1 P2 sece prave b i c. Ako, potom, obelezimo
sa B i P podnozja upravnih iz tacaka B i P na pravu b, bice:
2BB < 2P P < P1 Q + P Q < P1 Q + Q R + R P
= P Q + Q R + R P .
Odatle sledi da tacka P kojoj odgovara minimalni zbir P Q + Q R + R P nije
na produzenju duzi BC. Normale iz tacke A, na pravama b i c, seku stranicu
BC u tackama, oznacimo ih sa, B1 i C1 . Ako su X i Y proizvoljne tacke duzi
BB1 i CC1 , a X i Y podnozja upravnih iz tacaka X i Y na pravama b i c, bice
prema zadatku 1940.,zbirovi koji odgovaraju tackama X i Y jednaki dvostrukim
duzima XX i Y Y . Ako je, pak, Z bilo koja unutrasnja tacka duzi B1 C1 , prema
istom zadatku, njoj odgovara zbir jednak dvostrukoj duzi AZ. Najzad, ako je P
podnozje visine iz temena A, bice: 2AP 2AZ, 2AP < 2XX , 2AP < 2Y Y .
Stoga je za tacku P prave a odgovarajuci zbir najmanji.
1947. Konstruisati trougao ABC ako su date tri tacke Ha , Hb , Hc koje su
simetricne s ortocentrom H tog trougla u odnosu na prave odredene stranicama
BC, CA, AB.
Uputstvo: Tacke Ha , Hb , Hc koje su simetricne sa ortocentrom H pripadaju
krugu l opisanom oko trougla ABC. S obzirom da su te tri tacke poznate, moze
753

se konstruisati krug l. Iz jednakosti uglova ABHb i ACHc sledi da je tacka A


srediste kruznog luka Hb Hc na kome nije tacka Ha .

A
Hb

Hc
H

Ha

Slika 749
1948. Konstruisati trougao ABC ako su date tri tacke Oa , Ob , Oc koje su simetricne sa sredistem O opisanog kruga u odnosu na prave odredene stranicama
BC, CA, AB.
Uputstvo: Ako obelezimo sa A1 , B1 , C1 sredista stranica BC, CA, AB, bice
OOa BC, BC k B1 C1 i B1 C1 k Ob Oc pa je OOa Ob Oc . Isto tako je OOb
Oc Oa i OOc Oa Ob , pa je tacka O ortocentar trougla Oa Ob Oc . S obzirom da
su tacke Oa , Ob , Oc poznate, mozemo konstruisati, najpre, ortocentar O trougla
Oa Ob Oc , a zatim simetrale duzi OOa , OOb , OOc koje odreduju trougao ABC.

Oc
B1

C1

Ob

A1

Oa

Slika 750
1949. Konstruisati trougao ABC ako su ate tri tacke P , Q, R u kojima
simetrale uglova A, B, C seku opisani krug.
754

Uputstvo: Slicno kao u prethodnom zadatku, srediste S opisanog kruga


trougla P QR je ortocentar trougla P QR.(slika 751.)

A
Q

R
S

Slika 751
1950. Date su tri konkurentne prave sa , sb , sc i na pravoj sa tacka A.
Odrediti na pravama sb , sc tacke B i C takve da prave sa , sb , sc budu simetrale
unutrasnjih i spoljasnjih uglova trougla ABC.
Uputstvo: Tacke A1 i A2 simetricne sa tackom A u odnosu na prave sb i sc
pripadaju pravoj BC. Stoga su B i C tacke u kojima prava A1 A2 sece prave sb
i sc . (slika 752 i slika 753)
sb

sc

A
sb
sc

A1

A2

Sa
B

A2

sa

A1

C
sa

Slika 752

Slika 753

1951. Dati su krug k sa sredistem S i tri prave sa , sb , sc koje se seku u tacki


S. Odrediti na pravama sa , sb , sc tacke A, B, C takve da prave BC, CA, AB
budu tangente kruga k.
Uputstvo: Ako obelezimo sa A bilo koju tacku prave sa razlicitu od tacke S,
kao u prethodnom zadatku odredimo na pravama sb i sc tacke B i C takve da
prave sa , sb , sc budu simetrale unutrasnjih i spoljasnjih uglova trougla A B C .
Tangente kruga k paralelne sa pravama B C , C A , A B odreduju trazeni trougao ABC. (slika 754)
755

sb

sc

A
A1

C
B

A2

sa

Slika 754
1952. Date su tri konkurentne prave sa , sb , sc i na pravoj sa tacka A1 .
Konstruisati trougao ABC kome je tacka A1 srediste stranice BC i kome su
prave sa , sb , sc simetrale stranica BC, CA, AB.
1953. Dat je konacan skup od n tacaka s1 , . . . , sn . Konstruisati n tougao
A1 . . . An kome su prave s1 , . . . , sn respektivno simetrale unutrasnjih ili spoljasnjih uglova A1 , . . . , An . Analizirati posebno slucaj kada je n paran broj i
slucaj kada je n neparan broj.
1954. Dat je konacan skup od n komplanarnih pravih s1 , . . . , sn . Konstruisati n-tougao A1 . . . An kome su prave s1 , . . . , sn respektivno simetrale stranica
A1 A2 , . . . , An A1 . Analizirati posebno slucaj kada je n paran i slucaj kada je n
neparan broj.
1955. Dati su tacka S1 i konacan skup od n 1 pravih s2 , . . . , sn . Konstruisati n-tougao A1 . . . An kome se srediste stranice A1 A2 poklapa s tackom S, a
simetrale stranica A2 A3 , . . . , An A1 poklapaju respektivno s pravama s2 , . . . , sn .
1956. U dati krug k upisati n-tougao A1 . . . An kome su stranice A1 A2 , . . . , An A
1 respektivno paralelne s datim pravama p1 , . . . , pn . Analizirati posebno slucaj
kada je n neparan i slucaj kada je n paran broj.
1957. U dati krug k upisati n-tougao A1 . . . An kome prava odredena stranicom A1 A2 sadrzi datu tacku P1 , dok su stranice A2 A3 , . . . , An A 1 respektivno
paralelne s datim pravama p2 , . . . , pn . Analizirati posebno slucaj kada je n paran
i slucaj kada je n neparan broj.

13.6

Rotacija

1958. Date su dve linije a i b od kojih svaka predstavlja pravu ili krug i
tacka S. Odrediti na linijama a i b tacke A i B takve da ugao ASB bude jednak
datom uglu a duzi SA i SB srazmerne dvema datim duzima m i n.
Resenje: (a)Ako bi problem imao resenje, tada bi, prema definiciji, u obrtnoj
slicnosti (S, , m : n) tacki A odgovarala tacka B. S obzirom da se tacka A
nalazi na pravoj a, bice i tacka B na pravoj a , koja u pomenutoj transformaciji
odgovara pravoj a. Da bismo konstruisali pravu a , obelezimo sa M podnozje

756

upravne iz tacke S na pravoj a, sa SM polupravu koja sa polupravom SM


sa bilo koje strane zahvata ugao jednak uglu i sa M tacku te poluprave
takvu da je SM : SM = m : n. Kako u pomenutoj transformaciji tacki M
odgovara tacka M , pravoj a, koja je u tacki M upravna na polupravu SM
odgovara prava a koja je u tacki M upravna na polupravu SM . Prave a i b
ili imaju zajednickih tacaka ili ne. Pretpostavimo da imaju zajednickih tacaka,
neka je to tacka B. Ako obelezimo sa SA polupravu takvu da je ugao M SM
jednak i istosmeran sa uglom ASB, bice ugao M SA jednak sa uglom M SB. U
pravouglom trouglu M SB ugao M SB je ostar, pa je i ugao M SA ostar. Stoga,
prava a upravna na krak SM sece drugi krak ugla M SA u tacki A. Dokazimo
da su A i B trazene tacke.
Kako je ASB = M SM i M SM = , bice i ASB = . Iz slicnosti
trouglova M SA i M SB imamo da je SA : SB = SM : SM . No SM : SM =
m : n, pa je i SA : SB = m : n.
Ako polupravu SM konstruisemo sa druge strane poluprave SM , tada umesto prave a dobijamo neku drugu pravu a . Istim postupkom dokazujemo da
ce i svakoj zadatoj tacki B pravih a i b odgovarati na pravoj a tacka A takva
da ASB = i SA : SB = m : n. Stoga broj resenja datog problema zavisi od
broja zajednickih tacaka prave b sa pravama a i a . Prema tome, ako prava b
sece obe prave a i a , problem ima dva resenja; ako prava b sece jednu od tih
pravih a i a , a sa drugom je paralelna, problem ima jedno resenje; ako je prava
b paralelna sa obema pravama a i a problem nema resenja. Specijalno, ako
se prava b poklapa sa nekom od pravih a i a problem ima beskonacno mnogo
resenja.(slika 755)

M
A

B
a

Slika 755
(b)Ako bi problem imao resenje, tada bi, prema definiciji, u obrtnoj slicnosti
(S, , m : n) tacki A odgovarala tacka B. S obzirom da se tacka A nalazi na
krugu a, bice tacka B na krugu a koji u pomenutoj transformaciji odgovara
krugu a.
Da bismo konstruisali krug a , obelezimo sa O srediste kruga a, sa SO
polupravu koja sa polupravom SO sa bilo koje strane zahvata ugao jednak
uglu i sa O tacku te poluprave takvu da je SO : SO = m : n. Kako u
pomenutoj transformaciji tacki O odgovara tacka O , krugu a(O, r) odgovara
krug a (O , r ), gde je r : r = m : n. Krugovi a i b ili imaju zajednickih tacaka
ili ne. Pretpostavimo da imaju zajednickih tacaka, neka je tacka B bilo koja od
njih. Obelezimo sa SA polupravu takvu da je ugao OSO jednak i istosmeran
757

sa uglom ASB i sa A tacku te poluprave takvu da je SA : SB = SO : SO ,


zatim dokazimo da su A i B trazene tacke.
Najpre dokazimo da je tacka A pripada krugu a. Iz jednakosti i istosmernosti
uglova OSO i ASB sledi da je OSA = O SB. Odatle i iz proporcije SO :
SO = SA : SB sledi da je SOA SO B, pa je OA : O B = SO : SO
i, prema tome, OA : r = m : n. Iz ove jednakosti i proporcije r : r = m : n
sledi da je OA = r, pa je tacka A tacka kruga a. Kako je ASB = OSO i
OSO = , bice i ASB = . Najzad, iz proporcija SA : SB = SO : SO i
SO : SO = m : n sledi da je SA : SB = m : n.
Ako polupravu SO konstuisemo sa druge strane poluprave SO, tada umesto
kruga a dobijamo neki drugi krug a . Istim postupkom dokazuje se da ce i
svakoj zajednickoj tacki B krugova a i b odgovarati na krugu a tacka A takva
da je ASB = i SA : SB = m : n. Stoga, broj resenja datog problema
zavisi od broja zajednickih tacaka kruga b sa krugovima a i a . Prema tome,
problem moze da ima cetiri, tri, dva, jedno ili nijedno resenje. Specijalno, ako se
krug b poklapa sa nekim od krugova a i a , problem ima neograniceno mnogo
resenja.(slika 756)

Slika 756
(v)Koristiti transformaciju iz jednog od prethodna dva slucaja.
1959. Date su dve linije a i b od kojih svaka predstavlja pravu ili krug i tacka S.
Odrediti na linijama a i b tacke X i Y takve da ugao XSY bude jednak datom
uglu , a proizvod duzi SX i SY jednak kvadratu date duzi r.
Resenje: (a)Ako bi problem imao resenje, tada bi, prema definiciji, u obrtnoj inverziji (S, , r) tacki X odgovarala tacka Y . S obzirom da se tacka X
nalazi na pravoj a, tacka Y se nalazi na liku a koji u pomenutoj transformaciji
odgovara pravoj a.
Stoga, da bismo konstruisali tacke X i Y , konstruisimo, najpre, pravu a1 koja
u rotaciji R(S, ) odgovara pravoj a, zatim lik a koji u inverziji kruga I(S, r)
odgovara pravoj a1 . Prema poznatom stavu iz inverzije, lik a predstavlja pravu
758

ili krug zavisno od toga da li je tacka S na pravoj a1 ili ne, odnosno da li je


tacka S na pravoj a ili ne. Na slici 757. predstavljen je slucaj kada tacka S nije
na pravoj a. Linije a i b ili imaju zajednickih tacaka razlicitih od tacke S ili
nemaju zajednickih tacaka. Pretpostavimo da imaju, i neka je to tacka Y . S
obzirom da tacka Y pripada liku b, ona u obrtnoj inverziji (S, , r) odgovara
nekoj tacki X prave a.
Slucajeve pod (b) i (v) resavaju se analognim postupkom.

X
a

M1
a1
M

Slika 757
1960. Dati su prava ili krug l i tacka A. Odrediti na liniji l tacke B i C takve
da ugao BAC bude jednak datom uglu , a duzi AB i AC srazmerne dvema
datim duzima m i n.
1961. Dati su prava l ili krug l i tacka A. Odrediti na liniji l tacke B i C
takve da ugao BAC bude jednak datom uglu i da proizvod duzi AB i AC
bude jednak kvadratu date duzi r.
Konstruisati trougao ABC kada znamo:
1962. A, ha , bc .
1963. A, r, bc .
1964. A, a, bc .
1965. B C, r, bc .
1966. B c, ha , bc .
1967. B c, la , bc .
1968. a, r, bc.
1969. ha , la , bc .
1970. la , la , bc.
1971. Dat je trougao M AN i u njegovoj ravni tacka S1 . Odrediti na kracima
AM i AN tog ugla tacke B i C takve da ABC bude trougao kome je tacka S1
srediste Ojlerovog kruga.

759

Uputstvo: Sredista A1 , B1 , C1 stranica BC, CA, AB pripadaju Ojlerovom


krugu trougla ABC, pa je B1 S1 C1 = 2B1 A1 C1 . No, B1 A1 C1 = M AN ,
pa je i B1 S1 C1 = 2M AN . Stoga mozemo, kao u zadatku 1958, na kracima
AM i AN odrediti tacke C1 i B1 takve da je S1 C1 = S1 B1 i C1 S1 B1 =
2M AN . Tacke B i C bice simetricne sa tackom A u odnosu na tacke C1 i B1 .
(slika 758)
A
C

B
B1
S1

C1

B
M

A
A1

Slika 758
1972. U dati paralelogram ABCD upisati paralelogram P QRS kome su
dijagonale P R i QS srazmerne dvema datim duzima m i n, a jedan od uglova
koje odreduju dijagonale jednak datom uglu.
1973. U dati paralelogram ABCD upisati paralelogram P QRS kome su
stranice P Q i RS srazmerne dvema datim duzima m i n, a dijagonale P R i QS
seku se pod uglom jednakim datom uglu .
1974. U dati paralelogram ABCD upisati paralelogram P QRS kome je
jedan od uglova izmedu dijagonala jednak datom uglu , a proizvod dijagonala
jednak kvadratu date duzi r.
1975. U dati paralelogram ABCD upisati pravougaonik P QRS kome dijagonale zahvataju ugao jednak datom uglu .
1976. U dati paralelogram ABCD upisati romb P QRS kome su dijagonale
srazmerne dvema datim duzima m i n.
Uputstvo: Srediste romba P QRS poklapa se sa sredistem O datog paralelograma ABCD. Stoga se kao u problemu 1724 na stranicama AB i BC mogu
odrediti tacke P i Q takve da je P OQ prav i OP : OQ = m : n. Tacke R
i S su simetricne sa tackama P i Q u odnosu na tacku O.
1978. Konstruisati tetivan cetvorougao ABCD kome su stranice AB, BC,
CD, DA respektivno jednake datim duzima.
1979. Konstruisati cetvorougao ABCD kome su stranice AB, BC, CD, DA
respektivno jednake datim duzima a, b, c, d a zbir naspramnih uglova B i D
jednak datom uglu .
1980. Konstruisati cetvorougao ABCD kome su uglovi B i D jednaki datim
uglovima i , stranice BC i CD jednake datim duzima b i c, a stranice AB i
AD srazmerne dvema datim duzima m i n.

760

1981.U dati kvadrat ABCD upisati kvadrat P QRS kome je stranica jednaka
datoj duzi l. Uputstvo: Ako je tacka O srediste kvadrata ABCD i prav ugao,
tada u obrtnoj slicnosti (O, , AB : 1) kvadratu ABCD odgovara kvadrat
P QRS (Sl. 759).

Sl. 759
1982. Konstruisati kvadrat ABCD takav da prave odredene njegovim stranicama AB, BC, CD, DA respektivno sadrze date tacke P , Q, R, S.
1983. Dati su krug k i dve tacke A i B. Konstruisati tangentu t kruga k
takvu da odstojanja AM i AN tacke A od prave t i upravne kroz tacku B na
pravoj t budu srazmerne dvema datim duzima m i n.
Uputstvo. Ako obelezimo sa prav ugao, tada u obrtnoj slicnosti (A, , n :
m) pravoj BN odgovara prava t (Sl. 760).

761

Sl. 760
1984. Dati su krug k sa sredistem S, dve tacke A, B i ugao . Odrediti na
krugu k tacke C i D takve da je AC
= BD i CSD
= .
Uputstvo: U rotaciji R(S, ) tacki C odgovara tacka D, a tacki A izvesna
tacka A takva da je BDA = . Stoga treba konstruisati skup temena svih
uglova koji su jednaki sa uglom i kojima kraci sadrze tacke B i A . Svaka od
presecnih tacaka tog skupa sa krugom k predstavlja trazenu tacku D (Sl. 761).

762

Sl. 761
1985. Dat je krug l i na njemu tacka A. Odrediti na krugu tacke B i C
takve da ugao BAC bude jednak datom uglu , a zbir ili razlika tetiva AB i
AC jednaka datoj duzi D.
Uputstvo: Ako su p i q dve poluprave koje imaju zajednicki kraj A, zahvataju ugao jednak sa uglom i seku krug l u nekim tackama B i C , tada se kao
u problemu 1986 moze na krugu l odrediti tacka A takva da je zbir ili razlika
tetiva A B i A C jednaka duzi ?. Rotacija oko sredista O kruga l za ugao
A OA prevodi tacke A , B , C u tacke A, B, C.
1986. Date su dve prave m i n i tacka S. Konstruisati krug k koje je srediste
S i koji sece pravu m u tackama A, B i pravu n u tackama C, D takvim da je
zbir ili razlika tetiva AB i CD jednaka datoj duzi l.
1987. Data su dva koncentricna kruga k1 , k2 i tacka P . Konstruisati pravu
s koja sadrzi tacku P sece krug k1 u tackama A, B i krug k2 u tackama C, D
takvima da tetive AB i CD budu srazmerne dvema duzima m i n.
1988. Data su dva koncentricna kruga k1 , k2 i tacka P . Odrediti na krugovima k1 i k2 tacke X1 i X2 takve da duz X1 X2 budu jednake datoj duzi l, a
ugao X1 P X2 jednak datom uglu .

763

1989. Dati su dva kruga k1 i k2 , tacka O i ugao . Kroz tacku O konstruisati


dve prave p1 i p2 koje zahtevaju ugao jednak s uglom i na kojima krugovi k1
i k2 odsecaju jednake tetive.
Uputstvo: U rotaciji (O, ), krugu k2 odgovara izvestan krug k2 , dok pravoj p2 odgovara prava p1 . Sad mozemo kao u problemu 1985, konstruisati pravu
p1 koja sadrzi tacku O i na kojoj krugovi k1 i k2 odsecaju jednake tetive (Sl.
762).

Sl. 762
1990. Date su dve prave a, b i tacka S. Konstruisati dva kruga k1 i k2 koji se
medu sobom dodiruju u tacki S, kojima su poluprecnici srazmerni dvema datim
duz ima m i n, i od kojih prvi dodiruje pravu a, a drugi pravu b.
1991. Data su dva kruga k1 , k2 sa sredistima S1 , S2 i dve tacke M1 , M2 .
Odrediti na krugovima k1 i k2 tacke X1 i X2 takve da ugao S1 M1 X1 bude

764

jednak s uglom S2 M2 X2 , a jedan od uglova koje zahvataju prave S1 X1 i S2 X2


jednak je sa datim uglom .
1992. Data su dva kruga k1 , k2 sa sredistima S1 , S2 i dve tacke M1 , M2 .
Odrediti na krugovima k1 i k2 tacke X1 i X2 takve da je razlika uglova S1 M1 X1
i S2 M2 X2 jednaka s datim uglom , a jedan od uglova koje odreduju prave S1 X1
i S2 X2 jednak s datim uglom .
1993. Date su dve konkurentne prave a, b i dve tacke C, D. Odrediti na
pravama a i b tacke A i B takve da su duzi AC i BD srazmerne dvema datim
duzima m i n, a jedan od uglova koje odreduju prave AC i BD jednak datom
uglu .
1994. Date su tri paralelne prave a, b, c i krug d. Konstruisati kvadrat
ABCD kome temena A, B, C, D pripadaju respektivno linijama a, b, c, d.
1995. Dati su tri koncentricna kruga a, b, c i prava d. Konstruisati kvadrat
ABCD kome temena A, B, C, D pripadaju respektivno linijama a, b, c, d.
1996. Konstruisati srediste S obrtne slicnosti dvaju obrtno slicnih likova
i ako su data dva para odgovarajucih tacaka A, A i B, B .
1997. Date su dve prave a, b na njima respektivno tacke A, B i van tih pravih
tacka P . Konstruisati pravu p koja sadrzi tacku P i sece prave a i b u tackama
X i Y takvima da su duzi AX i BY srazmerne dvema datom duzima m i n.
Uputstvo: Koristiti obrtnu slicnost u kojoj tackama A i X odgovaraju tacke
B i Y.
1998. Date su tri prave a, b, c i na pravama a, b respektivno tacke X i Y
takve da prave XY i c budu medu sobom paralelne, a duzi AX i BY srazmerne
dvema datim duzima m i n.
Uputstvo: Analizirajmo slucaj kada se prave a i b seku u nekoj tacki O. Ako
pretpostavimo da na pravama a i b postoje tacke X i Y takve da je XY || c
i AX : BY = m : n. Zatim obelezimo sa S srediste obrtne slicnosti u kojoj
tackama A i B odgovaraju tacke X i Y , tada u toj obrtnoj slicnosti pravoj
AB odgovara prava XY , a podnozju P prave upravne iz tacke S na pravoj AB
odgovara podnozje Q upravne iz tacke S na pravoj XY . Stoga je ugao ASX
jednak i istosmeran sa uglom P SQ. No ugao P SQ je poznat, pa je poznat i
ugao ASX. Problem u opstem slucaju ima dva resenja. Slucaj kada su prave a
i b paralelne je jednostavniji, neka ga citalac analizira sam.

765

Sl. 763
1867. Uputstvo. Razlikova`cemo tri slucaja.
1. Neka je
1
S(AP B) < S(ABC)
2
i
1
S(AP C) < (ABC).
2
Ako obelezimo sa D,E,F sredista stranica BC,CA,AB, tada iz predpostavljenih
nejednakosti sleduje da se tacka P nalazi u paralelogramu AF DE, a trazena
tacka Q na stranici BC. Ako je pri tome tacka P na dijagonali AD pomenutog
paralelograma, tacka Q se poklapa sa tackom D. Ako je tacka P u jednom od

trouglova AF D i ADE, npr. trouglu ADE tacka Q je unutar duzi BD te duZ


P Q sece duz AD u nekoj tacki R. Da bi bilo
S(ABP Q) = S(AP QC),
mora biti
S(AP R) = S(DQR),
766

dakle i
S(AP D) = S(DQP ).
No, to `ce nastupiti samo u slucaju kada je AQ k P D. Otuda sleduje i postupak
za konstrukciju tacke Q.

F
R
B

sl.728
2. Ako je

1
S(ABC),
2
trazena tacka Q poklapa se sa tackom B.
3. Ako je
1
S(AP C) > S(ABC),
2
tada je tacka P u trouglu BF D, gde su kao i u prvom slucaju tacke F i D
sredista stranica AB i BC. Pri tome tacka Q pripada stranici AC, te duz P Q
sece duz AD u nekoj tacki R. Da bi bilo
S(AP B) =

S(AP Q) =

1
S(ABC) = S(ADC),
2

mora biti
S(AF R) = S(CQRD).
Ako zatim obelezimo sa K tacku prave KC takvu da je QD k CK, bi`ce
S(CQRD) = S(KQR),
dakle i
S(AP R) = S(KQR).
Otuda je
S(AP K) = S(QP K),
767

pa je P K k AC. Ovaj uslov daje i postupak za konstrukciju tacke Q.

F
R

P
B

D
K

sl.729
1868. Uputstvo. Ako na pravoj CD odredimo tacke M i N takve da je
P C k BM i P D k AN , bi`ce srediste Q duzi M N na duzi CD ili na njenom
podnozju. Ako je tacka Q na duzi CD, tada je(v. sl. 730)
S(P BCQ) = S(P M Q) = S(P N Q) = S(P ADQ).

A
P

768

sl.730
Ako je tacka Q na produznoj duzi CD, npr. iza tacke D u odnosu na tacku C,
a R tacka duzi AD takva da je AN k QR, bi`ce(v.sl. 731)
S(P BCDR) = S(P M Q) = S(P N Q) = S(P N D)S(P QD) = S(P AD)S(P RD) = S(AP R).
Stoga je trazena prava u prvom slucaju odredena tackama P i Q, a u drugom
slucaju tackama P i R.

Q
D
R

C
M

A
P

sl.731
1869. Uputstvo. Obelezimo sa A1 Ai+1 bilo koju od stanica poligonske
povrsi (A1 ...An ) koje se ne susticu u temenu A1 , sa A2 tacku prave A3 A4 takvu
da je A2 A2 k A1 A3 , sa A2 tacku prave A4 A5 takvu da je A2 A2 k A1 A4 , ...,
(i3) (i2)
(i2)
k A1 Ai , sa An tacku
A2
tacku prave Ai Ai+1 takvu da je A2
sa A2

prave An1 An2 takvu da je An An k A1 An3 , sa An tacku prave An2 An3


(ni1)
takvu da je An An k A1 A4 , ..., sa An
tacku prave Ai Ai+1 takvu da je
(ni2) (ni1)
(i2) (ni1)
An
An
k A1 Ai+1 . Neka su zatim F1 , ..., Fk tacke duzi A2
An
(i2)
(ni1)
]i
takve da je [A2
P1 ...Pk1 An
(i2)

A2

P1 = P1 F2 = ... = Fk1 An(ni1) .

Svaka od tacaka Pj za j = 1, ..., k 1 pripada duzi Ai Ai+1 . Konstruisimo


pravu A1 Pj . Ako je tacka Pj na produznoj duzi A1 Ai+1 , npr. iza A1 u odnosu
na Ai+1 , odredimo na pravoj Ai1 Ai tacku P1 takvu da je Pj Pj k A1 Ai . Pri
tome tacka Fj pripada stranici Ai1 Ai ili je iza Ai1 u odnosu na Ai . Ako
tacka Pj pripada stranici Ai1 Ai konstruisimo pravu A1 Fj . Ako je tacka Pj
iza Ai1 u odnosu na Ai odredimo na pravoj Ai2 Ai1 tacku Pj takvu da je
Pj Pj k A1 Ai1 . Pri tome tacka Pj pripada stranici Ai2 Ai1 ili je iza Ai2
u odnosu na Ai1 . Ako tacka Fj pripada stranici Ai2 Ai1 konstr. pravu
769

A1 Fj , ako se nalazi iza Ai2 u odnosu na Ai1 , postupak mozemo nastaviti


(

dok se ne dobije tacka Pj l) na nekoj stranici date poligonske povrsi. Nije tesko
sad dokazati da konstruisane prave razlazu poligonsku povrs na k ekvivalentnih
povrsi(v. sl. 732).

A1

A2
A6

A2
A3

A2

P 1A4

P2

A5

A6

sl.732
1872. Kroz presecnu tacku P datih krugova k1 i k2 konstruisati pravu p na
kojoj krugovi k1 i k2 odsecaju jednake tetive.
Uputstvo. Krug k1 koji je simetrican s krugom k1 u odnosu na tacku P sece
krug k2 u tacki P i jos nekoj tacki R. Pri tome prava P R sece krug k1 u tacki
P i jos jednoj tacki Q koja je simetricna s tackom R u odnosu na P (sl.733).

770

A
M
Q

Slika 733
1876. Dat je ugao XAY i u njegovoj ravni tacka S1 . Konstruisati trougao
ABC kome se srediste Ojlerovog kruga poklapa s tackom S1 , ortocentar H
nalazi na pravoj AX, a srediste upisanog kruga na polupravoj AY .
Uputstvo. Ako obelezimo sa AX polupravu simetricnu sa polupravom AX
u odnosu na pravu AY i sa s pravu simetricnu s pravom AX u odnosu na tacku
S1 , tada je tacka O u kojoj poluprava AX sece pravu s srediste kruga opisanog
oko trougla ABC (sl. 734).

H
B

P
B

Slika 734
1897. Date su dve razne prave a i b i izvan njih tacka P . Na pravoj a odrediti
771

tacku Q takvu da duz P Q bude jednaka zbiru odstojanja tacaka P i Q od prave


b.
Uputstvo. Ako obelezimo sa P tacku simetricnu s tackom P u odnosu na
pravu b i sa b pravu koja sadrzi P a uporedna je s pravom b, bice duz ? jednaka odstojanju tacke Q od prave b , pa se problem svodi na problem...
1901. Dati su prava p i sa iste strane od te prave dve tacke A, B. Odrediti
na pravoj p tacku X takvu da konveksni ugao AXB bude maksimalan.
Uputstvo. Ako obelezimo sa k proizvoljan krug kroz tacke A i B, sa X bilo
koju tacku kruga k razlicitu od tacaka A i B, sa Y bilo koju tacku koja se
nalazi u krugu k a s one strane prave AB skoje je tacka X i sa Z bilo koju
tacku koja se nalazi izvan kruga k a sone strane prave AB skoje je tacka X ,...

B
A

B
S

O
C

sl.735
1872. Kroz presecnu tacku P datih krugova k1 i k2 konstruisati pravu p na
kojoj krugovi k1 i k2 odsecaju jednake tetive.

772

S1
P

k1

S2
R
k2

Slika 1.
Uputstvo: Krug k1 koji je simetrican s krugom k1 u odnosu na tacku P sece krug
k2 u tacki P i u jos nekoj tacki R. Pri tome prava P R sece krug k1 u tacki P i
jos jednoj tacki Q koja je simetricna s tackom R u odnosu na tacku P . (Slika 1.)
1876. Dat je ugao XAY i u njegovoj ravni tacka S1 . Konstruisati trougao
ABC kome se srediste Ojlerovog kruga poklapa sa tackom S1 , ortocentar H
nlazi se na pravoj AX a srediste upisanog kruga na polupravoj AY .

A
B2

C2

S1
O

A2

A1

C
X

N
X

Slika 2.
Uputstvo: Ako obelezimo sa AX polupravu simetricnu s polupravom AX u
odnosu na pravu AY i sa s pravu simetricnu s pravom AX u odnosu na tacku
S1 , tada je tacka O u kojoj poluprava AX sece pravu s srediste kruga opisanog
oko trougla ABC. (Slika 2.)

773

1897. Date su dve razne prave a i b i izvan njih tacka P . Na pravoj a odrediti
tacku Q takvu da duz P Q bude jednaka zbiru odstojanja tacaka P i Q od prave
b.
Uputstvo: Ako obelezimo sa P tacku simetricnu s tackom P u odnosu na pravu
b i sa b pravu koja sadrzi tacku P a uporedna je s pravom b, bice jednaka
odstojanju tacke Q od prave b , pa se problem svodi na problem...
1901. Dati su prava p i sa iste strane od te prave tacke A i B. Odrediti na
pravoj p tacku X takvu da konveksni ugao AXB bude maksimalan.

O1

Z
O2
Y

Slika 3.
Uputstvo: Ako obelezimo sa k proizvoljan krug kroz tacke A i B, sa X bilo koju
tacku kruga k razlicitu od tacaka A i B, sa Y bilo koju tacku koja se nalazi
u krugu k a s one strane prave AB s koje je tacka X i sa Z bilo koju tacku
koja se nalazi izvan kruga k a s one strane prave AB s koje je tacka X , bice
AZ B < AX B < AY B. Iz ovih nejednakosti sleduje da ce tacka u kojoj
manji krug kroz tacke A i B dodiruje pravu p. Na taj nacin se ovaj problem
svodi na...
1902. Date su tri prave a,b i c i na pravama a i b respektivno tacke A i B.
Odrediti na pravama a i b tacke X i Y takve da je XY paralelno c i AX = BY .

774

a
c

X
A

A
X
Y

Slika 4.
Uputstvo: Pretpostavimo najpre da se prave a i b seku i da na pravama a i b
postoje tacke X i Y takve da je XY k c i AX = BY . Ako obelezimo sa s pravu
koja sadrzi srediste S duzi AB i koja je paralelna sa simetralom jednog od uglova
koji su odredeni pravama a i b, a sa A i X tacke simetricne s tackama A i X
u odnosu na pravu s, bice cetvorougao BY X A paralelogram kome su stranice
BA i Y X paralelne s pravom s. Pri tom su kod trougla XX Y poznati stranica Y X i nalegli uglovi Y i X , pa se moze konstruisati najpre njemu pomerno
podudaran trougao KA B, zatim tacke X i Y . Problem u opstem slucaju ima
dva resenja. Slucaj kada su prave a i b paralelne neka citalac analizira sam.
1903. Date su dve prave a i b i na njima respektivno tacke A i B. Odrediti
na pravama a i b tacke X i Y takve da je AX = BY , a duz XY jednaka datoj
duzi l.
Uputstvo: Postupak isti kao u prethodnom problemu, samo sto kod trougla
XX Y nisu poznati stranica Y X i nalegli uglovi Y i X , vec stranice Y X , Y X
i ugao X . Problem u opstem slucaju ima dva resenja.
1904. Date su tri prave a,b i c i na pravama a,b respektivno tacke A,B.
Odrediti na pravama a i b tacke X i Y takve da je AX = BY , a srediste duzi
XY na pravoj c.
Uputstvo: Postupak isti kao . . ., samo sto trugao XX Y mozemo konstruisati
neposredno, jer su poznata sredista Z i Z stranica XY i XX , jer je tacka Z
istovetna s tackom u kojoj se seku prave s i c, a tacka Z na pravoj s takva da
je duz ZZ istosmerna i jednaka polovini poznate duzi BA . Zadatak u opstem
775

slucaju ima dva resenja.


1936. Dati su prava p i sa iste strane od te prave dve tacke A,B. Odrediti
na pravoj p tacku X takvu da zbir duzi AX i BX bude minimalan.

X
B

Slika 5.
Uputstvo: Ako obelezimo sa B tacku simetricnu s tackom B u odnosu na pravu
p i sa X promenljivu tacku prave p, bice AX + BX = AX + B X . No zbir
AX + BX bice najmanji ako je zbir AX + B X najmanji, tj. ako je tacka X
presek duzi AB s pravom p.
1937. Dati su prava p i sa raznih strana od te prave dve tacke A,B. Odrediti
na pravoj p tacku X takvu da razlika njenih rastojanja od tacaka A i B bude
minimalna.

X
B

776

Slika 6.
Uputstvo: Ako obelezimo sa B tacku simetricnu s tackom B u odnosu na pravu
p i sa X promenljivu tacku prave p, bice AX BX = AX B X . Stoga ce
razlika |AX BX | biti maksimalna ako je i razlika AX B X maksimalna.
No to ce nastupiti u slucaju kada je tacka X presek duzi AB s pravom p.
1938. Data je prava p i izvan nje date su dve tacke M i N . Odrediti na
pravoj p dve tacke X i Y takve da duz XY bude jednaka datoj duzi l i da zbir
duzi M X,XY ,Y N bude minimalan.

N1

Slika 7a .
Uputstvo: Tacke M i N su s raznih strana ili s iste strane od prave p.
U prvom slucaju (Slika 7a ) ako je N1 tacka takva da je N N1 k p i N N1 = l, duz
M N1 sece pravu p u tacki X, tacka Y je cetvrto teme paralelograma N N1 XY .
U drugom slucaju (Slika 7b ):

777

N1

Y
N 1

Slika 7b .
Ako je N1 tacka takva da je N N1 k p i N N1 = l, a N1 tacka simetricna s tackom
N1 u odnosu na pravu p, duz M N1 sece pravu p u tacki X, tacka Y je cetvrto
teme paralelograma N N1 XY .
1939. Data je prava p i izvan nje date su dve tacke M i N . Odrediti na
pravoj p tacku X takvu da duzi M X i N X odreduju s pravom p jednake uglove.

Slika 740.
Upustvo. Ako su tacke M i N s raznih strana prave p, duz M N sece pravu p
u tacki X; ako su take M i N s raznih strana od prave p, a M tacka simetricna
s tackom M u odnosu na pravu p, duz M N sece pravu p u tacki X (slika 740).

778

1940. Data je prava P Q i izvan nje date su dve tacke M i N . Odrediti na


pravoj P Q tacku X takvu da je M XP = 2N XQ.

N
p

X
N

N
m

Slika 741.
Upustvo. Ako su take M i N s raznih stana od prave p i ako je k krug sa
sredistem N koji dodiruje pravu p, bice X tacka u kojoj tangenta kruga k kroz
tacku M sece pravu p. Ako su tacke M i N s raznih strana od prave p i ako
je N tacka simetricna s tackom N u odnosu na pravu p, a k krug sa sredistem
N koji dodiruje pravu p, bice X tacka u kojoj tangenta kruga k kroz tacku M
sece pravu p (slika 741).
1941. Date su dve prave p i q a izvan njih dve tacke M i N . Odrediti na
pravoj p tacku A i na pravoj q tacke B i C takve da tacka M bude na pravoj
AB, tacka N na pravoj AC i da bude AB = AC.

779

A
M
M
N

Slika 742.

p
A

C
q

Slika 743.
Upustvo. Prave pi q se seku u nekoj tacki O, ili su medusobno uporedne.
U prvom slucaju (Slika 742), ako je M tacka simetricna s tackom M u odnosu
na pravu p, bice M AN =M AM +M AN =2OAM +BAC=2*(ABCAOB)+BAC=ABC+BCA+BAC-2*AOB=2*R-2*AOB, gde je R
prav ugao. Stoga je ugao M AH poznat, pa se moze konstruisati tackaA. U
drugom slucaju (Slika 743) prava AC sadrzi tacku N i tacku M koja je sime780

tricna sa tackom M u odnosu na pravu p. U oba slucaj aproblem ima resenje.


1942. Dati su prava p i izvan nje dve tacke A i B. Odrediti na pravoj p
tacku C takvu da trougao ABC bude minimalnog obima.

(a)

(b)
Slika 744.
Upustvo. Razlikovacemo dva slucaja: 1) Tacke A i B su sa raznih strana
prave p (Slika (a)). U tom slucaju duz AB sece pravu p u nekoj tacki C. Ako
obelezimo sa C bilo koju drugu tacku prave p, bice AB+BC+CA= 2ABAB+BC +C A.Stoga
tazeni prav ugao ABC degenerise dvostruku duz AB. 2) Tacke A i B su sa iste
strane prave p (Slika (b)). Ako obelezimo sa B tacku simetricnu s tackom B u
odnosu na pravu p, sa C tacku u kojoj duz AB sece pravu p i sa C bilo koju
drugu tacku prave p, bice
AB+BC+CA = AB+B C+CA = AB+B A < AB+B C +C A = AB+BC +C A.
Pri tome duz AB moze da bude upravna pravoj p, pa je i tacka C na pravoj
AB. Trazeni trougao ABC u tom slucaju degenerise u dvostruku duz AC ili
781

BC zavisno od toga da li je [ABC] ili [BAC].


1943. Dati su osrougli trougao ABC i na stranici BC tacka P . Odrediti na
stranicama CA i AB tacke Q i R takve da obim trougla P QR bude minimalan.

A
P1

R
P2

Q
B

Slika 745.
Upustvo. Ako obelezimo sa P1 i P2 tacke simetricne s tackom P u odnosu
na prave CA i AB, a sa Q iR promenljive tacke stranica AC i AB, bice
P Q = P Q i P R = P R pa je obim trougla P Q R jednak obimu izlomljene
linije P1 Q R P2 . Otuda sledi da ce obim trougla P Q R biti minimalan ako je
obim izlomljene linije P1 Q R P2 minimalan. Obim te linije bice minimalan ako
su tacke Q i R preseci duzi P1 P2 sa stranicama CA i AB.
1944. U dati ostrogli trougao P QR upisati trougao P QR minimalnog obima.

A
P2
P2

R
Q

R
Q

P1
P1

P P

Slika 746.
Upustvo. Ako obelezimo sa P bilo koju unutrasnju tacku stranice BC i sa
P1 i P2 tacke simetricne s tackom P u odnosu na prave CA i AB, tadas duz
P1 P2 sece stranice CA i AB tackama Q i R takvim da je za tu tacku P trougao
P Q R minimalnog obima. Pri tome je kod trougla AP1 P2 P1 AP2 = 2BAC,
AP1 = AP2 = AP i P1 P2 = P Q +Q R +R P .Dopustimo li da tacka P menja
polozaj ostajuci pri tome izmedu tacaka B i C, trougao AP1 P2 se takode menja
imajuzci uvek konstantan ugao P1 AP2 i jednake stranice AP1 i AP2 . Stranica
782

P1 P2 tog trougla bice ..................AP1 i AP2 najmanje.NO duzi AP1 i AP2 bice
najmanje ako je njima jednaka duz AP namanja, tj. ako se tacka P poklapa
sa P koja je podnozje visine iz temena A.
1945. Date su dve prave p i q i izvan njih tacka A. Odrediti na pravama
p i q tacke B i C takve da obim trougla ABC bude minimalan.
Upustvo. Razlikovacemo cetiri slucaja:

Slika 747.
1. slucaj
1) Prave p i q su paralelne. Ako obelezimo sa B i C podnozja zajednicke
normale iz tacke A na pravama p i q, a sa B i C proizvoljne tacke pravih p i q,
bice AB AB, AC AC, B C BC, pa je (Slika 747a) AB + BC + CA
A B + B C + C A . Stoga trazeni trougao ABC se degenerise u dvostruku duz
AC, AB, BC zavisno od toga da li je [ABC], [ACB], [BAC].

q
A2

C
C

B
A1

783

Slika 747.
2. slucaj
2) Prave p i q se seku pod ostrim uglom, a tacka A je u tom ostrom uglu
P OQ. Najore dokazimo da tacke B i C ne mogu na produzenjima krakova tog
ugla. Ako bi se obe tacke B i C nalazile produzenjima polupravih OP i OQ,
bice OAAB i OAAC, pa je 2OAAB + BC + CA. Ako je jedna od tih dveju
tacaka B i C na jednom kraku ugla P OQ a druga na podnozju drugog kraka,
npr. tacka B na produzenju kraka OP , a tacka C na kraku OQ i ako obelezimo
sa B1 tacku simetricnu sa tackom B u odnosu na tacku O, bice AB1 AB i
CB1 CB, pa je AB1 + B1 C + CAAB + BC + CA. na taj nacin obe tacke B i C
pripadaju polupravama OP i OQ. Ako obelezimo sa A1 i A2 tacke simetricne s
tackom A u odnosu na prave p i q, duz A1 A2 sece poluprave OP i OQ u izvesnim
tackama B i C. Ako zatim obelezimo sa B i C proizvoljne tacke polupravih
OP i OQ, bice (Slika 747b) AB + BC + CA=A1 B + BC + CA2 =A1 A2
A1 B + B C + C A2 =AB + B C + C A.

A2
A
C
O

A1
q

Slika 747.
3. slucaj
3) Prave p i q su upravne. U tom slucaju tacka A je u jednom od pravih
uglova koji su odedeni pravama p i q, npr. u uglu P OQ. Kao u prethodnom
slucaju tacke B i C ne mogu biti na produzenjima polupravih OP i OQ. No
duz A1 A2 odredena tackama A1 i A2 koje su simetricne s tackom A u odnosu na
prave p i q sadrzi tacku O u kojoj se seku prave p i q, pa je za proizvoljne tacke
B i C polupravih OP i OQ tacka O srediste hipotenuze pravouglog trougla
AA1 A2 , i prema tome AB + B C + C A=A1 B + B C + C A2 A1 A2 =2OA.
Stoga trazeni trougao degenerise u dvostruku duz OA.

784

p
q

A
C
p

p
q

C
A
p

Slika 747.
4. slucaj
4) Slucaj kada se prave a i b seku ali nisu upravne a tacka A nalazi u jednom
od tupih uglova, ostavljamo da citalac analizira sam.
1946. Na trima datim pravama a, b, c odrediti tacke P , Q, R takve da zbir
duzi P Q, QR, RP bude minimalan.
Razlikovacemo sledeca dva slucaja:

785

b
P
A
Q
R

Slika uz zadatak 1946.a


1o Prave a, b, c se seku i obrazuju ostrougli trougao ABC (vidi sliku 1946.a).
Prema zadatku ..., podnozja visina trougla ABC odreduju trougao P QR koji
od svih upisanih trouglova u trougao ABC ima minimalni obim. Dokazimo da
su i u ovom problemu P , Q, R trazene tacke. Obelezimo sa P bilo koju tacku
prave a koja se ne nalazi na stranici BC trougla ABC, vec npr. iza tacke B u
odnosu na tacku C, sa Q i R promenljive tacke pravih b i c, a sa P podnozje
upravne iz tacke P na pravoj b, bice
P Q + QR + RP < 2BQ < 2P P < P Q + Q R + R P .

P2

P Q
B

c
X

Y
A

P
a
R

X B1ZP C1

P1

Slika uz zadatak 1946.b


2o Prave a, b, c se seku i obrazuju tupougli trougao ABC s tupim uglom
kod temena A (vidi sliku 1946.b). Prema zadatku ..., svakoj tacki P prave a
odgovaraju na pravama b i c tacke Q i R takve da je zbir duzi P Q +Q R +R P
786

minimalan. Odredimo sad onu tacku P na pravoj a za koju ce odgovarajuci


zbir P Q + Q R + R P biti najmanji.
Ako bi tacka P bila na pravoj a tako da je tacka B izmedu tacaka P i C,
tacka P bi bila uostrom uglu koji je odreden pravama b i c. Stoga bi prema
zadatku ... duz P1 P2 , koja spaja tacke P1 i P2 simetricne sa tackom P u odnosu
na prave b i c, bila jednaka odgovarajucem zbiru P Q + Q R + R P , gde su Q
i R tacke u kojima duz P1 P2 sece prave b i c. Ako zatim obelezimo sa B i P
podnozja upravnih iz tacaka B i P na pravoj b, bice
2BB < 2P P < P1 Q + P Q < P1 Q + Q R + P R .
Stoga tacka P kojoj odgovara minimalni zbir P Q + Q R + R P nije sa iste
strane tacke C kao i tacka B. Normale u tacki A na pravama b i c seku stranicu
BC u tackama koje cemo obeleziti sa B1 i C1 . Ako su zatim X i Y proizvoljne
tacke duzi BB1 i CC1 , X i Y podnozja upravnih iz tacaka X i Y na pravama
b i c, tada ce prema zadatku ... zbirovi koji odgovaraju tackama X i Y jednaki
dvostrukim duzima XX i Y Y . Ako je pak Z bilo koja tacka duzi B1 C1 , prema
istom zadatku, njoj odgovara zbir jednak dvostrukoj duzi AZ. Najzad, ako je P
podnozje visine iz temena A, bice 2AP < 2AZ, 2AP < 2XX , 2AP < 2Y Y .
Stoga je za tacu P prave a odgovarajuci zbir najmanji.
1947. Konstruisati trougao ABC ako su date tri tacke Ha ,Hb , Hc koje su
simetricne s ortocentrom H tog trougla u odnosu na prave odredene stranicama
BC, CA, AB.

Hb

A
Hc
H

Ha

Slika uz zadatak 1947.


Tacke Ha , Hb , Hc koje su simetricne s ortocentrom H pripadaju krugu l
opisanom oko trougla ABC. S obzirom da su te tri tacke poznate, moze se
konstruisati krug l. Iz jednakosti uglova ABHb i ACHc sledi da je tacka A
srediste kruznog luka Hb Hc na kome nije tacka Ha
1948. Konstruisati trougao ABC ako su date tri tacke Oa ,Ob , Oc koje su simetricne sa sredistem O opisanog kruga u odnosu na prave odredene stranicama
BC, CA, AB.

787

Ob

Oc

B1

C1
O
A1

C
Oa

Slika uz zadatak 1948.


Ako obelezimo sa A1 , B1 , C1 sredista stranica BC,CA, AB tada ce biti
OOa BC, BC k B1 C1 i B1 C1 k Ob Oc , pa je OOa Ob Oc . Isto tako je OOb Oc Oa
i OOc Oa Ob , pa je tacka O ortocentar trougla Oa Ob Oc . S obzirom da su tacke
Oa , Ob , Oc poznate, mozemo konstruisati najpre ortocentar O trougla Oa Ob Oc
zatim simetrale duzi OOa , OOb , OOc koje odreduju trougao ABC.
1949. Konstruisati trougao ABC ako su date tri tacke P , Q, R u kojima
simetrale uglova A, B, C seku opisani krug.

R
Q
S
B

Slika uz zadatak 1949.


Srediste S upisanog kruga trougla ABC je ortocentar trougla P QR.
1950. Date su tri konkurentne prave sa , sb , sc i na pravoj sa tacka A.
Odrediti na pravama sb , sc tacke B i C takve da prave sa , sb , sc budu simetrale
unutrasnjih i spoljasnjih uglova trougla ABC.

788

sa
sb
A

sc

S
A1
A2

sa
sc
A
sb
A2
C
B
Sa
A1

Slika uz zadatak 1950.


Tacke A1 i A2 simetricne sa tackom A u odnosu na prave sb i sc pripadaju
pravoj BC. Stoga su B i C tacke u kojima prava A1 A2 sece prave sb i sc .
1951. Dati su krug k sa sredistem S i tri prave sa , sb , sc koje se seku u
tacki S. Odrediti na pravama sa , sb , sc tacke A, B, C takve da prave BC, CA,
AB budu tangente kruga k.

789

sa
sc
A
sb

B
S

Slika uz zadatak 1951.


Obelezimo sa A bilo koju tacku prave sa razlicitu od tacke S, kao u prethodnom zadatku odredimo na pravama sb i sc take B i C takve da prave
sa , sb , sc budu simetrale unutrasnjih ili spoljasnjih ugova trougla A B C . Tangente kruga k paralelne sa pravama B C , C A , A B odreduju trazeni trougao
ABC.

13.5. Rotacija
1958. Date su dve linije a i b od kojih svaka predstavlja pravu ili krug i
tacka S. Odrediti na linijama a i b tacke A i B takve da ugao ASB bude jednak
datom uglu a duzi SA i SB srazmerne dvema datim duzima m i n.

M B
A
M

Slika 755

790

a
a
O

A
B

Slika 756
a) Ako bi problem imao resenje, tada bi prema definiciji u obrtnoj slicnosti
R(s, w, m : n) tacki A odgovarala tacka B. S obzirom da se tacka A nalazi na
pravoj a, bice i tacka B na pravoj a koja u pomenutoj transformaciji odgovara
pravoj a. Da bismo kontruisali pravu a , obelezimo sa M podnozje upravne iz
tacke S na pravoj a, sa SM polupravu koja s polupravom SM sa bilo koje
strane zahvata ugao jednak s uglom w i sa M tacku te poluprave takvu da
je SM : SM = m : n. Kako u pomenutoj transformaciji tacki M odgovara
tacka M , pravoj a koja je u tacki M upravna na polupravoj SM odgovara
prava a koja je u tacki M upravna na polupravoj SM . Prave a i b imaju
zajednickih tacaka ili ne. Pretpostavimo da imaju zajednickih tacaka i neka
je B bilo koja od njih. Ako obelezimo sa SA polupravu takvu da je ugao
M SM jednak i istosmeran s uglom ASB, bice ugao M SA jednak s uglom
M SB. U pravouglom trouglu M SB ugao M SB je ostar, pa je i ugao M SA
ostar. Stoga prava a upravna na kraku SM sece drugi krak M SA u nekoj
tacki A. Dokazimo da su A i B trazene tacke. Kako je ASB = M SM i
M SM = w, bice i ASB = w. Iz slicnosti trouglova M SA i M SB imamo
da je SA : SB = SM : SM . No SM : SM = m : n, pa je i SA : SB = m : n.
Ako polupravu SM konstruisemo s druge strane poluprave SM , tada umesto
prave a dobijamo neku drugu pravu a . Istim postupkom dokazuje se da ce i
svakoj zajednickoj tacki B pravih a i b odgovarati na pravoj a tacka A takva
da je ASB = w i SA : SB = m : n. Stoga broj resenja datog problema zavisi
od broja zajednickih tacaka prave b s pravama a i a . Prema tome, ako prava
b sece obe prave a i a problem ima dva resenja, ako prava b sece jednu od
pravih a i a a sa drugom je paralelna problem ima jedno resenje, ako je prava
b paralelna sa obema pravama a i a , problem nema resenja. Specijalno, ako
se prava b poklapa s nekom od pravih a i a problem ima beskonacno mnogo
resenja.
b) Ako bi problem imao resenje, tada bi prema definiciji u obrtnoj slicnosti
R(s, w, m : n) tacki A odgovarala tacka B. S obzirom da se tacka A nalazi na
krugu a, bice tacka B na krugu a koji u pomenutoj transformaciji odgovara
krugu a. Da bismo konstruisali krug a , obelezimo sa O srediste kruga a, sa

791

SO polupravu koja s polupravom SO s bilo koje strane zahvata ugao jednak


s uglom w i sa O tacku te poluprave takvu da je SO : SO = m : n. Kako
u pomenutoj transformaciji tacki O odgovara tacka O , krugu a(O, r) odgovara
krug a (O , r ), gde je r : r = m : n. Krugovi a i b imaju zajednickih tacaka
ili ne. Pretpostavimo da imaju zajednickih tacaka, neka je B bilo koja od
njih. Obelezimo sa SA polupravu takvu daje ugao OSO jednak i istosmeran s
uglom ASB i sa A tacku te poluprave takvu da je SA : SB = SO : SO , zatim
dokazimo da su A i B trazene tacke. Najpre dokazimo da je tacka A na krugu a.
Iz jednakosti i istosmernosti uglova OSO i ASB sledi da je OSA = O SB.
Otuda i iz proporcije SO : SO = SA : SB sledi da je SOA SO B, pa
je OA : O B = SO : SO , i prema tome OA : r = m : n. Iz ove i proporcije
r : r = m : n sledi da je Oa = r, pa je A tacka kruga a. Kako je ASB = OsO
i OSO = w, bice i ASB = w. Najzad, iz proporcija SA : SB = SO : SO
i SO : SO = m : n sledi da je SA : SB = m : n. Ako polupravu SO
konstruisemo s druge strane poluprave SO tada umesto kruga a dobijamo neki
drugi krug a . Istim postupkom dokazuje se da ce i svakoj zajednickoj tacki
B krugova a i b odgovarati na krugu a tacka A takva da je ASB = w i
SA : SB = m : n. Stoga broj resenja datog problema zavisi od broja zajednickih
tacaka kruga b sa krugovama a i a . Prema tome, problem moze da ima cetiri,
tri, dva, jedno ili nijedno resenje. Specijalno, ako se krug b poklapa sa nekim
od krugova a i a , problem ima neograniceno mnogo resenja.
v) Koristiti transformaciju iz jednog od prethodna dva slucaja.
1959. Date su dve linije a i b od kojih svaka predstavlja pravu ili krug i
tacka S. Odrediti na linijama a i b tacke X i Y takve da ugao XSY bude jednak
datom uglu , a proizvod duzi SX i SY jednak kvadratu date duzi r.

a
A
M1
M
B
a1

Slika 757
a) Ako bi problem imao resenje, tada bi po definiciji u obrtnoj inverziji
R(S, w, r) tacki A odgovarala tacka B. S obzirom da se tacka A nalazi na
pravoj a, tacka D se nalazi na liku a koji u pomenutoj transformaciji odgovara
pravoj a. Stoga, da bismo konstruisali tacke A i B, konstruisemo najpre pravu
a1 koja u rotaciji (S,w) odgovara pravoj a, zatim lik a koji u inverziji I(S, r)
792

odgovara pravoj a1 . Prema poznatom stavu iz inverzije lik a predstavlja pravu


ili krug zavisno od toga da li je tacka S na pravoj a1 ili ne, odnosno da li je
tacka S na pravoj a ili ne. Na sl. 757 predstavljen je slucaj kada tacka S nije
na pravoj a. Linije a i b imaju zajednickih tacaka razlicitih od tacke S ili ne.
Pretpostavimo da imaju i neka je B bilo koja od njih. S obzirom da tacka B
pripada liku a, ona u obrtnoj inverziji R(S, w, r) odgovara nekoj tacki A prave
a.
Slucajevi pod b) i v) resavaju se analognim postupkom.
1971. Dat je trougao M AN i u njegovoj ravni tacka S1 . Odrediti na kracima AM i AN tog ugla tacke B i C takve da ABC bude trougao kome je tacka
S1 srediste Ojlerovog kruga.

A2
A1

B1
B2
O

C2

C1

Slika 758
Sredista A1 , B1 , C1 stranica BC, CA, AB pripadaju Ojlerovom krugu trougla ABC, pa je B1 S1 C1 = 2B1 A1 C1 . No, B1 A1 C1 = M AN , pa je i
B1 S1 C1 = 2M AN . Stoga mozemo, kao u zadatku , na kracima AM i AN
odrediti tacke C1 i B1 takve da je S1 C1 = S1 B1 i C1 S1 B1 = 2M AN . Tacke
B i C bice simetricne sa tackom A u odnosu na tacke C1 i B1 (sl.758).
1976. U dati paralelogram ABCD upisati romb P QRS kome su dijagonale
srazmerne dvema datim duzima m i n.
Srediste romba P QRS poklapa se sa sredistem O datog paralelograma ABCD.
Stoga se kao u zadatku 1958. na stranicama AB i BC mogu odrediti tacke P
i Q takve da je P OQ prav i OP : OQ = m : n. Tacke R i S su simetricne s
tackama P i Q u odnosu na tacku O.
1981. U dati kvadrat ABCD upisati kvadrat P QRS kome je stranica jednaka datoj duzi l.

793

O
Q

Slika 759
Ako je tacka O srediste kvadrata ABCD i w prav ugao, tada u obrtnoj
slicnosti R(O, w, AB : l) kvadratu ABCD odgovara kvadrat P QRS (sl. 759).
1983. Dati su krug k i dve tacke A i B. Konstruisati tangentu t kruga k
takvu da odstojanja AM i AN tacke A od prave t i upravne kroz tacku B na
pravoj t budu srazmerne dvema datim duzima m i n.

Slika 760
Ako obelezimo sa w prav ugao, tada u obrtnoj slicnosti R(A, w, n : m) pravoj
BN odgovara prava t (sl.760).
1984. Dati su krug k sa sredistem S, dve tacke A, B i ugao . Odrediti na
krugu k tacke C i D takve da je AC
= BD i CSD
= .

794

C
B

Slika 761
U rotaciji R(S, w) tacki c odgovara tacka D, a tacki A izvesna tacka A takva
da je BDA = w. Stoga treba konstruisati skup temena svih uglova koji su
jednaki s uglom w i kojima kraci sadrze tacke B i A . Svaka od presecnih tacaka
tog skupa s krugom k predstavlja trazenu tacku D (sl.761).
1985. Dat je krug l i na njemu tacka A. Odrediti na krugu tacke B i C
takve da ugao BAC bude jednak datom uglu , a zbir ili razlika tetiva AB i
AC jednaka datoj duzi d.
Ako su p i q dve poluprave koje imaju zajednicki kraj A, zahvataju ugao
jednak s uglom w i seku krug l u nekim tackama B i C , tada se kao u zadatku
1984. moze na krugu l odrediti tacka A takva da je zbir ili razlika tetiva A B
i A C jednaka duzi d. Rotacija oko sredista O kruga l za ugao A OA prevodi
tacke A , B , C u tacke A, B, C.
1989. Dati su dva kruga k1 i k2 , tacka O i ugao . Kroz tacku O konstruisati dve prave p1 i p2 koje zahvataju ugao jednak s uglom i na kojima krugovi
k1 i k2 odsecaju jednake tetive.

795

p1

B2
S2
A2

p1

S1
B1 A1

B1

A1
O

S2

Slika 762
U rotaciji R(O, w), krugu k2 odgovara izvestan krug k2 , dok pravoj p2 odgovara prava p1 . Sad mozemo, kao u zadatku , konstruisati pravu p1 koja sadrzi
tacku O i na kojoj krugovi k1 i k2 odsecaju jednake tetive (sl.762).
1997. Date su dve prave a, b na njima respektivno tacke A, B i van tih
pravih tacka P . Konstruisati pravu p koja sadrzi tacku P i sece prave a i b u
tackama X i Y takvima da su duzi AX i BY srazmerne dvema datom duzima
m i n.
Koristiti obrtnu slicnost u kojoj tackama A i X odgovaraju tacke B i Y .
1998. Date su tri prave a, b, c i na pravama a, b respektivno tacke X i Y
takve da prave XY i c budu medu sobom paralelne, a duzi AX i BY srazmerne
dvema datim duzima m i n.

b
a

c
B
Y

P Q
X
S
A

Slika 763

796

Analizirajmo slucaj kada se prave a i b seku u nekoj tacki O. Ako pretpostavimo da na pravama a i b postoje tacke X i Y takve da je XY k c i
AX : BY = m : n, zatim obelezimo sa S srediste obrtne slicnosti u kojoj
tackama A i B odgovaraju tacke X i Y , tada u toj obrtnoj slicnosti pravoj AB
odgovara prava XY , a podnozju P upravne iz tacke S na pravoj AB odgovara
podnozje Q upravne iz tacke S na pravoj XY . Stoga je ugao ASX jednak i
istosmeran s uglom P SQ. No, ugao P SQ je poznat, pa je poznat i ugao ASX.
Problem u opstem slucaju ima dva resenja. Slucaj kada su prave a i b paralelne
je jednostavniji, neka ga citalac analizira sam.

797

You might also like